ML Aggarwal I S Chawla J Agarwal Munish Sethi Ravinder Singh - Self-Help To ICSE Class 10 X Understanding Mathematics Solutions of ML Aggarwal I S Chawla J Agarwal Munish Sethi Ravinder Singh and Sons

You might also like

Download as pdf or txt
Download as pdf or txt
You are on page 1of 692

(Rupees Four Hundred Sixty only)

1
GOODS AND SERVICES TAX

EXERCISE 1
1. An article is marked at `15000. A dealer sells it to a consumer at 10% profit. If the rate of GST is
12%, find :
(i) the selling price (excluding tax) of the article.
(ii) the amount of tax (under GST) paid by the consumer.
(iii) the total amount paid by the consumer.
Sol. (i) Purchase price of an article = `15000,
Profit = 10%

 10   11 
Selling price of an article =  1   of `15000 = `   15000  = `16500
 100  10 
(ii) Let us assume that the sale is in the same state, so this sale is intra-state.
Rate of GST = 12%, it comprises of CGST at 6% and SGST at 6%
Amount of GST collected by the dealer from the consumer.
6
SGST = CGST = 6% of `16500 = × 16500 = `990
100
 Amount of tax paid by the consumer = CGST + SGST = `990 + `990 = 1980
(iii) Total amount paid by the consumer = S.P. of an article + GST paid
= `16500 + `1980 = `18480
2. A shopkeeper buy goods worth `4000 and sells these at a profit of 20% to a consumer in the same
state. If GST is charged at 5%, find :
(i) the selling price (excluding tax) of the goods. (ii) CGST paid by the consumer.
(iii) SGST paid by the consumer. (iv) the total amount paid by the consumer.
Sol. Purchase price of an article = `4000
Profit = 20%
 20   12 
(i) Selling price of an article =  1   of `4000 = `  10  4000  = `4800
 100   
(ii) As the sale is in the city, so this sale is intra-state.
Rate of GST is 5%, it comprises of CGST at 2.5% and SGST at 2.5%.
Amount of GST collected by the shopkeeper from the consumer

 2.5 
CGST = SGST = 2.5% of `4800 = `   4800  = `120
 100 

1 Arun Deep's Understanding Math-10


2 Arun Deep's Understanding Math-10
(iii) Hence, the total amount paid by the consumer = S.P. of an article + Taxes paid
= `4800 + `120 + `120 = `5040
3. The marked price of an article is `12500. A dealer in Kolkata sells the article to a consumer in the
same city at a profit of 8%. If the rate of GST is 18%, find
(i) the selling price (excluding tax) of the article.
(ii) IGST, CGST and SGST paid by the dealer to the Central and State Governments.
(iii) the amount which the consumer pays for the article.
Sol. Marked price of the article = `12500
Profit = 8%
 8   108 
Selling price of the article =  1   of `12500 =  `12500  = `13500
 100   100 
As the sale is in the same city, so this sale is intra-state. Rate of GST is 18%, it comprises of CGST
at 9% and SGST at 9%
Amount of GST collected by the dealer from the consumer
IGST = Nil [ sale is intra-state]
 9 
CGST = SGST = 9% of `13500 = `   13500  = `1215
 100 
Amount of tax paid by the dealer to the central and state governments :
CGST = `1215 to the central government and SGST = `1215 to the state government
(iii) Total amount paid by the consumer = S.P. of the article + CGST + SGST
= `13500 + `1215 + `1215 = `15930
4. A shopkeeper buys an article from a wholesaler for `2000 and sells it to a consumer at 10% profit.
If the rate of GST is 12%, find the tax liability of the shopkeeper.
Sol. Marked price of an article = `2000
Rate of GST = 12%
 Amount of GST paid by the shopkeeper i.e. CGST and SGST = 6% of `2000 = `120
 Total GST paid by shopkeeper = `120 + `120 = `240
5. A dealer buys an article for `6000 from a wholesaler. The dealer sells the article to a consumer at
15% profit. If the sales are intra-state and the rate of GST is 18%, find
(i) input CGST and input SGST paid by the dealer.
(ii) output CGST and output SGST collected by the dealer.
(iii) the net CGST and SGST paid by the dealer.
(iv) the total amount paid by the consumer.
Sol. As all the sales are intra-state and the rate of GST is 18%, so GST comprises of CGST at 9% and
SGST at 9%
C.P. of an article for dealer = `6000
Dealer sell the article to consumer at the profit of 15%
 15   115 
 S.P. of an article for dealer =  1   of `6000 = `   6000  = `6900
 100   100 
(i) Input CGST and Input SGST paid by the dealer = 9% of `6000
3 Arun Deep's Understanding Math-10

9
= × `6000 = `540
100
Hence, Input CGST = Input SGST = `540
(ii) Output CGST and Output SGST collected by the dealer = 9% of `6900
9
= × `6900 = `621
100
Output CGST and Output SGST = `621
(iii) Net CGST paid by the dealer to the central government = Output CGST – Input CGST
= `621 – `540 = `81
Also, net SGST paid by the dealer to the state government = output CGST – Input CGST
= `621 – `540 = `81
(iv) Total amount paid by the consumer = S.P. of an article + Output CGST + Output SGST
= `6900 + `621 + `621 = `8142
6. A manufacturer buys raw material worth `7500 paying GST at the rate of 5%. He sells the finished
product to a dealer at 40% profit. If the purchase and the sale both are intra-state and the rate of
GST for the finished product is 12%, find :
(i) the input tax (under GST) paid by the manfacturer.
(ii) the output tax (under GST) collected by the manufacturer.
(iii) the tax (under GST) paid by the manufacturer to the Central and State Governments.
(iv) the amount paid by the dealer for the finished product.
Sol. As, all the sales are intra-state and the rate of GST is 5% and 12%, so GST comprises of CGST at
2.5% and 6% and CGST at 2.5% and 6%.
(i) C.P. of raw material for manufacturer = `7500
Rate of GST = 8%
 Input CGST = Input SGST = 2.5% of `7500
5
= × `7500 = `187.50
100
Hence, Input CGST and Input SGST paid by the manufacturer is `187.50 each
(ii) Marked price of raw material = `4500
Profit = 40%

 40   140 
S.P. of finished goods =  1   of `7500 = `  100  7500  = `10500
 100   
Rate of GST = 12%
6
 Output GST = Output SGST = 6% of `10500 = × `10500 = `630
100
Hence, Output CGST and Output SGST collected by the manufacturer is `630 each
(iii) Net CGST paid by the manufacturer = Output CGST – Input CGST = `630 – `187.50 = `442.50
4 Arun Deep's Understanding Math-10
Net SGST paid by the manufacturer = Output SGST – Input SGST = `630 – `187.50 = `442.50
(iv) Total amount paid by the dealer for finished good = S.P. of finished goods + CGST + SGST
= `10500 + `630 + `630 = `11760
7. A manufacturer sells a T.V. to a dealer for `18000 and the dealer sells it to a consumer at a profit
of `1500. If the sales are intra-state and the rate of GST is 12%, find:
(i) the amount of GST paid by the dealer to the State Government.
(ii) the amount of GST received by the Central Government.
(iii) the amount of GST received by the State Government.
(iv) the amount that the consumer pays for the T.V.
Sol. Since, it is a case of intra-state transaction of goods and services.

1
 SGST = CGST = GST
2
Given as manufacturer sells T.V. to a dealer = `18000
Amount of GST collected by manufacturer from Dealer

6
CGST = SGST = 6% of `18000 =  18000 = `1080
100
 Manufacturer will pay `1080 as CGST and `1080 as SGST
Amount of input GST of dealer = Input CGST = `1080 and Input SGST = `1080
 C.P. of a T.V. for dealer = `18000
Profit = `1500
Selling price of a TV for dealer to customer = C.P. + Profit = `18000 + `1500 = `19500
Amount of GST collected by dealer from customer:

6
CGST = SGST = 6% of `19500 =  19500 = `1170 and
100
 Amount of output GST of dealer : Output CGST = `1170 and Output SGST = `1170
(i) Amount of GST paid by the dealer to the State Government = output CGST – Input CGST
= `1170 – `1080 = `90
(ii) Amount of CGST paid by the dealer = Output CGST – Input CGST = `1170 – `1080 = `90
 Amount of CGST received by Central Govt. = CGST paid by manufacturer + CGST paid by dealer
= `1080 + `90 = `1170
(iii) Amount of SGST paid by dealer = Output SGST – Input SGST = `1170 – `1080 = `90
 Amount of SGST received by State Govt. = SGST paid by manufacturer + SGST paid by dealer
= `1080 + `90 = `1170
(iv) Amount paid by customer for T.V. = Cost price of T.V. + CGST paid by the customer + SGST paid
by the customer = `19500 + `1170 + `1170 = `21840
5 Arun Deep's Understanding Math-10
8. A shopkeeper buys a camera at a discount of 20% from a wholesaler, the printed price of the
camera being `1600. The shopkeeper sells it to a consumer at the printed price. If the sales are
intra-state and the rate of GST is 12%, find:
(i) GST paid by the shopkeeper to the Central Government.
(ii) GST received by the Central Government.
(iii) GST received by the State Government.
(iv) the amount at which the consumer bought the camera.
Sol. Since, it is a case of intra-state transaction of goods and services.
1
 SGST = CGST = GST
2
Given, printed price of a camera = `1600 ; Rate of discount = 20%
C.P. of a camera for shopkeeper = Printed price – Discount = `1600 – 20% of `1600
20
= `1600 –  1600 = `1600 – `320 = `1280
100
Given, rate of GST = 12%
Amount of GST paid by the shopkeeper to the wholesaler;
6
CGST= SGST = 6% of `1280 =  1280 = `76.80
100
(i) Amount of input GST of the shopkeeper : CGST = `76.80 and SGST = `76.80
Selling price of a camera by the shopkeeper to a consumer = `1600
Amount of GST paid by the consumer to shopkeeper :
6
CGST = SGST = 6% of `1600 =  1600 = `96
100
Amount of output GST of the shopkeeper : Output CGST = `96 and Output SGST = `96
Amount of GST paid by the shopkeeper to the Central Govt. = Output CGST – Input CGST
= `96 – `76.80 = `19.20
(ii) Amount of CGST paid by the shopkeeper = Output CGST – Input CGST = `96 – `76.80 = `19.20
 Amount of GST received by the Central Government = CGST paid by the wholesaler + CGST paid
by the shopkeeper = `76.80 + `19.20 = `96
(iii) Amount of SGST paid by the shopkeeper = Output SGST – Input SGST = `96 – `76.80 = `19.20
 Amount of GST received by the State Government = SGST paid by the wholesaler + SGST paid
by the shopkeeper =`76.80 + `19.20 = `96
(iv) The amount paid by the customer for camera = Cost price of a camera to consumer + CGST paid
by the consumer + SGST paid by consumer = `1600 + `96 + `96 = `1792
PQ. A manufacturer sells a washing machine to a wholesaler for `15000. The wholesaler sells it to a
trader at a profit of `1200 and the trader sells it to a consumer at a profit of `1800. If all the sales
6 Arun Deep's Understanding Math-10
are intra-state and the rate of GST is 12%, find:
(i) the amount of tax (under GST) received by the State Government from the wholesaler.
(ii) the amount of tax (under GST) received by the Central Government from the trader.
(iii) the amount that the consumer pays for the machine.
Sol. (i) Since, it is a case of intra-state transaction of goods and services.
1
 SGST = CGST = GST
2
Given, C.P. of a washing machine for wholesaler = `15000; Rate of GST = 12%
Amount of GST paid by wholesaler to manufacturer :
6
CGST = SGST = 6% of `15000 =  15000 = 900
100
 Selling price of a washing machine by the wholesaler to a Trader = C.P. + Profit
= `15000 + `1200 = `16200
Amount of GST paid by the trader to the wholesaler :
6 6
CGST = SGST = 6% of `16200 =  16200   16200 = `972
100 100
(i) Amount of input GST of the wholesaler : Input CGST = `900 and Input SGST = `900
 Amount of SGST paid by the wholesaler to the State Government = Output SGST – Input SGST
= `972 – `900 = `72
(ii) Amount of input GST for trader : Input CGST = `972 and Input SGST = `972
Selling price of a washing machine for trader to a consumer = C.P. of washing machine + Profit
= `16200 + `1800 = `18000
Amount of GST paid by the consumer to Trader :
6
CGST = SGST = 6% of `18000   18000 = `1080
100
Amount of CGST paid by the trader to the Central Government = Output CGST – Input CGST
= `1080 – `972 = `108
(iii) Amount paid by the consumer for washing machine = C.P. of washing machine for consumer +
CGST paid by the consumer + SGST paid by the consumer
= `18000 + `1080 + `1080 = `20160
9. A dealer buys an article at a discount of 30% from the wholesaler, the marked price being `6000.
The dealer sells it to a consumer at a discount of 10% on the marked price. If the sales are intra-
state and the rate of GST is 5%, find:
(i) the amount paid by the consumer for the article.
(ii) the tax (under GST) paid by the dealer to the State Government.
(iii) the amount of tax (under GST) received by the Central Government.
Sol. Since, it is a case of intra-state transaction of goods and services.
1
 SGST = CGST = GST;; Rate of GST = 5%
2
7 Arun Deep's Understanding Math-10
Given, Marked price of an article = `6000
Rate of discount given by the wholesaler = 30%
 C.P. of an article for dealer = Marked price – Discount = `6000 – 30% of `6000
30
= ` 6000   6000 = `6000 – 1800 = `4200
100
Amount of GST paid by the dealer to the wholesaler :
2.5
CGST = SGST = 2.5% of `4200 =  4200 = `105
100
 Amount of input GST for dealer : Input CGST = `105, Input SGST = `105
(i) Rate of discount given by dealer to consumer = 10% of marked price
Selling price of an article for consumer = Marked price – Discount = `6000 – 10% of `6000
10
= ` 6000   6000 = `5400
100
Amount of GST paid by consumer to dealer :
2.5
CGST = SGST = 2.5% of `5400 =  5400 = `135
100
 Amount of Output GST for dealer : Output CGST = `135, Output SGST = `135
 Amount paid by the consumer for an article = C.P. of an article for consumer + CGST paid by the
consumer + SGST paid by the consumer = `5400 + `135 + `135 = `5670
(ii) Amount of SGST paid by the dealer to the State Government = Output SGST – Input SGST
= `135 – `105 = `30
(iii) Amount of CGST paid by the dealer to Central Government = Output CGST – Input CGST
= `135 – `105 = `30
 Amount of GST received by Central Government = CGST paid by the wholesaler + CGST paid by
dealer = `105 + `30 = `135
10. The printed price of an article is `50000. The wholesaler allows a discount of 10% to a shopkeeper.
The shopkeeper sells the article to a consumer at 4% above the marked price. If the sales are intra-
state and the rate of GST is 18%, find:
(i) the amount inclusive of tax (under GST) which the shopkeeper pays for the article.
(ii) the amount paid by the consumer for the article.
(iii) the amount of tax (under GST) paid by the shopkeeper to the Central Government.
(iv) the amount of tax (under GST) received by the State Government.
Sol. Since, it is a case of intra-state transaction of goods and services.
1
 SGST = CGST = GST;; Rate of GST = 18%
2
Given, Marked price of an article = `50000
(i) Rate of discount given by the wholesaler = 10%
 C.P. of an article for shopkeeper = Marked price – Discount = `50000 – 10% of `50000
10
= ` 50000   50000 = `50000 – 5000 = `45000
100
8 Arun Deep's Understanding Math-10
Amount of GST paid by the dealer to the wholesaler :
9
CGST = SGST = 9% of `45000 =  45000 = `4050
100
 Amount of input GST for dealer : Input CGST = `4050, Input SGST = `4050
 Amount paid by the shopkeeper for an article = C.P. of an article for shopkeeper + CGST paid by
the consumer + SGST paid by the consumer = `45000 + `4050 + `4050 = `53100
(ii) Rate of discount given by shopkeeper to consumer = 4% of marked price
Selling price of an article for consumer = Marked price – Discount = `50000 – 4% of `50000
4
= ` 50000   50000 = `52000
100
Amount of GST paid by consumer to dealer :
9
CGST = SGST = 9% of `52000 =  52000 = `4680
100
 Amount of Output GST for dealer : Output CGST = `4680, Output SGST = `4680
 Amount paid by the consumer for an article = C.P. of an article for consumer + CGST paid by the
consumer + SGST paid by the consumer = `52000 + `4680 + `4680 = `61360
(iii) Amount of CGST paid by the shopkeeper to Central Government = Output CGST – Input CGST
= `4680 – `4050 = `630
(iv) Amount of SGST paid by the shopkeeper to the State Government = Output SGST – Input SGST
= `4680 – `4050 = `630
 Thus, required amount of tax (under GST) received by State Government = SGST paid by the
wholesaler + SGST paid by shopkeeper = `4050 + `630 = `4680
11. A retailer buys a T.V. from a wholesaler for `40000. He marks the price of the T.V. 15% above his
cost price and sells it to a consumer at 5% discount on the marked price. If the sales are intra-state
and the rate of GST is 12% find:
(i) the marked price of the T.V.
(ii) the amount which the consumer pays for the T.V.
(iii) the amount of tax (under GST) paid by the retailer to the Central Government.
(iv) the amount of tax (under GST) received by the State Government.
Sol. (i) Given C.P. of T.V. for retailer = `40,000
 15 
 Marked price of T.V. = `(40,000 + 15% of 40,000) = `  40, 000   40, 000 
 100 
= `(40,000 + 6000) = `46,000
 5 
(ii) Discount given by retailer = 5% of `46000 =   46000  = ` 2300
 100 
 Amount paid by consumer without GST for T.V. = `46,000 – `2300 = `43,700
Rate of GST = 12%
 12 
 Amount of GST paid by consumer = 12% of `43700 = `   43700  = `5244
100 
9 Arun Deep's Understanding Math-10
 Amount which the consumer pays for TV = `(43,700 + 5244) = `48,944
6
(iii) Input CGST paid by shopkeeper = 6% of `40,000 =  40,000 = `2400
100
and SGST = 6% of `40000 = `2400
The shopkeeper sells the article to consumer = `43,700
 GST collected by shopkeeper = 12% of `43,700 = `5244

 6 
 Output CGST of shopkeeper = Output SGST = 6% of 43,700 =   43, 700  = `2622
100 
 Amount of tax (under GST) paid by retailer to Central Government = Output CGST – Input CGST
= `2622 – `2400 = `222
(iv) SGST paid by wholesaler = `2400 and SGST paid by shopkeeper = `222
 Total tax (under GST) received by the State Government = `(2400 + 222) = `2622
12. A shopkeeper buys an article from a manufacturer for `12000 and marks up it price by 25%. The
shopkeeper gives a discount of 10% on the marked up price and he gives a further off-season
discount of 5% on the balance to a customer of T.V. If the sales are intra-state and the rate of GST
is 12%, find:
(i) the price inclusive of tax (under GST) which the consumer pays for the T.V.
(ii) the amount of tax (under GST) paid by the shopkeeper to the State Government.
(iii) the amount of tax (under GST) received by the Central Government.
Sol. (i) C.P. of an article for shopkeeper = `12,000

 25 
M.P. of article = `(12000 + 25% of 12,000) = ` 12, 000   12000  = `15000
 100 

 10 
Amount of discount given by shopkeeper = 10% of `15,000 =   15000  = `1500
 100 
Further, shopkeeper gives off season discount of 5% on balance = 5% of (15000 – 1500)
5
=  135, 00 = `675
100
 C.P. of T.V. for consumer = `(13500 – 675) = `12,825

 12 
Amount of GST paid by consumer = 12% of `12,825 =   12825  = `1539
100 
Thus, price inclusive of Tax (under GST) which the consumer pays for T.V. = `(12825 + 1539)
= `14,364
(ii) GST paid by shopkeeper to wholesaler:
6
CGST = SGST = 6% of 12000 =  12000 = `720
100
 Amount of input GST of the shopkeeper; (CGST = `720 ; SGST = `720)
10 Arun Deep's Understanding Math-10
GST paid by consumer to shopkeeper:
 6 
CGST = SGST = 6% of 12,825 =   12825  = `769.50
100 
 Amount of output GST of the shopkeeper: CGST = `769.50 and SGST = `769.50
 The amount of GST paid by shopkeeper to State Government = Output SGST – Input SGST
= `(769.50 – 720) = `49.50
(iii) CGST paid by manufacturer = `720
CGST paid by shopkeeper = `(769.50 – 720) = `49.50
 The amount of tax (under GST) received by Central Government = `(720 + 49.50) = `769.50
13. The printed price of an article is `40000. A wholesaler in Uttar Pradesh buys the article from a
manufacturer in Gujarat at a discount of 10% on the printed price. The wholesaler sells the article
to a retailer in Himachal at 5% above the printed price. If the rate of GST on the article is 18%, find:
(i) the amount inclusive of tax (under GST) paid by the wholesaler for the article.
(ii) the amount inclusive of tax (under GST) paid by the retailer for the article.
(iii) the amount of tax (under GST) paid by the wholesaler to the Central Government.
(iv) the amount of tax (under GST) received by the Central Government.
Sol. Given printed price of an article = `40000 ; Rate of GST = 18%
Here both given sales from manufacturer to wholesaler and wholesaler to retailer are inter-state
 CGST = SGST = 0  GST = IGST
 10 
Discount given by manufacturer = 10% of `40000 = `   40000  = `4000
100 
 Cost price of article for wholesaler = `(40000 – 4000) = `36,000
 5 
The C.P. of article without tax for retailer = `40000 + 5% of `40000 = `  40000   40000 
 100 
= `42,000
(i) Amount of IGST (GST) paid by wholesaler to manufacturer = 18% of `36,000
 18 
= `   36000  = `6480
100 
 Amount inclusive of tax paid by wholesaler for the article = `(36000 + 6480) = `42,480
 18 
(ii) amount of GST paid by retailer to wholesaler = 18% of `42,000 = `   42, 000  = `7560
100 
 Amount inclusive of tax paid by retailer for the article = `(42000 + 7560) = `49,560
(iii) Amount of GST paid by wholesaler to manufacturer = 18% of `36000 = `6480
amount of GST collected by wholesaler or paid by retailer = 18% of `42,000 = `7560
 required amount of tax (under GST) paid by wholesaler to Central Government = Output IGST –
Input IGST = `(7560 – 6480) = `1080
IGST paid by wholesaler to Central Government = `1080
IGST paid by manufacturer = `6480
 Total tax (under GST) received by Central Government = `(1080 + 6480) = `7560
14. A dealer in Delhi buys an article for `16000 from a wholesaler in Delhi. He sells the article to a
consumer in Rajasthan at a profit of 25%. If the rate of GST is 5%, find :
11 Arun Deep's Understanding Math-10
(i) the tax (under GST) paid by the wholesaler to Government.
(ii) the tax (under GST) paid by the dealer to the Government.
(iii) the amount which the consumer pay for the article.
Sol. (i) As the dealer in Delhi buys an article for `16000 from a wholesaler in Delhi, so this sale is intra state.
Rate of GST is 5%. On this sale CGST and SGST is levied at 2.5%
2.5
CGST collected by wholesaler from dealer = 2.5% of `16000 =  16000 = `400
100
2.5
SGST collected by wholesaler from dealer = 2.5% of `16000 =  16000 = `400
100
 Tax paid by wholesaler to the Government = CGST collected + SGST collected = `400 + `400 = `800
(ii) The amount of Input CGST of the dealer = `800
The dealer sells the article to a consumer in Rajasthan at a profit of 25%
 25   25 
 S.P. of article by the dealer = `  1   ×16000 = `   16000  = `20000
 100   100 
As the dealer sells the article to a consumer in Rajasthan, so this sale is inter-state
 GST comprises of IGST at 5%
Amount of GST collected by dealer from consumer :
 5 
IGST = 5% of `20000 = `   20000  = `1000
 100 
 Amount of tax paid by the dealer to the government = Output IGST – Input GST = `1000 – `800 = `200
(iii) Total amount paid by the consumer = S.P. of an article + IGST = `20000 + `1000 = `21000
15. A shopkeeper in Delhi buys an article at the printed price of `24000 from a wholesaler in Mumbai.
The shopkeeper sells the article to a consumer in Delhi at a profit of 15% on the basic cost price.
If the rate of GST is 12%, find:
(i) the price inclusive of tax (under GST) at which the wholesaler bought the article.
(ii) the amount which the consumer pays for the article.
(iii) the amount of tax (under GST) received by the State Government of Delhi.
(iv) the amount of tax (under GST) received by the Central Government.
Sol. (i) C.P. of an article for shopkeeper = `24000; Rate of GST = 12%
As the shopkeeper buys an article from a wholesaler in Mumbai
12
IGST collected by wholesaler from shopkeeper = 12% of `24,000 =  24000 = `2880
100
The price of an article at which shopkeeper bought = Cost price of an article for shopkeeper +
IGST paid by the shopkeeper to wholesaler = `24000 + `2880 = `26880
(ii) C.P. of an article for shopkeeper = `24,000
Profit on the C.P. of an article = 15% of C.P.
S.P. of an article by the shopkeeper to consumer = C.P. + Profit = `24000 + 15% of `24000
 15 
= `24000 + `   24000  = `24000 + `3600 = `27600
100 
Amount paid by customer for an article = C.P. of an article for consumer + GST paid by the
consumer = C.P. + CGST paid by the consumer + SGST paid by the consumer
12 Arun Deep's Understanding Math-10

 6   6 
=`27600+ `   27600  + `   27600  = `27600 + `1656 + `1656 = `30912
100   100 
(iii) Amount of input IGST for shopkeeper = `2880
The shopkeeper sells an article to consumer at the profit of 15% on the basic C.P. of an article.
 S.P. of an article to consumer = C.P. of an article for shopkeeper + Profit on Basic cost price
 15 
= `24000 + 15% of `24000 = `24000 + `   24000  = `24000 + `3600 = `27600
 100 
As the shopkeeper sells an article to consumer in Delhi; so this sales is Intra-state sales.
Amount of GST collected by shopkeeper from consumer ;
6
CGST = SGST = 6% of `27600 =  27600 = `1656
100
Amount of output GST of shopkeeper = CGST = `1656 and SGST = `1656
Amount of tax paid by the shopkeeper to State Govt. = First set off `2880 input IGST against
`1656 output IGST
Then Set off the balance `1224 (`2880 – `1656) Input IGST against output SGST
 SGST paid by the shopkeeper to the State Government (Delhi) = Output SGST – Balance of input
IGST = `1656 – `1224 = `432
(iv) Amount of Tax received by the Central Govt. = IGST received from wholesaler + CGST received
from shopkeeper = `2880 + NIL = `2880
16. A dealer in Maharashtra buys an article from a wholesaler in Maharashtra at a discount of 20%, the
printed price of the article being `20000. He sells the article to a consumer in Telengana at a
discount of 10% on the printed price. If the rate of GST is 12%, find :
(i) the tax (under GST) paid by the wholesaler to Governments.
(ii) the tax (under GST) paid by the dealer to the Government.
(iii) the amount which the consumer pays for the article.
Sol. The printed price of the article is `20000. The GST on the sale or purchase of the article is 12%.
Here, the given sales between dealer and wholesaler is intra-state, so CGST and SGST at 6% will be
levied. And the given sale between dealer and consumer is inter-state. So IGST will be levied at 12%.
(i) As the dealer buys the article from a wholesaler at 20% discount on the printed price.
 20   80 
 Cost price of the article for dealer = `  1    20000 = `   20000  = `16000
 100   100 
6
Amount of CGST and SGST collected by wholesaler from dealer = 6% of `16000 = × 16000
100
= `960
Hence, the amount collected by wholesaler and deposited to the central government is `960 and to
the state government is `960.
(ii) Rate of discount offered by dealer = 10%
 10   90 
 S.P. of the article for consumer = ` 1   × 20000 = `  100  20000  = 18000
 100   
12
Amount of IGST collected by dealer from consumer = 12% of `18000 = × 18000 = `2160
100
13 Arun Deep's Understanding Math-10
 Amount deposited by the dealer to the government = Amount of IGST collected – Amount of
CGST and SGST paid = `2160 – `960 – `960 = `240
(iii) Total amount paid by the consumer = `18000 + `2160 = `20160
17. Kiran purchases an article for `5310 which includes 10% rebate on the marked price and 18% tax
(under GST) on the remaining price. Find the marked price of the article.
Sol. Let the marked price of an article = `x ; Rate of Rebate on the M.P. = 10%
10 90 9
 C.P. of an article = x – 10% of x = x  x= x x
100 100 10
Given Rate of GST = 18%
9 18 9
 Amount of GST on C.P. of an article = 18% of x=  x
10 100 10
9 x  18 9 x  9x  18  118 9 x
 Total C.P. of an article =    = 1   =  (Including GST)
10  100 10  10  100  100 10
Given C.P. of an article including tax = `5310
118 9 x 100 10
   5310  x = 5310   = 5000
100 10 118 9
Hence the required marked price of an article is `5000.
18. A shopkeeper buys an article whose list price is `8000 at some rate of discount from a wholesaler.
He sells the article to a consumer at the list price. The sales are intra-state and the rate of GST is
18%. If the shopkeeper pay a tax (under GST) of `72 to the State Government, find the rate of
discount at which he bought the article from the wholesaler.
Sol. Given list price of an article = `8000
Let the rate of discount given by wholesaler = x%
 x 
 discount given by wholesaler = x% of `8000 =   8000  = `80x
100 
Thus, C.P. of article for shopkeeper = `(8000 – 80x)
Further given C.P. of article for consumer = `8000.
Since the sales are intra state and rate of GST = 18%
 CGST = SGST = 9%
amount of GST paid by shopkeeper to wholesaler :
9
SGST = CGST = 9% of `(8000 – 80x) = (8000 – 80x)
100
9
 amount of input GST of shopkeeper : CGST = SGST = (8000 – 80x)
100
amount of GST paid by consumer to shopkeeper:
 9 
CGST = SGST = 9% of 8000 =   8000  = `720
100 
 Amount of output GST of shopkeeper :CGST = SGST = `720
Thus, tax paid by shopkeeper to State Government = output SGST – Input SGST
9
= 720  (8000 – 80x)
100
14 Arun Deep's Understanding Math-10
Also tax paid by shopkeeper to State Government = `72
9  80 720 72
 72 = 720 – (100  x ) 720 – 72 = (100  x)  648 = (100  x )
100 100 10
648  10
 100 – x =  90  x = 100 – 90 = 10
72
 Required rate of discount = 10%
MULTIPLE CHOICE QUESTIONS
A retailer purchases a fan for `1500 from a wholesaler and sells it to a consumer at 10% profit. If the
sales are intra-state and the rate of GST is 12%, then choose the correct answer from the given four
options for questions 1 to 6:
1. The selling price of the fan by the retailer 4. The tax (under GST) paid by the retailer to
(excluding tax) is the State Government is
(a) `1500 (b) `1650 (a) `99 (b) `9
(c) `1848 (d) `1800 (c) `18 (d) `198
Sol. Cost price of fan for retailer = `1500 Sol. Amount of input SGST of the retailer = 6%
given profit% = 10% of `1500
 selling price of fan by the retailer  6 
=   1500  = `90
= C.P. + 10% of C.P. 100 
 10  Since the retailer sells the article to the
=Rs. 1500   1500  =`1650 Ans.(b)
 100  consumer at 10% profit.
2. The selling price of the fan including tax  10 
(under GST) by the retailer is  S.P. of article = 1500   1500  = `1650

100
(a) `1650 (b) `1800  Amount of output SGST of the retailer
(c) `1848 (d) `1830
 6 
Sol. Given GST (rate) = 12% = 6% of `1650 =   1650  = `99
100 
 12 
 GST = 12% of S.P. =   1650  = `198  Amount of tax (under GST) paid by retailer
100 
to State Government = Output SGST
Thus, the required selling price of an (fan)
– Input SGST
including tax by the retailer (under GST)
= S.P. + GST = `(1650 + 198) = `1848 = (99 – 90) = `9  Ans. (b)
 Ans. (c) 5. The tax (under GST) received by the Central
3. The tax (under GST) paid by the wholesaler Government is
to the Central Government is (a) `18 (b) `198
(a) `90 (b) `9 (c) `90 (d) 99
(c) `99 (d) `180 Sol. Amount of CGST paid by the retailer
Sol. The tax (under GST) paid by wholesaler to = Output CGST – Input CGST
Central Government = `(99 – 90) = `9 [ CGST = SGST]
 6  Thus, amount of tax (under GST) received
= 6% of `1500 =   1500  = `90
100  by Central Government
1 = CGST paid by distributor + CGST paid by
[ SGST = CGST =  rate of GST = 6%] retailer = (6% of `1500) + 9 = 90 + 9 = `99
2
 Ans. (a)  Ans. (d)
15 Arun Deep's Understanding Math-10
6. The cost of the fan to the consumer inclusive Thus S.P. of T.V. including Tax (under GST)
of tax is by the distributor
(a) `1650 (b) `1800 = `(24000 + 4320) = `28320
(c) `1830 (d) `1848  Ans. (c)
Sol. Here selling price of fan = `1650 8. The tax (under GST) paid by the distributor
GST on fan = 12% of `1650 to the State Government is
(a) `4320 (b) `2160
 12 
=  1650  = `198 (c) `2880 (d) `720
 100 
Sol.  Tax (under GST) paid by distributor to
Thus, cost of a fan to the consumer inclusive the State Government
of tax = SGST = 9% of `24000
= `(1650 + 198) = `1848
 9 
 Ans. (d) =   24000  = `2160
100 
7. A shopkeeper bought a TV from a distributor
 Ans. (b)
at a discount of 25% of the listed price of
`32000. The shopkeeper sells the TV to a 9. The tax (under GST) paid by the shopkeeper
consumer at the listed price. If the sales are to the Central Government is
intra-state and the rate of GST is 18%, then (a) `720 (b) `1440
choose the correct answer from the given (c) `2880 (d) `2160
four options for questions 7 to 11.
Sol. Amount of input CGST by the shopkeeper
The selling price of the TV including tax
CGST = `2160; SGST = 2160
(under GST) by the distributor is
Amount of GST collected by the shopkeeper
(a) `32000 (b) `24000
or paid by the consumer
(c) `28320 (d) `26160
= 18% of (32000)
Sol. Given listed price of T.V. by the distributor
 18 
= `32000 rate of discount = 25% =   32000  = `5760
100 
 25  5760
 Discount for shopkeeper =   32000 
 100   SGST = = `2880 = CGST
2
= `8000  Amount of CGST paid by shopkeeper to
Central Government = Output CGST – Input
Thus, C.P. of T.V. for shopkeeper or S.P. of
CGST
T.V. for distributor = `(32000 – 8000) =
`24,000, Rate of GST = 18% = (2880 – 2160) = `720
 Ans. (a)
 Rate of SGST = Rate of CGST = 9%
10. The tax (under GST) received by the State
[in case of Intra-state]
Government is
it is given that S.P. of T.V. for shopkeeper (a) `5760 (b) `4320
`32,000
(c) `1440 (d) `2880
 GST = 18% of `24000
Sol. Amount of SGST paid by distributor to state
government
= Rs.  
18
 24000  = `4320
 100  = 9% of 24000 = `2160
16 Arun Deep's Understanding Math-10
Amount of SGST paid by shopkeeper to state Sol. Given S.P. of T.V. for consumer = `32,000
government = `720
 18 
 Total tax (under GST) received by State GST = 18% of `32000 = `   32000 
100 
Government
= `2160 + `720 = `2880 = `5760
 Ans. (d)  Price (including tax) under GST of the T.V.
paid by consumer
11. The price including tax (under GST) of the
TV paid by the consumer is = `32,000 + `5760
(a) `28320 (b) `37760 = `37760
(c) `34880 (d) `32000  Ans. (b)

CHAPTER TEST
1. A shopkeeper bought a washing machine at  10 
a discount of 20% from a wholesaler, the =   18000  = `1800
 100 
printed price of the washing machine being
`18000. The shopkeeper sells it to a  C.P. of washing machine for customer
consumer at a discount of 10% on the printed without tax
price. If the sales are intra-state and the rate = `(18000 – 1800) = `16,200
of GST is 12%, find: Rate of GST = 12%
(i) the price inclusive of tax (under GST) Since it is a case of intra state
at which the shopkeeper bought the machine.
 Rate of SGST = rate of CGST = 6%
(ii) the price which the consumer pays for amount of GST paid by shopkeeper = 12%
the machine. of `14,400
(iii) the tax (under GST) paid by the
 12 
wholesaler to the State Government. =   14, 400  = `1728
100 
(iv) the tax (under GST) paid by the
shopkeeper to the State Government. Thus, price inclusive of tax at which
(v) the tax (under GST) received by the shopkeeper bought the machine
Central Government. = `(14,400 + 1728) = `16,128
Sol. Given printed price of washing machine (ii) GST paid by customer = 12% of 16,200
= `18000  12 
=   16, 200  = `1944
(i) Discount given by wholesaler = 20% of 100 
`18000
 Price for which the customer buys washing
 20  machine
= `   18000  = `3600
100  = `(16,200 + 1944) = `18144
 C.P. of washing machine for shopkeeper (iii) Tax paid by the wholesaler to State
Government
= `(18000 – 3600)
= 6% of `14,400
= `14,400
 6 
Discount given by shopkeeper = 10% of =   14, 400  = `864
`18000 100 
17 Arun Deep's Understanding Math-10
(iv) Amount of GST paid by shopkeeper to (v) the tax (under GST) paid by the retailer
wholesaler: to the Central Government for the article.
SGST = 6% of 14,400 = `864 = CGST (vi) the tax (under GST) received by the State
 amount of input GST of shopkeeper: Government.
 SGST = CGST = `864 Sol. Given list price of goods = `1600
amount of GST paid by customer to Discount given by manufacturer = 25% of
shopkeeper `1600
SGST = 6% of 16,200  25 
=   1600  = `400
100 
 6 
=   16, 200  = `972
 100   C.P. of per article for wholesaler = `(1600 –
400) = `1200
 amount of output GST of shopkeeper
Discount given by wholesaler = 20% of
CGST = SGST = `972
 Tax (under GST) paid by shopkeeper to State  20 
`1600 =   1600  = `320
Government 100 
= output SGST – Input SGST  C.P. per article for retailer = `(1600 – 320)
= `(972 – 864) = `108 = `1280
(v) CGST paid by wholesaler = `864 Discount given by retailer = 5% of `1600
CGST paid by shopkeeper
 5 
= `(972 – 864) = `108 =   1600  = `80
100 
 Total Tax (under GST) paid to Central
Government  C.P. for consumer without tax
= `(1600 – 80) = `1520
= `(864 + 108) = `972
It is case of Intra-state
2. A manufacturer listed the price of his goods
at `1600 per article. He allowed a discount 5
of 25% to a wholesaler who in turn allowed  Rate of SGST = rate of CGST = %
2
a discount of 20% on the listed price to a
(i) GST paid by wholesaler = 5% of `1200
retailer. The retailer sells one article to a
consumer at a discount of 5% on the listed  5 
price. If all the sales are intra-state and the =   1200  = `60
100 
rate of GST is 5%, find:
 price per article inclusive of tax which the
(i) the price per article inclusive of tax (under
wholesaler pay
GST) which the wholesaler pays.
= `(1200 + 60) = `1260
(ii) the price per article inclusive of tax (under
GST) which the retailer pays. (ii) GST paid by retailer = 5% of 1280
(iii) the amount which the consumer pays  5 
= Rs   1280  = `64
for the article. 100 
(iv) the tax (under GST) paid by the  price per article inclusive of tax (under GST)
wholesaler to the State Government for the which the retailer pays
article. = `(1280 + 60) = `1344
18 Arun Deep's Understanding Math-10
(iii) GST paid by customer = 5% of `1520 (vi) SGST paid by manufacturer to State
Government = `30
 5 
=   1520  = `76 SGST paid by retailer to State Government
100 
= `2
 amount which the consumer pays for article SGST paid by retailer to State Government
= `(1520 + 76) = `1596 = `6
(iv) Amount of GST paid by wholesaler to  Total tax received by State Government
manufacturer :
= `(30 + 2 + 6) = `38
CGST = SGST = 5% of `1200
3. Mukerjee purchased a movie camera for
5 1  `25488, which includes 10% rebate on the
=    1200  = `30

2 100 list price and 18% tax (under GST) on the
 amount of input GST wholesaler: remaining price. Find the marked price of
CGST = SGST = `30 the camera.
amount of GST paid by retailer to wholesaler: Sol. Let the marked price of movie camera = `x

5 Given rate of rebate on M.P. = 10%


SGST = CGST = % of `1280  C.P. of camera without GST
2
 5  = `(x – 10% of x)
=   1280  = `32
200   x 9x
= `  x   = `
 amount of output GST of wholesaler 10 10
= output SGST – Input SGST rate of GST = 18%
= `(32 – 30) = `2  9x 
 amount of GST on C.P. = 18% of 
(v) amount of GST paid by retailer to wholesaler 10 
5 18 9 x
SGST = CGST = % of Rs.1280 = `32 = 
2 100 10
 amount of input GST of retailer :  C.P. of movie including GST
SGST = CGST = `32 9x  18  118 9 x
= 1  = 
amount of GST paid by customer to retailer; 10 100  100 10
5 also C.P. of movie camera including all tax
SGST = CGST = % of `1520
2 and rebate = `25488
 5 
=   1520  = `38 118 9 x
200    = 25488
100 10
 amount of output GST of the retailer:
SGST = CGST = `38 25488  10  100
 x=
9  118
Thus tax (under GST) paid by retailer to
Central Government  x = 24,000
= output CGST – Input CGST Hence, the required marked price of movie
= `(38 – 32) = `6 camera = `24,000.
19 Arun Deep's Understanding Math-10
4. The marked price of an article is `7500. A Tax paid by shopkeeper to State Government
shopkeeper buys the article from a = Output SGST – Input SGST
wholesaler at some discount and sells it to a
consumer at the marked price. The sales are 6
(given) 90 = 450 – (7500  75 x )
intra-state and the rate of GST is 12%. If 100
the shopkeeper pays `90 as tax (under GST)
450
to the State Government, find:  90 = 450  (100  x)
(i) the amount of discount. 100
(ii) the price inclusive of tax (under GST) 450
of the article which the shopkeeper paid to  (100  x )  450  90
100
the wholesaler.
Sol. Given marked price of article = `7500 9
 (100  x) = 360
Let the required rate of discount given by 2
wholesaler to shopkeeper be x%.
360  2
 discount given by wholesaler  100 – x =
9
= x% of 7500
 x = 100 – 80 = 20
 x   rate of discount = 20%
= `   7500 
100 
(ii) amount of discount = 20% of 7500
= `75x
 20 
Thus, C.P. of article for shopkeeper =   7500 
100 
= `(7500 – 75x)
C.P. of article for consumer = `7500 = `1500
rate of GST = 12% C.P. of article for shopkeeper = `(7500 – 1500)
= `6000
It is case of inter-state, rate of SGST = rate
of CGST = 6% amount of GST paid by amount of GST paid by shopkeeper = 12%
shopkeeper to wholesaler of `6000
SGST = CGST = 6% of (7500 – 75x)  12 
=   6000  = `720
6 100 
= (7500  75 x)
100  required price of article inclusive of tax
Amount of input GST of shopkeeper = `6000 + `720 = `6720.
6 5. A retailer buys an article at a discount of 15%
SGST = CGST = (7500  75 x) on the printed price from a wholesaler. He
100
amount of GST paid by consumer to marks up the price by 10% on the printed
shopkeeper price but due to competition in the market,
he allows a discount of 5% on the marked
SGST = CGST = 6% of 7500 price to a buyer. If the rate of GST is 12%
 6  and the buyer pays `468.16 for the article
=  7500  `450 inclusive of tax (under GST), find
 100 
 amount of output GST of shopkeeper: (i) the printed price of the article.
SGST = CGST = `450 (ii) the profit percentage of the retailer.
20 Arun Deep's Understanding Math-10
Sol. Let the printed price of the article = `x
15 x
discount given by wholesaler = 15% of `x=
100

15 x 85 x 17 x
 C.P. of article for retailer = x   =`
100 100 20

10 11x
It is given that retailer, marked the price of article = x + 10% of x = x  x
100 10

 11x  5 11x 11x


But discount given by retailer =  5% of  =  =
10 100 10 200

11x 11x 220 x  11x 209 x


 C.P. of article for buyer =  = =
10 200 200 200
Given rate of GST = 12%
209 x 12 209 x
Thus, GST paid by buyer for an article = 12% of = 
200 100 200

209 x 12 209 x 209 x 112


 Buyer pays for the article inclusive of tax =   = 
200 100 200 200 100
also given buyer pays for article inclusive of tax = ` 468.16
209 x 112 46816  100  200
  = 468.16 x = = 400
200 100 100  112  209
Hence the printed price of article = `400

 17 
(ii) C.P. of article for retailer = `   400  = `340
20

209
S.P. of article for retailer = C.P. of article for buyer =  400 = `418
200
 profit of retailer = S.P. – C.P. = `(418 – 340) = `78
profit 78 390 16
and profit % =  100 =  100 = = 22 %
C.P. 340 17 17
2
BANKING
POINTS TO REMEMBER
1. Calculation of interest on a Saving Bank Account ?
(i) Interest for the month is calculated on the mininmum balance between the 10th. day and the
last day of that month.
(ii) Convert the minimum balance of each month as a multiple of Rs. 10.
(iii) Add all the balance.
(iv) Find simple interest on this sum for one month.
(v) If the interest is less than Re. 1, neglect it.
(vi) No interest is paid for the month in which the account is closed.
2. Calculation of interest on a recurring deposit is a bit difficult. So, for this purpose, maturity value
table is available in the banks and post offices. This table shows the maturity value of an RD or C.D.
with Rs. 100 per month for various periods and at various rates of interest (Compounded quarterly).
3. We can also use a formula given below to calculate the maturity amount of an R.D. Equivalent
x ( x  1)
principal for one month  P×
2
where P is the amount deposited each month.
Pr t
Then Interest is calculated as
100
where P is the principal for one month, r is net rate % and t is the the time i.e. one month.
 Amount of maturity = Total principal + interest = P × n + interest.

EXERCISE 2
1. Mrs. Goswami deposits Rs. 1000 every month in a recurring deposit account for 3 years at the
rate at 8 % p.a. Find the matured value.
Sol. Deposit per month = Rs. 1000,
Period = 3 years or 36 months
Rate of interest = 8% of p.a.
36(36  1)
 Total principal for one month =  1000
2
PRT 36  37  1000  8
Interest = = = 12 × 37 × 10 = Rs. 4440
100 2  12  100
Amount of Maturity = P × n + S.I. = Rs. (1000 × 36 + 4440) = Rs. (36000 + 4440)
= Rs. 40440 Ans.
2. Sonia had a recurring deposit account in a bank and deposited ` 600 per month for 2½ years. If
the rate of interest was 10% p.a., find the maturity value of this account.
21 Arun Deep's Understanding Math-10
22 Arun Deep's Understanding Math-10
Sol. Period(n) = 30 months,
Principal(P) = ` 600, rate(r) = 10% p.a.

P  n( n  1)  r  1 600  30  3110 1
S.I. = =
2  100 12 2 100 12
 S.I. = ` 2325
 Maturity Value = (P × n) + S.I. = (600 × 30) + 2325
 Maturity Value = ` 20,325 Ans.
3. Kiran Deposited Rs. 200 per month for 36 months in a bank's recurring deposit account. If the
bank pays interest at the rate of 11% per annum, find the amount she gets on maturity.
Sol. Amount deposited per month (P) = Rs. 200
Period (n) = 36 months, Rate (R) = 11% p.a.
36(36  1) 36  37
 Total principal for one month = 200  = 200 × = 100 × 36 × 37
2 2

P  R  T 100  36  37  11  1
Interest = = = Rs. 1221
100 100  12
Kiran will get maturity value = P × n + S.I. = Rs. (200 × 36 + 1221) = Rs. 8421

4. Heneef has a cumulative bank account and deposits Rs. 600 per month for a period of 4 years. If
he gets Rs. 5880 as interest at the time of maturity, find the rate of interest.
Sol. Interest = Rs. 5880
Monthly deposit (P) = Rs. 600
Period (n) = 4 years or 48 months
Pn  n  1 600  48  49
 Deposit for 1 month = = = Rs. 705600
2 2
Let, rate of interest = r % p.a.
Prt 705600  r  1
Interest =  5880 =  5880 = 588r
100 100  12
5880
 r= = 10
588
 Rate of interest = 10% p.a.
5. David opened a Recurring Deposit Account in a bank and deposited Rs. 300 per month for two
years. If he received Rs. 7725 at the time of maturity, find the rate of interest per annum.
(2008)
Sol. Deposit during one month (P) = Rs. 300
Period = 2 years = 24 months.
Maturity value = Rs. 7725
Let R be the rate percent, then
P  nn  1 300  2424  1 300  24  25
Now principal for 1 month = = = = Rs. 90000
2 2 2
23 Arun Deep's Understanding Math-10
PRT 90000  R  1 = 18 months
 Interest earned = =
100 100  12 n(n  1) r
= 75R  Interest = P × ×
2  12 100
Now 300 × 24 + 75R = 7725
 7200 + 75R = 7725 18(18  1) r
= `800 × ×
 75R = 7725 – 7200 = 525 2  12 100
525 18  19 r
 R= =7 = `800 × × = 114
14r
75 2  12 100
 Rate of Interest = 7% p.a.  Maturity amount = 114r + 800 × 18
6. Mr. Gupta opened a recurring deposit
account in a bank. He deposited Rs. 2500
 `15084 = 114r + `14400
per month for two years. At the time of  `15084 – `14400 = 114r  684 = 114r
maturity he got Rs. 67500. Find : 684
(i) the total interest earned by Mr. Gupta.  r= = 6%
114
(ii) the rate of interest per annum.
Sol. Deposit per month = Rs. 2500 8. Rekha opened a recurring deposit account
Period = 2 years = 24 months for 20 months. The rate of interest is 9%
Maturity value = Rs. 67500 per annum and Rekha receives `441 as
interest at the time of maturity. Find the
P  n n  1
 Total principal for 1 month = amount Rekha deposited each month.
2
Sol. n = 20 months r % = 9%
2500  24  25
=` = `750000 I = 441
2
Let required amount deposited by Rekha
 Interest = `67500 – 24 × 2500
= `67500 – 60000 = `7500 each month = P
1
P  n  ( n  1)  r
Period = 1 month = year I 
12 2  12  100
S.I.  100 P  20  21  9
 Rate of interest =  441 
PT 2  12  100
7500  100  12 P  20  21  9
= = 12%  441 
750000  1 2  12  100
7. Shahrukh opened a Recurring Deposit
441  2  12  100
Account in a bank and deposited `800 per  P= = ` 280
20  21  9
1  Rekha deposited `280 each month.
month for 1 years. If he received `15084
2 9. Mohan has a recurring deposit account in
at the time of maturity, find the rate of a bank for 2 years at 6% p.a. simple interest.
interest per annum. If he gets `1200 as interest at the time of
Sol. Money deposited by Shahrukh per month maturity, find :
(P) = `800 (i) the monthly instalment
r=? (ii) the amount of maturity. (2016)
1 3 Sol. Interest = `1200
No. of months (n) = 1 years = × 12
2 2 Period (n) = 2 years = 24 months
24 Arun Deep's Understanding Math-10
Rate (r) = 6% p.a. Then, Principal for one month = 2000 ×
Let monthly deposit = `P n ( n  1)
= 1000 n (n + 1)
P  n(n  1) r 2
 Interest = ×
2  12 100
1000 n (n  1)  10  1
P  24  25 6 and interest =
100  12
 1200 = ×
24 100
100 n ( n  1)
6 =
 1200 = P 12
4
100 n ( n  1)
1200  4  Maturity value = 2000 × n +
 P= = 800 12
6
100 n ( n  1)
 Monthly deposit = `800  2000n + = 83100
12
and Maturity value = P × n + Interest
= `800 × 24 + `1200 = `19200 + `1200  24000n + 100n2 + 100n = 83100 × 12
= `20400
10. Mr. R.K. Nair gets `6,455 at the end of  240 n + n2 + n = 831 × 12
one year at the rate of 14% per annum in a  n2 + 241n – 9972 = 0
recurring deposit account. Find the monthly  n2 + 277n – 36n – 9972 = 0
instalment.  n (n + 277) – 36 (n + 277) = 0
Sol. Let monthly instalment is ` P
 (n + 277) (n – 36) = 0
Here n = 1 year = 12 months
Either n + 277 = 0, then n = – 277, which
 n = 12
is not possible.
nn  1 P  R or n – 36 = 0, then x = 36
 M.V. = 2  12  100 + P..n
 Period = 36 months or 3 years
1212  1 P  14 MULTIPLE CHOICE QUESTIONS
 `6455 = × + P.12
2  12 100
Choose the correct answer from the given
13  P  7 four options (1 to 4) :
 `6455 =  P.12
100 1. If Sharukh opened a recurring deposit
91P  1200P account in a bank and deposited `800 per
 `6455 =
100 1
month for 1 years, then the total money
645500 2
 `645500 = 1291 P  P = =`500 deposited in the account is
1291
11. Samita has a recurring deposit account in (a) `11400 (b) `14400
a bank of `2000 per month at the rate of (c) `13680 (d) none of these
10% p.a. If she gets `83100 at the time of Sol. Monthly deposit = `800
maturity. Find the total time for which the
account was held. 1
Period (n) = 1 years = 18 months
Sol. Deposit per month = `2000, 2
Rate of interest = 10%,  Total money deposit = `800 × 18 = `14400 (b)
Let period = n months
25 Arun Deep's Understanding Math-10
2. Mrs. Asha Mehta deposit `250 per month 1
for one year in a bank's recurring deposit Period (n) = 2 years = 30 months
2
account. If the rate of (simple) interest is
8% per annum, then the interest earned by Interest = `1085
her on this account is Let r% be the rate of interest
(a) `65 (b) `120 P  n  (n  1) r
 Interest = ×
(c) `130 (d) `260 2  12 100
Sol. Deposit per month (P) = `250
400  30  31  r
Period (n) = 1 year = 12 months  1085 = `
2  12  100
Rate (r) = 8% p.a.
1085
P  n  (n  1) r  1085 = 155r  r= =7
 Interest = × 155
2  12 100
 Rate 7% p.a. (b)
250  12  13 8
= × = `130 (c) CHAPTER TEST
2  12 100
3. Mr. Sharma deposited `500 every month 1. Mr. Dhruv deposits `600 per month in a
in a cumulative deposit account for 2 years. recurring deposit account for 5 years at
If the bank pays interest at the rate of 7% the rate of 10% per annum (simple interest).
per annum, then the amount he gets on Find the amount he will receive at the time
maturity is of maturity.
(a) `875 (b) `6875 Sol. Deposit per month = `600
Rate of interest = 10% p.a.
(c) `10875 (d) `12875
Period (n) = 5 years 60 months.
Sol. Deposit (P) = `500 per month
Total principal for one month
Period (n) = 2 years = 24 months
n ( n  1) 60 ( 60  1)
Rate (r) = 7% p.a. = `600 × = ` 600×
2 2
P  n  (n  1) r
 Interest = × 600 × 60 × 61
2  12 100 =` = `1098000
2
500  24  25  7
= = `875 prt 1098000 ×10 ×1
2  12  100 Interest   = `9150
100 100 ×12
 Maturity value = P × 24 + Interest = `500
 Amount of maturity = `600 × 60 + `9150
× 24 + 875 = `12000 × 875 = `12875
= `36000 + `9150 = `45150
(d)
2. Ankita started paying `400 per month in a
4. John deposited `400 every month in a 3 years recurring deposit. After six months
1 her brother Anshul started paying `500 per
bank's recurring deposit account for 2 1
2 month in a 2 years recurring deposit.
years. If he gets `1085 as interest at the 2
The bank paid 10% p.a. simple interest for
time of maturity, then the rate of interest both. At maturity who will get more money
per annum is and by how much?
(a) 6% (b) 7% Sol. In case of Ankita,
(c) 8% (d) 9% Deposit per month = `400
Sol. Deposit (P) = `400 per month Period (n) = 3 years = 36 months
26 Arun Deep's Understanding Math-10
Rate of interest = 10%
n ( n  1) 36 (36  1) 400×36×37
 Total principal for one month = 400 × = 400 × =` = `266400
2 2 2
prt 266400 ×10 × 1
Interest   = `2220
100 100 ×12
 Amount of maturity = `400 × 36 + `2220 = `14400 + `2220 = `16620
In case of Anshul,
Deposit p.m. = `500
Rate of interest = 10%
1
Period (n)  2 years = 30 months
2
n ( n  1) 30 (30  1) 500×30×31
 Total principal for one month = `500 × = 500 × =` = `232500
2 2 2
232500 ×10 ×1
Interest  = `1937·50
100 ×12
Amount of maturity = `500 × 30 + `1937·50 = `15000 + `1937·50 = `16937·50
At maturity Anshul will get more amount
Difference = `16937·50 – `16620·00 = `317·50
3. Shilpa has a 4 year recurring deposit account in Bank of Maharashtra and deposits `800 per
month. If she gets `48200 at the time of maturity, find (i) the rate of simple interest, (ii) the total
interest earned by Shilpa
Sol. Deposit per month (P) = `800
Amount of maturity = `48200
Period (n) = 4 years = 48 months
Let rate of interest be R% p.a.

P(n) (n  1) 800  48  ( 48  1) 800  48  49


Total principal for one month = = =` = `940800
2 2 2
Total deposit = `800 × 48 = `38400
and amount of maturity = `48200
 Interest earned = `48200 – `38400 = `9800

S.I.  100 9800  100  12


(i)  Rate of interest = = = 12.5%
PT 940800  1
(ii) Total interest earned by Shilpa = `9800
1
4. Mr. Chaturvedi has a recurring deposit account in Grindlay’s Bank for 4 years at 11% p.a.
2
(simple interest). If he gets `101418·75 at the time of maturity, find the monthly instalment.
Sol. Let each monthly instalment = ` x
27 Arun Deep's Understanding Math-10
Rate of interest = 11%
1
Period (n)  4 years or 54 months
2
n ( n  1) 54 (54  1) 54 × 55
 Total principal for one month = ` x × =`x×  x× = 1485x
2 2 2

1485 x × 11× 1
Interest  = 13·6125x
100 ×12
 Total amount of maturity = 54x + 13·6125x = 67·6125x
 67·6125x = 101418·75
101418. 75
x = `1500
67. 6125
 Deposit per month = `1500
5. Rajiv Bhardwaj has a recurring deposit account in a bank of `600 per month. If the bank pays
simple interest of 7% p.a. and he gets `15450 as maturity amount, find the total time for which
the account was held.
Sol. Deposit during the month (P) = `600
Rate of interest = 7% p.a.
Amount of maturity = `15450
Let time = n months

P(n ) (n  1) 600  n ( n  1) 600 ( n 2  n)


 Total principal = = = = 300 (n2 + n)
2 2 2

PRT 300(n 2  n)  7  1 7
 Interest = = = (n2 + n)
100 100  12 4

7 2
 600n + (n + n) = 15450
4
 2400n + 7n2 + 7n = 61800  7n2 + 2407n – 61800 = 0
 7n2 – 168n + 2575n – 61800 = 0  7n (n – 24) + 2575 (n – 24) = 0
 (n – 24) (7n + 2575) = 0
Either n – 24 = 0, then n = 24 or 7n + 2575 = 0, then
2575
7n = –2575  n =
7
Which is not possible being negative.
 n = 24
 Period = 24 months or 2 years
3
SHARES AND DIVIDENDS
POINTS TO REMEMBER

1. Face Value : Actual value of each share is called face value. It can be Rs. 10, Rs. 25, Rs. 50, Rs.
100 etc : It is also called the nominal value.
2. Market Value : The value of each share which are being sold or purchased from the market , is
called the market value. It can be more, equal or less to the face value. If it is more than the face
value, it is called ‘at premium’, when it is less than the face value, it is called ‘at discount’ and if it is
equal to the face value, then it is called ‘at par’. Market value always changes from time to time.
3. Dividend : The profit which a shareholder gets from the company on his investment is called
dividend and it is expressed in percentage of the face value of the share and is given to the
shareholder on face value irrespective of market value.
EXERCISE 3
1. Find the dividend received on 60 shares of Sol. Amount of 500 shares at the rate of Rs. 10
Rs, 20 each if 9% dividend is declared. each. = Rs. 10 × 500 = Rs. 5000
Sol. Value of one share = Rs. 20 Amount of dividend = Rs. 400
 Value of 60 shares = Rs. 20 × 60 400×100
 Rate of dividend   8% Ans.
= Rs. 1200 5000
Rate of dividend = 9% 3. A man buys 200 ten rupee shares at ` 12.50
 Total dividend = Rs. 1200 × 9% and receives a dividend of 8%. Find the
amount invested by him and the dividend
9 received by him in cash.
= Rs. 1200 × = Rs. 108 Ans.
100 Sol. Face value of 200 shares = Rs. 10 × 200
2. A company declares 8 percent dividend to
= Rs. 2000
the share holders. If a man receives Rs.
2840 as his dividend, find the nominal value (i) Amount invested for the purchase of 200
of his shares. shares at the rate of Rs. 12·50 each
Sol. Rate of dividend = 8% = Rs. 12·50 × 200 = Rs. 2500
Amount of dividend = Rs. 2840 (ii) Rate of dividend = 8%
2840 ×100 2000 ×8
 Nominal value of shares  Total amount of dividend  Rs.
8 100
= Rs. 35500 Ans. = Rs. 160 Ans.
P.Q. A man bought 500 shares, each of face 4. Find the market price of 5% Rs. 100 share
value Rs. 10 of a certain business concern when a person gets a dividend of Rs. 65
and during the first year after purchase by investing Rs. 1430.
received Rs. 400 as dividend on his shares. Sol. Amount of dividend = Rs. 65
Find the rate of dividend on shares. Rate of dividend = 5%
28 Arun Deep's Understanding Math-10
29 Arun Deep's Understanding Math-10
65×100  Total investment = Rs.140 × 1800
 Total face value   Rs.1300 = Rs.252000
5
27000×100
If face value is Rs. 1300, then market value  Percentage on his return   10. 7
= Rs. 1430 252000
= 11% (in integers) Ans.
and if face value is Rs. 100, then market 7. What sum should a person invest in `25
1430 × 100 shares, selling at `36 to obtain an income of
value  = Rs. 110 Ans. `720 if the dividend declared is 12%? Also
1300
5. Salman buys 50 shares of face value `100 find the percentage return on his income.
(i) The number of shares bought by him.
available at `132.
(ii) The percentage return on his income.
(i) What is his investment ? Sol. Nominal value of each share = Rs. 25
(ii) If the dividend is 7.5% p.a., what will be his Market value of each share = Rs. 36
annual income ? Total income = `720.
(iii) If he wants to increase his annual income Rate of dividend = 12%
by `150, how many extra shares should he 100
buy? Total nominal value  ×720 = `6000
12
Sol. Face value = `100
6000
(i) Market value = `132, No. of shares = 50 (i)  Number of shares   240
Investment = no. of shares × Market value 25
= 50 × 132 = `6600 Total investment = 240 × `36 = `8640
(ii) Income per share = 7.5% of Face value 720×100 1
(ii) Percentage return  8 %
75 8640 3
= × 100 = `7.5
10  100 8. Ashok invests `26400 on 12% `25 shares of a
 Annual income = 7.5 × 50 = `375 company. If he receives a dividend of `2475,
find :
(iii) New annual income = 375 + 150 = `525
(i) the number of shares he bought.
525 (ii) the market value of each share. (2016)
 No. of shares = = 70
7.5 Sol. Investment = `26400
 No. of extra share to be increased = 70 – 50 Face value of each share = `25
= 20 Rate of dividend = 12%
6. A lady holds 1800, Rs. 100 shares of a and total dividend = `2475
company that pays 15% dividend annually. We know,
Calculate her annual dividend. If she had Dividend earned = M.P. of share × No. of
bought these shares at 40% premium, what r
shares ×
percentage return does she get on her 100
investment ? Give your answer to the 2475 100
(i)  Number of shares = ×
nearest integer. 12 25
Sol. Total number of shares = 1800 = 825 shares
Nominal value of each share = Rs. 100 26400
Rate of dividend = 15% (ii) Market value of each share = = `32
825
(i) Total face value of 1800 shares 9. A man invests `4500 in shares of a company
= Rs. 100 × 1800 = Rs. 180000 which is paying 7.5% dividend. If `100 shares
15 are available at a discount of 10%. find :
 Total dividend = Rs. 180000 ×
100 (i) number of shares he purchases.
= Rs. 27000 Ans. (ii) his annual income.
(ii) Market value of each share Sol. N.V. of each share = ` 100 and discount
= Rs. 100 + Rs.40 = Rs. 140 = 10%
30 Arun Deep's Understanding Math-10
 M.V. of each share = ` 100 – 10% of 15
` 100 = `90 = × 100 = 12.5%
120
(i) Number of shares purchased 12. A man buys shares at the par value of `10
yielding 8% dividend at the end of a year.
Investment 4500
= =  50 Ans. Find the number of shares bought if he
M.V. of each share 90 receives a dividend of `300.
(ii) N.V. of each share = ` 100 and Sol. Face value of each share = `10
dividend = 7.5% Rate of dividend = 8% p.a.
 Dividend on each share = 7.5% of ` 100 = `7.50 Total dividend = `300
His annual income = Dividend on 50 300 × 100
shares = 50 × ` 7.50 = 375 Ans.  Total face value of shares 
8
10. Amit Kumar invests `36,000 in buying `100 = `3750
shares at `20 premium. The dividend is 3750
15% per annum. Find : and no. of shares = ` = 375
(i) The number of shares he buys 10
13. A man invests `8800 on buying shares of
(ii) His yearly dividend
face value of rupees hundred each at a
(iii) The percentage return on his investment.
premium of 10%. If he earns `1200 at the
Give your answer correct to the nearest
end of year as dividend, find :
whole number.
(i) the number of shares he has in the company.
Sol. Investment = `36000 (ii) the dividend percentage per share.
Face value = `100 Sol. Investment = `8800
Premium = `20, dividend = 15% Face value of each share = `100
36000 and market value of each share
(i) No. of shares = = 300 = `100 + `10 = `110
120
(ii) Dividend = 15% of (100 × 300) = `4500 Total income = `1200
4500 450 8800×100
(iii) % Return = × 100 =  Total face value = ` = `8000
36000 36 110
= 12.5% = 13% 8000
(i) No. of shares   80
11. Mr. Tiwari invested `29,040 in 15% `100 100
shares at a premium of 20%. Calculate: 1200 × 100
(i) The number of shares bought by Mr. (ii) Rate of dividend  = 15%
8000
Tiwari.
(ii) Mr. Tiwari's income from the investment. 14. A man invested Rs. 45000 in 15% Rs. 100
(iii) The percentage return on his investment. shares quoted at Rs. 125. When the market
value of these shares rose to Rs. 140, he
 20  sold some shares, just enough to raise Rs.
Sol. (i) M.V. of one share =   100  100
100  8400. Calculate :
= `120 (i) the number of shares he still holds. (2004)
Investment (ii) the dividend due to him on these shares
No. of shares = M.V. of 1 share Sol. Investment on shares = Rs. 45000
Face value of each share = Rs. 125
29040 45000
=` = `242  Total number of shares  = 360 shares
120 125
(ii) Income = 242 × 15 = `3,630 Income from sold shares = Rs. 8400
dividend Income from shares
(iii)  rate % =
M.V.
× 100 No. of shares sold  Market value of each share
31 Arun Deep's Understanding Math-10


8400
 60  Total face value of 10000 shares
140 = Rs. 100 × 10000 = Rs. 1000000
 60 shares were sold. and amount of dividend
(i) No. of shares he still hold 1000000 8
= Total number of shares – shares sold = Rs.  = Rs. 80000
100
= 360 – 60 = 300 shares.
(ii) Number of shares = 90
 Number of shares he still holds = 300
Face value of each share = Rs. 150
(ii) Market value of 300 shares = Rs..300 × 140  Total face value of 90 shares
= Rs. 42000 = 100 × 90 = Rs. 9000
Face value of 300 shares = Rs.300 × 125
= Rs. 37500 9000 8
 Amount of dividend = Rs.  = Rs. 720
 Dividend due = Market value – Face value 100
= Rs. 42000 – Rs. 37500 = Rs. 4500 Market value of 90 shares = 90 × 150
15. Ajay owns 560 shares of a company. The = Rs. 13500
face value of each share is Rs. 25. The com- 720  100 16 1
pany declares a dividend of 9%. Calculate.  Rate of interest   5 %
13500  1 3 3
(i) the dividend Ajay will get. 17. A company with 4000 shares of nominal
(ii) the rate of interest, on his investment if value of Rs. 110 declares annual dividend of
Ajay has paid Rs. 30 for each share.(2007) 15%. Calculate : (i) the total amount of
Sol. No. of shares = 560 dividend paid by the company. (ii) the annual
Face value of each share = Rs. 25 income of Shah Rukh who holds 88 shares
Rate of dividend = 9% p.a. in the company. (iii) if he received only 10%
Total face value of 560 shares on his investment, find the price Shah Rukh
= Rs. 25 × 560 = Rs. 14000 paid for each share.
9 Sol. Number of shares = 4000
(i)  Amount of dividend = Rs. 14000 × Nominal (face) value of each share = Rs. 110
100
= Rs. 1260  Total face value of 4000 shares
(ii) Market value of each share = Rs. 30 = Rs. 110 × 4000 = Rs. 440000
 Total investment = Rs. 30 × 560 Rate of annual dividend = 15%
= Rs. 16800 440000  15
(i) Amount of dividend= =Rs. 66000
 Percentage of interest on his investment 100
1260 × 100 (ii) Number of shares, Shah Rukh has = 88
 = 7·5% Ans.
16800  Face value of 88 shares = 88 × 110 = Rs. 9680
16. A company with 10000 shares of nominal 9680  15
value of Rs. 100 declares an annual divi- and annual dividend = Rs. = Rs. 1452
100
dend of 8% to the share holders.
(iii) Rate of annual incomes on his investment
(i) Calculate the total amount of dividend paid
by the company ? = 10%
(ii) Ramesh bought 90 shares of the company 1452  100
 His investment = = Rs. 14520
at Rs. 150 per share. 10
Calculate the dividend he received and the 14520
and Market value of each share =
percentage return on his investment. 88
Sol. (i) Number of shares = 10000 = Rs. 165
Nominal value of each share = Rs. 100 18. By investing Rs. 7500 in a company paying
Rate of annual dividend = 8% 10 percent dividend, an income of Rs. 500
32 Arun Deep's Understanding Math-10
is received. What price is paid for each Market value of each share = Rs. 143
Rs. 100 share. 11440×10 . 4
Sol. Investment = Rs. 7500  Total dividend  Rs.
143
Rate of dividend = 10% = Rs. 832
Total income = Rs. 500 21. Two companies have shares of 7% at
Face value of each share = Rs. 100 Rs. 116 and 9% at Rs. 145 respectively. In
100×500 which of the shares would the investment
Total face value   Rs.5000 be more profitable ?
10
If face value is Rs. 5000, then investment Sol. Let the investment in each case = Rs. 116
= Rs. 7500 × 145
and if face value is Rs. 100 then market  Dividend in first case
116×145× 7
7500 × 100  Rs. = Rs. 1015
value of each share  116
5000
= Rs. 150 Ans. and dividend in second case
19. A man buys 400 ten-rupee shares at a 116×145× 9
 Rs. = Rs. 1044
premium of Rs. 2·50 on each share. If the 145
rate of dividend is 8%, Find. (i) his From the above it is clear that the second
investment (ii) dividend received (iii) yield. type of shares i.e. 9% at Rs. 145 are more
Sol. No. of shares = 400 profitable. Ans.
Face value of each share = Rs. 10 22. Which is better investment : 6% Rs. 100 shares
Market value of each share at Rs. 120 or 8% Rs. 10 shares at Rs. 15
= Rs. 10 + Rs. 2·50 = Rs. 12·50
Sol. Let the investment in each case = Rs. 120
Rate of dividend = 8%
 Face value of 400 shares = Rs. 10 × 400 In the fist case,
= Rs. 4000 Dividend on Rs. 120 = Rs. 6
(i) Total investment = Rs. 12·50 × 400 In second case, Dividend on Rs. 10
= Rs. 5000 8 × 10
8   0.8
(ii) Total dividend = Rs. 4000 × = Rs. 320 100
100 Now dividend on Rs. 15 = 0·8
320 ×100 32
(iii) Yield percent   = 6·4% 0. 8×120
5000 5  Dividend on Rs. 120   Rs. 6. 4
15
20. A man invests Rs. 10400 in 6% shares at
Rs. 104 and Rs. 11440 in 10·4% shares at It is clear that, second investment i.e. 8%
Rs. 143. How much income would he get in Rs. 10 shares at 15 is more profitable. Ans.
all ? 23. A man invests Rs. 10080 in 6% hundred-
Sol. In first case, rupee shares at Rs. 112. Find his annual
Total investment = Rs. 10400 income. When the shares fall to Rs. 96 he
Rate of dividend = 6% sells out the shares and invests the proceeds
Market value of each share = Rs. 104 in 10% ten-rupee shares at Rs. 8. Find the
10400× 6 change in his annual income.
 Total dividend   Rs. 600 Sol. Investment = Rs. 10080
104
In second case, Face value of each share = Rs. 100
Investment = Rs. 11440 Market value of each share = Rs. 112
Rate of dividend = 10·4% Rate of dividend = 6%
33 Arun Deep's Understanding Math-10
 Total income for the year = (10% of ` 100) × ` 50 = ` 500
10080× 6 Annual income in 2nd case = Dividend
  Rs. 540
112 on 1 share × no. of shares
= (12% of ` 100) × ` 80 = ` 960
10080
No. of shares   90  The change in his annual income = ` 960 –
112
Selling price of 90 shares at the rate of ` 500 = ` 460
Rs. 96 each = 90 × 96 = Rs. 8640 25. A person invests Rs. 4368 and buys cer-
Rate of dividend in new shares = 10% tain hundred-rupee shares at 91. He sells
Face value of each share = Rs. 10 out shares worth Rs. 2400 when they have
and market value of each share = Rs. 8 risen to 95 and the remainder when they
have fallen to 85. Find the gain or loss on
8640
No. of shares   1080 the total transaction.
8 Sol. Investment = Rs. 4368
Face value of 1080 shares = 1080 × 10 Market value of each share = Rs. 91
= Rs. 10800 Face value of each share = Rs. 100
10800×10 4368
 Dividend  Rs. = Rs. 1080  No. of shares = = 48
100 91
Difference in income = Rs. 1080 – Rs. 540 Face value of 24 shares = 24 × 100
= Rs. 540 more Ans. = Rs. 2400
24. Sachin invests ` 8500 in 10%, ` 100 shares Sale price of shares worth Rs. 2400
at ` 170. He sells the shares when the
2400×95
price of each share rises by ` 30. He   Rs.2280
100
invests the proceeds in 12% ` 100 shares
Face value of remaining shares = 24 × 100
at ` 125. Find: = Rs. 2400
(i) the sale proceeds. Sale price of shares of remaining amount
(ii) the number of ` 125 shares he buys.
(iii) the change in his annual income. 2400×85
  Rs. 2040
Sol. (i) Investment = ` 8,500 and M.V. of 100
each share = ` 170 Total amount received = Rs. 2280 + Rs.
 Number of shares bought 2040 = Rs 4320
 Loss = Rs. 4368 – Rs. 4320 = Rs. 48 Ans.
Investment `8,500
=   50 26. By purchasing Rs. 50 gas shares for Rs.
M.V. of each share `170 80 each, a man gets 4% profit on his inves-
Sachin sells each share for ` 170 + ` 30 tment. What rate percent is company
= ` 200 paying ? What is his dividend if he buys
 The sale proceeds = No. of shares sold × 200 shares ?
S.P. of each share = 50 × ` 200 = ` 10,000 Sol. Market value of each share = Rs. 80
(ii) He invests 10,000 in shares of M.V. = ` 125 Face value of each share = Rs. 50
Number of ` 125 shares be buys Interest on investment = 4%
80 × 4 32
Investment `10,000 Dividend on Rs. 80  
=   80 Ans. 100 10
M.V. of each share `125
(iii) Annual income in 1st case = Dividend on 32
Now dividend on face value Rs. 50 
1 share × no. of shares 10
34 Arun Deep's Understanding Math-10
32 100 64 1000000 × 5
 Percent dividend  ×  = 6·4%  Total amount of dividend 
10 50 10 100
No. of shares purchased = 200 = Rs. 50000
 Face value of 200 shares = Rs. 200 × 50
(ii) Income of 72 shares = 72 × 5 = Rs. 360
= Rs. 10000
. (iii) Rate of interest on investment = 4%
64
 Dividend = Rs. 10000 × = Rs. 640 Ans.  Market value of each share
100
27. Rs. 100 shares of a company are sold at a 100
discount of Rs. 20. If the return on the  ×5  Rs.125 Ans.
4
investment is 15%. Find the rate of dividend
30. A man sold some Rs. 100 shares paying
declared.
10% dividend at a discount of 25% and
Sol. Market value of each shares = 100 – 20
invested the proceeds in Rs. 100 shares
= Rs.80
paying 16% dividend quoted at Rs. 80 and
 Interest on investment of Rs. 80
thus increased his income by Rs. 2000.
15 Find the number of shares sold by him.
= 15% × 80  ×80  Rs.12
100 Sol. Face value of each share = Rs. 100
 Dividend on face value of Rs.100 = Rs. 12
Market value of each share
Rate of dividend = 12%. Ans.
28. A company declared a dividend of 14%. = Rs. 100 – Rs.25 = Rs. 75
Find the market value of Rs. 50 shares if Rate of dividend = 10%
the return on the investment was 10%. Let no. of shares = x
Sol. Rate of dividend = 14%  Selling price = x × 75 = Rs. 75x
14 ×50
 Dividend on Rs. 50   Rs. 7 Face value of x share = 100 x
100
Now Rs. 10 is interest on the investment 10
of = Rs. 100 Dividend annually  100 x ×  10 x
100
100× 7 75 x 15 x
 Rs. 7 will be the interest on   Rs. 70 No. of shares purchased  
10 80 16
Hence Market value of Rs. 50 shares 15 x 15
= Rs. 70Ans. Face value of shares  x × 100
16 16
29. A company with 10000 shares of Rs. 100
each, declares an annual dividend of 5%. 1500
 x.
(i) What is the total amount of dividend paid 16
by the company ? (ii) What would be the 1500 16
annual income of a man, who has 72 shares, Dividend  x  15 x
16 100
in the company ? (iii) If he received only  Increase in income = 15x – 10x = 5x
4% on his investment, find the price he Now 5x = 2000
paid for each share. (1998) 2000
Sol. No. of shares = 10000  x  400
5
Face value of each share = Rs. 100  No. of shares purchased = 400
Rate of dividend = 5% 31. A man invests Rs. 6750, partly in shares
of 6% at Rs. 140 and partly in shares of
(i) Total face value of 10000 shares
5% at Rs. 125. If his total income is Rs.
= Rs. 100 × 10000 = Rs. 1000000 280, how much has he invested in each ?
35 Arun Deep's Understanding Math-10
Sol. Let the investment in first case = x  23x = 24 × 20304 – 24x
Then investment in second case = (6750 – x)  23x + 24x = 24 × 20304
In first case, the dividend  47x = 24 × 20304
6 3 24  20304
 x×  Rs. x  x= = 10368
140 70 47
and dividend in second case  Amount invested in first kind of shares
5 6750  x
 ( 6750  x ) ×  Rs. = `10368
125 25 and in second kind of shares
3 6750  x = `20304 – `10368 = `9936
 Total dividend  x
70 25
3 6750  x MULTIPLE CHOICE QUESTIONS
 x  280
70 25 Choose the correct answer from the given four
 15x + 14 (6750 – x) = 280 × 350 options (1 to 7) :
(L.C.M. = 350) 1. If Jagbeer invest `10320 on `100 shares at a
 15x + 14 × 6750 – 14x = 280 × 350 discount of `14, then the number of shares
 x = 280 × 350 – 14 × 6750 he buys is
= 98000 – 94500 = Rs. 3500 (a) 110 (b) 120 (c) 130 (d) 150
 Investment in first case = Rs. 3500 Sol. Investment = `10320
and investment in second case Face value of each share = `100
= 6750 – 3500 = Rs. 3250 Ans. M.V. of each share = `100 – 14 = `86
32. Divide Rs. 20304 into two parts such that
10320
part is invested in 9% Rs. 50 shares pre-  No. of shares = = 120 (b)
mium and the other part is invested Rs. 25 86
shares at 8% discount, then the incomes 2. If Nisha invests `19200 on `50 shares at a
from both the investment are equal. premium of 20%, then the number of shares
Sol. Total amount = `20304 she buys is
Let amount invested in 9% `50 at 8% (a) 640 (b) 384 (c) 320 (d) 160
premium = x Sol. Investment = `19200
Then amount invested in 8% `25 at 8% Face value of each share = `50
Discount = 20304 – x 120
Income from both investments are equal M.V. = `50 × = `60
100
Now income from first type of shares
19200
x9 9x x  Number of shares = = 320 (c)
= = = 60
100  8 108 12
3. `40 shares of a company are selling at 25%
Income from second type of shares
premium. If Mr. Jacob wants to buy 280
(20304  x )  8 shares of the company, then the investment
=
100  8 required by him is
(20304  x )  8 2( 20304  x ) (a) `11200 (b) `14000 (c) `16800 (d) `8400
= = Sol. Face value of each share = `40
92 23
In both cases, annual income is same 125
 M.V. = 40 × = `50
x 2( 20304  x ) 100
 =
12 23  Number of shares = 280
 23x = 24(20304 – x) (By cross multiplication)  Total investment = `280 × 50 = `14000 (d)
36 Arun Deep's Understanding Math-10
4. Arun possesses 600 shares of `25 of a Dividend = 15%
company. If the company announces a Annual income = `3000
dividend of 8%, then Arun's annual income
is Let x be the share, then

(a) `48 (b) `480 F.V. of shares = x × 50 = `50x

(c) `600 (d) `1200 50 x  15


Dividend = = 30
Sol. Number of shares = 600 100
F.V. of each share = `25 15 3000  2
Rate of dividend = 8% = x = 3000  x =
2 15
8  x = 400
 Annual income = 600 × 25 ×
100  Number of shares = 400 (b)
= `1200 (d) 7. `25 shares of a company are selling at `20.
5. A man invests `24000 on `60 shares at a If the company is paying a dividend of 12%,
discount of 20%. if the dividend declared by then the rate of return is
the company is 10%, then his annual income (a) 10% (b) 12%
is (c) 15% (d) 18%
(a) `3000 (b) `2880 Sol. F.V. of each share = `25
(c) `1500 (d) 1440 M.V. = `20
Sol. Investment = `24000 Rate of dividend = 12%
F.V. of each share = `60
12
80  Dividend on each share = × 25 = `3
100
M.V. at discount of 20% = 60 × = `48
100
 Return on `20 = `3
Rate of dividend = 10%
3 5
500  60 and on `100 = ` × = 15% (c)
20 100
 Face value of all the share =
48
CHAPTER TEST
= `30000
1. If a man received `1080 as dividend from
30000  10 9% `20 shares, find the number of shares
Annual dividend = ` purchased by him.
100
9 9
= `3000 (a) Sol. Income on one share  × 20  `
100 5
6. Salman has some shares of `50 of a
company paying 15% dividend. If his annual 5
 No. of shares  1080 ×
income is `3000, then the number of shares 9
he possesses is = 120 × 5 = 600 Ans.
(a) 80 (b) 400 2. Find the percentage interest on capital
(c) 600 (d) 800 invested in 18% shares when a `10 share
Sol. F.V. of each share = `50 costs `12.
37 Arun Deep's Understanding Math-10
Sol. Dividend on one share = 18% of `10 (ii) the number of shares purchased.

18 10 9
(iii) the new income.
= =` (iv) the rate percent which he earns on his
100 5
investment.
9
Income on `12  Sol. Rate of dividend = 2·5% and market price
5
= `92
9 100
Then income of `100  ×  15 Let number of shares purchased = x.
5 12
 Percentage interest on capital = 15% Ans.  Selling price of x shares = 92 x

3. Rohit Kulkarni invests `10000 in 10% `100  Income from investing


shares of a company. If his annual dividend 92 x × 2 . 5
is `800, find : `x 
92
(i) The market value of each share.
92 x × 25 5
(ii) The rate percent which he earns on his   x
92 × 10 2
investment.
Sol. Investment = `10000 Again by investing 92 x in 5% at `115
Face value of each share = `100 92 x × 5
the dividend   4x
Rate of dividend = 10% 115
Annual dividend = `800 5 3
Difference  4 x  x  x
10000×10 2 2
(i)  Market value   `125
800 3
 x  90
(ii) Rate percent on investment 2
800 × 100 90 × 2
  8% Ans.  x   60
10000 3
4. At what price should a 9% `100 share be (i)  No. of shares = 60
quoted when the money is worth 6% ?
92 x
Sol. If interest is 6 then investment = `100 (ii) No. of shares sold 
115
and if interest is 9, then investment
92 × 60
100  9  = 48
=` = `150 115
6
(iii) New income = 4x = 4 × 60 = `240
 Market value of each share = `150 Ans.
(iv) Rate percent interest on investment
5. By selling at `92, some 2·5% `100 shares
and investing the proceeds in 5% `100 5 × 100 100
 
shares at `115, a person increased his 115 23
annual income by `90. Find:
8
(i) the number of shares sold. 4 % Ans.
23
38 Arun Deep's Understanding Math-10
6. A man has some shares of Rs. 100 par value Sol. Total investment = Rs. 101520
paying 6% dividend. He sells half of these at a
discount of 10% and invests the proceeds in Let investment in first part = x
7% Rs. 50 shares at a premium of Rs. 10. This and in second part = (101520 – x)
transaction decreases his income from divi-
dends by Rs. 120. Calculate : Market value of first kind of shares
(i) the number of shares before the transaction. = Rs. 100 – Rs. 8 = Rs. 92
(ii) the number of shares he sold. and rate of dividend = 8%
(iii) his initial annual income from shares. x ×8 2x
Sol. Let no. of shares = x  Dividend   Rs.
92 23
Value of x shares = x × 100 = 100 x Market value of second kind = (101520 – x)
100 x  6 Rate of dividend = 9%
and dividend = = Rs. 6x
100
100  8 108
6x and market value = Rs.  50   50
and dividend on half-shares = Rs.  Rs. 3 x 100 100
2 = Rs. 54
x
Now, no. of shares he sold out  9 101520  x
2 Dividend = (101520 – x) × =
2  54 12
Amount received at 10% discount
x 2 x 101520  x
  90 = Rs. 45x  According to the sum 
2 23 12
In investing Rs. 45x, no. of shares he  24x = 101520 × 23 – 23x
45 x  24x + 23x = 101520 × 23
purchased =  47 x = 101520 × 23
60
45 x 225 x 101520 × 23
Amount of shares   50 = Rs. x   49680
60 6 47
225 7 21x  Investment of first part = Rs. 49680
Income at the rate of 7%  x 
6 100 8 and in second part
21x 3 x
Difference in income  3 x   = Rs. 101520 – Rs. 49680 = Rs. 51840 Ans.
8 8
8. A man buys Rs. 40 shares of a company
3x
According to the condition,  120 which pays 10% dividend. He buys the
8 shares at such a price that his profit is 16%
120  8 on his investment. At what price did he
 x   320
3 buy each share ?
(i) No. of share he hold initially = 320 Sol. Face value of each share = Rs. 40
320 Dividend = 10%
(ii) No. of shares he hold later =  160
2 Gain on investment = 10%
(iii) Amount of income initially = 320 × 6 40×10
 Dividend on Rs. 40   Rs. 4
= Rs. 1920 Ans. 100
Now Rs. 16 is interest on the market value
7. Divide Rs. 101520 into two parts such that = Rs. 100
if one part is invested in 8% Rs. 100 shares
 Market value if interest is Rs. 4
at 8% discount and the other in 9% Rs. 50
shares at 8% premium, the annual incomes 100 4
are equal. = = Rs. 25 Ans.
16
39 Arun Deep's Understanding Math-10
9. A person invested 20%, 30% and 25% of his 30
savings in buying shares at par values of = Rs. 50000 × = Rs. 15000
100
three different companies A, B and C which
declare dividends of 10%, 12% and 15% and amount of investment in C company
respectively. If his total income on account
25
of dividends be Rs. 4675, find his savings = Rs. 50000 × = Rs. 12500
and the amount which he invested in buying 100
shares of each company. 10. Virat and Dhoni invest `36000 each in buying
Solution— shares of two companies. Virat buys 15%
Investment in 3 companies A, B and C = 20%, `40 shares at a discount of 20%, while Dhoni
30% and 25% buys `75 shares at a premium of 20%. If
Let total investment = Rs. 100 both receive equal dividends at the end of
 Investment in A company = Rs. 20 the year, find the rate percent of the dividend
Rate of dividend = 10% declared by Dhoni's company.
Sol. In the first case :
20  10
 Dividend = Rs. = Rs. 2 Investment made by Virat = Rs. 36000
100
and market value at a discount of 20%
Investment in B company = Rs. 30
Rate of dividend = 12% 80
= Rs. 40 × = Rs. 32
30  12 100
 Dividend = Rs.
100 36000  40
 Total face value = Rs.
36 32
= Rs. = Rs. 3.60
10 = Rs. 45000
Investment in company = Rs. 25 Rate of dividend = 15%
Rate of dividend = 25%
45000  15
25  15 375  Total dividend = Rs. = Rs. 6750
 Dividend = Rs. = Rs. 100
100 100
In second case,
= Rs. 3.75
Investment = Rs. 36000
Total dividend = Rs. 2 + 3.60 + 3.75
= Rs. 9.35 Dividend of Dhoni = Rs. 6750
If dividend is Rs. 9.35, then total savings = Face value of each share = Rs. 75
Rs. 100 and market value at premium of 20%
If dividend is Rs. 4675, then total savings
120
4675  100 = Rs. 75 × = Rs. 90
100
=
9.35
75
4675  100  100 Face value = 36000 ×
90
= Rs. 30000
= = Rs. 50000
935
and amount of investment in shares of A 6750  100
 Rate of dividend = %
30000
20
company = Rs. 50000 × = Rs. 10000
100 45
= % = 22.5%
Amount of investment in B co mpany 2
4
Linear Inequations
POINTS TO REMEMBER
1. Inequation among real numbers :
(i) Let a and b be two real numbers, then a is less than b and is written as a < b if and only if b –
a is a positive.
(ii) ‘a’ is less than or equal to ‘b’ and is written as a  b if and only if b – a is either positive or zero.
(iii) ‘a’ is greater than ‘b’ and is written as a > b if and only if a – b is positive.
(iv) ‘a’ is greater than or equal to ‘b’ and is written as a  b if and only if a – b is positive or zero.
2. Number Line : Any straight line which represents number positive or negative is called a
number line.
3. Linear Inequation : An equation which is written as x < 5, x + 3  4 or 3x + 4 > 7 etc. are
called linear inequations or linear inequalities. It is, in general, written as
ax + b < 0 or ax + b  0, ax + b > 0 or ax + b  0.
4. Replacement Set : The set from which values of the variable in the given inequation are
chosen is called Replacement set.
5. Solution Set : A solution to an inequation is a number (chosen from replacement set) which
makes the inequation true if substituted in that inequation. The set of all solutions of the given
inequation is called its solution set
6. Addition or subtraction to both sides of an inequation is same as we do in the case of equations.
7. If both sides of an inequation, are multiplied or divided by the same positive number, the
resulting inequation has the same solution set as in original.
8. When we multiply or divide the given inequation by a negative number, the symbol of inequali-
ties are reversed.

EXERCISE 4
1. Solve the inequation 3x – 11  3 where x  2. Solve 2(x – 3) < 1, x  {1, 2, 3, ....., 10}
{1, 2, 3, ........., 10}. Also express its 1
solution on a number line. Sol. Given, 2 (x – 3) < 1  x – 3 
2
Sol. Given, 3x – 11  3 1 1
14  x 3  x 3
 3x  3 + 11  3x < 14  x  2 2
3
But x  {1, 2, 3, ........... , 10}
But x  {1, 2, 3, .........., 10}
 Solution set = {1, 2, 3} Ans.
 Solution set is {1, 2, 3, 4}Ans.
Solution set represented on real line is given
Solution set on number line is represented below :
as below :
X’ X
0 1 2 3 4
40 Arun Deep's Understanding Math-10
41 Arun Deep's Understanding Math-10
3. Solve : 5 – 4x > 2 – 3x, x  w. Also 2x  3 1
represent its solution on the number line. P.Q. Solve :  , x  {0, 1, 2, ... , 8}
4 2
Sol. Given, 5 – 4x > 2 – 3x
 – 4x + 3x > 2 – 5  – x > – 3 2x  3 1 4
Sol. Given,   2x – 3 
 x < 3 ( if a > b  a c < b c when c < 0) 4 2 2
. . . x  W,  2x – 3  2  2x  2 + 3
 Solution set = {0, 1, 2} 5
 2x  5  x 
Solution set on Number Line is represented 2
below x  {0, 1, 2, ............, 8}
. ..
X’ X  Solution set = {3, 4, 5, 6, 7, 8} Ans.
0 1 2 3
4. List the solution set of 30 – 4(2x – 1) < 30, 7. Solve x – 3 (2 + x) > 2 (3x – 1), x  {– 3,
given that x is a positive integer. – 2, – 1, 0, 1, 2, 3}. Also represent its
Sol. Given, 30 – 4 (2x – 1) < 30 solution on the number line.
 30 – 8x + 4 < 30 Sol. Given, x – 3 (2 + x) > 2 (3x – 1)
 – 8x < 30 – 30 – 4  x – 6 – 3x > 6x – 2
 – 8x < – 4  x – 3x – 6x > – 2 + 6  – 8x > 4
4 1 4 1
 x  x  x  x
8 2 8 2
... x  {– 3, – 2, – 1, 0, 1, 2}
[if c < 0, a < b  ac > bc]
. ..
x is a positive integer  Solution set = {– 3, – 2, – 1}
 x = {1, 2, 3, 4 ......... } Ans. Solution set on Number Line is represented
by dark dots as shown below :
5. Solve : 2 (x – 2) < 3x – 2, x  {– 3, – 2, –
1, 0, 1, 2, 3} X’ X
–3 –2 –1 0 1 2 3
Sol. Given, 2 (x – 2) < 3x – 2
8. Given x  {1, 2, 3, 4, 5, 6, 7, 9} solve
 2x – 4 < 3x – 2
 2x – 3x < – 2 + 4  – x < 2 x – 3 < 2x – 1.
 x>–2 Sol. Given, x – 3 < 2x – 1
But x {–3, –2, –1, 0, 1, 2, 3}  x – 2x < – 1 + 3  – x < 2
 Solution set = {– 1, 0, 1, 2, 3} Ans. x>–2
6. If x is a negative integer, find the solution  But x  {1, 2, 3, 4, 5, 6, 7, 9}
2 1
set of  ( x  1)  0 .  Solution set = {1, 2, 3, 4, 5, 6, 7, 9} Ans.
3 3
2 1 1 P.Q. Given A = {x : x  I, – 4  x  4}, solve
Sol. Given,  x   0 2x – 3 < 3 where x has the domain A. Graph
3 3 3
1 1 the solution set on the number line.
 x 1 0  x  1 Sol. Given, 2x – 3 < 3
3 3
3
x  1×  x > – 3  2x < 3 + 3  2x < 6  x < 3

1 But x has the domain A = {x : x  I – 4  x
... x is a negative integer  4}
 Solution set = {– 2, – 1} Ans.  Solution set = {– 4, – 3, – 2, – 1, 0, 1, 2}
42 Arun Deep's Understanding Math-10
Solution set on Number line is represented 5 8
2x
by dark dots is shown below :   <– <
2 3 6
X’ X
–4 –3 –2 –1 0 1 2 3   15 <  4x < 8
9. List the solution set of the inequation 15
1 3  15 > 4x >  8  > x > 2
 8 x  5 x  , x  Z. 4
2 2
3
 3 > x > 2
1 3 4
Sol. Given,  8 x  5x 
2 2 But x  N, hence only possible solution
3 1 for x = {1, 2, 3}
 8 x  5x   
2 2 Solution set represented on number line
2 by dark dots as shown below :
 3x > – 2  x  
3
given, x  Z,
... –4 –3 –2 –1 0 1 2 3 4
 Solution set = {0, 1, 2, 3, 4, .......} Ans. 12. If x  W, find the solution set of
10. List the solution set of 3 2x  1
x 1
11  2 x 9  3 x 3 5 3
  , x  N. Also graph the solution set on the number
5 8 4
line, if possible.
11  2 x 9  3 x 3
Sol. Given,   3 2x  1
5 8 4 Sol. Given, x  1
5 3
 88 – 16x  45 – 15x + 30 9x – (10x – 5) > 15 (L.C.M. of 5, 3 = 15)
(L.C.M. of 8, 5, 4 = 40}  9x – 10x + 5 > 15
 – 16x + 15x  45 + 30 – 88  – x > 15 – 5  –x > 10  x < – 10
 – x  – 13  x  13 But x  W
... x  N.  Solution set = .
 Solution set = {1, 2, 3, 4, 5, .. , 13} Ans. Hence it can’t be represented on number
11. Find the values of x, which satisfy the line.
1 2x 5 13. Solve :
inequation : 2    1 , x  N.
2 3 6
Graph the solution set on the number line. x x
(i)  5   6, where x is a positive odd
(2001) 2 3
integer.
1 2x 5
Sol. Given,  2 <  <1 ,x  N 2 x  3 3x  1
2 3 6 (ii)  , where x is positive even
3 4
1 1 2x 1 11 1 integer.
 2 <   < 
2 2 3 2 6 2 x x
Sol. (i) Given, 5 6
1 2 3
[By subtracting on both sides of inequality] x x 3x  2 x
2    6  5  1
2 3 6
43 Arun Deep's Understanding Math-10
x 24
  1  x  6  5x  12 + 12  x 
6 5
... x is a positive odd integer.
4
 x4
x = {1, 3, 5} Ans. 5
2 x  3 3x  1 4
(ii) Given,  Thus 4  x  6
3 4 5
But x  I
2 x 3 3x 1
     x = { 5, 6} Ans.
3 3 4 4
Solution represented on number line by dark
2 x 3 x 1 dots as shown below :
   1
3 4 4
8x  9 x 5
  P.Q. Given x  {1, 2, 3, 4, 5, 6, 7, 9}, find the
12 4
values of x for which –3 < 2x – 1 < x + 4.
 x 5 x 5 Sol. Given, – 3 < 2x – 1 < x + 4
   
12 4 12 4  – 3 < 2x – 1 and 2x – 1 < x + 4
[If a < b and c < 0  ac > bc]  – 2x < – 1 + 3 and 2x – x < 4 + 1
5  – 2x < 2 and x < 5
 x   12  x  15
4  –x<1  x>–1
x is positive even integer
 –1<x<5
. . . x  {1, 2, 3, 4, 5, 6, 7, 9}
x = {2, 4, 6, 8, 10, 12, 14} Ans.
14. Given that x  I, solve the inequation and  Solution set = {1, 2, 3, 4} Ans.
graph the solution on the number line : 15. Solve : 1  15 – 7x > 2x – 27, x  N
Sol. Given, 1  15 – 7x > 2x – 27
x4 x
3   2. (2004) 1  15 – 7x and 15 – 7x > 2x – 27
2 3
 7x  15 – 1 and – 7x – 2x > – 27 – 15
x4 x x4 x  7x  14 and –9x > – 42
Sol. Given, 3   and  2
2 3 2 3
42
3 x  12  2 x  x  2 and  x  
(i) Now, 3  9
6
14 14
5 x  12  2  x and  x    x
3 3 3
6
14
 18  5x – 12  5x – 12  18  2x
3
 5x  18 + 12 But x  N
 5x  30  x  6  Solution set = {2, 3, 4} Ans.
x4 x 16. If x  Z, solve 2 + 4x < 2x – 5  3x. Also
(ii) Also,  2 represent its solution on the number line.
2 3
3 x  12  2 x Sol. Given, 2 + 4x < 2x – 5  3x
 2 2 + 4x < 2x – 5 and 2x – 5  3x
6
5 x  12  4x – 2x < – 5 – 2 and 2x – 3x  5
 2  5x – 12  12  2x < – 7 and – x  5
6
44 Arun Deep's Understanding Math-10
7 1
 x and x  – 5 and – 5  x  Solution set = {x : x  R, x  }
2 7
7 Solution set on the number line is
 5  x  
2 1
But x  Z represented by dark line and dark dot at
7
 Solution set = {– 5, – 4}
1
Solution set on Number line is represented shows that included in the solution set.
by dark dots as shown below : 7
X’ X
–6 –5 –4 –3 –2 –1 0 1 2 3
3x 2 x  1
5 18. Solve   1 , x  R and repre-
P.Q. Solve the inequation = 12  1 x  5  3 x , 5 3
6 sent the solution set on the number line.
x  R. Represent the solution on a number
3x 2 x  1
line. (1999) Sol. Given,  1
5 3
5  9x – (10x – 5) > 15
Sol. Given, 12  1 x  5  3 x
6  9x – 10x + 5 > 15
11  – x > 15 – 5  –x > 10  x < – 10
 12  x  5  3x ... x  R.
6
 72 + 11x  30 + 18x (Multiplying by 6)  Solution set = {x : x  R, x < – 10}
Solution set on the number line is repre-
 11x – 18x  30 – 72 sented by dark line is shown below
42
 – 7x  – 42   x  
7
 –x  – 6  x  6
P.Q. Solve the inequation – 3  3 – 2x < 9, x 
 xR R. Represent your solution on a number
 Solution set = {x : x  R, x  6} line. (2000)
Solution set on Number line is represented Sol. Given, – 3  3 – 2x < 9
by dark dots is given below : – 3  3 – 2x and 3 – 2x < 9
 2x  3 + 3 and – 2x < 9 – 3
 2x  6 and – 2x < 6  x  3 and – x < 3
4 x  10 5 x  7  x  3 and – 3 < x
17. Solve :  , x  R and  – 3 < x  3.
3 2
represent the solution set on the number Solution set = {x : x  R, – 3 < x  3}
line. Solution set represented on number line is
4 x  10 5 x  7 shown below :
Sol. Given, 
3 2 hollow dot at –3 represents that –3 is not
 8x – 20  15x – 21 included in the solution set.
(L.C.M. of 3, 2 = 6)
 8x – 15x  – 21 + 20
P.Q. Solve 2  2x – 3  5, x  R and mark it on
1 1
 – 7x  – 1   x    x number line. (2003)
7 7
Sol. Given, 2  2x – 3  5
... xR
45 Arun Deep's Understanding Math-10
 2  2x – 3 and 2x – 3  5 we get
 2 + 3  2x and 2x  5 + 3   8 < 3x + 1 < 10
  8  1 < 3x + 1  1 < 10  1
 5  2x and 2x  8. [Add  1]
5
 x and x  4   9 < 3x < 9

2  3 < x < 3
5 [Dividing by 3]
 x4
2
RS 5 UV Hence, the solution set is {x : x  R,  3
 Solution set = x : x R , 2  x  4 < x < 3}
T W
Solution set on number line is represented
below : –3 –2 –1 0 1 2 3
The graph of the solution set is shown
by the thick portion of the number line.
The solid circle at  3 indicates that the
19. Given that x  R, solve the following number  3 is included among the
inequality and graph the solution on the solutions whereas the open circle at 3
number line: indicates that 3 is not included in the
 1 < 3 + 4x < 23. (2006) solutions.
Sol. We have 21. Solve the following inequation and represent
 1 < 3 + 4x < 23, x  R
the solution set on the number line :

  1  3 < 4x < 23  3   4 < 4x < 20 1 2x 5


–3 < – – < , x  R. (2010)
2 3 6
  1 < x < 5, x R
 Solution set = {  1 < x < 5 ; x  R} Ans.
1 2x 5
Sol. Given, –3 < – – < ,xR
2 3 6
Hollow dot at x = 5 shows that +5 is not
included in solution set. 1 2x  1 2x 
(i) –3 < – –  –3 < –   
The graph of the solution set is shown 2 3 2 3 
below :
 1 2x  2x 1
 –   > –3  – > –3 +
-2 -1 0 1 2 3 4 5 6
2 3  3 2
20. Solve the following inequation and graph 2x 5 2x 5 5 3
the solution on the number line. (2007)  – >  < x< ×
3 2 3 2 2 2

2 1 1 15
2 <x+ <3+ ;x  R.  x< ....(i)
3 3 3 4
1 2x 5 2x 5 1
2 1 1 (ii) Also, – – < – < +
Sol. Given  2 < x +
3 3
<3+ ;x
3
R 2 3 6 3 6 2
2x 53 2 8 2 8
 <  x<  x>
8 1 10 3 6 3 6 3 6
 <x+ <
3 3 3 8 3
 x> ×  x > –2
Multiplying by 3, L.C.M. of fractions, 6 2
46 Arun Deep's Understanding Math-10
 –2 < x ....(ii)  –3x + 21 > 15 – 7x
 From (i) and (ii), we have  –3x + 7x > 15 – 21  4x > –6
15 6 3 3
–2 < x <
4  x> x>  <x ...(1)
4 2 2
 15  x 1
 Solution =  x : x  R ,2  x   and 15 – 7x >
 4 3
Now solution set on number line represented  45 – 21x > x + 1
15 15  45 – 1 > x + 21x
hollow dot at x = means that x = is
4 4  44 > 22x
not included in solution set. 2>xx<2 ...(2)
2 15/4 From (1) and (2) ; we have
6 5 4 3 2 1 O 1 2 3 4 5 6 7 Solution set represented on number line is
given below hollow dot at x = 2 means that
2x 1
P.Q. Solve + 2 (3 – x)  7, x  R. Also the point x = 2 is not included in solution set.
2
graph the solution set on the number line. 3
 < x < 2, x  R
2x 1 2
Sol. Given,  2 (3  x)  7
2
2x 1 3 2 1 0 1 2 3 4 5
  6  2 x7
2 23. Solve the following inequation, write down
2x 1 the solution set and represent it on the real
  2 x7  6 number line :
2
2 x 1 4 x –2 + 10x 13x + 10 < 24 + 10x, x  Z
 1  2 x + 1 – 4 x  2
2 Sol. Given, –2 + 10x  13x + 10 < 24 + 10x, x  Z
 –2x2–1–2x1
 –2 – 10  13x – 10x
1 1
 x  x  –12  3x
2 2

RS 1
Solution set x : x R, x   2
UV  –4  x
T W
Solution set on number line is represented Also 13x + 10 < 24 + 10x
as shown below :  13x – 10x < 24 – 10

2
22. Solving the following inequation, write the 3x < 14 x<4
3
solution set and represent it on the number
line. 2
 –4  x < 4
x 1 3
–3(x – 7) > 15 – 7x > , x  R (2016)
3
–4 –3 –2 –1 0 1 2 3 4
x 1
Sol. Given, –3(x – 7) > 15 – 7x > ,xR  Solution set = {–4, –3, –2, –1, 0, 1, 2, 3, 4]
3 Ans.
 –3(x – 7) > 15 – 7x 24. Solve the inequation 2x – 5 < 5x + 4 < 11,
747
2 Arun Deep's Understanding Math-10
where x  I. Also represent the solution set  4 x < 12 + 5  4 x < 17
on the number line. (2011)
17
Sol. Given, 2x – 5 < 5x + 4 < 11  x
4
2x – 5 < 5x + 4 and 5x + 4 < 11
 2x – 5 – 4 < 5x and 5x + 4 < 11 But x  W
 2x – 9 < 5x and 5x < 11 – 4  Solution set Q = {4, 3, 2, 1, 0}.
and 5x < 7 (i) P  Q = {2, 3, 4} (ii) Q – P = {1, 0}.
7 27. A = {x : 11x  5 > 7x + 3, x  R} and
 2x – 5x < 9 and x<
5
B = {x : 18x  9 > 15 + 12x, x  R}.
 3x > –9 and x < 1.4
 x > –3 Find the range of set A  B and represent
Thus, –3  x < 1.4 it on a number line
Solution set represented on real line is given Sol. Given, A = {x : 11x–5 > 7x+3, x R}
below : and B = {x : 18x – 9  15 + 12x, x  R}
X’ X Now, A = 11x – 5 > 7x + 3
–3 –2 –1 0 1 2 3  11x – 7x > 3 + 5  4x > 8
25. If x  I, A is the solution set of  x > 2, x  R
2 (x – 1) < 3 x – 1 and B is the solution set and B = 18x – 9 15+12x
of 4 x – 3  8 + x, find A  B.  18x – 12x 15+9  6x 24  x 4
Sol. Given, 2 (x – 1) < 3 x – 1  A  B = x  4, x  R
 2x–2<3x–1 Hence Range of A  B = {x : x 4, x
R} and its graph will be.
 2x–3x<–1+2–x<1
 x > – 1 but x  I -3 -2 -1
 Solution set A = {0, 1, 2, 3, ....} 28. Given : P = {x : 5 < 2 x – 1  11, x  R}
Also, 4 x – 3  8 + x Q = {x : – 1  3 + 4 x < 23, x  I} where
R = {real numbers}, I = {integers}
11
 4 x – x  8 + 3  3 x  11  x  Represent P and Q on number line. Write down
3
the elements of P Q. (1996)
 Solution set B = {3, 2, 1, 0, – 1, .....}
Sol. Given, P = {x : 5 < 2 x – 1  11}
 A  B = {0, 1, 2, 3} Ans.
5 < 2 x – 1  11
26. If P is the solution set of – 3 x + 4 < 2 x – 5 < 2 x – 1 and 2 x – 1  11
3, x  N and Q is the solution set of 4 x – 5
–2x<–5–1 and 2 x  11 + 1
< 12, x  W, find (i) P  Q (ii) Q – P.
– 2 x < – 6 and 2 x  12
Sol. (i) Given, – 3 x + 4 < 2 x – 3
–x<–3 and x6
 –3x–2x<–3–4–5x<–7
x>3 or 3 < x and x6
7 7
 x  x  Solution set = {x R ; 3 < x < 6}
5 5
Thus, 3 < x < 6
(if a < b  ac > bc if c < 0)
Solution set on number line is represented
but x N below :
 Solution set P = {2, 3, 4, 5, .....}
(ii) Also, 4 x – 5 < 12
48 Arun Deep's Understanding Math-10
Q = {– 1  3 + 4 x < 23}  x < 5 and –4 < x, xR
– 1  3 + 4 x < 23  –4 < x < 5, x  R
–1<3+4x and 3 + 4 x < 23 Hence, solution set is {x : –4 < x < 5, x  R}
The solution set is represented on the number
–4x<3+1 4 x < 23 – 3
line as below.
–4x<4 4 x < 20
–x<1 x<5
5 4 3 2 1 O 1 2 3 4 5
 –x>–1
 –1<x 32. Solve the given inequation and graph the
 –1<x<5 solution on the number line :
 Solution set = {x  I, –1 < x < 5} = {0, 1, 2, 3, 2y – 3 < y + 1 < 4y + 7; y  R.
4} Solution— Given, 2y – 3 < y + 1 < 4y + 7; y  R
Solution set on number line (a) 2y – 3 < y + 1
 2y – y < 1 + 3 y < 4  4 > y ....(i)
and P  Q = {4} (b) Also, y + 1 < 4y + 7
29. If x  I, find the smallest value of x which  y – 4y < 7 – 1  –3y < 6
5 5x 6
satisfies the inequation 2 x   2  3y > –6  y >
2 3 3
5 5x
Sol. Given, 2 x  2  y > –2 ....(ii)
2 3
5x 5 From (i) and (ii), we have
 2x  2 
3 2 4 > y > –2 or –2 < y < 4
3 Now representing it on a number given below
 12 x – 10 x > 12 – 15  2 x > – 3  x  
2
xI
.. . 5 4 3 2 1 O 1 2 3 4 5
Thus, x = {–1, 0, 1, 2, 3, ....} 33. Solve the inequation and represent the
 Smallest value of x = – 1 Ans. solution set on the number line.
30. Given 20 – 5 x < 5 (x + 8), find the smallest 8x 14
value of x, when –3 + x < +2< + 2x, where x  I.
(i) x  I (ii) x  W (iii) x  N. 3 3
Sol. Given, 20 – 5 x < 5 (x + 8) 8x 14
 20 – 5 x < 5 x + 40 Sol. Given : –3 + x < + 2 < + 2x,
3 3
 – 5 x – 5 x < 40 – 20
 – 10 x < 20  – x < 2  x>–2 where x  I
(i) When x  I, then smallest value = – 1. 8x
(ii) When x  W, then smallest value = 0. (i) –3 + x < +2
3
(iii) When x  N, then smallest value = 1. Ans.
31. Solve the following inequation and represent 8x 5x x
 –3 – 2 < – x  –5 <  –1 <
the solution set on the number line : 3 3 3
3x 2  –3 < x ...(i)
4x – 19 <  2    x, x  R
5 5 8x 14
3x 2 and +2< + 2x
Sol. We have 4x – 19 < –2< + x, xR 3 3
5 5
3x 3x 2 8x 14 2x 8
 4x – 19 < – 2 and –2< + x, x  R  – 2x < –2 <
5 5 5 3 3 3 3
3x 2 3x  x<4 ...(ii)
 4x – < 17 and –2 + <x– , xR
5 5 5 From (i) and (ii), we have
17x 8 2x –3 < x < 4, But x  I
 < 17 and < , xR Solution set = {–3, –2, –1, 0, 1, 2, 3, 4}
5 5 5
49 Arun Deep's Understanding Math-10
Solution set on number line  Solution set = {–3, –1, 0, 1, 3}  Ans. (b)
X’ X 2. If x  W, then the solution set of the
4 3 2 1 0 1 2 3 4 inequation 3x + 11 > x + 8 is
34. Find the greatest integer which is such that (a) {–2, –1, 0, 1, 2, ...}
if 7 is added to its double, the resulting (b) {–1, 0, 1, 2, ...}
number becomes greater than three times (c) {0, 1, 2, 3, ...}
the integer.
Sol. Let the greatest integer = x  3
(d)  x : x  R , x   
According to the condition,  2
2x + 7 > 3x 2x – 3x > – 7 Sol. Given, 3x + 11 > x + 8
 – x > – 7  x < 7  3x – x > 8 – 11
Value of x which is greatest = 6 Ans.
35. One-third of a bamboo pole is burried in 3 1
 2x > –3  x >  x > –1 but x W
mud, one-sixth of it is in water and the 2 2
part above the water is greater than or equal Solution set = {0, 1, 2, 3, .....}  Ans. (c)
to 3 metres. Find the length of the shortest 3. If x  W, then the solution set of the
pole. inequation 5 – 4x > 2 – 3x is
Sol. Let the length of the shortest pole = x metre
(a) {..., –2, –1, 0, 1, 2, 3}
x
Length of pole which is burried in mud = (b) {1, 2, 3}
3
x (c) {0, 1, 2, 3}
Length of pole which is in the water = (d) {x : x  R, x < 3}
6
According to this problem, we have Sol. Given, 5 – 4x > 2 – 3x
x LM
x x
 OP
 3  x  2 x  x  3
F I  5 – 2 > –3x + 4x  x < 3 but x W
N
3 6 Q H 6 K  Solution set = {0, 1, 2, 3,}  Ans. (c)
x x 4. If x  I, then the solution set of the inequation
 x   3   3 x  6
2 2 1 < 3x + 5 < 11 is
Length of pole (shortest in length) 6 metres
(a) {–1, 0, 1, 2}
MULTIPLE CHOICE QUESTIONS (b) {–2, –1, 0, 1}
Choose the correct answer from the given four (c) {–1, 0, 1}
options (1 to 5) : 4
1. If x  {–3, –1, 0, 1, 3, 5}, then the solution (d) {x : x  R,  < x < 2}
3
set of the inequation 3x – 2 < 8 is Sol. Given, 1 < 3x + 5 < 11
(a) {–3, –1, 1, 3}  1 < 3x + 5  1 – 5 < 3x
(b) {–3, –1, 0, 1, 3}
(c) {–3, –2, –1, 0, 1, 2, 3} 4
 –4 < 3x  <x
(d) {–3, –2, –1, 0, 1, 2} 3
Sol. Given, x  {–3, –1, 0, 1, 3, 5} and 3x + 5 < 11  3x < 11 – 5  3x < 6
Also, 3x – 2 < 8 6
 3x < 8 + 2  3x < 10  x< x<2
3
10 1
 x< x<3 4
3 3  < x < 2 but x I
3
50 Arun Deep's Understanding Math-10
 Solution set = {–1, 0, 1, 2}  Ans. (a)  x–2x3–5–x–2  x2
5. If x  R, the solution set of 6 < –3 (2x – 4) ... x  R

< 12 is  Solution set is given by {x : x R ; x > 2}


(a) {x : x  R, 0 < x < 1} Solution set on number line is represented
(b) {x : x  R, 0 < x < 1} below
(c) {0, 1}
(d) none of these
Sol. Given, 6 < –3(2x – 4) < 12
Now 6 < –3(2x – 4)  6 < – 6x + 12 4. If x  R (real numbers) and – 1 < 3 – 2 x  7,
find solution set and represent it on a
6
 6x < 12 – 6  6x < 6  x < number line.
6 Sol. Given, – 1 < 3 – 2 x  7
 x<1 ...(i) –1<3–2x and 3–2x7
and –3(2x – 4) < 12  –6x + 12 < 12  2x<3+1 and –2x7–3
 –6x < 12 – 12  –6x < 0  2x<4 and –2x4
 x>0 ...(ii)  x<2 and –x2
From (i) and (iii), we have and x–2
 0<x<1 ... x  R
Solution set = {x : x  R, 0 < x < 1}  Solution set is given by – 2  x < 2 ; x  R
 Ans. (a) Solution set on number line is represented
CHAPTER TEST below
1. Solve the inequation : 5 x – 2  3 (3 – x)
where x  {– 2, – 1, 0, 1, 2, 3, 4}. Also
represent its solution on the number line. 5. Solve the inequation :
Sol. Given, 5 x – 2  3 (3 – x)
 5x–29–3x5x+3x9+2 5x  1 F
x 2 I
3 3x  1
4  1  , x  R.
11
7 H
7 5 K
5 7
 8 x  11  x 
8 5x  1 F x 2 I 3 3x  1
. . . x  {– 2, – 1, 0, 1, 2, 3, 4} Sol. Given, 7  4H 7  5 K  1 
5 7
 Solution set = {– 2, – 1, 0, 1}
5x  1
 4F  I  8  3 x  1
Solution set on number line is represented x 2
 7 H 7 5K 5 7
below
X’ X Multiplying by L.C.M. of 7 and 5 i.e. 35
–2 –1 0 1 2 3
2. Solve the inequations :  25x + 5 – 4 (5x + 14)  56 + 15x – 5
6 x – 5 < 3 x + 4, x  I.  25x + 5 – 20x – 56  56 + 15x – 5
Sol. Given, 6 x – 5 < 3 x + 4  25x – 20x – 15x  56 – 5 – 5 + 56
 6x–3x<4+53x<9  x<3
... x  I  – 10x  102
 Solution set = {– 1, – 2, 2, 1, 0, .....}. 102 51
 x   x 
3. Find the solution set of the inequation 10 5
51
x + 5  2 x + 3 ; x  R.  x
5
Graph the solution set on the number line. ... x  R
Sol. Given, x + 5  2 x + 3
51 Arun Deep's Understanding Math-10

R 51 U
Solution set = S x: x  R , x   5 V
... xR
T W RS 5 UV
6. Find the range of values of x, which satisfy  Solution set = x: x  R , x  2
7  – 4x + 2 < 12, x  R. Graph these
T W
Solution set on number line is represented
values of x on the real number line. as below :
Sol. Given, 7  – 4x + 2 < 12
7  – 4x + 2 and – 4x + 2 < 12
 4x  2– 7 and – 4x < 12 – 2
 4x  –5 and – 4x < 10 8. Find positive integers which are such that
if 6 is subtracted from five times the inte-
5 10
 x and  x  ger than the resulting number cannot be
4 4 greater than four times the integer.
5 5 5 Sol. Let the required positive integer = x
 x and  x  or x >
4 2 2 According to the problem, we have
... x  R
5x – 6 < 4x
5 5
 Solution set  x  5x – 4x < 6  x < 6
2 4
 Solution set = {x : x < 6} and x  I
RS 5 5 UV
= x: x  R ,  2  x  4 = { 1, 2, 3, 4, 5, 6} Ans.
T W 9. Find three smallest consecutive natural
Solution set on number line is represented
below numbers such that the difference between
one-third of the largest and one-fifth of the
smallest is atleast 3.
1 x 2 Sol. Let first least natural number = x
7. If x  R, solve 2x – 3  x   x. then second number = x + 1
3 5
Also represent the solution set on the and third number = x + 2
number line.
According to the condition ;
1 x 2
Sol. Given, 2x – 3  x   x 1 1
3 5 ( x  2)  ( x)  3
3 5
1 x 1 x 2 5x + 10 – 3x > 45
2x  3  x  and x   x
3 3 5 (Multiplying by 15 the LCM of 3 and 5)
3x  1  x 3x  1  x 2  2x > 45 – 10  2x > 35
 2x  3  and  x
3 3 5
35 1
 6x – 9  3x + 1 – x and 15x + 5 – 5x > 6x x>  x > 17
2 2
 6x – 3x + x  1 + 9 and 15x – 6x – 5x > – 5
x is a natural least number
 4x  10 and 4x > – 5
 x = 18
10 5  First least natural number = 19
 x and x  
4 4
Second number = 18 + 1 = 19
5
 x and third number = 18 + 2 = 20
2
5 Hence required least natural numbers are
 x 18, 19, 20 Ans.
2
5
Quadratic Equations in One Variable
POINTS TO REMEMBER.
1. ax2 + bx + c = 0 is a quadratic equation where a, b and c  R and a  0, b  0 and c is a constant.
2. It has two roots, b2 – 4 ac is called discriminant and is denoted by D
 D = b2 – 4ac
3. (i) If D > 0, then roots are real and unequal.
(ii) If D = 0, then roots are real and equal.
(iii) If D < 0, then roots are not real.
4. By factorisation,
ax2 + bx + c = (x – ) ( x – )  x = ,  are roots

b  b 2  4 ac b  D
5. By formula x  or 
4a 2a
EXERCISE 5.1
1. In each of the following, determine whether 2
(ii) Given, 3x2 – 13x – 10 = 0; 5,
the given numbers are roots of the given 3
equations or not : When x = 5, we have
(i) x2 – 5x + 6 = 0; 2, –3 3(5)2 – 13 × 5 – 10 = 75 – 65 – 10
2 = 75 – 75 = 0
(ii) 3x2 – 13x – 10 = 0; 5,  x = 5 is the root of given eqn.
3
(iii) x2 – x + 1 = 0; 1, –1 2
If x = , then
3
1 2
(iv) 6x2 – x – 2 = 0; , 2
2 3 2 2
3  – 13 × – 10
Sol. (i) x2 – 5x + 6 = 0 ; 2, –3  3  3
When x = 2, then
3 4 26
(2)2 – 5 × 2 + 6 = 4 – 10 + 6 = 10 – 10 = 0 = + – 10
9 3
 x = 2 is its root as it satisfies the given eqn.
Where, x = –3, then 4 26 30
= + – 10 = – 10
(–3)2 – 5(–3) + 6 3 3 3
= 9 + 15 + 6 = 30  0 = 10 – 10 = 0
 x = –3 is not its solution as it does not satisifes 2
the given eqn.  x= is also its root as it satisfies the given
3
 2 is root of the equation and –3 is not a root. eqn.
52 Arun Deep's Understanding Math-10
53 Arun Deep's Understanding Math-10

2 2
Hence both 5, are its roots.  x= is also its root.
3 3
(iii) Given, eqn. be, x2 – x + 1 = 0 1 2
Hence , are both the roots of given
Where x = 1, then 2 3
(1)2 – 1 + 1 = 1 – 1 + 1 = 1  0 equation.
P.Q. Check whether the following are quadratic
 x = 1 does not satisfy the given eqn.
equations :
and (–1)2 – (–1) + 1 = 0
2
3
1 + 1 + 1 which is false (i) 3 x – 2x + 5 = 0
 x = –1, does not satisfy the given eqn. (ii) (2x + 1) (3x – 2) = 6(x + 1) (x – 2)
 x = 1, –1 are not the roots of the equation. (iii) (x – 3)3 + 5 = x3 + 7x2 – 1

1 2 3
(iv) Given, 6x2 – x – 2 = 0; , (iv) x – = 2, x  0
2 3 x

1 
If x = , (v) x + = x 2, x  0
2 x

2 Sol.
 1  1
then 6x 2 – x – 2 = 6   –   – 2 3
 2   2  2
(i) Given, 3 x – 2x + 5 = 0
1 1 3 1
=6× + –2= + –2 It is a quadratic equation as it is power of 2.
4 2 2 2
(ii) Given, (2x + 1) (3x – 2) = 6(x + 1) (x – 2)
4
= –2=0  6x2 – 4x + 3x – 2 = 6(x 2 – 2x + x – 2)
2
 6x2 – x – 2 = 6x2 – 12x + 6x – 12
1
Then x =  satisfies the equation  12x – 6x – x = –12 + 2
2
 5x = –10
1
 x= is be root.  x = –2
2
It is not a quadratic equation as it is of degree
2
If x = , then 6x2 – x – 2 1.
3
(iii) Given, (x – 3)3 + 5 = x3 + 7x2 – 1
2
2 2  x3 – 3x2 × 3 + 3x × 9 – 27 + 5 = x3 + 7x2 – 1
= 6  –   – 2
3 3  –9x2 + 27x – 22 – 7x2 + 1 = 0
4 2 8 2  –16x2 + 27x – 21 = 0
=6× – –2= – –2
9 3 3 3  16x2 – 27x + 21 = 0
6 It is a quadratic equation as it is a polynomial
= –2=0 of degree 2.
3
54 Arun Deep's Understanding Math-10

3 L.H.S. = x2 – 2x–4
(iv) Given, x – = 2, x  0
 
x
x2 – 3 = 2x  x2 – 2x – 3 = 0
=  2 
2
– 2  2 –4=2+2–4=0
= R.H.S.
It is a quadratic equation as it is a polynomial
of degree 2.  x = – 2 be its solution.

2 (b) x = –2 2
(v) Given, x + = x 2, x  0
x Substituting x = 2 2 in L.H.S., we have
 x2 +2= x3
 x3 – x2 – 2 = 0

2

=  2 2 – 2 2 2 – 4  
=8–4–4=8–8=0
It is not a quadratic equation as it is a
= R.H.S.
polynomial of degree 3.
2. In each of the following, determine whether  x = –2 2 be its solution as it satisfies the
the given numbers are solutions of the given given eqn.
equation or not : 1
3. (i) If  is a solution of the equation
2
(i) x – 3 3 x + 6 = 0; 3 , –2 3 2
3x2 + 2kx – 3 = 0, find the value of k.
(ii) x2 – 2 x – 4 = 0, x = – 2 , 2 2
2
Sol. (i) x2 – 3 3 x + 6 = 0, x = (ii) If is a solution of the equation
3 , x = –2 3 3
7x2 + kx – 3 = 0, find the value of k.
(a) When x = 3
1
L.H.S. = x2 – 3 3 x + 6 Sol. (i) Since, x = is the solution of given
2

=  32
–3 3 × 3 +6=3–9+6=0
3x2 + 2kx – 3 = 0
 it satisfies given eqn.
= R.H.S. Substituting the value of x in the given
 x= equation, we get
3 be its solution as it satisfies given
2
eqn.  1  1
3   + 2k   – 3 = 0
(b) x = –2 3  2   2 
1
L.H.S. = x2 – 3 3 x + 6  3× –k–3=0
4

= 2 3 
2

– 3 3 2 3 + 6  
3
–k–3=0
= 12 + 18 + 6 = 36  0 4

x = – 2 3 is not its solution 3 9


 k= –3= 
4 4
as x = – 2 3 not satisifes the given eqn.
9
(ii) Given eqn. be, x2 – 2 x – 4 = 0, Hence k = 
4
(a) When x = – 2 (ii) Given, eqn. be, 7x2 + kx – 3 = 0.
55 Arun Deep's Understanding Math-10
2 x = a is its solution so it satisfies given eqn.
x= as its solution so it satisfies given  (a)2 – a (a + b) + k = 0
3
eqn.  a2 – a2 – ab + k = 0  –ab + k = 0
2  k = ab
2 2
 7   +k   –3=0 2
3 3 5. If and –3 are the roots of the equation
3
4 2 px2 + 7x + q = 0, find the values of p and q.
 7× + k–3=0
9 3
2
28 2 Sol. Since, and –3 are the roots of the equation
 –3+ k=0 3
9 3
px2 + 7x + q = 0 ...(1)
2 28
 k=3– 2
3 9 Substituting the value of x = and –3
3
2 27  28 2 1 respectively in given eqn. (1), we get
 k=  k=
3 9 3 9
2
2 2
1 3 1 p  + 7  + q = 0
 k= × = 3 3
9 2 6
4 14
1  p+ +q=0
Hence k = 9 3
6
 4p + 42 + 9q = 0
4. (i) If 2 is a root of the equation  4p + 9q = –42 ...(i)
2
and p(–3) + 7(–3) + q = 0
kx2 + 2 x – 4 = 0, find the value of k.
9p – 21 + q = 0
(ii) If a is a root of the equation
 9p + q = 21 ...(ii)
x2 – (a + b)x + k = 0, find the value of k.
q = 21 – 9p
Sol. (i) Given, kx2 + 2x–4=0 Substituting the value of q in (i) ; we get
x= 4p + 9(21 – 9p) = –42
2 be its root
 4p + 189 – 81p = –42
 it must satisfies given the eqn.
 –77p = –42 – 189 = –231
 k  2 2
+ 2 × 2 –4=0
 p=
231
=3
 2k + 2 – 4 = 0  77
 2k – 2 = 0  q = 21 – 9 × 3
 2k = 2 = 21 – 27 = –6
 p = 3, q = –6
2
 k= =1
2 EXERCISE 5.2
 k=1
Solve the following equations (1 to 24) by
(ii) Given, x2 – x (a + b) + k = 0 factorisation
56 Arun Deep's Understanding Math-10
1.(i) x2 – 3x – 10 = 0 PQ.(i) 4x2 = 3x
Sol. Given, eqn. be, x2 – 3x – 10 = 0 x 2  5x
 x2 – 5x + 2x – 10 = 0 (ii) 0
2
 x (x – 5) + 2 (x – 5) = 0 Sol. (i) Given eqn. be, 4x2 – 3x = 0
 (x – 5) (x + 2) = 0 x (4x – 3) = 0
Either x – 5 = 0, then x = 5
Either x = 0, or 4x – 3 = 0
or x + 2 = 0, then x = – 2
3
Hence x = 5, – 2 Ans.  x = 0 or 4x = 3  x 
4
(ii) x (2x + 5) = 3 (1994)
Sol. Given, eqn. be, x (2x + 5) = 3 3
 x = 0, Ans.
 2x2 + 5x – 3 = 0 4
 2x2 + 6x – x – 3 = 0 x 2  5x
(ii) Given eqn. be, 0
 2x (x +3) – 1 (x + 3) = 0 2
 (x + 3) (2x – 1) = 0  x2 – 5x = 0
Either x + 3 = 0, then x = – 3  x (x – 5) = 0
1 Either x = 0 or x – 5 = 0, then x = 5
or 2x – 1 = 0, then 2x = 1  x 
2 Hence x = 0, 5 Ans.
1
 x = – 3, . PQ.(i) (x – 3) (2x + 5) = 0
2
2.(i) 3x2 – 5x – 12 = 0 Sol. Given, (x – 3) (2x + 5) = 0
Sol. Given, eqn. be, 3x2 – 5x – 12 = 0 Either x – 3 = 0, then x = 3
5
 3x2 – 9x + 4x – 12 = 0 or 2x + 5 = 0 then 2x = – 5  x 
2
 3x (x – 3) + 4 (x – 3) = 0 5
Hence x = 3, Ans.
 (x – 3) (3x + 4) = 0 2
Either x – 3 = 0, then x = 3 (ii) x (2x + 1) = 6 (1993)
4 Sol. Given, eqn. be x (2x + 1) = 6
or 3x + 4 = 0, then 3x = –4  x 
3  2x2 + x – 6 = 0
4
Hence x = 3,  2x2 + 4x – 3x – 6 = 0
3
(ii) 2
21x – 8x – 4 = 0 (1990)  2x (x + 2) – 3 (x + 2) = 0
2  (x + 2) (2x – 3) = 0
Sol. Given, eqn. be, 21x – 8x – 4 = 0
Either x + 2 = 0, then x = – 2
 21x2 – 14x + 6x – 4 = 0
3
 7x (3x – 2) + 2 (3x – 2) = 0 or 2x – 3 = 0, then 2x = 3  x 
2
 (3x – 2) (7x + 2) = 0 3
2 Hence x = – 2, Ans.
Either 3x – 2 = 0, then 3x = 2  x  2
3
3.(i) 3x2 = x + 4 (1992)
2
or 7x + 2 = 0, then 7x = – 2  x  2
Sol. Given, eqn. be, 3x = x + 4
7
2 2  3x2 – x – 4 = 0
Hence x , Ans. we want to find two numbers whose, sum
3 7
57 Arun Deep's Understanding Math-10
be –1 and product = 12  2x2
– 3x + 2x – 3 = 0
by inspection such two numbers are – 4  x (2x – 3) + 1 (2x – 3) = 0
and 3.  (2x – 3) (x + 1) = 0
 3x2 – 4x + 3x – 4 = 0 3
Either 2x – 3 = 0, then 2x = 3  x 
 x (3x – 4) + 1 (3x – 4) = 0 2
or x + 1 = 0, then x = – 1
 (3x – 4) (x + 1) = 0
4 Hence x = 3/2, – 1 Ans.
Either 3x – 4 = 0, then 3x = 4  x 
3 PQ. (i) (x – 4)2 + 52 = 132
or x + 1 = 0, then x = – 1 Sol. Given eqn. be, (x – 4)2 + 52 = 132
4  x2 – 8x + 16 + 25 = 169
Hence x  , 1 Ans.
3  x2 – 8x + 41 – 169 = 0
(ii) x (6x – 1) = 35 (1991)  x2 – 8x – 128 = 0
Sol. Given eqn. be, x (6x – 1) = 35  x2 – 16x + 8x – 128 = 0
 6x2 – x – 35 = 0  x (x – 16) + 8 (x – 16) = 0
 6x2 – 15x + 14x – 35 = 0  (x – 16) (x + 8) = 0
 3x (2x – 5) + 7 (2x – 5) = 0 Either x – 16 = 0, then x = 16
 (2x – 5) (3x + 7) = 0 or x + 8 = 0, then x = – 8
5 Hence x = 16, – 8 Ans.
Either 2x – 5 = 0, then 2x = 5  x 
2 2
5. (i) 3 (x – 2) = 147
7
or 3x + 7 = 0, then 3x = – 7  x  Sol. Given, eqn. be, 3 (x – 2)2 = 147
3
5 7 3 (x2 – 4x + 4) = 147
Hence x  ,  3x2 – 12x + 12 – 147 = 0
2 3
4. (i) 6p2 + 11p – 10 = 0  3x2 – 12x – 135 = 0
Sol. Given eqn. be, 6p2 + 11p – 10 = 0  x2 – 4x – 45 = 0 (dividing by 3)
 6p2 + 15p – 4p – 10 = 0  x2 – 9x + 5x – 45 = 0
 3p (2p + 5) – 2 (2p + 5) = 0  x (x – 9) + 5 (x – 9) = 0
(2p + 5) (3p – 2) = 0  (x – 9) (x + 5) = 0
5 Either x – 9 = 0, then x = 9
Either 2p + 5 = 0, then 2p = – 5  p 
2 or x + 5 = 0, then x = – 5
2
or 3p – 2 = 0, then 3p = 2  p  Hence x = 9, – 5 Ans.
3
5 2 1
Hence p , (ii) ( 3 x  5) 2 = 28
2 3 7
2 2 1 1
(ii) x  x 1 Sol. Given eqn. be, ( 3 x  5) 2 = 28
3 3 7
2 1 (3x – 5)2 = 28 × 7  9x2 – 30x + 25 = 196
Sol. Given eqn. be, x 2  x  1  9x2 – 30x + 25 – 196 = 0
3 3
 2x2 – x = 3  2x2 – x – 3 = 0  9x2 – 30x – 171 = 0
 3x2 – 10x – 57 = 0 (Dividing by 3)
58 Arun Deep's Understanding Math-10
 3x2
– 19x + 9x – 57 = 0 Hence x = 6, – 2 Ans.
 x (3x – 19) + 3 (3x – 19) = 0 PQ. 5x2 – 8x – 4 = 0 ; when x  Q.
 (3x – 19) (x + 3) = 0 Sol. Given eqn. be, 5 x2 – 8 x – 4 = 0 ... 5 ×
Either 3x – 19 = 0, then 3x = 19 (– 4) = – 20
19 – 8 = – 10 + 2
 x or x + 3 = 0, then x = – 3  5 x – 10 x + 2 x – 4 = 0
2
3
19  5 x (x – 2) + 2 (x – 2) = 0
Hence x  , 3 Ans.
3  (x – 2) (5 x + 2) = 0
(ii) 3 (y2 – 6) = y (y + 7) – 3 (Zero Product Rule)
Sol. Given eqn. be, 3 (y2 – 6) = y (y + 7) – 3 Either x – 2 = 0, then x = 2
 3 (y2 – 6) = y2 + 7y – 3 or 5 x + 2 = 0, then 5 x = – 2
 3y2 – 18 = y2 + 7y – 3 2
 3y2 – y2 – 7y – 18 + 3 = 0  x
5
 2y2 – 7y – 15 = 0 2
 2y2 – 10y + 3y – 15 = 0  x  2,  Ans.
5
2y (y – 5) + 3 (y – 5) = 0 7. 2x2 – 9x + 10 = 0, when (i) x  N (ii) x  Q
 (y – 5) (2y + 3) = 0 Sol. Given eqn. be, 2x2 – 9x + 10 = 0
Either y – 5 = 0, then y = 5  2x2 – 4x – 5x + 10 = 0
or 2y + 3 = 0, then 2y = – 3  y = –3/2  2x (x – 2) – 5 (x – 2) = 0
Hence y = – 3/2, 5 Ans.  (x – 2) (2x – 5) = 0
6. x2 – 4x – 12 = 0 when x  N. Either x – 2 = 0, then x = 2,
Sol. Given eqn. be, x2 – 4x – 12 = 0 5
or 2x – 5 = 0, then 2x = 5  x 
 x2 – 6x + 2x – 12 = 0 2
 x (x – 6) + 2 (x – 6) = 0 5
(i) when x  N, then x = 2 as  
2
 (x – 6) (x + 2) = 0
5
Either x – 6 = 0, then x = 6 (ii) when x  Q, then x  2 , Ans.
2
or x + 2 = 0, then x = – 2
8.(i) a2 x2 + 2ax + 1 = 0, a  0
But – 2 is not a natural number
Sol. Given eqn. be, a2 x2 + 2ax + 1 = 0
 x = 6 Ans.
 a2 x2 + ax + ax + 1 = 0
PQ. 2x2 – 8x – 24 = 0 when x  I
 ax (ax + 1) + 1 (ax + 1) = 0
Sol. Given eqn. be, 2x2 – 8x – 24 = 0
 (ax + 1) (ax + 1) = 0
 x2 – 4x – 12 = 0 (Dividing by 2)  (ax + 1)2 = 0
 x2 – 6x + 2x – 12 = 0 1
 ax + 1 = 0  ax = –1 x  
 x (x – 6) + 2 (x – 6) = 0 a
1 1
 (x – 6) (x + 2) = 0 Hence x   ,
a a
Either x – 6 = 0, then x = 6 (ii) 2
x – (p + q) x + pq = 0
or x + 2 = 0, then x = – 2 Sol. x2 – (p + q) x + pq = 0
59 Arun Deep's Understanding Math-10
x2
– px – qx + pq = 0
4 3x 2 + 8x – 3x – 2 3 = 0
x (x – p) – q (x – p) = 0
 4x ( 3 x + 2) – 3 ( 3 x + 2) = 0
 (x – p) (x – q) = 0
Either x – p = 0, then x = p,  ( 3 x + 2) (4x – 3) = 0
or x – q = 0, then x = q Either 3 x + 2 = 0, then 3x = – 2
Hence x = p, q Ans.
2
9. a2 x2 + (a2 + b2) x + b2 = 0, a  0.  x
3
Sol. Given eqn. be, a2 x2 + a2 x + b2 x + b2 = 0
 a2 x (x + 1) + b2 (x + 1) = 0  2 3 2 3
 x 
 (x + 1) (a2 x+ b 2) = 0. 3 3 3
 (x + 1) = 0, then x = – 1 or 4x – 3 = 0, then 4x = 3
or a2 x + b2 = 0, then a2x = – b2
b 2 3
 x
 x 2 4
a
b 2 2 3 3
Hence x = – 1, 2 Ans. Hence x  , Ans.
a 3 4
2
10.(i) 3 x + 10x + 7 3 = 0 11. (i) x2 – (1 + 2) x + 2 =0
Sol. Given eqn. be, 3 x2 + 3x + 7x + 7 3 = 0 Sol. Given, eqn. be, x2 – (1 + 2) x + 2 =0
 x2 – x – 2x + 2 =0
[. . . 3 × 7 3 = 7 × 3 = 21]
 x (x – 1) – 2 (x – 1) = 0
 3 x (x + 3 ) + 7 (x + 3) = 0
 (x – 1) (x – 2) = 0
 (x + 3 ) ( 3 x + 7) = 0
Either x – 1 = 0, then x = 1
Either x + 3 = 0, then x = – 3 or x – 2 = 0, then x = 2
or 3 x + 7 = 0, then 3 x=–7 Hence x = 1, 2 Ans.
7
 x 1 1
3 (ii) Given, eqn. be, x  2
x 20
7 3 7 3
 x  x 2  1 41
3 3 3    20x2 + 20 = 41x
x 20
7 3  20x2 – 41x + 20 = 0
Hence x = – 3,  3
 20x2 – 16x – 25x + 20 = 0
(ii) 4 3 x2 + 5x – 2 3 = 0
  16  (25)  41 
 
Sol. Given eqn. be, 4 3 x2 + 5x – 2 3 = 0  (16)  ( 25)  400 

4 3  ( 2 3)  8  ( 3)  24and 8 + (–3) = 5   4x (5x – 4) – 5 (5x – 4) = 0


 (5x – 4) (4x – 5) = 0
60 Arun Deep's Understanding Math-10
4 or x + 10 = 0, then x = –10
Either 5x – 4 = 0, then 5x = 4  x 
5  x = 15, –10
5 8
or 4x – 5 = 0, then 4x = 5  x  13. (i) 3 x  =2
4 x
4 5 x2 2x  3
Hence x  , Ans. (ii) 
5 4 x  3 3x  7
2  8
12. (i) – + 2 = 0, x  0 Sol. (i) Given eqn. be, 3 x  =2
x2 x x

x2 x 3x 2  8
(ii) – – 10 = 0  =2
15 3 x
 3x2 – 8 = 2x
2   3x2 – 2x – 8 = 0
Sol. (i) Given eqn. be, 2 – + 2 = 0, x  0
x x  3x2 – 6x + 4x – 8 = 0
 2 – 5x + 2x2 = 0  3x (x – 2) + 4 (x – 2) = 0
 (x – 2) (3x + 4) = 0
 2x2 – 5x + 2 = 0  2  2  4 
  Either x – 2 = 0, then x = 2
 4  4  (1)
  5  4  1  4
  or 3x + 4 = 0, then 3x = – 4  x 
3
 2x2 – 4x – x + 2 = 0 Hence x = 2, –4/3 Ans.
 2x(x – 2) – 1(x – 2) = 0
x2 2x  3
 (x – 2) (2x – 1) = 0 (ii) Given eqn. be, 
x  3 3x  7
Either x – 2 = 0, then x = 2
 (x + 2) (3x – 7) = (2x – 3) (x + 3)
1
or 2x – 1 = 0, then 2x = 1  x =  3x2 – 7x + 6x – 14 = 2x2 + 6x – 3x – 9
2
 3x2 – x – 14 = 2x2 + 3x – 9
1  3x2 – x – 14 – 2x2 – 3x + 9 = 0
 x = 2, .
2
 x2 – 4x – 5 = 0  x2 – 5x + x – 5 = 0
x2 x x (x – 5) + 1 (x – 5) = 0
(ii) Given eqn. be, – – 10 = 0
15 3  (x – 5) (x + 1) = 0
 x2 – 5x – 150 = 0 Either x – 5 = 0, then x = 5
or x + 1 = 0, then x = – 1
  150  15  10
  Hence x = 5, – 1 Ans.
  5  15  10 
8 3
 x2 – 15x + 10x – 150 = 0 14. (i) x  3  2  x = 2
 x(x – 15) + 10(x – 15) = 0
8 3
 (x – 15) (x + 10) = 0 Sol. Given, x  3  2  x = 2
Either x – 15 = 0, then x = 15
61 Arun Deep's Understanding Math-10
16  8 x  3 x  9 x x  1 34
 =2 P.Q. (i)  
( x  3) ( 2  x ) x 1 x 15
11x  7 x x  1 34
 2 x  x 2  6  3x = 2 Sol. Given eqn. be,  
x 1 x 15
 – 11x + 7 = 4x – 2x2 + 12 – 6x
x 2  x 2  2 x  1 34
 – 11x + 7 – 4x + 2x2 – 12 + 6x = 0  
x ( x  1) 15
 2x2 – 9x – 5 = 0
 2x2 – 10x + x – 5 = 0 2 x 2  2 x  1 34
 
 2x (x – 5) + 1 (x – 5) = 0 x2  x 15
 (x – 5) (2x + 1) = 0  30x2 + 30x + 15 = 34x2 + 34x
Either x – 5 = 0, then x = 5  30x2 + 30x + 15 – 34x2 – 34x = 0
1  – 4x2 – 4x + 15 = 0
or 2x + 1 = 0, then 2x = – 1  x  
2  4x2 + 4x – 15 = 0
1  4x2 + 10x – 6x – 15 = 0
Hence x = 5,  Ans.
2 [ 10 + (–6) = 4 and 10 × (–6) = –60 = 4 × (–15)]
x x 1 1  2x (2x + 5) – 3 (2x + 5) = 0
(ii)  2
x 1 x 2  (2x + 5) (2x – 3) = 0
x x 1 1 5
Sol. Given eqn. be,  2 Either 2x + 5 = 0, then 2x = – 5  x 
x 1 x 2 2
x x 1 5 3
   or 2x – 3 = 0, then 2x = 3  x 
x 1 x 2 2
x2  x2  2x  1 5 5 3
  Hence x  , Ans.
x ( x  1) 2 2 2

2x2  2x  1 5 x 1 x  2
  15. (i)  =3
x2  x 2 x1 x2
 4x2 – 4x + 2 = 5x2 – 5x
x 1 x  2
 4x2 – 4x + 2 – 5x2 + 5x = 0 Sol. Given, eqn. be,  =3
x1 x2
 – x2 + x + 2 = 0  x2 – x – 2 = 0
 x2 – 2x + x – 2 = 0 ( x  1) ( x  2 )  ( x  2 ) ( x  1)
 =3
( x  1) ( x  2 )
 x (x – 2) + 1 (x – 2) = 0
 (x – 2) (x + 1) = 0
x 2  2x  x  2  x 2  x  2x  2
Either x – 2 = 0, then x = 2 
x 2  2x  x  2
or x + 1 = 0, then x = – 1
x 2  3x  2  x 2  3x  2 3
Hence x = 2, – 1 Ans.  
x2  x  2 1
62 Arun Deep's Understanding Math-10
 2x2
+4= 3x2
+ 3x – 6  a(2a – 1) – 2(2a – 1) = 0
 2x2 + 4 – 3x2 – 3x + 6 = 0  (2a – 1) (a – 2) = 0
 – x2 – 3x + 10 = 0 1
 x2 + 3x – 10 = 0 Either 2a – 1 = 0, then a =
2
 x2 + 5x – 2x – 10 = 0 or a – 2 = 0, then a = 2
[ 5 + (–2) = 3 and 5 × (–2) = –10 = 1 × (–10)] 1 x3 1
 x (x + 5) – 2 (x + 5) = 0 (a) When a = , then =
2 x3 2
 (x + 5) (x – 2) = 0  2x – 6 = x + 3
Either x + 5 = 0, then x = – 5  2x – x = 3 + 6  x = 9
or x – 2 = 0, then x = 2 x3 2
(b) When a = 2, then =
Hence x = – 5, 2 Ans. x3 1
1 1 1  2x + 6 = x – 3  2x – x = –3 – 6
(i)  
x 3 x 5 6  x = –9
 x = 9, –9 Ans.
1 1 1
Sol. Given eqn. be,   a b
x 3 x 5 6 16. (i)  = a + b,
ax  1 bx  1
x 5 x 3 1 8 1 a + b  0, ab  0
  2 
( x  3) ( x  5) 6 x  2 x  15 6
1 1 1 1
 x2 + 2x – 15 = 48 (ii)
2a  b  2 x
=
2a
+
b
+
2x
 x2 + 2x – 15 – 48 = 0
a b
 x2 + 2x – 63 = 0  x2 + 9x – 7x – 63 = 0 Sol. (i) Given,  =a+b
ax  1 bx  1
 x (x + 9) – 7 (x + 9) = 0
 (x + 9) (x – 7) = 0 
F a  bI  F b  a I  0
Either x + 9 = 0, then x = – 9
H ax  1 K H bx  1 K
or x – 7 = 0, then x = 7 a  abx  b b  abx  a
  0
Hence x = – 9, 7 Ans. ( ax  1) (bx  1)

x3 x3 1
 (a + b – abx)
LM 1  1 OP  0
P.Q.
x3
+
x3
=2
2 N ax  1 bx  1 Q
Sol. Given eqn. be,
x3
+
x3
=2
1
 (a + b – abx) M
L bx  1  ax  1 OP  0.
x3 x3 2 N (ax  1) (bx  1) Q
x3 x3 1 ( a  b  abx) ( ax  bx  2 )
Put
x3
= a, then
x3
=  0
a ( ax  1) (bx  1)
1 5  (a + b – abx) (ax + bx – 2) = 0
 a+ =
a 2 Either a + b – abx = 0, then a + b = abx
 2a2
+ 2 = 5a a b
 2a2 – 5a + 2 = 0  2a2 – a – 4a + 2 = 0  x
ab
63 Arun Deep's Understanding Math-10
or ax + bx – 2 = 0, then x (a + b) = 2 2x2 – 8x – 4x + 16 = 3 (x2 – 4x – 60)
2 2x2 – 8x – 4x + 16 = 3x2 – 12x – 180
 x  a b 2x2 – 12x + 16 – 3x2 + 12x + 180 = 0
 – x2 + 196 = 0
a b 2
Hence x  , Ans.  x2 – 196 = 0
ab a  b
 (x)2 – (14)2 = 0
(ii) Given eqn. be,
 (x + 14) (x – 14) = 0
1 1 1 1
= + + Either x + 14 = 0, then x = – 14
2a  b  2 x 2a b 2x
or x – 14 = 0, then x = 14
1 1 1 1  x = 14, – 14 Ans.
 – = +
2a  b  2 x 2x 2a b
18.(i) 3x  4 = x
2 x  (2a  b  2 x) b  2a
 (2a  b  2 x)2 x = Sol. Given eqn. be,
2 ab
3 x  4 = x, Squaring both sides, we have
 ( 2a  b) ( 2a  b)
 ( 2a  b  2 x ) 2 x = 3x + 4 = x2  x2 – 3x – 4 = 0
2ab
 x2 – 4x + x – 4 = 0,
1 1
 ( 2a  b  2 x ) 2 x =  x (x – 4) + 1 (x – 4) = 0
2ab
 (x – 4) (x + 1) = 0
 –2ab = (2a + b + 2x)2x
Either x – 4 = 0, then x = 4
 4ax + 2xb + 4x2 = –2ab
 4x2 + 2bx + 4ax + 2ab = 0 or x + 1 = 0, then x = – 1
 2x(2x + b) + 2a (2x + b) = 0  x = 4, – 1
 (2x + 2a)(2x + b) = 0 Check (i) If x = 4, then
 2x + 2a = 0 or 2x + b = 0
L.H.S. = 3x  4  3× 4  4
b
x = –a or x =
2  12  4  16  4
Hence, the roots of the given equation are
R.H.S. = x = 4  L.H.S. = R.H.S.
b Hence x = 4 is its root
–a and .
2
(ii) If x = – 1, then
1 1 3
17.   . L.H.S.  3×( 1)  4  3  4  1  1
x  6 x  10 x  4
R.H.S. = x = – 1
1 1 3
Sol. Given eqn. be,   L.H.S.  R.H.S.
. ..
x  6 x  10 x  4
 x = – 1 is not its root
x  10  x  6 3
  Hence x = 4 Ans.
( x  6) ( x  10) x  4
(ii) x ( x  7)  3 2
2x  4 3
 
( x  6) ( x  10) x  4 Sol. Given, eqn. be, x ( x  7)  3 2
 (2x – 4) (x – 4) = 3 (x + 6) (x – 10) Squaring both sides,
64 Arun Deep's Understanding Math-10
 x (x – 7) = 9 × 2  – 7x = 18
x2 4
 x2 – 7x – 18 = 0, (ii) If y = , then
5
 x2 – 9x + 2x – 18 = 0 4 4 1
 x (x – 9) + 2 (x – 9) = 0  3x + 1 =  3x = –1=
5 5 5
 (x – 9) (x + 2) = 0 1 1 1
Either x – 9 = 0, then x = 9  x= × =
5 3 15
or x + 2 = 0, then x = – 2 1
Check : (i) If x = 9, then Hence x = –1, .
15
L.H.S.  20. Find the values of x if p + 1 = 0 and
x ( x  7)  9 ( 9  7)
x2 + px – 6 = 0
 9 × 2  18  9 × 2  3 2 = R.H.S. Sol. ... p + 1 = 0, then p = – 1
x = 9 the required root Substituting the value of p in the given
quadratic equation
(ii) If x = – 2, then
x2 + (– 1) x – 6 = 0
L.H.S.  x ( x  7)  2 ( 2  7 ) x2 – x – 6 = 0  x2 – 3x + 2x – 6 = 0
x (x – 3) + 2 (x – 3) = 0  (x – 3) (x + 2) = 0
 2 × 9  18  9 × 2  3 2 = R.H.S
Either x – 3 = 0, then x = 3
 x = – 2 is also its root or x + 2 = 0, then x = – 2
Hence x = 9, – 2 Ans. Hence x = 3, – 2 Ans.
19. Use the substitution y = 3x + 1 to solve for 21. Find the values of x if p + 7 = 0,
x : 5 (3x + 1)2 + 6(3x + 1) – 8 = 0 q – 12 = 0 and x2 + px + q = 0,
Sol. y = 3x + 1 Sol. ... p + 7 = 0, then p = – 7
Now, 5(3x + 1)2 + 6(3x + 1) – 8 = 0, we and q – 12 = 0, then q = 12
get Substituting the values of p and q in the
5y2 + 6y – 8 = 0  5y2 + 10y – 4y – 8 = 0 given quadratic equation, we get
x2 – 7x + 12 = 0  x2 – 3x – 4x + 12 = 0
5  ( 8)  40 
  x (x – 3) – 4 (x – 3) = 0  (x – 3) (x – 4) = 0
 40  10  (4) 
 6  10  4  Either x – 3 = 0, then x = 3
  or x – 4 = 0, then x = 4
 5y(y + 2) – 4(y + 2) = 0 Hence x = 3, 4 Ans.
 (y + 2) (5y – 4) = 0 22. If x = p is a solution of the equation x(2x +
Either y + 2 = 0, then y = –2 5) = 3, then find the value of p.
Sol. Given, x = p and x(2x + 5) = 3
4
or 5y – 4 = 0, then 5y = 4  y = Sbustituting the value of p, we get
5
p(2p + 5) = 3  2p2 + 5p – 3 = 0
(i) If y = –2, then
3x + 1 = –2  3x = –2 – 1  2p2 + 6p – p – 3 = 0 2  (3)  6 
 
3  6  6  (1) 
 3x = – 3  x = = –1  5  6 1 
3  
65 Arun Deep's Understanding Math-10
 2p(p + 3) – 1(p + 3) = 0 Here a = 2, b = – 7, c = 6
 (p + 3) (2p – 1) = 0  D = b2 – 4ac = (– 7)2 – 4 × 2 × 6
Either p + 3 = 0, then p = –3 = 49 – 48 = 1
1  b  b 2  4 ac b  D
or 2p – 1 = 0, then 2p = 1  p =  x 
2 2a 2a
1  ( 7 )  1 7  1
 p= , –3  
2 2×2 4
23. If x = 3 is a solution of the equation, 7 1 8
(k + 2)x2 – kx + 6 = 0, find the value of k.  x1   =2
4 4
Hence, find the other root of the equation.
7 1 6 3
Sol. (k + 2)x2 – kx + 6 = 0 ...(1) x2   
4 4 2
Substitute x = 3 in equation (1)  x = 2, 3/2 Ans.
(k + 2) (3)2 – k(3) + 6 = 0
(ii) 2x2 – 6x + 3 = 0
 9(k + 2) – 3k + 6 = 0
Sol. On comparing with ax2 + bx + c = 0
 9k + 18 – 3k + 6 = 0
Here a = 2, b = – 6, c = 3
 6k + 24 = 0
then D = b2 – 4ac = (– 6)2 – 4 × 2 × 3
 6k = –24
= 36 – 24 = 12
24
 k= =4  ( 6)  12
6 b  D 6 2 3
Now x   
2a 2×2 4
 k = –4
Now, substituting = –4 in equation (1), we
get  x1 
6 2 3
=
2 3 3 

3 3 
4 4 2
(–4 + 2)x2 – (–4)x + 6 = 0
 –2x2 + 4x + 6 = 0
x2 
62 3
=
2 3 3 

3 3 
2
 x – 2x – 3 = 0 (Dividing by 2) 4 4 2
2
 x – 3x + x – 3 = 0
3 3 3 3
 x(x – 3) + 1(x – 3) = 0 Hence x  , Ans.
2 2
 (x + 1) (x – 3) = 0
2.(i) 256 x2 – 32x + 1 = 0
So, the roots are x = –1 and x = 3
Sol. Here a = 256, b = – 32, c = 1
Thus, the other root of the equation is x = –1
D = b2 – 4ac = (– 32)2 – 4 × 256 × 1
EXERCISE 5.3 = 1024 – 1024 = 0
1. Solve the following (1 to 8) equations b  D  (32)  0 32 1
by using formula :
. ..
x =  
2a 2  256 512 16
1.(i) 2x2 – 7x + 6 = 0 1 1
Sol. On comparing with ax2 + bx + c = 0 x1  , x2 
16 16
66 Arun Deep's Understanding Math-10
1 1 Sol. On comparing with ax2 + bx + c = 0
Hence x  , Ans.
16 16 Here a  3 , b = 10, c  8 3
(ii) 25x2
+ 30x + 7 = 0
D = b2 – 4ac = (10)2 – 4 × 3 × ( 8 3 )
Sol. On comparing with ax2 + bx + c = 0
Here a = 25, b = 30, c = 7 = 100 + 96 = 196
Here, D = b2 – 4ac = (30)2 – 4 × 25 × 7  b  D  10  196
. ..
x
 D = 900 – 700 = 200 2a 2× 3
b  D 30  200 10  14
...
x  
2a 2 × 25 2 3
30  100 × 2 10  14 4 2× 3 2 3
 x  x1    
50 2 3 2 3 3× 3 3
30  10 2 3  2 10  14 24 12 × 3
  x2   
50 5
2 3 2 3 3× 3
3  2 3  2
 x1  and x 2 
5 5 12 3
 x2  =–4 3
3  2 3  2 3
Hence x  ,
5 5
2 3
3.(i) 2x2 + Hence x  , 4 3 Ans.
5 x–5=0 3
Sol. On comparing with ax2 + bx + c = 0
x  2 x2
4.(i) Given eqn. be,  =4
Here a = 2, b = 5, c = – 5 x2 x  2

Here, D = b2 – 4ac  e 5 j2 – 4 × 2 × (– 5) 
( x  2) 2  ( x  2) 2
4
= 5 + 40 = 45 ( x  2) ( x  2)

b  D  5  45 x2  4x  4  x2  4x  4
. .. x   =4
2a 2×2 x2  4
 2x2 + 8 = 4x2 – 16
 5  9 ×5  5 3 5
   2x2 + 8 – 4x2 + 16 = 0
4 4
 – 2x2 + 24 = 0
 53 5 2 5 5
 x1     x2 – 12 = 0
4 4 2
On comparing with ax2 + bx + c = 0
 53 5 4 5 Here a = 1, b = 0, c = – 12
and x2  
4 4  5 D = b2 – 4ac = (0)2 – 4 × 1 (– 12)
5 = 0 + 48 = 48
Hence x  ,  5 Ans.
2
b  D 0  48  48
(ii) 3 x 2  10 x  8 3  0
... x   
2a 2 ×1 2
67 Arun Deep's Understanding Math-10

 16 × 3 4 3 a2 1  a2 1 2 1
 = =2 3 x2   
2 2 2a 2a a
1
Hence roots are 2 3 , 2 3 Ans. Hence x = a, Ans.
a
x  1 3x  2 (ii) 4x2 – 4ax + (a2 – b2) = 0
(ii)  Sol. On comparing with ax2 + bx + c = 0
x  3 2x  3
Here a = 4, b = – 4a, c = a2 – b2
Sol. Given eqn. be, D = b2 – 4ac = (– 4a)2 – 4 × 4 (a2 – b2)
(x + 1) (2x + 3) = (3x + 2) (x + 3) = 16a2 – 16 (a2 – b2)
 2x2 + 3x + 2x + 3 = 3x2 + 9x + 2x + 6 = 16a2 – 16a2 + 16b2
D = 16 b2
 2x2 + 5x + 3 – 3x2 – 11x – 6 = 0 b  D  ( 4 a )  16b 2
. .. x
 
 – x2 – 6x – 3 = 0  x2 + 6x + 3 = 0 2a 2×4
On comparing with ax2 + bx + c = 0 4 a  4b a b
 
Here a = 1, b = 6, c = 3 8 2

 D = b2 – 4ac = (6)2 – 4 × 1 × 3 a b a b
 x1  , x2 
2 2
= 36 – 12 = 24
a b a b
Hence x  , Ans.
 b  D  6  24 2 2
... x 
2a 2 ×1 1
6. (i) x – = 3, x  0
6  4×6 x
6  2 6
  =–3 6
2 2 1 1
(ii) + = 3, x  0, 2
x x2
 x1 = –3 + 6 , x2 = –3 – 6
1
Hence x = – 3 + 6 , – 3 – 6 Ans. Sol. (i) Given, eqn. be, x – =3
x
5.(i) a (x2 + 1) = (a2 + 1) x , a  0
 x2 – 1 = 3x  x2 – 3x – 1 = 0
Sol. Given eqn. be, ax2 + a = (a2 + 1) x On comparing with ax2 + bx + c = 0
 ax2 – (a2 + 1) x + a = 0 Here a = 1, b = –3, c = –1
On comparing with Ax2 + Bx + C = 0  b2 – 4ac = (–3)2 – 4 × 1 × (–1)
Here A = a, B = – (a2 + 1), C = a = 9 + 4 = 13
D = B2 – 4AC = [– (a2 + 1)]2 – 4 × a × a
= a4 + 2a2 + 1 – 4a2 = a4 – 2a2 + 1  b  b 2  4ac
x=
= (a2 – 1)2 2a

b  D  ( 3)  13 3  13
. .. x  = =
2a 2 1 2
( a 2  1)  ( a 2  1) 2 ( a 2  1)  ( a 2  1) 3  13 3  13
   x= and
2×a 2a 2 2

a 2 1  a 2 1 2a 2 1 1
 x1 =  =a (ii) Given eqn. be, + =3
2a 2a x x2
68 Arun Deep's Understanding Math-10

x2 x 2x  2 9 3
x ( x  2) = 3  =3 = (Dividing by 2)
x2  2x 3
 3x2 – 6x = 2x – 2  3x2 – 6x – 2x + 2 = 0 9 3 9 3
 3x2 – 8x + 2 = 0  x ,
3 3
On comparing with ax2 + bx + c = 0
Here a = 3, b = –8, c = 2 3 3
 3 , 3
 b2 – 4ac = (–8)2 – 4 × 3 × 2 3 3
= 64 – 24 = 40 1 1
 3 , 3 Ans.
3 3
 b  b 2  4ac
Thus, x =
2a  2x  1   x3 
7. Solve for x : 2   – 3  = 5,
 x3   2x  1 
 (8)  40 8  2 10 4  10
= = = 1
23 6 3
x  –3,
2
4  10 4  10
 x= and  2x  1   x3 
3 3 Sol. Given eqn. be, 2   – 3  =5
 x  3   2x  1 
1 1 1
PQ.    0.
x2 x3 x4 2x  1 x3 1
Let = y, then = y
1 1 1 x3 2x  1
Sol. Given eqn. be,   0
x2 x3 x4 Thus given eqn. becomes ;
1 1 1 3
  
x2 x3 x4 2y – y = 5
x  3 x  2 1
   2y2 – 3 = 5y  2y2 – 5y – 3 = 0
( x  2 )( x  3) x4
On comparing with ax2 + bx + c = 0
2x  5 1 Here, a = 2, b = –5, c = –3
 
x 2  5x  6 x  4  b2 – 4ac = (–5)2 – 4 × 2 × (–3)
 (2x – 5) (x – 4) = –1 (x2 – 5x + 6) = 25 + 24 = 49
 2x2 – 8x – 5x + 20 = –x2 + 5x – 6
 2x2 – 8x – 5x + 20 + x2 – 5x + 6 = 0  b  b 2  4ac
Now, y =
 3x2 – 18x + 26 = 0 2a
On comparing with ax2 + bx + c = 0
Here, a = 3, b = – 18, c = 26  (5)  49 57
 y= =
2 2 4
b  b 2  4 ac
 x= 57 12
2a y= = =3
4 4
 ( 18)  ( 18) 2  4  3  26
 57 2 1
23 or y = = =
4 4 2
18  324  312 18  12 18  2 3
   1
6 6 6  y = 3,
2
69 Arun Deep's Understanding Math-10
2x  1
When y = 3, then =3  b  b 2  4ac
x3  x=
 3x + 9 = 2x – 1 2a
 3x – 2x = –1 – 9  x = –10
1  7  ( 7 ) 2  ( 4  1  7 )
When y = , then =
2 2 1
2x  1 1
=  7  49  28  7  77
x3 2 x= 
 4x – 2 = –x – 3 2 2
 4x + x = –3 + 2  5x = –1
 7  8.774
1 =
 x= 2
5
1  7  8.774
 x = –10, x=
5 2
8. Solve the following equation by using
quadratic equations for x and give your  7  8.774 1.774
x= =
answer correct up to two decimal places: 2 2
(i) x2  5x  10 = 0 (2005)
Sol. x2  5x  10 = 0  15.774
x=
On comparing with, ax2 + bx + c = 0 2
a = 1, b =  5, c =  10 x = 0.887
 b  b 2  4ac x = –7.89
 x= x = 0.89
2a
Solution set = {–7.89, 0.89} Ans.
  5   52  41 10
x= 9. Solve the following equations by using
2 1 quadratic formula and give your answer
5  25  40 correct to 2 decimal places :
 x= (i) 4x2 – 5x – 3 = 0 (2017)
2
5  65 5  8.06 1
 x= = (ii) 2x  =7 (2006)
2 2 x
5  8.06 13.06 Sol. (i) Given equation is 4x2 – 5x – 3 = 0
Either x = = = 6.53 Comparing with ax2 + bx + c = 0, we have,
2 2
a = 4, b = –5, c = –3
5  8.06  3.06
or x = = = 1.53
2 2  b  b 2  4ac
 x=
 x = 6.53, x =  1.53 Ans. 2a
(ii) x2 + 7x = 7
Sol. x2 + 7x – 7 = 0  (5)  ( 5) 2  4  4  (3)
=
 a = 1, b = 7, c = –7 24
70 Arun Deep's Understanding Math-10
a = 1, b = –4, c = –8
5  25  48 5  73 5  8.544
= = =
8 8 8 4  (  4) 2  4 1 8 4  16  32
x 
5  8.544 5  8.544 2 1 2
= or
8 8 4  48 4  6.928
 
13.544 3.544 2 2
= or 4  6.928 4  6.928
8 8
 x or
= 1.693 or –0.443 2 2
= 1.69 or –0.44 10.928 2.928
x or 
(correct to 2 decimal places) 2 2
(ii) Given eqn. be, x = 5.464 or – 1.464
1 x = 5.46 or – 1.46
2x  = 7  2x2  1 = 7x 18
x (ii) x – = 6. (2011)
 2x2  7x  1 = 0 ....(i) x
2
Comparing (i) with ax + bx + c, we get, 18
a = 2, b =  7, c =  1 Sol. Given eqn. be, x –
x
=6

 b  b 2  4ac  x2 – 6x – 18 = 0
 x=
2a On comparing with ax2 + bx + c = 0
a = 1, b = –6, c = –18
  7    7 2  42   1
 x=
22  b  b 2  4ac 6  36  72
x= =
7  49  8 7  57 2a 2
 =
61  1.73
4 4
6  108 66 3
x = = or
7  57 7  57 2 2 2
 x= or x =
4 4 = 3 × 2.73 or 3 × –0.73
7  7.55 7  7.55 = 8.19 or –2.19
 x= or x=
4 4
11. Solve the equation 5x2 – 3x – 4 = 0 and
14.55  0.55 give your answer correct to 3 significant
 x= or x= figures:
4 4
 x = 3.64 or x =  0.14 Ans. Sol. Given eqn. by, 5x2 – 3x – 4 = 0
10. Solve the following quadratic equations and On comparing with ax2 + bx + c = 0
give your answer correct to two significant Here, a = 5, b = –3, c = –4
figures :
(i) x2 – 4x – 8 = 0.  b  b 2  4ac
 x=
Sol. Given equation is x2 – 4x – 8 = 0 2a
Comparing with ax2 + bx + c = 0
3  9  4  5  4 3  89
b  b 2  4ac = 
Now x  25 10
2a
71 Arun Deep's Understanding Math-10
3  9.43 3  9.43  Root are not real. Ans.
x= or x = 2. Discuss the nature of the roots of the
10 10
following equations :
12.43 6.43 1
 x= or x = (i) 3x2 – 4 3 x + 4 = 0 (ii) x2 – x+4=0
10 10 2
 x = 1.24 or x = – 0.643 (iii) –2x2 + x +1 = 0 (iv) 2 3 x2 – 5x + 3 = 0
1
EXERCISE 5.4 (v) x2 – 4x – 1 = 0 (vi) 3x3 – 2x + =0
3
1. Find the discriminant of the following Sol.
equations and hence find the nature of roots: (i) Given eqn. be,
(i) 3x2 – 5x – 2 = 0 (ii) 2x2 – 3x + 5 = 0 3x2 – 4 3 x + 4 = 0
(iii) 16x2 – 40x + 25 = 0 (iv) 2x2 + 15x + 30 = 0 On comparing with ax2 + bx + c = 0
Sol. (i) Given eqn. be, Here a = 3, b = –4 3 , c = 4
3x2 – 5x – 2 = 0  D = b2 – 4ac
On comparing with ax2 + bx + c = 0
Here a = 3, b = –5, c = –2

= 4 3 2
– 4 × 3 × 4 = 48 – 48 = 0
 D=0
 D = b2 – 4ac  Roots are real and equal
= (–5)2 – 4 × 3 × (–2) = 25 + 24 = 49
1
 Discriminant = 49 (ii) Given eqn. be, x2 – x+4=0
 D>0
2
 Roots are real and distinct. On comparing with ax2 + bx + c = 0
(ii) Given eqn. be, 2x2 – 3x + 5 = 0 1
On comparing with ax2 + bx + c = 0 Here a = 1, b = – ,c=4
2
Here a = 2, b = –3, c = 5  D = b2 – 4ac
 D = b2 – 4ac
= (–3)2 – 4 × 2 × 5 = 9 – 40 = –31  1 1 63
=   – 4 × 1 × 4 = – 16 = –
 Discriminant = –31 < 0  2 4 4
 D < 0,  D<0
 Roots are not real.  Roots are not real.
(iii) 16x2 – 40x + 25 = 0 (iii) Given eqn. be, –2x2 + x +1 = 0
a = 16, b = –40, c = 25 On comparing with ax2 + bx + c = 0
 D = b2 – 4ac = (–40)2 – 4 × 16 × 25 Here, a = –2, b = 1, c = 1
= 1600 – 1600 = 0 D = b2 – 4ac = (1)2 – 4 × (–2) × 1
 Discriminant = 0 =1+8=9
 D=0  D>0
 Roots are real and equal.  Roots are real and distinct.
(iv) 2x2 + 15x + 30 = 0 (iv) Given eqn. be, 2 3 x2 – 5x + 3 = 0
Here a = 2, b = 15, c = 30 On comparing with ax2 + bx + c = 0
 D = b2 – 4ac
Here a = 2 3 , b = –5, c = 3
= (15)2 – 4 × 2 × 30 = 225 – 240 = –15
 Discriminant = –15  D = b2 – 4ac = (–5)2 – 4 × 2 3 × 3
 D<0 = 25 – 24 = 1
72 Arun Deep's Understanding Math-10
 D>0 4. Without solving the following quadratic
 Roots are real and distinct. equation, find the value of ‘p’ for which the
(v) Given eqn. be, x2 – 4x – 1 = 0 roots are equal :
On comparing with ax2 + bx + c = 0 (i) px2 – 4x + 3 = 0 (ii)x2 + (p – 3) x + p = 0.
Here a = 1, b = –4, c = –1
Sol.(i) Given eqn. be, px2 – 4x + 3 = 0
 D = b2 – 4ac = (–4)2 – 4 × 1 × (–1)
On comparing with ax2 + bx + c = 0
= 16 + 4 = 20
Here a = p, b = –4, c = 3
 D>0
Thus, Roots are real and distinct.  D = b2 – 4ac = (–4)2 – 4 × p × 3 = 16 – 12p
The roots are equal.
1
(vi) Given eqn. be, 3x2 – 2x + =0  D=0
3
On comparing with ax2 + bx + c = 0  b2 – 4ac = 0 16 – 12p = 0  12p = 16
1 16 4 4
Here a = 3, b = –2, c =  p= =  p=
3 12 3 3
1 2
(ii) Given, eqn. be, x + (p – 3) x + p = 0
 D = b2 – 4ac = (–2)2 – 4 × 3 × =4–4=0
3 On comparing with ax2 + bx + c = 0
D=0
 Roots are equal and equal. Here a = 1, b = (p – 3) , c = p
3. Find the nature of the roots of the following
quadratic equations : Equation has real and equal roots.
 b2 – 4ac = 0
1 1
(i) x2 – x– = 0 (ii) x2 – 2 3 x – 1 = 0  (p – 3)2 – 4(1) (p) = 0 (p – 3)2 – 4p = 0
2 2
If real roots exist, find them.  p2 + 9 – 6p – 4p = 0  p2 – 10p + 9 = 0
 p2 – 9p – p + 9 = 0
1 1
Sol. (i) Given eqn. be, x2 – x– =0  p (p – 9) – 1 (p – 9)= 0
2 2
 (p – 1) (p – 9) = 0 p = 1, 9 Ans.
1 1
Here a = 1, b = – ,c=– 5. Find the value (s) of k for which each of the
2 2
following quadratic equation has equal roots:
 D = b2 – 4ac
(i) x2 + 4kx + (k2 – k + 2) = 0
2
 1  1 1 9 (ii) (k – 4) x2 + 2(k – 4) x + 4 = 0
=   –4×1×   = +2=
 2   
2 4 4 Sol. (i) Given eqn. be, x2 + 4kx + (k2 – k + 2) = 0
9 On comparing with ax2 + bx + c = 0
D= >0 a = 1, b = 4k, c = (k2 – k + 2)
4
 Roots are real and unequal  D = b2 – 4ac
(ii) Given eqn. be, x2 – 2 3 x – 1 = 0 For equal roots,
On comparing with ax2 + bx + c = 0 b2 – 4ac = 0
Here a = 1, b = –2 3 , c = –1  (4k)2 – [4 × 1 × (k2 – k + 2)] = 0
 D = b2 – 4ac 16k2 – (4k2 – 4k + 8) = 0
 
2
=  2 3 – 4 × 1 × (–1) = 12 + 4 = 16  16k2 – 4k2 + 4k – 8 = 0
D>0  12k2 + 4k – 8 = 0
 Roots are real and unequal. 3k2 + k – 2 = 0 (dividing by 4)
73 Arun Deep's Understanding Math-10
 3k2
+ 3k – 2k – 2 = 0  Roots are equal.
 3k (k + 1) – 2(k + 1) = 0  D=0
 (3k – 2) (k + 1) = 0  –8m2 + 4m + 4 = 0
 2m2 – m – 1 = 0 (Dividing by 4)
2  2
2m – 2m + m – 1 = 0
 k= or k = – 1 Ans.
3  2m (m – 1) + 1 (m – 1) = 0
(ii) Given eqn. be,  (m – 1) (2m + 1) = 0
(k – 4) x2 + 2(k – 4) x + 4 = 0 Either m – 1 = 0, then m = 1
On comparing with ax2 + bx + c = 0 or 2m + 1 = 0, then 2m = –1
Here a = k – 4, b = 2 (k – 4), c = 4
D = b2 – 4ac 1
 m=–
= [2(k – 4)]2 – 4 × (k – 4) × 4 2
= 4 (k2 + 16 – 8k) – 16 (k – 4) 1
= 4 (k2 – 8k + 16) – 16 (k – 4)  m = 1, –
2
= 4 [k2 – 8k + 16 – 4k + 16] (ii) Given eqn. be,
= 4 (k2 – 12k + 32) x2 + 2(m – 1) x + (m + 5) = 0
 Roots are equal. On comparing with ax2 + bx + c = 0
 D=0 Here a = 1, b = 2 (m – 1), c = (m + 5)
 4 (k2 – 12k + 32) = 0 So, discriminant, D = b2 – 4ac
 k2 – 12k + 32 = 0 = [2(m – 1)]2 – 4 × 1 × (m + 5)
 k2 – 8k – 4k + 32 = 0 = 4(m2 – 2m + 1) – 4(m + 5)
 k (k – 8) – 4 (k – 8) = 0 = 4m2 – 8m + 4 – 4m – 20
 (k – 8) (k – 4) = 0 = 4m2 – 12m – 16
Either k – 8 = 0, then k = 8 For real and equal roots D = 0
or k – 4 = 0, then k = 4 So, 4m2 – 12m – 16 = 0
But k – 4  0 k4  m2 – 3m – 4 = 0 (Dividing by 4)
Hence, k = 8 Ans.  2
m – 4m + m – 4 = 0
6. Find the value(s) of m for which each of the  m(m – 4) + 1(m – 4) = 0
following quadratic equation has real and
equal roots:  (m – 4) (m + 1) = 0
(i) (3m + 1)x2 + 2(m + 1)x + m = 0  m = 4 or m = –1
(ii) x2 + 2(m – 1) x + (m + 5) = 0 7. Find the values of k for which each of the
following quadratic equation has equal roots:
Sol. (i) Given eqn. be,
(i) 9x2 + kx + 1 = 0
(3m + 1)x2 + 2(m + 1)x + m = 0
(ii) x2 – 2kx + 7k – 12 = 0
On comparing with ax2 + bx + c = 0
Also, find the roots for those values of k in
Here a = 3m + 1, b = 2(m + 1), c = m each case.
 D = b2 – 4ac Sol. (i) Given eqn. be,
= [2(m + 1)]2 – 4 × (3m + 1) (m) 9x2 + kx + 1 = 0
= 4 (m2 + 2m + 1) – 12m2 – 4m On comparing with ax2 + bx + c = 0
= 4m2 + 8m + 4 – 12m2 – 4m Here a = 9, b = k, c = 1
= –8m2 + 4m + 4  D = b2 – 4ac
74 Arun Deep's Understanding Math-10
= k2 – 4 × 9 × 1 = k2 – 36
b D   2  4  0 8
Roots are equal. x= = = =4
 D=0 2a 2 1 2
 x = 4, 4
 k2 – 36 = 0  (k + 6) (k – 6) = 0
8. Find the value(s) of p for which the quadratic
Either k + 6 = 0, then k = –6 equation
k – 6 = 0, then k = 6 (2p + 1)x2 – (7p + 2)x + (7p – 3) = 0 has
 k = 6, –6 equal roots. Also find these roots.
(a) If k = 6, then given eqn. be, Sol. The quadratic equation given is
9x2 + 6x + 1 = 0 (2p + 1)x2 – (7p + 2)x + (7p – 3) = 0
 (3x)2 + 2 × 3x × 1 + (1)2 = 0 Comparing with ax2 + bx + c = 0, we have
 (3x + 1)2 = 0 a = 2p + 1, b = –(7p + 2), c = (7p – 3)
Since given eqn. have equal roots.
 3x + 1 = 0  3x = –1
 D = b2 – 4ac  0 = [–(7p + 2)]2 – 4(2p + 1)
1 1 (7p – 3)
x = – 3, – 3  0 = 49p2 + 4 + 28p – 4(14p2 – 6p + 7p – 3)
(b) If k = –6, then given eqn. be,  0 = 49p2 + 4 + 28p – 56p2 – 4p + 12
9x2 – 6x + 1 = 0  0 = –7p2 + 24p + 16
 (3x)2 – 2 × 3x × 1 + (1)2 = 0  0 = –7p2 + 28p – 4p + 16
 (3x – 1)2 = 0  3x – 1 = 0  0 = –7p(p – 4) – 4(p – 4)
 0 = (–7p – 4) (p – 4)
1  –7p – 4 = 0 or p – 4 =0
 3x = 1  x =
3 4
1 1 Hence, the vaue of p = or p = 4
7
x= ,
3 3 PQ. If –5 is a root of the quadratic equation
(ii) Given eqn. be, x2 – 2kx + 7k – 12 = 0 2x2 + px – 15 = 0 and the quadratic equation
On comparing with ax2 + bx + c = 0 p(x2 + x) + k = 0 has equal roots, find the
value of k.
Here a = 1, b = –2k, c = 7k – 12 Sol. –5 is a root of the quadratic equation
 D = b2 – 4ac 2x2 + px – 15 = 0, then
= (–2k)2 – 4 × 1 × (7k – 12)  2(–5)2 + p(–5) – 15 = 0
= 4k2 – 4 (7k – 12) = 4k2 – 28k + 48  50 – 5p – 15 = 0  35 – 5p = 0
Roots are equal. 35
 D=0  5p = 35  p = =7
5
 4k2 – 28k + 48 = 0k2 – 7k + 12 = 0 Since given eqn.
 k2 – 3k – 4k + 12 = 0 p(x2 + x) + k = 0 has equal roots.
 k (k – 3) – 4 (k – 3) = 0  px2 + px + k = 0
 7x2 + 7x + k = 0
 (k – 3) (k – 4) = 0 Here, a = 7, b = 7, c = k
Either k – 3 = 0, then k = 3  D = b2 – 4ac = (7)2 – 4 × 7 × k = 49 – 28k
or k – 4 = 0, then k = 4 Roots are equal.
(a) If k = 3, then  b2 – 4ac = 0
 49 – 28k = 0  28k = 49
b D 4k  0 4  3 12 49 7 7
x= = = = =6  k= =  k=
2a 2 1 2 2 28 4 4
x = 6, 6 9. Find the value(s) of p for which the equation
(b) If k = 4, then 2x2 + 3x + p = 0 has real roots.
75 Arun Deep's Understanding Math-10
Sol. Given eqn. be,  (x + 6) (x – 5) = 0
2x2 + 3x + p = 0 Either x + 6 = 0, then x = – 6
On comparing with ax2 + bx + c = 0
or x – 5 = 0, then x = 5
Here, a = 2, b = 3, c = p . . . The numbers are positive.
b2 – 4ac = (3)2 – 4 × 2 × p = 9 – 8p
Roots are real.  x = – 6 is not possible.
 b2 – 4ac > 0  9 – 8p > 0 Hence the required first natural number = 5
9 and second natural number = 5 + 1 = 6 Ans.
9 > 8p  8p < 9  p <
8 (ii) Let the first integer = x
10. Find the least positive value of k for which Then second integer = x + 1
the equation x2 + kx + 4 = 0 has real roots.
Sol. Given eqn. be, According to the condition, (x)2 + (x + 1)3
x2 + kx + 4 = 0 = 61
On comparing with ax2 + bx + c = 0  x2 + x 2 + 2x + 1 = 61
Here, a = 1, b = k, c = 4  2x2 + 2x + 1 – 61 = 0
 b2 – 4ac = k2 – 4 × 1 × 4 = k2 – 16  2x2 + 2x – 60 = 0
Roots are real and positive.
 k2 – 16 > 0  k2 > 16  x 2 + x – 30 = 0 (Dividing by 2)
 k>4  x 2 + 6x – 5x – 30 = 0
Thus, the least the value of k be equal to 4  x(x + 6) – 5(x + 6) = 0
11. Find the values of p for which the equation  (x + 6) (x – 5) = 0
3x2 – px + 5 = 0 has real roots.
Sol. Given eqn. be, Either x + 6 = 0, then x = –6 or x – 5 = 0,
3x2 – px + 5 = 0 then x = 5
On comparing with ax2 + bx + c = 0 (i) If x = –6, then
Here, a = 3, b = –p, c = 5 First integer = –6
 b2 – 4ac = (–p)2 – 4 × 3 × 5 = p2 – 60 and second = –6 + 1 = –5
Since roots are real
 b2 – 4ac > 0 (ii) If x = 5, then
 p2 – 60 > 0  p2 > 60 First integer = 5
 |p| > 60 = 2 15 and second = 5 + 1 = 6
 Required integers are (–6, –5), (5, 6).
 p < –2 15 or p > 2 15
2.(i) If the product of two positive consecutive
[ |x| > a  x > a or x < – a]
even integers is 288, find the integers.
EXERCISE 5.5 (ii) If the product of two consecutive even
1. (i) Find two consecutive natural numbers integers is 224, find the integers.
such that the sum of their squares is 61. (iii) Find two consecutive even natural num-
(ii) Find two consecutive integers such that bers such that the sum of their squares is
the sum of their squares is 61. 340.
Sol. (i) Let the first natural number = x (iv) Find two consecutive odd integers such
then second natural number = x + 1 that the sum of their squares is 394.
According to the condition, (x)2 + (x + 1)2 = 61 Sol.(i) Let first positive even integer = 2 x
 x2 + x2 + 2 x + 1 – 61 = 0 then second even integer = 2 x + 2
 2 x2 + 2 x – 60 = 0 According to the condition, we have
 x2 + x – 30 = 0 2 x × (2 x + 2) = 288
 x2 + 6 x – 5 x – 30 = 0  4 x + 4 x – 288 = 0
2

 x (x + 6) – 5 (x + 6) = 0  x2 + x – 72 = 0 (Dividing by 4)
76 Arun Deep's Understanding Math-10
 x2 + 9 x – 8 x – 72 = 0 Then second odd integer = 2x + 3
 x (x + 9) – 8 (x + 9) = 0 According to the condition, we have
 (x + 9) (x – 8) = 0 (2x + 1)2 + (2x + 3)2 = 394
Either x  9 = 0, then x = – 9  4x2 + 4x + 1 + 4x2 + 12x + 9 = 394
But it is not possible as it is not a natural  8x2 + 16x – 394 + 10 = 0
number.  8x2 + 16x – 394 = 0
or x – 8 = 0, then x = 8  x2 + 2x – 48 = 0 (Dividing by 8)
 First even integer = 2 x = 2 × 8 = 16  x2 + 8x – 6x – 48 = 0
and second even integer = 16 + 2 = 18 Ans.  x(x + 8) – 6(x + 8) = 0
(ii) Let first even integer = 2 x  (x + 8) (x – 6) = 0
then second even integer = 2 x + 2 Either x + 8 = 0, then x = –8
According to the condition, we have or x – 6 = 0, then x = 6
2 x × (2 x + 2) = 224 (i) If x = –8, then first odd integer = 2x + 1
 4 x2 + 4 x – 224 = 0
= 2 × (–8) + 1 = –16 + 1 = –15
 x2 + x – 56 = 0
and second integer = –15 + 2 = –13
 x2 + 8 x – 7 x – 56 = 0
(ii) If x = 6, then first odd integer = 2x + 1
 x (x + 8) – 7 (x + 8) = 0
= 2 × 6 + 1 = 13
 (x + 8) (x – 7) = 0
Either x  8 = 0, then x = – 8 and second integer = 13 + 2 = 15
 First even integer = 2 × (–8) = –16  Required integer are –15, –13, or 13, 15
and second even integer = –16 + 2 = –14 3. The sum of two numbers is 9 and the sum
or x – 7 = 0, then x = 7 of their squares is 41. Taking one number
as x, form an equation in x and solve it to
 First even integer = 2 x = 2 × 7 = 14
find the numbers.
and second even integer = 14 + 2 = 16 Ans.
(iii) Let first even natural number = 2x Sol. Given sum of two numbers = 9
Then second number = 2x + 2 Let first number = x
According to the condition, we have then second number = 9 – x
(2x) 2 + (2x + 2)2 = 340 Now according to the condition, we have
 4x2 + 4x + 8x + 4 = 340 (x)2 + (9 – x)2 = 41
 8x2 + 8x + 4 – 340 = 0  x2 + 81 – 18 x + x2 – 41 = 0
 8x2 + 8x – 336 = 0  2 x2 – 18 x + 40 = 0
 x2 + x – 42 = 0 (Dividing by 8)  x2 – 9 x + 20 = 0 (Dividing by 2)
 x2 + 7x – 6x – 42 = 0  x2 – 4 x – 5 x + 20 = 0
 x(x + 7) –6 (x + 7) = 0  x (x – 4) – 5 (x – 4) = 0
 (x + 7) (x – 6) = 0  (x – 4) (x – 5) = 0
Either x  7 = 0, then x = – 7 Either x – 4 = 0, then x = 4
But it is not a even natural number. or x – 5 = 0, then x = 5
or x – 6 = 0, then x = 6 (i) If x = 4, then first number = 4
 First even natural number = 2x = 2 × 6 =
and second number = 9 – 4 = 5
12
and second = 12 + 2 = 14 (ii) If x = 5, then first number = 5
 Required numbers are 12, 14 and second number = 9 – 5 = 4
(iv) Let first odd integer = 2x + 1 Hence numbers are 4 and 5 Ans.
77 Arun Deep's Understanding Math-10
4. Five times a certain whole number is equal  x2
– 20x – 3x + 60 = 0
to three less than twice the square of the  x(x – 20) – 3(x – 20) = 0
number. Find the number.
 (x – 3) (x – 20) = 0
Sol. Let number = x
either (x – 3) = 0 or (x – 20) = 0
Now according to the condition, we have
 x = 3 or x = 20
5 x = 2 x2 – 3
Since sum of two natural numbers is 8,
 2 x2 – 5 x – 3 = 0
 x  20.
 2 x2 – 6 x + x – 3 = 0
Thus x = 3
 2 x (x – 3) + 1 (x – 3) = 0
From equation (i), y = 8 – x = 8 – 3 = 5
 (x – 3) (2 x + 1) = 0
Thus the values of x and y are 3 and 5
Either x – 3 = 0, then x = 3 respectively.
1 6. The difference of the squares of two num-
or 2 x + 1 = 0, then 2 x = – 1  x  
2 bers is 45. The square of the smaller num-
But it is not possible as the given number ber is 4 times the larger number. Deter-
is whole number. mine the numbers.
 Required number = 3 Ans. Sol. Let the larger number = x
5. Sum of two natural numbers is 8 and the then smaller number = y
difference of their reciprocal is 2/15. Find Now according to the condition, we have
the numbers.
x2 – y2 = 45 ...(i)
Sol. Let x and y be two required numbers
s.t. x < y and y2 =4x ...(ii)
Given that, x + y = 8 ....(i) Substituting the value of y2 from (ii) in (i);
we get,
1 1 2 x2 – 4 x = 45  x2 – 4 x – 45 = 0
and x  y  15 ....(ii)
 x2 – 9 x + 5 x – 45 = 0
From equation (i), we have, y = 8 – x  x (x – 9) + 5 (x – 9) = 0
Substituting the value of y in equation (ii),  (x – 9) (x + 5) = 0
we have,
Either x – 9 = 0, their x = 9
1 1 2 or x + 5 = 0, then x = – 5
 
8 8  x 15 (i) When x = 9, the larger number = 9
8 x x 2 and smaller number
 
x (8  x ) 15
y  4x  4×9  36
8  2x 2 4 x 1  y=6
 x(8  x)  15  
x (8  x ) 15 (ii) When x = – 5, then larger number = – 5
 15(4 – x) = x(8 – x) y  4 x  4  5   20
 60 – 15x = 8x – x2
which is not possible.
 x2 – 15x – 8x + 60 = 0
Hence numbers are 6, 9.
 x2 – 23x + 60 = 0
7. There are three consecutive positive inte-
78 Arun Deep's Understanding Math-10
gers such that the sum of the square of the possible.
first and the product of other two is 154. or x – 4 = 0, then x = 4
What are the integers ?
 First even natural number = 2 x = 2 × 4 = 8
Sol. Let the first integer = x second number = 8 + 2 = 10
then second integer = x + 1 and the third number = 10 + 2 = 12 Ans.
and third integer = x + 2 (ii) Let the three numbers be x, x + 2, x + 4
Now according to the condition, we have According to statement, we have
x2 + (x + 1) (x + 2) = 154 (x)2 + (x + 2)2 + (x + 4)2 = 83
 x2 + x2 + 3 x + 2 – 154 = 0  x2 + x2 + 4x + 4 + x2 + 8x + 16 = 83
 2 x2 + 3 x – 152 = 0  3x2 + 12x + 20 = 83
 2 x2 + 19 x – 16 x – 152 = 0  3x2 + 12x + 20 – 83 = 0
 x (2 x + 19) – 8 (2 x + 19) = 0  3x2 + 12x – 63 = 0  x2 + 4x – 21 = 0
 (2 x + 19) (x – 8) = 0  x2 + 7x – 3x – 21 = 0
19  x (x + 7) – 3 (x + 7) = 0
Either 2 x + 19 = 0, then 2 x = – 19  x  
2  (x – 3) (x + 7) = 0
But it is not possible as it is not an positive integer. Either x – 3 = 0 then x = 3
or x – 8 = 0, then x = 8 or x + 7 = 0 then x = – 7
 Numbers are 8, (8 + 1) = 9 and (8 + 2) = 10 Ans. Numbers will be 3, 3 + 2, 3 + 4 = 3, 5, 7 Ans.
8. (i) Find three successive even natural num- or Numbers will be –7, –7 + 2, –7 + 4
bers, the sum of whose squares is 308. = –7, –5, –3 Ans.
(ii) Find three consecutive odd integers, the 9. In a certain positive fraction, the denomi-
sum of whose squares is 83. nator is greater than the numerator by 3. If
Sol. (i) Let first even number = 2 x 1 is subtracted from both the numerator
and denominator, the fraction is decreased
second even number = 2 x + 2
1
third even number = 2 x + 4 by . Find the fraction.
14
Now according to the condition, we have Sol. Let the numerator of a fraction = x
(2 x)2 + (2 x + 2)2 + (2 x + 4)2 = 308 then denominator = x + 3
 4 x2 + 4 x2 + 8 x + 4 + 4 x2 + 16 x + 16 = x
then required fraction 
308 x3
Now according to the condition, we have
 12 x2 + 24 x + 20 – 308 = 0
x 1 x 1
 12 x2 + 24 x – 288 = 0 new fraction =  
x  31 x  3 14
 x2 + 2 x – 24 = 0 (Dividing by 12) x  1 14 x  x  3
 x2  
+ 6 x – 4 x – 24 = 0 x2 14 ( x  3)
 x (x + 6) – 4 (x + 6) = 0
x 1 13 x  3
 (x + 6) (x – 4) = 0  
x  2 14 x  42
Either x + 6 = 0, then x = – 6
 (x – 1) (14 x + 42) = (13 x – 3) (x + 2)
But it is not a natural number, hence not
[after cross multiplying]
79 Arun Deep's Understanding Math-10
 14 x2 + 42 x – 14 x – 42  x2
4 – 132 x + 560 = 0
= 13 x2 + 26 x – 3 x – 6  x2 – 33 x + 140 = 0
 14 x + 28 x – 42 – 13 x2 – 23 x + 6 = 0
2  x2 – 28 x – 5 x + 140 = 0
 x2 + 5 x – 36 = 0  x (x – 28) – 5 (x – 28) = 0
 x2 + 9 x – 4 x – 36 = 0  (x – 28) (x – 5) = 0
 x (x + 9) – 4 (x + 9) = 0 Either x – 28 = 0, then x = 28, but it is not
 (x + 9) (x – 4) = 0 possible as sum of numerator and
denominator is 8.
Either x + 9 = 0, then x = – 9, but it is not
possible as the fraction is positive. or x – 5 = 0, then x = 5
or x – 4 = 0, then x = 4 8 x 85 3
 Required fraction   
x 5 5
x 4 4
 Required fraction    Ans.
x3 43 7
Ans. 11. A two digit number contains the bigger at
ten’s place. The product of the digits is 27
10. The sum of the numerator and denomina-
and the difference between two digits is 6.
tor of a certain positive fraction is 8. If 2
Find the number.
is added to both the numerator and denomi-
4 Sol. Let unit's digit = x
nator, the fraction is increased by . Find
35 then tens digit = x + 6
the fraction.
Since difference between two digit is 6.
Sol. Let the denominator of a positive fraction=x
 Number = x + 10 (x + 6) = x + 10 x + 60
then numerator = 8 – x
= 11x + 60
8 x
 Fraction  According to the given condition, we have
x
According to the condition, we have x (x + 6) = 27
8 x2 8 x 4  x2 + 6 x – 27 = 0
 
x2 x 35  x2 + 9 x – 3 x – 27 = 0
10  x 8 x 4
    x (x + 9) – 3 (x + 9) = 0
x2 x 35
 (x + 9) (x – 3) = 0
10  x 8 x 4
   Either x + 9 = 0, then x = – 9, but it is not
x2 x 35
possible as it is negative.
10  x 8  x 4
   or x – 3 = 0, then x = 3
x2 x 35
 Required number = 11 x + 60
10 x  x 2  8 x  x 2  16  2 x 4
  = 11 × 3 + 60
x ( x  2) 35
= 33 + 60 = 93 Ans.
4 x  16 4 12. A two digit positive number is such that the
 
x 2x
2 35 product of its digits is 6. If 9 is added to the
number, the digits interchange their places.
 4 x2 + 8 x = 140 x – 560 Find the number. (2014)
 4 x2 + 8 x – 140 x + 560 = 0 Ans. Let the required 2-digit number = xy = 10x + y
80 Arun Deep's Understanding Math-10
Reversed digits = yx = 10y + x  x2 – 15 x – 7 x + 105 = 0
According to question, we have  x (x – 15) – 7 (x – 15) = 0
xy = 6  (x – 15) (x – 7) = 0
Either x – 15 = 0, then x = 15
6
 y= ...(i) or x – 7 = 0, then x = 7
x
As length > breadth, x = 7 is not admissible.
and 10x + y + 9 = 10y + x
 Length = 15 cm
6 6 and breadth = 22 – 15 = 7 cm. Ans.
 10x + + 9 = 10 × +x
x x 14. A rectangular garden 10 m by 16 m is to be
6 surrounded by a concrete walk of uniform
(From (i) y = ) width. Given that the area of the walk is
x
120 square metres, assuming the width of
 10x2 + 6 + 9x = 60 + x2 the walk to be x, form an equation in x and
 10x2 – x2 + 9x + 6 – 60 = 0 solve it to find the value of x. (1992)
 9x2 + 9x – 54 = 0 Sol. Given length of garden = 16 m
 x +x–6=0
2 and width = 10 m
 x2 + 3x – 2x – 6 = 0 Let width of walk = x m
 x(x + 3) – 2(x + 3) = 0  Outer length = 16 + 2 x
 (x – 2) (x + 3) = 0 and outer width = 10 + 2 x
 x = 2 or –3 (rejecting –3) Now according to the condition,
(16 + 2 x) (10 + 2 x) – 16 × 10 = 120
putting the value of x in (i) ; we have
 160 + 32 x + 20 x + 4 x2 – 160 = 120
6
y= =3  4 x2 + 52 x – 120 = 0
2
 x2 + 13 x – 30 = 0(Dividing by 4)
 Required 2-digit = 10x + y = 10 × 2 + 3 = 23
 x2 + 15 x – 2 x – 30 = 0
13. A rectangle of area 105 cm2 has its length
equal to x cm. Write down its breadth in  x (x + 15) – 2 (x + 15) = 0
terms of x. Given that the perimeter is 44  (x + 15) (x – 2) = 0
cm, write down an equation in x and solve Either x + 15 = 0, then x = – 15
it to determine the dimensions of the rect-
angle. But it is not possible as width can't be negative

Sol. Given perimeter of rectangle = 44 cm or x – 2 = 0, then x = 2 Ans.


P.Q. Harish made a rectangular garden, with its
44
 length + breadth   22 cm length 5 metres more than its width. The
2 next year, he increased the length by 3
Let length = x metres and decreased the width by 2
then breadth = 22 – x metres. If the area of the second garden
According to the condition, we have was 119 sq m, was the second garden
x (22 – x) = 105  22 x – x2 = 105 larger or smaller ?
 x2 – 22 x + 105 = 0 Sol. In first case,
Let length of the garden = x m
81 Arun Deep's Understanding Math-10
then width = (x – 5) m 2 (x – 9) (x – 5) = x (x – 5) + 140
Area = l × b = x (x – 5) sq. m  2 (x2 – 14 x + 45) = x2 – 5 x + 140
In second case,  2 x2 – 28 x + 90 = x2 – 5 x + 140
Length = (x + 3) m  2 x2 – 28 x + 90 – x2 + 5 x – 140 = 0
and width = x – 5 – 2 = (x – 7) m  x2 – 23 x – 50 = 0
 Area of garden = (x + 3) (x – 7), we have  x2 – 25 x + 2 x – 50 = 0
According to the condition, we have  x (x – 25) + 2 (x – 25) = 0
(x + 3) (x – 7) = 119  (x – 25) (x + 2) = 0
 x – 7 x + 3 x – 21 = 119
2 Either x – 25 = 0, then x = 25
 x2 – 4 x – 21 – 119 = 0 or x + 2 = 0, then x = – 2, but it is not pos-
 x2 – 4 x – 140 = 0 sible as it is negative.
 x2 – 14 x + 10 x – 140 = 0  Length of the rectangle = 25 m
and width = 25 – 5 = 20 m Ans.
 x (x – 14) + 10 (x – 14) = 0
16. The perimeter of a rectangular plot is 180m
 (x – 14) (x + 10) = 0
and its area is 1800 m2. Take the length of
Either x – 14 = 0, then x = 14 the plot as x m. Use the perimeter 180 m to
or x + 10 = 0, then x = – 10, but it is not write the value of the breadth in terms of
possible as it is negative. x. Use the values of length, breadth and
 Length of first garden = 14 m the area to write an equation in x. Solve
the equation to calculate the length and
and width = 14 – 5 = 9 m breadth of the plot. (1993)
Area = l × b = 14 × 9 = 126 m2 Sol. Perimeter of a rectangular field = 180 m
Difference of areas of two rectangles and area = 1800 m2
= 126 – 119 = 7 sq. m. Let length = x m
 Area of second garden is smaller than the 180
But length + breadth   90 m
area of the first garden by 7 sq. m. 2
15. The length of a rectangle exceeds its breadth  breadth = (90 – x) m
by 5 m. If the breadth were doubled and According to the condition, we have
the length reduced by 9 m, the area of the x (90 – x) = 1800
rectangle would have increased by 140 m2.  90 x – x2 – 1800 = 0
Find its dimensions.
 x2 – 90 x + 1800 = 0
Sol. Let length of the rectangle = x m
 x2 – 60 x – 30 x + 1800 = 0
then width = (x – 5) m
 x (x – 60) – 30 (x – 60) = 0
Area = x (x – 5) sq. m
In second case,  (x – 60) (x – 30) = 0
Length of the second rectangle = x – 9 Either x – 60 = 0, then x = 60
and width = 2 (x – 5) m or x – 30 = 0, then x = 30
... Length is greater than its breadth
 Area = (x – 9) 2 (x – 5)
= 2 (x – 9) (x – 5) sq. m  Length = 60 m
According to the condition, we have and breadth = 90 – 60 = 30 m Ans.
82 Arun Deep's Understanding Math-10
17. The lengths of the parallel sides of a trape-  (x – 24) (x – 10) = 0
zium are (x + 9) cm and (2 x – 3) cm and Either x – 24 = 0, then x = 24
the distance between them is (x + 4) cm. or x – 10 = 0, then x = 10
If its area is 540 cm2, find x. ... Length is greater than breadth.
1  Length = 24 m
Sol. Area of a trapezium 
2
(sum of parallel sides) × height and breadth = (34 – 24) = 10 m
Given, lengths of parallel sides are (x + 9) and Area = l × b = 24 × 10 = 240 m2.
and (2 x – 3) 19. If the sum of two smaller sides of a right –
and height = (x + 4) angled triangle is 17cm and the perimeter
According to the condition, we have is 30cm, then find the area of the triangle.
Sol. Perimeter of the triangle = 30 cm.
1
( x  9  2 x  3)×( x  4 )  540 Let one of the two small sides = x
2
 (3 x + 6) (x + 4) = 540 × 2 then, other side = 17 – x
 3 x2 + 12 x + 6 x + 24 – 1080 = 0
 3 x2 + 18 x – 1056 = 0
 x2 + 6 x – 352 = 0 (Dividing by 3)
 2
x + 22 x – 16 x – 352 = 0
 x (x + 22) – 16 (x + 22) = 0
 (x + 22) (x – 16) = 0
Either x + 22 = 0, then x = – 22 Length of hypotenuse
But it is not possible as it is negative. = perimeter – sum of other two sides
or x – 16 = 0, then x = 16 Ans. = 30cm – 17cm = 13cm.
18. If the perimeter of a rectangular plot is 68m According to the problem, we have
and length of its diagonal is 26 m, find its x2 + (17 – x)2 = (13)2 (Pythagoras theorem)
area.  x + 289 + x – 34x = 169
2 2
Sol. Perimeter = 68 m and diagonal = 26 m  x2 – 34x + 289 – 169 = 0
68  2 x2 – 34x + 120 = 0
 Length + breadth   34 m
2  x2 – 17x + 60 = 0 (Dividing by 2)
Let length of rectangular plot = x m  x – 12x – 5x + 60 = 0
2
then breadth = (34 – x) m
 x (x – 12) – 5 (x – 12) = 0
According to the condition, we have
 (x – 12) (x – 5) = 0
l 2 + b2 = h2
Either x – 12 = 0, then x = 12
 (x)2 + (34 – x)2 = (26)2
 x + 1156 + x2 – 68 x = 676
2 or x – 5 = 0, then x = 5
 2 x2 – 68 x + 1156 – 676 = 0 (i) When x = 12, then first side = 12cm
 2 x2 – 68 x + 480 = 0 and second side = 17 – 12 = 5cm
 x2 – 34 x + 240 = 0 (Dividing by 2) (ii) When x = 5, then first side = 5
 x2 – 24 x – 10 x + 240 = 0 and second side = 17 – 5 = 12
 x (x – 24) – 10 (x – 24) = 0  Sides are 5cm. 12cm
83 Arun Deep's Understanding Math-10
Now, area of the triangle  x2= 2 x2 – 12 x + 20
1  2 x2 – 12 x + 20 – x2 = 0
= × base × altitude
2  x2 – 12 x + 20 = 0
5  12 60  x2 – 10 x – 2 x + 20 = 0
   30cm 2 Ans.
2 2  x (x – 10) – 2 (x – 10) = 0
20. The hypotenuse of a grassy land in the  (x – 10) (x – 2) =0
shape of a right triangle is 1 metre more
Either x – 10 = 0, then x = 10
than twice the shortest side. If the third
side is 7 metres more than the shortest side, or x – 2 = 0, then x = 2, but it is not pos-
find the sides of the grassy land. sible as the hypotenuse is the longest side.
Sol. Let the shortest side = x  Hypotenuse = 10 cm
Hypotenuse = 2 x + 1 Base = 10 – 2 = 8 cm
and third side = x + 7 and shortest side = 10 – 4 = 6 cm Ans.
According to the condition, we have 22. In a P.T. display, 480 students are arranged in
(2 x + 1)2 = x2 + (x + 7)2 rows and columns. If there are 4 more stu-
 4 x2 + 4 x + 1 = x2 + x2 + 14 x + 49 dents in each row than the number of rows,
find the number of students in each row.
 4 x2 + 4 x + 1 – 2 x2 – 14 x – 49 = 0
Sol. Total number of students = 480
 2 x2 – 10 x – 48 = 0
Let the number of students in each row = x
 x2 – 5 x – 24 = 0 (Dividing by 2)
480
 x2 – 8 x + 3 x – 24 = 0 then the number of rows =
x
 x (x – 8) + 3 (x – 8) = 0
480
 (x – 8) (x + 3) = 0 According to the condition, x  4
x
Either x – 8 = 0, then x = 8  x 2 = 480 + 4x  x2 – 4x – 480 = 0
or x + 3 = 0, then x = – 3, but it is not  x2 – 24x + 20x – 480 = 0
possible as it is negative.  x (x – 24) + 20 (x – 24) = 0
 Shortest side = 8 m  (x – 24) (x + 20) = 0
Third side = x + 7 = 8 + 7 = 15 m Either x – 24 = 0 or x + 20 = 0  x = 24
and hypotenuse = 2 x + 1 = 8 × 2 + 1 or x = –20
= 16 + 1 = 17 m Ans. which is not possible as it is negative
 Number of students in each row = 24 Ans.
21. Mohini wishes to fit three rods together in
the shape of a right triangle. If the hypot- 23. In an auditorium, the number of rows are
enuse is 2 cm longer than the base and 4 equal to the number of seats in each row.
cm longer than the shortest side, find the If the number of rows is doubled and num-
lengths of the rods. ber of seats in each row is reduced by 5,
then the total number of seats is increased
Sol. Let the length of hypotenuse = x cm
by 375. How many rows were there ?
then base = (x – 2) cm
Sol. Let the number of rows = x
and shortest side = x – 4
then no. of seats in each row = x
According to the condition, we have
and total number of seats = x × x = x2
(x)2 = (x – 2)2 + (x – 4)2
According to the condition, we have
 x2 = x2 – 4 x + 4 + x2 – 8 x + 16
84 Arun Deep's Understanding Math-10
x2
2 x × (x – 5) = + 375 Due to heavy rains
 2 x2 – 10 x = x2 + 375 Speed of the bus = (x – 10) km/hr
 2 x2 – 10 x – x2 – 375 = 0  Time taken by bus to cover the same distance
 x2 – 10 x – 375 = 0 240
=
 x2 – 25 x + 15 x – 375 = 0 x  10
 x (x – 25) + 15 (x – 25) = 0 According to the condition, we have
 (x – 25) (x + 15) = 0
240 240
Either x – 25 = 0, then x = 25  = –2
x x  10
or x + 15 = 0, then x = – 15, but it is not
possible as it is negative. 240 240
 – =2
 Required number of rows = 25. x  10 x
24. At an annual function of a school, each 240 x  240 x  2400
student gives gift to every other student.  x( x  10) =2
If the number of gifts is 1980, find the
number of students. 2400
Sol. Let the number of students = x = =2
x  10 x
2

then the number of gifts given = x – 1  2400 = 2x2 – 20x


 Total number of gifts = x (x – 1)  2x2 – 20x – 2400 = 0
According to the condition, we have  x2 – 10x – 1200 = 0
x (x – 1) = 1980  x2 – 40x + 30x – 1200 = 0
 x – x – 1980 = 0
2
 x(x – 40) + 30(x – 40) = 0
 x2 – 45 x + 44 x – 1980 = 0  (x – 40) (x + 30) = 0
 x (x – 45) + 44 (x – 45) = 0 Either x – 40 = 0, then x = 40
 (x – 45) (x + 44) = 0 or x + 30 = 0, then x = –30 which is not
Either x – 45 = 0, then x = 45 possible, speed can't be negative.
or x + 44 = 0, then x = – 44, but it is not  Required speed of bus = 40 km/hr
possible as it is negative. 26. The speed of an express train is x km/hr and
Hence number of students = 45 Ans. the speed of an ordinary train is 12 km/hr
25. A bus covers a distance of 240 km at a less than that of the express train. If the
uniform speed. Due to heavy rain its speed ordinary train takes one hour longer than the
gets reduced by 10 km/h and as such it takes express train to cover a distance of 240 km,
two hours longer to cover the total distance. find the speed of the express train.
Assuming the uniform speed to be 'x' km/h,
form an equation and solve it to evaluate x. Sol. Let the speed of express train = x km
(2016) Then speed of the ordinary train
Sol. Distance = 240 km = (x – 12) km
Let speed of a bus = x km/hr
Time taken to cover 240 km by the express
 Time taken by bus to cover a distance of
D 240 240
240 km = = hours train = hours
S x x
85 Arun Deep's Understanding Math-10
Time taken to cover 240 km by the ordinary In second case,
speed of car = (x + 12) km/h
240
train = hours New time taken by car to cover 400 km
x  12
400
= h
According to the condition, we have x  12
According to the condition, we have
240 240
– =1 400 400 40 2 5
x  12 x – = 1 1 =
x x  12 60 3 3
 1 1  x  12  x  5 400  12 5
  =1
 240   400     =
 ( x  12) x   x( x  12)  3 x  12 x
2
3
 400 × 12 × 3 = 5x2 + 60x
 x  x  12   14400 = 5x2 + 60x
 240  x( x  12)  = 1
   5x2 + 60x – 14400 = 0
 x2 + 12x – 2880 = 0
 12  (dividing both side by 5)
 240  2  =1  x + 60x – 48x – 2880 = 0
2
 x  12 x 
 x (x + 60) – 48 (x + 60) = 0
 2880 = x2 – 12x x2 – 12x – 2880 = 0  (x + 60) (x – 48) = 0
 x2 – 60x + 48x – 2880 = 0  x = 48 or x = –60
 x(x – 60) + 48(x – 60) = 0  x = 48 (Rejecting x = –60, being speed)
 (x – 60) (x + 48) = 0 Hence, required original speed of the car
= 48 km/h.
 x = 60 or x = –48
28. An aeroplane travelled a distance of 400
 x = 60 km at an average speed of x km/hr. On the
(Rejecting x = –48, as speed can't be return journey, the speed was increased by
negative) 40 km/hr. Write down an expression for
the time taken for
Hence, required speed of the express train
(i) the onward journey,
= 60 km/h
(ii) the return journey.
27. A car covers a distance of 400 km at a certain If the return journey took 30 minutes less
speed. Had the speed been 12 km/h more, than the onward journey, write down an
the time taken for the journey would have equation in x and find its value. (2002)
been 1 hour 40 minutes less. Find the original Sol. Given distance = 400 km
speed of the car. (1996) and Speed of aeroplane = x km/hr
Sol. Let the original speed of the car = x km/h. (i)  Time taken by aeroplane to cover of
Distance covered = 400 km 400
400 km  hours
Time taken to cover a distance of 400 km x
On increasing the speed by 40 km/hr,
400
= h on the return journey, the speed = (x + 40)
x
km/hr.
86 Arun Deep's Understanding Math-10
400
(ii) Time taken  hours 216 x  216 ×16  208 x 2
x  40  
Now according to the condition, we have x ( x  16) 1
400 400 1 8 x  3456
  30 minutes = 
2
x x  40 2 
x 2  16 x 1
 1 1  1
 400     8 x + 3456 = 2 x2 + 32 x
 x x  40  2
 2 x2 + 32 x – 8 x – 3456 = 0
 x  40  x  400 × 40 1
 400   x 2  40 x  2  2 x2 + 24 x – 3456 = 0
 x( x  40)   x2 + 12 x – 1728 = 0 (Dividing by 2)
 x + 40 x = 400 × 40 × 2
2
 2
x + 48 x – 36 x – 1728 = 0
 x2 + 40 x – 32000 = 0  x (x + 48) – 36 (x + 48) = 0
 x2 + 200 x – 160 x – 32000 = 0  (x + 48) (x – 36) = 0
 x (x + 200) – 160 (x + 200) = 0 Either x + 48 = 0, then x = – 48,
 (x + 200) (x – 160) = 0 which is not possible as it is negative.
Either x + 200 = 0, then x = – 200, which or x – 36 = 0, then x = 36 Ans.
is not possible as it is negative.
(iv) Speed of the train = (x + 16) km/hr.
or x – 160 = 0, then x = 160 Ans.
= (36 + 16) km.hr. = 52 km/hr.
29. The distance by road between two towns
30. An aeroplane flying with a wind of 30 km/hr
A and B, is 216 km, and by rail it is 208
takes 40 minutes less to fly 3600 km, than
km. A car travels at a speed of x km/hr,
what it would have taken to fly against
and the train travels at a speed which is 16
the same wind. Find the planes’ speed of
km/hr faster than the car. Calculate :
flying in still air.
(i) The time taken by the car, to reach town B
from A, in terms of x ; Sol. Let the speed of the plane in still air = x
(ii) The time taken by the train, to reach town km/hr
B from A, in terms of x ; Speed of wind = 30 km/hr
(iii) If the train takes 2 hours less than the car, Distance = 3600 km
to reach town B, obtain an equation in x
3600
and solve it.  Time taken with the wind 
x  30
(iv) Hence find the speed of the train. (1998)
3600
Sol. Given the distance by road between and time taken against the wind 
x  30
A and B = 216 km
and the distance by rail = 208 km According to the condition, we have
speed of car = x km/hr 3600 3600 2
  40 minutes  hour
and speed of train = (x + 16) km/hr. x  30 x  30 3
216
(i) Time taken by car  hours .  3600 F 1  1 I2
x
208
H x  30 x  30 K 3
(ii) Time taken by train  hours .
F x  30  x  30 IJ  2
x  16  3600 G
(iii) According to the condition, we have H ( x  30) ( x  30) K 3
216 208 3600 × 60 2
 2  
x x  16 x 2  900 3
87 Arun Deep's Understanding Math-10
 x2
2 – 1800 = 3 × 3600 × 60 Either x – 30 = 0, then x = 30
 2 x2 – 1800 = 648000 or x + 25 = 0, then x = – 25, but it is not
 2 x2 – 1800 – 648000 = 0 possible as it is negative.
 2 x2 – 649800 = 0  Speed of bus = 30 km
 x2 – 324900 = 0 (Dividing by 2) and time taken while returning
 x2 – (570)2 = 0 150 150 150
    6 hours Ans.
 (x + 570) (x – 570) = 0 x  5 30  5 25
Either x + 570 = 0, then x = – 570 32. A boat can cover 10 km up the stream and
which is not possible as it is negative 5 km down the stream in 6 hours. If the
speed of the stream is 1.5 km/hr, find the
or x – 570 = 0, then x = 570 speed of the boat in still water.
Hence required speed of plane in still air Sol. Distance up stream = 10 km
= 570 km/hr. Ans.
and down stream = 5 km
31. A school bus transported an excursion
party to a picnic spot 150 km away. While Total time taken = 6 hours
returning, it was raining and the bus had to Speed of stream = 1.5 km/hr
reduce its speed by 5 km/hr, and it took Let speed of boat in still water = x km/hr
one hour longer to make the return trip. According to the condition, we have
Find the time taken to return.
10 5
Sol. Distance = 150 km  6
x  1.5 x  1.5
Let the speed of bus = x km/hr
 10x + 15 + 5x + 5x – 7.5 = 6 (x – 15) (x + 1.5)
 Time taken by bus to cover a distance
150  15x + 7.5 = 6(x2 – 2.25)
 hour
x  15x + 7.5 = 6x2 – 13.5
On returning speed of the bus = (x – 5) km/hr
 6x2 – 15x – 13.5 – 7.5 = 0
 Time taken by bus to cover a distance 150
 6x2 – 15x – 21 = 0
150
  2x2 – 5x – 7 = 0 (Dividing by 3)
x5
According to the condition, we have  2x2 – 7x + 2x – 7 = 0
150 150
 1  2  (7)  14
x 5 x 
F I   14  7  2
 150 1  1  1  5  7  2
H
x5 x K 


F x  x  5IJ  1
150 G
 x(2x – 7) + 1 (2x – 7) = 0
H x ( x  5) K  (2x – 7) (x + 1) = 0
150 × 5 Either 2x – 7 = then 2x = 7
  1  x2 – 5 x = 750
x2  5x
 x= 7
2
 x2 – 5 x – 750 = 0
 x2 – 30 x + 25 x – 750 =0 or x + 1 = 0, then x = –1
 x (x – 30) + 25 (x – 30) = 0 But it is not possible being negative.
 (x – 30) (x + 25) = 0  x= 7 = 3.5
2
88 Arun Deep's Understanding Math-10
 Required speed of boat = 3.5 km/hr Sol. (i)  Given amount received by x children
33. Two pipes running together can fill a tank = Rs. 480
1 480
in 11 minutes. If one pipe takes 5 min-  Share of each child = Rs.
9 x
utes more than the other to fill the tank,
According to the question ; we have
find the time in which each pipe would fill
the tank. 480 480
= – 12
Sol. Let time taken by one pipe = x minutes x  20 x
Then time taken by second pipe = (x + 5)
minutes 480 480  12 x
 =
1 x  20 x
Time taken by both pipes  11 minutes
9
Now according to the condition, we have 480 1240  x 
 =
1 1 9 x  20 x
 
x x  5 100  (x + 20) (40 – x) = 40x
( x  5)  x 9 x 5 x 9  40x – x2 + 800 – 20x = 40x
 x( x  5)  100  
x  5x
2 100  x2 + 20x – 800 = 0
2x  5 9  x2 + 40x – 20x – 800 = 0
 2 
x  5 x 100  x (x + 40) – 20 (x + 40) = 0
 9x2 + 45x = 200x + 500
 (x + 40) (x – 20) = 0
 9x2 + 45x – 200x – 500 = 0
 x = –40, x = 20
 9x2 – 155x – 500 = 0
Since – ve value of x is not possible.
 9x2 – 180x + 25x – 500 = 0
 Required no. of children = 20
 9x (x – 20) + 25 (x – 20) = 0
(ii) Let the no. of children be x.
 (x – 20) (9x + 25) = 0
Either x – 20 = 0, then x = 20. Amount = ` 7500
Or 9x + 25 = 0, then 9x = – 25 7500
 Each gets =
25 x
x but is not possible as it is in negative.
9 New number = x – 20
 x = 20
Amount = ` 7500
Hence the first pipe can fill the tank in 20
minutes and second pipe can do the same 7500
in 20 + 5 = 25 minutes. Ans. Each gets =
x  20
34. (i) ` 480 is divided equally among ‘x’ chil-
dren. If the number of children were 20 more 7500 7500
   100
then each would have got ` 12 less. Find ‘x’. x  20 x
(ii) ` 7500 were divided equally among a certain 7500 x  7500( x  20)
number of children. Had there been 20 less   100
x( x  20)
children, each would have received ` 100
more. Find the original number of children.  7500x – 7500x + 150000 = 100 (x2 – 20x)
89 Arun Deep's Understanding Math-10
 150000 = 100(x2 – 20x) (ii) Write down the equation in x for the above
 x2 – 20x – 1500 = 0 situation and solve it to find x. (1999)
x2 – 50x + 30x – 1500 = 0 Sol. Total cost = Rs. 600, No. of articles = x
600
x (x – 50) + 30 (x – 50) = 0 (i)  Cost of one article  Rs.
x
(x – 50) (x + 30) = 0 In second case price of one article
 x = 50 or x = –30 but x cannot be negative. 600
 Rs. 5
x
 Original number of children = 50 Ans. (ii) According to given condition, we have
35. 2x articles cost Rs. (5x + 54) and (x + 2) 600  5 x 600

similar articles cost Rs. (10x – 4), find x. x x4
Sol. Given, cost of 2x articles = 5x + 54  (x – 4) (600 + 5x) = 600x
 600x – 2400 + 5x2 – 20x = 600x
5 x  54  5x2 + 600x – 20x – 600x – 2400 = 0
 Cost of 1 article  ...(i)
2x  5x2 – 20x – 2400 = 0
Again cost of x + 2 articles = 10x – 4  x2 – 4x – 480 = 0 (Dividing by 5)
10 x  4  x2 – 24x + 20x – 480 = 0
 Cost of 1 article  ...(ii)  x (x – 24) + 20 (x – 24) = 0
x2
 (x – 24) (x + 20) = 0
From (i) and (ii) ; we have
Either x – 24 = 0, then x = 24
5 x  54 10 x  4 or x + 20 = 0, then x = – 20, but it is

2x x2 not possible as it is in negative.
 (5x + 54) (x + 2) = 2x (10x – 4)  x = 24 Ans.
 5x2 + 10x + 54x + 108 = 20x2 – 8x 37. A shopkeeper buys a certain number of books
 5x2 + 10x + 54x + 108 – 20x2 + 8x = 0 for 960. If the cost per book was 8 less, the
number of books that could be bought for 960
 – 15x2 + 72x + 108 = 0 would be 4 more. Taking the original cost of
 5x2 – 24x – 36 = 0 (Dividing by –3) each book to be x, write an equation in x and
 5x2 – 30x + 6x – 36 = 0 solve it to find the original cost of each book.
 5x (x – 6) + 6 (x – 6) = 0 Sol. Let original cost = x
960
 (x – 6) (5x + 6) = 0 No. of books bought =
x
Either x – 6 = 0, then x = 6 New cost of books = (x – 8)
960
6  No. of books bought =
or 5x + 6 = 0, then 5x = – 6  x  , x8
5 960
but it is not possible as it is in nega- If no. of books bought is 4 more then cost = +4
x
tive.  According to condition, we have
 x = 6. Ans. 960 960  1 1
– = 4 960  x  8  x  = 4
36. A trader buys x articles for a total cost of x8 x  
Rs. 600. x  x  8 4 x x8 4
(i) Write down the cost of one article in terms  x  x  8 =  =
960 x  8x
2
960
of x. If the cost per article were Rs. 5 more,
8 1
the number of articles that can be bought  2 = x2 – 8x = 8 × 240
for Rs. 600 would be four less. x  8x 240
90 Arun Deep's Understanding Math-10

 x2 – 8x – 1920 = 0  750 = x2 + 5x  x2 + 5x – 750 = 0


on comparing with ax2 + bx + c = 0  x2 + 30x – 25x – 750 = 0
Here a = 1 ; b = –8 ; c = – 1920  x (x + 30) –25 (x + 30) = 0
 (x + 30) (x – 25)= 0
 b  b 2  4 ac Either x + 30 = 0, then x = –30 which is not
 x
2a possible being negative.
  8   82  41 1920 or x – 25 = 0, then x = 25
x=
2  Length of cloth piece in the begining
= 25 metres
8  64  7680 8  7744
= = 300
2 2 and rate per metre = Rs. = Rs. 12
25
8  88 8  88 8  88 96 80 39. The hotel bill for a number of people for
= = , = ,
2 2 2 2 2 overnight stay is Rs. 4800. If there were 4
= 48, –40 (rejecting) more, the bill each person had to pay would
 Required cost of book = 48 have reduced by Rs. 200. Find the number
38. A piece of cloth costs Rs. 300. If the piece of people staying overnight. (2000)
was 5 metre longer and each metre of cloth Sol. Let the number of people = x
costs Rs. 2 less, the cost of the piece would Amount of bill = Rs. 4800
have remaind unchanged. How long is the
4800
original piece of cloth and what is the rate Then bill for each person  Rs.
x
per metre ?
In second case,
Sol. Total cost of cloth piece = Rs. 300 the number of people = x + 4
Let the length of the piece of cloth in the 4800
then bill of each person 
beginning = x m x4
300 According to the condition, we have
Then cost of 1 metre = Rs. 4800 4800
x   200
In second case, length of cloth = (x + 5) x x4

 Cost of 1 metre = Rs.


300 F
 4800 1  1  200 I
x5 H
x x4 K
According to the condition, we have
 x4x 4800 × 4
300 300  4800  x ( x  4)   200   200
– =2   x ( x  4)
x x5
1 1   19200 = 200x2 + 800x
 300  x  x  5  = 2  200x2 + 800x – 19200 = 0.
 
 x2 + 4x – 96 = 0 (Dividing by 200)
 x5 x   x2 + 12x – 8x – 96 = 0
 300  x x  5  =2  x (x + 12) – 8 (x + 12) = 0
 
 (x + 12) (x – 8) = 0
300  5 Either x + 12 = 0, then x = –12, but it is not
 xx  5 =2 possible as it is in negative.
or x – 8 = 0, then x = 8.
 Required no. of people = 8 Ans.
150  5 40. A person was given Rs. 3000 for a tour. If
 x x  5  =1 (Dividing by 2)
he extends his tour programme by 5 days,
91 Arun Deep's Understanding Math-10
he must cut down his daily expenses by 250
Or 3x – 250 = 0, then 3x = 250  x 
Rs. 20. Find the number of days of his 3
tour programme. But it is not possible.
Sol. Let the number of days of tour programme = x Loss = 10%
Given total amount = Rs. 3000  Cost price = 60x = 60 × 10 = Rs. 600 Ans.
3000 42. (i) The sum of the ages of Vivek and his
 Expenses for each day  younger brother Amit is 47 years. The
x
In second case, no. of days = x + 5. product of their ages in years is 550. Find
their ages. (2017)
3000
then expenses of each day  (ii) Paul is x years old and his father’s age is
x5 twice the square of Paul’s age. Ten years
Now according to the condition, we have hence, father’s age will be four times Paul’s
3000 3000 age. Find their present ages.
  20
x x5 Sol. (i) Let Vivek's present age be x years.
F I  His brother's age = (47 – x) years
 3000 1  1  20
x x5H K According to question, we have
x(47 – x) = 550
( x  5  x)  47x – x2 = 550
 3000  20
x 2  5x  x2 – 47x + 550 = 0
 3000 × 5 = 20x2 + 100x  x2 – 25x – 22x + 550 = 0
 20x2 + 100x – 15000 = 0  x(x – 25) – 22(x – 25) = 0
 x2 + 5x – 750 = 0 (Dividing by 20)  (x – 25)(x – 22) = 0
 x2 – 25x + 30x – 750 = 0  x – 25 = 0 or x – 22 = 0
 x (x – 25) + 30 (x – 25) = 0  x = 25 or x = 22
 (x – 25) (x + 30) = 0 When x = 25, then 47 – x = 47 – 25 = 22
Either x – 25 = 0, then x = 25 When x = 22, then 47 – x = 47 – 22 = 25
or x + 30 = 0, then x = – 30, but it is not (does not satisfy the given condition)
possible as it is in negative.  Vivek's age = x = 25 years.
 Required no. of days = 25 Ans. His younger brother's age = 22 years.
41. Ritu bought a saree for Rs. 60 x and sold it (ii) Let age of Paul = x years.
for Rs. (500 + 4x) at a loss of x%. Find Father’s age = 2x2
the cost price. 10 years hence,
Sol. Given cost price of saree = Rs. 60x Age of Paul = x + 10
and selling price = Rs. (500 + 4x) and father’s age = 2x2 + 10
Loss = x% According to the conditions, we have
Now according to the condition, we have 2x2 + 10 = 4(x + 10)
100  Loss%  2
2x + 10 = 4x + 40
S.P. = C.P. ×  2x2 + 10 – 4x – 40 = 0
100
 2x2 – 4x – 30 = 0
60 x (100  x )
500  4 x   x2 – 2x – 15 = 0 (Dividing by 2)
100  x2 – 5x + 3x – 15 = 0
 50000 + 400x = 6000x – 60x2  x (x – 5) + 3 (x – 5) = 0
 60x2 – 6000x + 400x + 50000 = 0  (x – 5) (x + 3) = 0
 60x2 – 5600x + 50000 = 0 Either x – 5 = 0, then x = 5
 3x2 – 280x + 2500 = 0 (Dividing by 20) or x + 3 = 0, then x = – 3, but it is not
 3x2 – 30x – 250x + 2500 = 0 possible as it is in negative.
 3x (x – 10) – 250 (x – 10) = 0  Age of Paul = 5 years.
 (x – 10) (3x – 250) = 0 and his father’s age = 2x2 = 2(5)2 = 2 × 25
Either x – 10 = 0, then x = 10 = 50 years Ans.
92 Arun Deep's Understanding Math-10
43. The age of a man is twice the square of the If x = 3, then
age of his son. Eight years hence, the age Present age of man = 3x2 + 2
of the man will be 4 years more than three = 3 (3)2 + 2 = 27 + 2 = 29 years
times the age of his son. Find their present and age of daughter = x + 2 = 3 + 2 = 5 years.
age. 45. The length (in cm) of the hypotenuse of a
Sol. Let the present age of the son = x years right angled triangle exceeds the length of
then, present age of the man = 2x2 years. one side by 2 cm and exceeds twice the
8 years hence, length of other side by 1 cm. Find the length
The age of son will be = (x + 8) years of each side. Also find the perimeter and
and the age of man = (2x2 + 8) years the area of the triangle.
According to the problem, we have Sol. Let the length of one side = x cm
2x2 + 8 = 3 (x + 8) + 4 and other side = y cm.
 2x2 + 8 = 3x + 24 + 4 then hypotenues = x + 2, and 2y + 1
 2x2 – 3x – 24 – 4 + 8 = 0  x +2 = 2y + 1  x – 2y = 1 – 2
 2x2 – 3x – 20 = 0  x – 2y = – 1  x = 2y – 1 ...(i)
 2x2 – 8x + 5x – 20 = 0 and by using Pythagorous theorem,
 2x (x – 4) + 5 (x – 4) = 0 x2 + y2 = (2y + 1)2
 (x – 4) (2x + 5) =0  x + y = 4y2 + 4y + 1
2 2
Either x – 4 = 0, then x = 4  (2y – 1)2 + y2 = 4y2 + 4y + 1 [From (i)]
5  4y2 – 4y + 1 + y2 = 4y2 + 4y + 1
or 2x + 5 = 0 then 2x = – 5  x  
2  4y2 – 4y + 1 + y2 – 4y2 – 4y – 1 = 0
But, it is not possible.  y2 – 8y = 0 y (y – 8) = 0
 Required present age of the son = 4 years Either y = 0, but it is not possible.
and present age of the man = 2x2
or y – 8 = 0 then y = 8
= 2 (4)2 years = 32 years Ans.
44. Two years ago, a man's age was three times Substituting the value of y in (i) ; we get
the square of his daughter’s age. Three x = 2(8) – 1 = 16 – 1 = 15
years hence, his age will be four times his  Length of one side = 15 cm
daughter’s age. Find their present ages. and length of other side = 8 cm
Sol. 2 years ago,
and hypotenuse = x + 2 = 15 + 2 = 17
Let the age of daughter = x
age of man = 3x2  Perimeter = 15 + 8 + 17 = 40 cm
then present age, of daughter = x + 2 1
and mean = 3x2 + 2 and Area  ×one side × other side
2
and 3 years hence, the age of
the daughter = x + 2 + 3 = x + 5 1
 × 15 × 8  60 cm 2 Ans.
and of man = 3x2 + 2 + 3 = 3x2 + 5 2
According to the condition,we have 46. If twice the area of a smaller square is
3x2 + 5 = 4 (x + 5)  3x2 + 5 = 4x + 20 subtracted from the area of a larger square,
 3x2 – 4x + 5 – 20 = 0  3x2 – 4x – 15 = 0 the result is 14 cm2. However, if twice the
 3x2 – 9x + 5x –15 = 0
area of the larger square is added to three
 3x (x – 3) + 5 (x – 3) = 0
 (x – 3) (3x + 5) = 0
times the area of the smaller square, the
Either x – 3 = 0, then x = 3 result is 203 cm2. Determine the sides of
the two squares.
5
or 3x + 5 = 0, then 3x = –5  x = Sol. Let the side of smaller square = x cm
3 and side of bigger square = y cm
which is not possible, as age can't be
negative. According to the condition, we get
93 Arun Deep's Understanding Math-10
y2 – 2x2 = 14 ...(i) It is a quadratic equation as it is a polynomal
and 2y2 + 3x2 = 203 ...(ii) of degree 2.
(b) Given eqn. be, x2 + 3x = (–1) (1 – 3x)
Multiply (i) by 2 and (ii) by 1
 x2 + 3x = –1 + 3x
2y2 – 4x2 = 28
 x2 + 1 = 0
2y2 + 3x2 = 203
It is also quadratic equation as it is a
Subtracting, we get, – 7x2 = – 175 polynomal of degree 2.
175 (c) Given eqn. be, (x + 2) (x – 1) = x2 – 2x – 3
 x2   25
7 x2 – x + 2x – 2 = x2 – 2x – 3
x2 – 25 = 0  (x + 5) (x – 5) = 0 x2 – x2 + x + 2x – 2 + 3 = 0
Either x + 5 = 0 then x = – 5, but it is not  3x + 1 = 0
possible, or x – 5 = 0, then x = 5.
It is not a quadratic equation as it is a
Substitute the value of x in (i) ; we have polynomal of degree 1.
y2 – 2(5)2 = 14 (d) (x + 1)3

 y2 = 14 + 2 × 25 x3 – x2 + 2x + 1 = x3 + 3x2 + 3x + 1

y2 = 14 + 50 = 64 = (8)2  x3 – x2 + 2x + 1 – x3 – 3x2 – 3x – 1 = 0

 y=8  3x2 + x2 – 2x – 1 + 3x + 1 = 0
 4x2 + x = 0
Hence required side of smaller square
= 5 cm It is a quadratic equation.
 Ans. (c)
and side of bigger square = 8 cm Ans.
1
MULTIPLE CHOICE QUESTIONS 2. If is a root of the quadratic equation
2
Choose the correct answer from the given four 4x2 – 4kx + k + 5 = 0, then the value of k is
oprtions (1 to 15) : (a) –6 (b) –3
1. Which of the following is not a quadratic (c) 3 (d) 6
equation ?
1
(a) (x + 2)2 = 2(x + 3) Sol. Since,
2
is a root of the equation
(b) x2 + 3x = (–1) (1 – 3x)
4x2 – 4kx + k + 5 = 0
(c) (x + 2) (x – 1) = x2 – 2x – 3
1
(d) x3 – x2 + 2x + 1 = (x + 1)3 Substituting the value of x = in the given
2
Sol. (a) Given eqn. be, (x + 2)2 = 2(x + 3) equation, we have
 x2 + 4x + 4 = 2x + 6
2
 x2 + 4x – 2x + 4 – 6 = 0 1 1
4  – 4 × k × +k+5=0
 x2 + 2x – 2  
2 2
94 Arun Deep's Understanding Math-10
1 – 2k + k + 5 = 0  –k + 6 = 0 Sol. (a) Given eqn. be,
k=6 (x – 2) (x + 1) = (x – 1) (x – 3)
 Ans. (d)
 x2 + x – 2x – 2 = x2 – 3x – x + 3
3. The roots of the equation x2 – 3x – 10 = 0
are  3x + x – 2x + x = 3 + 2

(a) 2, –5 (b) –2, 5  3x = 5

(c) 2, 5 (d) –2, –5 It is not a quadratic equation as it is a


polynomal of degree 1.
Sol. Given eqn. be, x2 – 3x – 10 = 0
(b) Given eqn. be, (x + 2)3 = 2x(x2 – 1)
On comparing with ax2 + bx + c = 0
 x3 + 6x2 + 12x + 8 = 2x3 – 2x
Here a = 1 ; b = –3 ; c = –10
 x3 + 6x2 + 12x + 8 – 2x3 + 2x = 0
 b  b 2  4 ac  –x3 + 6x2 + 14x + 8 = 0
 x
2a
It is not a quadratic equation as it is a
polynomal of degree 3.
 (3)  (3)  4  1  (10)
2
x=
2 1 (c) Given eqn. be, x2 + 3x + 1 = (x – 2)2
 x2 + 3x + 1 = x2 – 4x + 4
3  9  40
=  3x + 1 + 4x – 4 = 0
2
 7x – 3 = 0
3  49
= It is not a quadratic equation as it is a
2 polynomal of degree.

37 (d) Given eqn. be, 8(x – 2)3 = (2x – 1)3 + 3


=
2  8(x3 – 6x2 + 12x – 8)

3 7 37 4 = 8x3 – 12x2 + 6x – 1 + 3


 x= = 5 and x = = = –2  8x3 – 48x2 + 96x – 64 – 8x3 + 12x2 – 6x + 1
2 2 2
–3=0
 x = 5, –2 or –2, 5 (b)
 –36x2 + 90x – 66 = 0
P.Q. Which of the following is a quadratic
It is a quadratic equation as it is polynomal
equation ?
of degree 2.
(a) (x – 2) (x + 1) = (x – 1) (x – 3)
Ans. (d)
(b) (x + 2)3 = 2x(x2 – 1)
4. If one root of a quadratic equation with
(c) x2 + 3x + 1 = (x – 2)2
3 5
(d) 8(x – 2)3 = (2x – 1)3 + 3 rational coefficients is , then the other
2
95 Arun Deep's Understanding Math-10
root is Sol. Given, eqn. be,
2x2 – kx + k = 0
3 5 3 5
(a) (b) On comparing with ax2 + bx + c = 0
2 2
a = 2, b = –k, c = k
3 5 3 5  b2 – 4ac = (–k)2 – 4 × 2 × k
(c) (d) = k2 – 8k
2 2
Sol. One root of a quadratic equation is Roots are equal.
 b2 – 4ac = 0
3 5 k2 – 8k = 0  k(k – 8) = 0
, then other root will be
2 Either k = 0
or k – 8 = 0, then k = 8
3 5
.  Ans. (c) k = 0, 8  Ans. (d)
2 2
7. If the equation 3x – kx + 2k = 0 has equal
Since we know that if a  b be the root of roots, then the the value(s) of k is (are)
(a) 6 (b) 0 only
quadratic eqn. then a – b be also the root
(c) 24 only (d) 0 or 24
of similar equation.
Sol. Given eqn. be,
5. If the equation 2x2 – 5x + (k + 3) = 0 has
equal roots then the value of k is 3x2 – kx + 2k = 0
On comparing with ax2 + bx + c = 0
9 9 Here, a = 3, b = –k, c = 2k
(a) (b) 
8 8  b2 – 4ac = (–k)2 – 4 × 3 × 2k
1 1 = k2 – 24k
(c) (d)  Roots are equal.
8 8
Sol. Given eqn. be,  b2 – 4ac = 0
2x2 – 5x + (k + 3) = 0  k2 – 24k = 0  k(k – 24) = 0
On comparing with ax2 + bx + c = 0 Either k = 0,
a = 2, b = –5, c = k + 3 or k – 24 = 0, then k = 24
 b2 – 4ac = (–5)2 – 4 × 2 × (k + 3)  k = 0, 24  Ans. (d)
= 25 – 8(k + 3) 8. If the equation (k + 1)x2 – 2(k – 1)x + 1 = 0
has equal roots, then the values of k are
Roots are equal.
(a) 1, 3 (b) 0, 3
 b2 – 4ac = 0
 25 – 8(k + 3) = 0 3
(c) 0, 1 (d) 0,
 25 – 8k – 24 = 0 4
 1 – 8k = 0  8k = 1 Sol. Given eqn. be,
1 (k + 1)x2 – 2(k – 1)x + 1 = 0
 k=  Ans. (c) On comparing with ax2 + bx + c = 0
8
6. The value(s) of k for which the quadratic Here, a = k + 1, b = –2(k – 1), c = 1
equation 2x2 – kx + k = 0 has equal roots is  b2 – 4ac = [–2(k – 1)]2 – 4(k + 1)(1)
(are) = 4(k2 – 2k + 1) – 4k – 4
(a) 0 only (b) 4 = 4k2 – 8k + 4 – 4k – 4
(c) 8 only (d) 0, 8 = 4k2 – 12k
96 Arun Deep's Understanding Math-10
Roots are equal.
 b2 – 4ac = 0  
2
b2 – 4ac =  5 – 4 × 2 × 1
 4k2 – 12k = 0 = 5 – 8 = –3
 4k(k – 3) = 0  k(k – 3) = 0 b2 – 4ac < 0
Either k = 0  It has no real roots.  Ans. (c)
or k – 3 = 0, then k = 3 PQ. Which of the following equations has two
 k = 0, 3  Ans. (b) distinct real roots ?
9. If the equation 2x2 – 6x + p = 0 has real and 9
different roots, then the values of p are given (a) 2x2 – 3 2 x + =0
4
by
(b) x2 + x – 5 = 0
9 9
(a) p < (b) p < (c) x2 + 3x + 2 2 = 0
2 2
(d) 5x2 – 3x + 1 = 0
9 9 Sol. (a) Given eqn. be,
(c) p > (d) p >
2 2
9
Sol. Given eqn. be, 2x2 – 3 2 x + =0
4
2x2 – 6x + p = 0
On comparing with ax2 + bx + c = 0
On comparing with ax2 + bx + c = 0
Here, a = 2, b = –6, c = p 9
Here, a = 2, b = –3 2 , c =
b2 – 4ac = (–6)2 – 4 × 2 × p 4
= 36 – 8p 9
Roots are real and unequal.  b2 – 4ac = (–3 2 )2 – 4 × 2 ×
4
 b2 – 4ac > 0
= 18 – 18 = 0
 36 – 8p > 0
 Roots are real and equal.
36 (b) Given eqn. be, x2 + x – 5 = 0
 36 > 8p  >p
8 Here, b2 – 4ac = (1)2 – 4 × 1 × (–5)
= 1 + 20 = 21 > 0
36 9
 p< p<  Ans. (a)  Roots are real and distinct.  Ans. (b)
8 2
PQ. Which of the following equations has no real
10. The quadratic equation 2x2 – 5x+1=0 roots ?
has (a) x2 – 4x + 3 2 = 0
(a) two distinct real roots
(b) x2 + 4x – 3 2 = 0
(b) two equal real roots
(c) no real roots (c) x2 – 4x – 3 2 = 0
(d) more than two real roots
(d) 3x2 + 4 3 x + 4 = 0
Sol. Given eqn. be,
Sol. (a) Given eqn. be,
2x2 – 5x+1=0
x2 – 4x + 3 2 = 0
On comparing with ax2 + bx + c = 0
On comparing with ax2 + bx + c = 0
Here, a = 2, b =  5 , c = 1
 b2 – 4ac = (–4)2 – 4 × 1 × 3 2
97 Arun Deep's Understanding Math-10
a
= 16 – 12 2 = 16 – 12(1.4) Hence x = – a, Ans.
2
= 16 – 16.8 = –0.8
2
b2 – 4ac < 0 (ii) 3 x + 10x + 7 3 = 0
 Roots are not real.  Ans. (a) Sol. Given eqn. be,
2
CHAPTER TEST 3 x + 10x + 7 3 = 0
 2
3 x + 3x + 7x + 7 3 = 0
Solve the following equations (1 to 4) by
factorisation :  3 x (x + 3 ) + 7 (x + 3) = 0
1.(i) x2 + 6x – 16 = 0  (x + 3 ) ( 3 x + 7) = 0
Sol. Given eqn. be,
Either x + 3 = 0, then x = – 3
x2 + 6x – 16 = 0
 x2 + 8x – 2x – 16 = 0 or 3 x + 7 = 0, then 3 x = –7
 x (x + 8) – 2 (x + 8) = 0 7
 (x + 8) (x – 2) = 0  x
3
Either x + 8 = 0, then x = – 8
or x – 2 = 0, then x = 2 7 × 3 7 3
Hence x = – 8, 2 Ans. x 
3× 3 3
(ii) 2
3x + 11x + 10 = 0
Sol. Given eqn. be, 7 3
Hence x =  3 ,
3x2 + 11x + 10 = 0 3
 3x2 + 6x + 5x + 10 = 0 3.(i) x (x + 1) + (x + 2) (x + 3) = 42
 3x (x + 2) + 5 (x + 2) = 0 Sol. Given eqn. be,
 (x + 2) (3x + 5) = 0 x (x + 1) + (x + 2) (x + 3) = 42
Either x + 2 = 0, then x = – 2 x2 + x + x2 + 5x + 6 = 42
2x2 + 6x + 6 – 42 = 0
or 3x + 5 = 0, then
 2x2 + 6x – 36 = 0
5  x2 + 3x – 18 = 0 (Dividing by 2)
3x = – 5  x 
3  2
x + 6x – 3x – 18 = 0
5  x (x + 6) – 3 (x + 6) = 0
Hence x = – 2, Ans.
3  (x + 6) (x – 3) = 0
2.(i) 2x2 + ax – a2 = 0 Either x + 6 = 0, then x = – 6
Sol. Given eqn. be, or x – 3 = 0, then x = 3
2x2 + ax – a2 = 0 Hence x = – 6, 3 Ans.
 2x2 + 2ax – ax – a2 = 0 6 2 1
(ii)  
 2x (x + a) – a (x + a) = 0 x x 1 x2
 (x + a) (2x – a) = 0 Sol. Given eqn. be,
Either x + a = 0, then x = – a 6 2 1
 
or 2x – a = 0, then x x 1 x2
a 6x  6  2 x 1
2x = a  x   
2 x ( x  1) x  2
98 Arun Deep's Understanding Math-10
4x  6 1  1  15  16 = 4
 
x x
2 x2 R.H.S. = x + 3 = 1 + 3 = 4
 (4x – 6) (x – 2) = x2 – x . . . L.H.S. = R.H.S.
 4x2 – 8x – 6x + 12 = x2 – x  x = 1 is a root of this equation.
 4x2 – 14x + 12 – x2 + x = 0 Hence x = 1 Ans.
 3x2 – 13x + 12 = 0 (ii) 3 x 2  2 x  1 = 2x – 2
 3x2 – 9x – 4x + 12 = 0
Sol. Given eqn. be,
 3x (x – 3) – 4 (x – 3) = 0
 (x – 3) (3x – 4) = 0 3 x 2  2 x  1 = 2x – 2
Either x – 3 = 0, then x = 3 On squaring both sides, we have
or 3x – 4 = 0, then 3x2 – 2x – 1 = (2x – 2)2
4  3x2 – 2x – 1 = 4x2 – 8x + 4
3x = 4  x   4x2 – 8x + 4 – 3x2 + 2x + 1 = 0
3
 x2 – 6x + 5 = 0
4
Hence x = 3, Ans.  x2 – 5x – x + 5 = 0
3
 x (x – 5) – 1 (x – 5) = 0
4.(i) x  15 = x + 3  (x – 5) (x – 1) = 0
Sol. Given eqn. be, Either x – 5 = 0, then x = 5
or x – 1 = 0, then x = 1
x  15 = x + 3
Check. (i) If x = 5, then
On squaring both sides ; we have
x + 15 = (x + 3)2 L.H.S.  3 x 2  2 x  1
 x + 15 = x2 + 6x + 9
 3 × (5) 2  2 × 5  1
 x2 + 6x + 9 – x – 15 = 0
 x2 + 5x – 6 = 0  3× 25  10  1  75  10  1
 x2 + 6x – x – 6 = 0  64  8
 x (x + 6) – 1 (x + 6) = 0 R.H.S. = 2x – 2 = 2 × 5 – 2
 (x + 6) (x – 1) = 0 = 10 – 2 = 8
Either x + 6 = 0, then x = – 6 . . . L.H.S. = R.H.S.
or x – 1 = 0, then x = 1  x = 5 be its root.
 x = – 6, 1 (ii) If x = 1, then
Check : L.H.S.  3 x 2  2 x  1
(i) If x = – 6 then
 3 (1) 2  2 (1)  1
L.H.S.  x  15
 3×1  2  1  3  2  1  0
 6  15  9 = 3
R.H.S. = 2x – 2 = 2 × 1 – 2 = 2 – 2 = 0
R.H.S. = x + 3 = – 6 + 3 = – 3 . . . L.H.S. = R.H.S.
... L.H.S.  R.H.S.
 x = 1 is also its root.
 x = – 6 is not a root.
Hence x = 5, 1 Ans.
(ii) If x = 1, then
Solve the following equations (5 to 8)
L.H.S.  x  15 by using formula :
99 Arun Deep's Understanding Math-10
5. (i) 2x2
– 3x – 1 = 0 Sol. Given eqn. be,
Sol. Given eqn. be, 2x  5 x  1
2x2 – 3x – 1 = 0 
3x  4 x  3
On comparing with ax2 + bx + c = 0  (2x + 5) (x + 3) = (x + 1) (3x + 4)
Here a = 2, b = – 3, c = – 1  2x2 + 6x + 5x + 15 = 3x2 + 4x + 3x + 4
Here, D = b2 – 4ac = (–3)2 – 4 × 2 × (–1)  3x2 + 7x + 4 – 2x2 – 11x – 15 = 0
= 9 + 8 = 17
 x2 – 4x – 11 = 0
b  D Here a = 1, b = – 4, c = – 11
... x
2a  D = b2 – 4ac = (–4)2 – 4 × 1 × (– 11)
 ( 3)  17 3  17 = 16 + 44 = 60
 
2×2 4 b  D
...
x
3  17 3  17 2a
 x , Ans.
4 4  ( 4 )  60 4  4 ×15
 
F 1
(ii) x 3 x  2  6
I 2 ×1 2
H K 4  2 15
  2  15
x 2
Sol. Given eqn. be, 3 x 2  6
2
 x  2  15 , 2  15 Ans.
 6x2 + x = 12
2 1 4 3
 6x2 + x – 12 = 0 (ii)   
x  2 x 1 x  4 x  3
On comparing with ax2 + bx + c = 0
Sol. Given eqn. be,
Here a = 6, b = 1, c = –12
2 1 4 3
D = b2 – 4ac = (1)2 – 4 × 6 × (–12)    
x  2 x 1 x  4 x  3
= 1 + 288 = 289
2 x  2  x  2 4 x  12  3 x  12
b  D 
... x ( x  2 ) ( x  1) ( x  4 ) ( x  3)
2a
x x
 
1  289 1  17 ( x  2 ) ( x  1) ( x  4 ) ( x  3)
 
26 12
 1 1 
1  17 16 4  x  ( x  2)( x  1)  ( x  4) ( x  3)   0
 x1     
12 12 3
1  17 18  1 1 
3  x 2  2 0
and x 2     x  3 x  2 x  7 x  12 
12 12 2
4 3  x(x2 + 7x + 12 – x2 – 3x – 2) = 0
 x ,  Ans.
3 2  x(4x + 10)
2x  5 x  1  2x(2x + 5) = 0
6. (i) 
3x  4 x  3 which 2x = 0 or 2x + 5 = 0
100 Arun Deep's Understanding Math-10
5 18
 2x = – 5  x   x= =6
2 3
If x = 0, then
5
5  x = 6,
Hence x = 0, Ans. 2
2
(ii) Given eqn. be,
3x  4 7 5 4
7. (i) + = ,x 4 5 3
7 3x  4 2 3 –3= , x  0, 
x 2x  3 2
4 5 3
(ii) –3= , x  0,  4  3x 5
x 2x  3 2  =
x 2x  3
Sol. (i) Given eqn. be,
 (4 – 3x) (2x + 3) = 5x
3x  4 7 5 4  8x + 12 – 6x2 – 9x – 5x = 0
+ = ,x
7 3x  4 2 3  –6x2 – 6x + 12 = 0
3x  4  x2 + x – 2 = 0
Let = y, then given eqn. be,  x2 + 2x – x – 2 = 0
7
 x(x + 2) – 1(x + 2) = 0
1 5
 y+ y =  2y2 + 2 = 5y  (x + 2) (x – 1) = 0
2 Either x + 2 = 0, then x = –2
 2y2 – 5y + 2 = 0 or x – 1 = 0, then x = 1
 2y2 – y – 4y + 2 = 0  x = 1, –2
 y(2y – 1) – 2(2y – 1) = 0 8. (i) x2 + (4 – 3a)x – 12a = 0.
 (2y – 1) (y – 2) = 0 (ii) 10 ax2 – 6x + 15ax – 9 = 0, a  0
Either 2y – 1 = 0, then Sol. (i) Given eqn. be,
x2 + (4 – 3a)x – 12a = 0
1
2y = 1  y = On comparing with ax2 + bx + c = 0
2
Here a = 1, b = 4 – 3a, c = –12a
or y – 2 = 0, then y = 2  D = b2 – 4ac
1 = (4 – 3a)2 – 4 × 1 × (–12a)
When y = , then = 16 – 24a + 9a2 + 48a
2
= 16 + 24a + 9a2 = (4 + 3a)2
3x  4 1
=  6x – 8 = 7 b  D
7 2  x=
 6x = 7 + 8  6x = 15 2A

15 5  4  3a   4  3a 2
 x= = =
6 2 2 1
y = 2, then
3a  4  3a  4
3x  4 2 =
=  3x – 4 = 14 2
7 1
3a  4  3a  4
 3x = 14 + 4 = 18  x1 =
2
101 Arun Deep's Understanding Math-10

6a Sol. Given eqn. be,


= = 3a (x – 1)2 – 3x + 4 = 0
2
2
 x + 1 – 2x – 3x + 4 = 0
3a  4  3a  4
and x2 =  x2 – 5x + 5 = 0
2
On comparing with ax2 + bx + c = 0
8
= = –4 Here a = 1, b = –5 and c = 5
2
 Roots are 3a, –4  b  b 2  4 ac
Thus, x 
(ii) Given eqn. be, 2a
10ax2 – (6 – 15a) x – 9 = 0
 ( 5)  (5) 2  4(1)(5)
On comparing with ax2 + bx + c = 0 x=
2
Here a = 10a, b = – (6 – 15a), c = – 9
 D = b2 – 4 ac 5  25  20
=
= [– (6 – 15a)]2 – 4 × 10a (– 9) 2
= 36 – 180a + 225a2 + 360a
5 5 5  2.236 5  2.236
= 36 + 180a + 225a2 = (6 + 15a)2 = = or
2 2 2
b  D 7.236 2.764
... x = or = 3.618, 1.382
2a 2 2
 x = 3.6, 1.4
[  ( 6  15a )]  ( 6  15a ) 2
 10. Discuss the nature of the roots of the
2 × 10a
following equations :
( 6  15a )  ( 6  15a )
 1
20a (i) 3x2 – 7x + 8 = 0 (ii) x2 – x–4=0
2
6  15a  6  15a
  x1  (iii) 5x2 – 6 5 x + 9 = 0
20a
12 3 (iv) 3 x2 – 2x – 3 =0
 
20a 5a In case real roots exist, then find them.
6  15a  6  15a Sol. (i) Given eqn. be,
x2 
20a 3x2 – 7x + 8 = 0
30a 3 On comparing with ax2 + bx + c = 0
 
20a 2 Here a = 3, b = –7, c = 8
3 3  D = b2 – 4ac = (–7)2 – 4 × 3 × 8
Hence x  , Ans. = 49 – 96 = – 47 < 0
5a 2
9. Solve for x using the quadratic formula.  D<0
Write your answer correct to two  Roots are not real.
significant figures :
1
(x – 1)2 – 3x + 4 = 0. (2014) (ii) Given eqn. be, x2 – x–4=0
2
102 Arun Deep's Understanding Math-10
On comparing with ax2 + bx + c = 0 = 4 (k + 2)2 – 4 (32k + 4 – 8k2 – k)
= 4 (k2 + 4k + 4) –4 (32k + 4 – 8k2 – k)
1
Here a = 1, b = , c = –4 = 4k2 + 16k + 16 – 128k – 16 + 32 k2 + 4k
2
= 36k2 – 108k = 36k (k – 3)
2
1  Roots are equal
 D = b2 – 4ac =   – 4 × 1 × (–4)
2  D=0
 36k (k – 3) = 0  k (k – 3) = 0
1 65
= + 16 = >0 Either k = 0
4 4 or k – 3 = 0, then k = 3
 D>0  k = 0, 3 Ans.
 Roots are real and distinct. 12. Find the values of m so that the quadratic
(iii) Given, 5x2 – 6 5 x + 9 = 0 equation 3x2 – 5x – 2m = 0 has two distinct
real roots.
On comparing with ax2 + bx + c = 0
Sol. Given eqn. be,
Here a = 5, b = –6 5,c=9 3x2 – 5x – 2m = 0
 D = b2 – 4ac On comparing with ax2 + bx + c = 0
Here a = 3, b = –5, c = –2m

= 6 5 2
– 4 × 5 × 9 = 180 – 180 = 0
 D = b2 – 4ac
 D=0 = (–5)2 –4 × 3 × (–2m) = 25 + 24m
 Roots are real and equal.  Roots are real and distinct.
 D>0
(iv) Given, 3 x2 – 2x – 3 =0
 25 + 24m > 0
On comparing with ax2 + bx + c = 0
 24m > –25
Here a = 3 , b = –2, c = – 3
25
 D= b2 – 4ac i.e. m > – Ans.
24
= (–2)2 – 4 ×  
3 ×  3 = 4 + 12 = 16 13. Find the value(s) of k for which each of the
following quadratic equation has equal roots:
 D>0
(i) 3kx2 = 4(kx – 1)
 Roots are real and distinct Ans. (ii) (k + 4)x2 + (k + 1)x + 1 =0
11. Find the values of k so that the quadratic equation Also, find the roots for that value (s) of k in
(4 – k) x2 + 2 (k + 2) x + (8k + 1) = 0 has equal each case.
roots. Sol. (i) Given eqn. be,
Sol. Given eqn. be, 3kx2 = 4 (kx – 1)
 3kx2 = 4kx – 4 3kx2 – 4kx + 4 = 0
(4 – k) x2 + 2 (k + 2) x + (8k + 1) = 0
On comparing with ax2 + bx + c = 0
On comparing with ax2 + bx + c = 0 Here a = 3k, b = –4k, c = 4
Here a = (4 – k), b = 2 (k + 2), c = 8k + 1  D = b2 – 4ac = (–4k)2 – 4 × 3k × 4
 D = b2 – 4ac = 16k2 – 48k
Roots are equal.
= [2 (k + 2)]2 – 4 × (4 – k) (8k + 1)
 D=0
103 Arun Deep's Understanding Math-10
16k – 48k = 0 k – 3k = 0 k (k – 3) =0
2 2
According to the condition ; we have
Either k = 0 x2 + (x + 3)2 = 117
or k – 3 = 0 then k = 3  x2 + x2 + 6x +9 = 117
b D b  2x2 + 6x + 9 – 117 = 0
 x= = ( D = 0)  2x2 + 6x – 108 = 0
2a 2a
 x2 + 3x – 54 = 0 (Dividing by 2)
4k 43 12 2  x2 + 9x – 6x – 54 = 0
 x= = = =
2  3k 2  3 3 18 3  x (x + 9) – 6 (x + 9) = 0
2 2  (x + 9) (x – 6) = 0
 x= ,
3 3 Either x + 9 = 0, then x = – 9, but it is not
(ii) Given eqn. be, a natural number.
(k + 4)x2 + (k + 1)x + 1 = 0 or x – 6 = 0, then x = 6
On comparing with ax2 + bx + c = 0  First natural number = 6
Here a = k + 4, b = k + 1, c = 1 and second number = 6 + 3 = 9 Ans.
 D = b2 – 4ac = (k + 1)2 – 4 × (k + 4) × 1 15. Divide 16 into two parts such that the twice
= k2 + 2k + 1 – 4k – 16 = k2 – 2k – 15 the square of the larger part exceeds the
Root are equal. square of the smaller part by 164.
 k2 – 2k – 15 = 0 Sol. Let larger part = x
 k2 – 5k + 3k – 15 = 0 then smaller part = 16 – x (... sum = 16)
 k (k – 5) + 3 (k – 5) = 0 According to the condition ; we have
 (k – 5) (k + 3) = 0
2x2 – (16 – x)2 = 164
Either k – 5 = 0, then k = 5
or k + 3 = 0, then k = –3  2x – (256 – 32x + x2) = 164
2

(a) When k = 5, then  2x2 – 256 + 32x – x2 = 164


 x2 + 32x – 256 – 164 = 0
b D b k  1 5  1
x= = = 2k  4  = 25  4   x2 + 32x – 420 = 0
2a 2a  x2 + 42x – 10x – 420 = 0
6 1  x (x + 42) – 10 (x + 42) = 0
= =  (x + 42) (x – 10) = 0
18 3
Either x + 42 = 0, then x = – 42, but it is
1 1
 x= , not possible.
3 3
or x – 10 = 0, then x = 10
(b) When k = –3, then
 Larger part = 10
b D b k  1 and smaller part = 16 – 10 = 6 Ans.
x= = = 2k  4 
2a 2a 16. Two natural numbers are in the ratio 3 : 4.
Find the numbers if the difference between
 3  1 2
their squares is 175.
= = =1
2  3  4  2 1
Sol. Ratio in two natural numbers = 3 : 4
 x = 1, 1 Let the numbers be 3x and 4x
14. Find two natural numbers which differ by According to the condition, we have
3 and whose squares have the sum 117.
(14x)2 – (3x)2 = 175
Sol. Let first natural number = x
 16x2 – 9x2 = 175
then second natural number = x + 3
104 Arun Deep's Understanding Math-10
 7x2 = 175  x2+ 12x = 16x + 96
175  x2 + 12x – 16x – 96 = 0
 x2 = = 25 = (5) 2  x2 – 4x – 96 = 0
1
 x = 5, – 5  x2 – 12x + 8x – 96 = 0
But x = – 4 is not a natural number.  x (x – 12) + 8 (x – 12) = 0
 x=5  (x – 12) (x + 8) = 0
 Required natural numbers are 3x, 4x Either x – 12 = 0, then x = 12
= 3 × 5, 4 × 5 = 15, 20. or x + 8 = 0, then x = – 8, but it is not
17. Two squares have sides x cm and (x + 4) possible as it is in negative.
cm. The sum of their areas is 656 sq. cm.  Breadth of rectangular garden = 12 m
Express this as an algebraic equation and and length of rectangular garden = 12 + 12
solve it to find the sides of the squares. = 24 m. Ans.
Sol. Side of first square = x cm 19. A farmer wishes to grow a 100 m2 rectangu-
and side of second square = (x + 4) cm lar vegetable garden. Since he has with him
Now according to the condition, we have only 30 m barbed wire, he fences three sides
(x)2 + (x + 4)2 = 656 of the rectangular garden letting compound
 x2 + x2 + 8x + 16 = 656 wall of his house act as the fourth side fence.
Find the dimensions of his garden.
 2x2 + 8x + 16 – 656 = 0
Sol. Area of rectangular garden = 100 cm2
 2x2 + 8x – 640 = 0
Length of barbed wire = 30 m = perimeter
 x2 + 4x – 320 = 0 (Dividing by 2)
of wire
 x2 + 20x – 16x – 320 = 0
Let the lengh of side opposite to wall = x
 x (x + 20) – 16 (x + 20) = 0
 (x + 20) (x – 16) = 0 30  x
and length of other each side 
2
Either x + 20 = 0, then x = – 20, but it not
x ( 30  x )
possible as it is in negative. According to the condition,  100
2
or x – 16 = 0 then x = 16
 x (30 – x) = 200
 Side of first square = 16 cm
 30x – x2 = 200  x2 – 30x + 200 = 0
and side of second square = 16 + 4 = 20 cm Ans.
 x2 – 20x – 10x + 200 = 0
18. The length of a rectangular garden is 12 m
more than its breadth. The numerical value  x (x – 20) – 10 (x – 20) = 0
of its area is equal to 4 times the numerical  (x – 20) (x – 10) = 0
value of its perimeter. Find the dimensions Either x – 20 = 0, then x = 20
of the garden. or x – 10 = 0, then x = 10
Sol. Let breadth of rectangular garden = x m (i) If x = 20, then side opposite to the wall
then length rectangular garden = (x + 12) m = 20m
 Area = l × b = x (x + 12) m2 30  20 10
and other side   5m
and perimeter = 2 (l + b) 2 2
= 2(x + 12 + x) = 2 (2x + 12) m (ii) If x = 10, then side opposite to wall
= 10 m
According to the condition, we have
30  10 20
x (x + 12) = 4 × 2 (2x + 12) and other side    10 m
2 2
105 Arun Deep's Understanding Math-10
 Sides are 20 m, 5m or 10m, 10 m Ans.  x2 – 62x + 672 = 0 (Dividing by 2)
20. The hypotenuse of a right angled triangle  x2 – 48x – 14x + 672 = 0
is 1 m less than twice the shortest side. If  x (x – 48) – 14 (x – 48) = 0
the third side is 1m more than the shortest
side, find the sides of the triangle.  (x – 48) (x – 14) = 0
Sol. Let the length of shortest side = x m Either x – 48 = 0, then x = 48
 Length of hypotenuse = 2x – 1 or x – 14 = 0, then x = 14
and third side = x + 1 (i) If x = 48, then one side = 48 cm
Now according to the condition, we have and other side = 62 – 48 = 14 cm
(2x – 1)2 = (x)2 + (x + 1)2 (ii) If x = 14, then one side = 14 cm
(By Pythagorus Theorem) and other side = 62 – 14 = 48
 4x2 – 4x + 1 = x2 + x2 + 2x + 1 Hence sides are 14 cm, 48 cm Ans.
 4x2 – 4x + 1 – 2x2 – 2x – 1 = 0 PQ. Car A travels x km for every litre of petrol,
 2x2 – 6x = 0 while car B travels (x + 5) km for every
litre of petrol.
 x2 – 3x = 0 (Dividing by 2)
(i) Write down the number of litres of petrol
 x (x – 3) = 0
used by car A and car B in covering a dis-
Either x = 0, but it is not possible tance of 400 km.
or x – 3 = 0, then x = 3 (ii) If car A uses 4 litres of petrol more than
 Shortest side = 3m car B in covering 400 km, write down an
equation, in x and solve it to determine the
Hypotenuse = 2 × 3 – 1 = 6 – 1 = 5
number of litres of petrol used by car B
Third side = x + 1 = 3 + 1 = 4 for the journey.
Hence sides are 3, 4, 5 (in m) Ans. Sol. Distance travelled by car A in one litre
21. A wire ; 112 cm long is bent to form a = x km
right angled triangle. If the hypotenuse is
and distance travelled by car B in one litre
50 cm long, find the area of the triangle. = (x +5) km
Sol. Perimeter of a right angled triangle = 112 cm (i) Consumption of car A in covering 400 km
Hypotenuse = 50 cm 400
 litres and
Sum of other two sides = 112 – 50 = 62 cm x
Let the length of first side = x 400
consumption of car B  litres.
x5
and length of other side = 62 – x
(ii) According to the condition, we have
According to the condition, we have
400 400
(x)2 + (62 – x)2 = (50)2  4
x x5
(By Pythagorus Theorem)
 x5 x 
 x2 + 3844 – 124x + x2 = 2500  400  x( x  5)   4
 
 2x2 – 124x + 3844 – 2500 = 0
400 ( x  5  x ) 400 × 5
 2x2 – 124x + 1344 = 0  4  4
x ( x  5) x 2  5x
106 Arun Deep's Understanding Math-10
 2000 = 4x2 + 20x  (x + 5) (x – 5) = 0
 4x2 + 20x – 2000 = 0 Either x + 5 = 0, then x = – 5, but it is not
 x2 + 5x – 500 = 0 (Dividing by 4) possible as it is in negative.
 x2 + 25x – 20x – 500 = 0 or x – 5 = 0, then x = 5

 x (x + 25) – 20 (x + 25) = 0 Hence speed of stream = 5 km/hr. Ans.

 (x + 25) (x – 20) = 0 P.Q. By selling an article for Rs. 21, a trader


loses as much percent as the cost price of
Either x + 25 = 0, then x = – 25, but it is the article. Find the cost price.
not possible as it is in negative.
Sol. S.P. of an article = Rs. 21
or x – 20 = 0, then x = 20.
Let cost proce = Rs. x
 Petrol used by car B = 20 – 4
C.P. (100 – Loss%)
= 16 litres. Ans.  S.P. =
100
22. The speed of a boat in still water is 11 km/
hr. It can go 12 km up-stream and return x (100 – x)
21 =
downstream to the original point in 2 hours 100
45 minutes. Find the speed of the stream.
 2100 = 100x – x2
Sol. Speed of boat in still water = 11 km/hr
 x2 – 100x + 2100 = 0
Let the speed of stream = x km/hr.  x2 – 30x – 70x + 2100 = 0
Speed of boat upstream = (11 – x) km/hr  x(x – 30) – 70 (x – 30) = 0
and Speed of boat downstream = (11 + x) km/hr  (x – 30) (x – 70) = 0
Distance covered = 12 km. Either x – 30 = 0, then x = 30
Given total time taken = 2 hours 45 minutes or x – 70 = 0, then x = 70
3 11  Cost price = Rs. 30 or Rs. 70
2  hours.
4 4 23. A man spent Rs. 2800 on buying a number
Now according to the condition, we have of plants priced at Rs x each. Because of
the number involved, the supplier reduced
12 12 11
  the price of each plant by Rupee 1. The
11  x 11  x 4
man finally paid Rs. 2730 and received 10
12 (11  x  11  x ) 11 more plants. Find x.
 
(11  x ) (11  x ) 4 Sol. Amount spent by man = Rs. 2800
Price of each plant = Rs. x
12 × 22 11
   Reduced price = Rs. (x – 1)
121  x 2 4
2800
 1331 – 11x2 = 4 × 12 × 22 = 1056 Thus, no. of plants in first case 
x
 1331 – 11x2 = 1056 No. of plants received in second case
 1331 – 1056 – 11x2 = 0 2800
  10
x
 – 11x2 + 275 = 0 Amount paid = Rs. 2730
 x2 – 25 = 0 (Dividing by –11) According to the given condition, we have
107 Arun Deep's Understanding Math-10
F 2800  10I ( x  1)  2730
H x K
( 2800  10 x ) ( x  1)
  2730  (2800 + 10x) (x – 1) = 2730 x
x
 2800x – 2800 + 10x2 – 10x = 2730x  10x2 + 2800x – 10x – 2730x – 2800 = 0
 2
10x + 60x – 2800 = 0  x2 + 6x – 280 = 0 (Dividing by 10)
 x2 + 20x – 14x – 280 = 0 x (x + 20) – 14 (x + 20) = 0.
 (x + 20) (x – 14) = 0
Either x + 20 = 0, then x = – 20, but it is not possible as it is in negative.
or x – 14 = 0, then x = 14. Ans.
24. Forty years hence, Mr. Partap’s age will be the square of what it was 32 years ago. Find his
present age.
Sol. Let Partap’s present age = x years
40 years hence his age = x + 40
and 32 years ago his age = x – 32
According to the condition, we have
x + 40 = (x – 32)2
 x + 40 = x2 – 64x + 1024 x2 – 64x + 1024 – x – 40 = 0
 x2 – 65x + 984 = 0  x2 – 24x – 41x + 984 = 0
 x(x – 24) – 41 (x – 24) = 0 (x – 24) (x – 41) = 0
Either x – 24 = 0, then x = 24 but it is not possible as it is less than 32
or x – 41 = 0, then x = 41
Hence Pratap's present age = 41 years Ans.
6. Factorisation
EXERCISE 6 Substituting the value of x in f (x) :

1. Find the remainder (without divisions) on (i) Here, f (x) = 4 x2 + 5 x + 3


dividing f (x) by x – 2, where  1  2
(i) f (x) = 5 x2 – 7 x + 4
 f 4  1 FG IJ
5× 
1
3
FG IJ
 2  2 H K 2 H K
(ii) f (x) = 2 x3 – 7 x2 + 3
1 5 5 5 3
Sol. Let x – 2 = 0, then x = 2  4×   3  1   3  4  
4 2 2 2 2
(i) Substituting value of x in
3
f (x) = 5 x2 – 7 x + 4  Remainder  Ans.
2
 f (2) = 5 (2)2 – 7 × 2 + 4
(ii) Here, f (x) = 3 x3 – 7 x2 + 4 x + 11
 f (2) = 20 – 14 + 4 = 10
Hence Remainder = 10 Ans.  1  FG 1 IJ 3  7 FG  1 IJ 2
 f   3 
(ii) f (x) = 2 x3 – 7 x2 + 3  2  H 2K H 2K
 f (2) = 2 (2)3 – 7 (2)2 + 3 = 16 – 28 + 3 F 1I
 4 G  J  11
Hence Remainder = – 9 Ans. H 2K
2. Using remainder theorem, find the F 1I F 1I F 1I
 3 G  J  7 G J  4 G  J  11
remainder on dividing f (x) by (x + 3) where H 8K H 4K H 2K
(i) f (x) = 2 x2 – 5 x + 1 3 7
(ii) f (x) = 3 x3 + 7 x2 – 5 x + 1    2  11
8 4
Sol. Let x + 3 = 0  x = – 3 3  14  16  88 55 7
Substituting the value of x in f (x),    6 Ans.
8 8 8
(i) f (x) = 2 x2 – 5 x + 1 P.Q. (i) Find the remainder when 2 x3 – 3 x2 +
 f (– 3) = 2 (–3)2 – 5 (– 3) + 1 7 x – 8 is divided by x – 1 (2000)
= 18 + 15 + 1 = 34 Ans. (ii) Find the remainder (without division) on
Hence remainder = 34. dividing 3 x2 + 5 x – 9 by (3 x + 2)
(ii) f (x) = 3 x3 + 7 x2 – 5 x + 1 Sol. (i) Let x – 1 = 0, then x = 1
= 3 (– 3)3 + 7 (– 3)2 – 5 (– 3) + 1 Substituting value of x in f (x).
= – 81 + 63 + 15 + 1 = – 2 Ans. Here, f (x) = 2 x3 – 3 x2 + 7 x – 8
Hence remainder = –25.
 f (x) = 2 (1)3 – 3 (1)2 + 7 (1) – 8
3. Find the remainder (without division) on
dividing f (x) by (2 x + 1) where =2×1–3×1+7×1–8=2–3+7–8
(i) f (x) = 4 x2 + 5 x + 3 = – 2 Ans.
(ii) f (x) = 3 x3 – 7 x2 + 4 x + 11 2
1 (ii) Let 3 x + 2 = 0, then 3 x = – 2  x 
Sol. Let 2 x + 1 = 0, then x   3
2
108 Arun Deep's Understanding Math-10
109 Arun Deep's Understanding Math-10
Substituting the value of x in f (x) 2 x2 – 5 x so that the resulting polynomial leaves
the remainder 2, when divided by 2 x + 1?
f (x) = 3 x2 + 5 x – 9
(ii) What number must be added to 2 x3 – 7 x2
2
 2  FG IJ
2 FG 2 IJ  9 + 2 x so that the resulting polynomial leaves
  f    3  5  the remainder – 2 when divided by 2 x – 3?
 3  H K
3 H 3K
Sol. (i) Let a be subtracted from 2 x2 – 5 x,
4 2 4 10
 3×  5×  9   9 Dividing 2 x2 – 5 x – a by 2 x + 1, by long
9 3 3 3 division method, we have
6 2 x + 1 ) 2 x2 – 5 x – a ( x – 3
  9  2  9  11
3 2 x2 + x
 Remainder = –11 Ans. –6x–a
4. Using remainder theorem, find the value of k –6x–3
if on dividing 2x3 + 3x2 – kx + 5 by x – 2, + +
leaves a remainder 7. (2016) –a+3
Sol. f(x) = 2x3 + 3x2 – kx + 5 Here remainder is (3 – a)
g(x) = x – 2, if x – 2 = 0, then x = 2 but we are given that remainder is 2.
 3–a=2
 Dividing f(x) by g(x) the remainder will be  –a=2–3=–1a=1
f(2) = 2(2)3 + 3(2)2 – k × 2 + 5 Hence required number be 1 which is to
= 16 + 12 – 2k + 5 = 33 – 2k be subtracted from given expression. Ans.
(ii) Let a be added to 2 x3 – 7 x2 + 2 x on
Also given remainder = 7
dividing it by 2 x – 3, then
 33 – 2k = 7
2 x – 3 ) 2 x3 – 7 x2 + 2 x + a ( x2 – 2 x – 2
 33 – 7 = 2k 2 x3 – 3 x2
26 – +
 2k = 26  k = = 13 – 4 x2 + 2 x
2
– 4 x2 + 6 x
 k = 13
+ –
5. Using remainder theorem, find the value –4x+a
of a if the division of x3 + 5 x2 – ax + 6 by –4x+6
(x – 1) leaves the remainder 2a. + –
Sol. Let x – 1 = 0, then x = 1 a–6
But remainder is – 2, then
Substituting the value of x in f (x)
a – 6 = – 2  a = – 2 + 6  a = 4
f (x) = x3 + 5 x2 – ax + 6
Hence 4 is to be added. Ans.
 f (1) = (1)3 + 5 (1)2 – a (1) + 6 7.(i) When divided by x – 3 the polynomials x3 –
= 1 + 5 – a + 6 = 12 – a px2 + x + 6 and 2x3 – x2 – (p + 3) x – 6 leave
the same remainder. Find the value of ‘p’.
Also given remainder = 2 a
(2010)
 12 – a = 2a Sol. Let p(x) = x3 – px2 + x + 6
 12 = a + 2a 3a = 12 and q(x) = 2x3 – x2 – (p + 3) x – 6
 a = 4 Ans. On divided p(x) and q(x) and q(x) by x – 3,
6.(i) What number must be subtracted from the remainder is same
110 Arun Deep's Understanding Math-10
Let x – 3 = 0, then x = 3 By remainder theorem,
Now by Remainder Theorem, p(3) = q(3) f(4) = p ....(1)
p (3) = (3)3 – p (3)2 + 3 + 6 x–4=0 x=4
= 27 – 9p + 9 = 36 – 9p and g(4) = q ....(2)
and q (3) = 2 (3)2 – (3)2 – (p + 3) × 3 – 6 x–4=0 x=4
= 2 × 27 – 9 – 3p – 9 – 6 Now, substitute the value of x = 4 in eq. (1)
= 54 – 24 – 3p = 30 – 3p
a(4)3 + 3(4)2 – 3 = p
The remainder in each case is same.
 64a + 48 – 3 = p
 36 – 9p = 30 – 3p
 64a + 45 = p ...(3)
36 – 30 = 9p – 3p
Also, substitute the value of x = 4 in eq. (2)
6
 6 = 6p  p= =1 2(4)3 – 5(4) + a = q
6
 p=1  128 – 20 + a = q
(ii) Find 'a' if the twopolynomials ax3 + 3x2–9  108 + a = q ...(4)
and 2x3 + 4x + a, leaves the same remainder
According to the question,
when divided by x + 3.
2p = q
Sol. The given polynomials are ax3 + 3x2 – 9
 2(64a + 45) = 108 + a
and 2x3 + 4x + a
 128a – a = 108 – 90
Let p(x) = ax3 + 3x2 – 9
 127a = 18
and q(x) = 2x3 + 4x + a
Given that p(x) and q(x) leave the same 18
 a=
remainder when divided by (x + 3). 127
Thus by Remainder Theorem, we have 8. Using the remainder theorem, find the
p(–3) = q(–3) remainders obtained when x3 + (kx + 8) x + k
is divided by x + 1 and x – 2. Hence, find k
 a(–3)3 + 3(–3)2 – 9 = 2(–3)3 + 4(–3) + a
if the sum of the two remainders is 1.
 –27a + 27 – 9 = –54 – 12 + a
Sol. x  1  0  x  1
 –27a + 18 = –66 + a
Remainder when x3 + (kx + 8)x + k is
 –27a – a = –66 – 18 divided by x + 1
 –28a = –84 = The value of x3 + (kx + 8)x + k for x =
84 –1
 a=
28 = (–1)3 + (k × –1 + 8) × –1 + k
 a=3
= –1 + k – 8 + k = 2k – 9
(iii) The polynomials ax 3 + 3x 2 – 3 and
x–2=x=2
2x3 – 5x + a when divided by x – 4 leave
the remainder r1 and r2 respectively. If Remainder when x3 + (kx + 8)x + k is
2r1 = r2, then find the value of a. divided by x – 2
Sol. Let f(x) = ax3 + 3x2 – 3 = The value of x3 + (kx + 8)x + k for x = 2
and g(x) = 2x3 – 5x + a = (2)3 + (k × 2 + 8) × 2 + k
111 Arun Deep's Understanding Math-10
= 8 + 4k + 16 + k = 5k + 24 From above, we see that
Given, the sum of two remainder = 1 (x + 3) (x – 1), i.e., x2 + 2x – 3 is a factor
of p(x)
 (2k – 9) + (5k + 24) = 1
 p(x) is exactly divisible by (x2 + 2x – 3).
 7k = –14 and k = –2 Ans.
11. Show that (x – 2) is a factor of 3 x2 – x – 10
9. By factor theorem, show that (x + 3) and
(2 x – 1) are factors of 2 x2 + 5 x – 3. Hence factorise 3 x2 – x – 10.
Sol. Let x + 3 = 0 then x = – 3 Sol. Let x – 2 = 0, then x = 2
Substituting the value of x in f (x) Let f (x) = 3 x2 – x – 10
f (x) = 2 x2 + 5 x – 3 Substituting the value of x in f (x)
 f (–3) = 2 (– 3)2 + 5 (– 3) –3 f(2) = 3 (2)2 – 2 – 10 = 12 – 2 – 10 = 0
f (2) = 18 – 15 – 3 = 0
Remainder is zero
Remainder = 0, then x + 3 is a factor
 x – 2 is a factor of f (x).
1
Again let 2 x – 1 = 0, then x  On dividing 3 x2 – x – 10 by x – 2, we get
2
Substituting the value of x in f (x),
x  2 ) 3 x2  x  10 ( 3 x  5
f (x) = 2 x2 + 5 x – 3 3 x2  6 x
 
 f
FG 1 IJ  2 FG 1 IJ 2  5 FG 1 IJ  3 5 x  10
H 2K H 2K H 2K 5 x  10
 
1 5 1 5 ×
 2×  3   30
4 2 2 2  3 x2 – x – 10 = (x – 2) (3 x + 5) Ans.
Remainder = 0,
12. Using the factor theorem, show that
 2 x – 1 is also a factor of f (x) (x – 2) is a factor of x3 + x2 – 4x – 4.
Hence proved. Hence factorise the polynomial
10. Without actual division, prove that x4 + 2x3 completely.
– 2x2 – 3 is exactly divisible by x2 + 2x – 3.
Sol. Let f ( x )  x 3  x 2  4 x  4
Sol. x2 + 2x – 3 = x2 + 3x – x – 3
and x  2  0  x  2
= x(x + 3) – 1(x + 3) = (x + 3) (x – 1)
 f (2)  23  22  4 (2)  4  8  4  8  4  0
Let p(x) = x4 + 2x3 – 2x2 + 2x – 3
 (x – 2) is a factor of f (x).
We see that
Now dividing f (x) by x – 2, we get
p(–3) = (–3)4 + 2(–3)3 – 2(–3)2 + 2(–3) –3
= 81 – 54 – 18 – 6 – 3 = 0 x – 2 ) x3 + x2 – 4x – 4 (x2 + 3x + 2
Hence by converse of factor theorem,  x3  2 x 2
(x + 3) is a factor of p(x) 3x 2  4 x
Also, we see that 3x 2  6 x
p(1) = (1)4 + 2(1)3 – 2(1)2 + 2(1) – 3 = 0 2x  4
Hence by converse of factor theorem, 2 x  4
(x – 1) is a factor of p(x). 0
112 Arun Deep's Understanding Math-10
 f(x) = (x – 2) (x2 + 3x + 2) expression completely, using the factor
= (x – 2) (x2 + 2x + x + 2) theorem. (2006)
= (x – 2) [x (x + 2) + 1 (x + 2)] Sol. Let 2x + 7 = 0, then 2x = –7
 f(x) = (x – 2) (x + 2) (x + 1)
P.Q. Show that (2 x + 1) is a factor of 7
x=
4 x3 + 12 x2 + 11 x + 3 2
Hence factorise 4 x3 + 12 x2 + 11 x + 3. Subsitituting the value of x in f (x),
1 Here, f (x) = 2x3 + 5x2 – 11x – 14
Sol. Let 2 x + 1 = 0, then x  
2
3 2
Substituting the value of x in f (x),  7  7  7
Here, f (x) = 4 x3 + 12 x2 + 11 x + 3 f    = 2    + 5   
 2  2  2

 f
FG  1 IJ  4 FG  1 IJ 3  12 FG  1 IJ 2  7
H 2K H 2K H 2K  111     14
 2
F 1I
 11 G  J  3
H 2K =
 343 245 77
   14
F 1I F 1I F 1I
 4 G  J  12 G J  11 G  J  3 4 4 2
H 8K H 4K H 2K
1 11  343 245154 56  399 399
  3  3 = (6) – (6) = 0 = = =0
2 2 4 4
... Remainder = 0
Hence, (2x + 7) is a factor of f (x).
 2 x + 1 is a factor of Proved.
4 x3 + 12 x2 + 11 x + 3 Now, 2x + 5x2 – 11x – 14 = (2x + 7) (x2 – x – 2)
3
Now dividing f (x) by 2 x + 1, we get
= (2x + 7) [x2 – 2x + x – 2]
= (2x + 7) [x (x – 2) + 1 (x – 2)]
2 x  1 ) 4 x3  12 x2  11 x  3 ( 2 x2  5 x  3
4 x 3  2 x2 = (2x + 7) (x + 1) (x – 2) Ans.
  2x + 7) 2x3 + 5x2 – 11x – 14 (x2 – x – 2
10 x2  11 x
10 x2  5 x 2x3 + 7x2
  – –
6x  3
6x  3 – 2x2 – 11x
  – 2x2 – 7x
×
+ +
– 4x – 14
 4 x3 + 12 x2 + 11 x + 3
= (2 x + 1) (2 x2 + 5 x + 3) – 4x – 14
= (2 x + 1) [2 x2 + 2 x + 3 x + 3] + +
= (2 x + 1) [2 x (x + 1) + 3 (x + 1)] ×
= (2 x + 1) [(x + 1) (2 x + 3)] 14. (i) Use factor theorem to factorise the following
polynominals completely.
= (2 x + 1) (x + 1) (2 x + 3) Ans.
(i) x3 + 2x2 – 5x – 6 (ii) x3 – 13x – 12.
13. Show that 2x + 7 is a factor of 2x 3 +
5x2  111x  14. Hence factorise the given
113 Arun Deep's Understanding Math-10
Sol. (i) Let f (x) = x3 + 2x2 – 5x – 6 x–4 ) x3
– 13x –12 (x2 + 4x + 3
Factors of 6 are (...6 =  1 ;  2,  3,  6) x3 – 4x2
Let x = – 1, then – +
f (– 1) = (– 1)3 + 2 (– 1)2 – 5 (– 1) – 6 4x2 – 13x
4x2 – 16x
= – 1 + 2 ( 1) + 5 – 6
– +
=–1+2+5–6=7–7=0 3x – 12
... f (– 1) = 0, 3x – 12
 x + 1 is a factor of f (x) Ans. – +
×
Now, dividing f (x) by x + 1, we get
15. Use Remainder Theorem to factories the
f (x) = (x + 1) (x2 + x – 6) following polynomials completely :
= (x + 1) (x2 + 3x – 2x – 6) (i) 2x3 + x2 – 13x + 6 (ii) 3x3 + 2x3 – 19x + 6
= (x + 1) {x (x + 3) – 2 (x + 3)} (iii) 2x3 + 3x2 – 9x – 10 (iv) x3 + 10x2 – 37x + 26
= (x + 1) (x + 3) (x – 2) Ans. Sol. (i) Let f (x) = 2x3 + x2 – 13x + 6
Factors of 6 are +1, +2, +3, +6
x + 1 ) x 3  2 x 2  5x  6 ( x2 + x – 6 Let x = 2, then
x3  x2 f (2) = 2 (2)3 + (2)2 – 13 × 2 + 6
= 16 + 4 – 26 + 6 = 26 – 26 = 0
– –
f (2) = 0
x 2  5x  x – 2 is the factor of f (x)
x2  x (By Remainder Theorem)
Dividing f (x) by x – 2, we get
– –
x 2) 2x + x 13x + 6 (2x + 5x 3
3 2 2

 6x  6
2x  4x
3 2
 6x  6
 +
5x  13x
2
+ +
5x  10x
2

×  +
x3
(ii) f (x) = – 13x – 12 3x + 6
Factors of 12 are ±1, ±2, ±3, ±4, ±6, 3x + 6
+ 
and ±12 ×
Let x = 4, then
f (x) = (4)3 – 13(4) – 12  f (x) = (x – 2) (2x2 + 5x – 3)
= 64 – 52 – 12 = 64 – 64 = 0 = (x – 2) {2x2 + 6x – x – 3}
f (x) = 0 = (x – 2) {2x (x + 3) – 1 (x + 3)}
= (x – 2) (x + 3) (2x – 1)
 x – 4 is a factor of f (x)
(ii) P(x) = 3x3 + 2x2 – 19x + 6
Now, dividing f (x) by (x – 4), we get,
P(1) = 3 + 2 – 19 + 6 = –8  0
f (x) = (x – 4) (x2 + 4x + 3)
P(–1) = –3 + 2 + 19 + 6 = –24  0
= (x – 4) (x2 + 3x + x + 3) P(2) = 24 + 8 – 38 + 6 = 0
= (x – 4) [x (x + 3) + 1 (x + 3)] Hence, (x – 2) is a factor of P(x)
= (x – 4) (x + 3) (x + 1) Ans.  P(x) = 3x3 + 2x2 – 19x + 6
114 Arun Deep's Understanding Math-10
2 (x – 1) is factor of f (x)
= 3x (x – 2) + 8x (x – 2) – 3 (x – 2)
= (x – 2) (3x2 + 8x – 3) 2
 f (x) = (x – 1) (x + 11x – 26)
= (x – 2) (3x2 + 9x – x – 3) = (x – 1) (x2 + 13x – 2x – 26)
= (x – 2) {3x (x + 3) – 1 (x + 3)
= (x – 1) [x(x + 13) – 2(x + 13)]
= (x – 2) (x + 3) (3x – 1)
(iii) f(x) = 2x3 + 3x2 – 9x – 10 = (x – 1) [(x – 2) (x + 13)]
f(–1) = 2(–1)3 + 3(–1)2 – 9(–1) –10 16. If (2 x + 1) is a factor of 6x3 + 5x2 + ax – 2
 f(–1) = –2 + 3 + 9 – 10 = 0 find the value of a.
 (x + 1) is a factor. 1
Sol. Let 2x + 1 = 0, then x  
2
2x2 + x – 10
Substituting the value of x in f (x),
x 1 ) 2x 3 + 3x2 – 9x – 10
2x 3 + 2x 2 f (x) = 6x3 + 5x2 + ax – 2
 – FG 1 IJ  6 FG  1 IJ 3  5 FG  1 IJ 2
f 
x 2 – 9x H 2K H 2K H 2K
x2+ x F 1I
a G J  2
– –
F 1I F 1I F 1I
H 2K
–10 x – 10  6 G J  5 G J aG J  2
–10 x – 10
H 8K H 4K H 2K
 3 5 a
+    2
0 0 4 4 2
3  5  2 a  8 6  2 a
 
 2x2 + x – 10 = 2x2 + 5x – 4x – 10 4 4
. . . 2 x + 1 is a factor of f (x)
= x(2x + 5) – 2(2x + 5) = (2x + 5) (x – 2)
 Remainder = 0
 factors are (x + 1) (x – 2) (2x + 5) Ans.
6  2 a
(iv) Use the Remainder and Factor Theorem,  0–6–2a=0
4
factorise the following polynomials :
 2a=–6a=–3
x3 + 10x2 – 37x + 26.
 a = –3
Sol. f (x) = x3 + 10x2 – 37x + 26
17. If (3 x – 2) is a factor of 3 x3 – kx2 + 21 x
f (1) = (1)3 + 10(1)2 – 37(1) + 26 – 10, find the value of k.
= 1 + 10 – 37 + 26 = 0
2
 x = 1 be the root of f(x) = 0 Sol. Let 3 x – 2 = 0, then 3 x = 2  x 
3
Then x – 1 is factor of f (x) Substituting the value of x in f (x),
f (x) = 3x3 – kx2 + 21 x – 10
x  1)x3 + 10x2  37x + 26(x2 + 11x  26
x3  x2
Here, f
FG 2 IJ  3 FG 2 IJ 3  k FG 2 IJ 2  21 FG 23 IJ  10
 + H 3K H 3K H 3K H K
11x2  37x 8 4 2
 3×  k ×  21×  10
11x2  11x 27 9 3
 + 8 4k 8 4k
   14  10  4
 26x + 26 9 9 9
. . . Remainder is 0
 26x + 26
+  8 4k
 4 =0
0 9
115 Arun Deep's Understanding Math-10
 8 – 4 k + 36 = 0  K = –4
 –4 k + 44 = 0  4 k = 44 Now, substituting = –4 in equation (1), we
 k = 11 Ans. get
18. If x – 2 is a factor of 2x3 – x2 + px – 2, (–4 + 2)x2 – (–4)x + 6 = 0
then (i) find the value of p. (ii) With this  –2x2 + 4x + 6 = 0
value of p, factorise the above expression
 x2 – 2x – 3 = 0 (Dividing by 2)
completely. 2
 x – 3x + x – 3 = 0
Sol. (i) Let x – 2 = 0, then x = 2
 x(x – 3) + 1(x – 3) = 0
Now f (x) = 2x3 – x2 + px – 2
 f (2) = 2(2)3 – (2)2 + p × 2 – 2  (x + 1) (x – 3) = 0
= 2 × 8 – 4 + 2p – 2 = 16 – 4 + 2p – 2 So, the roots are x = –1 and x = 3
= 10 + 2p Thus, the other root of the equation is x =
f (2) = 0, then 10 + 2p = 0 –1
 2p = – 10 p = – 5 19. What number should be subtracted from
Now, the polynomial will be 2x 3 – 5x 2 + 5x so that the resulting
2x3 – x2 – 5x – 2 and (x – 2) be its factor polynomial has 2x – 3 as a factor?
 f (x) = (x – 2) (2x2 + 3x + 1] Sol. Let the number to be subtracted be k and
the resulting polynomial be f (x), then
= (x – 2) (2x2 + 2x + x + 1]
= (x – 2) [2x (x + 1) + 1 (x + 1)] f (x) = 2x3 – 5x2 + 5x – k
= (x – 2) (x + 1) (2x + 1) Ans. Since, 2x – 3 is a factor of f (x),
Then by fator theorem ; we have
x – 2 ) 2 x 3  x 2  5x  2 ( 2x2 + 3x + 1
2x3  4x2 3
– + f  = 0
2
3x 2  5x 3 2
3x 2  6 x 3 3 3
 2  – 5  + 5  – k = 0
– + 2 2 2
x2
x2 27 9 3
 2× –5× +5× –k=0
– + 8 4 2
×
27 45 15
P.Q. Find the value of 'K' for which x = 3 is a  – + –k=0
solution of the quadratic equation, (K + 2) 4 4 2
x2 – Kx + 6 = 0.  27 – 45 + 30 – 4k = 0  –4k + 12 = 0
Sol. Given eqn. be,
12
(K + 2) x2 – Kx + 6 = 0 ....(1)  k=
Substitute x = 3 in equation (1) ; we have 4
(K + 2) (3)2 – K(3) + 6 = 0  k=3
(–4 + 2)x2 – (–4)x + 6 = 0 20. (i) Find the value of the constants a and b,
 9(K + 2) – 3K + 6 = 0 if (x – 2) and (x + 3) are both factors of
 9K + 18 – 3K + 6 = 0 the expression x3 + ax2 + bx – 12.
 6K + 24 = 0  6K = –24 Sol. Let x – 2 = 0, then x = 0
24 Substituting value of x in f (x)
 K= where f (x) = x3 + ax2 + bx – 12
6
116 Arun Deep's Understanding Math-10
 f (2) = (2)3 +a (2)2 + b (2) – 12  –2(–19) + b = 8  38 + b = 8
= 8 + 4a + 2b – 12 = 4a + 2b – 4  b = –30 Ans.
x – 2 is a factor of f(x) 21. If (x + 2) and (x – 3) are factors of x3 +
 4a + 2b – 4 = 0  4a + 2b = 4 ax + b, find the values of a and b. With
 2a + b = 2 ...(i) these values of a and b, factorise the given
expression.
Again let x + 3 = 0, then x = – 3
Sol. Let x + 2 = 0, then x = – 2
Substituting the value of x in f (x)
Substituting the value of x in f (x),
f (x) = x3 + ax2 + bx – 12
f (x) = x3 + ax + b
 Here, f(–3) = (–3)2 + a (–3)2 + b(– 3) – 12
Here, f (– 2) = (– 2)3 + a (– 2) + b
= –27 + 9 a – 3 b – 12 = – 39 + 9 a – 3 b
. . . x + 3 is a factor of f (x) =–8–2a+b
x + 2 is a factor of f(x).
 – 39 + 9 a – 3 b = 0  9 a – 3 b = 39
 Remainder is zero.
 3 a – b = 13 ...(ii)
 – 8 – 2 a + b = 0 – 2 a + b = 8
Adding (i) and (ii) ; we get
5 a = 15  a = 3  2a–b=–8 ...(i)
Substituting the value of a in (i) ; we have Again let x – 3 = 0, then x = 3,
2 (3) + b = 2  6 + b = 2 Substituting the value of x in f (x),
 b=2–6 where f (x) = x3 + ax + b
 b=–4  f (3) = (3)3 + a (3) + b = 27 + 3 a + b
Hence a = 3, b = – 4 Ans. x – 3 is a factor of f (x)  Remainder = 0
(ii) If (x + 2) and (x – 3) are factors of x3 + ax  27 + 3 a + b = 0
+ b, find the values of 'a' and 'b'.  3 a + b = – 27 ...(ii)
Sol. Since (x + 2) = 0  x = –2 Adding (i) and (ii) ; we get
3
f (x) = x + ax + b 35
5 a = – 35  a   7
 f (–2) = (–2)3 + a (–2) + b 5
Since x + 2 is a factor, by factor theorem Substituting value of a in (i) ; we get
–8 – 2a + b = 0 2 (– 7) – b = – 8  – 14 – b = – 8
 –2a + b = 8 ...(1) – b = – 8 + 14  – b = 6  b = – 6
Since x + 3 = 0 Hence a = – 7, b = – 6
Thus, (x + 2) and (x – 3) are the factors of
 x = –3
x3 + ax + b i.e. x3 – 7 x – 6
f (x) = x3 + ax + b
Now dividing x3 – 7 x – 6 by (x + 2)
 f (–3) = (–3)3 + a (–3) + b
(x – 3) or x2 – x – 6, we get
 f(–3) = –27 – 3a + b
By factor theorem, –27 – 3a + b = 0 x2  x  6 ) x3  7 x  6 ( x  1
 –3a + b = 27 ...(2) x3  x 2  6 x
Subtracting eq. (2) from eq. (1)   
x2  x  6
–2a + b = 8
x2  x  6
–3a + b = 27   
(+) (–) (–) ×
a = –19  Factors are f(x) are (x + 2), (x – 3) and
Now, substituting the value of a in eq. (1) (x + 1) Ans.
117 Arun Deep's Understanding Math-10
22. (x  2) is a factor of the expression Also, (x – 3) it leaves remainder = 52
x3 + ax2 + bx + 6. When this expression is  f (3) = 52
divided by (x  3), it leaves the remainder 2(3)3 + a(3)2 + b(3) – 14 = 52
3. Find the values of a and b. (2005)
54 + 9a + 3b – 14 = 52  9a + 3b = 52 – 40
Sol. As x  2 is a factor of
9a + 3b = 12
f (x) = x3 + ax2 + bx + 6
 3a + b = 4 ...(ii)
Then by factor theorem
From (i) and (ii) ; we have
 f (2) = 0
3 2 2a + b = –1
 (2) + a (2) + b (2) + 6 = 0
3a + b = 4
 8 + 4a + 2b + 6 = 0
– – –
 4a + 2b =  14
on subtracting ; –a = –5
 2a + b =  7 ....(i)
 a=5  from (i) ; we have
as on dividing f (x) by x  3
 2(5) + b = – 1
remainder = 3
 b = –1 – 10 b = –11
 f (3) = 3
3 2  a = 5, b = –11
 (3) + a (3) + b (3) + 6 = 3
 27 + 9a + 3b + 6 = 3 24. If ax3 + 3 x2 + bx – 3 has a factor (2x + 3)
and leaves remainder –3 when divided by
 9a + 3b =  30 (x + 2), find the values of a and b. With these
 3a + b =  10 ....(ii) values of a and b, factorise the given expression.
Solving simultaneously equation (i) and (ii), 3
Sol. Let 2 x + 3 = 0 then 2 x = – 3  x  2
 2a + b =  7
3a + b =  10 Substituting the value of x in f (x),
On subtracting, _ _ + f (x) = ax3 + 3 x2 + bx – 3
a = 3
 f
FG 3 IJ  a FG 3 IJ 3  3 FG 3 IJ 2 b FG 3 IJ  3
Substituting value of a in equation (i) ; H2K H2K H2K H2K
we get F 27 IJ  3 FG 9 IJ  b FG 3 IJ  3
2(3) + b = 7 a G
H 8 K H 4K H 2 K
 6 + b = 7 27 a 27 3 b
   3
 b = 1 8 4 2
 a =  3, b =  1 Ans. 2 x + 3 is a factor of f (x)
23. If (x – 2) is a factor of the expression  Remainder = 0
2x3 + ax2 + bx – 14 and when the expression is
27 a 27 3 b
divided by (x – 3), it leaves a remainder 52,    30
find the values of a and b. 8 4 2
Sol. f (x) = 2x3 + ax2 + bx – 14  – 27 a + 54 – 12 b – 24 = 0
(x – 2) is factor of f (x) (Multiplying by 8)
 f (2) = 0  – 27 a – 12 b + 30 = 0
2(2)3 + a(2)2 + b(2) – 14 = 0  – 27 a – 12 b = – 30
 16 + 4a + 2b – 14 = 0 4a + 2b = –2  9 a + 4 b = 10 [Dividing by (–3)]
2a + b = –1 ...(i) 9a + 4b = 10 ...(i)
118 Arun Deep's Understanding Math-10
Again let x + 2 = 0 then x = – 2 f (x) + g (x) + 4x2 + 7x.
Substituting the value of x in f (x) Sol. f (x) = ax2 + bx + 2
f (x) = ax3 + 3 x2 + bx – 3 g (x) = bx2 + ax + 1
x – 2 is a factor of f (x)
f (– 2) = a (– 2)3 + 3 (– 2)2 + b (– 2) – 3
Let x – 2 = 0  x = 2
= – 8 a + 12 – 2 b – 3 = – 8 a – 2 b + 9  f (2) = a (2)2 + b × 2 + 2 = 4a + 2b + 2
Remainder = – 3  4a + 2b + 2 = 0 ( x – 2 is its factor)
 –8a–2b+9=–3  2a + b + 1 = 0 ...(i) (Dividing by 2)
 –8a–2b=–3–9 Dividing g (x) by x – 2, remainder = –15
 – 8 a – 2 b = – 12 (Dividing by 2) Let x – 2 = 0  x = 2
 g (2) = b (2)2 + a × 2 + 1
 4a+b=6 ...(ii)
= 4b + 2a + 1
Multiplying (ii) by 4, we get Remainder is –15.
16 a + 4 b = 24  4b + 2a + 1 = –15  4b + 2a + 1 + 15 = 0
9 a + 4 b = 10  4b + 2a + 16 = 0  2b + a + 8 = 0
– – –
(Dividing by 2)
On substracting, 7 a = 14
 a + 2b + 8 = 0 ....(ii)
14
7 a = 14  a   2. Multiplying (i) by 2 and (ii) by 1
7
Substituting the value of a in (i) ; we get 4a + 2b + 2 = 0
9 (2) + 4 b = 10  18 + 4 b = 10 a + 2b + 8 = 0
 4 b = 10 – 18  4 b = – 8  6
8 3a 6 = 0  3a = 6  a =
 b  2 3
4
Hence a = 2, b = – 2  a=2
Substituting the value of a in (i) ; we get
 f (x) = ax3 + 3x2 + bx – 3
2×2+b+1=04+b+1=0
= 2 x3 + 3 x2 – 2 x – 3
 b + 5 = 0  b = –5
2 x + 3 is a factor of (x) Hence a = 2, b = –5
 On dividing f (x) by x + 2 ; we have Now f (x) + g (x) + 4x2 + 7x
2 x  3 ) 2 x3  3 x2  2 x  3 ( x2  1 = 2x2 – 5x + 2 + (–5x2 + 2x + 1) + 4x2 + 7x
2 x3  3 x 2 = 2x2 – 5x + 2 – 5x2 + 2x + 1 + 4x2 + 7x
  = 6x2 – 5x2 – 5x + 2x + 7x + 2 + 1
2x3
2x3 = x2 + 4x + 3
  = x2 + x + 3x + 3
× = x (x + 1) + 3 (x + 1) = (x + 1)(x +3)
 2 x3 + 3 x2 – 2 x – 3 = (2 x + 3) (x2 – 1)
= (2 x + 3) [(x2) – (1)2] MULTIPLE CHOICE QUESTIONS

= (2 x + 3) (x + 1) (x – 1) Ans. Choose the correct answer from the given


25. Given f (x) = ax2 + bx + 2 and g (x) = bx2 + ax + four options (1 to 5) :
1. If x – 2 is a factor of f (x) but leaves 1. When 2x3 – x2 + 3x + 5 is divided by 2x + 1,
the remainder –15 when it divides g (x), find then the remainder is
the values of a and b. With these values of a (a) 6 (b) –6 (c) –3 (d) 0
and b, factorise the expression. Sol. f (x) = 2x – x3 – 3x + 5
3
119 Arun Deep's Understanding Math-10
g(x) = 2x + 1  6k – 16 = k – 1
1  6k – k = –1 + 16  5k = 15
Let 2x + 1 = 0, then x =
2 15
 k= =3
Then remainder will be 5
3 2  k=3 (d)
 1  1  1  1
f   = 2  –   – 3  + 5 4. 3 2
If x + 1 is a factor of 3x + kx + 7x + 4,
 
2  2   2   2  then the value of k is
1 1 3 (a) –1 (b) 0
=2× – + +5
8 4 2 (c) 6 (d) 10
1 1 3 Sol. f(x) = 3x3 + kx2 + 7x + 4
= – + +5 g(x) = x + 1
4 4 2
1  1  6  20 24 Remainder = 0
= = =6 Let x + 1 = 0, then x = –1
4 4
 Remainder = 6 (a) f(–1) = 3(–1)3 + k(–1)2 + 7(–1) + 4
2. 2
If on dividing 4x – 3kx + 5 by x + 2, the = –3 + k – 7 + 4 = k – 6
remainder is –3 then the value of k is  Remainder = 0
(a) 4 (b) –4 (c) 3 (d) –3  k–6=0k=6 (c)
2 5. 3 2
When x – 3x + 5x – 7 is divided by x – 2,
Sol. f(x) = 4x – 3kx + 5
g(x) = x + 2 then the remainder is
Remainder = –3 (a) 0 (b) 1 (c) 2 (d) –1
Sol. 3 2
f(x) = x – 3x + 5x – 7
Let x + 2 = 0, then x = –2
Now remainder will be g(x) = x – 2, if x – 2 = 0, then x = 2
f(–2) = 4(–2)2 – 3k(–2) + 5 Remainder will be
= 16 + 6k + 5 = 21 + 6k  f(2) = (2)3 – 3(2)2 + 5 × 2 – 7
 21 + 6k = –3  6k = –3 – 21 = –24 = 8 – 12 + 10 – 7 = 18 – 19 = – 1
24  Remainder = –1 (d)
 k= = –4 CHAPTER TEST
6
 k = –4 (b) 1. Find the remainder when 2 x3–3 x2 +4 x+ 7
3 2 is divided by
3. If on dividing 2x + 6x – (2k – 7)x + 5 by
x + 3, the remainder is k – 1 then the value (i) x – 2 (ii) x + 3 (iii) 2 x + 1
of k is Sol. f (x) = 2 x3 – 3 x2 + 4 x + 7
(a) 2 (b) –2 (i) Let x – 2 = 0, then x = 2
(c) –3 (d) 3 Substituting value of x in f (x) ; we get
Sol. f(x) = 2x3 + 6x2 – (2k – 7)x + 5 f (2) = 2 (2)3 – 3 (2)2 + 4 (2) + 7
g(x) = x + 3 =2×8–3×4+4×2+7
Given, remainder = k – 1 = 16 – 12 + 8 + 7 = 19
If x + 3 = 0, then x = –3  Remainder = 19 Ans.
 Given, remainder will be (ii) Let x + 3 = 0, then x = – 3
f(–3) = 2(–3)3 + 6(–3)2 – (2k – 7) (–3) + 5 Substituting the value of x in f (x) ; we get
= –54 + 54 + 3(2k – 7) + 5 f (– 3) = 2 (– 3)3 – 3 (– 3)2 + 4 (– 3) + 7
= –54 + 54 + 6k – 21 + 5 = 6k – 16 = 2 × (– 27) – 3 (9) + 4 (– 3) + 7
120 Arun Deep's Understanding Math-10
= – 54 – 27 – 12 + 7 = – 93 + 7 = – 86 Now f (x) will be 6 x2 + x – 15
 Remainder = – 86 Ans. Dividing 6 x2 + x – 15 by 2 x – 3, we get
(iii) Let 2 x + 1 = 0, then 2 x = – 1  x  
1 2 x  3 ) 6 x2  x  15 ( 3 x  5
2 6 x2  9 x
Now substituting the value of x in f (x)  
10 x  15
FG 1 IJ  2 FG  1 IJ 3  3 FG  1 IJ 2  4 FG  1 IJ  7 10 x  15
f   
H 2 K H 2 K H 2 K H 2K ×
F 1 I F 1 I F 1I  6 x2 + x – 15 = (2 x – 3) (3 x + 5) Ans.
 2 G  J  3G J  4 G  J  7 4. When 3x2 – 5x + p is divided by (x – 2), the
H 8 K H 4 K H 2K remainder is 3. Find the value of p. Also
1 3 factorise the polynomial 3x2 – 5x + p – 3.
=  27 =–1–2+7=4
4 4 Sol. f (x) = 3x2 – 5x + p
 Remainder = 4 Ans. Let (x – 2) = 0, then x = 2
2. When 2 x3 – 9 x2 + 10 x – p is divided by  f (2) = 3 (2)2 – 5(2) + p
(x + 1), the remainder is – 24. Find the = 3 × 4 – 10 + p = 12 – 10 + p = 2 + p
. . . Remainder = 3
value of p.
 2 + p = 3  p = 3 – 2 = 1
Sol. Let x + 1 = 0 then x = – 1,
Hence p = 1
Substituting the value of x in f (x)
Now f (x) = 3x2 – 5x + p – 3
f (x) = 2 x3 – 9 x2 + 10 x – p = 3x2 – 5x + 1 – 3 = 3x2 – 5x – 2
Here, f (– 1) = 2 (– 1)3 – 9 (– 1)2 + 10 (– On dividing g(x) by (x – 2), we get
1) – p
x – 2 ) 3 x 2  5 x  2 ( 3x + 1
= – 2 – 9 – 10 – p = – 21 – p
3x 2  6 x
the remainder = – 24 – +
 – 21 – p = – 24  – p = – 24 + 21 = – 3
x2
 p = 3 Ans. x2
3. If (2 x – 3) is a factor of 6 x2 + x + a, find – +
the value of a. With this value of a, ×
factorise the given expression. 3x2 – 5x – 2 = (x – 2) (3x + 1) Ans.
3 5. Prove that (5x + 4) is a factor of
Sol. Let 2 x – 3 = 0 then 2 x = 3  x 
2 5x3 + 4x2 – 5x – 4. Hence factorise the
Substituting the value of x in f (x) given polynomial completely.
f (x) = 6 x2 + x + a Sol. f (x) = 5x3 + 4x2 – 5x – 4
4
Here, f
FG 3 IJ  6 FG 3 IJ 2  3  a Let 5x + 4 = 0, then 5x = – 4  x 
5
H 2 K H 2K 2
9 3
 6×   a  f
F 4I  5 F 4I 3
4 2 H 5K H 5K
27 3 30
  a   a  15  a 4 2
 4 F I  5 F I  4
4
2 2 2
 2 x – 3 is the factor
H 5K H 5K
 .5  F 
 Remainder = 0 64 I 16
 15 + a = 0  a = – 15 H 125K 25  4  4
4
121 Arun Deep's Understanding Math-10
64 64 = – 8 + 38 – 30 = 38 – 38 = 0
  44 0
25 25  (x + 2) is a factor of f (x)
f F I  0
...
4 Now dividing f (x) by (x + 2), we get
H 5K f (x) = x3 – 19x – 30
 (5x + 4) is a factor of f (x).
= (x + 2) (x2 – 2x – 15)
Now dividing f (x) by 5x + 4, we get
5x3 + 4x2 – 5x – 4 = (x + 2) {(x2 – 5x + 3x – 15}
= (5x + 4) (x2 – 1) = (x + 2) {x (x– 5) + 3 (x – 5)}
= (5x + 4) {(x)2 – (1)2} = (x + 2) (x – 5) (x + 3) Ans.
= (5x + 4) (x + 1) (x – 1) Ans. x + 2 ) x 3  19 x  30 ( x2 – 2x – 15
5x + 4 ) 5 x 3  4 x 2  5 x  4 (x2 – 1 x3  2x2
5x 3  4 x 2 – –
– –  2 x 2  19 x
5 x  4  2x2  4x
5 x  4 + +
+ + 15 x  30
× 15 x  30
+ +
6. Use factor theorem to factorise the
following polynomials completely : ×
(i) 4x3 + 4x2 – 9x – 9 (ii) x3 – 19x – 30 7. If x3 – 2 x2 + px + q has factor (x + 2) and
leaves a remainder 9 when divided by
Sol. (i) f (x) = 4x3 + 4x2 – 9x – 9
(x + 1). Find the values of p and q. With
Let x = – 1, then
these values of p and q factorise the given
f (– 1) = 4 (– 1)3 + 4 (– 1)2 – 9 (– 1) – 9
polynomial completely.
= 4 (– 1) + 4 (1) + 9 – 9 Sol. f (x) = x3 – 2x2 + px + q
= – 4 + 4 + 9 – 9 = 13 – 13 = 0 (x + 2) is a factor of f(x)
 (x + 1) is a factor of f (x)  f (– 2) = (– 2)3 – 2 (– 2)2 + p (– 2) + q
Now dividing f (x) by x + 1, we get = – 8 – 2 × 4 – 2p + q = – 8 – 8 – 2p + q
f (x) = 4x3 + 4x2 – 9x – 9 = 16 – 2 p + q
= (x + 1) (4x2 – 9) x + 2 is a factor of f (x)
= (x + 1) {(2x)2 – (3)2}
 f (– 2) = 0
= (x + 1) (2x + 3) (2x – 3) Ans.
 – 16 – 2 p + q = 0
x + 1 ) 4 x 3  4 x 2  9 x  9 (4x2 – 9  2p – q = – 16 ...(i)
4x3  4x2 Again let x + 1 = 0, then x = – 1
– –  f (– 1) = (– 1)3 – 2 (– 1)2 + p (– 1) + q
 9x  9 =–1–2–p+q
 9x  9 =–1–2–p+q=–3–p+q
+ + Remainder = 9, then
× –3–p+q=9
(ii) f (x) = x3 – 19x – 30  – p + q = 9 + 3 = 12
Let x = – 2, then – p + q = 12 ...(ii)
f (– 2) = (– 2)3 – 19 (– 2) – 30 Addition (i) and (ii) ; we get
122 Arun Deep's Understanding Math-10
p=–4 16 a – 4 b + 88 = 0  16 a – 4 b = – 88
Substituting the value of p in (i) ; we get  4 a – b = – 22 ...(ii)
– (– 4) + q = 12 Adding (i) and (ii), we get
4 + q = 12  q = 12 – 4 = 8 7 a = – 21, a = – 3
 p = – 4, q = 8 Substituting the value of a in (i) ; we have
 Dividing f (x) by x + 2, we get 3 (–3) + b = 1  – 9 + b = 1
f(x) = (x + 2) (x2 – 4 x + 4)  b = 1 + 9 = 10
= (x + 2) {(x)2 – 2 × x (–2) + (2)2}  a = – 3, b = 10
= (x + 2) (x – 2)2 Ans. Now f (x) will be
x2 ) x3  2 x2  4 x  8 ( x2  4 x  4 f (x) = x3 – 3 x2– 10 x + 24
x3  2 x2 . . . x + 3 and x – 4 are factors of f (x)

   Dividing f (x) by (x + 3) (x – 4)
 4 x2  4 x or x2 – x – 12
 4 x2  8 x
  x 2  x  12 ) x 3  3 x 2  10 x  24 ( x  2
4x 8 x 3  x 2  12 x
4x 8   
   2 x 2  2 x  24
×  2 x 2  2 x  24
8. If (x + 3) and (x – 4) are factors of   
x3 + ax2 – bx + 24, find the values of a and ×
b. With these values of a and b, factorise
the given expression.  x3 – 3 x2 – 10 x + 24
Sol. f (x) = x3 + ax2 – bx + 24 = (x2 – x – 12) (x – 2) = (x + 3) (x – 4) (x – 2)Ans.
Let x + 3 = 0, then x = – 3 P.Q. If 2 x3 + a2 – 11 x + b leaves remainder 0
and 42 when divided by (x – 2) and (x – 3)
Substituting the value of x in f (x)
respectively, find the values of a and b.
f (– 3) = (– 3)3 + a (– 3)2 – b (– 3) + 24, With these values of a and b, factorise the
= – 27 + 9 a + 3 b + 24 = 9 a + 3 b – 3 given expression.
... x + 3 is a factor, of f(x) Sol. f (x) = 2 x3 + ax2 – 11 x + b
 Remainder = 0, Let x – 2 = 0, then x = 2,
Substituting the vaue of x in f (x), we have
 9a+3b–3=03a+b–1=0
f (2) = 2 (2)3 + a (2)2 – 11 (2) + b
 3a+b=1 ...(i)
= 2 × 8 + 4 a – 22 + b
Again Let x – 4 = 0, then x = 4
= 16 + 4 a – 22 + b = 4 a + b – 6
Substituting the value of x in f (x) ; we get ... Remainder = 0,
f (x) = (4)3 + a (4)2 – b (4) + 24
 4a+b–6=04a+b=6 ...(i)
= 64 + 16 a – 4 b + 24 = 16 a – 4 b + 88 Again let x – 3 = 0, then x = 3
... x – 4 is a factor of f(x) Substituting the value of x is f (x) ; we have
 Remainder = 0 f (3) = 2 (3)3 + a (3)2 – 11 × 3 + b
123 Arun Deep's Understanding Math-10
= 2 × 27 + 9 a – 33 + b Sol. Let 2 x + 1 = 0, then 2 x = – 1
= 54 + 9 a – 33 + b 1
 x
 9 a + b + 21 2
...
Substituting the value of x in
Remainder = 42
f (x) = 2 x2 – 5 x + p
 9 a + b + 21 = 42
 9 a + b = 42 – 21
f 
FG 1 IJ  2 FG 1 IJ 2  5 FG 1 IJ  p
 9 a + b = 21 ...(ii) H 2K H 2 K H 2 K
Subtracting (i) from (ii) ; we have
1 5 1 5
 5 a = 15  2×   p    p  3 p
4 2 2 2
15 2 x + 1 is the factor of p (x).
 a 3
5  Remainder = 0
Substituting the value of a is (i) ; we have  3+p=0p=–3
4 (3) + b = 6 Again substituting the value of x in q (x)
 12 + b = 6 q (x) = 2 x2 + 5 x + q
 b = 6 – 12
 b=–6 FG 1 IJ =2 FG  1 IJ 2  5 FG  1 IJ  q  2× 1  5  q
q 
 f (x) will be 2 x3 + 3 x2 – 11 x – 6
H 2K H 2K H 2K 4 2
... x – 2 is a factor (as remainder = 0) 1 5 4
  q   q q 2
 Dividing f (x) by x – 2, we get 2 2 2
x  2 ) 2 x 3  3 x 2  11 x  6 ( 2 x 2  7 x  3 2 x + 1 is the factor of q (x)
2 x3  4 x2  Remainder = 0
   q – 2 = 0  q = 2
7 x 2  11 x Hence p = – 3, q = 2
7 x 2  14 x Now (i) ... 2 x + 1 is the factor of p (x)
 
3x  6 = 2 x2 – 5 x – 3
3x  6  Dividing p (x) by 2 x + 1,
 
× 2 x  1 ) 2 x2  5 x  3( x  3
2 x2  x
 2 x3 + 3 x2 – 11 x – 6
 
= (x – 2) (2 x2 + 7 x + 3) 6x 3
= (x – 2) [2 x2 + 6 x + x + 3] 6x 3
= (x – 2) [2 x (x + 3) + 1 (x + 3)]  
= (x – 2) (x + 3) (2 x + 1) Ans. ×
9. If (2x + 1) is a factor of both the expressions  2 x2 – 5 x – 3 = (2 x + 1) (x – 3)
2 x2 – 5 x + p and 2 x2 + 5 x + q, find the (ii) ... 2 x + 1 is the factor of q (x) = 2 x2 + 5 x + 2
value of p and q. Hence find out factors of
 Dividing q (x) by 2 x + 1,
both the polynomials.
124 Arun Deep's Understanding Math-10
2 x  1 ) 2 x2  5 x  2 ( x  2 11. When a polynomial f(x) is divided by (x – 1),
2 x2  x the remainder is 5 and when it is divided by
  (x – 2), the remainder is 7. Find the remainder
4x 2 when it is divided by (x – 1) (x – 2).
4x 2 Solution—
  When f (x) is divided by (x – 1),
×
Remainder = 5
2 x2 + 5 x + 2 = (2x + 1) (x + 2) Ans.
 Let r – 1 = 0  x = 1
10. If a polynomial f(x) = x4 – 2x3 + 3x2 – ax – b
 f (1) = 5
leaves remainders 5 and 19 when divided by
(x – 1) and (x + 1) respectively, find the When divided by (x – 2), Remainder = 7
values of a and b. Hence, determine the Let x – 2 = 0  x = 2
remainder when f(x) is divided by (x – 2).  f (2) = 7
Sol. f(x) = x4 – 2x3 + 3x2 – ax – b Let f (x) = (x – 1) (x – 2) q (x) + ax + b
f(1) = 5 and f(1)2 = 19 Where q (x) is the quotient and ax + b is
 (1)4 – 2(1)3 + 3(1)2 – a(1) + b = 5 remainder
Putting x = 1, we get :
and (–1)4 – 2(–1)3 + 3(–1)2 – a(–1) + b = 19
f (1) = (1 – 1) (1 – 2) q (1) + a × 1 + b
 1–2+3–a+b=5
=0+a+b=a+b
and 1 + 2 + 3 + a + b = 19
and x = 2, then
 –a + b = 5 – 2 and a + b = 19 – 6
f (2) = (2 – 1) (2 – 2) q (2) + a × 2 + b
 a + b = 3 ...(1) and a + b = 13 ...(2)
= 0 + 2a + b = 2a + b
On subtracting (1) from (2), we get
 a+b =5 ....(i)
a + b – (–a + b) = 13 – 3
2a + b = 7 ....(ii)
a + b + a – b = 10 Subtracting, we get
2a = 10 –a = –2  a = 2
a=5 Substituting the value of a in (5)
putting a = 5 in equation 1, we get 2+b=5b=5–2=3
–5 + b = 3, b = 8  a = 2, b = 3
a = 5, b = 8  Remainder = ax + b
= 2x + 3
7
Ratio and Proportion
POINTS TO REMEMBER
1. If a and b are two quantities of same kind, then the relation between a and b is called ratio
a
and it is written as a : b or where ‘a’ is the first term or antecedent and ‘b’ is the second
b
term or consequent.
Note : Multiplication or division by a number to both the terms, does not effect the ratio.
2. Compounded Ratio : When two or more ratios are multiplied together, the new ratio so
formed is called the compound ratio.
3. Duplicate Ratio : When a ratio is multipied by itself, then it is called duplicate ratio as
duplicate ratio of a : b will be a2 : b2.
4. Triplicate ratio : When a ratio is compounded three times of itself, it is called triplicate ratio
such as triplicate ratio of a : b is a3 : b3.
5. Sub-duplicate Ratio : Subduplicate ratio of a : b is a : b
Sub-triplicate Ratio : Sub-duplicate ratio of a : b is 3 3
6. a : b
7. Reciprical ratio of a : b is b : a
8. Comparision of Ratios : We can compare the ratios after converting them into equivalent
like fractions.

PROPORTION 4. Mean proportional : If a, b, c are in


1. When two ratios are equivalent, they are continued proportion, then b is called the
called proportion. Four terms a, b,c,d are means proportional If a, b, c are in
called proportion if and only if a : b = c : d continued portion, then b2 = ac or b  ac
and is written as
Some Properties of Ratio and Proportion
a c
a:b::c:d or  1. Invertendo : If a : b : : c : d, then b : a : :
b d
or ad = bc, d is called the fourth d : c is called invertendo
bc 2. Alternendo : If a : b : : c : d, then
proportion and is formed d 
a a : c : : b: d is called alternendo.
2. Continued proportion : Quantities of the 3. Componendo : If a : b : : c : d then (a + b):
same kind a, b, c, d, e, f,....... are called to b : : (c + d) : d is called componendo.
be in continued proportion if and only if 4. Dividendo : If a : b : : c : d then
a b c e
   (a – b) : b : : (c – d) : d is called dividendo
b c d f ...........
3. Third proportional : If a, b, c are in 5. Componendo and dividendo : If a : b : : c
continued proportion, then c is called the : d, then (a + b) : (a – b) : : (c + d) : (c – d)
third proportion is called componendo and dividendo and is

125 Arun Deep's Understanding Math-10


126 Arun Deep's Understanding Math-10
a b cd a c e
written as  also. 7. If b  d  f , then each ratio will be
a b cd
6. Convertendo : If a : b : : c : d, then ace Sum of antecedents
equal to 
a : (a – b) : : c : (c – d) is called convertendo. bd  f Sum of consequents

EXERCISE 7.1
1
1. An alloy consists of 27 kg of copper a  b (a  b) 2 a 4  b4
2  × 2 ×
a  b a  b 2 (a 2  b 2 ) 2
3
and 2 kg of tin. Find the ratio by weight a  b (a  b) (a  b)
4  ×
of tin to the alloy. a b a 2  b2

Sol. Quantity of Copper given in alloy  27


1
kg ( a 2  b 2 ) ( a  b) ( a  b)
2 ×
55 ( a  b) 2 ( a  b) 2
 kg,
2 1
 or 1 : 1 Ans.
3 1
Weight of Tin in given alloy  2 kg
4 3. Find the duplicate ratio of
11
 kg (i) 2 : 3 (ii) 5 :7 (iii) 5a : 6b
4
55 11 110  11 121 Sol. (i) Duplicate ratio of 2 : 3 = (2)2 : (3)2
 Total alloy     kg
2 4 4 4 =4:9
11 121 (ii) Duplicate ratio of 5 : 7 = ( 5 )2 : (7)2
Now Ratio between tin and alloy  kg : kg
4 4 = 5 : 49
= 11 : 121 = 1 : 11 Ans.
(iii) Duplicate ratio of 5a : 6b = (5a)2 : (6b)2
2. Find the compounded ratio of :
= 25a2 : 36b2 Ans.
(i) 2 : 3 and 4 : 9
4. Find the triplicate ratio of
(ii) 4 : 5, 5 : 7 and 9 : 11
(iii) (a – b) : (a + b), (a + b)2 : (a2 + b2) and 1 1
(i) 3 : 4 (ii) : (iii) 13 : 23
(a4 – b4) : (a2 – b2)2 2 3
Sol. (i) 2 : 3 and 4 : 9 Sol. (i) Triplicate ratio of 3 : 4 = (3)3 : (4)3

2 4 8 = 27 : 64
Compounded ratio  ×  or 8 : 27
3 9 27 3 3
(ii) 4 : 5, 5 : 7 and 9 : 11 (ii) Triplicate ratio of
1 1
: 
1 FG IJ FG IJ
:
1
2 3 2 H K HK
3
4 5 9 36
Compound ratio =    
1 1
: = 27 : 8
5 7 11 77 8 27
or 36 : 77 (iii) Triplicate ratio of 13 : 23 = (13)3 : (23)3
(iii) (a – b) : (a + b), (a + b)2 : (a2 + b2) = (1)3 : (8)3 = 1 : 512 Ans.
and (a4 – b4) : (a2 – b2)2 5. Find the sub-duplicate ratio of
Compound ratio
127 Arun Deep's Understanding Math-10
1 1 order of magnitude :
(i) 9 : 16 (ii)  : (iii) 9a2 : 49b2
4 9 2 : 3, 17 : 21, 11 : 14 and 5 : 7
Sol. (i) Sub-duplicate ratio of 9 : 16 Sol. Writing the given ratios in fraction
 16 = 3 : 4
9 : 2 17 11 5
, , ,
1 1 1 1 3 21 14 7
(ii) Sub-duplicate ratio of :  :
4 9 4 9 LCM of 3, 21, 14, 7 = 42
1 1 Converting the given ratio as equivalent
 : =3:2
2 3 2 × 14 17 × 2
2 28 17 34
(iii) Sub-duplicate ratio of 9a 2 : 49b 2   ;  
3 3× 14 42 21 21× 2 42
 9 a 2 : 49b 2 = 3a  3a = 7b  7b 11 11× 3 33 5 5× 6 30
= 3a : 7b Ans.   ;  
14 14 × 3 42 7 7 × 6 42
6. Find the sub-triplicate ratio of Since 28 < 30 < 33 < 34
1 1 28 30 33 34
(i) 1 : 216 (ii) : (iii) 27a3 : 64b3   
8 125 42 42 42 42
Sol. (i) Sub-triplicate ratio of 1 : 216 From above, numbers writing in ascending
3
order,
= 1 : 3 216
28 30 33 34 2 5 11 17
1 1 , , , or , , ,
 (13 ) 3 :( 6 3 ) 3 =1:6 42 42 42 42 3 7 14 21
1 1 or 2 : 3 ; 5 : 7 ; 11 : 14 and 17 : 21 Ans.
(ii) Sub-triplicate ratio of : 9.(i) If A : B = 2 : 3, B : C = 4 : 5 and
8 125
C : D = 6: 7, find A : D
1 1
1 1 A 2 B 4 C 6
F 1 I F 1 IJ  LMFG 1 IJ 3 OP 3 : LMFG 1 IJ 3 OP 3
 G J :G
3 3 Sol.  ,  , 
B 3 C 5 D 7
H 8 K H 125K MNH 2 K PQ MNH 5 K PQ A B C 2 4 6
Multiplying × × = × ×
1 1 B C D 3 5 7
 : =5:2
2 5 A 16
   A : D = 16 : 35 Ans.
[on multiplying by L.C.M. of 2 and 5 i.e. 0 ] D 35
(iii) Sub-triplicate ratio of 27a3 : 64b3 (ii) If x : y = 2 : 3, and y : z = 4 : 7, find x : y : z
1 1 Sol. LCM of y’s terms 3 and 4 = 12
 3ab g 3 3
b g
: 4b
3 3
= 3a : 4b Ans. Making equals of y as 12
x 2 2×4 8
7. Find the reciprocal ratio of    or 8 : 12
y 3 3× 4 12
1
(i) 4 : 7 (ii) 32 : 42 :2 (iii) y 4 3 12
9  ×  or 12 : 21
z 7 3 21
Sol. (i) Reciprocal ratio of 4 : 7 = 7 : 4 Then x : y : z = 8 : 12 : 21 Ans.
(ii) Reciprocal ratio of 32 : 42 = 42 : 32
1 1 1 1
= 16 : 9 10. (i) If A : B  : and B : C = : , find
4 5 7 6
1 1 A : B : C.
(iii) Reciprocal ratio of : 2 = 2 : = 18 : 1
9 9 1 5 5
Sol. A : B  × 
8. Arrange the following ratios in ascending 4 1 4
128 Arun Deep's Understanding Math-10

and B : C =
1 6 6
 
FG Substituting the value of a IJ
7 1 7 H bK
LCM of B’s terms 4 and 6 = 12 45 45  33 12
3
Making terms of B’s; as 12  11  11  11
27 27  55 82
A 5  3 15 5
=  = 15 : 12 11 11 11
B 4  3 12 12 11 12 6
 ×  
B 6  2 12 11 82 82 41
and =  = 12 : 14  (15a – 3b) : (9a + 5b) = 6 : 41 Ans.
C 7  2 14
 A : B : C = 15 : 12 : 14 Ans. 12. (i) If (4x2 + xy) : (3xy – y2) = 12 : 5,
(ii) If 3A = 4B = 6C, find A : B : C find (x + 2y) : (2x + y).
A 4 (ii) If y (3x – y) : x (4x + y) = 5 : 12.
Sol. Given, 3A = 4B   or A : B = 4 : 3
B 3 Find (x2 + y2) : (x + y)2.
and 4B = 6C Sol. (i) Given, (4x2 + xy) : (3xy – y2) = 12 : 5
B 6 3
   or B : C = 3 : 2 4 x 2  xy 12
C 4 2  
 A : B : C = 4 : 3 : 2 Ans. 3 xy  y 2 5

3x  5 y 7  20x2 + 5xy = 36xy – 12y2


11. (i) If   20x2 + 5xy – 36xy + 12y2 = 0
3x  5 y 3 , find x : y (2006)
 20x2 – 31xy + 12y2 = 0
3x  5 y 7
 20 x 2 31xy 12 y 2
Sol. Given,
3x  5 y 3    0
y2 y2 y2
 9x + 15y = 21x  35y [By cross multiplication] (Dividing by y2)
 21x  9x = 15y + 35y
x  20
FG x IJ 2  31 FG x IJ  12  0
 12x = 50y  =
50
=
25 H yK H yK
y 12 6
Hence, x : y = 25 : 6 Ans.
F xI2 F xI F xI
 20 G J  15 G J  16 G J + 12 = 0
(ii) If a : b = 3 : 11, find (15a – 3b) : (9a + 5b).
H yK H yK H yK
Sol. Given, a : b = 3 : 11 or
a

b 11
3 F xI L F xI O L F xI O
 5 G J M 4 G J  3P 4 M4 GH JK  3P  0
15a 3b
H yK N H yK Q N y Q

15a  3b b b L F xI O L F xI O
Now 9 a  5b  (Dividing by b)  M 4 G J  3 P M5 G J  4 P  0
9 a 5b
 N H yK Q N H yK Q
b b
15a
 3 15× 3  3
x F xI
Either 4 FG IJ  3  0, then 4 G J  3
b 11
H yK H yK
 
9a 3 x 3
5 9×  5  
b 11 y 4
129 Arun Deep's Understanding Math-10
 36xy – 12y2 = 20x2 + 5xy
or 5
FG x IJ  4  0,  20x2 + 5xy – 36xy + 12y2 = 0
H yK
 20x2 – 31xy + 12y2 = 0
F xI x 4
then 5 G J  4  
H yK y 5 x2 xy 12 y 2
 20  31  0
y2 y2 y2
x
2 (Dividing by y2)
x  2y y
Now  (Dividing by y)
2x  y x FG x 2 IJ  31FG xy IJ  12  0
2 1  20
y
H y2 K H y2 K
x 3
(a) When  , then 2 F xI
 20FG IJ  15 F I  16 GH JK  12  0
y 4 x x
H yK H y K y
x 3
2 2

y
 4
11
4
11 F xI L F xI O L F xI O
 5 G J M 4 G J  3P   4 M 4 GH JK  3P  0
x 3   4 H yK N H yK Q N y Q
2 1 2  1 3 5
y 4 1
2 2
L F x I  3OP LM5 FG x IJ  4 OP  0
 M4

11 2 11
  N H 4K Q N H yK Q
4 5 10
(x + 2y) : (2x + y) = 11 : 10
L F xI O
Either M 4 G J  3P  0,
x 4
N H yK Q
(b) When y  5 , then
then 4 FG IJ  3 
x x 3

H yK y 4
x 4 14
2 2 L F xI O
x  2y y 5 or M5 G J  4 P  0
   5 N H yK Q
2x  y x 4 8
2 1 2  1 1
y 5 5
then 5 FG IJ  4 
x x 4

(Dividing by y) H yK y 5
x 3
14 (a) when 
14 5 14 y 4
 5   
13 5 13 13 then (x2 + y2) : (x + y)2
5
x2 y2

x  2 y 11 14 x2  y2 y2 y2
Hence  or  
2 x  y 10 13 ( x  y) 2 1 (Dividing by y2)
( x  y) 2
y2
x  2y
 = 11 : 10 or 14 : 13 Ans.
2x  y
x2
(ii) Given, y (3x – y) : x (4x + y) = 5 : 12 1
y2


3 xy  y 2

5 F x  1I 2
4 x 2  xy 12 Hy K
130 Arun Deep's Understanding Math-10
2 (ii) Given, (3x + 1) : (5x + 3) is the triplicate
F 3I  1 9  1 ratio of 3 : 4.

H 4 K  16
F 3  1I 2 F 7 I 2 3 x  1 ( 3) 3
 5x  3  
27
H 4 K H 4K (4) 3 64
 64 (3x + 1) = 27 (5x + 3)
25
16 25 16 25  192x + 64 = 135x + 81
   
49 16 49 49  192x – 135x = 81 – 64
16
17
. .. (x2 + y2) : (x + y)2 = 25 : 49  57x = 17  x 
57
x 4 17
(b) When  , then Hence x  Ans.
y 5 57
(iii) Given, (x + 2y) : (2x – y) is the duplicate
x2
1 F xI2 1 ratio of 3 : 2,
x2  y2 y2 H yK
  x  2 y ( 3) 2 9
( x  y) 2 2
FG x  1IJ F x  1I 2  2x  y  
(2) 2 4
Hy K Hy K  9 (2x – y) = 4 (x + 2y)
 18x – 9y = 4x + 8y
F 4 I 2  1 16  1 41  18x – 4x = 8y + 9y

H 5 K  25  25 41 25 41
  
F 4  1I 2 F 9 I 2 2581 25 81 81  14x – 17y
x 17
 y  14 .
H 5 K H 5K
 x : y = 17 : 14 Ans.
. ..(x2 + y2) : (x + y)2 = 41 : 81 Ans.
14. (i) Find two numbers in the ratio of 8 : 7
13. (i) If (x – 9) : (3x + 6) is the duplicate ratio
of 4 : 9, find the value of x. 1
such that when each is decreased by 12 ,
(ii) If (3x + 1) : (5x + 3) is the triplicate ratio 2
of 3 : 4, find the value of x. they are in the ratio 11 : 9.
(iii) If (x + 2y) : (2x – y) is equal to the duplicate (ii) The income of a man is increased in the
ratio of 3 : 2, find x : y. ratio of 10 : 11. If the increase in his income
is `600 per month, find his new income.
2
x9 4 Sol. (i) Let the required numbers be 8x and 7x.
Sol. (i) Given, =  
3x  6 9 According to given condition, we have

x9 16 25 16 x  25
 = 8x 
3x  6 81 2  11 2 
11

25 9 14 x  25 9
 81x – 729 = 48x + 96 7x 
2 2
 81x – 48x = 96 + 729
(16 x  25)  2 11
825  
 33x = 825  x = = 25 Ans. 2 (14 x  25) 9
33
131 Arun Deep's Understanding Math-10
16 x  25 11 4
  and Blind School’s share = ` 2100 × = `1200
14 x  25 9 7
 154x – 275 = 144x – 225 16. (i) The sides of a triangle are in the ratio
 154x – 144x = 275 – 225 7 : 5 : 3 and its perimeter is 30 cm. Find
the lengths of sides.
 10x = 50
(ii) If the angles of a triangle are in the ratio
50 2 : 3 : 4, find the angles.
 x  =5
10 Sol. (i) Given, Perimeter of a triangle = 30 cm.
 Required numbers are 8x = 8 × 5 = 40 Ratio among their sides = 7 : 5 : 3
and 7x = 7 × 5 = 35 Ans.  Sum of ratio terms = 7 + 5 + 3 = 15
(ii) Let the present income = 10x 7
 Length of first side  30 × = 14cm
then increased income = 11x 15
 Increase per month = 11x – 10x = x 5
Length of second side  30 × = 10cm
 x = `600 15
Now his new income = 11x = 11 × 600 3
and Length of third side  30 × = 6 cm.
15
= `6600
 Sides of triangle are 14cm, 10cm, 6 cm.
15. (i) A woman reduces her weight in the ratio
7 : 5. What does her weight become if (ii) Since, sum of angles of a triangle = 180º
originally it was 91 kg. Ratio among angles = 2 : 3 : 4
(ii) A school collected `2100 for charity. It was  Sum of ratio terms = 2 + 3 + 4 = 9
decided to divide the money between an
2
orphanage and a blind school in the ratio  First angle  180º × = 40º
of 3 : 4. How much money did each 9
receive? 3
Second angle  180º × = 60º
Sol. (i) Ratio between the original weight and 9
reduced weight = 7 : 5
4
Let original weight of woman = 7x and Third angle  180º × = 80º
9
then her reduced weight = 5x
 Required angles of triangle are 40º, 60º and
If original weight = 91 kg. 80º
91× 5 x 1 1 1
then reduced weight  = 65 kg. 17. Three numbers are in the ratio : : .
7x 2 3 4
(ii) Total amount to be distributed = `2100 If the sum of their squares is 244, find the
Ratio between orphanage and a blind school numbers.
=3:4 Sol. Given, the ratio of three numbers be
 Sum of ratio terms = 3 + 4 = 7 1 1 1
: :
2 3 4
3
 Orphanage school’s share = ` 2100 ×
7 1 1 1
i.e.  12 :  12 :  12 = 6 : 4 : 3
= `900 2 3 4
132 Arun Deep's Understanding Math-10
Let the required first number 6x, second  Ratio between their investments
4x and third number be 3x. = 300000 : 240000 : 400000
 According to the given condition, we have = 30 : 24 : 40
(6x)2 + (4x)2 + (3x)2 = 244  Sum of ratios = 30 + 24 + 40 = 94
 36x2 + 16x2 + 9x2 = 244 Given total earnings = `18800
 61 x2 = 244
30
 A’s share  × 18800 = `6000
244 94
 x2  = 4 = (2)2
61
24
 x=2 B’s share  × 18800 = `4800
94
 first number = 6x = 6 × 2 = 12
40
second number = 4x = 4 × 2 = 8 C’s share  × 18800 = `8000
94
and third number = 3x = 3 × 2 = 6
19. (i) In a mixture of 45 litres, the ratio of
18. (i) A certain sum was divided among A, B milk to water is 13 : 2. How much water
and C in the ratio 7 : 5 : 4. If B got `500 must be added to this mixture to make the
more than C, find the total sum divided. ratio of milk to water as 3 : 1 ?
(ii) In a business, A invests `50000 for 6 (ii) The ratio of the number of boys to the
months, B `60000 for 4 months and C, number of girls in a school of 560 pupils is
`80000 for 5 months. If they together earn 5 : 3. If 10 new boys are admitted, find
`18800 find the share of each. how many new girls may be admitted so
Sol. (i) Ratio between A , B and C = 7 : 5 : 4 that the ratio of the number of boys to the
number of girls may change to 3 : 2.
Let A’s share = 7x
Sol. (i) Given, Mixture of milk and water = 45
B’s share = 5x
litres
and C’s share = 4x
Ratio of milk and water = 13 : 2
 Total sum = 7x + 5x + 4x = 16x  Sum of ratio terms = 13 + 2 = 15
Now according to the condition, we have
45×13
5x – 4x = 500  x = 500  Quantity of milk  = 39 litres
15
 Total sum = 16x = 16 × 500 = `8000
2
(ii) A’s 6 months investment = `50000 and quantity of water  45 × = 6 litres
15
 A’s 1 month investment
Let x litre of water be added, then quantity
= `50000 × 6 = `300000 of water
B’s 4 month’s investment = `60000 = (6 + x) litres
 B’s 1 month investment Now new ratio = 3 : 1
= Rs. 60000 × 4 = `240000  39 : (6 + x) = 3 : 1
C’s 5 months investment = `80000 39 3
 C’s 1 month investment    39 = 18 + 3x
6 x 1
= `80000 × 5 = `400000
 3x = 39 – 18 = 21
133 Arun Deep's Understanding Math-10
21 5x – 3y = 80 ...(i)
 x = 7 litres. and 7x – 5y = 80 ...(ii)
3
 7 litres of water is to be added. Multiplying (i) by 7 and (ii) by 5 and
subtracting, we get
(ii) Given, ratio between boys and girls = 5 : 3
35x – 21y = 560
Given, no. of pupils = 560
35x – 25y = 400
Sum of ratio terms = 5 + 3 = 8
– + –
5
 No. of boys  × 560 = 350 On subtracting, 4y = 160  y = 40
8
From (i), 5x = 80 + 3 × 40 = 200  x = 40
3
and no. of girls  × 560 = 210 So, monthly pocket money of Ravi
8
= `5 × 40 = `200
No. of new boys admitted = 10
(ii) Let the number of students in the class
 Total number of boys = 350 + 10 = 360 =x
Let the no. of girls admitted = x
Ratio of boys and girls = 4 : 3
 Total number of girls = 210 + x
4x
Now according to the condition, we have  No. of boys 
7
360 3
360 : 210 + x = 3 : 2  210  x  3x
2 and no. of girls 
7
 630 + 3x = 720
According to the problem, we have
 3x = 720 – 630 = 90
90
F 4 x  20I : F 3x  12I = 2 : 1
 x 
3
= 30 H7 K H7 K
 No of girls to be admitted = 30 4 x  140 3 x  84
i.e. : : 2 :1
20. (i) The monthly pocket money of Ravi and 7 7
Sanjeev are in the ratio 5 : 7. Their 4 x  140 7 2
expenditures are in the ratio 3 : 5. If each   
7 3 x  84 1
saves `80 every month, find their monthly
pocket money. 4 x  140 2
 
(ii) In class X of a school, the ratio of the 3 x  84 1
number of boys to that of the girls is 4 : 3.  6x – 168 = 4x + 140
If there were 20 more boys and 12 less  6x – 4x = 140 + 168
girls, then the ratio would have been 2 : 1,
308
How many students were there in the class?  2x = 308  x  = 154
2
Sol. (i) Let the monthly pocket money of Ravi
and Sanjeev be 5x and 7x respectively. Also, Hence, required number of students = 154
let their expenditure be 3y and 5y 21. In an examination, the ratio of passes to
respectively. failures was 4 : 1. If 30 less had appeared
Then according to given condition, we and 20 less passed, the ratio of passes to
have failures would have been 5 : 1. How many
134 Arun Deep's Understanding Math-10
students appeared for the examination.  x × 24 = 3 × 2
Sol. Let the number of students who passes 3× 2 1
the examination = 4x  x 
24 4
and number of students who fails = x (iii) Given, 2·5 : 1·5 = x : 3
Then total number of students appeared in Product of means = Product of extremes
exam = 4x + x = 5x  1·5 × x = 2·5 × 3
In second case, the number of students
appeared = 5x – 30 2. 5 × 3
x = 5·0
1. 5
and number of students who passes the
examination = 4x – 20 (iv) x : 50 : : 3 : 2
 No. of failures students = (5x – 30) – (4x Product of means = Product of extremes
– 20)  x × 2 = 50 × 3
= 5x – 30 – 4x + 20 = x – 10
50 × 3
According to the given condition, we have x = 75
2
4 x  20 5 2. Find the fourth proportional to

x  10 1 (i) 3, 12, 15
 5x – 50 = 4x – 20
1 1 1
 5x – 4x = – 20 + 50 (ii) , ,
3 4 5
 x = 30
(iii) 1·5, 2·5, 4·5
 Required number of students appeared =
5x (iv) 9·6 kg, 7·2 kg, 28·8 kg
= 5 × 30 = 150 Sol. (i) Let fourth proportional to
3, 12, 15 be x.
EXERCISE 7.2
then 3 : 12 : : 15 : x
1. Find the value of x in the following Product of extremes = Product of means
proportions :
 3 × x = 12 × 15
(i) 10 : 35 = x : 42
(ii) 3 : x = 24 : 2 12 × 15
x = 60
(iii) 2·5 : 1·5 = x : 3 3
(iv) x : 50 : : 3 : 2 1 1 1
(ii) Let fourth proportional to , , be x
Sol. (i) Given, 10 : 35 = x : 42 3 4 5
Product of means = Product of extremes
1 1 1
 35 × x = 10 × 42 then : :: : x
3 4 5
10 × 42 1 1 1
 x = 2 × 6 = 12  ×x  ×
35 3 4 5
(ii) Given, 3 : x = 24 : 2
1 1 3 3
Product of means = Product of extremes  x × × 
4 5 1 20
135 Arun Deep's Understanding Math-10
(iii) Let fourth proportional to 1
(iv) Let x be the third proportional to 5 and 7
1·5, 2·5, 4·5 be x. 4
then 1·5 : 2·5 : : 4·5 : x 1 21
then 5 : 7 :: 7 : x  : 7 :: 7 : x
 1·5 × x = 2·5 × 4·5 4 4
21
2 .5× 4 .5  ×x = 7 × 7
x = 7·5 4
1.5 7×7×4 28 1
x  9
(iv) Let fourth proportional to 9·6 kg, 7·2 kg, 21 3 3
28·8 kg be x 1
 Required third proportional  9 Ans.
then 9·6 : 7·2 : : 28·8 : x 3
4. Find the mean proportion of :
Product of means = Product of extremes
1 1
 9·6 × x = 7·2 × 28·8 (i) 5 and 80 (ii) and
12 75
7 .2 × 28 .8 (iii) 8·1 and 2·5
x = 21·6
9 .6 (iv) (a – b) and (a3 – a2b), a > b
3. Find the third proportional to Sol. (i) Let x be the mean proportion of 5 and
(i) 5, 10 (ii) 0·24, 0·6 80
then 5 : x : : x : 80
1
(iii) Rs. 3, Rs. 12 (iv) 5 and 7.  Product of extremes = Product of means
4
 x2 = 5 × 80
Sol. (i) Let x be the third proportional to 5,10,
then 5 : 10 : : 10 : x  x  5 80  400 = 20
 Product of extreme = Product of means x = 20
10 ×10 Hence required mean proportion = 20
 5 × x = 10 × 10  x  = 20 1
5 (ii) Let x be the mean proportion of and
 Required third proportional = 20 12
1
(ii) Let x be the third proportional to 0·24, 0·6 75
then 0·24 : 0·6 : : 0·6 : x 1 1
then : x :: x :
 0·24 × x = 0·6 × 0·6 12 75
0 .6 × 0 .6 1 1 1
x  x2  × 
= 1·5 12 75 900
0 .24
1 1 1
 Required third proportional be = 1·5  x = 
900 30  30 30
(iii) Let x be the third proportional to 1
Rs. 3 and Rs. 12 Hence the required mean proportion 
30
then Rs. 3 : Rs. 12 : : Rs. 12 : x (iii) Let the x be the mean proportion of 8·1
and 2·5.
12 ×12  8·1 : x : : x : 2·5
 x = 48
3
 x2 = 8·1 × 2·5
 Required third proportional = Rs. 48
136 Arun Deep's Understanding Math-10

 x  8 .1× 2 .5  20 .25 = 4·5 each of the numbers 23, 30, 57 and 78 so


that the remainders are in proportion ?
Hence required mean proportion = 4·5 Ans. (2004)
(iv) Let x be the mean proportion to Sol. Let x be subtracted from each term, then
(a – b) and (a3 – a2 b), a > b 23 – x, 30 – x, 57 – x and 78 – x are
then (a – b) : x : : x : (a3 – a2b) proportional
x2 = (a – b) (a3 – a2b)  23 – x : 30 – x : : 57 – x : 78 – x
= (a – b) a2 (a – b) = a2 (a – b)2 23  x 57  x
 
 x = a (a – b) 30  x 78  x
Hence the required mean proportion = a (a  (23 – x) (78 – x) = (30 – x) (57 – x)
– b) Ans.  1794 – 23x – 78x + x2
5. If a, 12, 16 and b are in continued proportion, = 1710 – 30x – 57x + x2
find a and b.  x2 – 101x + 1794 = x2 – 87x + 1710
Sol. ... a, 12, 16, b are in continued proportion,
 x2 – 101x + 1794 – x2 + 87x – 1710 = 0
a 12 16
then    – 14x + 84 = 0  14x = 84
12 16 b
84
a 12 x  =6
   16 a = 144 14
12 16 Hence 6 is to be subtracted Ans.
144 8. If k + 3, k + 2, 3k – 7 and 2k – 3 are in proportion,
 a 9
16 find k.
12 16 Sol.  k + 3, k + 2, 3k – 7 and 2k – 3 are in
and   12 b = 16 × 16 = 256 proportion.
16 b
256 64 1  k  3 : k  2  3k  7 : 2k  3
b   21
12 3 3  (k  3) (2k  3)  (k  2)(3k  7)
64 1
Hence a = 9, b  or 21 Ans.  2k 2  3k  6k  9  3k   6k  7 k  14
3 3
6. What number should be added to each of  2k 2  3k  9  3k   k 14
the numbers 5, 11, 19 and 37 so that they
are in proportion ? (2009)   k 2  4k  5  0  k 2  4k  5  0
Sol. Let x be added to 5, 11, 19 and 37 to make
them in proportion.  k 2  5k  k  5  0
 5 + x : 11 + x : : 19 + x : 37 + x  k (k  5)  1 ( k  5)  0
 (5 + x) (37 + x) = (11 + x) (19 + x)
 (k  5) (k  1)  0  k = – 1, 5
 185 + 5x + 37x + x2 = 209 + 11x + 19x + x2
9. If x + 5 is the mean proportion between
 185 + 42x + x2 = 209 + 30x + x2 x + 2 and x + 9, find the value of x.
 42x – 30x + x2 – x2 = 209 – 185 Sol. ... x + 5 is the mean proportion between
 12x = 24  x=2 x + 2 and x + 9, then
(x + 5)2 = (x + 2) (x + 9)
 Required least number to be added = 2
 x2 + 10x + 25 = x2 + 11x + 18
7. What number should be subtracted from
 x2 + 10x – x2 – 11x = 18 – 25
137 Arun Deep's Understanding Math-10
 –x=–7 Hence required numbers are 14, 56 Ans.
 x = 7 Ans. 12. If b is the mean proportional between a and c,
10. What number must be added to each of the prove that a, c, a2 + b2 and b2 + c2 are in
numbers 16, 26 and 40 so that the resulting proportion.
numbers may be in continued proportion ? Sol. ... b is the mean proportional between a
Sol. Let x be added to each number then 16 + and c, then b2 = a × c  b2 = ac ...(i)
x, 26 + x and 40 + x are in continued Now a, c, a 2 + b 2 and b 2 + c 2 are in
proportion. proportion
16  x 26  x a a2  b2
  if 
26  x 40  x c b2  c2
After Cross Multiplying, we have if a (b2 + c2) = c (a2 + b2)
(16 + x) (40 + x) = (26 + x) (26 + x) if a (ac + c2) = c (a2 + ac) [from (i)]
 640 + 16 x + 40 x + x2 if ac (a + c) = a2 c + ac2
= 676 + 26 x + 26 x + x2 if ac (a + c) = ac (a + c)
 640 + 56 x + x2 = 676 + 52 x + x2 which is true.
 56 x + x2 – 52 x – x2 = 676 – 640 Hence proved.
36 13. If b is the mean proportional between a
 4 x = 36  x  9
and c, prove that (ab + bc) is the mean
4
 Required number to be added is 9. Ans. proportional between (a2 + b2) and
11. Find two numbers such that the mean (b2 + c2).
proportional between them is 28 and the Sol. ... b is the mean proportional between a
third proportional to them is 224. and c then b2 = ac ...(i)
Sol. Let the two numbers are a and b. Now if (ab + bc) is the mean proportional
... 28 is the mean proportional between a and between (a2 + b2) and (b2 + c2), then
b
(ab + bc)2 = (a2 + b2) (b2 + c2)
 a : 28 : : 28 : b
Now L.H.S. = (ab + bc)2 = a2 b2 + b2 c2 +
784 2ab2c
 ab = (28)2 = 784  a  ...(i)
b
... = a2 (ac) + ac (c)2 + 2a ac. c [from (i)]
224 is the third proportional to a and b
 a : b : : b : 224 = a 3c + ac3 + 2a2 c2
 b2 = 224a ...(ii) = ac (a2 + c2 + 2ac) = ac (a + c)2
Substituting the value of a in (ii) ; we get R.H.S. = (a2 + b2) (b2 + c2)
784 = (a2 + ac) (ac + c2) [from (i)]
b2  224 ×  b3 = 224 × 784
b = a (a + c) c (a + c) = ac (a + c)2
 b3 = 175616 = (56)3  L.H.S. = R.H.S.
 b = 56 14. If y is mean proportional between x and z,
Now substituting the value of b in (i) ; we prove that
get xyz (x + y + z)3 = (xy + yz + zx)3.
784
a = 14 Sol. ... y is the mean proportional between
56
138 Arun Deep's Understanding Math-10
x and z, then cz  ax
 =3
y2 = xz ...(i) (c  a ) ( z  x )
L.H.S. = xyz (x + y + z) 3
x y z
Sol. ...   = k (say)
= (xz) y (x + y + z)3 a b c
= y2 y (x + y + z)3 [from (i)]  x = ak, y = bk, z = ck
= y (x + y + z) = [y (x + y + z)]3
3 3
x3 y3 z3
= [xy + y2 + yz]3 = (xy + yz + zx)3 (i) L.H.S.   
a2 b2 c2
[using (i)]
= R.H.S a 3k 3 b 3k 3 c 3k 3
  
a2 b2 c2
1 1 m = a k + bk + ck = k3 (a + b + c)
3 3 3
15. If a + c = mb and + = , prove that
b d c
( x  y  z) 3
a, b, c and d are in proportion. R.H.S. 
( a  b  c) 2
1 1 m
Sol. Given, a + c = mb and
b
+
d
=
c ( ak  bk  ck ) 3 k 3 ( a  b  c) 3
 
a + c = mb ( a  b  c) 2 (a  b  c) 2
= k3 (a + b + c )
a c
+ =m (Dividing by b) ...(i) Hence L.H.S. = R.H.S.
b b
1 1 m L a x  b y  c z OP
2 2 2 2 2 2 3
and + = (ii) L.H.S M
b d c N a xb yc z Q
3 3 3

c c
+ =m (Multiplying by c) ...(ii) 3
b d L a 2 . a 2 k 2  b 2 .b 2 k 2  c 2 .c 2 k 2 O
M
From (i) and (ii), we have
a c c c
MN a 3 . a. k  b 3 . bk  c 3 . ck PPQ
+ = +
b b b d L a k  b k  c k OP
4 2 4 2 4 2 3
M
a
=
c N a k b k c k Q
4 4 4

b d
3
Hence, a, b, c and d are in proportion. L
M
k 2 (a 4  b4  c4 ) O
MN k (a 4  b4  c4 ) PPQ = k
3
x y z
16. If   , prove that
a b c
xyz ak . bk . ck
R.H.S   3
x3 y3 z3 ( x  y  z) 3 abc abc = k
(i)   
a 2 b2 c2 ( a  b  c) 2  L.H.S = R.H.S
3
 a2 x2  b2 y 2  c2 z 2  xyz ax  by by  cz
(ii)  3   (iii) L.H.S = 
3 3
 a x  b y  c z  abc ( a  b ) ( x  y ) (b  c ) ( y  z )
ax  by by  cz cz  ax
(iii)  
(a  b) ( x  y ) (b  c ) ( y  z ) (c  a ) ( z  x )
139 Arun Deep's Understanding Math-10
a . ak  b. bk b. bk  c. ck (a 3  c3 )2 (b 3 k 3  d 3 k 3 ) 2
  
( a  b ) ( ak  bk ) (b  c ) (bk  ck ) (ii) L.H.S =
(b 3  d 3 ) 2 (b 3  d 3 ) 2
c. ck  a . ak [ k 3 (b 3  d 3 )] 2 k 6 (b 3  d 3 ) 2
  
( c  a ) ( ck  ak ) = k6
(b 3  a 3 ) 2 (b 3  d 3 ) 2
a 2 k  b2 k b2 k  c2 k f 6k 6
  e6
( a  b ) k ( a  b ) (b  c ) k (b  c ) R.H.S  6
 = k6
f f6
c2 k  a 2 k  L.H.S = R.H.S

(c  a ) k (c  a )
a 2 c2 e2
(iii) L.H.S   
k (a 2  b2 ) k (b2  c2 ) k ( c2  a 2 ) b2 d 2 f2
  
k ( a 2  b 2 ) k (b2  c2 ) k ( c2  a 2 )
b2 k 2 d 2 k 2 f 2k2
= 1 + 1 + 1 = 3 = R.H.S   
b2 d2 f 2
a c e
17. If b  d  f , prove that : = k2 + k2 + k2 = 3k2

(i) (b2 + d2 + f 2) (a2 + c2 + e2) ac  ce  ae


R.H.S 
= (ab + cd + ef )2 bd df bf

(a 3  c3 )2 e6 bk . dk dk . fk bk . fk
(ii)    
(b  d )
3 3 2 f6 b. d d. f b. f

a 2 c2 e2 ac ce ae = k2 + k2 + k2 = 3k2
(iii) 2  2  2   
bd df df  L.H.S = R.H.S
b a f
3
LM a  b  c  d  c  f OP 3
(iv) L.H.S  b d f M
L a b cd e f O
 
(iv) b d f
N b d f Q
N b d f PQ
= 27 (a + b) (c + d) (e + f )  bd f M
L bk  b  dk  d  fk  f OP 3
a c e N b d f Q
Sol. ... b  d  f = k (say)
L b ( k  1)  d ( k  1)  f ( k  1) OP3
 bd f M
 a = b k, c = dk, e = fk
(i) L.H.S. = (b2 + d2 + f 2) (a2 + c2 + e2)
N b d f Q
= b d f (k + 1 + k + 1 + k + 1)3
= (b2 + d2 + f 2) (b2 k2 + d2 k2 + f 2 k 2)
= b d f (3k + 3)3 = 27 b d f (k + 1)3
= (b2 + d2 + f 2) k2 (b2 + d2 + f 2)
R.H.S = 27 (a + b) (c + d) (e + f)
= k2 (b2 + d2 + f 2)2
R.H.S = (ab + cd + ef )2 = 27 (bk + b) (dk + d) (fk + f)
= (b. kb. + dk. d + fk. f )2 = 27 b (k + 1) d (k + 1) f (k + 1)
= (kb2 + kd2 + kf 2)2 = k2 (b2 + d2 + f 2)2 = 27 b d f (k + 1)3
 L.H.S = R.H.S  L.H.S = R.H.S
140 Arun Deep's Understanding Math-10
18. If ax = by = cz ; prove that a = bk, c = dk.
x2 y2 z2 bc ca ab (i) L.H.S. = (5a + 7b) (2c – 3d)
    
yz zx xy a 2 b2 c2 = (5·bk + 7b) (2 dk – 3d)
Sol. Let ax = by = cz = k = k (5b + 7b) k (2d – 3d)
k k k = k2 (12b) × (– d) = – 12 bd k2
 x , y  ,z 
a b c R.H.S. = (5c + 7d) (2a – 3b)
x2 y2 z2 = (5dk + 7d) (2 k·b – 3b)
L.H.S.   
yz zx xy = k (5d + 7d) k (2b – 3b)
= k2 (12d) (– b) = – 12k2 bd = – 12 bd k2
k2 k2 k2 k2 k2 k2
2 2 2 2 2 2  L.H.S = R.H.S
 a  b  c  a  b  c
k .k k .k k .k k 2 k 2 k2 (ii) T.P. (ma + nb) : b = (mc + nd) : d
b c c a a b bc ca ab ma  nb mc  nd
 
b d
k2 bc k2 ca k2 ab
 ×  ×  × mbk  nb b ( mk  n )
a 2
k 2
b 2
k 2
c 2
k2 L.H.S.  
b b
bc ca ab = mk + n
   = R.H.S
a 2 b2 c2 mc  nd mdk  nd
R.H.S.  
19. If a, b, c and d are in proportion, prove d d
that : d ( mk  n )
 = mk + n.
(i) (5a + 7b) (2c – 3d) = (5c + 7d) (2a – 3b) d
(ii) (ma + nb) : b = (mc + nd) : d  L.H.S = R.H.S.
(iii) (a4 + c4) : (b4 + d4) = a2 c2 : b2 d2. (iii) (a4 + c4) : (b4 + d4) = a2 c2 : b2 d2
a 2  ab b 2  2 ab a 4  c4 a 2 c2
(iv)  T.P. 
c 2  cd d 2  2 cd b4  d 4 b2 d 2
(a  c) 3 a ( a  c) 2
(v)  a 4  c4 b4 k 4  d 4 k 4
(b  d ) 3 b (b  d ) 2 L. H.S.= 
b4  d 4 b4  d 4
a 2  ab  b 2 c 2  cd  d 2
(vi)  k 4 (b 4  d 4 )
a 2  ab  b 2 c 2  cd  d 2   k4
(b 4  d4)
a 2  b2 ab  ad  bc
(vii) 
c2  d 2 bc  cd  ad a 2 c2 k 2 b2 . k 2 d 2
R. H.S.    k4
b2 d2 b2 .d 2
(viii) abcd
LM 1 
1 1 1
 2  2
OP Hence L.H.S. = R.H.S.
Na 2 b 2 c d Q
= a 2 + b 2 + c2 + d 2 a 2  ab k 2 b 2  bk . b
(iv) L.H.S  2 
Sol. ... a, b, c, d are in proportion c  cd k 2 d 2  dk . d
kb 2 ( k  1) b2
a c
  
 = k (say) d 2 k ( k  1) d2
b d
141 Arun Deep's Understanding Math-10
b 2  2 ab b 2  2. bkb bk . b  bk . d  b. dk
R.H.S   
d  2 cd
2 d 2  2 dkd b. kd  dk . d  bk . d
b 2 (1  2 k ) b2 k (b 2  bd  bd )
  
b2
d (1  2 k )
2
d2 
k (bd  d 2  bd ) d2
 L.H.S = R.H.S
 L.H.S = R.H.S.
( a  c) 3
( bk  dk ) 3
(v) L.H.S.   FG 1  1  1  1 IJ
(b  d ) 3 (b  d ) 3 (viii) L.H.S. = abcd
H a 2 b2 c2 d 2 K
k 3 (b  d ) 3

(b  d ) 3
= k3
= bk .b. dk.d
LM 1  1  1  1 OP
N b2 k 2 b2 d 2 k 2 d 2 Q
a ( a  c) 2 bk (bk  dk ) 2
R.H.S. 
b (b  d ) 2

b (b  d ) 2
LM d 2  d 2 k 2  b2  b 2 k 2 OP
= k2 b2 d2
MN b2d 2 k 2 PQ
bk . k 2 (b  d ) 2
 = k3 = d2 (1 + k2) + b2 (1 + k2)
b (b  d ) 2
= (1 + k2) (b2 + d2)
 L.H.S = R.H.S. R.H.S = a2 + b2 + c2 + d2
a 2  ab  b 2 = b 2 k2 + b 2 + d 2 k2 + d 2
(vi) L.H.S 
a 2  ab  b 2 = b2 (k2 + 1) + d2 (k2 + 1)
b 2 k 2  bk . b  b 2 = (k2 + 1) (b2 + d2)

b 2 k 2  bk . b  b 2  L.H.S = R.H.S.
b 2 ( k 2  k  1) k2  k 1 20. If x, y, z are in continued proportion, prove
 
b 2 ( k 2  k  1) k2  k 1
x  y 2 x
 cd c2 d2 that : = . (2010)
R.H.S  2  y  z 2 z
c  cd  d 2
Sol. Since, x, y, z are in continued proportion
d 2 k 2  dkd  d 2
 x y
d 2 k 2  dk . d  d 2  y =
z
d 2 ( k 2  k  1) k2  k 1
  x y
d ( k  k  1)
2 2 k2  k 1 Let y = =k
z
 L.H.S = R.H.S
Then y = kz
a2  b2 b2 k 2  b2 x = yk = kz × k = k2z
(vii) L.H.S  2 
c  d2 d 2k 2  d 2
x  y 2
 1) Now L.H.S. =

b2 (k 2

b2  y  z 2
d2 (k 2  1) d2
ab  ad  bc k 2
z  kz 
2
kzk  12
R.H.S  =
kz  z 2
=
z k  12
bc  cd  ad
142 Arun Deep's Understanding Math-10
(v) abc (a + b + c)3 = (ab + bc + ca)3
k 2 z 2 k  1
2

= = k2 (vi) (a + b + c) (a – b + c) = a2 + b2 + c2
z 2 k  12
Sol. As a, b, c are in continued proportion

x k 2z a b
R.H.S. = = = k2   = k (say)
z z b c
 b = ck, a = bk = c k k = ck2
 L.H.S. = R.H.S.
a b ck 2  ck
21. If a, b, c are in continued proportion, prove (i) L.H.S  
that: bc ck  c
ck ( k  1)
pa 2  qab  rb 2  =k
a c ( k  1)
= .
pb 2  qbc  rc 2 c a 2 (b  c ) ( ck 2 ) 2 ( ck  c )
R.H.S  2 
Sol. Given a, b, c are in continued proportion b (a  b) ( ck ) 2 ( ck 2  ck )
a b c 2 k 4 c ( k  1) c 3 k 4 ( k  1)
 = = k (say)   =k
b c c 2 k 2 ck ( k  1) c 3 k 3 ( k  1)
 a = bk and b = ck ...(i)  L.H.S = R.H.S
 a = (ck)k = ck2 [Using (i)] 1 1 1
and b = ck (ii) L.H.S.  3

3

a b c3
a ck 2 1 1 1
  
L.H.S. = = = k2 ...(ii) 2
( ck ) 3 ( ck ) 3 c3
c c
1 1 1
p (ck 2 ) 2  q (ck 2 )ck  r (ck ) 2   
3
c k 6 3
c k 3 c3
R.H.S. =
p (ck ) 2  q(ck )c  rc 2 1 LM
1 1 1 OP
  
pc 2 k 4  qc 2 k 3  rc 2 k 2
c 3 k N
6 k 3 1 Q
a b c
=
pc 2 k 2  qc 2 k  rc 2 R.H.S.  2 2  2 2  2 2
b c c a a b
ck 2 ck c
c 2 k 2  pk  qk  r 
2
  
=  2  = k2 ...(iii) 2
( ck ) c 2 2
c ( ck ) 2 2
( ck ) ( ck ) 2
2 2
c 2  pk  qk  r 
ck 2 ck c
From (ii) and (iii), L.H.S. = R.H.S.   
22. If a, b, c are in continued proportion, prove c2k 2c2 c2c2k 4 c2k 4c2k 2
that : ck 2 ck c
  
c 4 k 2 c4 k 4 c 4 k 6
a b a 2 (b  c )
(i) 
bc b 2 (a  b) 
1

1

1
c 3 3
c k 3 c k6
3
1 1 1 a b c
(ii)     
a 3 b 3 c 3 2
b c 2 2
c a 2 a b2
2

1
1
LM
1

1 OP  1 LM 1  1  1OP
(iii) a : c = (a2 + b2) : (b2 + c2) c3 N
k3 k6 Q c3 Nk 6 k 3 Q
(iv) a2 b2 c2 (a–4 + b–4 + c–4) = b–2 (a4 + b4 + c4)  L.H.S = R.H.S.
143 Arun Deep's Understanding Math-10
(iii) a : c = (a2 + b 2) : (b2 + c 2)
c4 c2
 [ k 8  k 4  1]  [1  k 4  k 8 ]
a a 2  b2 c2 k 2 k2
 
c b2  c2  L.H.S. = R.H.S.
(v) L.H.S. = abc (a + b + c)3
a ck 2
L.H.S.   k2 = ck2.ck.c [ck2 + ck + c]3
c c
= c3k3 [c (k2 + k + 1)]3
( ck 2 ) 2  ( ck ) 2
R.H.S.  = c3k3.c3. (k2 + k + 1)3
( ck ) 2  c 2
= c6k3 (k2 + k + 1)3
c 2 k 4  c 2 k 2  c 2 k 2 ( k 2  1)  k 2 R.H.S. = (ab + bc + ca)3
 = (ck2 . ck + ck . c + c . ck2)3
c2 k 2  c2 c 2 ( k 2  1)
 L.H.S. = R.H.S. = (c2k3 + c2k + c2k2)3
(iv) L.H.S. = a2 b2 c2 (a–4 + b–4 + c–4) = (c2k3 + c2k2 + c2k)3
= [c2k (k2 + k + 1)]3 = c6k3 (k2 + k + 1)3
 a 2b 2 c 2
LM 1  1  1 OP  L.H.S. = R.H.S.
N a 4 b4 c4 Q (vi) L.H.S. = (a + b + c) (a – b + c)
a 2b2 c2 a 2b2c2 a 2b2c2 = (ck2 + ck + c) (ck2 – ck + c)
  
a4 b4 c4 = c (k2 + k + 1) c (k2 – k + 1)
= c2 (k2 + k + 1) (k2 – k + 1)
b2c2 c2a 2 a 2b 2
   = c2 (k4 + k2 + 1)
a2 b2 c2
R.H.S. = a2 + b2 + c2
( ck ) 2 . c 2 c 2 ( ck 2 ) 2 ( ck 2 ) 2 ( ck ) 2 = (ck2)2 + (ck)2 + (c)2
  
( ck 2 ) 2 ( ck ) 2 c2 = c2k4 + c2k2+ c2 = c2 (k4 + k2 + 1)
 L.H.S. = R.H.S.
c 2 k 2 . c 2 c2 . c2 k 4 c 2 k 4 . c2 k 2
   P.Q. If a, b, c are in continued proportion, prove
c2 k 4 c2 k 2 c2
that :
c2 c2 k 2 c2 k 6 a b a 2 (b  c )
   (i) 
k2 1 1 bc b 2 (a  b)
1 1 1 a b c
(ii)     
 c2
LM 1  k 2  k 6 OP [1  k 4  k 8 ]
c2 a 3 b 3 c 3 2
b c 2 2
c a 2 a b2
2

N k2 Q k2 (iii) a : c = (a2 + b2) : (b2 + c2)


R.H.S. = b–2 [a4 + b4 + c4] (iv) a2 b2 c2 (a–4 + b–4 + c–4) = b–2 (a4 + b4 + c4)
1 (v) abc (a + b + c)3 = (ab + bc + ca)3
 [a 4  b 4  c 4 ]
b2 (vi) (a + b + c) (a – b + c) = a2 + b2 + c2
1 Sol. As a, b, c are in continued proportion
 [( ck 2 ) 4  ( ck ) 4  c 4 ]
2
( ck ) a b
1   = k (say)
 [c 4 k 8  c 4 k 4  c 4 ] b c
c k2
2
 b = ck, a = bk = c k k = ck2
144 Arun Deep's Understanding Math-10
(i) L.H.S 1 1 1
  
c 3 3
c k 3 c k6
3
a b ck 2  ck ck ( k  1)
   =k
bc ck  c c ( k  1)

LM
1
1
1

1 OP
a 2 (b  c )
c3N k3 k6 Q
R.H.S  2
b (a  b)
1 L 1 1 O
   1P
3 M 6 3
c Nk k Q
( ck 2 ) 2 ( ck  c )

( ck ) 2 ( ck 2  ck )  L.H.S = R.H.S.
(iii) a : c = (a2 + b2) : (b2 + c2)
c2 k 4 c ( k  1)
 a a 2  b2
c 2 k 2 ck ( k  1)  
c b2  c2
c 3 k 4 ( k  1)
 =k a ck 2
c 3 k 3 ( k  1) L.H.S.   k2
c c
 L.H.S = R.H.S
( ck 2 ) 2  ( ck ) 2
R.H.S. 
1 1 1 ( ck ) 2  c 2
(ii) L.H.S.   
a 3 b3 c3
c2 k 4  c2 k 2


1

1

1 c2 k 2  c2
2
( ck ) 3 ( ck ) 3 c3
c 2 k 2 ( k 2  1)
  k2
1 1 1 c 2 ( k 2  1)
  
c3k 6 c3k 3 c3  L.H.S. = R.H.S.
1 LM 1  1  1 OP (iv) L.H.S. = a2 b2 c2 (a–4 + b–4 + c–4)

c3 N k6 k3 1Q  a 2b 2 c 2
LM 1  1  1 OP
a b c N a 4 b4 c4 Q
R.H.S.   
b2c2 c2a 2 a 2b 2
a 2b2 c2 a 2b2c2 a 2b2c2
  
ck 2 ck c a4 b4 c4
  
( ck ) 2 c 2 c 2 ( ck 2 ) 2 ( ck 2 ) 2 ( ck ) 2 b2c2 c2a 2 a 2b2
  
a2 b2 c2
ck 2 ck c
  
( ck ) 2 . c 2 c 2 ( ck 2 ) 2 ( ck 2 ) 2 ( ck ) 2
c2k 2c2 c2c2k 4 c2k 4c2k 2   
( ck 2 ) 2 ( ck ) 2 c2
ck 2 ck c
   c 2 k 2 . c 2 c2 . c2 k 4 c 2 k 4 . c2 k 2
c 4 k 2 c4 k 4 c 4 k 6   
c2 k 4 c2 k 2 c2
145 Arun Deep's Understanding Math-10
= c2k4 + c2k2+ c2 = c2 (k4 + k2 + 1)
c2 c2 k 2 c2 k 6
    L.H.S. = R.H.S.
k2 1 1
23. If a, b, c, d are in continued proportion,
 c2
LM 1  k 2  k 6 OP prove that :
N k2 Q a 3  b3  c3 a
(i) 
c2 b c d
3 3 3 d
 [1  k 4  k 8 ]
k2 (ii) (a – b ) (c – d ) = (b2 – c2)2
2 2 2 2

R.H.S. = b–2 [a4 + b4 + c4] (iii) (a + d) (b + c) – (a + c) (b + d) = (b – c)2


1 (iv) a : d = triplicate ratio of (a – b) : (b – c)
 [a 4  b 4  c 4 ]
b2 2 2
 a b a c  d b d c
FG IJ
1 (v)     
 [( ck 2 ) 4  ( ck ) 4  c 4 ]  c b  c bH K
( ck ) 2


1
[c 4 k 8  c 4 k 4  c 4 ]  (a  d ) 2
FG 1  1 IJ
c k2
2 H c2 b2 K
Sol. ... a, b, c, d are in continued proportion
c4
 [ k 8  k 4  1]
c2 k 2 
a b
 
c
 k (say)
b c d
c2  c = dk, b = ck = dk . k = dk2,
 [1  k 4  k 8 ]
k2 a = bk = dk2 . k = dk3
 L.H.S. = R.H.S.
a3 b3 c3
(v) L.H.S. = abc (a + b + c)3 (i) L.H.S.  3
b  c3  d 3
= ck2.ck.c [ck2 + ck + c]3
= c3k3 [c (k2 + k + 1)]3 ( dk 3 ) 3  ( dk 2 ) 3  ( dk ) 3

= c3k3.c3. (k2 + k + 1)3 = c6k3 (k2 + k + 1)3 ( dk 2 ) 3  ( dk ) 3  d 3
R.H.S. = (ab + bc + ca)3 d 3k 9  d 3k 6  d 3k 3

= (ck2 . ck + ck . c + c . ck2)3 d 3k 6  d 3k 3  d 3
= (c2k3 + c2k + c2k2)3 = (c2k3 + c2k2 + c2k)3
d 3 k 3 ( k 6  k 3  1)
= [c2k (k2 + k + 1)]3 = c6k3 (k2 + k + 1)3   k3
d 3 ( k 6  k 3  1)
 L.H.S. = R.H.S.
(vi) L.H.S. = (a + b + c) (a – b + c) a dk 3
R.H.S.    k3
d d
= (ck2 + ck + c) (ck2 – ck + c)
 L.H.S. = R.H.S.
= c (k2 + k + 1) c (k2 – k + 1)
(ii) L.H.S. = (a2 – b2) (c2 – d2)
= c2 (k2 + k + 1) (k2 – k + 1)
= [(dk3)2 – (dk2)2] [(dk)2 – d2]
= c2 (k4 + k2 + 1)
= (d2 k6 – d2k4) (d2k2 – d2)
R.H.S. = a2 + b2 + c2
= d2k4 (k2 – 1) d2 (k2 – 1) = d4k4 (k2 – 1)2
= (ck2)2 + (ck)2 + (c)2
R.H.S. = (b2 – c2)2 = [(dk2)2 – (dk)2]2
146 Arun Deep's Understanding Math-10
= [d2k4– d 2k 2] 2
= [d2 k2
– (k2 1)]2 2 2
 k 2  1   1 k 2 1 k 
= d4k4 (k2– 1)2   k ( k  1)    2 
 k   k k 
 L.H.S. = R.H.S.  
(iii) L.H.S. = (a + d) (b + c) – (a + c) (b + d)  k 2 ( k  1)  ( k 2  1) 
2

= (dk3 + d) (dk2 + dk) – (dk3 + dk) (dk2 + d)  


 k 
 
= d (k3 + 1) dk (k + 1) – dk (k2 + 1) d (k2 + 1)
2
= d2k (k + 1) (k3 + 1) – d2k (k2 + 1) (k2 + 1)  k (1  k 2 )  1  k 
 
= d2k [k4 + k3 + k + 1 – k4 – 2 k2 – 1]  k2 
 
= d2k [k3 – 2 k2 + k] = d2k2 [k2 – 2 k + 1] 2 2
= d2 k2 (k – 1)2
F
G
k 3  k 2  k 2  1I F k  k 3  1  k I
R.H.S. = (b – c)2 = (dk2 – dk)2 = d2k2 (k – 1)2 H k JK GH k 2 JK
 L.H.S. = R.H.S. ( k 3  1) 2 (  k 3  1) 2
 
(iv) a : d = triplicate ratio of (a – b) : (b – c) k2 k4
= (a – b)3 : (b – c)3 ( k  1)
3 2 (1  k 3 ) 2
 
k2 k4
a dk 3
L.H.S. = a : d    k3 2
d d F
G
( k 3  1) I F
1
1 I
( a  b) 3 H k K k 2 JK
2 J G
H
R.H.S.  ( k 3  1) 2 ( k 2  1)
(b  c ) 3 
k4
( dk 3  dk 2 ) 3 d 3 k 6 ( k  1) 3
  = k3 ( k 3  1) 2 ( k 2  1)
( dk 2  dk ) 3 d 3 k 3 ( k  1) 3  .
k4
 L.H.S. = R.H.S.
R.H.S.  ( a  d ) 2
1 FG

1 IJ
 a b a c 

2
c2 H b2 K
(v) L.H.S. =  
 c b 
 ( dk 3  d ) 2
FG 1

1 IJ
H2
d k 2 d k4
2 K

FG d  b  d  c IJ 2 F k  1IJ
2
H c b K  d 2 (k 3  1) G
2
Hd k K
2 4
2
 dk 3  dk 2 dk 3  dk  ( k 3  1) 2 ( k 2  1)
=    
 dk dk 2  k4
 L.H.S. = R.H.S.
2
 d  dk 2 d  dk 
–  dk  2 
 EXERCISE 7.3
 dk 
1. If a : b : : c : d, prove that
2
 dk ( k  1) dk ( k  1) 
2 2
=    2 a  5b 2 c  5d
dk dk 2  (i) 
  2 a  5b 2 c  5d
2
 d (1  k 2 ) d (1  k )  5 a  11 b 5 a  11b
   (ii) 
 dk dk 2  5 c  11 d 5 c  11 d

147 Arun Deep's Understanding Math-10
(iii) (2 a + 3b) (2c – 3d) la lc FG l IJ
  Multiply by
= (2a – 3b) (2c + 3d) mb md H m K
(iv) (la + mb) : (lc + md) : : (la – mb) Applying componendo and dividendo, we
have
: (lc – md)
la  mb lc  md
. .
Sol. (i) . a : b :: c : d 
la  mb lc  md
a c
then la  mb la  mb
b d  
lc  md lc  md

2a 2c
 FG
multiply by
2 IJ (By alternendo)
5b 5 d H 5 K  (la + mb) : (lc + md) :: (la – mb)
Applying componendo and dividendo, we : (lc – md)
have 5x  7 y 5x  7 y
2. (i) If  , show that
2 a  5b 2 c  5d 5 u  7 v 5u  7v

2 a  5b 2 c  5d x u
 .
(ii) ... a : b :: c : d y v

a c 5a 5c FG Multiply by 5 IJ 8a  5b 8a  5b a c
    (ii) If  , prove that 
b d 11b 11 d H 11 K 8c  5d 8c  5d b d

Applying componendo and dividendo, 5x  7 y 5x  7 y


Sol. (i) Given, 
5u  7 v 5u  7 v
5 a  11b 5 c  11 d
 5 x  7 y 5u  7 v
5 a  11b 5 c  11 d Applying alternendo ; 
5 x  7 y 5u  7 v
5 a  11 b 5 a  11 b Applying componendo and dividendo ; we
  have
5 c  11 d 5 c  11 d
5 x  7 y  5 x  7 y 5u  7 v  5u  7 v
(Applying alternendo) 
5 x  7 y  5 x  7 y 5u  7 u  5u  7 v
(iii) . .. a : b :: c : d 10 x 10 u x u
a c    y  v
  14 y 14 v
b d
 10 
2a 2c FG2 IJ  Dividing by 
  Multiply by  14 
3b 3d H3 K Hence proved.
Applying componendo and dividendo, we
8a  5b 8a  5b
have (ii) Given, 
8c  5d 8c  5d
2 a  3b 2 c  3d
 8a  5b 8c  5d
2 a  3b 2 c  3d   (using alternendo)
8a  5b 8c  5d
 (2 a + 3 b) (2 c – 3 d)
Applying compoundo and dividendo, we have
= (2 a – 3 b) (2 c + 3 d)
(By cross multiplication) 8a  5b  8a  5b 8c  5d  8c  5c

. .
(iv) . a : b :: c : d 8a  5b  8a  5b 8c  5d  8c  5d

a c 16a 16c
   
b d 10b 10d
148 Arun Deep's Understanding Math-10
a c 16 5. If (ma + nb) : b :: (mc + nd) : d, prove that
 (Dividing by )
b d 10 a, b, c, d are in proportion.
Hence proved. Sol. Given, (ma + nb) : b :: (mc + nd) : d
3. If (4a + 5b) (4c – 5d) = (4a – 5b) (4c + 5 ma  nb mc  nd
 
d), prove that a, b, c, d are in proporton. b d
Sol. Given, (4a + 5b) (4c – 5d)  mad + nbd = mbc + nbd
= (4a – 5b) (4c + 5d)  mad = mbc  ad = bc

4 a  5b a c
4 c  5d   .
  b d
4 a  5b 4 c  5d
Hence a : b :: c : d.
 Applying componendo and dividendo ; we
6. If (11a2 + 13 b2) (11 c2 – 13 d 2 ) = (11 a2
get
– 13 b2) (11 c2 + 13 d 2 ),
4 a  5b  4 a  5b 4 c  5d  4 c  5d prove that a : b :: c : d.

4 a  5b  4 a  5b 4 c  5d  4 c  5d Sol. Given, (11 a2 + 13 b2) (11 c2 – 13 d 2)
8a 8c = (11 a2 – 13 b2) (11 c2 + 13 d 2)
a c
   
10 b 10 d b d 11 a  13 b 2 11 c 2  13 d 2
 
Hence, a, b, c, d are in proportion. 11 a 2  13 b 2 11 c 2  13 d 2
4. If (pa + qb) : (pc + qd) :: (pa – qb) : (by alternendo)
(pc – qd) Applying componendo and dividendo, we
prove that a : b :: c : d have
Sol. Given, (pa + qb) : (pc + qd) :: (pa – pb) : 11 a 2  13 b 2  11 a 2  13 b 2
(pc – qd)
11 a 2  13 b 2  11 a 2  13 b 2
pa  qb pa  qb
  11 c 2  13 d 2  11 c 2  13 d 2
pc  qd pc  qd 
11 c 2  13 d 2  11 c 2  13 d 2
pa  qb pc  qd
  (by alternendo)
pa  qb pc  qd
(by alternendo) 22 a 2 22 c 2
 
Applying componendo and dividendo ; we 26 b 2 26 d 2
have
pa  qb  pa  qb pc  qd  pc  qd 
a2

c2 FG Dividing by 22 IJ
 b 2 d2 H 26 K
pa  qb  pa  qb pc  qd  pc  qd
a c
2 pa 2 pc  
 b d

2 qb 2 qd Hence a : b :: c : d
PQ. If (a + 3b + 2c + 6d) (a – 3b – 2c + 6d) =

a

c FG Dividing by 2 p IJ (a + 3b – 2c – 6d) (a – 3b + 2c – 6d),
b d H 2q K prove that a : b :: c : d.
Hence a : b :: c = d. Sol. Given,
149 Arun Deep's Understanding Math-10
a  3b  2 c  6d a  3b  2 c  6d xa 2b  a  b 3b  a
   ...(i)
a  3b  2 c  6d a  3b  2 c  6d xa 2b  a  b ba
(by alternendo) x 2a
Again 
a  3b  2 c  6 d b ab

a  3b  2 c  6 d Applying componendo and dividendo, we
have
a  3b  2 c  6 d
 (By alternendo) x  b 2 a  a  b 3a  b
a  3b  2 c  6 d   ...(ii)
x  b 2a  a  b a b
Applying componendo and dividendo
Adding (i) and (ii) ; we have
a  3b  2 c  6 d  a  3b  2 c  6 d
a  3b  2 c  6 d  a  3 b  2 c  6 d xa xb 3b  a 3 a  b
  
xa xb ba a b
a  3b  2 c  6 d  a  3b  2 c  6 d
 a  3b 3 a  b
a  3b  2 c  6 d  a  3b  2 c  6 d  
a b a b
2 (a  3b) 2 ( a  3 b)
  a  3b  3 a  b
2 (2 c  6 d ) 2 (2 c  6 d ) 
a b
a  3b a  3b
  (Dividing by 2) 2 a  2b 2 ( a  b)
2c  6d 2c  6d    2 Ans.
a b a b
a  3b 2c  6d
  (by alternendo) 8 ab
a  3b 2 c  6d 8. If x , find the value of
a b
Again applying componendo and x  4a x  4b
dividendo)  .
x  4a x  4b
a  3b  a  3b 2 c  6 d  2 c  6 d
 8 ab x 2b
a  3b  a  3b 2 c  6 d  2 c  6 d Sol. x   
a b 4a ab
2a 4c 2c
   Applying componendo and dividendo, we
6b 12 d 6d get

a c  2 x  4a 2b  a  b 3b  a
  Dividing by 6    ...(i)
b d   x  4a 2b  a  b ba
2 ab x 2a
7. If x , find the value of Again 
a b 4b a b
xa xb Applying compondndo and dividendo, we
 .
xa xb have
2 ab x 2b x  4b 2 a  a  b 3a  b
Sol. x      ...(ii)
a b a ab x  4b 2a  a  b a b
Applying componendo and dividendo, we Adding (i) and (ii) ; we have
have
150 Arun Deep's Understanding Math-10

x  4a x  4 b 3b  a 3 a  b 3 3 2 3 2  3
 
   3 2 3 2
x  4a x  4b ba a b
a  3b 3 a  b 3 3 2 3 2  3
  
a b a b 3 2
a  3b  3 a  b 2a  2b
  2 3 2 2 2( 3  2)
a b a b =  = 2 Ans.
3 2 3 2
2 ( a  b)
  2 Ans.
a b 36 x  1  6 x
P.Q. Solve for x :  9.
4 6 36 x  1  6 x
9. If x  , find the value of
2  3 36 x  1  6 x 9
Sol. 
x2 2 x2 3 36 x  1  6 x 1

x2 2 x2 3 Applying componendo and dividendo,
4 6 4 2× 3 36 x  1  6 x  36 x  1  6 x
Sol. Given, x  
2  3 2  3 36 x  1  6 x  36 x  1  6 x
x 2 3 9 1
 
2 2 2  3 9 1
Applying componendo and dividendo, we 2 36 x  1 10
have  
12 x 8
x2 2 2 3 2  3
 36 x  1 5
x2 2 2 3 2  3   (Squaring both sides)
6 x 4
3 3 2 36 x  1 25
 ...(i) 
3 2 36 x 16
x 2 2  36x × 25 = 16 (36x + 1)
Again 
2 3 2  3  900x = 576x + 16 900x – 576x = 16
Applying componendo and dividendo, we  324x = 16
have 16 4
 x  Ans.
x2 3 2 2  2  3 324 81

x2 3 2 2  2  3 10. Using properties of proportion, find x from
the following equations :
3 2  3
 ...(ii) 2x  2x
2  3 (i) 3
Adding (i) and (ii), we have 2x  2x

x2 2 x2 3 x  4  x  10 5
 (ii) 
x  4  x  10 2
x2 2 x2 3
3 3 2 3 2  3 1 x  1 x a
  (iii) 
3 2 2  3 1 x  1 x b
151 Arun Deep's Understanding Math-10
5x  2x  6
(iv) 4
5x  2x  6 1 x  1 x a
(iii) Given, 
1 x  1 x b
ax  ax c
(v)  Applying componendo and dividendo,
ax  ax d
1 x  1 x  1 x  1 x
a  a 2  2ax
(vi) b 1 x  1 x  1 x  1 x
a  a 2  2ax
ab

2x  2x 3 a b
Sol. (i) Given, 
2x  2x 1
2 1 x a b 1 x ab
Applying componendo and dividendo, we   
have 2 1 x a  b  1 x a b

2x  2x  2 x  2x 31


 1 x (a  b) 2
2x  2x  2x  2x 31 Squaring both sides, 1  x 
(a  b) 2
2 2x 4 2x 2
  
2 2x 2  2x 1 Again applying componendo and dividendo,
we have
Squaring both sides ; we have
2x 4 1  x  1  x  (a  b) 2  (a  b) 2
 8+4x=2–x
2x 1 1 x 1 x ( a  b) 2  ( a  b ) 2
4x+x=2–85x= –6
6 2 2 (a 2  b 2 ) 1 a2  b2
 x Ans.    
5 2x 4 ab x 2 ab

x4  x  10 5 2 ab
(ii) Given,   x Ans.
x4  x  10 2 a2  b2
Applying componendo and dividendo, we
have 5x  2x  6
(iv) 4
x4  x  10  x4  x  10 5 2 5x  2x  6

x4  x  10  x4  x  10 52 Apply componendo and dividendo ; we
2 x4 have
7 x4 7
  
3 
2 x  10 x  10
     4 1
3
5x  2 x  6  5x  2x  6
Squaring both sides, we have 
5x  2 x  6  5x  2x  6 4 1
x4 49
  49x – 490 = 9 x + 36
x  10 9 2 2x  6 3 2x  6 3
   
 49 x – 9 x = 36 + 490  40 x = 526 2 5x 5 5x 5
526 263
 x  Ans. on squaring both sides, we have
40 20
152 Arun Deep's Understanding Math-10

2x  6 9 2 a 2  2ax b  1
 
5x 25 
2a b 1
 25(2x – 6) = 9 × 5x
 50x – 150 = 45x a 2  2ax b  1
  ; on squaring both
a b 1
 50x – 45x = 150  5x = 150
sides ; we have
150 
 x=  30 a 2  2ax  b 1 
5   
a2  b 1
ax  ax c
(v) Given, 
ax  ax d 2 (b  1)2
 a2 – 2ax = a
Applying componendo and dividendo, we (b  1)2
have
ax  ax  ax  ax cd 2
 (b  1)2 
  2ax = a 1  
ax  ax  ax  ax cd  (b  1) 2 
2 ax cd ax cd
     2 2
2 ax cd ax cd 2 (b  1)  (b  1)
 2ax = a  
ax (c  d ) 2  (b  1) 2 
Squaring both sides ; a  x 
(c  d ) 2
Again applying componendo and dividendo;  b 2  1  2b  b2  1  2b 
 2ax = a2  
we have  (b  1)2 
a  x  a  x  (c  d ) 2  (c  d ) 2
axax (c  d ) 2  (c  d ) 2 a  4b  2ab
 x=    x =
2a 2 (c 2  d 2 ) a c2  d 2
2  (b  1)2  (b  1)2
   
2x 4 cd x 2 cd
11. Using properties of proportion, solve for
 x (c2 + d2) = 2 acd x. Given that x is positive.
2 acd
 x Ans. 3x  9 x2  5 2 x  4x2  1
c2  d2 (i) 5 (ii) 4
3x  9 x 2  5 2 x  4 x2  1
a  a 2  2ax
(vi) b 3x  9 x2  5
a  a 2  2ax Sol.(i) 5
3x  9 x 2  5
Apply componendo and dividendo ;
By componendo and dividendo,
 2   2 
 a  a  2ax    a  a  2ax  b  1 (3 x  9 x 2  5 )  (3 x  9 x 2  5 ) 5 1
    
2 2
a  a  2ax  a  a  2ax b 1 (3 x  9 x 2  5 )  (3 x  9 x 2  5 ) 5 1
153 Arun Deep's Understanding Math-10
 25 = 64x2
3x  9 x 2  5  3x  9 x 2  56
 25
3x  9 x2  5  3x  9 x2  5 4   x2
64
6x 6
 5
2 4  x=+ (Since x is positive) Ans.
2 9x  5 8

3x 6 1 x 
x2 62 (1  x )
 12. Solve 
 1 x 
x 2 63 (1  x )
9 x2  5 4
1 x  x2 62 (1  x )
Sol. Given, 
 12x = 6 9 x 2  5 1 x  x 2 63 (1  x )

Squaring both sides; we get (1  x ) (1  x  x 2 ) 62


 
 144x2 = 36(9x2 – 4) (1  x ) (1  x  x2 ) 63

 144x2 = 324x2 – 180 (1  x ) (1  x  x 2 ) 63


 
 180 = 180x2 (1  x ) (1  x  x 2 ) 62
180 1  x3 63
  x2  
180 1 x 3 62
 x = + 1 (Since x is positive) Applying componendo and dividendo, we
have
2 x  4x2  1 1  x3  1  x3 63  62
(ii) 4 
2x  4x 1 2
1  x3  1  x3 63  62
By componendo and dividendo, 2 125 1 125
   
3 1
2 x3 1 x
( 2 x  4 x 2  1)  ( 2 x  4 x 2  1 ) 4 1

4 1  x3 
1

1 FG IJ 3
( 2 x  4 x  1)  ( 2 x  4 x  1)
2 2
125 5 H K
1
 x Ans.
2x  4x 1  2x  4x 1
2 2
5 5

2x  4x 1  2x  4x 1
2 2 3
13. Solve for x :16
FG a  x IJ 3  a  x
H a xK ax
4x 5 3

3 Sol. Given, 16 G
F a  x IJ  a  x
2 4x 1 2
H a  xK a  x
2x

5 
FG a  x IJ × FG a  x IJ 3  16

4x 1 2 3
H a xK H axK

FG a  x IJ 4  16  (  2) 4
 6x = 5 4 x 2  1 H axK
 36x2 = 25(4x2 – 1) ax
  2
 36x2 = 100x2 – 25 ax
154 Arun Deep's Understanding Math-10
ax 2
When 
ax 1
Applying componendo and dividendo, we have
axax 2 1 2a 3 a 3
     3x=a
axax 2 1 2x 1 x 1
a
 x
3
ax 2
Case II when 
ax 1
Applying componendo and dividendo, we have
axax 2  1 2 a 1 a 1
      x = 3 a
axax 2  1 2 x 3 x 3
a
Hence x  , 3a Ans.
3
a 1  a 1
14. If x  ,
a  1  a 1
prove that x2 – 2ax + 1 = 0.
x a 1  a 1
Sol. We have 
1 a 1  a 1
Applying componendo and dividendo
x 1 a  1  a 1  a  1  a 1

x 1 a  1  a 1  a  1  a 1
x 1 a 1

x 1 a 1
Squaring both sides.
( x  1)2a 1

( x  1) 2a 1
Again applying componendo and dividendo
( x  1) 2  ( x  1) 2 a 1  a 1 2( x 2  1) 2 a x2  1
    a
( x  1) 2  ( x  1) 2 a 1 a 1 4x 2 2x
 x2 + 1 = 2 ax  x2 – 2 ax + 1 = 0.
Hence proved.

a 2  b2  a2  b2 2a 2 x
15. Given x  , use componendo and dividendo to prove that b2 = .
a 2  b2  a 2  b2 x2  1
(2010)

x a 2  b2  a 2  b2
Sol. Given, 1 
a 2  b2  a 2  b2
155 Arun Deep's Understanding Math-10
Applying componendo and dividendo both sides

x 1 a 2  b2  a 2  b2  a 2  b2  a 2  b2

x 1 a 2  b2  a 2  b2  a 2  b2  a 2  b2

2 2
x 1  2 a b x 1 a 2  b2
  
x  1 2 a2  b2 x 1 a 2  b2
Squaring both sides we have

( x  1) 2 a 2  b2 x2  1  2 x a 2  b2
   
( x  1) 2 a2  b2 x2  1  2 x a2  b2
Applying componendo and dividendo on both sides ; we have

x2  1  2 x  x 2  1  2 x a 2  b2  a 2  b 2 2 ( x 2  1) 2a2 x2  1 a2
     
x2  1  2 x  x2  1  2 x a 2  b2  a 2  b 2 4x 2 b2 2x b2

2a 2 x
 b2 =
x2  1

a 3  3ab 2 63
16. Given that = . Using Componendo and dividendo, find a : b. (2009)
b  3a b
3 2
62

a 3  3ab 2 63
Sol. Given : =
b  3a b
3 2
62

a 3  3ab 2  b 3  3a 2b 63  62 125
By componendo and dividendo ; we have = =
a  3ab  b  3a b
3 2 3 2
63  62 1

a  b 3 5
3
ab
 =    =5 a + b = 5a – 5b
a  b3 1 ab

a 6 a 3
 4a – 6b = 0  4a = 6b  =  =
b 4 b 2
 a:b=3:2

x 3  12 x y 3  27 y
17. Given = . Using componendo and dividendo, find x : y.
6x 2  8 9 y 2  27

x 3  12 x y 3  27 y
Sol. Given that =
6x 2  8 9 y 2  27
Using componendo-dividendo, we have
156 Arun Deep's Understanding Math-10

x 3  12 x  6 x 2  8 y 3  27 y  9 y 2  27
= 3 
( x  1) 3

216

FG 6 IJ 3
x 3  12 x  6 x 2  8 y  27 y  9 y 2  27 ( x  1) 3 125 H 5K
x 1 6
( x  2) 3 ( y  3)3   6x–6=5x+5
x 1 5
 =
( x  2) 3 ( y  3)3  6x – 5x = 5 + 6  x = 11 Ans.
xy yz zx
3
 y  3
3 19. If ax  by  ay  bz  , prove
 x 2 az  bx
   =  
 x2  y 3 2
that each of these ratio is equal to
ab
x2 y3
 = y3 unless x + y + z = 0.
x2
xy yz zx
Again using componendo-dividendo, we Sol. Given, ax  by  ay  bz  az  bx
get
x y yzzx
x2 x2 y 3 y 3 
= y 3 y 3 ax  by  ay  bz  az  bx
x2 x2
2 ( x  y  z)
2x 2y 
 = x ( a  b)  y ( a  b)  z ( a  b)
4 6
2 ( x  y  z) 2
x y   if x + y + z  0.
 = (a  b) ( x  y  z) ab
2 3
Hence proved.
x 2
 y = 3
MULTIPLE CHOICE QUESTIONS

Thus the required ratio is x : y = 2 : 3 Choose the correct answer from the given
18. Using the properties of proportion, solve options (1 to 10) :
the following equation for x ; given 1. The ratio of 4 litres to 900 mL is
x3  3x 341 (a) 4 : 9 (b) 40 : 9

3x2 1 91 (c) 9 : 40 (d) 20 : 9
x3  3x 341 Sol. 4l : 900 ml
Sol. 
3x2  1 91  4000 ml : 900 ml
Applying componendo and dividendo,  4000 : 900 = 40 : 9  Ans. (b)
x3  3x  3 x2  1 341  91 2. When the number 210 is increased in the ratio

x3  3x  3 x2 1 341  91 5 : 7, the the new number is
(a) 150 (b) 180
x3  3 x2  3 x  1
432 216
 3   (c) 294 (d) 420
x  3 x  3 x  1 250
2 125
157 Arun Deep's Understanding Math-10
Sol. 210 is increased in the ratio 5 : 7, then Sol. x, 12, 8, 32 are in proportion, then
New increased number will be x × 32 = 12 × 8 ( ad = bc)

7 12  8
= 210 × = 294  Ans. (c)  x= =3
5 32
 x=3  Ans. (c)
3. Two numbers are in the ratio 7 : 9. If the
sum of the numbers is 288, then the smaller 6. The fourth proportional to 3, 4, 5 is
number is 20
(a) 6 (b)
(a) 126 (b) 162 3
(c) 112 (d) 144 15 12
Sol. Ratio in two number = 7 : 9 (c) (d)
4 5
Given, Sum of numbers = 288 Sol. The fourth proportion to 3, 4, 5 will be
and Sum of ratio terms = 7 + 9 = 16 45  bc 
=  d  
288  7 3  a 
 Smaller number = = 126
16 20
= Ans. (b)
 Ans. (a) 3

2 5 3
P.Q. A ratio equivalent to the ratio : is P.Q. The ratio of 45 minutes to 5 hours is
3 7 4
(a) 180 : 23 (b) 3 : 23
(a) 4 : 6 (b) 5 : 7
(c) 23 : 3 (d) 6 : 23
(c) 15 : 14 (d) 14 : 15
3
2 5 Sol. Ratio of 45 minutes to 5 hours
Sol. : 4
3 7 3
45 minutes : 5 hours
Multiply by L.C.M of 3 and 7 i.e. 3 × 7 = 21 4
2 5 3
i.e. × 21 : × 21  14 : 15 = 45 : 5
4
× 60
3 7
 Ans. (d) 23
4. The ratio of number of edges of a cube to = 45 : × 60
4
the number of its faces is
= 45 : 345 = 3 : 23
(a) 2 : 1 (b) 1 : 2
 Ans. (b)
(c) 3 : 8 (d) 8 : 3
Sol. No. of edges of the cube = 12 1
7. The third proportional to 6 and 5 is
No. of faces = 6 4
 Ratio in edges a cube to the number of faces 1
= 12 : 6 = 2 : 1  Ans. (a) (a) 4 (b) 7
2
5. If x, 12, 8 and 32 are in proportion, then the
value of x is (c) 3 (d) none of these
(a) 6 (b) 4 1
(c) 3 (d) 2 Sol. Let the third proportional to 6 and 5 be x
4
158 Arun Deep's Understanding Math-10

1 Sol. Given, (7p + 3q) : (3p – 2q) = 43 : 2


Thus, 6 : 5 :: 5 : x
4 7 p  3q 43
 
3 p  2q 2
25
 : 5 :: 5 : x  129 p – 86 q = 14 p + 6 q
4
 129 p – 14 p = 6 q + 86 q
25 55  115 p = 92 q
 × x = 25 = x = ×4=4
4 25
p 92 4
 q  115  5
 bc 
 d  
 a   p : q = 4 : 5 Ans.
 Ans. (a) 3. If a : b = 3 : 5, find (3a + 5b) : (7a – 2b).

1 a 3
8. The mean proportional between and 128 Sol. Given, a : b = 3 : 5  =
2 b 5
is 3a  5b
3a + 5b : 7a – 2b =
(a) 64 (b) 32 7a  2b
(c) 16 (d) 8 Dividing each term by b
1 a 3
Sol. The mean proportional between and 128 3  5 3  5
2 b 5
 
a 3
1 7  2 7  2
=  128 = 64 = 8  Ans. (d) b 5
2
9  25 34
CHAPTER TEST 34
 5  5 
21  10 11 11
1. Find the compound ratio of :
5 5
(a + b)2 : (a – b)2 , (a2 – b2) : (a2 + b2),
(a4 – b4) : (a + b)4 = 34 : 11
Sol. (a + b)2 : (a – b)2, (a2 – b2) : (a2 + b2), 4. The ratio of the shorter sides of a right-
(a4 – b4) : (a + b)4 angled triangle is 5 : 12. If the perimeter of
the triangle is 360 cm, find the length of
(a  b) 2 a 2  b2 a4  b4 the longest side.
 × ×
(a  b) 2 a 2  b2 ( a  b) 4 Sol. Let the two shorter sides of a right angled
triangle be 5x and 12x.
(a  b) 2 ( a  b) ( a  b)
 × Then third (longest side) using pythagoras
(a  b) 2 a 2  b2 theorem
( a 2  b 2 ) (a  b) (a  b)  (5 x ) 2  (12 x ) 2
×
( a  b) 4
 25 x 2  144 x 2
1
 1 : 1 Ans.
1  169 x 2  13 x
2. If (7 p + 3 q) : (3 p – 2 q) = 43 : 2 find p : q But given 5x + 12x + 13x = 360 cm
159 Arun Deep's Understanding Math-10
 30x = 360 Now according to the condition, we have
360 3x  6 2
 x  12 
30 x  14 1
 Required length of the longest side = 13x
 3 x – 6 = 2 x + 28
= 13 × 12cm = 156 cm. Ans.
 3 x – 2 x = 28 + 6
5. The ratio of the pocket money saved by
Lokesh and his sister is 5 : 6. If the sister  x = 34
saves `30 more, how much more the  Required no. of candidates appeared
brother should save in order to keep the = 4 x = 4 × 34 = 136 Ans.
ratio of their savings unchanged ?
7. What number must be added to each of
Sol. Let the savings of Lokesh and his sister
the numbers 15, 17, 34 and 38 to make
are 5x and 6x.
them proportional ?
According to the problem, we have
Sol. Let x be added to each number, then
5x  y 5 numbers will be 15 + x, 17 + x, 34 + x, and

6 x  30 6 38 + x.
 30x + 6y = 30x + 150
Now according to the condition, we have
 30x + 6y – 30x = 150
15  x 34  x
 6y = 150 
17  x 38  x
150  (15 + x) (38 + x) = (34 + x) (17 + x)
 y  25
6  570 + 53 x + x2 = 578 + 51 x + x2
Hence, Lokesh should save `25 more Ans.  x2 + 53 x – x2 – 51 x = 578 – 570
6. In an examination, the number of those  2 x = 8  x = 4
who passed and the number of those who  Required number to be added be 4 Ans.
failed were in the ratio of 3 : 1. Had 8 more 8. If (a + 2 b + c), (a – c) and (a – 2 b + c)
appeared, and 6 less passed, the ratio of are in continued proportion, prove that b is
the mean proportional between a and c.
passed to failures would have been 2 : 1.
Sol. (a + 2 b + c), (a – c) and (a – 2 b + c) are
Find the number of candidates who in continued proportion.
appeared.
a  2b  c ac
Sol. Let number of students passed = 3 x  
ac a  2b  c
and no. of students failed = x
 (a + 2 b + c) (a – 2 b + c) = (a – c)2
 Total candidates appeared = 3x + x = 4x.  a2 – 2 ab + ac + 2 ab – 4 b2 + 2 bc
In second case, + ac – 2 bc + c2 = a2 – 2 ac + c2
No. of candidates appeared = 4 x + 8  a2 – 2 ab + ac + 2 ab – 4 b2 + 2 bc
and No. of passed students = 3 x – 6 + ac – 2 bc + c2 – a2 + 2 ac – c2 = 0
 4 ac – 4 b2 = 0  ac – b2 = 0
and no. of failed students = 4 x + 8 – 3 x +
 b2 = ac
6 = x + 14
Hence b is the mean proportional between
Given ratio will be = 2 : 1 a and c. Ans.
160 Arun Deep's Understanding Math-10
9. If 2, 6, p, 54 and q are in continued From (i) and (ii) ; we have
proportion, find the values of p and q.
256 y 2
Sol. .. . 2, 6, p, 54 and q are in continued   y3 = 256 × 128 = 32768
y 128
proportional then
2 6 p 54  y3 = (32)2  y = 32
  
6 p 54 q 256 256
 x=  8
2 6 y 32
(i) ...  then 2 p = 36  p = 18.
6 p  Required numbers are 8, 32.
p 54 12. If q is the mean proportional between p
(ii) 54  q  pq = 54 × 54 and r, prove that :
54 × 54  1 3 1 
 18 q = 54 × 54  q   162 p2 – 3q2 + r2 = q4  2  2  2 
18
p q r 
Hence p = 18, q = 162.
Sol. q is mean proportional between p and r
10. If a, b, c, d, e are in continued proportion,
prove that : a : e = a4 : b4.  q2 = pr
L.H.S. = p2 – 3q2 + r2 = p2 – 3pr + r2
Sol. ... a, b, c, d, e are in continued proportion
a b c d  1 3 1 
  =   k (say) and R.H.S. = q4  2  2  2 
b c d e p q r 
d = ek, c = ek2, b = ek3 and a = ek4
 1 3 1 
a ek 4 = (q2)2  2  2  2 
Now L.H.S.    k4 p q r 
e e

a4 ( ek 4 ) 4  1 3 1 
e 4 k 16
R.H.S.  = = (pr)2  2  2  2 
b4 ( ek 3 ) 4 e 4 k 12 p q r 

 k 16 12  k 4  1 3 1 
 L.H.S. = R.H.S. = (pr)2  2  pr  2 
p r 
11. Find two numbers whose mean
proportional is 16 and the third proportional  r 2  3 pr  p 2 
is 128. = p2r2   = r2 – 3pr + p2

 p2r 2 
Sol. Let x and y be two numbers
Their mean proportion = 16  L.H.S. = R.H.S. Hence proved.
a c e
 xy  16  xy = 256 13. If b  d  f , prove that each ratio is
and third proportion = 128 equal to :
256 3 a 2  5 c2  7 e2
 x= ....(i) (i)
y 3b2  5 d 2  7 f 2
1
y2 y2 LM 2 a 3  5 c3  7 e3 PO 3
and  128 x= ....(ii)
x 128 (ii)
MN 2 b3  5 d 3  7 f 3 PQ
161 Arun Deep's Understanding Math-10
a c 3 3
e  3ak  5bk  4ck   k (3a  5b  4c) 
Sol. Let    k (say)
b d f =  =  (3a  5b  4c) 
 3a  5b  4c   
 a = bk, c = dk, e = fk = (k)3 = k3
 L.H.S. = R.H.S.
3a 2  5c2  7 e2 15. If x : a = y : b, prove that
(i)
3b 2  5 d 2  7 f 2 x 4  a 4 y4  b4 ( x  y ) 4  ( a  b) 4
 
x3  a3 y 3  b3 ( x  y ) 3  (a  b) 3
3b 2 k 2  5 d 2 k 2  7 f 2 k 2 x y
 Sol. Given,   k (say)
3b 2  5 d 2  7 f 2 a b
 x = ak, y = bk.
3b 2  5 d 2  7 f2
k k x4  a 4 y 4  b4
3b 2  5 d 2  7 f2 L.H.S.  
x 3  a 3 y 3  b3
Hence proved,
1 a 4 k 4  a 4 b4 k 4  b4
 3 3 
(ii)
LM 2 a  5 c  7 e PO
3 3 3 3
a k  a 3 b 3k 3  b 3
N 2 b  5d  7 f Q
3 3 3
a 4 ( k 4  1) b 4 ( k 4  1)
1  
L 3 3 3 3 3 3 O3 a 3 ( k 3  1) b 3 ( k 3  1)
2 b k  5d k  7 f k
M
MN 2 b3  5d 3  7 f 3 PPQ 
a ( k 4  1) b ( k 4  1)

1 k3 1 k3 1

kM 3
L 2 b3  5 d 3  7 f 3 O 3
k a ( k 4  1)  b ( k 4  1)
MN 2 b  5 d 3  7 f 3 PPQ 
k3  1
Hence proved.
( k 4  1) ( a  b )
x y z 
14. If   , prove that k3 1
a b c
3 ( x  y ) 4  ( a  b) 4
3 x3  5 y 3  4 z 3
 3x  5 y  4 z  R.H.S. 
3 3

3  3a  5b  4c  ( x  y ) 3  (a  b) 3
3a  5b  4c  
x y z ( ak  bk ) 4  ( a  b ) 4
Sol. Let   = k (suppose) 
a b c ( ak  bk ) 3  ( a  b ) 3
 x = ak, y = bk, z = ck
k 4 ( a  b) 4  ( a  b) 4
3 x3  5 y 3  4 z 3 
L.H.S. = k 3 ( a  b) 3 ( a  b) 3
3a 3  5b3  4c3
3a 3k 3  5b3k 3  4c 3k 3 ( a  b ) 4 ( k 4  1)

= ( a  b ) 3 ( k 3  1)
3a 3  5b3  4c3
k 3 (3a3  5b3  4c3 ) ( a  b ) ( k 4  1) ( k 4  1) ( a  b )
=  k3  
3
3a  5b  4c 3 3
k3 1 k 3 1
 3x  5 y  4 z 
3  L.H.S. = R.H.S.
R.H.S. =   Hence proved.
 3a  5b  4c 
162 Arun Deep's Understanding Math-10
x y z
16. If 
bca cab

a bc
, F a I 2  3 2F 9 I 2  3
2
prove that each ratio's equal to : 
H b K  H 10 K
xyz 2
F a I 2  3 2F 9 I 2  3
abc
. H bK H 10 K
x y z 81 81 81  150
Sol. Let   =k 2× 3 3
bca ca b a bc 100 50 50
  
(say) 81 81 81  150
2× 3 3
 x = k (b + c – a), y = k (c + a – b), 100 50 50
z = k (a + b – c)
69 50 69 23
xyz  ×   Ans.
50 231 231 77
abc 18. If (3x2 + 2y2) : (3x2 – 2y2) = 11 : 9, find
k (b  c  a )  k ( c  a  b )  k ( a  b  c )
 3 x 4  25 y 4
abc the value of
3 x 4  25 y 4
k (b  c  a  c  a  b  a  b  c ) 3x  2 y2
2 11
 
a bc Sol. Given, 2
3x  2 y 2 9
k (a  b  c) Applying componendo and dividendo
  k . Hence proved.
a bc 3x2  2 y2  3x2  2 y2 11  9
17. If a : b = 9 : 10, find the value of 
3x  2 y  3x  2 y
2 2 2 2 11  9
5a  3b 2 a 2  3b 2
(i) (ii) 6x2 x2
5a  3b 2 a 2  3b 2 
20 3
 10
 2 
a 9 4y 2 2 y2
Sol. Given, a : b = 9 : 10  
b 10
x2 2 20
5a 3b 5a   10 × 
 3 y 2 3 3
5a  3b
 b b  b
(i) 5a  3b 5a  3b 5a  3
3x 4 25 y 4
b b b 
(Dividing by b) 3 x 4  25 y 4 y4 y4

9 9 15 3 x 4  25 y 4 3x 4 25 y 4
5× 3 3 
 10  2  2
9 9 3 y4 y4
5× 3 3
10 2 2
a
(Substituting the value of )
FG x 2 IJ 2  25 3 ×F 20 I 2  25
3
b

H y2 K 
H 3K
15 2
 ×  5 Ans.
2 3
F3G x 2 IJ  25 3F 20 I 2  25
2

H y2 K H 3K
2a 2
3b 2
2 
2
2 a  3b 2 2
(ii) Now,  b 2 b2 400 400 25
2
2 a  3b 2
2a 3b 3×  25 
_ 2  9  3 1
2 400 400 25
b b 3×  25 
(Dividing by b2) 9 3 1
163 Arun Deep's Understanding Math-10
Applying componendo and dividendo,
400  75
3 475 3 19 x  pa b  a  b a  2b
  ×    ...(i)
400  75 3 325 13 Ans. x  pa b  a  b a
3
x b
2 mab Again, pa  a  b
19. If x  , find the value of
ab Applying componendo and dividendo.
x  ma x  mb
 x  pb a  a  b 2a  b
x  ma x  mb   ...(ii)
x  pb a  a  b b
2 mab x 2b
Sol. x    x  pa x  pb
ab ma ab L.H.S. = x  pa  x  pb
Applying componendo and dividendo, we
get a  2b 2a  b a  2b 2a  b
=   
x  ma 2b  a  b 3b  a a b a b
  ...(i)
x  ma 2b  a  b ba
ab  2b2  2a 2  ab 2b 2  2a 2
x 2a = 
Again  ab ab
mb ab
Applying componendo and dividendo, we 2a 2  2b2
=
get ab
x  mb 2a  a  b 3a  b
  2(a 2  b 2 ) 2(a 2  b 2 )
x  mb 2a  a  b ab
...(ii) =  = R.H.S.
ab ab
Adding (i) and (ii) ; we have 21. Find x from the equation
x  ma x  mb 3b  a 3 a  b
   a  x  a2  x2 b
x  ma x  mb ba ab 
2 2 x
3b  a 3 a  b ax a x
 
a b a b ax a2  x2 b
Sol. Given, 
3 b  a  3 a  b ax  a2 x2 x

ab Applying componendo and dividendo,
2a  2b 2 (a  b)
  = 2 Ans. ax a2  x2  a  x  a2  x2
ab ab
ax a2  x2  a  x  a2  x2
pab
20. If x = , prove that bx
ab 
bx
x  pa x  pb 2( a 2  b2 )
  2 (a  x) bx
x  pa x  pb ab  
2 a2  x2 bx
pab
Sol. Given, x = ax bx
ab  
a2  x2 bx
x b
 Squaring both sides, we get
pa a  b
164 Arun Deep's Understanding Math-10
( a  x )2  (b  x )2  x3
+ 3 x = 3 ax2 + a
a2  x2 (b  x )2  x – 3 ax2 + 3 x – a = 0
3

Hence proved.
( a  x )2 (b  x )2
  by  cz cz  ax ax  by
(a  x ) (a  x) (b  x )2 23. If 2  2  2 , prove
b c 2 c a 2 a  b2
a  x (b  x ) 2
  that each of these ratio is equal to
a  x (b  x ) 2
x y z
Again applying componendo and dividendo,   .
a b c
axax (b  x )2  (b  x )2 by  cz cz  ax ax  by
 Sol.  2  2
axax (b  x )2  (b  x )2 b c
2 2 c a 2 a  b2
2a 2 ( b2  x 2 ) a b2  x 2 2 ( ax  by  cz ) ax  by  cz
      2
2x 4 bx x 2 bx 2 ( a 2  b 2  c2 ) a  b2  c2
 2 abx = x (b2 + x2)  2 ab = b2 + x2 (Adding)
 x 2  2 ab  b 2  x  2 ab  b 2 Ans. by  cz ax  by  cz
Now  2
b c
2 2 a  b2  c2
3
a 1  3 a 1
22. If x  , prove that : by  cz b 2  c2
  2
3
a 1  3 a 1 ax  by  cz a  b2  c2
x3 – 3 ax2 + 3 x – a = 0. (By alternendo)
3
a 1  3 a 1 by  cz  ax  by  cz
Sol. x  
3
a 1  3 a 1 ax  by  cz
b  c2  a 2  b 2  c2
2
Applying componendo and dividendo, 
a 2  b2  c2
x  1 3 a 1  3 a 1  3 a 1  3 a 1  ax a2
   2
x  1 3 a 1  3 a 1  3 a 1  3 a 1 ax  by  cz a  b 2  c 2
3 x a
x 1 2 a 1 x 1 3 a 1   2
 3   ax  by  cz a  b2  c2
x 1 2 a 1 x 1 3 a 1
x ax  by  cz
Cubing both sides ; we have   2 ...(i)
a a  b2  c2
( x  1) 3 a 1 Similarly we can prove that

( x  1) 3 a 1 y ax  by  cz
 ...(ii)
Again applying componendo and dividendo, b a 2  b2  c2
( x  1) 3  ( x  1) 3 a 1 a 1 ax  by  cz
 z
 2
( x  1)  ( x  1)
3 3 a 1 a 1 and
c a  b 2  c2
...(iii)
2 ( x3  3 x) 2 a
  from (i), (ii) and (iii); we get
2 ( 3 x 2  1) 2
x y z
x3  3 x a Hence   .
  a b c
3 x2  1 1
8
Matrices
POINTS TO REMEMBER

1. Matrix : A rectangular arrangement of numbers in the form of horizontal and vertical lines is
called a matrix. Horizontal lines are called ‘rows’ and vertical lines are called ‘columns.’
Each number of a matrix is called ‘element’.
2. Order of a matrix : If a matrix contains. m rows and n columns, then it is called matrix of
order m × n, then it has m n elements. Every matrix is denoted by a capital letter. Elements of
a matrix are enclosed in brackets [ ].
3. Equal Matrices : Two matrices A are B are called equal if they are of the same order and is
written A = B.
4. Types of matrices :
(i) Row Matrix : A matrix having one row is called row matrix.
(ii) Column Matrix : A matrix having one column is called column matrix.
(iii) Square Matrix : A matrix which has same number of rows and columns, is called a square
matrix.
(iv) Zero or Null Matrix : A matrix which has each element as zero is called a zero matrix.
(v) Identity or Unit Matrix : A square matrix in which each diagonal elements is 1 and all other
elements are zero, is called an Identity or unit matrix.
5. Operation on Matrices
(i) Addition of matrices : If A and B are two matrices of the same order, then their sum A + B is
the matrix obtained by adding the corresponding elements of A and B.
(ii) Subtraction of Matrices
If A and B are two matrices of the same order, their difference A – B is the matrix obtained by
subtracting the elements of B from the corresponding elements of A.
(iii) Multiplication of a Matrix by a number
If k is any number and A is a matrix, then the matrix kA is obtained by multiplying each element
of the matrix A by the number k.
(iv) Multiplication of two Matrices
Two matrices A and B are said to be conformable for the product AB if and only if the number
of columns in A is equal to the number of rows in B. If A is of order m × n and B is of order ‘n
× p’ then AB is the order of m × p and is defined as AB = [cik]m × p when
(i, k)th element of AB = Sum of the products of the elements of the ith row of A with the
corresponding elements of the kth column of B.
Note : 1. Multiplication of two matrices is not commutative i.e. AB  BA.
2. Multiplication of matrices is associative. If A, B and C are matrices, then (AB) C = A (BC).
165 Arun Deep's Understanding Math-10
166 Arun Deep's Understanding Math-10
3. Multiplication of matrices is distributive with respect to addition. If A, B and C are matrices,
then
A (B + C) = AB + AC
and (A + B) C = AC + BC.
Note : Cancellation law for the multiplication of matrices may not hold i.e.
If AB = AC, then it may not imply B = C, A  0.

EXERCISE 8.1
 a11 a12  1 2 
1. Classify the following matrices :  A=   .
 a21 a22   2 4 
(i)
LM 25 11OP (ii) 2 3 7 4. Find the values of x and y if
N Q
LM 03 OP LM20 40OP LM 2 x  y OP  L 5O
(iii) (iv)
N 3 x  2 y Q NM 4 QP
MN1 PQ MN 1 7 PQ Sol. Comparing corresponding elements,
2x + y = 5 ...(i)
(v)
LM 2 7 8 OP (iv) L 0 0 0O 3x – 2y = 4 ...(ii)
N 1 2 0 Q MN 0 0 0PQ Multiply (i) by 2 and (ii) by ‘1’ we get
Sol. (i) It is square matrix of order 2 4x + 2y = 10
(ii) It is row matrix of order 1 × 3 3x – 2y = 4
(iii) It is column matrix of order 3 × 1 Adding we get,
(iv) It is matrix of order 3 × 2 7x = 14  x = 2
(v) It is matrix of order 2 × 3 Substituting the value of x in (i) ; we get
(vi) It is zero matrix of order 2 × 3 2×2+y=54+y=5
2. (i) If a matrix has 4 elements, what are the  y=5–4=1
possible order it can have ? Hence x = 2, y = 1 Ans.
(ii) If a matrix has 8 elements, what are 5. Find the value of x if
the possible order it can have ?
3 x  y  y   1 2 
Sol. (i) The factors of 4 are 1, 2, 3  2 y  x 3    5 3  .
It can have 1 × 4, 4 × 1 or 2 × 2 order    
(ii) It can have 1 × 8, 8 × 1, 2 × 4 or 4 × 2 order. 3 x  y  y   1 2
Sol. Given,  2 y  x 
3. Construct a 2 × 2 matrix whose elements
 3   5 3 
aij are given by
Comparing the corresponding terms, we
(i) aij = 2i – j (ii) a ij = i.j
get
Sol. (i) Given, aij = 2i – j ; 1 < i.j = 2
–y = 2  y = –2
a11 = 2 – 1 = 1 ; a21 = 4 – 1 = 3
3x + y = 1 3x = 1 – y
a12 = 2 – 2 = 0 ; a22 = 4 – 2 = 2  3x = 1 – (–2) = 1 + 2 = 3
(ii) Given, aij = i j ; 1 < i, j < 2
3
a11 = 2 ; a12 = 1 × 2 = 2 ; a21 = 2 × 1 = 2;  x= =1
3
a22 = 2 × 2 = 4
Hence x = 1, y = –2
167 Arun Deep's Understanding Math-10
x3 4  5 4  Multiplying (i) by 1 and (ii) by 2
6. If  y  4 x  y   3 9  , find the values a + 2b = 5
   
of x and y. 6a – 2b = 2
Adding, we get
x3 4  5 4 
Sol. Given,  y  4 x  y   3 9  7a = 7  a = 1
    Substituting the value of a in (i) ; we get
Comparing the corresponding terms, we 1 + 2b = 5  2b = 5 – 1 = 4  b = 2
get
Hence x = 5, y = 1, a = 1, b = 2 Ans.
x+3=5x=5–3=2
9. Find the values of a, b, c and d if
 y–4=3 y=3+4=7
 x = 2 and y = 7 a  b 3   6 d 
7. Find the values of x, y and z if  5  c ab    1 8 
   
LM x 3 2 56z OP  LM5y2  y OP a  b 3 6 d
N Q N3 20 Q Sol. Given,  5  c ab    1 8 
Sol. Comparing the corresponding elements of    
equal matrices ; Comparing the corresponding terms, we
x + 2 = – 5  x = – 5 – 2 = – 7 get
 x = – 7, 5z = – 20 3 = d  d = 3  5 + c = –1  c = –1 –
20 5 c = –6
 z   4 z = –4
5 a + b = 6 and ab = 8
y2 + y = 6  (a – b)2 – (a + b)2 –4ab
 y + y – 6 = 0  y2 + 3y – 2y – 6 = 0
2
= (6)2 – 4 × 8 = 36 – 32 = 4 = (+2)2
 y (y + 3) – 2 (y + 3) = 0  a – b = +2
 y + 3) (y – 2) = 0 (i) If a – b = 2
Either y + 3 = 0, a+b=6
then y=–3 Adding, we get 2a = 8  a = 4
or y – 2 = 0, then y = 2 a+b=6 4+b=6b=6–4=2
Hence x = – 7, y = – 3, 2, z = – 4.  a = 4, b = 2
8. Find the values of x, y, a and b if (ii) If a = b = –2
LM x  2 y OP  L 3 1O a+b=6
N a  2b 3a  b Q MN 5 1PQ Adding, we get, 2a = 4  a =
4
=2
Sol. Comparing corresponding elements : 2
x – 2 = 3, y = 1 a+b=6  2+b=6b=6–2=4
x=3+2=5  a = 2, b = 4
a + 2b = 5 ...(i) Hence, a = 4, b = 2, or a = 2, b = 4
3a – b = 1 ...(ii) c = –6 and d = 3.
Given aij = ij ; 1 < i, j < 2 10. Find the values of x, y, a and b, if
a11 = 2 ; a12 = 1 × 2 = 2 ; a21 = 2 × 1 = 2
; a22 = 2 × 2 = 4
LM 3 x  4 y 2 x  2y OP
N ab 2a  b 1 Q
 a11 a12 
 L 5 5 OP
 A = a a  2 2 4
 21 22  MN 1 Q
168 Arun Deep's Understanding Math-10
Sol. Comparing the corresponding elements : 4y = 2 – 6 = – 4  y = – 1
3x + 4y = 2 ...(i)  x = 2, y = – 1
x – 2y = 4 ...(ii) a+b=5
Multiplying (i) by 1 and (ii) by 2 ; we get ...(iii)
3x + 4y = 2 2a – b = – 5
2x – 4y = 8 ...(iv)
Adding we get Adding we get
5x = 10  x = 2 3a = 0  a=0
Substituting the value of x in (i) ; we get Substituting the value of a in (iii) ; we get
3 × 2 + 4y = 2  6 + 4y = 2 0 + b = 5  b=5
Hence x = 2, y = – 1 a = 0, b = 5 Ans.
EXERCISE 8. 2
2
1. Given that M  1
LM 0OP 2
2 and N  1
LM 0OP
2 , find M + 2N
N Q N Q
2
Sol. Given, M  1
LM 0OP 2
2 , N  1
LM 0
2
OP
N Q N Q
2
 M + 2N  1 LM OP LM 20 OP
0 2
2  2 1
N Q N Q
24 00
 L 1 2 O  L 2 4 O  L 1  2 2  4 O  L 1 6 O Ans.
2 0 4 0 6 0
MN PQ MN PQ MN PQ MN PQ
2.
L 2 0O L 0 1O
If A  M 3 1P and B  M 2 3P find 2A – 3B.
N Q N Q
Sol.
L 2 0 O L 0 1O
Given, A  M 3 1P , B  M 2 3 P
N Q N Q
 L 2 0 O L 0 1O
2A – 3B  2 M 3 1P  3 M 2 3P
N Q N Q
40 03 3
 L 6 2 O  L 6 9 O  L 6  6 2  9 O  L 0 7 O Ans.
4 0 0 3 4
NM QP NM QP NM QP NM PQ
 sinA
–cosA   cosA sinA 
3. Simplify, sin A   + cos A  
 cosA
sinA   –sinA cosA 
 sin 2 A  sin A.cos A   cos 2 A cosA.sin A 
Sol.   
sin A cos A sin 2 A    sin A.cos A cos 2 A 
 sin 2 A+ cos 2 A  sin A.cos A + cos A.sin A  1 0 
    0 1 
sin A.cos A  sin A.cos A sin 2 A + cos 2 A   

1LM 2OP 2 LM 1
OP 0 LM 3OP
4. If A  2 3 , B 1 2 and C  2 1 find A + 2B – 3C
N Q N Q N Q
169 Arun Deep's Understanding Math-10

Sol. Given, A 
1 LM 2 OP 2
3 , B 1
LM 1 OP LM 25  27 1 4 OP 9
0  6  3 X  3
LM 3 OP
6  3 X
2 N Q N 2 , Q N Q N Q
0 LM 3 OP 1 9 LM 3
OP LM
3 1 OP
C 2 X= 6  1 2 Ans.
N 1 Q 3 3 N Q N Q
 A + 2B – 3C  1 4 3 2 
7. If   + 2M = 3 
 3 
, find the
LM
1 2 2
OP LM 1
OP LM 31OP 0  2 3 0
 2 3 2 1 2 3 2
N Q N Q N Q matrix M.
2 O L 42 O L 0 Sol. Given,
 L 2
1 9O
MN 3PQ  MN 24 PQ  MN 6 3PQ
1 4 3 2
1  4  0 2  2  9 O L 3 9 O  2  + 2M = 3 0  3
 L 2  2  6  3 
NM 3  4  3QP  NM 6 10 QP Ans.
3 2 1 4
L 0 21OP and B  LM 11 21OP ,
5. If A  M 1  2M = 3 0 
 3 –  2 3
N Q N Q  
find the matrix X if :
(i) 3A + X = B (ii) X – 3 B = 2A 9 6 1 4
= 0 
 9 –  2 3
 
0 LM 1OP LM
1 2 OP
Sol. Given, A  1 2 , B  1 1
N Q N Q  9 1 64 
(i) 3A + X = B  X = B – 3A = 0   2   9  3

1
 X  1
LM 21OP  3 LM 01 21OP 8 2 
N Q N Q = 2 12

0 3 O
 L 1 1 O  L 3
1 2
NM QP NM 6 QP 1 8 2  4 1 
 M= 2 12 = 1  6
1 0 2  3
 L 1  3 1  6 O  L 4 OP 
1 5 2
MN PQ MN 5 Ans.
Q (Dividing by 2)
(ii) Given, X – 3B = 2A  X = 2A + 3B
0 1 LM OP LM OP
1 2  2 6   3 2 
 X = 2 1 2  3 1 1 8. Given A    , B   4 0 ,
N Q N Q  2 0   
0 2
 L2
MN 4 OPQ  LMN 3 3OPQ
3 6
4 0
C   Find the matrix X such that A
0  3 2  6 O L 3 4 O 0 2
 L2  3 4  3PQ  MN 1 7 PQ
MN + 2X = 2B + C.
6. Solve the matrix equation : Sol. Given,

LM25 01OP  3 X = LM72 46OP  2 6   3 2  4 0


(1991) A   ,B   , C  
N Q N Q  2 0   4 0  0 2
L 2 1 O L 7 4 O
Sol. Given, M 5 0 P  M 2 6 P  3 X x y
N Q N Q Let X  
z t 
170 Arun Deep's Understanding Math-10
Since, A + 2X = 2B + C
5 0  2 0
 2X = 2B + C – A Hence X    ,Y 
 1 4  1 1
x y  3 2   4 0   2 6  10. Find the value of ‘x’ and ‘y’ if
 2 = 2   
z t   4 0  0 2  2 0

x 7  6  7  10 7 
2  
 6 4   4 0   2 6  9 y  5 4 5  22 15
=  8 0  0 2   2 0 
    
x 7  6  7  10 7 
Sol. 2  
 6  4  2 4  0  6   4 10  9 y  5 4 5  22 15
=  8  0  2 0  2  0    6 2
   
2 x 14  6  7  10 7 
x y   4 10     
 2z  18 2 y  10 4 5  22 15
 t   6 2 
2 x  6 14  7  10 7 
x y  1  4 10   2 5 18  4 2 y  10  5   22 15
 z      
 t  2  6 2   3 1 
 2x + 6 = 10 x=2
7 LM 0 OP  2y – 10 + 5 = 15  2y = 20
9. Find X and Y if X + Y  2 5 and X –
N Q  y = 10 Ans.
Y 0 3
LM
3 0 OP 3 4  1 y  z 0
N Q 11. If 2 5 x  + 0 1  = 10 5 find

LM
7
Sol. Given, X + Y  2
0
5
OP ...(i) the value of x, y and z.
N Q
3 4  1 y  z 0
L 3 0O
X – Y  M 0 3P ...(ii) Sol. Given, 2 5 x  + 0 1  = 10 5
N Q 

7 0  3 0  6 8 1 y  z 0
Adding (i) and (ii) we get, 2X     10  0

 2 5   0 3  2 x +
 1  = 10 5

73
LM
 20
00 OP LM
10 0 OP  6 1 8 y   z 0
N 53  2
Q N 8 Q  10  0 2 x  1 = 10 5

1 10 0  5 0  7 8 y   z 0
 X  
2  2 8  1 4   10
 2 x  1 = 10 5
7 0  3 0 On comparing, the corresponding
Subtracting (ii) from (i), 2Y     elements,
 2 5   0 3
2x + 1 = 5  2x = 5 – 1 = 4

 2Y  2  0 LM 7  3 00 OP LM
4
5 3  2
0 OP 4
N Q N 2 Q  x= =2
2
1  4 0  2 0 8 + y = 0,  y = –8
 Y  2  2 2   1 1 z=7
   
Hence x = 2, y = –8, z = 7 Ans.
171 Arun Deep's Understanding Math-10
5A + 2B = C, find the values of a, b and c.
5 2  1 2 x  1  3 8
12. If  1 y  1  2 3 2    7 2  LM
2 a 2 3
OP LM OP
      Sol. Given, A = 3 5 , B = 7 b and
find the value of x and y.
N Q N Q
Sol. Given, C   c1 911 
 
5 2  1 2 x  1 3 8  Now 5A + 2B = C
 1  2 
 y  1 3  2   7
 2 
 5 2 a  2 2 3 =  1 11 
LM OP LM OP c 9
5 2   2 4 x  2   3 8  3 5
N Q N Q
7 b  
  1 y  1   6 
4   7 2 
  
L 10 5a O L4 6 O c 9
 M 15 25P  M 14 2b P =  1 11 
 5  x 2  4 x  2  3 8
  1  6 y  1  4    7 2 
N Q N Q  
    L 10  4 5a  6 O c 9
 M 15  14 25  2b P   1 11 
 3 4  4 x   3 8  N Q  
  7 y  5    7 2 
    L 6 5 a + 6O c 9
 M 1 25  2b P   1 11 
Comparing the corresponding terms, we N Q  
get Comparing the corresponding elements,
4 – 4x = – 8  4x = –8 –4 5a + 6 = 9
12 3
 –4x = –12  x = 3  5a = 9 – 6 = 3  a 
4 5
and y + 5 = 2  y = 2 – 5 = –3  25 + 2b = – 11
 x = 3, y = –3  2b = – 11 – 25 = – 36
13. If
36
LM a4 23OP  LM 21 2b OP  LM 21 c1OP  LM 75 03OP ,  b  18
2
N findQtheNvalue ofQa ,Nb and c.Q N Q c=6
3
Sol. Given, Hence a  , b = – 18 and c = 6 Ans.
5
LM a4 OP LM 2b OP  LM 21 c1OP  LM 75
3 2
2  1
0
3
OP EXERCISE 8.3
N Q N Q N Q N Q
a 21 3b 1 3 5  2
 LM 4  1  2 2  2  c OP  LM 7
5 0O
N Q N 3PQ 1. If A =  4 2  and B =  4  , is the product
   
a  1 b  2 O L 5 0O
 LM 7
AB possible ? Give a reason. If yes, find
N  c PQ  MN 7 3PQ AB.
Comparing the corresponding elements ; Sol. Yes, the product is possible because number
we have of column in A = number of row in B
a+1=5 a=4 i.e., (2 × 2), (2 × 1) = (2 × 1) is the order of
b+2=0 b=–2 matrix AB.
– c = 3  c = – 3 Ans.  3 5   2  3 2  5 4 
AB =  4 2   4 =  4  2  (2)  4 
LM 2 a
14. If A = 3 5 , B =
OP LM72 b3OP,      
N Q N Q
6  20   26 
C   c1 911  and =  88    0 
     
172 Arun Deep's Understanding Math-10

2 LM 5 OP 1 LM 1OP  1 5 
2. If A  1 3 ; B  3 2 , find AB  4 6  . Find AB – 5C
N Q N Q  
and BA. Is AB  BA ?
3 7  0 2 
2 LM 5OP 1 LM 1 OP Sol.  AB = 2 4  5 3 
Sol. Given, A  1    
N 3 , B  3
Q N 2 Q 3  0  7  5 3  2  7  3
2 5  1 1 =  2  0  4  5 2  2  4  3
 AB   1  
 3  3 2 
 
 0  35 6  21  35 27 
2  15
 1 9LM 2  10 13 OP LM 8 OP = 0  20 4  12    20 16 
N 1  6  8 Q N 5 Q    
 1 1  2 5   1 5  5 25
and BA   and 5C = 5   
 3 2   1 3  4 6   20 30 
21
LM 53 1 2 OP LM OP 35 27   5 25
 6  2 15  6  4 9  AB – 5C = 20 16    20 30 
N Q N Q    
Hence AB  BA.
30 52 
4 6 LM2 3OP LM OP  
P.Q. If P  2 8 , Q = 1 1 , find  40 14 
2PQ.
N Q N Q
2 3
4. If A  1 LM OP
2 and LM OP
2 1
B  1 2 , find
Sol. Given, P  4 6 LM
2 8 , Q = 1
OP LM
1
OP 2 N 1 Q N Q
N Q N Q A (BA)

 2PQ  2 
 4 6   2  3
   Sol.
L 1 21OP , B  LM 21 21OP
Given, A  M 2
 2 8  1 1 N Q N Q
BA  1 2 O × L 2
L 2 1 1 2O
86LM 12  6 2 OP LM 6 OP 
 2 48 6  8  2 12 14 NM QP NM 1QP
N Q N Q 22 41
 L 1  4 2  2O = L 5 4O
12 4 5
LM
4
 24 28 Ans.
OP MN PQ MN PQ
N Q  1 2  4 5
1
P.Q. Given A  8
LM 31OP , evaluate A – 4A 2 Thus, A (BA)     5 4
N Q  2 1  
Sol. Given, A  L 1 1O 4  10
LM 58
 8  5 10  4 = OP LM 14 13 OP
MN8 3PQ N Q N 13 14 Ans.
Q
5. Given the matrices :
 L 1 1O L 1 1O L 1 1O
A – 4A  M 8 3 P M 8 3 P  4 M 8 3P
2
N QN Q N Q 2LM OP LM
1
A  4 2 , B = 1
3 4OP
2 and
18 13
N Q N Q
 L 8  24 8  9 O  L 32 12 O
4 4
3
C  L 0 2 O
1
NM QP NM QP MN PQ
 L 32 17 O  L 32 12 O
9 4 4 4 Find the product of (i) ABC (ii) ACB and
MN PQ MN PQ state whether they are equal.
94 44
 L 32  32 17  12 O  L 0 5O Ans.
5 0
NM QP NM QP 4 2
LM OP LM
Sol. Given, A  2 1 , B= 3 4 ,
1 2
OP
3 7  0 2 
N Q N Q
3. If A = 2 4  , B = 5 3  and C = LM03 21OP
    C
N Q
173 Arun Deep's Understanding Math-10

 2 1  3 4   3
ABC  
1
7. If A 
LM21 43OP and B  L 2 3OP find
     N Q NM 4 6 ,
Q
 4 2   1 2   0 2 
the matrix AB + BA.
61
LM 8  2 O L 3 1O
 12  2 16  4 QP NM 0 2 QP  1 3  2 3
N Sol. Given, AB     
 2 4   4 6 
3
 L10 12 O × L 0 2 O
5 6 1
NM QP NM QP  LM 24  1216 3  18 14
OP LM
6  24  12
15
30
OP
15  0 5  12 15 7
N Q N Q
 L 30  0 10  24 O  L 30 14
OP
MN QP MN Q BA  L 4
2 3 1
OP LM OP
3
MN 6 × 2
Q N 4Q
 2 1  3 1  3 4
ACB       1 2  2  6 6  12 O L 8 6
 4 2   0 2     L 4  12 12  24 QP  NM 8 36
OP
NM Q
6  0
LM
 12  0
2  2O L 3 4O  AB + BA
N 4  4 QP × NM 1 2 QP
14
LM 15 O L 8 6 O
 12 30 QP  NM 8 36 QP
6 0 N
 L 12 0 O × L 1 2 O
3 4
NM QP NM QP 14  8 15  6 22 21
 L 12  8 30  36 O  L 20 66
OP
18  0 24  0
 L 36  0 48  0 O  L 36
18 24
48
OP NM QP NM Q
NM QP NM Q L 1 2 O L 6 1O
 ABC  ACB. P.Q. If A  M 3 4 P , B  M 1 1P and
N Q N Q
4 sin 30º 2 cos 60º  4 5 2 3O
6. Evaluate :  sin 90º C L 0
 2 cos 0º  5 4 NM 1QP
find each of the following and state if they
4 sin 30º 2 cos 60º  4 5 are equal.
Sol.  sin 90º 2 cos 0º  5 4
 (i) CA + B (ii) A + CB

sin 30º =
1
, cos 60º =
1
Sol. (i) Thus, CA 
LM 20 3
OP LM 31 42 OP
1
2 2 N QN Q
sin 90º = 1 and cos 0º = 1 2  9 4  12 O  L 11 16O
 L 03 0  4 PQ MN 3 4PQ
 1 1
MN
4  2   4 5
=  2
 1
2 
2  1  5 4  CA + B  M 3
L 11 164 OP  LM 61 11OP
N Q N Q
11  6 16  1O L 5 15 O
2 1  4 5  L 31 4  1PQ  MN 4
= 1
 2 5 4 MN 5PQ
(ii) A + CB
2  4  1  5 2  5  1  4 LM
1 2 OP LM 31OP × LM 61 11OP
2
= 1  4  2  5  3 4  0
 1  5  2  4 N Q N Q N Q
2 O L 12  3 2  3O
 L3
1
 85 10  4 13 14 MN 4 PQ  MN 0  1 0  1PQ
= 4  10 58  = 14 13
 
174 Arun Deep's Understanding Math-10

 LM 1 2 OP  LM15 5OP 1
10. If A  2
LM 23OP , B = LM 23 21OP and
N3 4 Q N 1 1Q N Q N Q
1  15 2  5O L 14 3
 LM OP C  L 3 1O
1 3
N 3  1 4  1PQ  MN 4 5 Q MN PQ
We can say that CA + B  A + CB. find the matrix C (B – A) (1993)

8. If A  2
1 LM 2
OP 3
1 and B  2
LM 2OP
1 ,
Sol.  B  A 
2 LM OP LM 23OP  LM11 11OP
1 1
2  2
N Q N Q 3 N Q N Q N Q
find 2B – A2
L 1 3O L1 1O
C (B – A)  M 3 1P × M1 1P
3
Sol. Now, 2 B  2 2 LM OP LM 24 OP
2 6
1  4
N Q N Q
N Q N Q 1  3 1  3 4 4
 L 3  1 3  1O  L 4 4 O Ans.
L 1 2O L 1 2 O
and A = A × A  M2 1P M2 1P
2 NM QP NM QP
N Q N Q 1 0  2 3
11. Let A =   and B =  1 0
1  4 2  2 O L 3
 L2  2
0O .
MN 4  1PQ  MN 0 3 PQ 2 1 
Find A2 + AB + B2.
3
 2B – A  LM 4 2 OP  LM 0 3OP
2 6 4 0
N Q N Q 1 0 2 3
Sol. Given A = 
1  1 0
and B =
6  ( 3) 4  0O L6  3 2 
 LM
4O
N 4  0 2  ( 3) QP  NM 4 2  3QP 1 0  1 0
A2 = A . A = 
 L 4 1  2 1
9 4O ×
NM 5QP Ans. 2

L 1 2 O L 2 1O
9. If A  M 3 4 P , B  M 4 2 P and
11  0  2 1 0  0 1
N Q N Q =  
2 1 1 2 2  0 11
C  L 7 4 O , compute
5 1
MN PQ 1  0 0  0 1 0
=   = 
(i) A (B + C) (ii) (B + C) A. (1992) 2  2 0  1 4 1
Sol. Now,
1 0  2 3
 1 2    2 1  5 1  A.B= 
1   1 0
A (B + C)       2
3 4    4 2   7 4  
1 2  0 1 13  0 0  2 3
= 2 2 1 1 23 1 0 = 
1 2 25
 L3 4 O L4  7
11
OP LM 42 OP LM117 26 OP
1 6
NM QP MN 24  3
Q N QN Q   3

7  22 2  12 O L 29 14 O 2 3  2 3
 L 21  44 6  24 PQ  MN 65 30 PQ B2 = B . B = 
0   1 0
MN ×
 1
L 7 2O L1 2O
(B + C) A  M11 6 P × M 3 4 P  2  2  3  ( 1) 2  3  3 0 
N Q N Q =  1 2  0  (1)  1 3  0  0
 

76 14  8 1 6
 L11  18 22  24 O  L 29 46 OAns.
13 22  4 3 6  0 
MN PQ MN PQ =  20 30  = 
   2  3
175 Arun Deep's Understanding Math-10

1 0 2 3  1 0
 A2 + AB + B2 = 
1  3 6 C=   , find AC + B2 – 10C.
+ +
4   1 4 
1 6
 2  2 3  0 4
  3 Sol. Given, A =  ,B=  1 7  ,
5 7   
1 2 1 0  3 6  4 9
=  4  3  2 1  6  (3)  = 
  5 4  1 0
C=  
 1 4
2 1  4 1
12. Let A = 0 2  , B =  3 2  AC + B2 – 10C
   
2 3  1 0    1 12 
 3 2  AC =    =  
and C =  1 4  , find A2 + AC – 5B. (2014) 5 7   1 4  2 28
 
 0 4  0 4  4 28
2 1  2 1  B2 =    
Sol.  A2 + AC – 5B = 0  2 0  2 +  1 7   1 7   7 45
   
 AC + B2 – 10C
2 1   3 2 4 1 
    –5    1 12    4 28  1 0
0  2    1 4  3  2  2 28    7 45  10  1 4
     
(Substituting the values from question)
 15 40
4  0 2  2   6  1 4  4 = 
33
Ans.
= 0  0 0  4  +  0  2 0  8   1
   

4 1 
1
14. If A = 0
LM 0 OP 2 3
1 , find A and A . Also
– 5  3  2
N Q
  state which of these equal to A.

4 0   7 8   20 5  Sol. Given, A = 1
0
LM 0
1
OP
= 0 4 +  2  8 –  15  10 N Q
     
 A2 = A × A = 0 LM 1
OP LM 01 01OP
0
1
4  7  20 0  8  5 
NQN Q
=  0  2  15 4  8  10 1  0 0  0 O L 1 0O
  = L0  0 0  1PQ = MN 0 1PQ
MN
 23 3 L 1 0O L 1 0O
=  17 6 Ans. 3 2
A = A – A = M 0 1P × M 0 1P
  N QN Q
1 0 0  0
= L 0  0 0  1O = L 0 1O
1 0
 2 3  0 4 MN PQ MN PQ
13. If A =  , B =  1 7  ,
5 7    From above, it is clear that
A3 = A.
176 Arun Deep's Understanding Math-10

4
15. If X = 1
LM 1OP 2
2 , show that 6X – X = 9I
LM 45OP  LM 24 42 OP  LM 03 03OP
5
 4
N Q N Q N Q N Q
where I is the unit matrix.
5  2  3 4  4  0O L0 0O
 L4  4  0 5  2  3 PQ  MN 0 0 PQ
Sol. Given, X = 
 4 1 MN

 1 2   X2 – 2X – 3 I = O Hence proved.
17. Find the matrix X of order 2 × 2 which
4
X2 = X × X = 1
LMOP LM 41 21OP
1
2 satisfies the equation
N QN Q
16  1 42 LM 23 74 OP LM 05 23OP  2 X  LM 41 5 OP
= L 4  2 1  4 O = L 6 3 O
15 6
MN PQ MN PQ N QN Q N 6 Q
(1999)
L 4 1O L 15 6O
L.H.S.= 6X – X = 6 M 1 2 P  M 6 3P
2
3 LM OP LM 05 23OP  2 X
7
N Q N Q Sol. Given, 2
N QN Q4
 L 6 12 O  L 6 3O 5
 L 4 OP
24 6 15 6 1
NM QP NM QP NM 6 Q
 24  15 6  6  9 0   LM 00  2035 46  1221OP  2 X  LM 41 5O
6 PQ
   N Q N
 6  6 12  3 0 9  LM 2035 2716OP  2 X  LM 41 65OP

N Q N Q
 9 L 0 1O  9I = R.H.S.
1 0
1 5
NM PQ 2 X   L20 16O  L 4 6O
35 27
 MN PQ MN PQ
Hence proved.
  34  32 
=   24 10 
16.
L 1 21OP is a solution of the
Show that M 2

N Q 1 34 32
LM 17 OP L 16 OP
matrix equation X2 – 2X – 3I = 0, where I  X  12 5 Ans.
2 24 10
N Q MN Q
is the unit matrix of order 2.
Sol. Given, X2 – 2X – 3I = 0 1 LM 1OP
18. If A  x x , find the value of x, so
 1 2
N Q
X that A2 = 0.

 2 1
1
Sol. Given, A 2  x
LM 1 OP LM
1
x × x
1
x
OP
 1 2  1 2
N Q N Q
 X2     LM 1  2x 1  xO
 2 1  2 1 
Nx  x x  x 2 PQ
1 4 LM 22 OP LM
5 4 OP ... A2 = O
 X2  2  2 41  4 5
N Q N Q
Now x2 – 2x – 3I  1 x 1  x 0 LM 0 OP
  2
O  0 0
 x  x
2
x  x  N Q
LM
5
 4
4 OP LM
1
5 2 2
2OP LM
1
1 3 0
0
1
OP
N Q N Q N Q On comparing ; 1 + x = 0  x = – 1 Ans.
177 Arun Deep's Understanding Math-10
and x + = 0  x (1 + x) = 0
x2
6 x  2 x  16
 x=0–1  3 y  6 y  =  9 
   
1 3   2   x 
P.Q. If 0 0   1   0  , find the value of x. 8 x  16
      9 y  =  9 
   
1 3   2   x  On comparing, we get
Sol. Given 0 0   1   0 
     16
8x = 16  x = =2
8
 2  3  x  1  x 
 0  0  =  0   0    0  9
        and 9y = 9  y = =1
9
Comparing the corresponding elements, we Hence x = 2, y = 1
have
20. Find the values of x and y if
x = –1 Ans.
LM x  y y OP L 2 O  L 3 O
19.
L 3
(i) Find x and y if M 0
N
2
5
OP LM 2x OP  LM y5OP
QN Q N Q N 2x xy Q MN 1PQ MN 2 PQ
2 x x  3 16 Sol.
Lx  y
Given, M 2 x
y O L 2 O L 3O

N x  y PQ MN 1PQ MN 2 PQ
(ii) Find x and y, if  y   
3 y  2 = 9.
  
2x  2 y  y  L 3O
(2009)   4x  x  y   M2 P
  NQ
Sol. (i) Given,
LM 03 25OP LM 2x OP  LM y5OP
N QN Q N Q 2x  y  L 3O
  3x  y   M 2 P
 3x  4  L 5O   NQ
  0  10   M y P
  N Q Comparing the corresponding elements
2x + y = 3 ...(i)
 3x  4  L 5 O 3x + y = 2 ...(ii)
  0  10   M y P
  N Q Subtracting, we get
Comparing the corresponding elements –x=1x=–1
– 3x + 4 = – 5  – 3x = – 5 – 4 = – 9 Substituting the value of x in (i) ; we get
9
 x= =3 2 (–1) + y = 3  – 2 + y = 3
3
–10 = y  y = – 10  y=3+2=5
Hence x = 3, y = – 10 Ans. Hence x = – 1, y = 5Ans.
1 2   x 0   x 0 
2 x x  3 16 21. If 3 3   0 y    9 0  , find the
(ii) Given,  y 3 y  2 =  9      
 values of x and y. (1998)
2x  3  x  2 16 1 2   x 0   x 0 
  y  3  3 y  2 =  9  Sol. Given, 3 3   0 y    9 0 
        
178 Arun Deep's Understanding Math-10
 x  0 0  2 y   x 0 Comparing the corresponding elements of two
 3 x  0 0  3 y    9 0  equal matrices, x = 36 Ans.
   
 x 2 y   x 0 24. If A  2 LM OP
x and 4
B 0 LM OP
36 , find
 3 x 3 y    9 0  0 N 1Q N 1 Q
    the value of x, given that A2 = B.
Comparing the corresponding elements
2y = 0  y = 0 Sol. Now, A 2 
2
0
LM x
1
OP LM 20 x1OP
3x = 9 x = 3 N QN Q
Hence x = 3, y = 0 Ans.
40
LM 2 x  x O L4 3x OP
 00 0  1PQ  MN 0
22. If M 2
L 3 45OP  LM ac db OP LM 01 01OP , write N 1 Q
N Q N QN Q
down the values of a, b, c and d.
... A2 = B

Sol.
L 3 45OP  LM ac db OP LM 01 01OP
Given, M 2  LM 40 3x 4
1  0
OP LM 36
1
OP
N Q N QN Q N Q N Q
 LM 23 45OP  LM ac  00 00  db OP Corresponding the corresponding elements
N Q N Q 3x = 36  x = 12

 LM 23 45OP  LM ac db OP Hence x = 12 Ans.


N Q N Q
Comparing the corresponding elements 3 x  9 16 
25. If A = 0 1  and B = 0  y  , find x
a = 3, b = 4, c = 2, d = 5 Ans.    
23. Find the value of x given that A2 = B and y when A 2 = B. (2015)

3 x  9 16 
2 12 4 x Sol. Given : A = 0 1  and B =   , and
Where A = 
1 
and B = 0 1 
(2005)   0  y 
0 
A2 = B
 2 12  4 x  Now, A2 = A × A
Sol. Given, A =  ,B= 0 1  and A = B
2

0 1    3 x  3 x 
 A. A = B = 0 1  × 0 1 
   
 2 12   2 12   4 x 
     9 3 x  x  9 4 x 
0 1  0 1  0 1  = 0 1 
 =  
 0 1 
2  2  12  0 2  12  12  1  4 x  We have A2 = B
  = 
 0  2  1  0 0  12  1  1  0 1  Two matrices are equal if each and every
corresponding element is equal.
 4  0 24  12  4 x 
   =   9 4 x  9 16 
0  0 0  1  0 1  Thus, 0 1  = 0  y 
   
 4 36   4 x   4x = 16 and 1 = –y
     x = 4 and y = –1
0 1  0 1 
179 Arun Deep's Understanding Math-10

 2 0  1  2  y 1
PQ. If A = 0
LM LM 2 1x OP ,
4 OP
26. Find x, y if  3 1  2 x  + 3  1  = 2  3  1 and B =
        N MN 0  2 PQQ
(2014) find the value of x if AB = BA.

4 L
 2 0  1  2  y 2 xO
 AB = L 0 1O × M
1 1P
Sol. Given,  3 1  2 x  + 3  1  = 2  3  Sol.
MN PQ MN 0  2 PQ
       

  2  1  0  2 x    6 2 y  L 2  0 x  12 OP  LM 2 x  12 OP
 M
  +   =  
 3   1  1  2 x   3 6 MN 0  0 0  2 PQ MN 0 2 PQ
 2    6 2 y  L 2 1x OP L 1 4 O
  +   =   and BA = M
 3  2 x  3 6 MN 0  2 PQ MN 0 1PQ
 26  2 y  L2  0 8  1x OP  LM2 8  1x OP
  =   M
 3  2 x  3 6 MN 0  0 0  2 PQ MN 0 2 PQ
...
 4  2 y  AB = BA
  =  
 2x  6 LM 2 x2 2 OP LM 8x OP
 1  1
 2x = 6 and  2y = –4 MN 0 0 PQ MN PQ
2 2
x = 3 and  y = –2
Comparing the corresponding elements ;
27.
La 1O L 4 3O L b
If M 1 0 P M 3 2 P  M 4
11OP
c , find a, b,
we get
N QN Q N Q x – 2 = 8 – x  x + x = 8 + 2  2x = 10
and c.
10
L a 1O L 4 3O L b 11c OP  x = 5 Ans.
Sol. Given, M 1 0 P M 3 2 P  M 4 2
N QN Q N Q
LM 4 4a  03 3a3  20 OP  LM 4b 11c OP 2
28. If A = 1 LM 3 OP
2 , find x, y so that
 N Q
N Q N Q A2 = x A + yI
 LM 4 a  43 3a  23OP  LM 4b 11c OP 2 LM OP LM 21 23OP
3
N Q N Q Sol.  A 2 = 1
N 2QN Q
Comparing the corresponding elements '
we get 43LM 6  6O L7 12 OP
 22 3  4 PQ  MN 4 7
3a + 2 = 11  3a = 11 – 2 = 9 N Q
... A2 = xA + yI
9
 a  =3 7 LM 12OP LM2 3 OP LM
1 0 OP
3  4
N 7  x 1
Q N 2 y 0
Q N 1 Q
4a – 3 = b  b = 4 × 3 – 3 = 12 – 3 = 9
and 3 = c LM 74 12 O L 2 x 3xO L y 0O

N 7 QP  NM x 2 x QP  MN 0 y PQ
Hence a = 3, b = 9, c = 3 Ans.
180 Arun Deep's Understanding Math-10

7 12  x = 1 and x + 2y = 2  1 + 2y = 2
2x  y LM 3x OP
 4 7   x 2x  y 1
  N Q  2y = 2 – 1 = 1  y 
2
Comparing the corresponding elements ;
we have 1
Hence x = 1, y 
3x = 12  x = 4 2
and 2x + y = 7  2 × 4 + y = 7
 8+y=7 y=7–8=–1  M= 1
LM 1OP
Ans.
Hence x = 4, y = – 1 Ans.
N 2 Q
2 6 LM 3 OP LM x OP  2 1 7 
29. If P = 3 9 , Q = y 2 , find x 31. Given   X =   . Write :
N Q N Q  3 4 6 
and y such that PQ = O.
(i) the order of the matrix X (ii) the matrix X.
Sol.
L2 6O L 3
Given, P = M 3 9 P , Q = M y
x
2
OP  2 1 7 
N Q N Q Sol. Given,  3 4 X =  
  6 
PQ = L 3 9 O L y
MN PQ MN 2 OPQ
2 6 3 x
 (i) We see that (2 × 2) × (2 × 1) = 2 × 1
L 6  6 y 2 x  12 OP  The order of X is 2 × 1.
=M
N 9  9 y 3 x  18 Q  x
. .. PQ = O (ii) Let X =  y 
 
LM 6  6 y
 9  9y
2 x  12
OP L0
 0
0 OP  2 1  x  7   2x  y  7 
N 3 x  18Q MN 0 Q So,  3 4  y  = 6   3x  4 y  = 6
Comparing the corresponding elements ;          
we have 2x + y = 7 ...(i)
6 + 6y = 0  6y = – 6 –3x + 4y = 6 ...(ii)
 y=–1 Multiplying (i) by 3 and (ii) by 2, and adding
2x + 12 = 0  2x = – 12  x = – 6 we get :
6x + 3y = 21
Hence x = – 6, y = –1 Ans.
–6x + 8y = 12
1
30. Let M × 0
LM 1OP
2 = [1 2]1×2 where M is a On adding, 11y = 33  y = 3
N Q
matrix. From (i), 2x = 7 – 3 = 4  x = 2
(i) State the order of the matrix M. 2
(ii) Find the matrix M. So, X = 3
 
Sol. (i) M must be of the order 1 × 2
32. Solve the matrix equation :
Let M = [x y]
LM41OP X  LM41 82OP
y × L0 2O  NQ N Q
1 1
 x 1 2
NM PQ L 4 O L 4 8 O
 [x + 0 x + 2y]  1 Sol. Given, M 1 P X  M 1 2 P
2 NQ N Q
Comparing the corresponding elements Let matrix X = [x, y]
181 Arun Deep's Understanding Math-10

LM4 OP [ x , y ]  LM4 OP 4

8  y
N1Q N 1 2 Q 3
LM 13 OP

LM4 x 4 y OP  L4 8O 6
N x y Q MN 1 2 PQ  Matrix C = M 4 P Ans.
Comparing the corresponding elements.
MN 3 PQ
4x = – 4  x=–1 2  1
4y = 8  y=2 (ii) A =  4 5  and B = [0 – 3]

 X = [–1 2] Ans. Let matrix C = [x y]1×2
33. (i) If A  LM 2 1OP and B  L3O , find Since the matrix A is 2 × 2 and B = 7 × 2
N 4 5 Q MN 2 PQ  CA = B
matrix C such that AC = B.
2 1
 2  1  [x y]  4 5 
= [0 –3]
(ii) If A =  4 5  and B = [0, – 3], find the 
   [2x – 4y – x + 5y] = [0 –3]
matrix C such that CA = B.
On comparing, we have
Sol. (i) Given, A 
LM 2 1OP, B  LM3OP 2x – 4y = 0  x – 2y = 0
N 4 5 Q N 2 Q 
x = 2y
Let the matrix C  L x O and –x + 5y = – 3  –2y + 5y = – 3
MN y PQ  3y = –3  y = –1
 x = 2y = 2 × (–1) = –2
2 1O L x O L 2 x  y O
AC  LM

N4 5PQ MN y PQ  MN4 x  5 y PQ Hence C = [x y] = [–2 –1]

But AC = B  3  4
34.. If A=  1 2  , find matrix B such that
LM 2 x  y OP  L 3O  

N 4 x  5 y Q MN 2 PQ BA = I, where I is unity matrix of order 2.

Comparing the corresponding elements ; 3  4


we have Sol.A  1 2  and BA = I, where I is the unity

2x–y=–3 ...(i)
matrix of order 2
–4x+5y=2 ...(ii)
Multiplying (i) by 5 and (ii) by 1 1 0
10 x – 5 y = – 15  I = 0 1

– 4 x + 5 y= 2
13 a b
On adding, we get 6 x = – 13  x  Let B =  c
6  d 
Substituting the value of x in (i) ; we have
a b 3  4
2
FG 13 IJ  y  3  13  y  3  B A = c
 d  ×  1
 2 
H 6K 3
13 9  13 4 3a  b  4a  2b 
  y  3   
3 3 3 = 3c  d  4c  2d 

182 Arun Deep's Understanding Math-10

3a  b  4a  2b  1 0  4 p  2 r 4q  2 s   6 0 
  pr 
 3c  d


 4c  2 d  = 0
 1   q  s   0 6 

Comparing the corresponding terms, we get  4p + 2r = 6 ...(i) –p + r = 0 ...(ii)


3a – b = 1, –4a + 2b = 0 4q + 2s = 0 ...(iii) and –q + s = 6 ...(iv)
 2b = 4a  b = 2a On solving equation (i) and (ii), we get, p = 1
 3a – b = 1  3a – 2a = 1  a = 1 and r = 1 and On solving eq. (ii) and (iv)
we get, q = –2 and s = 4
and b = 2a  b = 2 × 1 = 2
 a = 1, b = 2 p q   1 2 
 Matrix M = 
s   1 4 
= Ans.
and 3c – d = 0  d = 3c r
–4c + 2d = 1  –4c + 2 × 3c = 1
1  4 2  17 1 
 –4c + 6c = 1  2c = 1  c = 36. If B =   and C = 
 13 
2 ,
 5  1  47
1 3 find matrix A such that AB = C.
and d = 3c = 3 × =
2 2
 4 2 17 1 
1 3 Sol. Given, B  5
Hence a = 1, b = 2, c = ,d=   1 , C = 47  13 and
2 2
1 2 AB = C
 Matrix B =  1 3
 2 2  a b
Let A =  c d 
 4 2 
35. Given   M = 6I, where M is a matrix
 –1 1  a b  4 2
and I is unit matrix of order 2 × 2. Then AB =  c d  ×  5  1

(i) State the order of matrix M.
(ii) Find the matrix M.   4a  5b 2a  b 
=  4c  5d 2c  d 
Sol.(i) Let order of matrix M = a × b 
 4 2 AB = C
   M = 6I
 1 1    4a  5b 2a  b  17 1 
  4c  5d 
2c  d  = 47  13
 4 2  1 0  
  1 1  × Ma×b = 6 ×  0 1
  22   22 Comparing corresponding elements, we get
 a = 2 and b = 2 –4a + 5b = 17 ....(i)
 order of matrix M = a × b = 2 × 2 Ans. 2a – b = –1 ....(ii)
–4c + 5d = 47 ....(iii)
p q
(ii) Let matrix M = 
s 
2c – d = –13 ....(iv)
r
Multiplying (i) by 1 and (ii) by 2
 4 2  p q 1 0  
  1 1   r   6   –4a + 5b = 17
  s 0 1  4a – 2b = –2
183 Arun Deep's Understanding Math-10
15 (c) x = –2, y = 3 (d) x = 3, y = 2
On adding ; 3b = 15 b= =5
3 x  2y  y  4 3 
2a – b = –1  2a – 5 = –1 2a = –1 + 5 Sol. Given,  3 x 4 =  
  6 4
4 On comparing, we get
=4a= =2
2
6
 a = 2, b = 5 3x = 6  x = =2
3
Again multiplying (iii) by 1 and (iv) by 2,
–4c + 5d = 47 –y = 3  y = –3
4c – 2d = –26  x = 2, y = –3  Ans. (b)
21 x  2 y 5 6 5 
On adding 3d = 21  d = =7 3. If  3  =   , then the value
3
 y 3  2
and 2c – d = –13  2c – 7 = –13
of x is
6 (a) –2 (b) 0 (c) 1 (d) 2
 2c = –13 + 7 = –6  c = = –3
2
 c = –3, d = 7 x  2 y 5  6 5 
Sol. Given,  3   
y  = 3  2

2 5
Thus, required matrix A =  3 7  On comparing, we get

y = –2
MULTIPLE CHOICE QUESTIONS and x – 2y = 6  x – 2 × (–2) = 6
Choose the correct answer from the given four  x+4=6x=6–4=2  Ans. (d)
oprtions (1 to 14) :
 x  2 y 3 y  0  3
x  3 4  5 4 4. If  4 x   
2  = 8 2  , then the value
1. If  y  4 x  y  = 3 9 , then the values 
    of x – y is
of x and y are
(a) –3 (b) 1 (c) 3 (d) 5
(a) x = 2, y = 7 (b) x = 7, y = 2
(c) x = 3, y = 6 (d) x = –2, y = 7 x  2 y 3y 0  3
Sol. Given,  4 x  = 
2

 8 2 
x  3 4  5 4
Sol. Given,  y  4  = 
x  y
 On comparing, we get
 3 9
On comparing, we get 3
3y = –3  y = = –1
x+3=5x=5–3=2 3
and y – 4 = 3  y = 3 + 4 = 7 8
 x = 2, y = 7 Ans. (a) and 4x = 8  x = =2
4
x  2y  y  4 3   x – y = 2 – (–1) = 2 + 1 = 3 Ans. (c)
2. If  3 x
 4  =  6 4 , then the
2  1 10
values of x and y are 5. If x 3 + y  0  =  6  , then the values of
     
(a) x = 2, y = 3 (b) x = 2, y = –3 x and y are
184 Arun Deep's Understanding Math-10
(a) x = 2, y = 6 (b) x = 2, y = –6 On adding (i) and (ii), we get
(c) x = 3, y = –4 (d) x = 3, y = –6 2 0   1 1  1 1
3A = 2 2 +  0  1 = 2 1
2  1 10      
Sol. Given, x 3 + y  0  =  6 
     
1 1 1
 A=    Ans. (a)
2 x   y  10 3 2 1
 3 x  +  0  =  6 
      1 0
P.Q. If A = 0 1 , then A2 =
2 x  y  10  
  3x  0  =  6 
    1 1 0 0  1 0 0 1 
(a) 0 0 (b) 1 1 (c) 0 1 (d) 1 0
       
2 x  y  10
  3x  =  6  1 0
   
Sol. Given, A = 0 1
Comparing, we get  

6 1 0 1 0
3x = 6  x = =2 A2 = A . A =  0 1   0 1 
3    
and 2x – y = 10 1  0 0  0 1 0
2 × 2 – y = 10  4 – y = 10 = 0  0 0  1  = 0 1   Ans. (d)
   
 –y = 10 – 4 = 6  y = –6
0 1 
 x = 2, y = –6  Ans. (b) 6. If A = 1 0 , then A2 =
 
1 0  1 1 
P.Q. If A + B = 1 1 and A – 2B =  0  1 , 1 1 0 0 
    (a) 0 0 (b) 1 1
then A is equal to    

1 1 1 1 2 1  0 1  1 0
(a)   (b)   (c) 1 0 (d) 0 1
3 2 1 3 1 2     

1 1 2 1  0 1 
(c) 2 1 (d) 1 2 Sol. Given, A = 1 0
     

1 0  1 1  0 1  0 1 
Sol. Given, A + B = 1 1 , A – 2B =  0  1  A2 = A . A = 1 0 1 0
       

 0  1 0  0 1 0
2 0  = 0  0 1  0  = 0 1   Ans. (d)
 2A + 2B = 2 2 (Multiplying by 2) ...(i)    
 
0 0 
 1 1  7. If A = 1 0 , then A2 =
and A – 2B =  0  1 ...(ii)  
 
(a) A (b) O (c) I (d) 2A
185 Arun Deep's Understanding Math-10

0 0  9  ( 1) 3  2   8 5
Sol. Given, A = 1 0 =   3  2  1  4 =  5 3 (a)
     

0 0  0 0   2  2
 A2 = A A = 1 0 1 0 10. If A =  2 2  , then A2 = pA, then value
     
of p is
0  0 0  0  0 0  (a) 2 (b) 4 (c) –2 (d) –4
= 0  0 0  0 = 0 0 = O  Ans. (b)
     2  2
Sol. A =  2 2  and A2 = pA
1 0  
8. If A = 1 1 , then A2 =
   2  2  2  2
A2 = A A =   2 2    2 2 
2 0 1 0    
(a) 1 1 (b) 1 2
     4  4  4  4  8  8
=  4  4 4  4  =  8 8 
1 0     
(c) 2 1 (d) none of these
   2  2  2 p  2 p
and pA = p  2 2  =  2 p 2 p 
1 0    
Sol. Given, A = 1 1 2
A = pA
 
 8  8  2 p  2 p
1 0 1 0   8 8  =  2 p 2 p 
 A2 = A . A = 1 1 1 1    
    Comparing, we get
8 = 2p  p = 4 (b)
1  0 0  0 1 0 
= 1  1 0  1 = 2 1  Ans. (c) CHAPTER TEST
   
1. Find the values of a and b if
 3 1
9. If A =  1 2 , then A2 =
  LM a  3 b 2  2  2 a  1 3b
OP LM OP
N0 6 Q N
0 b 2  5b Q
 8 5 8  5 Sol. Given,
(a)  5 3 (b) 5 3 
   
LM a  3 b 2  2  2 a  1 3b
OP LM OP
 8  5  8  5 N0 6 Q N
0 b 2  5b Q
(c)  5  3 (d)  5 3  Comparing the corresponding elements, we
   
have
 3 1 a+3=2a+12a–a=3–1
Sol. Given, A =  1 2  a=2
 
b2 + 2 = 3 b  b2 – 3 b + 2 = 0
 3 1  3 1 b2 – b – 2 b + 2 = 0
A2 = A . A =   1 2    1 2 
     b (b – 1) – 2 (b – 1) = 0
186 Arun Deep's Understanding Math-10
 (b – 1) (b – 2) = 0. 3. Determine the matrices A and B when
Either b – 1 = 0, then b = 1 LM
1 2 OP 2 1 LM OP
or b – 2 = 0, then b = 2 A + 2 B  6 3 and 2 A – B  2 1
Hence a = 2, b = 2 or 1 Ans.
N Q N Q
2. Find a, b, c and d if 3 a b LM OP Sol.
L 1 2O
Given, A + 2 B  M6 3P , ...(i)
c d N Q N Q
LM4 a  bO L a L2 1O

Nc  d 3 PQ  MN 1 2d6OPQ 2 A – B  M2 1P
N Q ...(ii)
Sol. Given,
Multiplying (i) by 1 and (ii) by 2
a  bO L a
3 LM
a b O L4 6O
Nc d PQ  MNc  d 3 PQ  NM 1 2d QP LM
1 2
A + 2 B  6 3
OP
LM3 a 3b OP  L4  a a  b  6O N Q

N 3 c 3 d Q MNc  d  1 3  2 d PQ 2 1
LM 4 2
4 A – 2 B  2 2 1  4 2
OP LM OP
Comparing the corresponding elements ; N Q N Q
we get
On adding, we get
3a=4+ a3a–a=42a=4
 a=2
LM1 2 OP LM OP LM OP
4 2 5 0
3b=a+b+63b–b=2+6 5 A  6 3  4 2  10 5
N Q N Q N Q
 2b=8
 b=4 A  LM
1 5 0O L 1 0O

3d=3+2d3d–2d=3 5 N10 5QP NM2 1QP
 d=3
3c=c+d–13c–c=3–1 L 1 2O
From (i) ; A + 2 B  M6 3P
2c=2c=1
N Q
Hence a = 2, b = 4, c = 1, d = 3.

LM 1 0OP + 2 B  LM 1 2 OP
3 2
PQ. Find X if Y  1 4
LM OP and 2 X + N2 1Q N6 3Q
N Q
2 B  LM
1 2 O L 1 0O L 0 2 O
LM
1 0
Y  3 2 OP N6 3PQ  MN2 1PQ  MN4 2 PQ
N Q
Sol. Given,
1 0 2 O  L 0 1O
LM 1 0OP  2 X  L 1 0O – Y  B  LM
2X+Y  2 N4 2 QP MN2 1PQ
N 3 2 Q MN3 2PQ
 2 X  LM
1 0O L3 2 O Hence A  M
L 1 0OP and B  LM 0 1OP
N 2PQ – MN 1 4 PQ
 3 N2 1Q N2 1Q
L 1  3 0  2 O L2 2 O
= M 3  1 2  4 P  M 4 2 P L4 1O
N Q N Q 4.(i) Find the matrix B if A  M2 3P and
N Q
1 2 2 O L 1 1O
X  LM  Ans. A2 = A + 2 B.
2 N 4 2 PQ MN 2 1PQ
187 Arun Deep's Understanding Math-10

Sol. Given, A  LM4 1OP , Let B  LMa bOP Sol. Given, A =


LM 1 2 OP , B = LM 0 1OP ,
N 2 3Q Nc dQ N  3 4 Q N  2 5Q
L4 1O L4 1O
A = A × A  M2 3P M 2 3P
2
N QN Q C=
LM 2 0OP
16  2 4  3O L18 7 O
N  1 1Q
 LM
N 8  6 2  9QP  NM14 11QP 4 B  3C = 4 M
L 0 1OP  3 LM2 0OP
L4 1O La b O N 2 5Q N 1 1Q
and A + 2 B  M2 3P + 2 M c d P
N Q N Q M
L 0 4OP  LM 6 0OP
4 1O L2 a 2 b O
N 8 20Q N 3 3Q
 LM
N2 3PQ  MN 2 c 2 d PQ M
L 0  ( 6) 4  0OP  LM 0  6 4  0OP
L4  2 a 1  2 bOP N8  ( 3) 20  3Q N8  3 20  3Q
=M
N2  2c 3  2dQ M
L 6 4 OP
... A2 = A + 2 B N5 17Q
LM18 7 OP = LM4  2 a 1 2b OP L 1 2O L 6 4O
Now A (4 B – 3 C)  M 3 4 P M 5 17 P
 N QN Q
N14 11Q N 2  2 c 3  2d Q
Comparing the corresponding elements ;
M
L 1  6  2 ( 5) 1  4  2  17OP
we have N 3  6  4  ( 5) 3  4  4  17Q
4 + 2 a = 18  2 a = 18 – 4 = 14
 a=7 M
L 6  10 4  34OP
1+2b=7 2b=7–1=6
N18  20 12  68Q
 b=3
M
L 4 38OP Ans.
2 + 2 c = 14  2 c = 14 – 2 = 12 N38 56Q
 c=6
A  LM
1 4O L2 1O

and 3 + 2 d = 11  2 d = 11 – 3 = 8
d=4
PQ. If
N1 0PQ , B  MN 3 1PQ and
C  LM
2 3O
Hence a = 7, b = 3, c = 6, d = 4
N 0 5PQ , compute (AB) C and (CB) A.
 B
LM7 3OP Ans. Is (AB) C = (CB) A ?
N6 4Q LM L1 4 O ×L2 1O OP L2 3O
Sol. (AB) C 
L 1 2O L 0 1O
(ii) If A = M  3 4 P , B = M  2 5P and
N MN1 0PQ MN 3 1PQ Q MN 0 5PQ
N Q N Q 2  12 1  4 O L 2 3O
L 2 0OP = LM
C=M N2  0 1  0PQ MN 0 5PQ
N  1 1Q
14 3O L 2 3O
find A (4 B – 3 C). = LM
N 2 1PQ MN 0 5PQ
188 Arun Deep's Understanding Math-10
2 2
LM28  0 42  15
OP LM OP
28 27 (ii) A – B
= 
N 40 65
Q N Q 4 11 3 2 3 2 1 0 1 0
= 0 5 0 5 – 1 
2 × 1 2
 
L L2 3O L2 1O O L1 4 O
and (CB) A  M M 0 5P M 3 1P P M1 0P
NN Q N QQ N Q 9  0 6  10  1  0 0  0
= 0  0 0  25 – 1  2 0  4
 
4  9 2  3O L1 4 O
= LM
N0  15 0  5PQ MN1 0PQ 9
= 0
16 

1
3
0

9  1
0  3
16  0 
 25 –
 4 =
 25  4
13 1O L1 4 O L13  1 52  0 O
= LM
N15 5PQ MN1 0PQ = MN15  5 60  0PQ 8 16 
=  3 21
 
= LM
12 52 O
We see that, (A + B) (A – B)  A2 – B2
N10 60PQ
It is clear from above that 6. If A = LM 3 5OP , find A 2 – 5 A – 14I.
N 4 2 Q
(AB) C  (CB) A where I is unit matrix of order 2 × 2.

3 5 LM OP LM 3 5OP
3 2 1 0 Sol. Given, A2 = A × A = 4 2
5. If A 
0

5 and B = 
1 2  , find each of
N Q N 4 2Q
9  20 15  10O L 29 25O
the following and state it they are equal : = LM
(i) (A + B) (A – B) (ii) A2 – B2. N 12  8 20  4PQ = MN 20 24PQ

Solution—
3 5
5A =5 L
3 2 1 0
and
MN4 2OPQ = LMN 2015 1025OPQ
Given, A = 0 5 , B = 1 2
    A2 – 5 A – 14 I
(i) (A + B) (A – B)
= LM 29 25OP  LM2015 1025OP 14 LM01 01OP
3 2 1

0  N 20 24Q N Q N Q
= 0 5 1

2 
 29 25O L 15 20O L14 0 O
 LM
3 2 1 0 
N20 24 PQ  MN20 10PQ  NM 0 14QP
  
0 5 1 2  L 29  15  14 25  25  0OP
M
3  1 2  0 3  1 2  0
N20  20  0 24  10  14 Q
= 0  1  × 0  1 5  2  LM
 5  2  0 0O
N0 0PQ = 0
4 2  2 2
= 1 7  1 3 L 3 3O
 7. If A  M p q P and A = O, find p and q.
2
N Q
8  2 86  6 14 
= 2  7


2  21 =  5
 23 Sol.
L 3 3O L 3 3O
 A = A × A  M p qP M p qP
2
N QN Q
189 Arun Deep's Understanding Math-10


LM9  3 p 9  3q OP y
6 5 3
× 
N3 p  pq 3p  q2 Q 25 2 5

But A2 = O 8 3
Hence x  , y Ans.
5 5
LM 9  3 p 9  3q OP  L 0 0 O

N 3 p  pq 3p  q 2
Q MN 0 0 PQ 8. If
LM 1 0OP LMa bOP  LM 1 0OP , find a, b, c
N 0 1Q N c d Q N0 1Q
Comparing the corresponding elements and d.
9+3p=03p=–9p=–3
9+3q=03q=–9q=–3 Sol. Given,
LM 1 0OP LMa bOP  LM 1 0OP
N 0 1Q N c d Q N0 1Q
Hence p = – 3, q = – 3 Ans.

LM a  0 b  0OP  LM 1 0OP
L3 2 OP N 0  c 0  d Q N0 1Q
PQ. If A  M 5 5 and A2 = I, find x, y.
MN x y PQ LM a b OP  LM 1 0OP

3 LM 2 OP LM 3 2 OP N c d Q N0 1Q
Sol. A = A A  5
2 . 5 5 5 Comparing the corresponding elements
x MN y PQ MN x y PQ
–a=1 a=–1
LM 9  2 x 6
 y
2 OP –b=0b=0
 M 25 5 25 5
PP c = 0 and d = – 1
MN 53 x  xy 2
5
x  y2
Q Hence a = – 1, b = 0, c = 0, d = – 1 Ans.
1 0
But A2 = I  0 1
LM OP 9. Find a and b if
N Q
LMa  b b4 OP LM2 0OP  LM2 2 OP
LM 9  2 x 6
 y
2 OP Nb  4 a2 Q N 0 2 Q N 14 0Q
 MM 253 5 25 5
2

1 0
0 1 PP LMN OPQ La  b b  4 O L2 0O L2 2 OP
Sol. Given, Mb  4 a  2 P M 0 2 P  M 14
N 5 x  xy 5
x  y2
Q N Q N Q N 0Q
Comparing the corresponding elements, we  2 a  2 b  0 2b  8  L 2 2 O
have   2 b  8  0 0  2 a  4   M 14 0P
  N Q
9 2 2 9 16
 x  1  x  1   2 a  2 b 2b  8  L 2 2 O
25 5 5 25 25   2 b  8 2 a  4   M 14 0 P
  N Q
16 5 8
x ×  Comparing the corresponding elements
25 2 5
2 a – 4 = 0  2 a = 4  a = 2
6 2 2 6
and  y0  y
25 5 5 25 2a–2b=–2 2×2–2b=–2
190 Arun Deep's Understanding Math-10
 4 – 2 b = – 2  – 2 b = – 2 – 4 = – 6
 b=3
Hence a = 2, b = 3.

10. If A 
LMsec 60 cos 90
OP, and
N3 tan 45 sin 90Q
LM 0 cot 45 OP
B
N 2 3 sin 90 Q
find (i) 2 A – 3 B (ii) A2 (iii) BA

Sol. Given, A
LMsec 60 cos 90

OP L
2 0 OP
N3 tan 45 sin 90 Q MN
3 1 Q
(... sec 60° = 2, cos 90° = 0, tan 45° = 1, sin 90° =1)

and B  LM 0 OP LM
cot 45  0 1
2 3
OP (... cot 45° = 1)
N 2 3 sin 90 Q N Q
(i)  2 A – 3 B

2 LM 2 0OP  3 LM 0 1OP  LM 4 0OP  LM 0 3OP  LM 4  0 03 4 3


OP LM
 OP
N3 1Q N2 3Q N6 2Q N6 9Q N 6  6 29 0 7
Q N Q
(ii) 2 L 2 0O L 2 0O
 A = A. A  M 3 1P M 3 1P
N QN Q
4  0 0  0 O L 4 0O
 LM
N6  3 0  1PQ  MN9 1PQ
(iii)
L 0 1O L 2 0O
BA  M 2 3P M 3 1P
N QN Q
0  3 0  1O 3 1 O
 LM  LM
N  4  9 0  3Q N13 3PQ Ans.
P
9
Arithmetic and Geometric Progressions
Points to Remember : n 1
1. Arithmetic Progression (A.P.) : A list of th term is called the middle term of an
2
numbers in which each term is obtained by
A.P.
adding a fixed number to its preceding term,
6. For convenience, we take
except the first term, is called an arithmetic
(i) three numbers as a – d, a, a + d
progression (abbreviated A.P.). In other
words, a list of numbers is called an (ii) Four numbers as a – 3d, a – d, a + d, a + 3d
arithmetic progression (A.P.) if and only if (iii) Five numbers as a – 2d, a – d, a, a + d,
the difference of any term from its preceding a + 2d, etc.
term is constant i.e. a fixed number. If a, b, c are in A.P., then b – a = c – b
This constant (a fixed number) is usually ac
denoted by d and is called common  2b = a + c  b = .
2
difference. Note that this fixed number d 7. Sum of first n terms of an A.P. :
may be positive, zero or negative.
First, second, third, ... terms of an A.P. are n
Sn = [2a + (n – 1)d]
respectively denoted by a1, a2, a3, ... or by 2
t1, t2, t3, ... . n
Thus, a1, a2, a3, a4, ... is an A.P. if and only or (a + l) where l is the last term.
2
if an + 1 – an = d, a constant (independent of
(i) an = Sn – Sn – 1
n) i.e. an + 1 = an + d (except the first term).
The first term of an A.P. is also denoted by n(n  1)
(ii) Sum of first n natural numbers = .
a. 2
2. General term of an A.P. : Geometric Progression : A list of non-zero
an = a + (n – 1)d is the nth term or the numbers is called a geometric progression
general term of an A.P. (G.P.) if and only if the ratio of its any term
3. Last term : l = a + (m – 1)d is the last term to the proceding term is constant or fixed.
of an 'A'. a is called the first term and r is called the
4. nth term from the end of a finite A.P. : common ratio.
= a + (m – n)d an 1
Where an A.P. consists of m terms, with a r= a
n
as first term and d as common difference.
General term of a geometric progression is
5. Difference of any two terms of an A.P. : arn – 1 and last term (l) = arn – 1.
The difference between two terms say mth nth term from the end of a finite G.P. :
term and nth term
The nth term from the end of a finite G.P.
an – am = (n – m)d
= arm – n
191 Arun Deep's Understanding Math-10
192 Arun Deep's Understanding Math-10
 d = 0.2
b c
If a, b, c are in G.P. then =  b2 = ac. 2. Write first four terms of the A.P., when the
a b
first term a and the common difference d
Sum of first n term of a G.P. : are given as follows :
Sn of first n terms of a G.P.
(i) a = 10, d = 10 (ii) a = –2, d = 0
a (1  r n ) 1 1
Sn = if r < 1 (iii) a = 4, d = –3 (iv) a = ,d= 
1 r 2 6
a(r n 1 ) Sol. (i) Given, a = 10, d = 10
= if r > 1
r 1 Given, A.P. be a, a + d, a + 2d, ....
 A.P. be, 10, 20, 30, 40, ...
EXERCISE 9.1
(ii) Given, a = –2, d = 0
1. For the following A.P.s, write the first term  Required A.P. be, –2, –2, –2, –2, ...
a and the common difference d : (iii) Given, a = 4, d = –3
(i) 3, 1, –1, –3, ...  Required A.P. be, a, a + d, a + 2d, ....
1 5 9 13 i.e. 4, 4 – 3, 4 – 6, ...
(ii) , , , , ...
3 3 3 3  A.P. becomes, 4, 1, –2, –5, ...
(iii) –3.2, –3, –2.8, –2.6, ... 1 1
Sol. (i) 3, 1, –1, –3, ... (iv) Given, a = ,d= 
2 6
Here first term (a) = 3
1 1 1 2
and common difference (d) A.P. is ,    =
= 1 – 3 = –2, 2 2 6 6
= – 1 – 1 = –2, ... 2 1 1
= –2 = – = , ...
6 6 6
1 5 9 13 1 2 1
(ii) , , , , ...  A.P. becomes, , , , 0, ...
3 3 3 3 2 6 6
1 1 1 1
Here first common term (a) = i.e. , , , 0, ...
3 2 3 6
and common difference (d) 3. Which of the following lists of numbers form
5 1 4 9 5 4 an A.P.? If they form an A.P., find the
= – = , – = , ... common difference d and write the next
3 3 3 3 3 3
three terms :
4 (i) 4, 10, 16, 22, ... (ii) –2, 2, –2, 2, ...
=
3
5 7
(iii) –3.2, –3, –2.8, –2.6, ... (iii) 2, 4, 8, 16, ... (iv) 2, , 3, , ...
2 2
Here first term (a) = –3.2
(v) –10, –6, –2, 2, ... (vi) 12, 32, 52, 72, ...
and common difference (d)
(vii) 1, 3, 9, 27, ...
= –3 – (–3.2) = –3 +3.2 = 0.2
193 Arun Deep's Understanding Math-10

(viii) 2 , 8, 18 , 32 , ... 9
and next three terms are 4, ,5
2
(ix) 3, 3 + 2 , 3 + 2 2 , 3 + 3 2 , ...
(v) –10, –6, –2, 2, ...
(x) 3, 6, 9, 12 , ... Here, first term (a) = –10
(ix) a, 2a, 3a, 4a, ... d = –6 – (–10) = –6 + 10 = 4
(x) a, 2a + 1, 3a + 2, 4a + 3, ... –2 – (–6) = –2 + 6 = 4
Sol. (i) 4, 10, 16, 22, ... 2 – (–2) = 2 + 2 = 4
Here, a = 4, d = 10 – 4 = 6, 16 – 10 = 6,  Common difference is same.
and 22 – 16 = 6.  It is an A.P.
Common difference is same and next three terms are 2 + 4, 2 + 4 + 4,
 It forms an A.P. 2 + 4 + 4 + 4, i.e. 6, 10, 14, 20.
(vi) 12, 32, 52, 72, ...
and next three terms are 28, 34, 40
= 1, 9, 25, 49, ...
(ii) –2, 2, –2, 2, ...
Here, first term (a) = 12 = 1
Here, a = –2
d=9–1=8
d = 2 – (–2) = 2 + 2 = 4
25 – 9 = 16
–2 – 2 = –4
49 – 25 = 24
2 – (–2) = 4
Common difference is not same.
Common difference is not same.
 It is not an A.P.
 It does not forms an A.P.
(vii) 1, 3, 9, 27, ...
(iii) 2, 4, 8, 16, ...
Here, first term (a) = 1
Here, a = 2 d=3–1=2
d = 4 – 2 = 2, 8 – 4 = 4, 16 – 8 = 8 9–3=6
 Common difference is not same. 27 – 9 = 18
 It is not an A.P. Common difference is not same.
 It is not an A.P.
5 7
(iv) 2, , 3, , ...
2 2 (viii) 2 , 8, 18 , 32 , ...
Here a = 2,  2 , 2 2 , 3 2 , 4 2 , ...
5 1
d= –2= Here, first term (a) = 2
2 2
and common difference (d)
5 1
3– = =2 2 – 2 = 2
2 2
=3 2 –2 2 = 2
7 1
–3=
2 2 =4 2 –3 2 = 2
Common difference is same. The common difference is same.
 It is an A.P.  It is an A.P.
194 Arun Deep's Understanding Math-10
and next three terms are 4 2  2, and common difference (d)
= 2a + 1 – a = a + 1
4 2  2 2, 4 2  3 2
3a + 2 – 2a – 1 = a + 1
i.e. 25  2, 36  2, 49  2 4a + 3 – 3a – 2 = a + 1
Common difference is same.
50 , 72 , 98 , ...
 It is an A.P.
(ix) 3, 3 + 2 , 3 + 2 2 , 3 + 3 2 , ... and three next terms are
Here, first term (a) = 3 5a + 4, 6a + 5, 7a + 6, ...
and d = 3 + 2 –3= 2 EXERCISE 9.2
=3+2 2 –3– 2 = 2 1. Find the A.P. whose nth term is 7 – 3n. Also
find the 20th term.
=3+3 2 –3+2 2 = 2
Sol. Given, Tn = 7 – 3n
Common difference is same.
Giving values 1, 2, 3, 4, ... to n, we get
 It is an A.P.
T1 = 7 – 3 × 1 = 7 – 3 = 4
and next three terms are
T2 = 7 – 3 × 2 = 7 – 6 = 1
3 + 4 2 , 3 + 5 2 , 3 + 6 2 , ...
T3 = 7 – 3 × 3 = 7 – 9 = –2
(x) 3, 6, 9, 12 , ... T4 = 7 – 3 × 4 = 7 – 12 = –5
T20 = 7 – 3 × 20 = 7 – 60 = –53
Here, a = 3  A.P. is 4, 1, –2, –5, ...
d= 6 – 3 =( 3 × 2 – 3) and 20th term = –53
2. Find the indicated terms in each of following
= 3 ( 2 –1) A.P.s :
= (i) 1, 6, 11, 16, ...; a20
9 – 6 =3– 2 3= 3( 3 – 2)
(ii) –4, –7, –10, –13, ..., a25, an
 Common difference is not same.
Sol. (i) 1, 6, 11, 16, ...
 It is not an A.P.
Here, a = 1, d = 6 – 1 = 5
(xi) a, 2a, 3a, 4a, ...
 a20 = a + (n – 1)d = 1 + (20 – 1) × 5
Here first term = a
= 1 + 19 × 5 = 1 + 95 = 96
Common difference (d) = 2a – a = a
(ii) –4, –7, –10, –13, ..., a25, an
3a – 2a = a
Here, a = –4, d = –7 – (–4) = –7 + 4 = –3
4a – 3a = a
a25 = a + (25 – 1)d = –4 + (25 – 1) × –3
The common difference is same.
= –4 + 24 × (–3) = –4 – 72 = –76
 It is an A.P.
and next three terms are and an = a + (n – 1)d = –4 + (n – 1) (–3)

5a, 6a, 7a = –4 – 3n + 3 = –1 – 3n = –3n – 1


(xii) a, 2a + 1, 3a + 2, 4a + 3, ... 3. Find the nth term and the 12th term of the
list of numbers : 5, 2, –1, –4, ...
Here first term (a) = a
195 Arun Deep's Understanding Math-10
Sol. 5, 2, –1, –4, ... 5. Which term of the A.P.
Here, a = 5 (i) 3, 8, 13, 18, ... is 78 ?
d = 2 – 5 = –3 1
(i)  Tn = a + (n – 1)d = 5 + (n – 1) (–3) (ii) 18, 15 , 13, ... is –47 ?
2
= 5 – 3n + 3 = 8 – 3n (iii) 7, 13, 19, ... is 205 ?
(ii)  T12 = a + 11d = 5 + 11(– 3) Sol. (i) 3, 8, 13, 18, ... is 78
= 5 – 33 = –28 Let be the is nth term of given A.P.
P.Q. Find the 8th term of the A.P. whose first Here, a = 3, d = 8 – 3 = 5
term is 7 and common difference is 3.
 78 = a + (n – 1)d
Sol. First term (a) = 7
 78 = 3 + (n – 1)5
and common difference (d) = 3
 78 = 3 + 5n – 5
 A.P. = 7, 10, 13, 16, 19, ...
 78 + 5 – 3 = 5n  5n = 80
We know that
Tn = a + (n – 1)d T8 = 7 + (8 – 1) × 3 80
 n= = 16
= 7 + 7 × 3 = 7 + 21 = 28 5
4. (i) If the common difference of an A.P. is  78 is the 16th term of given A.P.
–3 and the 18th term is –5, then find its first
1
term. (ii) 18, 15 , 13, ... is –47
2
(ii) If the first term of an A.P. is –18 and its
10th term is zero, then find its common Let nth term of given A.P. is –47
difference.
1 1 5
Sol. (i) Common difference (d) = –3 a = 18, d = 15 – 18 = –2 =
2 2 2
T18 = –5
Since Tn = –47
a + (n – 1)d = Tn
 –47 = a + (n – 1)d
 a + (18 – 1) (–3) = –5
 a + 17(–3) = –5  a – 51 = –5  5
 –47 = 18 + (n – 1)  
 a = –5 + 51 = 46  2 
 First term = 46
5 5
(ii) First term (a) = –18  –47 – 18 = n+
2 2
T10 = 0
We know that 5 5 135 5
 –65 – = n = n
2 2 2 2
a + (n – 1)d = Tn
 –18 + (10 – 1)d = T10 = 0 2
 n= × = 27
 –18 + 9d = 0  9d = 18 135 5
2
18
 d= =2  –47 is the required 27th term of given A.P.
9
(iii) 7, 13, 19, ... is 205
 Common difference = 2
Let nth term of given A.P. is 205
196 Arun Deep's Understanding Math-10
Here, a = 7, d = 13 – 7 = 6 Let 0 be the nth term, then
205 = a + (n – 1)d Tn = a + (n – 1)d
 205 = 7 + (n – 1) × 6  0 = 31 + (n – 1)(–3)
 205 = 7 + 6n – 6  0 = 31 – 3n + 3  3n = 34
 6n = 205 – 7 + 6 = 204 34 1
n= = 11
1
204 3 3
 n= = 34 Hence 0 is not any term of the A.P.
6
 205 is 34th term. 7. (i) Find the 20th term from the last term of
the A.P. 3, 8, 13, ..., 253.
6. (i) Check whether –150 is a term of the A.P.
11, 8, 5, 2, ... (ii) Find the 12th from the end of the A.P. –2,
–4, –6, ..., –100.
(ii) Find whether 55 is a term of the A.P. 7, 10,
13, ... or not. If yes, find which term is it. Sol. (i) A.P. is 3, 8, 13, ..., 253
(iii) Is 0 a term of the A.P. 31, 28, 25, ... ? Justify 12th term from the end
your answer. Last term = 253
Sol. (i) A.P. is 11, 8, 5, 2, ... Here, a = 3, d = 8 – 3 = 5
Here, a = 11, d = 8 – 11 = –3  Last term (Tn) = a + (n – 1)d
Let –150 = Tn, 253 = 3 + (n – 1) × 5
Also, Tn = a + (n – 1)d  253 = 3 + 5n – 5
 –150 = 11 + (n – 1)(–3)  253 – 3 + 5 = 5n
 –150 = 3 – 3n + 11 255
 5n = 255  n = = 51
 3n = 3 + 150 + 11 = 153 + 11 = 164 5
 253 is 51th term
164 2
n= = 54 Let Tm be the 20th term from the last term
3 3
Then Tm be the 20th term from the last term
No, –150 is not any term of the given A.P. Then Tm = l – (m – 1)d = 253 – (20 – 1) × 5
(ii) A.P. 7, 10, 13, ... = 253 – 19 × 5
Here, a = 7, d = 10 – 7 = 3 = 253 – 95 = 158
Let 55 is the nth term, then  20th term from the end = 158
Tn = a + (n – 1)d (ii) A.P. = –2, –4, –6, ..., –100
a = –2, d = –4 – (–2) = –4 + 2 = –2
 55 = 7 + (n – 1) × 3
l = –100
 55 = 7 + 3n – 3  3n = 55 – 7 + 3 = 51
 Tn = a + (n – 1)d
51  –100 = –2 + (n – 1) × (–2)
 n= = 17
3 –100 = –2 – 2n + 2
 55 is a term of the given A.P. and it is 17th 100
term.  +2n = 100  n = = 50
2
(iii) A.P. 31, 28, 25, ...
Let mth term is the 12th term from the end
Here, a = 31, d = 28 – 31 = –3
Then Tm = l – (n – 1)d
197 Arun Deep's Understanding Math-10
= –100 – (12 – 1) × (–2) = –100 + 22 = –78 = 53 – 5n + 5 = 58 – 5n
8. Find the sum of the two middle most terms Now, Tn = 0
of the A.P.
 5n = 58
4 2 1
 , –1,  , ..., 4 . 58 3
3 3 3 n= = 11
1
5 5
4 2 1  12th term will be negative.
Sol. Given, A.P. is  , –1,  , ..., 4
3 3 3
P.Q. Determine the A.P. whose fifth term is 19
and the difference of the eighth term from
4 4 4 1
Here, a =  , d = –1 –   –1+ = the thirteenth term is 20.
3  3  3 3
Sol. In an A.P.,
1 T5 = 19
l=4
3 T13 – T8 = 20
1 Let a be the first term and d be the common
 Tn = l = 4 = a + (n – 1)d difference of required A.P.
3
 T5 = a + 4d = 19 ...(i)
1 4 1
 4 = + (n – 1) × T13 – T8 = (a + 12d) – (a + 7d)
3 3 3
 20 = a + 12d – a – 7d
13 4 1 17 3
 + = (n – 1)  × = (n – 1) 20
3 3 3 3 1  20 = 5d  d = =4
5
n – 1 = 17  n = 17 + 1 = 18
Substitute the value of d in eq. (i), we get
18 18
 Two middle term are and +1  a + 4 × 4 = 19  a + 16 = 19
2 2
 a = 19 – 16 = 3
= 9th and 10th term
 Required A.P. is 3, 7, 11, 15, ...
 a9 + a10 = a + 8d + a + 9d
10. Determine the A.P. whose third term is 16
= 2a + 17d and the 7th term exceeds the 5th term by
4 12.
1
=2×   + 17 × Sol. Given, T3 = 16
 3  3
and T7 – T5 = 12
8 17 9
= + = =3 Let a be the first term and d be the common
3 3 3 difference of given A.P.
9. Which term of the A.P. 53, 48, 43, ... is the T3 = a + 2d = 16 ...(i)
first negative term ?
T7 – T5 = (a + 6d) – (a + 4d) = 12
Sol. Let nth term is the first negative term of given
A.P. 53, 48, 43, ...  a + 6d – a – 4d = 12
Here, a = 53, d = 48 – 53 = –5 12
 2d = 12  d = =6
 Tn = a + (n – 1)d 2
= 53 + (n – 1) × (–5) Substitute the value of d in eq. (i), we get
198 Arun Deep's Understanding Math-10
 a + 2 × 6 = 16  a + 12 = 16
1 1
 a = 16 – 12 = 4 Sol. Given, a7 =  a + 6d = ....(i)
9 9
 A.P. is 4, 10, 16, 22, 28, ...
11. Find the 20th term of the A.P. whose 7th 1 1
term is 24 less than the 11th term, first term
and a9 =   a  8d   ....(ii)
7 7
being 12.
Sol. Given, T11 – T7 = 24 1 1
On subtracting, (ii) from (i),  2d  
and a = 12 9 7
Let a be the first term and d be the common
79 2
difference, then  –2d =  –2d =
63 63
(a + 10d) – (a + 6d) = 24
a + 10d – a – 6d = 24  4d = 24 1
 d=
24 63
 d= =6
4 Now, substitute the value of d in eq. (i),
a = 12 we get
 T20 = a + 19d = 12 + 19 × 6
 1  1
= 12 + 114 = 126 a + 6  
 63  9
12. Find the 31st term of an A.P. whose 11th
term is 38 and 6th term is 73. 1 6 76 1
Sol. Given, T11 = 38, T6 = 73 a=   
9 63 63 63
Let a be the first term and d be the common
difference of required A.P. then  a63 = a + 62d
a + 10d = 38 ...(i) 1  1  1  62 63
=  62    1
a + 5d = 73 ...(ii) 63  63  63 63
On subtracting, 5d = –35
14. (i) The 15th term of an A.P. is 3 more than
35 twice its 7th term. If the 10th term of the
d=  = –7
5 A.P. is 41, find its nth term.
Substitute the value of d in eq. (i), we get (ii) The sum of 5th and 7th terms of an A.P. is
a + 10d = 38 52 and the 10th term is 46. Find the A.P.
a –70 = 38  a = 38 + 70 = 108 (iii) The sum of 2nd and 7th terms of an A.P. is
 T31 = a + 30d = 108 + 30(–7) = –102 30. If its 15th term is 1 less than twice its
 31th term = –102. 8th term, find the A.P.
Sol. (i) Let a be the first term and d be the
1
13. If the seventh term of an A.P. is and its common difference of given A.P.
9
We have,
1
ninth term is , find its 63rd term. a10 = 41
7
199 Arun Deep's Understanding Math-10
 a + 9d = 41 ...(i) Hence, a2 = a1 + d
and a15 = 2a7 + 3 =1+5=6
 a + 14d = 2(a + 6d) + 3 a3 = a2 + d
 a + 14d = 2a + 12d + 3 = 6 + 5 = 11
 a – 2d = –3 ...(ii) a4 = a3 + d
Subtracting (ii) from (i), we get = 11 + 5 = 16
9d + 2d = 41 + 3  The required A.P. formed is 1, 6, 11, 16, .....
 11d = 44 15. If 8th term of an A.P. is zero, prove that its
38th term is triple of its 18th term.
 d=4
Sol. Given, T8 = 0
Now, from (i), we get
To prove that T38 = 3 × T18
a + 9 × 4 = 41
Let a be the first term and d be the common
 a + 36 = 41
difference of given A.P.
 a=5
T8 = a + 7d = 0  a = –7d
Now,
Now T38 = a + 37d
nth term = an = a + (n – 1)d
= –7d + 37d = 30d
= 5 + (n – 1)4 = 4n + 1
and T18 = a + 17d = –7d + 17d = 10d
(ii) Let a be the first term and d be the common
It is clear that T38 is triple of T18
difference, then
16. Which term of the A.P. 3, 10, 17, ... will be
a5 = a + (5 – 1)d = a + 4d
84 more than its 13th term ?
a7 = a + (7 – 1)d = a + 6d
Sol. Given, A.P. is 3, 10, 17, ...
 a5 + a7 = a + 4d + a + 6d = 52
Here, a = 3, d = 10 – 3 = 7
 2a + 10d = 52
T13 = a + 12d = 3 + 12 × 7 = 3 + 84 = 87
 a + 5d = 26 ...(i)
Let nth term is 84 more than its 13th term
Similarly,
 Tn = 84 + 87 = 171
a10 = a + (10 – 1)d = a + 9d
 a + (n – 1)d = 171
 a + 9d = 46 ...(ii)
 3 + (n – 1) × 7 = 171
Subtracting (i) from (ii),
 (n – 1) × 7 = 171 – 3 = 168
4d = 20
168
20  n–1= = 24
 d= =5d=5 7
4
i.e. n = 24 + 1 = 25
Now, put the value of d in eq. (i) ; we get
 25th term is the required term.
a + 5 × 5 = 26
P.Q. If the nth terms of the two A.P.s 9, 7, 5, ...
 a = 26 – 25  a = 1 and 24, 21, 18, ... are the same, find the
200 Arun Deep's Understanding Math-10
value of n. Also find that term. by 10 are 110, 120, 130, ....., 990
Sol. nth term of two A.P.s 9, 7, 5, ... and 24, 21, Here a = 110, d = 120 – 110 = 10
18, ... are same
and an = 990
In the first A.P. 9, 7, 5, ...
 a + (n – 1) d = 990
a = 9 and d = 7 – 9 = –2
 110 + (n – 1) (10) = 990
 Tn = a + (n – 1)d = 9 + (n – 1)(–2)
 (n – 1) (10) = 990 – 110 = 880
= 9 – 2n + 2 = 11 – 2n
and in second A.P. 24, 21, 18, ... 880
 (n – 1) = = 88
10
a1 = 24, d1 = 21 – 24 = –3
Tn = 24 + (n – 1)(–3)  n = 88 + 1 = 89

= 24 – 3n + 3 = 27 – 3n Hence, number between 101 and 999 which


are divisible by both 2 and 5 are 89.
The nth terms of both A.P.s is same
(iii) Numbers between 10 and 300, which when
 11 – 2n = 27 – 3n
divided by 4 leave a remainder 3 will be
–2n + 3n = 27 – 11  n = 16 11, 15, 19, 23, ... 299
and T16 = a + (n – 1)d Here, a = 11, d = 15 – 11 = 4, l = 299
= 9 + 15 × (–2) = 9 – 30 = –21  Tn = l = a + (n – 1)d
17. (i) How many two digit numbers are divisible 299 = 11 + (n – 1) × 4
by 3 ?
 299 – 11 = (n – 1)4
(ii) Find the number of natural numbers
between 101 and 999 which are divisible 288
 4(n – 1) = 288  n – 1 = = 72
by both 2 and 5. 4

(iii) How many numbers lie between 10 and 300,  n = 72 + 1 = 73


which when divided by 4 leave a remainder 18. If the numbers n – 2, 4n – 1 and 5n + 2 are
3? in A.P., find the value of n.
Sol. (i) Two digits numbers divisible by 3 are Sol. Since, n – 2, 4n – 1 adn 5n + 2 are in A.P.
12, 15, 18, 21, ..., 99  2(4n – 1) = n – 2 + 5n + 2
Here, a = 13, d = 15 – 12 = 3 and l = 99 8n – 2 = 6n  8n – 6n = 2
 Tn = l = a + (n – 1)d [ a, b, c are in A.P then 2b = a + c]
 99 = 12 + (n – 1) × 3  99 – 12 = 3(n – 1) 2
 2n = 2  n = =1
87 2
 3(n – 1) = 87  n – 1 = = 29
3  n=1

 n = 29 + 1 = 30 19. The sum of three numbers in A.P. is 3 and


their product is –35. Find the numbers.
(ii) Numbers divisible by both 2 and 5 i.e. divisible
Sol. Sum of three numbers which are in A.P. = 3
201 Arun Deep's Understanding Math-10
Their product = –35 terms exceeds the second term by 29, find
the A.P.
Let the three numbers which are in A.P.
Sol. Let the three numbers in A.P. are
a – d, a, a + d
a – d, a, a + d
 a – d + a + a + d = 3  3a = 3 Now, a – d + a + a + d = 33
 3a = 33
3
 a= =1 33
3
 a= = 111
3
and (a – d) × a × (a + d) = –35
 (1 – d) × 1 × (1 + d) = –35 and (a – d) (a + d) = a + 29
 a2 – d2 = a + 29
 12 – d2 = –35
 (11)2 – d2 = 11 + 29
 1 – d2 = –35  d2 = 35 + 1 = 36
 121 – d2 = 40
 d = +6
 d2 = 121 – 40 = 81 = (+9)2
when d = 6
 d = +9
 Numbers are 1 – 6, 1, 1 + 6
When d = 9, then
i.e. –5, 1, 7
 Numbers are 11 – 9, 11, 11 + 9
when d = –6
i.e. 2, 11, 20
 Numbers are, 1 + 6, 1, 1 – 6 i.e. 7, 1, –5
When d = –9, then numbers are
Hence numbers in A.P. are
11 + 9, 11, 11 – 9
–5, 1, 7 or 7, 1, –5 i.e. 20, 11, 2
20. The sum of three numbers in A.P. is 30 and Hence required numbers are 2, 11, 20 or 20,
the ratio of first number to the third number 11, 2
is 3 : 7. Find the numbers.
P.Q. Justify whether it is true to say that the
Sol. Sum of three numbers in A.P. = 30 following are the nth terms of an A.P.
Ratio between first and the third number = 3 : 7 (i) 2n – 3 (ii) n2 + 1
Let the numbers be a – d, a, a + d, then Sol. (i) Given Tn = 2n – 3
a – d + a + a + d = 30 Giving the some difference values to n such
30 as 1, 2, 3, 4, ... then
 3a = 30  a = = 10 T 1= 2 × 1 – 3 = 2 – 3 = –1
3
T 2= 2 × 2 – 3 = 4 – 3 = 1
ad 3 T 3= 2 × 3 – 3 = 6 – 3 = 3
and =  7a – 7d = 3a + 3d
ad 7 T 4= 2 × 4 – 3 = 8 – 3 = 5
 7a – 3a = 3d + 7d  4a = 10d We see that –1, 1, 3, 5, ... are in A.P. whose
first term = –1 and d = 1 – (–1) = 1 + 1 = 2
40
 10d = 4 × 10 = 40  d = =4 (ii) Tn = n2 + 1
10
Giving some different values to n such as
 Required numbers are 10 – 4, 10, 10 + 4 i.e.
1, 2, 3, 4, ...
6, 10, 14
T1 = (1)2 + 1 = 1 + 1 = 2
21. The sum of the first three terms of an A.P.
is 33. If the product of the first and the third T2 = (2)2 + 1 = 4 + 1 = 5
202 Arun Deep's Understanding Math-10
T3 = (3)2 + 1 = 9 + 1 = 10 11 9 33 13
T4 = (4)2 + 1 = 16 + 1 = 17 = × = or = 1
2 30 20 20
We see that a = 2, P.Q. How many terms of the A.P. 27, 24, 21,
d=5–2=3 ..., should be taken so that their sum is
= 10 – 5 = 5 zero?
= 17 – 10 = 7 Sol. Given, A.P. be 27, 24, 21, .....
The common difference is not same.  a = 27 and d = 24 – 27 = –3
and Sn = 0
 No. It does not form an A.P.
Let n terms be there in A.P.
EXERCISE 9.3 we know that
1. Find the sum of the following A.P.s : n
Sn = [2a + (n – 1)d]
(i) 2, 7, 12, ... to 10 terms 2
1 1 1 n
(ii) , ,
15 12 10
, ... to 11 terms  0= [(2 × 27) + (n – 1) (–3)]
2
Sol. (i) Given series be, 2, 7, 12, ... to 10 terms  0 = n[54 – 3n + 3]
Here a = 2, d = 7 – 2 = 5 and n = 10  n[57 – 3n] = 0
n  (57 – 3n) = 0 [ n  0]
Sn = [2a + (n – 1)d]  3n = 57
2

10 57
 S 10= [2 × 2 + (10 – 1) × 5]  n= = 19
2 3

= 5(4 + 45) = 5 × 49 = 245 2. Find the sums given below :


(i) 34 + 32 + 30 + ... + 10
1 1 1
(ii) Given series be, , , , ... to 11 terms (ii) –5 + (–8) + (–11) + ... + (–230)
15 12 10
Sol. (i) Given A.P. be, 34 + 32 + 30 + ... + 10
1 1 1 54 1 Here, a = 34, d = 32 – 34 = –2, l = 10
Here a = , d= – = =
15 12 15 60 60 Tn = a + (n – 1)d
 1 65 1  10 = 34 + (n – 1)(–2)
or – = =
10 12 60 60  –24 = –2(n – 1)
and n = 11 24
 = n – 1  n – 1 = 12
n 2
Sn = × [2a + (n – 1)d]
2  n = 12 + 1 = 13

11  1 1 n
 S11 = × 2   (11  1)   and Sn = [a + l]
2  15 60  2

11  2 1 11 4  5 13 13
= ×    = ×    S13 =
2
[34 + 10] =
2
× 44 = 286
2 15 6  2  30 
203 Arun Deep's Understanding Math-10
(ii) Given series be, 28 7 1
–5 + (–8) + (–11) + ... + (–230)  28 = 12d  d = = =2
12 3 3
Here, a = –5, d = –8 – (–5) = –8 + 5 = –3
n
l = –230 and S13 = [2a + (n – 1)d]
2
 l = a + (n – 1)d  –230 = –5 + (n – 1) (–3)
 –230 + 5 = –3(n – 1)  –225 = –3(n – 1) 13  7
= 2  7  (13  1)  3 
225 2  
 = n – 1  n – 1 = 75
3
13 13
 n = 75 + 1 = 76 = [14 + 28] = × (42)
2 2
n 76 = 13 × 21 = 273
 Sn = [a + l] = [–5 + (–230)]
2 2 (iii) Given, d = 5, S9 = 75
= 38[–5 – 230] = 38 × (–235) = –8930 an = a + (n – 1)d
3. In an A.P. (with usual notations) : a9 = a + (9 – 1) × 5
(i) given a = 5, d = 3, an = 50, find n and Sn = a + 40 ...(i)
(ii) given a = 7, a13 = 35, find d and S13 We know that
(iii) given d = 5, S9 = 75, find a and a9
n
(iv) given a = 8, an = 62, Sn = 210, find n and d Sn = [2a + (n – 1)d]
2
(v) given a = 3, n = 8, S = 192, find d.
Sol. (i) Given, a = 5, d = 3, an = 50 9
 75 = [2a + 8 × 5]
2
an = a + (n – 1)d
 50 = 5 + (n – 1) × 3  50 – 5 = 3(n – 1) 150
 = 2a + 40
45 9
 45 = 3(n – 1)  =n–1
3 150 50
 2a = – 40 = – 40
 n – 1 = 15  n = 15 + 1 = 16 9 3
 n = 16
70 70
 2a = a=
n 3 23
and Sn = [2a + (n – 1)d]
2
35
16 a=
3
 S16 = [2 × 5 + (16 – 1) × 3] = 8[10 + 45]
2
From (i), we have
= 8 × 55 = 440
35
(ii) Given, a = 7, a13 = 35 a9 = a + 40 = + 40
3
an = a + (n – 1)d
 35 = 7 + (13 – 1)d  35 – 7 = 12d 35  120 85
= =
3 3
204 Arun Deep's Understanding Math-10
35 85 (ii) The sum of first 15 terms of an A.P. is 750 and its
 a= , a9 = first term is 15. Find its 20th term.
3 3
(iv) Given, a = 8, an = 62, Sn = 210 Sol. (i) First term of an A.P. (a) = 5
Since, an = a + (n – 1)d Last term (l) = 45
Sum = 400
 62 = 8 + (n – 1)d
Let the required number of terms be n
 (n – 1)d = 62 – 8 = 54 ...(i)
s.t. l = a + (n – 1)d
n  45 = 5 + (n – 1)d
and Sn = [2a + (n – 1)d]
2  (n – 1)d = 45 – 5 = 40 ...(i)
n n
 210 = [2 × 8 + 54] [From (i)] and Sn = [2a + (n – 1)d]
2 2
 420 = n(16 + 54)  420 = 70n n
 400 = [2 × 5 + 40]  800 = n(10 + 40)
420 2
 n= =6
70
800
 (6 – 1)d = 54 50n = 800  n = = 16
50
 5d = 54
From (i), we have
54
 d= 40
5  (16 – 1)d = 40  15d = 40  d =
15
54
Hence d = and n = 6 8
5  d= and n = 16
3
(v) Given, a = 3, n = 8, S = 192
(ii) Let a be the first term and d be the common
n difference of given A.P.
Since, Sn = [2a + (n – 1)d]
2 Now, a = 15
and Sn = 192 Sum of first n terms of an AP is given by,
8 n
 192 = [2 × 3 + 7 × d] Sn = [2a + (n – 1)d]
2 2
192 15
 192 = 4[6 + 7d]  = 6 + 7d  S15 = [2a + (15 – 1)d]
4 2
 48 = 6 + 7d  7d = 48 – 6 = 42
15
42  750 = (2a + 14d)
 d= =6 2
7
 a + 7d = 50
 d=6  15 + 7d = 50
4. (i) The first term of an A.P. is 5, the last  7d = 35
term is 45 and the sum is 400. Find the
 d=5
number of terms and the common
Now, 20th term = a20 = a + 19d
difference.
205 Arun Deep's Understanding Math-10
= 15 + 19 × 5 = 15 + 95 = 110 41
5. The first and the last terms of an A.P. are 17 or 3n + 41 = 0, then 3n = –41  n =
3
and 350 respectively. If the common
which is not possible being negative.
difference is 9, how many terms are there
and what is their sum ?  n = 14
Sol. First term of an A.P. (a) = 17 Now, x = a + (n – 1)d
and last term (l) = 350 = 1 + (14 – 1) × 3 = 1 + 13 × 3
and d = 9 = 1 + 39 = 40
Since, l = Tn = a + (n – 1)d  x = 40
 350 = 17 + (n – 1) × 9 7. (i) How many terms of the A.P. 25, 22, 19,
... are needed to give the sum 116 ? Also
 350 – 17 = 9(n – 1) find the last term.
333 (ii) How many terms of the A.P. 24, 21, 18, ...
 333 = 9(n – 1)  n – 1 = = 37 must be taken so that the sum is 78 ? Explain
9
the double answer.
 n = 37 + 1 = 38
Sol. (i) Given, A.P. is 25, 22, 19, ...
n
Sn = [2a + (n – 1)d] and Sum = 116
2
Here, a = 25, d = 22 – 25 = –3
38 Let number of terms be n, then Sn = 116
 S38 = [2 × 17 + (38 – 1) × 9]
2
n
= 19[34 + 37 × 9] = 19 [34 + 333]  116 = [2a + (n – 1)d]
2
= 19 × 367 = 6973
 232 = n[2 × 25 + (n – 1) (–3)]
Hence n = 38 and Sn = 6973
 232 = n[50 – 3n + 3]  232 = n(53 – 3n)
6. Solve for x : 1 + 4 + 7 + 10 + ... + x = 287.
 232 = 53n – 3n2
Sol. Given, 1 + 4 + 7 + 10 + ... + x = 287
Here, a = 1, d = 4 – 1 = 3, Tn = x  3n2 – 53n + 232 = 0
l = x = a + (n – 1)d = 1 + (n – 1) × 3
 232  3  696 
 x – 1 = (n – 1)d  
 696  24  (29)
n   53  24  29 
Sn = [2a + (n – 1)d]  
2
 3n2 – 24n – 29n + 232 = 0
n
 287 = [2 × 1 + (n – 1)3]  3n(n – 8) – 29(n – 8) = 0
2
 (n – 8) (3n – 29) = 0
 574 = n(2 – 3n – 3)
Either n – 8 = 0, then n = 8
 3n2 – n – 574 = 0
 3n2 – 42n + 41n – 574 = 0 29
or 3n – 29 = 0, then 3n = 29  n =
 3n(n – 14) + 41(n – 14) = 0 3
 (n – 14) (3n + 41) = 0 which is not possible because of fraction
Either n – 14 = 0, then n = 14  n=8
206 Arun Deep's Understanding Math-10
Now, Tn = a + (n – 1)d n
= 25 + 7 × (–3) = 25 – 21 = 4  S22 = [2a + (n – 1)d]
2
(ii) Given, A.P. is 24, 21, 18, ...
and Sum = 78 22
= [2 × 2 + (22 – 1) (7)]
Here, a = 24, d = 21 – 24 = –3 2
Let the required no. of terms be n s.t. Sn = 78 = 11[4 + 21 × 7] = 11 × [4 + 147]
= 11 × 151 = 1661
n
Sn = [2a + (n – 1)d] 9. In an Arithmetic Progression (A.P.) the
2 fourth and sixth terms are 8 and 14
respectively. Find the :
n
 78 = [2 × 24 + (n – 1) (–3)] (i) first term (ii) common difference
2
(iii) sum of the first 20 terms.
 156 = n(48 – 3n + 3)
 156 = 51n – 3n2 Sol. t4  a  3d  8 ...(i)
 3n2 – 51n + 156 = 0 t6  a  5d  14 ...(ii)
 3n2 – 12n – 39n + 156 = 0
Eq. (i) – (ii)  – 2d = – 6  d 3
 156  3  468 
   Substituting ; value of d in eq. (i)
 468  12  39
  51  12  39   a + 3(3) = 8
 
 d=8–9=–1
 3n(n – 4) – 39(n – 4) = 0
 (n – 4) (3n – 39) = 0 (i) First term = a  1
Either n – 4 = 0, then n = 4
(ii) Common difference = d = 3
or 3n – 39 = 0, then 3n = 39  n = 13
 n = 4 and 13 n
(iii) S20 = [2a  (n  1) d ]
 T4 = a + (n – 1)d = 24 + 3(–3) 2
= 24 – 9 = 15 20
T13 = 24 + 12(–3) = 24 – 36 = –12 = [2  (1)  19  3]
2
 Sum of 5th term to 13 term = 0
= 10 [– 2 + 57] = 10  55
[ 12 + 9 + 6 + 3 + 0 + (–3) + (–6) + (–9) +
 S20 = 550
(–12) = 0]
10. (i) Find the sum of first 51 terms of the A.P.
8. Find the sum of first 22 terms of an A.P. in
whose second and third terms are 14 and
which d = 7 and a22 is 149.
18 respectively.
Sol. Sum of first 22 terms of an A.P. whose (ii) The 4th term of an A.P. is 22 and the 15th term is
d=7 66. Find the first term and the common
a22 = 149 and n = 22 difference. Hence, find the sum of the series to
8 terms of the A.P. (2018)
 149 = a + (n – 1)d = a + 21 × 7
149 = a + 147 Sol. (i) Sum of first 51 terms of an A.P. in which
 a = 149 – 147 = 2 T2 = 14, T3 = 18
207 Arun Deep's Understanding Math-10
 d = T3 – T2 = 18 – 14 = 4 6
and a = T1 = 14 – 4 = 10, n = 51  S6 = [2a + (6 – 1)d] = 36
2
n
Now, Sn = [2a + (n – 1)d]  3[2a + 5d] = 36
2
51  2a + 5d = 12 ...(i)
 S51 = [2 × 10 + (51 – 1) × 4]
2
16
51 51 and S16 = [2a + (16 – 1)d] = 256
= [20 + 50 × 4] = [20 + 200] 2
2 2
 8[2a + 15d] = 256
51
= × 220 = 5610  2a + 15d = 32 ...(ii)
2
On subtracting (i) from (ii), we have
(ii) a + (n – 1) d = tn, Since t4 = 22
 a + (4 – 1)d = 22  a + 3d = 22 20
–10d = –20  d = =2
 10
and t15 = 66
Substitute the value of d in eq. (i), we have
a + (15 – 1)d = 66  a + 14d = 66
2a + 5d = 12
 a + 3d = 22
 2a + 5 × 2 = 12 2a + 10 = 12
a + 14d = 66
– – – 2
 2a = 12 – 10 = 2 a = =1
–11d = –44 d=4 2
 a + 3d = 22  a + 3(4) = 22  a =1, d = 2
 a = 10 n
Now, Sn = [2a + (n – 1)d]
2
n
Sn = [2a + (n – 1) d] 10
2
 S10 = [2 × 1 + (10 – 1) × 2]
2
8 = 5[2 + 9 × 2] = 5[2 + 18]
 S8 = [2 × 10 + (8 – 1) 4]
2
= 5 × 20 = 100
 S8 = 4[20 + 28] = 4[48] = 192 Ans. 12. Show that a1, a2, a3, ... form an A.P. where
11. If the sum of first 6 terms of an A.P. is 36 an is defined as an = 3 + 4n. Also find the
and that of the first 16 terms is 256, find the sum of first 15 terms.
sum of first 10 terms.
Sol. an = 3 + 4n
Sol. Given, S6 = 36
 a1 = 3 + 4 × 1 = 3 + 4 = 7
and S16 = 256
a2 = 3 + 4 × 2 = 3 + 8 = 11
n
Sn = [2a + (n – 1)d] a3 = 3 + 4 × 3 = 3 + 12 = 15
2
Where a = first term and d = Common a4 = 3 + 4 × 4 = 3 + 16 = 19 and so on
difference Here, a = 7 and d = 11 – 7 = 4
208 Arun Deep's Understanding Math-10

n  2(20 + 6d) = (20 + 22d) ( a = 5)


Sn =
2
[2a + (n – 1)d]  40 + 12d = 20 + 22d
 10d = 20
15
 S 10= [2 × 7 + (15 – 1) × 4]  d=2
2
Thus, the common difference of A.P. is 2.
15 15 13. The sum of first six terms of an arithmetic
= [14 + 14 × 4] = [14 + 56]
2 2 progression is 42. The ratio of the 10th term
to the 30th term is 1 : 3. Calculate the first
15 and the thirteenth term.
= × 70 = 525
2
Sol. T10 : T30 = 1 : 3, S6 = 42
P.Q. (i) If an = 3 – 4n, show that a1, a2, a3, ... Let a be the first term and d be the common
form an A.P. Also find S20. difference, then
(ii) Find the common difference of an A.P.
whose first term is 5 and the sum of first a  9d 1
 =  3a + 27d = a + 29d
four terms is half the sum of next four a  29d 3
terms.  3a – a = 29d – 27d
Sol. (i) Given, an = 3 – 4n  2a = 2d  a = d
 a1 = 3 – 4 × 1 = 3 – 4 = –1
n
a2 = 3 – 4 × 2 = 3 – 8 = –5 Now, S6 = 42 = [2a + (n – 1)d]
2
a3 = 3 – 4 × 3 = 3 – 12 = –9
a4 = 3 – 4 × 4 = 3 – 16 = –13 and so on 6
 42 = [2a + (6 – 1)d]
Here, a = –1, d = –5 – (–1) = –5 + 1 = –4 2

n  42 = 3[2a + 5d]
Now, Sn = [2a + (n – 1)d]  14 = 2a + 5d  14 = 2a + 5a ( d = a)
2

20 14
 S20 = [2 × (–1) + (20 – 1) × (–4)]  7a = 14  a = =2
2 7

= 10[–2 + 19 × (–4)]  a=d=2


= 10[–2 – 76] = 10 × (–78) = –780 Now, T4 = a + (n – 1)d
(ii) Let a and d be the first term and common  T13 = 2 + (13 – 1) × 2 = 2 + 12 × 2
difference of A.P. respectively. = 2 + 24 = 26
Given, a = 5  1st term is 2 and thirteenth term is 26
1 14. In an A.P., the sum of its first n terms is
(a + a6 + a7 + a8)
a1 + a2 + a3 + a4 = 6n – n2. Find is 25th term.
2 5
 a + (a + d) + (a + 2d) + (a + 3d) Sol. Given, Sn = 6n – n2
1 T25 = ?
=
[(a + 4d) + (a + 5d) + (a + 6d) and S(n – 1) = 6(n – 1) – (n – 1)2
2
+ (a + 7d)] = 6n – 6 – (n2 – 2n + 1)
 2(4a + 6d) = (4a + 22d) = 6n – 6 – n2 + 2n – 1 = 8n – n2 – 7
209 Arun Deep's Understanding Math-10
Now an = Sn – Sn – 1 = 500[2 + 999] = 500 × 1001
= 6n – n2 – 8n + n2 + 7 = 500500
= –2n + 7 (ii) Sum of first 15 multiples of 8
 a25 = –2(25) + 7 = –50 + 7 = –43 8 + 16 + 24 + 32 + ... 120
15. If Sn denotes the sum of first n terms of an Here, a = 8, d = 8, n = 15
A.P., prove that S30 = 3(S20 – S10).
n
Sol. Sn denotes sum of first n terms of an A.P.  S15 = [2a + (n – 1)d]
2
To prove : S30 = 3(S20 – S10)
15
n = [2 × 8 + (15 – 1) × 8]
Sn = [2a + (n – 1)d] 2
2
15 15
10 = [16 + 14 × 8] = [16 + 112]
 S10 = [2a + (10 – 1)d] = 5(2a + 9d) 2 2
2
= 10a + 45d 15
= × 128 = 15 × 64 = 960
2
20
S20 = [2a + (20 – 1)d] = 10(2a + 19d) 17. (i) Find the sum of all two digit natural
2 numbers which are divisible by 4.
= 20a + 190d (ii) Find the sum of all natural numbers between
100 and 200 which are divisible by 4.
30
S30 = [2a + (30 – 1)d = 15(2a + 29d) (iii) Find the sum of all multiples of 9 lying
2
between 300 and 700.
= 30a + 435d
(iv) Find the sum of all natural numbers less than
Now, 100 which are divisible by 6.
R.H.S. = 3(S20 – S10) Sol. (i) Sum of two digit natural numbers which
= 3[20a + 190d – 10a – 45d] are divisible by 4 which are 12, 16, 20, 24,
= 3[10a + 145d] ..., 96
= 30a + 435d Here, a = 12, d = 16 – 12 = 4, l = 96
= S30 = L.H.S.  l = 96 = a + (n – 1)d = 12 + (n – 1) × 4
16. (i) Find the sum of first 1000 positive 96 = 12 + 4n – 4 = 8 + 4n
integers. 88
(ii) Find the sum of first 15 multiples of 8.  4n = 96 – 8 = 88  n = = 22
4
Sol. (i) Sum of first 1000 positive integers
n
i.e., 1 + 2 + 3 + 4 + ... + 1000  S22 = [2a + (n – 1)d]
2
Here, a = 1, d = 1, n = 1000
22
n = [2 × 12 + (22 – 1) × 4]
Sn = [2a + (n – 1)d] 2
2
= 11[24 + 21 × 4] = 11[24 + 84]
1000
 S1000 = [2 × 1 + (1000 – 1)1] = 11 × 108 = 1188
2
(ii) Sum of all natural numbers between 100 and
210 Arun Deep's Understanding Math-10
200 which are divisible by 4 which are (iv) Sum of all natural numbers less then 100
104, 108, 112, 116, ..., 196 which are divisible by 6 which are
Here, a = 104, d = 108 – 104 = 4, l = 196 6, 12, 18, 24, ..., 96
l = an = 196 = a + (n – 1)d Here, a = 6, d = 6, l = 96
 196 = 104 + (n – 1) ×4 an = l = 96 = a + (n – 1)d
 196 – 104 = (n – 1)4  92 = (n – 1)4  96 = 6 + (n – 1) × 6
 96 – 6 = 6(n – 1)
92
 n–1= = 23 90
4
 = n – 1  n – 1 = 15
 n = 23 + 1 = 24 6
n = 15 + 1 = 16
n
Now, S24 = [2a + (n – 1)d]
2 n
 S16 = [2a + (n – 1)d]
2
24
= [2 × 104 + (24 – 1) × 4]
2 16
= [2 × 6 + (16 – 1) × 6]
= 12[208 + 23 × 4] = 12 × [208 + 92] 2
= 12 × 300 = 3600 = 8[12 + 15 × 6] = 8[12 + 90]
(iii) Sum of all natural numbers multiple of 9 lying = 8 × 102 = 816
between 300 and 700 which are P.Q. (i) Find the sum of all two digit odd positive
306, 315, 324, 333, ..., 693 numbers.
Here, a = 306, d = 9, l = 693 (ii) Find the sum of all 3-digit natural numbers
l = an = 693 = a + (n – 1)d which are divisible by 7.
= 306 + (n – 1) × 9 (iii) Find the sum of all two digit numbers which
when divided by 7 yield 1 as the remainder.
 693 – 306 = 9(n – 1)
Sol. (i) Sum of all two-digit odd positive numbers
387 which are 11, 13, 15, ..., 99
 387 = 9(n – 1)  n – 1 = = 43
9 Here, a = 11, d = 2, l = 99
 n = 43 + 1 = 44 an = l = a + (n – 1)d
 99 = 11 + (n – 1) × 2  99 – 11 = 2(n – 1)
n
Sn = [2a + (n – 1)d]
2 88
 88 = 2(n – 1)  n – 1 = = 44
2
44
 S44 = [2 × 306 + (44 – 1) × 9]  n = 44 + 1 = 45
2
= 22[612 + 43 × 9] n
S45 = [2a + (n – 1)d]
= 22[612 + 387] = 22 × 999 = 21978 2

999 45
= [2 × 11 + (45 – 1) × 2]
×22 2
1998
1998× 45 45
= [22 + 44 × 2] = [22 + 88]
21978 2 2
211 Arun Deep's Understanding Math-10
45 13
= × 110 = 2475 = [30 + 12 × 7]
2 2
(ii) Sum of all 3-digit natural numbers which 13 13
are divisible by 7 which are 105, 112, 119, = [30 + 84] = × 114
14
2 2
..., 994
= 13 × 57 = 741
Here, a = 105, d = 112 – 105 = 7, l = 994
 l = Tn = 994 = a + (n – 1)d
EXERCISE 9.4
 994 = 105 + (n – 1) × 7
 994 – 105 = (n – 1)7  889 = 7(n – 1) 1. (i) Find the next term of the list of numbers
889 1 1 2
= n – 1  n – 1 = 127 , , , ...
7 6 3 3
 n = 127 + 1 = 128 (ii) Find the next term of the list of numbers
3 3 3 3
n ,  , ,  , ...
 S128 = [2a + (n – 1)d] 16 8 4 2
2
(iii) Find the 15th term of the series
128
= [2 × 105 + (128 – 1) × 7] 1 1
2 3 + + + ...
3 3 3
= 64[210 + 889] = 64 × 1099 = 70336
(iv) Find the 10th and nth terms of the list of
(iii) The 2-digit number which when divided
numbers 5, 25, 125, ...
by 7 gives remainder 1 are :
(v) Find the 6th and the nth terms of the list of
15, 22, 29, ..., 99
Here, a = 15 and d = 22 – 15 = 7 3 3 3
numbers , , , ...
We have, an = 99 2 4 8
nth term of an AP is an = a + (n – 1)d (vi) Find the 6th term from the end of the list of
numbers 3, –6, 12, –24, ..., 12288.
 99 = 15 + (n – 1)7
(vii) Find the nth term of the list of numbers
 99 = 15 + 7n – 7
 99 = 8 + 7n 1
, –2, 4 2 , –16, ...
 7n = 99 – 8 2
91 1 1 2
 n= = 13 Sol. (i) Given series be, , , , ...,
7 6 3 3
 n = 13
1 1 1 1 6
Now, we know that Here, a = ,r= ÷ = × =2
6 3 6 3 1
n
 Sn = [2a + (n – 1)d] 2 4
2  Next term = ×2=
3 3
13
S13 = [2 × 15 + (13 – 1) × 7] 3 3 3 3
2 (ii) Given series by, ,  , ,  , ...
16 8 4 2
212 Arun Deep's Understanding Math-10
3 3 3 3 16 3 3 3
Here, a = ,r= ÷ = × = –2  a6 = n = 6 =
16 8 16 8 3 2 2 64

3 (vi) Given series be, 3, –6, 12, –24, ..., 12288


 Next term = × (–2) = 3
2 6th term from the end of the list
Here, a = 3, r = –6 ÷ 3 = –2, l = 12288
1 1
(iii) Given series by, 3 + + + ... Now, 6th term from the end
3 3 3
n 1 6 1
1  1 
Here, a = 3, =l×   = 12288 ×  
r 2
1 1 1 1
r= ÷ 3 = × = 1 12288
3 3 3 3 = 12288 × (2) 5 = = –384
 32
We know that an = arn–1
1
151 (vii) Given series be, , –2, 4 2 , –16, ...
1 2
 a15 = arn – 1 = 3  3
 
1
Here, a = ,
14 2
1 1
= 3 × 3 = 3 × 314
  1
r = –2 ÷ = –2 × 2 = –2 2
(iv) Given series be, 5, 25, 125, ... 2
Here, a = 5, r = 25 ÷ 5 = 5 1
 a10 = arn – 1 =5× (5)10 – 1 an = arn – 1 = × (–2 2 )n– 1
2
= 5 × 59 = 59 + 1 = 510
1
 an = arn – 1 = 5 × 5n – 1 = 5n – 1 + 1 = 5n = × (–1)n – 1 × [( 2 )2 × 2]
n–1
2
3 3 3
(v) Given series be, , , , ... 1
2 4 8 = (–1)n – 1 × × [( 2 )3]n – 1
2
3 3 3 3 2 1
Here, a = ,r= ÷ = × = 1
2 4 2 4 3 2 = (–1)n – 1 × × ( 2 )3n – 3
2
n 1
3 1
 an = arn – 1 = × = (–1)n – 1 ( 2 )3n – 3 – 1
2 2
= (–1)n – 1 ( 2 )3n – 4
n 1 n 11
1 1 1
=3× ×   =3×   3n  4
2 2 2 = (–1)n – 1 × 2 2

n 2. Which term of the G.P.


1 3
=3×   = n
2 2 (i) 2, 2 2 , 4, ... is 128?
213 Arun Deep's Understanding Math-10

1 1 1 1 1 1 1
(ii) 1, , , ... is ? (iii) Given, series be, , , , ... is
3 9 243 3 9 27 19683

1 1 1 1 1 1 1
(iii) , , , ... is ? Here, a = ,r= ÷
3 9 27 19683 3 9 3
Sol. (i) Given series be, 2, 2 2 , 4, ... is 128
1 3 1 1
i.e. r = × = ,l=
9 1 3 19683
2 2
Here, a = 2, r = = 2 , l = 128
2 1
Let is the nth term, then
Let 128 be the nth term, then 19683
an = 128 = arn – 1
n 1
1 1 1
 128 = 2( 2 )n – 1  2( 2 )n – 1 = 27 an = = arn – 1 =  
19683 3  3
3 19683
( 2 )n – 1 = 2 7 – 1 = 2 6 3 6561
n11 n
1 1 3 2187
( 2 )n – 1 = ( 2 )12 =   =   3 729
 3  3 3 243
Comparing, we get
3 81
n – 1 = 12  n = 12 + 1 = 13 1 1
n 9 3 27
   =   3 9
 128 is the 13th term  3 3 3 3
1
1 1 1 On comparing, we get
(ii) Given series be, 1, , , ... is  n=9
3 9 243
1
1 1 1 Hence, is the 9th term.
Here, a = 1, r = ÷1= ,l= 19683
3 3 243
P.Q. Which term of the G.P. 3, –3 3 , 9, –9 3 ,
1
Let is the nth term, then ... is 729 ?
243
Sol. G.P. is 3, –3 3 , 9, –9 3 , ... is 729
n 1
1 1
an = = arn – 1 = 1 ×   3 3
243  3 Here, a = 3, d = =  3 , l = 729
3
n 1 5 Let 729 is the nth term, then
1 1
   =  
 3 3 an = 729 = arn – 1 = 3 (  3 ) n 1

On comparing, we get 729


 = (  3 ) n 1  243 = (  3 ) n 1
n–1=5n=5+1=6 3

1  (– 3 )10 = (– 3 )n – 1
 is the 6th term.
243 On comparing, we get
214 Arun Deep's Understanding Math-10
n – 1 = 10  n = 10 + 1 = 11 3
 729 is the 11th term. first term is
4
, common ratio is 2 and the
3. Determine the 12th term of a G.P. whose last term is 384.
8th term is 192 and common ratio is 2.
Sol. In a G.P. 3
Sol. First term of a G.P. (a) =
a8 = 192 and r = 2 4
Let a be the first term and r be the common and common ratio (r) = 2
ratio then Last term = 384
a8 = arn – 1  192 = a(2)n – 1 = a28 – 1 = a27 Let number of terms of given G.P. be n
192 192 3 then an = arn – 1
 a= 2 512
7 = =
2 128 2 3 2 256
an = a(r)n – 1  384 = (2)n – 1 2 128
4 2 64
3 3 2 32
 a12 = (2)12 – 1 = × 211 384  4
2 2  2n – 1 = = 512 = 29 2 16
3 2 8
3  n – 1 = 9  n = 9 + 1 = 10 2 4
= × 2048 = 3072 2 2
2  Number of terms in G.P. = 10 1
 a12 = 3072 6. Find the value of x such that
4. In a G.P., the third term is 24 and 6th term is
2 7
192. Find the 10th term. (i)  , x,  are three consecutive terms of
7 2
Sol. In a G.P.
a G.P.
a3 = 24 and a6 = 192, a10 = ?
(ii) x + 9, x – 6 and 4 are three consecutive
Let a be the first term and r be the common terms of a G.P.
ratio, therefore
(iii) x, x + 3, x + 9 are first three terms of a G.P.
a6 = ar6 – 1 = ar5 = 192 { an = arn – 1}
Sol. Find the value of x
a3 = ar3 – 1 = ar2 = 24
2 7
On dividing, we get (i)  , x,  are three consecutive terms of
7 2
ar 5 192 a G.P.
2 =  r3 = 8 = (2)3
ar 24
2 7
 r=2  x2 = × = 1 = (+1)2
7 2
Now, ar2 = 24  a × 22 = 24  x = +1
24 24 Hence, x = 1 or –1
 a= = =6
22 4 (ii) x + 9, x – 6 and 4 are three consecutive
 a=6 terms of a G.P., then
Now, a10 = ar10 – 1 = ar9 [If a, b, c are in G.P. Then b2 = ac]
= 6 × (2)9 = 6 × 512 = 3072 (x – 6)2 = (x + 9) × 4
5. Find the number of terms of a G.P. whose  x2 – 12x + 36 = 4x + 36
215 Arun Deep's Understanding Math-10
 x2 – 12x – 4x + 36 – 36 = 0 a8 = ar7 = q and
 x2 – 16x = 0  x(x – 16) = 0 a11 = ar10 = s
Either x – 16 = 0, then x = 16 q2 = (ar7)2 = ar14
or x = 0 and ps = ar4 × ar10 = a2r4 + 10 = a2r14
 x = 0, 16 Hence, q2 = ps
(iii) x, x + 3, x + 9 are first three terms of a G.P. P.Q. If a, b, c are in G.P., then show that a2, b2,
 (x + 3)2 = x(x + 9) c2 are also in G.P.
 x2 + 6x + 9 = x2 + 9x Sol. a, b, c are in G.P.
 9 = 9x – 6x = 3x Show that a2, b2, c2 are also in G.P.
a, b, c are in G.P., then
9
 x= =3 b2 = ac ...(i)
3
a2, b2, c2 will be in G.P.
7. If the fourth, seventh and tenth terms of a
if (b2)2 = a2 × c2
G.P. are x, y, z respectively, prove that x, y, z
are in G.P.  (ac)2 = a2c2 [From (i)]
Sol. In a G.P.  a c = a c which is true.
2 2 2 2

a4 = x, a7 = y, a10 = z Hence proved.


To prove : x, y, z are in G.P. P.Q. If a, b, c are in A.P., then show that 3a, 3b,
3c are in G.P.
Let a be the first term and r be the common
ratio, therefore Sol. a, b and c are in A.P.
a4 = arn – 1 = ar4 – 1 = ar3 = x Then, 2b = a + c
Similarly, Now, 3a, 3b, 3c will be in G.P.
a7 = ar6 = y if (3b)2 = 3a·3c
a10 = ar9 = z if 32b = 3a + c
If x, y and z are in G.P., then Comparing, we get
y2 = xz if 2b = a + c, which is true.
Now, xz = ar3 × ar9 = a2r3 + 9 = a2r12 P.Q. If a, b, c are in A.P., then show that 10ax + 10,
10bx + 10, 10cx + 10, x  0, are in G.P.
y2 = (ar6)2 = a2·r12
Sol. a, b, c are in A.P.
L.H.S. = R.H.S.
To show that 10ax + 10, 10bx + 10, 10cx + 10,
 x, y and z are in G.P.
x  0 are in G.P.
8. The 5th, 8th and 11th terms of a G.P. are p,
a, b, c are in A.P.
q and s respectively. Show that q2 = ps.
 2b = a + c ...(i)
Sol. In a given G.P.,
Now,
a5 = p, a8 = q and a11 = s
(10ax + 10), (10bx + 10), (10cx + 10) will be in
To show that q2 = ps
G.P.
Let a be the first term and r be the common
if (10bx + 10)2 = (10ax + 10) × (10cx + 10)
ratio, then
if 102bx + 20 = 10ax + 10 + cx + 10
a5 = arn – 1 = ar5 – 1 = ar4 = p
if 102bx + 20 = 10ax + cx + 20
Similarly,
216 Arun Deep's Understanding Math-10
On comparing, we get Hence m = 4
if 2bx + 20 = ax + cx + 20 10. Find the geometric progression whose 4th
if 2bx = ax + cx term is 54 and the 7th term is 1458.
if 2b = a + c Sol. In a G.P.,
Which is given. 4th term (a4) = 54
9. If a, a2 + 2 and a3 + 10 are in G.P., then find and 7th term (a7) = 1458
the values(s) of a. Let a be the first term and r be the common
Sol. a, a2 + 2 and a3 + 10 are in G.P. ratio, then
 (a2 + 2)2 = a(a3 + 10) ar3 = 54 and ar6 = 1458
 a4 + 4a2 + 4 = a4 + 10a On dividing, we have

 4a2 – 10a + 4 = 0 ar 6 1458


=  r3 = 27 = (3)3
 2a2 – 5a + 2 = 0 ar 3 54
 2a2 – a – 4a + 2 = 0  r=3
 a(2a – 1) – 2(2a – 1) = 0 Now, ar3 = 54
 (2a – 1) (a – 2) = 0 54
 a × 27 = 54  a = =2
1 27
Either 2a – 1 = 0, then 2a = 1  a =
2 Hence required G.P. is a, ar, ar 2 , ....
or a – 2 = 0, then a = 2 2, 6, 18, 54, ...
PQ. The fourth term of a G.P. is the square of its
1
Hence a = 2 or second term and the first term is –3.
2 Determine its seventh term.
PQ. The first and the second terms of a G.P. are Sol. In a G.P.
x–4 and xm. If its 8th term is x52, then find a4 is square of a2 i.e. a4 = (a2)2
the value of m.
a1 = –3
Sol. In a G.P.,
Let a be the first term and r be the common
First term (a1) = x–4 ...(i) ratio, then
Second term (a2) = x m
a4 = arn – 1 = ar4 – 1 = ar3
Eighth term (a8) = x52 and a2 = ar2 – 1 = ar
a2 xm  ar3 = (ar)2
r = a =  4 = xm + 4 ...(ii)  ar3 = a2r2
1 x
Now, a8 = arn – 1 = ar8 – 1 = ar7 r3 a2
x52 = x–4 × r7 = x–4 × x7(m + 4)  =  r = a = –3 ( a1 = –3)
r2 a
x52 = x–4 + 7m + 28 = x7m + 24 Now, a7 = ar7 – 1 = ar6 = (–3) (–3)6 = (–3)7
On comparing, we have = –2187
52 = 7m + 24  7m = 52 – 24 = 28
39
28 11. The sum of first three terms of a G.P. is
 m= =4 10
7 and their product is 1. Find the common ratio
217 Arun Deep's Understanding Math-10
and the terms. Then terms of G.P. be, when
39 5 25 125
Sol. Sum of first three terms of G.P. = 1, , , , ...
10 2 4 8
and their product = 1
2
Let a be the first term and r be the common if r = , then terms of G.P. will be
5
ratio, then
a 2 4 8
Let , a, ar be the three terms of G.P., then 1, , , , ...
r 5 25 125
12. Three numbers are in A.P. and their sum is
a 39 a 15. If 1, 4 and 19 are added to these numbers
+ a + ar = and × a × ar = 1
r 10 r respectively, the resulting numbers are in
G.P. Find the numbers.
1  39
 a  1 r  = .....(1) Sol. Given : Three numbers are in A.P.
r  10
and their sum = 15
and a3 = 1  a = 1
Let a – d, a, a + d be the three numbers in
Substitute the value of a in eqn. (1), we get A.P.
1  39  a – d + a + a + d = 15  3a = 15
 1  1 r  =
r  10 15
 a= =5
3
1 r  r2 39
 = By adding 1, 4, 19 in then, we get
r 10
 10 + 10r + 10r2 = 39r a – d + 1, a + 4, a + d + 19
 10r2 + 10r – 39r + 10 = 0 These numbers are in G.P.
 10r2 – 29r + 10 = 0  (a + 4)2 = (a – d + 1) (a + d + 19)
 a2 + 8a + 16 = a2 + ad + 19a – ad – d2 – 19d
 10  10  100  + a + d + 19
 
 100  25  (4)  a2 + 8a + 16 = a2 – d2 – 18d + 20a + 19
  29  25  4 
   8a + 16 = 20a – 18d – d2 + 19
 10r2 – 25r – 4r + 10 = 0  8a + 16 – 20a + 18d + d2 – 19 = 0
 5r(2r – 5) – 2(2r – 5) = 0  d2 + 18d – 12a – 3 = 0
 (2r – 5) (5r – 2) = 0  d2 + 18d – 12 × 5 – 3 = 0
 d2 + 18d – 60 – 3 = 0
5
Either 2r – 5 = 0, then r =
2  d2 + 18d – 63 = 0

2   63  21  3
or 5r – 2 = 0, then r =  
5   18  21  3 

5 2  d2 + 21d – 3d – 63 = 0
Hence r = or  d(d + 21) – 3(d + 21) = 0
2 5
218 Arun Deep's Understanding Math-10
 (d + 21) (d – 3) = 0
42 3
Either d + 21 = 0, then d = –21 = =2+ 3
2
or d – 3 = 0, then d = +3
If d = 3 and a = 5, then numbers becomes,  r = 2+ 3

5 – 3, 5, 5 + 3 i.e. 2, 5, 8 The numbers are increasing.


If d = –21, then numbers becomes ;  r=2+ 3
5 + 21, 5, 5 – 21 14. Three numbers whose sum is 70 are in G.P.
 26, 5, –16 If each of the extremes is multiplied by 4
and the mean by 5, the numbers will be in
13. Three numbers form an increasing G.P. If
A.P. Find the numbers.
the middle term is doubled, then the new
numbers are in A.P. Find the common ratio Sol. Three numbers are in G.P.
of the G.P. a
Sol. Three numbers form an increasing G.P. Let the three numbers be, , a, ar
r
a a 1 
Let
r
, a, ar be three numbers in G.P..  + a + ar = 70  a   1  r  = 70...(i)
r r 
Double the middle term, we get By multiplying the extremes by 4 and mean
by 5, then numbers becomes ;
a
, 2a, ar will be in A.P..
r a
× 4, a × 5, ar × 4
r
a
If 2(2a) = + ar 4a
r
, 5a, 4ar
r
1 
If 4a = a   r  But these are in A.P.
r 
4a
 2(5a) = + 4ar
1 r
If 4 = + r  4r = 1 + r2
r
1 
 r2 – 4r + 1 = 0  10a = 4a   r 
 r 

 b  b 2  4ac 1 
 r=  5 = 2  r  5r = 2 + 2r2
2a r 
 2r2 – 5r + 2 = 0
 ( 4)  (4) 2  4  1  1
=  2r2 – r – 4r + 2 = 0
2 1
 2  2  4 
4  16  4 4  12  
= =  4  1  ( 4) 
2 2   5  1  4 
 
219 Arun Deep's Understanding Math-10
 r(2r – 1) – 2(2r – 1) = 0  (a + c)2 – 4ac = 0  (a – c)2 = 0
 (2r – 1) (r – 2) = 0  a–c=0a=c ...(iii)
1 From (i), 2b = a + c = a + a = 2a
Either 2r – 1 = 0, then r =  b=a ...(iv)
2
From (iii) and (iv), we have
or r – 2 = 0, then r = 2
From (i), we have a=b=c
(ii) a, b, c are in A.P. as well as in G.P.
1   2b = a + c
a   1  2  = 70
 2  and b2 = ac
7 2 and a = b = c [Proved in (i)]
 a = 70  a = 70 × = 20 Now, a b – c +b c – a +c a – b
2 7
 Numbers are (if r = 2) Since, a = b = c
 b – c = 0, c – a = 0 and a – b = 0
20
, 20, 20 × 2 i.e., 10, 20, 40  a 0 + b 0 + c0 = 1 + 1 + 1 = 3 ( x0 = 1)
2
16. The terms of a G.P. with first term a and
1 common ratio r are squared. Prove that
If r = , then numbers are ; resulting numbers form a G.P. Find its first
2
term, common ratio and the nth term.
20 1 20  2 1 Sol. In a G.P.,
, 20, 20 × i.e. , 20, 20 × First term = a and common ratio = r
1 2 1 2
2  G.P. is a, ar, ar2
Squaring, we get
i.e. 40, 20, 10.
a2, a2r2, a2r4 are in G.P.
15. (i) If a, b, c are in A.P. as well in G.P., prove if b2 = ac
that a = b = c.  (a2r2)2 = a2 × a2r4 a4r4 = a4r4
(ii) If a, b, c are in A.P. as well as in G.P., then Which is true
find the value of ab – c + bc – a + ca – b. The first term is a2
Sol. (i) a, b, c are in A.P. as well as in G.P. and common ratio is r2
To prove : a = b = c The nth term will be
a, b, c are in A.P. an = arn – 1 = a2(rn – 1)2 = a2r2n – 2
17. Show that the products of the
ac corresponding terms of two G.P.'s a, ar,
 2b = a + c  b = ...(i)
2 ar2, ..., arn – 1 and A, AR, AR2, ..., ARn – 1
 a, b, c are in G.P. form a G.P. and find the common ratio.
Sol. It has to be proved that the sequence aA,
 b2 = ac ...(ii)
arAR, ar2AR2 ..., arn – 1ARn – 1, forms a G.P.
2
ac Second term arAR
  = ac Hence,
 2  First term = aA = rR

( a  c) 2 Third term ar 2 AR 2
 = ac  (a + c)2 = 4ac and Second term = = rR
4 arAR
220 Arun Deep's Understanding Math-10
Thus, the above sequence forms a G.P. and  2q = p + r
the common ratio is rR. i.e. p, q, r are in A.P. which is given
1 1 1 Hence proved.
18. (i) If a, b, c are in G.P. show that , ,
a b c 19. If a, b, c are in G.P., prove that the following
are also in G.P. are also in G.P.
(ii) If K is any positive real number and Ka, Kb, (i) a3, b3, c3
Kc are three consecutive terms of a G.P., (ii) a2 + b2, ab + bc, b2 + c2.
prove that a, b, c are three consecutive terms
Sol. (i) Given, a, b, c are in G.P.
of an A.P.
(iii) If p, q, r are in A.P., show that pth, qth and  b2 = ac
rth terms of any G.P. are themselves in G.P. Now, a3, b3, c3 are in G.P.
Sol. (i) a, b, c are in G.P. if (b3)2 = a3 × c3
 b2 = ac if (b2)3 = (a × c)3
1 1 1 if b2 = ac
, , will be in G.P.
.P.
a b c which is given
2 Hence proved.
1 1 1 1 1
if   = ×  2 = 2 2 2 2
(ii) a + b , ab + bc, b + c will be in G.P.
b a c b ac
 ac = b2 (By cross multiplication) ab  bc b2  c2
If 2 =
which is given a b
2
ab  bc
Hence proved. a, b, c are in G.P.
(ii) K is any positive number  b = ar and c = ar2
Ka, Kb, Kc are in G.P.
then (Kb)2 = Ka × Kc a( ar )  ar (ar 2 ) ( ar ) 2  ( ar 2 ) 2
i.e., if =
 K2b = Ka + c a 2  (ar ) 2 a( ar )  ar (ar 2 )
 2b = a + c
Hence, a, b, c are in A.P. a 2 r (1  r 2 ) a 2 r 2 (1  r 2 )
 If 2 = 2
(iii) p, q, r are in A.P. a (1  r 2 ) a r (1  r 2 )
 2q = p + r  If r = r which is true.
pth term in G.P. = ARp – 1  a2 + b2, ab + bc, b2 + c2 are in G.P.
qth term = ARq – 1 PQ. If a, b, c, d are in G.P., show that
rth term = ARr – 1 (i)a2 + b2, b2 + c2, c2 + d2 are in G.P.
Where A = first term and R = Common ratio (ii)(b – c)2 + (c – a)2 + (d – b)2 = (a – d)2.
These will be in G.P. Sol. a, b, c, d are in G.P.
if (ARq – 1)2 = ARp – 1 × ARr – 1 Let r be the common ratio, then
if A2R2q – 2 = A2Rp – 1 + r – 1 a=a
if A2R2q – 2 = A2Rp + r – 2 b = ar, c = ar2, d = ar3
if R2q – 2 = Rp + r – 2 (i) a2 + b2, b2 + c2, c2 + d2 are in G.P.
On comparing, we get  a2 + b2 = a2 + a2r2 = a2(1 + r2)
2q – 2 = p + r – 2 b2 + c2 = a2r2 + a2r4 = a2r2(1 + r2)
221 Arun Deep's Understanding Math-10
c2 + d2 = a 2r 4 + a 2r 6 = a2r4(1 + r 2) (v) n terms of the G.P. 7, 21 , 3 7 , ...
a2 + b 2, b2 + c 2, c2 + d2 will be in G.P.
(vi) n terms of the G.P. 1, –a, a 2 , –a 3 , ...
if (b2 + c 2) 2 = (a2 + b 2) (c2 + d 2) (a  –1)
Now, (b2 + c 2) 2 = [a2r2(1 + r2)]2 (vii) n terms of the G.P. x3, x5, x7, ... (x  +1).
= a4r4(1 + r2)2 ...(i) Sol. (i) Given series be 2 + 6 + 18 + ... 20 terms
(a2 + b 2) (c2 + d 2) Here, a = 2, r = 3, n = 20, r > 1
= [a2(1 + r2)][a2r4(1 + r2)]2
a(r n  1)
= a4r4(1 + r 2) 2 ...(ii) Sn =
r 1
From (i) and (ii), we have
(b2 + c2)2 = (a2 + b2) (c2 + d2) 2(320  1) 2(320  1)
 S20 = = = 320 – 1
3 1 2
Hence, a2 + b2, b2 + c2, c2 + d2 are in G.P.
(ii) Show that (ii) Given series be, 1 + 3 + 3 + ... 10 terms
(b – c)2 + (c – a)2 + (d – b)2 = (a – d)2
Here, a = 1, r = 3 , n = 10, r > 1
L.H.S. = (b – c)2 + (c – a)2 + (d – b)2
= (ar – ar2)2 + (ar2 – a)2 + (ar3 – ar)2 a(r10  1) 1[( 3 )10  1]
 S10 = =
= a2r2(1 – r)2 + a2(r2 – 1)2 + a2r2(r2 – 1)2 r 1 3 1
= a2[r2(1 + r2 – 2r) + r4 – 2r2 + 1 + r2(r4 –
2r2 + 1)] (r n  1)
[ Sn = a ]
r 1
= a2[r2 + r4 – 2r3 + r4 – 2r2 + 1 + r6 – 2r4 +
r2)] 10
( 3  1) ( 3  1)
= a2(r6 – 2r3 + 1) =
( 3  1) ( 3  1)
R.H.S. = (a – d)2 = (a – ar3)2 = a2(1 – r3)2
(Rationalising the denominator)
= a2(1 + r6 – 2r3)
= a2[r6 – 2r2 + 1] (35  1) ( 3  1) (243  1) ( 3  1)
 S10 = =
 L.H.S. = R.H.S. 3 1 2
EXERCISE 9.5 242 ( 3  1)
= = 121( 3 + 1)
1. Find the sum of : 2
(i) 20 terms of the series 2 + 6 + 18 + ... 2 4
(iii) Gives series be, 1,  , , ... 6 terms
(ii) 10 terms of series 1 + 3 + 3 + ... 3 9

2 4 2
(iii) 6 terms of the G.P. 1,  , , ... Here, a = 1, r =
3
, n = 6, r < 1
3 9
2    2 6 
(iv) 5 terms and n terms of the series 1 + +
3 1 1    
a (1  r 6 )   3  
4  S6 = = 2
9
+ ... 1 r 1
3
222 Arun Deep's Understanding Math-10

3  (2) 6  7 [( 3 ) n  1] 3 1
1    Sn = ×
=  36  3 1 3 1
5 

3  64  3  729  64  7 [( 3 ) n  1] ( 3  1)
= 1  729  = 5  729  =
( 3 ) 2  (1) 2
5    

3 665 133 7 [( 3 ) n  1] ( 3  1)
= × = =
5 729 243 3 1
(iv) Given series be,
7
2 4 = [( 3 )n – 1] ( 3 + 1)
1+ + + ... n terms, 5 terms 2
3 9
(vi) 1, –a, a2, –a3, ... (a  –1) upto n terms
2 2
Here, a = 1, r = ÷ 1 = , n = 5, n, (r < 1) Here, A = 1, r = –a,
3 3
A(1  r n ) 1[1  ( a) n ] 1  (a) n
  2 n  Sn = = =
1 r 1  ( a ) 1 a
1 1    
a(1  r n )   3   (vii) Given series be, x3, x5, x7, ... (x  +1)
 Sn = = 2
1 r 1 Here, a = x3, r = x2
3
a(1  r n ) x 3 [1  ( x 2 ) n ]
 Sn = = if r < 1
1 3 1   2     2 n 
n
1 r 1  x2
=     = 3 1   3  
1   3       x 3 (1  x 2 n )
=
1  x2
  2 5   32 
and S5 = 3 1   3   = 3 1   a(r n  1) x 3 [( x 2 ) n  1]
     243 
or Sn = =
1 r x2 1
 243  32  211
= 3  = x 3 ( x 2 n  1)
 243  81 =
x2 1
(v) Given series be,
PQ. Find the sum of the first 10 terms of the
7, 21 , 3 7 , ... geometric series

21 7 3 2 + 6 + 18 + ...
Here, a = 7 , r = = = 3>1
7 7 Sol. Given series be, 2 + 6 + 18 + ...
i.e. r > 1,
Here, a = 2,r= 3,r>1
a(r  1)
n
7 [( 3 ) n  1]
Now, Sn =  Sn = a(r n  1)
r 1 3 1  S10 =
r 1
Rationalising the denominator, we have
223 Arun Deep's Understanding Math-10

 S10 =

2 ( 3 )10  1 = 2
[(3)5 – 1] Tn = arn – 1  128 = a 2n – 1  a =
128
3 1 3 1 2 n 1
...(i)
2 2 a(r n  1) a(2 n  1)
= (243 – 1) = × 242  Sn =  255 =
3 1 3 1 r 1 2 1

2 ( 3  1)  242 255
=  255 = a (2n – 1)  a = ...(ii)
( 3  1) ( 3  1) 2n  1
From (i) and (ii), we have
(Rationalising the denominator)
255 128
242 ( 6  2 ) 242( 6  2)  = n 1  255 × 2n – 1 = 128(2n – 1)
= = 2 1 2
n

3 1 2
 255 × 2n – 1 = 128 × 2n – 128
= 121( 6 + 2) 255  2 n
= 128 × 2n – 128
2. Find the sum of the series 81 – 27 + 9 – ... 2
1  255 × 2n = 256 × 2n – 256
– .
27  256 × 2n – 255 × 2n = 256
 2n = 256 = 28
1
Sol. Given series be, 81 – 27 + 9 – ... – On comparing, we get
27
n=8
27 1 1 Now, 128 = a·27  128 = a × 128
Here, a = 81, r = = ,l= ,r<1
81 3 27
128
 a= =1
1 1 1 128
81   81 
a  lr 27 3 81  a=1
 Sn = = =
1 r 1
1 4 PQ. If the sum of first six terms of any G.P. is
3 3 equal to 9 times the sum of the first three
terms, then find the common ratio of the
6561  1 6560  3 1640 G.P.
= = =
4 81  4 27 Sol. Sum of first 6 terms of a G.P. = 9 × The
81 
3 sum of first 3 terms
3. The nth term of a G.P. is 128 and the sum of Let a be the first term and r be the common
its n terms is 255. If its common ratio is 2, ratio of given G.P.
then find its first term.  S6 = 9 × S3
Sol. In a G.P.
a(r n  1)
Tn = 128 Sn =
r 1
Sn = 255
r = 2, a(r 6  1) a (r 3  1)
 S6 = and S3 =
Let a be the first term, then r 1 r 1
224 Arun Deep's Understanding Math-10

a(r 6  1) a (r 3  1) 1(4 n  1)
 =9×  = 341
r 1 r 1 4 1
 r6 – 1 = 9(r3 – 1)
4n  1
 = 341
r6 1 (r 3  1) (r 3  1) 3
 = 9  =9
r3 1 r3 1  4n – 1 = 341 × 3 = 1023
 r3 + 1 = 9  r3 = 9 – 1 = 8 = (2)3 4n = 1023 + 1 = 1024 = 210 = 45
 r=2
 Required common ratio = 2 2 1024
4. (i) How many terms of the G.P. 3, 32, 33, ... 2 512
2 256
are needed to give the sum 120?
2 128
(ii) How many terms of the G.P. 1, 4, 16, ... 2 64
must be taken to have their sum equal to 2 32
341? 2 16
Sol. In G.P. 2 8
2 4
(i) 3, 32, 33, ... 2 2
1
32
Sum = 120, Here, a = 3, r = = 3, r > 1
3  n=5
Let number of terms in G.P. be n, then  Required number of terms = 5

a(r n  1) PQ. How many terms of the G.P. 1, 2 , 2, 2 2 , ...


Sn = = 120
r 1 are required to give a sum of 1023( 2 + 1)?

3(3n  1) 3(3n  1) Sol. Given, G.P. is 1, 2 , 2, 2 2 , ...,


 = 120  = 120
3 1 2
and given sum = 1023 ( 2 + 1)
120  2
 3n – 1 = = 80 Here, a = 1, r = 2,r>1
3
Let the required number of terms be n, then
3n = 80 + 1 = 81 = 34
 n=4 a(r n  1)
Sn = = 1023( 2 + 1)
 Required number of terms = 4 r 1
(ii) Given, G.P. is 1, 4, 16, ...
1[( 2 ) n  1]
Sum = 341  = 1023( 2 + 1)
2 1
4
Here, a = 1, r = = 4, r > 1  ( 2 )n – 1 = 1023( 2 + 1) ( 2 – 1)
1
Let number of terms be n, then  ( 2 )n – 1 = 1023 [( 2 )2 – (1)2]
a(r n  1)  ( 2 )n – 1 = 1023(2 – 1) = 1023
Sn = = 341
r 1
225 Arun Deep's Understanding Math-10
 ( 2 )n = 1023 + 1 = 1024 = 210 or ( 2 )20 n
  3 55 5 9 275
 1–   = × × =
2 1024  2  72 2 2 32
2 512
n
2 256   3 275 243
2 128    =1– =
 2  32 32
2 64
2 32 n 5
2 16  3  3 
2 8    =  
 2   2 
2 4
2 2 On comparing, we get
1 n=5
On comparing, we get  Required number of terms = 5
n = 20 6. The 2nd and 5th terms of a geometric series
5. How many terms of the series 1 1
2 1 1 55 are  and respectively. Find the sum
2 16
– + + ... will make the sum ?
9 3 2 72 of the series upto 8 terms.
2 1 1 Sol. In a given G.P.
Sol. Given, G.P. is – + + ...
9 3 2 1 1
a2 = and a5 =
55 2 16
and Sum =
72 Let a be the first term and r be the common
ratio of given G.P.
2 1 9 3
Here, a = ,r= × = ,r<1 1
9 3 2 2  a2 = arn – 1 = ar2 – 1 = ar = ...(i)
2
Let required number of terms of G.P. be n
1
a(1  r n ) 55 a5 = ar5 – 1 = ar4 = ...(ii)
 Sn = = 16
1 r 72
Dividing (ii) by (i), we have

2   3 
n 1  1 1 2 1
1     r3 = ÷   = × =
9   2   55 16  
2 16 1 8
 =
3 72 3
1  1
2 =  
 2 

1
2   3 
n
1    r=
  2
9   2   55
 =
5 72 1  1 1
and ar = a×   =
2 2  2  2
226 Arun Deep's Understanding Math-10
1 2
 a= × =1   1 10 
2 1 27 1    
a(1  r n )   3  
1 Now, S10 = =
Thus, a = 1, and r = 1 r 1
2 1
3
a(1  r n )
Now, Sn =
1 r  1 
27 1  10 
 3 
   1 8  = 3 1
1 1    
  2   3
 S8 =
1
1 27  3  1  81  1 
2 = 1  310  = 1  310 
2   2  
1 8. Find the first term of the G.P. whose
1
256 common ratio is 3, last term is 486 and the
= 3 sum of whose terms is 728.
2 Sol. Common ratio of a G.P. = 3
255 2 510 85 and last term = 486
= × = = and sum of terms = 728
256 3 768 128
7. The first term of a G.P. is 27 and 8th term is We know that

1 a(r n  1) a(3n  1)
. Find the sum of its first 10 terms. Sn = =
81 r 1 3 1
Sol. In a given G.P.
a(3n  1)
First term (a) = 27 = = 728
2
1  a(3n – 1) = 728 × 2 = 1456 ...(i)
a8 =
81  l = 486  arn – 1 = a·3n – 1 = 486
Let r be the common ratio, then
 3n 
1  a  3  = 486  a3n = 486 × 3 = 1458
arn – 1 = ar8 – 1 = ar7 =  
81
...(ii)
1 But a(3n – 1) = 1456 [From (i)]
 27r7 =
81 a3n – a = 1456
1 1458 – a = 1456 [From (ii)]
1 1
 r7 = = =  a = 1458 – 1456 = 2
81  27 2187 ( 3)7
Hence required first term = 2
1 9. In a G.P. the first term is 7, the last term is
 r= 448, and the sum is 889. Find the common
3
227 Arun Deep's Understanding Math-10
ratio. 4372
Sol. In a G.P.  4372 = 2186a  a = =2
2186
First term (a) = 7, last term (l) = 448
 a3 = ar3 – 1 = ar2 = 2 × 32 = 2 × 9 = 18
and sum = 889
11. If the first term of a G.P. is 5 and the sum of
Let r be the common ratio, then
31
l = arn – 1 first three terms is , find the common
5
 7rn – 1 = 448
ratio.
448 Sol. In a G.P.
 rn – 1 = = 64 ...(i)
7 First term (a) = 5
a(r  1)
n
31 a (r 3  1) 5(r 3  1)
and sum = = 889 and S3 = = =
r 1 5 r 1 r 1

7( r n  1) rn 1 889 r3 1 31 31
 = 889  = = 127 = =
r 1 r 1 7 r 1 55 25
...(ii)
From (i), we have (r  1) ( r 2  r  1) 31
 =
r 1 25
rn
= 64  rn = 64r
r 31
r2 + r + 1 =
25
64r  1
From (ii) ;
r 1
= 127  25r2 + 25r + 25 = 31
 25r2 + 25r – 6 = 0
 64r – 1 = 127r – 127
 25r2 + 30r – 5r – 6 = 0
 127r – 64r = –1 + 127  63r = 126
 5r(5r + 6) – 1(5r + 6) = 0
126
 r= =2  (5r + 6) (5r – 1) = 0
63
6
Hence required common ratio = 2 Either 5r + 6 = 0, then r =
5
10. Find the third term of a G.P. whose common
ratio is 3 and the sum of whose first seven 1
terms is 2186. or 5r – 1 = 0, then r =
5
Sol. In a G.P.
r = Common ratio = 3 1 6
Hence common ratio = or
5 5
a(r n  1) a(3n  1)
S7 = 2186 =  2186 = 12. The sum of first three terms of a G.P. is to
r 1 3 1 the sum of first six terms as 125 : 152. Find
the common ratio of the G.P.
a(3n  1)
 2186 =  4372 = a(37 – 1) Sol. Given, S3 ÷ S6 = 125 : 152
2
Let r be the common ratio and a be the first
 4372 = a(2187 – 1)
number, then
228 Arun Deep's Understanding Math-10
Sn = x2 + x4 + x6 + ... n terms + xy + x2y2
a ( r 3  1) a(r 6  1)
: = 125 : 152 + x3y3 + ...
r 1 r 1
(r3 – 1) : (r6 – 1) = 125 : 152 x 2 [( x 2 ) n  1] xy [( xy ) n  1]
Sn = +
(r3 – 1) : (r3 + 1)(r3 – 1) = 125 : 152 x2 1 xy  1
1 : (r3 + 1) = 125 : 152
In the first G.P. a  x 2 , r  x 2 
 (r3 + 1) × 125 = 152 × 1  
 125r3 + 125 = 152 In second G.P. a  xy, r  xy 
 125r3 = 152 – 125 = 27
x 2 ( x 2 n  1) xy[( xy) n  1]
Sn = +
27 3
3
x2 1 xy  1
 r3 = =  
125 5 15. Find the sum of the series

3 1 + (1 + x) + (1 + x + x2) + ... to n terms,


 r= x  1.
5
Sol. Let S = 1 + (1 + x) + (1 + x + x2) + ... n
3 terms, x  1
 Required common ratio =
5 Multiply and divide by (1 – x) ; we have
50 (1  x ) (1  x) (1  x)
13. Evaluate  (2
n 1
n
 1) . S = (1  x ) + (1  x)

50 (1  x) (1  x  x 2 )
Sol. Let Sn =  (2
n 1
n
 1) +
(1  x)
+ ... upto n terms

Here, n = 1, 2, 3, ..., 50 1
= (1  x ) [(1 – x) + (1 – x2) + (1 – x3) + ...
Sn = 21 – 1 + 22 – 1 + 23 – 1 + 24 – 1 + ...
+ 250 – 1 upto n terms]
 Sn = (21 + 22 + 23 + 24 ... 250) – 1 × 50
1
 Sn = (2 + 4 + 8 + 16 ... 250) – 50 = [(1 + 1 + 1 + ... upto n terms) – (x
(1  x )
a(r n  1) 2( 250  1) + x2 + x3 + ... upto n terms)]
 Sn = – 50 = – 50
r 1 2 1
1  x (1  x n ) 
Sn = 2 × 250 – 2 – 50  n  
= (1  x ) (1  x) 
Sn = 251 – 52 
14. Sum the series
 a (1  r n ) 
x(x + y) + x2 (x2 + y2) + x3 (x3 + y3) + ... to  Using Sn  as r  1
n terms.  1 r 
Sol. Let Sn = x(x + y) + x2 (x2 + y2) + x3 (x3 + y3)
+ ... n terms 1  n(1  x)  x(1  x n ) 
Sn = x2 + xy + x4 + x2y2 + x6 + x3y3 + ... n = (1  x )  (1  x)

 
terms
229 Arun Deep's Understanding Math-10

1 8
 S = (1  x) 2 [n(1 – x) – x(1 – xn)] = [10 + 100 + 1000 + ... n terms – 1 × n]
9

8 10(10  1)  n 
16. Find the sum of the following series to n n

terms : =  
9  10  1 
(i) 7 + 77 + 777 + ...
n 1
(ii) 8 + 88 + 888 + ... 8 10  10  n 
=  
(iii) 0.5 + 0.55 + 0.555 + ... 9  9 
Sol. (i) 7 + 77 + 777 + ... n terms
8
= 7[1 + 11 + 111 + ... n terms] = [10n + 1 – 10 – 9n]
81
7 8
= [9 + 99 + 999 + ... n terms] = [10n + 1 – 9n – 10]
9 81

7 (iii) 0.5 + 0.55 + 0.555 + ... n terms


= [(10 – 1) + (100 – 1) + (1000 – 1) + ... = 5[0.1 + 0.11 + 0.111 + ... n terms]
9
n terms] 5
= [0.9 + 0.99 + 0.999 + ... n terms]
9
7
= [10 + 100 + 1000 + ... n terms – (1 + 1
9 5
+ 1 ... n terms] = [(1 – 0.1) + (1 – 0.01) + (1 – 0.001) +
9
... n terms)
7 10(10  1)  n 
n

=   5
9  10  1  = [1 + 1 + 1 + ... n terms – (0.1 + 0.01 +
9
0.001) + ... n terms]
7 10  10  10  n 
n

=  
9  9  5  n  0.1[1  (0.1) 
n
=  
9  1  0.1 
7
= [10n + 1 – 10 – 9n]
81
5  1 1 
7 =  n  1  n  
= [10n + 1 – 9n – 10] 9  9  10 
81
(ii) 8 + 88 + 888 + ... n terms 5  1 
= 9n  1  10 n 
= 8[1 + 11 + 111 + ... n terms] 81  

8 MULTIPLE CHOICE QUESTIONS


= [9 + 99 + 999 + ... n terms]
9
Choose the correct answer from the given
8 four options (1 to 27) :
= [(10 – 1) + (100 – 1) + (1000 – 1) + ... 1. The 10th term of the A.P. 5, 8, 11, 14, ... is
9
n terms] (a) 32 (b) 35
230 Arun Deep's Understanding Math-10
(c) 38 (d) 185 (a) 37 (b) 40
Sol. Given A.P. be, 5, 8, 11, 14, ... (c) 43 (d) 58
{ a = 5, d = 3}. Sol. 4th term from the end of the A.P. –11, –8,
 a + (n – 1)d = 5 + (10 – 1) × 3 –5, ..., 49 is

= 5 + 9 × 3 = 5 + 27 = 32  Ans. (a) Here, a = –11, d = –8 – (–11) = –8 + 11 = 3


and l = 49
2. The 30th term of the A.P. 10, 7, 4, ... is
 l = 49 = a + (n – 1)d
(a) 87 (b) 77
 49 = –11 + (n – 1) × 3
(c) –77 (d) –87
 49 + 11 = 3(n – 1)
Sol. Given, A.P. 10, 7, 4, ... is
Tn = a + (n – 1)d 60
 =n–1
3
 a  10 
   n = 20 + 1 = 21
 d  7  10  3
Now, 4th term from the end = l – (n – 1)d
 T30 = 10 + (30 – 1) × (–3) = 49 – (4 – 1) × 3 = 49 – 9 = 40 Ans.(b)
= 10 + 29(–3) = 10 – 87 = –77 4. The 15th term from the last of the A.P. 7,
 Ans. (c) 10, 13, ..., 130 is
(a) 49 (b) 85
1
3. The 11th term of the A.P. –3,  , 2, ... is (c) 88 (d) 110
2
Sol. Given, A.P. be, 7, 10, 13, ..., 130
(a) 28 (b) 22
Here, a = 7, d = 10 – 7 = 3, l = 130
1
(c) –38 (d) –48  15th term from the end = l – (n – 1)d
2
= 130 – (15 – 1) × 3
1 = 130 – 42 = 88  Ans. (c)
Sol. Given, A.P. be, –3,  , 2, ...
2 5. If the common difference of an A.P. is 5,
then a18 – a13 is
1 1 5
a = –3, d =  – (–3) =  + 3 = (a) 5 (b) 20
2 2 2
(c) 25 (d) 30
 Tn = a + (n – 1)d
Sol. Common difference of an A.P. (d) = 5
5
 T11 = –3 + (11 – 1) × a18 – a15 = a + 17d – a – 12d
2
= 5d = 5 × 5 = 25  Ans. (c)
5 6. In an A.P., if a 18 – a 14 = 32 then the
= –3 + 10 × = – 3 + 25 = 22  Ans.(b)
2 common difference is
P.Q. The 4th term from the end of the A.P. –11, (a) 8 (b) –8
–8, –5, ..., 49 is (c) –4 (d) 4
231 Arun Deep's Understanding Math-10
Sol. Given, a18 – a14 = 32, then d = ? 9. Which term of the A.P. 21, 42, 63, 84, ... is
(a + 17d) – a – 13d = 32 210?
 a + 17d – a – 13d = 32 (a) 9th (b) 10th
(c) 11th (d) 12th
32
 4d = 32  d = =8  Ans. (a) Sol. Which term of an A.P. 21, 42, 63, 84, ...
4
is 210
7. In an A.P., if d = –4, n = 7, an = 4, then a is Let 210 be the nth term, of A.P. then
(a) 6 (d) 7 Here, a = 21, d = 42 – 21 = 21
(c) 20 (d) 28  210 = a + (n – 1)d
Sol. In an A.P., d = –4, n = 7, an = 4 then a = ? 210 = 21 + (n – 1) × 21
an = a + (n – 1)d = 4  210 – 21 = 21(n – 1)
a7 = a + (7 – 1)d = 4
189
 a + 6d = 4  = n – 1  9 = n – 1  n = 9 + 1 = 10
21
 a + 6 × (–4) = 4
 Required term be 10th term.
 a – 24 = 4  a = 4 + 24 = 28  Ans. (d)
 Ans. (b)
8. In an A.P., if a = 3.5, d = 0, n = 101, then an
10. If the last term of the A.P. 5, 3, 1, –1, ... is
will be
–41, then the A.P. consists of
(a) 0 (b) 3.5
(a) 46 terms (b) 25 terms
(c) 103.5 (d) 104.5
(c) 24 terms (d) 23 terms
Sol. In an A.P. Sol. Last term of an A.P. 5, 3, 1, –1, ... is –41
a = 3.5, d = 0, n = 101, then an = ? Then A.P. will consist of ___ terms
an = a101 = a + (101 – 1)d Here, a = 5, d = 3 – 5 = –2 and n = ___,
= 3.5 + 100d = 3.5 + 100 × 0 l = –41
= 3.5 + 0 = 3.5  Ans. (b) l = –41 = a + (n – 1)d
P.Q. In an A.P., if a = –7.2, d = 3.6, an = 7.2,  –41 = 5 + (n – 1) (–2)
then n is
46
(a) 1 (b) 3  –41 – 5 = (n – 1) (–2)  =n–1
2
(c) 4 (d) 5
 n – 1 = 23  n = 23 + 1 = 24
Sol. In an given, A.P.
 A.P. consists of 24 terms.  Ans. (c)
a = –7.2, d = 3.6, an = 7.2, n = ?
11. If k – 1, k + 1 and 2k + 3 are in A.P., then
an = 7.2
the value of k is
 a + (n – 1)d = 7.2
(a) –2 (b) 0
 –7.2 + (n – 1) 3.6 = 7.2 (c) 2 (d) 4
 (n – 1) × 3.6 = 7.2 + 7.2 = 14.4 Sol. k – 1, k + 1 and 2k + 3 are in A.P.
14.4  2(k + 1) = (k – 1) + (2k + 3)
 (n – 1) = =4
3. 6  2k + 2 = k – 1 + 2k + 3  2k + 2 = 3k + 2
n=4+1=5  Ans. (d) 3k – 2k = 2 – 2  k = 0  Ans. (b)
232 Arun Deep's Understanding Math-10
12. The 21st term of an A.P. whose first two 2
terms are –3 and 4 is  , ... is
3
(a) 17 (b) 137
(c) 143 (d) –143 2
(a) 0 (b) 
Sol. First two terms of an A.P. are –3 and 4 3
 a = –3, d = 4 – (–3) = 4 + 3 = 7 50
(c)  (d) –50
21st term = a + 20d 3
= –3 + 20(7) = –3 + 140 = 137  Ans. (b) 2 2 2
P.Q. If the 2nd term of an A.P. is 13 and the 5th Sol. Sum of 25 terms of an A.P.  ,  ,  ,
3 3 3
term is 25, then its 7th term is ... is
(a) 30 (b) 33
n
(c) 37 (d) 38 = [2a + (n – 1)d]
2
Sol. In an A.P.
2nd term = 13  a + d = 13 ...(i) 25  (2) 
= 2  3  (25  1)  0
5th term = 25  a + 4d = 25 ...(ii) 2  
Subtracting (i) and (ii), we have
25  4 50
12 =  3  =  Ans. (c)
3d = 12  d = =4
2   3
3
15. In an A.P., if a = 1, an = 20 and Sn = 399,
Substitute the value of d in eq. (i), we get then n is
 a = 13 – 4 = 9 (a) 19 (b) 21
Thus, 7th term = a7 = a + 6d = 9 + 6 × 4 (c) 38 (d) 42
= 9 + 24 = 33  Ans. (b) Sol. In an A.P., Given a = 1, an = 20, Sn = 399
13. If the first term of an A.P. is –5 and the Since, an = a + (n – 1)d = 20
common difference is 2, then the sum of its  1 + (n – 1)d = 20
first 6 terms is
 (n – 1)d = 20 – 1 = 19 ...(i)
(a) 0 (b) 5
(c) 6 (d) 15 n
Also, Sn = [2a + (n – 1)d]
Sol. First term (a) of an A.P. = –5 2
Common difference (d) is 2 n
399 = [2 × 1 + 19] [From (i)]
n 2
 Sum of first 6 terms = [2a + (n – 1)d]
2 n
 399 = × 21
6 2
= [2 × (–5) + (6 – 1) × 2]
2
399  2
= 3[–10 + 5 × 2] = 3 × [–10 + 10]  n= = 38
21
=3×0=0  Ans. (a)
 n = 38  Ans. (c)
2 2 16. In an A.P., if a = –5, l = 21 and Sn = 200,
14. The sum of 25 terms of the A.P.  ,  ,
3 3 then n is equal to
233 Arun Deep's Understanding Math-10
(a) 50 (b) 40 Sol. First 5 multiples of 3 :
(c) 32 (d) 25 3, 6, 9, 12, 15
Sol. In an A.P. Here, a = 3, d = 6 – 3 = 3
a = –5, l = 21, Sn = 200, n = ?
n
Now, l = a + (n – 1)d = –5 + (n – 1)d Since, Sn = [2a + (n – 1)d]
2
 21 = –5 + (n – 1)d
 (n – 1)d = 21 + 5 = 26 ...(i) 5
 S5 = [2 × 3 + (5 – 1) × 3]
2
n
Also, Sn = [2a + (n – 1)d]
2 5 5
= (6 + 12) = × 18 = 45  Ans. (a)
2 2
n
 200 = [2 × (–5) + 26] [From (i)] 18. The number of two digit numbers which are
2
divisible by 3 is
 400 = n[–10 + 26] = n(16)
(a) 33 (b) 31
400
 n= = 25 (c) 30 (d) 29
16
Sol. Two digit number which are divisible by 3 is
 n = 25  Ans. (d) 12, 15, 18, 21, ... 99
P.Q. In an A.P., if a = 3 and S8 = 192, then d is Here, a = 12, d = 3, l = 99
(a) 8 (b) 7  l = an = a + (n – 1)d
(c) 6 (d) 4  12 + (n – 1) × 3 = 99
Sol. In an A.P. we have given,  (n – 1)3 = 99 – 12 = 87
a = 3, S8 = 192, d = ?
87
n  n–1= = 29
3
S8 = [2a + (n – 1)d]
2
 n = 29 + 1 = 30  Ans. (c)
8 19. The number of multiples of 4 that lie between
 192 = [2 × 3 + (8 – 1)d] 10 and 250 is
2
(a) 62 (b) 60
 192 = 4[6 + 7d]
(c) 59 (d) 55
192
 = 6 + 7d  48 = 6 + 7d Sol. Multiples of 4 lying between 10 and 250
4
12, 16, 20, 24, ..., 248 be,
 7d = 48 – 6 Here, a = 12, d = 16 – 12 = 4, l = 248
 7d = 42 Now, l = an = a + (n – 1)d
42  248 = 12 + (n – 1) × 4
i.e. d = =6  Ans. (c)
7  248 – 12 = 4(n – 1)
17. The sum of first five multiples of 3 is 236
(a) 45 (b) 55  = n – 1  n – 1 = 59
4
(c) 65 (d) 75  n = 59 + 1 = 60  Ans. (b)
234 Arun Deep's Understanding Math-10
20. The sum of first 10 even whole numbers is 22. The 5th term from the end of the G.P. 2, 6,
(a) 110 (b) 90 18, ..., 13122 is
(c) 55 (d) 45 (a) 162 (b) 486
Sol. Sum of first 10 even whole numbers (c) 54 (d) 1458
Even numbers are 0, 2, 4, 6, 8, 10, 12, 14, Sol. Given G.P. be 2, 6, 18, ..., 13122
16, 18
6
Here, a = 0, d = 2, n = 10 Here, a = 2, r = = 3, l = 13122
2
n
S10 = [2a + (n – 1)d] mn
2 1
 5th term from the end = l  
r
10
= [2 × 0 + (10 – 1) × 2] Here, l = an = arn – 1
2
= 5[0 + 18] = 90  Ans. (b) 13122 = 2(3)n – 1

1 1 13122
P.Q. The list of number ,  , 1, –3, ... is a 
2
= 3(n – 1)
9 3
1  6561 = 3(n – 1)
(a) G.P. with r = –3 (b) G.P. with r =   (3)8 = 3n – 1
3
(c) G.P. with r = 3 (d) not a G.P. On comparing, we get
Sol. The given list of numbers n–1=8n=9
1 1 mn
,  , 1, –3, ... 1
9 3 5th term from the end = l  
r
1 1 1 1 9
Here, a = ,r=  ÷ =  × = –3 9 5
9 3 9 3 1 1
= 13122  
It is a G.P. with r = –3  Ans. (a)  3
1 1 4
21. The 11th of the G.P. ,  , 2, –1, ... is 1 13122
8 4 = 13122 ×   = = 162
3 3 3 3 3
(a) 64 (b) –64
(c) 128 (d) –128 81)13122(162
1 1 81
Sol. Given, G.P. be, ,  , 2, –1, ... 502
8 4
486
1 1 1 1 8 162
Here a = , r =  ÷ =  × = –2
8 4 8 4 1 (a)
1 23. If k, 2(k + 1), 3(k + 1) are three consecutive
 a11 = arn – 1 = (–2)10 terms of a G.P., then the value of k is
8
(a) –1 (b) –4
1
= × (–1)10 × 210 = 1 × 210 – 3 = 27 (c) 1 (d) 4
23
Sol. k, 2(k + 1), 3(k + 1) are in G.P.
= 128  Ans. (c)
235 Arun Deep's Understanding Math-10
 [2(k + 1)]2 = k × 3(k + 1) 81  1 80
 4(k + 1)2 = 3k(k + 1) =
3 1
=
3 1
 4(k + 1) = 3k
(Dividing by k + 1 if k + 1  0) 80( 3  1)
=
 4k + 4 = 3k ( 3  1) ( 3  1)
 4k – 3k = –4  k = –4 (b) (Rationalising the denominator)
24. Which term of the G.P. 18, –12, 8, ... is
80( 3  1) 80( 3  1)
512 = =
? 3 1 2
729
(a) 12th (b) 11th = 40( 3 + 1)  Ans. (a)
(c) 10th (d) 9th 2
26. The sum of first 6 terms of the G.P. 1,  ,
Sol. Given G.P. be, 3
512 4
18, –12, 8, ... , ... is
729 9
512
Let nth term of given G.P. be .
729 133 133
(a)  (b)
12 2 243 243
Here, a = 18, d = =
18 3 793
(c) (d) none of these
 an = arn – 1 1215
n 1 n 1 Sol. Given G.P. be,
512 2 512 1   2 
 = 18    × =  2 4
729  3  729 18  3  1,  , , ...
3 9
n 1 8 n 1
256 2 2 2 2
 =     =  Here, a = 1, r = 
729  9  3   3   3  3
,n=6
On comparing, n – 1 = 8  n = 8 + 1 = 9
 It is 9th term.  Ans. (d)   2 6 
25. The sum of the first 8 terms of the series 1 1    
a(1  r n )   3  
1+ 3 + 3 + ... is S6 = = 2
1 r 1
(a) 40( 3 + 1) (b) 40( 3 – 1) 3

(c) 80( 3 + 1) (d) 80( 3 – 1)  64  3 729  64 3


= 1   = ×
Sol. Given G.P. be, 1 + 3 + 3 + ...  729  5 729 5

3 665 133
Here, a = 1, r = = 3,n=8 = =  Ans. (b)
1 243  5 243
27. If the sum of the G.P., 1, 4, 16, ... is 341,
a(r n  1) 1[( 3 ) 8  1] then the number of terms in the G.P. is
Now, Sn = =
r 1 3 1 (a) 10 (b) 8
236 Arun Deep's Understanding Math-10
(c) 6 (d) 5 1 3
Sol. Given G.P. be, 1, 4, 16, ... is 341 a7 = a + 6d = 0 + 6 × =
4 2
Let n be the term of G.P. be 341 i.e. Sn = 341,
14 13
4 (ii) 5, , , 4, ...
Here, a = 1, r = = 4, Sn = 341 3 3
1
14 14  15 1
a(r n  1) 1(4 n  1) 4n  1 Here, a = 5, d = –5= =
3 3 3
 341 = = =
r 1 4 1 3  Next three terms will be
 341 × 3 = 4n – 1  1023 = 4n – 1
1 11
 1023 + 1 = 4n  4n = 1024 a2 = 4 – =
3 3
 4n = 45
 n=5  Ans. (d) 11 1 10
a3 = – =
3 3 3
CHAPTER TEST
1. Write the first four terms of the A.P. when 10 1 9
a4 = – = =3
its first term is –5 and common difference 3 3 3
is –3.
11 10
Sol. First 4 term of A.P. whose first term (a) = –5 i.e. , ,3
3 3
and common difference (d) = –3
3. The nth term of an A.P. is 6n + 2. Find the
Thus first four terms of A.P. are a, a + d, common difference.
a + 2d, a + 3d
Sol. Tn of an A.P. = 6n + 2
i.e. –5, –8, –11, –14
T1 = 6 × 1 + 2 = 6 + 2 = 8
2. Verify that each of the following lists of
numbers is an A.P., and the write its next T2 = 6 × 2 + 2 = 12 + 2 = 14
three terms : T3 = 6 × 3 + 2 = 18 + 2 = 20
 d = 14 – 8 = 6
1 1 3 14 13
(i) 0, , , , ... (ii) 5, , , 4, ... 4. Show that the list of numbers 9, 12, 15, 18,
4 2 4 3 3
... form an A.P. Find its 16th term and the
1 1 3 nth.
Sol. (i) Given, A.P., 0, , , , ...
4 2 4 Sol. Given series be, 9, 12, 15, 18, ...
Here, a = 9, d = 12 – 9 = 3 or 15 – 12 = 3 or
1
Here, a = 0, d = 18 – 15 = 3
4
Yes, it forms an A.P.
5 3  T16 = a + (n – 1)d = 9 + (16 – 1) × 3
 Next three terms will be 1, ,
4 2
= 9 + 15 × 3 = 9 + 45 = 54
1 and Tn = a + (n – 1)d = 9 + (n – 1) × 3
a5 = a + 4d = 4 × =1
4 = 9 + 3n – 3 = 3n + 6
1 5 5. Find the 6th term from the end of the A.P.
a6 = a + 5d = 0 + 5 × = 17, 14, 11, ..., –40.
4 4
237 Arun Deep's Understanding Math-10
Sol. 6th term from the end of a17 = 2 × a8 + 5
Given A.P. be, 17, 14, 11, ... –40 a11 = 43,
Here, a = 17, d = –3, l = –40 Let a be the first term and d be the common
difference, then
l = a + (n – 1)d
a11 = a + (n – 1)d = a + (11 – 1)d
 –40 = 17 + (n – 1) (–3)
= a + 10d = 43 ...(i)
 –40 = 17 + (n – 1) (–3)
 –40 – 17 = (n – 1) (–3) Similarly,
a17 = 2 × a8 + 5
57
 =n–1  a + 16d = 2(a + 7d) + 5
3
 a + 16d = 2a + 14d + 5
 19 = n – 1
 –5 + 16d – 14d = 2a – a  a = 2d – 5
n = 19 + 1 = 20 ...(ii)
 6th term from the end From (i) and (ii) ; we have
= l – (n – 1)d 2d – 5 + 10d = 43  12d = 43 + 5 = 48
= –40 – (6 – 1) (–3)
48
= –40 + 15 = –25  d= =4
12
6. If the 8th term of an A.P. is 31 and the 15th But a + 10d = 43
term is 16 more than its 11th term, then find
the A.P.  a + 10 × 4 = 43  a + 40 = 43
 a = 43 – 40 = 3
Sol. In an A.P.
 a = 3, d = 4
a8 = 31, a15 = a11 + 16
Now, an = a + (n – 1)d
Let a be the first term an d be the common
difference, then = 3 + 4(n – 1) = 3 + 4n – 4
a8 = a + (n – 1)d = 31  a + 7d = 31 ...(i) = 4n – 1
Similarly, 8. The 19th term of an A.P. is equal to three
times its 6th term. If its 9th term is 19, find
a15 = a + 14d = a + 10d + 16 the A.P.
 14d – 10d = 16  4d = 16 Sol. In an A.P.
16 a19 = 3 × a6 and a9 = 19
 d= =4
4 Let a be the first term and d be the common
From (i) ; a + 7 × 4 = 31 difference, then

 a + 28 = 31  a = 31 – 28 = 3 a9 = a + (n – 1)d = a + (9 – 1)d = a + 8d
a + 8d = 19 ...(i)
 a = 3, d = 4
Similarly,
Now, A.P. will be 3, 7, 11, 15, ...
a19 = 3 × a6
7. The 17th term of an A.P. is 5 more than twice
its 8th term. If the 11th term of the A.P. is  a + 18d = 3(a + 5d)
43, then find the nth term.  a + 18d = 3a + 15d  3a – a = 18d – 15d
Sol. In an A.P.  2a = 3d ...(ii)
238 Arun Deep's Understanding Math-10

3 Here, a = 5, d = 2 – 5 = –3
a= d  an = a + (n – 1)d
2
From (i), we have  –55 = 5 + (n – 1) × (–3)

3 19 60
d + 8d = 19  d = 19  –55 – 5 = –3(n – 1)  =n–1
2 2 3
 n – 1 = 20  n = 20 + 1 = 21
19  2
 d= =2  –55 is the 21st term.
19
11. The 24th term of an A.P. is twice its 10th
3 3 term. Show that its 72nd term is four times
and a = d = ×2=3
2 2 its 15th term.
 a = 3, d = 2 and required A.P. is 3, 5, 7, 9, ... Sol. In an A.P.
9. If the 3rd and the 9th terms of an A.P. are 4 24th term = 2 × 10th term
and –8 respectively, then which term of this T.P. 72nd term = 4 × 15th term
A.P. is zero?
Let a be the first term and d be the common
Sol. In an A.P. difference, then
a3 = 4, a9 = –8, which term of A.P. will be 24th term = a + (24 – 1)d = a + 23d
zero
and 10th term = a + 9d
Let a be the first term and d be the common
difference, then  a + 23d = 2(a + 9d)
a3 = a + (n – 1)d = a + (3 – 1)d  a + 23d = 2a + 18d
 a + 2d = 4 ...(i)  2a – a = 23d – 18d
Similarly, a + 8d = –8  a = 5d ...(i)
Subtracting, we get and 72nd term = a + 71d = 5d + 71d = 76d
12 and 15th term = a + 14d = 5d + 14d = 19d
6d = –12  d = = –2 Hence 72nd term is 4 times the 15th term.
6
and a + 2d = 4  a + 2 × (–2) = 4 1
12. Which term of the list of numbers 20, 19 ,
 a–4=4a=4+4=8 4
Let nth term be zero, then 1 3
a + (n – 1)d = 0  8 + (n – 1) × (–2) = 0 18 , 17 , ... is the first negative term?
2 4
 –2n + 2 = –8  –2n = –8 – 2 = –10 Sol. Given A.P. be,
10 1 1 3
 n= =5 20, 19 , 18 , 17 , ...
2 4 2 4
 0 will be the fifth term of given A.P.
1 3
10. Which term of the list of numbers 5, 2, –1, Here, a = 20, d = 19 – 20 =
4 4
–4, ... is –55?
Let nth term be first negative term
Sol. Given, A.P. is 5, 2, –1, –4, ...
 an = a + (n – 1)d
Let nth term of given A.P. is –55.
239 Arun Deep's Understanding Math-10
Let nth term be first negative term, then 13. How many three digit numbers are divisible
by 9?
  3
an = 20 + (n – 1)   Sol. 3-digit numbers which are divisible by 9 are
 4 
108, 117, 126, 135, ..., 999
  3 Here, a = 108, d = 9 and l = 999
 an = 20 + (n – 1)  
 4   l = an = a + (n – 1)d
 999 = 108 + (n – 1)9
3 3
 an = 20 – n +  999 – 108 = 9(n – 1)
4 4
Now, an < 0 is the first negative term 891
 891 = 9(n – 1)  =n–1
3 3 83 3 9
 20 + – n<0 – n<0
4 4 4 4  n – 1 = 99  n = 99 + 1 = 100
83 3  There are 100 numbers or terms.
 < n  83 < 3n 14. The sum of three numbers in A.P. is –3 and
4 4
the product is 8. Find the numbers.
83
 < n  28 < n Sol. Sum of three numbers of an A.P. = –3
3
and their product = 8
 28th is the first negative term.
Let the numbers be
P.Q. If the pth term of an A.P. is q and the
qth term is p, show that its nth term is a – d, a, a + d, then
(p + q – n). a – d + a + a + d = –3
Sol. In an A.P.
3
pth term = q  3a = –3  a = = –1
3
qth term = p
Show that (p + q – n) is nth term and (a – d) a (a + d) = 8
Let a be the first term and d be the common a(a2 – d2) = 8  –1[(–1)2 – d2] = 8
difference
8
 pth term = a + (p – 1)d = q ...(i)  1 – d2 = = –8
and qth term = a + (q – 1)d = p ...(ii) 1
Subtracting, we get d2 = 1 + 8 = 9 = (+3)2
q – p = (p – 1 – q + 1)d = (p – q)d  d = +3
q p ( p  q ) If d = 3
d = p  q = ( p  q ) = –1 Hence the numbers are –1 – 3, –1, –1 + 3
From (i), a + (p – 1) × (–1) = q  –4, –1, 2
a–p+1=q and if d = –3, then
a=q+p–1 –1 – (–3), –1, –1 – 3
L.H.S.  –1 + 3, –1, –4  2, –1, –4
nth term 15. The angles of a quadrilateral are in A.P. If
= a + (n – 1)d = (p + q – 1) + (n – 1) (–1) the greatest angle is double of the smallest
= p + q – 1 – n + 1 = p + q – n = R.H.S. angle, find all the four angles.
240 Arun Deep's Understanding Math-10
Sol. Angles of a quadrilateral are in A.P.  It is not an A.P.
Greatest angle is double of the smallest Hence, an  n2 + n + 1
Let the greatest angle of quadrilateral is 16. Find the sum of first 20 terms of an A.P.
a + 3d ...(i) whose nth term is 15 – 4n.
and other angles are a + d, a – d, a – 3d Sol. Given Tn = 15 – 4n
where a – 3d is the smallest Giving some different values such as 1 to
 a + 3d = 2(a – 3d) 20, we get,
 a + 3d = 2a – 6d a1 = 15 – 4 × 1 = 15 – 4 = 11
 2a – a = 3d + 6d  a = 9d ...(ii) a2 = 15 – 4 × 2 = 15 – 8 = 7
But sum of angles of a quadrilateral = 360° a3 = 15 – 4 × 3 = 15 – 12 = 3
 a – 3d + a – d + a + d + a + 3d = 360° a4 = 15 – 4 × 4 = 15 – 16 = –1
and so on,
360
 4a = 360°  a = = 90° a20 = 15 – 4 × 20 = 15 – 80 = –65
4
Now, A.P. is 11, 7, 3, –1, ..., –65
90 Here, a = 11, d = –4 and n = 20
 9d = a = 90°  d = = 10° [From (ii)]
9
n
Greatest angle = a + 3d = 90° + 30° = 120°  S 20 = [2a + (n – 1)d]
2
Other angles are = a + d = 90° + 10° = 100°
a – d = 90° – 10° = 80° 20
= [2 × 11 + (20 – 1) (–4)]
and a – 3d = 90° – 30° = 60° 2
Hence required angles are 60°, 80°, 100°, = 10[22 – 76]
120° = 10(–54) = –540
P.Q. The nth term of an A.P. cannot be n2 + n + 1. 17. Find the sum :
Justify your answer.
Sol. We want to prove that nth term of an A.P. 1  1
18 + 15 + 13 + ... +   49  .
can't be n2 + n + 1. 2  2
Giving some different values to n such as Sol. Given series be,
1, 2, 3, 4, ...
then a1 = 12 + 1 + 1 = 1 + 1 + 1 = 3 1  1
18 + 15 + 13 + ... +   49 
a2 = 2 2 + 2 + 1 = 4 + 2 + 1 = 7 2  2

a3 = 32 + 3 + 1 = 9 + 3 + 1 = 13 1 1 5
a4 = 42 + 4 + 1 = 16 + 4 + 1 = 21 Here, a = 18, d = 15 – 18 = –2 =
2 2 2
We see that,
d = a2 – a1 = 7 – 3 = 4 1 99
l = –49 =
2 2
d = a3 – a2 = 13 – 7 = 6
d = a4 – a3 = 21 – 13 = 8 We know that
We see that d is not constant an = a + (n – 1)d
241 Arun Deep's Understanding Math-10

99  5 n
 = 18 + (n – 1)   Since, Sn =
2
[2a + (n – 1)d]
2  2 

99 18 5 n   1 
 – = (n – 1)  –25 = 2  (6)  (n  1) 
2 1 2 2   2 

99  36 5  1 1
 = (n – 1)  –25 × 2 = n  12  n  
2 2  2 2

135 5
 = (n – 1)   25 1 
2 2  –50 = n   n
 2 2 
135 2
 × =n–1 1 2 25
2 5  n – n + 50 = 0
2 2
 n – 1 = 27
 n2 – 25n + 100 = 0
 n = 27 + 1 = 28
 100  20  5
n  
Now, Sn = [2a + (n – 1)d]   25  20  5 
2
 n2 – 5n – 20n + 100 = 0
28    5   n(n – 5) – 20(n – 5) = 0
 S28 = 2  18  (28  1) 
2   2   (n – 5) (n – 20) = 0
Either n – 5 = 0, then n = 5
   5   135 
S28 = 14 36   27   = 14 36  or n – 20 = 0, then n = 20
  2   2 
 Required number of terms are 5 or 20
(ii) Solve the equation 2 + 5 + 8 + ... + x = 155
 72  135    63 
S28 = 14   = 14 ×   = –441 Here, a = 2, d = 5 – 2 = 3, l = x
 2   2 
and Sum = 155
11 l = a + (n – 1)d
18. (i) How many terms of the A.P. –6,  ,
2  x = 2 + (n – 1)3 = 2 + 3n – 3
–5, ... make the sum –25?  x = 3n – 1 ...(i)
(ii) Solve the equation 2 + 5 + 8 + ... + x = 155.
n
Sn = [2a + (n – 1)d]
11 2
Sol. (i) Given, A.P. be, –6,  , –5, ...
2
n
 155 = [2 × 2 + (n – 1) × 3]
11 1 2
Here, a = –6, d = +6=
2 2  155 × 2 = n[4 + 3n – 3]
and Sum Sn = –25  310 = n(3n + 1) = 3n2 + n
Let n terms of given A.P. gives sum –25  3n2 + n – 310 = 0
242 Arun Deep's Understanding Math-10
 3n2 – 30n + 31n – 310 = 0 20
 3n(n – 10) + 31(n – 10) = 0 = [2 × (–1) + (20 – 1) × 3]
2
 (n – 10) (3n + 31) = 0 = 10[–2 + 57] = 10 × 55 = 550
Either n – 10 = 0, then n = 10 P.Q. In an A.P., the first term is 2 and the last
31 term is 29. If the sum of the terms is 155,
or 3n + 31 = 0, then 3n = –31  n = then find the common difference of the A.P.
3
which is not possible being negative. Sol. In an A.P.
 n = 10 First term (a) = 2
Last term (l) = 29
Now, x = 3n – 1 = 3 × 10 – 1 = 30 – 1 = 29
Sn = Sum of terms = 155
[From (i)]
l = an = a + (n – 1)d
19. If the third term of an A.P. is 5 and the ratio
of its 6th term to the 10th term is 7 : 13,  29 = 2 + (n – 1)d  29 – 2 = d(n – 1)
then find the sum of first 20 terms of this  d(n – 1) = 27 ...(i)
A.P.
n
Sol. 3rd term of an A.P. = 5 i.e. a3 = 5 Sn = [2a + (n – 1)d]
2
and Ratio in 6th term and 10th term = 7 : 13
To find S20 : n n
 155 = [2 × 2 + 27] = [4 + 27]
2 2
Let a be the first term and d be the common
difference of given A.P. 31 155 2
 a3 = a + (n – 1)d  a + (3 – 1) d = 5  155 = nn= = 10
2 31
 a + 2d = 5 ...(i) Now, d(n – 1) = 27  d(10 – 1) = 27
Similarly, 27
a6 = a + 5d and a10 = a + 9d  d × 9 = 27  d = =3
9
a  5d 7 20. The sum of first 14 terms of an A.P. is 1505
 =  7a + 63d = 13a + 65d and its first term is 10. Find its 25th term.
a  9d 13
 13a + 65d – 7a – 63d = 0 Sol. Given, sum of first 14 terms = 1505
First term (a) = 10
 6a + 2d = 0  3a + d = 0
 d = –3a ...(ii) n
Now Sn = [2a + (n – 1)d]
From (i) and (ii), we have 2
 a + 2(–3a) = 5  a – 6a = 5 14
 S14 = 1505 = [2 × 10 + (14 – 1)d]
5 2
 –5a = 5  a = = –1
5 1505
 1505 = 7[20 + 13d]  20 + 13d =
and d = –3a = –3 × (–1) = 3 7
Now sum of first 20 terms  13d = –20 + 215 = 195
n 195
=
2
[2a + (n – 1)d]  d= = 15
13
243 Arun Deep's Understanding Math-10
Now, an = a + (n – 1)d
400
 a25 = 10 + (25 – 1) (15) = 10 + 24(15)  d= = –4
100
= 10 + 360 = 370
From (i), we have
P.Q. The sum of first n term of an A.P. is 10a + 45 × (–4) = –150
3n2 + 4n. Find the 25th term of this A.P.
 10a – 180 = –150
Sol. Given, Sn = 3n2 + 4n
 10a = –150 + 180 = 30
 Sn – 1 = 3(n – 1)2 + 4(n – 1)
= 3(n2 – 2n + 1) + 4(n – 1) 30
a= =3
10
= 3n2 – 6n + 3 + 4n – 4
= 3n2 – 2n – 1 Required A.P. be 3, –1, –5, –9, ...

Now, an = Sn – Sn – 1 P.Q. The sum of first m terms of an A.P. is


4m2 – m. If its nth term is 107, find the
= (3n2 + 4n) – (3n2 – 2n – 1) value of n. Also find the 21st term of this
= 3n2 + 4n – 3n2 + 2n + 1 A.P.
= 6n + 1 Sol. Given, Sm = 4m2 – m
 a25 = 6(25) + 1 = 150 + 1 = 151  Sn = 4n2 – n
P.Q. In an A.P., the sum of first 10 terms is –150 and Sn – 1 = 4(n – 1)2 – (n – 1)
and the sum of next 10 terms is –550. Find = 4[n2 – 2n + 1] – n + 1
the A.P.
= 4n2 – 8n + 4 – n + 1 = 4n2 – 9n + 5
Sol. In an given A.P.
Now, an = Sn – Sn – 1
Sum of first 10 terms = –150
= 4n2 – n – 4n2 + 9n – 5
Sumof next 10 terms = –550,
= 8n – 5
Since, sum of first 10 terms = –150
Now, an = 107
n  8n – 5 = 107  8n = 107 + 5 = 112
 S10 = [2a + (n – 1)d]
2
112
n= = 14
10 8
–150 = [2a + 9d] = 5(2a + 9d)
2 and an = 8n – 5
 10a + 45d = –150 ...(i)  a21 = 8 × 21 – 5 = 168 – 5 = 163
S20 – S10 = –550 21. Find the geometric progression whose 4th
 S20 = –150 – 550 = –700 term is 54 and 7th term is 1458.
Sol. In given G.P.
20
Now S20 = [2a + 19d] = 10(2a + 19d) a4 = 54
2
a7 = 1458
 20a + 190d = –700 ...(ii) Let a be the first term and r be the common
20a + 90d = –300 [Multiplying (i) by 2] difference of given G.P.
– – +  a4 = arn – 1 = ar3 = 54
On subtracting, 100d = –400 Similarly, ar6 = 1458
244 Arun Deep's Understanding Math-10
On dividing, we get a16 = ar15 = z
ar 6 1458 x, y, z are in G.P.
3 =
ar 54 if y2 = xz
 r3 = 27 = (3)3 y2 = (ar9)2 = a2r18
 r=3 xz = ar3 × ar15 = a2 r3 + 15 = a2 r18
and ar3 = 54  a × 27 = 54  y2 = xz
54  x, y, z are in G.P.
 a= =2
27
3 3
 a = 2, r = 3 24. How many terms of the G.P. 3, , , ...
2 4
and required G.P. is 2, 6, 18, 54, ...
3069
22. The fourth term of a G.P. is the square of its are needed to give the sum ?
second term and the first term is –3. Find 512
its 7th term.
3 3
Sol. In G.P. Sol. Given, G.P. is 3, , , ...
2 4
a4 = (a2)2, a1 = –3
Let a be the first term and r be the common 3069
Sn =
ratio of given G.P. 512
an = arn – 1

 a4 = ar3 Here, a = 3, r =
2
and a2 = ar
 ar3 = (ar)2  ar3 = a2r2 Let n be the required number of terms such
 r=a a(1  r n )
that Sn =
 a1 = –3 1 r
 a = –3
 a7 = ar7 – 1 = ar6 = –3 × (–3)6   1 n    1 n 
31     31    
= –3 × 729 = –2187   2     2  
3069
23. If the 4th, 10th and 16th terms of a G.P. are = 1 = 1
512 1
x, y and z respectively, prove that x, y and z
2 2
are in G.P.
Sol. In a G.P.
3069   1 n 
a4 = x, a10 = y, a16 = z  = 2 × 3 1   2  
Show that x, y, z are in G.P.
512    
Let a be the first term and r be the common n
ratio, then 1 3069 1023
 1–   = =
an = arn – 1  
2 512  6 1024

a4 = ar4 – 1 = ar3 = x n
1 1024  1023 1
Similarly,    = =
2 1024 1024
a10 = ar9 = y
245 Arun Deep's Understanding Math-10

3
2 1024 = [10 × 10n – 10 – 9n]
81
2 512
2 256 1
2 128 = [10n + 1 – 9n – 10]
27
2 64
2 32 26. Find the sum of the series 7 + 7.7 + 7.77 +
2 16 7.777 + ... to 50 terms.
2 8 Sol. The given sequence is 7, 7.7, 7.77, 7.777,
2 4 ...
2 2
1 Required sum = S50
= 7 + 7.7 + 7.77 + ... 50 terms = 7(1 + 1.1
+ 1. 11 + ... 50 terms)
n 10
1 1
   =   7
2 2 = [9 + 9.9 + 9.99 + 50 terms]
9
On comparing, we get
7
 n = 10 = [(10 – 1) + 1(10 – 0.1) + (10 – 0.01) +
9
25. Find the sum of first n terms of the series : 50 terms)]
3 + 33 + 333 + ...
7
Sol. Series is = [(10 + 10 + 10 + ... 50 terms) – (1 +
9
3 + 33 + 333 + ... n terms
0.1 + 0.01 + ... 50 terms)]
= 3[1 + 11 + 111 + ... n terms]
7
3 = [500 – Sum of G.P. of 50 terms with
= [9 + 99+ 999 + ... n terms] 9
9 a = 1, r = 0.1]
3  1[1  (0.1) 50 ] 
= [(10 – 1) + (100 – 1) + (1000 – 1) + ... 7 
9 = 500 
9  1  0.1 
n terms]

3 7  10  1 
= [10 + 100 + 1000 + ... n terms – n × 1] = 500  1  50 
9 9  9  10 

3  a ( r n  1   7
   n = (4500 – 10 + 10–49)
=  81
9  r  1  
7
= [4490 + 10–49]
3 10(10  1)  n 
n
81
=  
9  10  1 

3 10 n 
=  (10  1)  n 
9  9 
10. REFLECTION
POINTS TO REMEMBER
1. Reflection : The reflection or image of a point P in a line AB is a point P such that AB is the
perpendicular bisector of the line segments PP. The line AB is called axis of reflection.
2. Invariant Point : A point is called an invariant point with respect to a given line if and only if it
lies on the line.
3. Reflection of a point in the x-axis :
(i) Retain the abscissa i.e. x-co-ordinate.
(ii) Change the sign of ordinate i.e. y-co ordinate.
4. Reflection of a-point in y-axis :
(i) Change the sign of abscissa i.e. x-cordinate.
(ii) Retain the ordinate i.e. y-coordinate.
5. Reflection of a point in a point : The reflection or image of a point P in a given point M is a
point P such that M is the mid-point of the line segment PP.
6. Reflection of a point in the origin :
Change the sign of abscissa as well as ordinate i.e. x-coordinate and also y-coordinate.

EXERCISE 10
1. Find the co-ordinates of the images of the (i) (2, – 5)
FG
3 1
(ii)  2 , 2
IJ
(iii) (0, – 7)
following points under reflections in the x-
H K
axis: Sol. We know that
My (x, y) = (–x, y)
(i) (2, – 5)
FG3 1
(ii)  2 ,  2
IJ Co-ordinates of the image of the points
H K under reflection in the y-axis
(iii) (– 7, 0) (i) Image of (2, – 5) will be (– 2, – 5)
Sol. We know that Mx (x, y) = (x, –y) 3 1
(ii) Image of  2 , 2
FG IJ will be FG 3 , 1 IJ
Thus, co-ordinates of the images of the H K H 2 2K
points under reflection in the x-axis will be (iii) Image of (0, – 7) will be (0, – 7) Ans.
(i) Image of (2, – 5) will be (2, 5) 3. Find the co-ordinates of the images of the
following points under reflection in the
(ii) Image of
FG  3 ,  1 IJ will be origin:
H 2 2K
FG  3 , 1 IJ (i) (2, – 5) (ii)
FG 3 , 1 IJ (iii) (0, 0)
H 2 2K H2 2K
Sol. We know that
(iii) Image of (–7, 0) will be (– 7, 0) Ans.
M0 (x, y) = (–x, –y)
2. Find the co-ordinates of the images of the
Co-ordinates of the images under reflection
following points under reflection in the in the origin
y-axis :
(i) Image of (2, – 5) will be (– 2, 5)

246 Arun Deep's Understanding Math-10


247 Arun Deep's Understanding Math-10

FG 3 , 1 IJ will be FG 3 , 1 IJ x = 4 to the point P ' . Find the co-ordinates


(ii) Image of H2 2K H 2 2K of the point P ' .
(ii) Find the image of the point P (1, –2) in the
(iii) Image of (0, 0) will be (0, 0) Ans.
line x = –1.
4. The image of a point P under reflection in
Sol.
the x-axis is (5, – 2). Write down the co-
ordinates of P. (i) (a) Draw axis XOX ' and YOY ' and take 1
Sol. As the image of a point (5, – 2) under x- cm = 1 unit.
axis is P. (b) Plot point P (2, 3) on it.
i.e. Mx (x, y) = (5, –2) (c) Draw a line x = 4 which is parallel to y-axis.
i.e. (x, –y) = (5, –2) (d) From P1 draw a perpendicular on x = 4,
i.e. x = 5 and –y = –2  y = 2 which interesects x = 4 at Q.
 Co-ordinates of P will be (5, 2) Ans. (e) Produce PQ to P ' , such that QP ' = QP..
5. A point P is reflected in the x-axis. Co-  P ' is the reflection of P in the line x = 4
ordinates of its image are (8, – 6). Co-ordinates of P ' are (6, 3)
(i) Find the co-ordinates of P.
Y
(ii) Find the co-ordinates of the image of P
under reflection in the y-axis. 4

Sol. The co-ordinates of image of P which is P (2, 3)


3 P’
reflected in x-axis are (8, –6), then Q
(i) Mx (x, y) = (8, –6)
2
 (x, –y) = (8, –6)
 x = 8, y = 6 1 x= 4
Co-ordinates of P will be (8, 6)
(ii) Since, My (x, y) = (–x, y) X` X
O 1 2 3 4 5 6
 My (8, 6) = (–8, 6)
 Co-ordinates of image of P under reflection 1
in the y-axis will be (– 8, 6) Ans.
6. A point P is reflected in the origin. 2

Co-ordinates of its image are (2, – 5). Find


3
(i) the co-ordinates of P.
(ii) the co-ordinates of the image of P in the
4
x-axis.
Y`
Sol. Since, M0 (x, y) = (2, –5)
 (–x, –y) = (2, –5) (ii) (a) Draw axis XOX and YOY and take 1
 x = –2 and y = 5 cm = 1 unit.
The co-ordinates of image of a point P which (b) Plot the point P (1, –2) on it.
is reflected in origin are (2, – 5), then
(c) Draw a line x = –1 which is parallel to y-axis.
(i) Co-odinates of P will be (– 2, 5)
(d) From P, draw a perpendicular on the line x = –1,
(ii) Co-ordinates of the image of P in the x- which meets it at Q.
axis will be (– 2, – 5) Ans.
(e) Produce PQ to P such that PQ = QP
[ Mx (x, y) = (x, –y)]
P ' is the image or reflection of P in the line x = –1
7. (i) The point P (2, 3) is reflected in the line
Co-ordinates of P ' are (–3, –2)
248 Arun Deep's Understanding Math-10

Y
Y
P (2, 4)
4
4

3
3

2
2
Q
1
1 y=1

X` X
X` X 3 2 1 O 1 2 3
3 2 1 O 1 2 3
1
1
x = 1
2
2 P` (2, 2)
P` (3, 2) Q P (1, 2)
3
3

4
4
Y`
Y`

(i)
(ii)

8. (i) The point P (2, 4) on reflection in the (ii) (a) Draw axis XOX` and YOY` and take
line y = 1 is mapped onto P`. Find the co- 1 cm = 1 unit.
ordinates of P`.
(ii) Find the image of the point P (–3, –5) in the (b) Plot point P (–3, –5) on it.
line y = –2.
Sol. (c) Draw a line y = –2 which is parallel to x-axis.
(i) (a) Draw axis XOX` and YOY` and take
1 cm = 1 unit. (d) From P, draw a perpendicular on y = –2
(b) Plot point P (2, 4) on it.
which meets it at Q.
(c) Draw a line y = 1, which is parallel to x-axis.
(d) From P, draw a perpendicular on y = 1
(e) Produce PQ to P` such that QP` = PQ.
meeting it at Q.
Then P` is the image of P, whose co-ordinates
(e) Produce PQ to P` such that QP` = PQ.
are (–3, 1).
P` is the reflection of P whose co-ordinates
are (2, –2)
249 Arun Deep's Understanding Math-10
Y (ii) Co-ordinates of the image reflected in y-
3 axis will be (– 3, – 2)
(iii) Co-ordinates of the point reflected in x-
2 axis followed by reflection in the y-axis will
P` (3, 1) be (– 3, 2)
1
(iv) Co-ordinates of the point reflected in the
X
origin will be (– 3, 2) Ans.
X`
O
4 3 2 1 1 2 3 11. Find the co-ordinates of the image of (3, 1)
1 under reflection in x-axis followed by
reflection in the line x = 1.
y = 2
2 Sol. (i) Draw axis XOX` and YOY` taking 1 cm
Q
= 1 unit.
3 (ii) Plot a point P (3, 1).
(iii) Draw a line x = 1, which is parallel to y-axis.
F
4 (iv) From P, draw a perpendicular on x-axis
meeting it at Q.
P ( 3,  5)
5 (v) Produce PQ to P` such that QP` = PQ, then
P` is the image of P is x-axis. Then co-
Y`
(ii) ordinates of P` will be (3, –1)
(vi) From P`, draw a perpendicular on x = 1
9. The point P (– 4, – 5) on reflection in meeting it at R.
y-axis is mapped on P. The point P on reflection (vii) Produce P`R to P`` such that RP`` = P`R
in the origin is mapped on P. Find the co-ordinates  P`` is the image of P` in the line x = 1
of P and P. Write down a single transformation Co-ordinates of P`` are (–1, –1)
that maps P onto P.
Y
Sol. P is the image of point P (– 4, – 5) in y-axis
4
[ My (x, y) = (–x, y)] x=1

 Co-ordinates of P will be (4, – 5) 3

Again P is the image of P under reflection


2
in origin will be (– 4, 5).
P (3, 1)
 The single transformation that maps P onto 1
P is x-axis
Q
10. Write down the co-ordinates of the image X` X
3 2 1 O 1 2 3 4
of the point (3, – 2) when :
1
(i) reflected in the x-axis (ii) reflected in the P`` (1, 1) R P` (3, 1)
y-axis (iii) reflected in the x-axis followed
2
by reflection in the y-axis (iv) reflected in
the origin. (2000)
3
Sol. Co-ordinates of the given points are
(3, –2). 4
(i) Co-ordinates of the image reflected in x- Y`
axis will be (3, 2)
250 Arun Deep's Understanding Math-10
12. If P` (–4, –3) is the image of a point P under reflection in the origin, find
(i) the co-ordinates of P.
(ii) the co-ordinates of the image of P under reflection in the line y = –2.
Sol.
(i) Reflection of P is P` (–4, –3) in the origin
 Co-ordinates of P will be (4, 3)
Draw a line y = –2, which is parallel to x-axis
(ii) From P, draw a perpendicular on y = –2 meeting it at Q
Produce PQ to P`` such that QP`` = PQ
 P`` will the image of P in the line y = –2
 Co-ordinates of P`` will be (4, –7)

P (4, 3)
3

X` X
5 4 3 2 1 O 1 2 3 4 5

1

Q
2
y = 2
P` (4, 3)
3

4

5

6

7
P`` (4, 7)
Y`

13. A Point P (a, b) is reflected in the x-axis to P` (2, –3), write down the values of a and b. P`` is the
image of P, when reflected in the y-axis. Write down the co-ordinates of P``. Find the co-ordinates
of P```, when P is reflected in the line parallel to y-axis such that x = 4. (1998)
251 Arun Deep's Understanding Math-10
Sol.
P` (2, –3) is the reflection of P (a, b) in the x-axis
 Co-ordinates of P` will be P` (a, –b) but P` is (2, –3)
Comparing a = 2, b = 3

P`` (a, b) P (a, b) P``` (6, 3)


3

X` X
2 1 O 1 2 3 4 5 6

1
P`` (1, 1)
x= 4
2

3
P` (2, 3)

4
Y`

 Co-ordiantes of P will be (2, 3)


P`` is the image of P when reflected in y-axis [ My (x, y) = (–x, y)]
 Co-ordinate of P`` will be (–2, 3)
Draw a line x = 4, which is parallel to y-axis and P``` is the image of P when it is reflected in the line
x = 4, then P``` is its reflection
 Co-ordinates of P``` will be (6, 3).
14. (i) Point P (a, b) is reflected in the x-axis to P (5, – 2). Write down the values of a and b.
(ii) P is the image of P when reflected in the y-axis. Write down the co-ordinates of P.
(iii) Name a single transformation that maps P to P. (1997)
Sol. (i) Image of P (a, b) reflected in the x-axis to P (5, – 2)
 a = 5 and b = 2
(ii) P is the image of P when reflected in the y-axis
 its co-ordinates will be (– 5, – 2).
(iii) The single transformation that maps P to P is origin. Ans.
252 Arun Deep's Understanding Math-10
15. Points A and B have co-ordinates (2, 5) and (0, 3). Find
(i) the image A of A under reflection in the x-axis.
(ii) the image B of B under reflection in the line AA.
Sol. Co-ordinates of A are (2, 5) and of B are (0, 3)

(i) Co-ordinates of A, the image of A reflected in the x-axis will be (2, – 5)
(ii) Co-ordinates of B, the image of B under reflection in the line AA will be (4, 3).
16. Plot the points A (2, – 3), B (– 1, 2) and C (0, – 2) on the graph paper. Draw the triangle formed
by reflecting these points in the x-axis. Are the two triangles congruent ?
253 Arun Deep's Understanding Math-10
Sol. The points A (2, – 3), B (– 1, 2) and C(0, – C. Write down a single transformation
2) has been plotted on the graph paper as that maps ABC onto A B C.
shown and are joined to form a triangle ABC.
Sol. The co-ordinates of  ABC are A (1, 2) B
The co-ordinates of the images of A, B and
(4, 8), C (6, 8) which are reflected in x-
C reflected in x-axis will be A (2, 3), B (–
axis as A, B and C.
1, – 2), C (0, 2) respectively and are joined
to from another  ABC
 The co-ordinates of A (1, – 2), B (4, – 8)
and C (6, – 8).
Yes, these two triangle are congruent.
A, B and C are again reflected in origins
17. The points (6, 2), (3, – 1) and (– 2, 4) are
to form a  A B C.
the vertices of a right angled triangle. Check
whether it remains a right angled triangle  The co-ordinates of A will be (– 1, 2), B
after reflection in the y-axis. (– 4, 8) and C (– 6, 8)
Sol. Let A (6, 2), B (3, – 1) and C (– 2, 4) be The single transformaton that maps ABC
the points of a right angled triangle then onto A B C is y-axis. Ans.
the co-ordinates of the images of A, B, C 19. The image of a point P on reflection in a
reflected in y-axis be A (– 6, 2), B (– 3, – line l is point P. Describe the location of
1) and C (2, 4). the line l.
Sol. The line will be the right bisector of the
line segment joining P and P.
20. Given two points P and Q, and that (1) the
image of P on reflection in y-axis is the
point Q and (2) the mid point of PQ is
invariant on reflection in x-axis. Locate
(i) the x-axis (ii) the y-axis and
(iii) the origin.
Sol. Q is the image of P on reflection in y-axis
and mid point of PQ is invariant on
reflection in x-axis

By joining these points, we find that 


ABC is also a right angled triangle.
18. The triangle ABC where A (1, 2), B (4, 8),
C (6, 8) is reflected in the x-axis to triangle
A B C. The triangle A B C is then
reflected in the origin to triangle ABC. (i)  x-axis will be the line joining the points
Write down the co-ordinates of A, B, P and Q.
254 Arun Deep's Understanding Math-10
(ii) The line perpendicular bisector of line segment PQ is the y-axis.
(iii) The origin will be the mid point of line segment PQ. Ans.
21. The point (– 3, 0) on reflection in a line is mapped as (3, 0) and the point (2, – 3) on reflection
in the same line is mapped as (– 2, – 3).
(i) Name the mirror line.
(ii) Write the co-ordinates of the image of (– 3, – 4) in the mirror line.
Sol. The point (– 3, 0) is the image of point (3, 0) and point (2, – 3) is image of point (– 2, – 3)
reflected on the same line.
(i) It is clear that the mirror line will be y-axis.
(ii) The co-ordinates of image of the point (– 3, – 4) reflected in the same line i.e.
y-axis will be (3, – 4). Ans.
22. Use graph paper for this question (take 2 cm = 1 unit along both x and y axis).
ABCD is a quadrilateral whose vertices are A(2, 2), B(2, –2), C(0, –1) and D(0, 1).
(i) Reflect quadrilateral ABCD on the y-axis
and name it as A B C D .
(ii) Write down the coordinates of A and B.
(iii) Name two points which are invariant
under the above reflection.
(iv) Name the polygon ABCD.
Sol. (i)

(ii) A = (–2, 2), B = (–2, –2)


(iii) C and D
(iv) Isosceles trapezium Ans.

23. Use a graph sheet for this question.


Take 1 cm = 1 unit along both x and y-axis.
(i) Plot the points:
A(0, 5), B(3, 0), C(l, 0) and D(l, –5).
(ii) Reflect the points B, C and D on the y-axis and name them as B, C and D respectively.
(iii) Write down the coordinates of B, C and D.
(iv) Join the points A, B, C, D, D, C, B, A in order and give a name to the closed figure
ABCDDCB.
255 Arun Deep's Understanding Math-10
y -axis the line y = 0, the co-ordinates of R are
Sol. (iii) B (3, 0)  B (– 3, 0) (  2, 4).
y -axis (iii) Figure PQR is the right angled triangle
C (1, 0)  C (– 1, 0)
PQR.
y -axis
D (1, – 5)  D (– 1, – 5)
(iv) The closed figure ABCDDCBA is like
an arrow head Y

6
Plot the points on graph paper.
5

Y R (-2, 4) 4 Q (2, 4)

6 3

5 2
A (0, 5)
4 1

3 X` X
7 6 5 4 3 2 1 O 1 2 3 4 5 6 7
2 1

1 2
B(–3, 0) C B (3, 0)
X` X 3
7 6 5 4 3 2 1 O 1 2 3 4 5 6 7
1 C (1, 0) 4 P (2, -4)

2 5

3 6

4 7
Y`
D (–1, 5) 5 D (1, –5)
6

7
Y`
1
24. Use graph paper for this question. (iv) Area of PQR = × QR × PQ
2
(i) The point P (2,  4) is reflected about
the line x = 0 to get the image Q. Find the = × 4 × 8 = 16 sq. units.
co-ordinates of Q.
25. Using a graph paper, plot the points
(ii) Point Q is reflected about the line y = 0 to A(6, 4) and B(0, 4).
get the image R. Find the co-ordinates of
(i) Reflect A and B in the origin to get the
R.
images A and B.
(iii) Name the figure PQR.
(ii) Write the co-ordinates of A and B.
(iv) Find the area of figure PQR. (2007)
(iii) State the geometrical name for the figure
Sol. (i) Since the point Q is the reflection of ABAB.
the point P (2,  4) in the line x = 0, the
(iv) Find its perimeter.
co-ordinates of Q are (2, 4).
Sol. (i) A(6, 4), B(0, 4)
(ii) Since R is the reflection of Q (2, 4) about
256 Arun Deep's Understanding Math-10
Y
(ii) A (6, 4) origin
 A (–6, –4) 5
B (0, 4) A (6, 4)
4
B (0, 4) origin
 B (0, –4)
3
(iii) ABAB is a parallelogram
2

(iv) AB = AB2  BB2 = 6 2  82 1

X` X
6 5 4 3 2 1 O 1 2 3 4 5 6
= 36  64 = 100 = 10 units 1

Perimeter = Sum of all sides 2

= 6 + 10 + 6 + 10 = 32 units 3

4 B (0, 4)
A ( 6,  4)
26. Use graph paper to answer this question : 5
Y`
(i) Plot the points A (4, 6) and B (1, 2). Y

(ii) If A is the image of A when 7


reflected in x-axis, write the A (4, 6)
6
co-ordinates of A. 5
(iii) If B is the image of B when B is 4
reflected in the line AA,write the 3
co-ordinates of B. 2 B (1, 2) B(7, 2)
(iv) Give the geometrical name for 1
the figure AB AB. (2009) X` X
7 6 5 4 3 2 1 O 1 2 3 4 5 6 7
Sol. (i) Plotting the points A (4, 6) 1
and B (1, 2) on the given graph. 2
(ii) A = (4, –6) 3
(iii) B = (7, 2) 4
(iv) In the quadrilateral ABAB, we have 5
AB = AB and AB = AB 6 A (4, 6)
Hence, ABAB is a kite. Ans. Y`
27. The points A (2, 3), B (4, 5) and C (7, 2) are the Sol. Points A (2, 3), B (4, 5) and C (7, 2) are the
vertices of ABC. (2006) vertices of  ABC.
(i) Write down the co-ordinates of A1, B1, C1 A1, B1 and C1 are the images of A, B and C
if  A1B1C1 is the image of  ABC when reflected in the origin.
reflected in the origin.
(i)  Co-ordinates of A1 = (– 2, – 3) of B1
(ii) Write down the co-ordinates of A2, B2, C2 (– 4, – 5) and of C1 (– 7, – 2).
if  A2B2C2 is the image of  ABC when
(ii) Co-ordinates of A2, B2 and C2 the images of
reflected in the x-axis. A, B and C when reflected in x-axis are
(iii) Assign the special name to the A2 (2, – 3), B2 (4, – 5), C2 (7, – 2)
quadrilateral BCC2B2 and find its area.
257 Arun Deep's Understanding Math-10
Y (v) The line AA is the line of symmetry ABAB.
7 Y
6 5
5 B (4, 5) 4  (2, 4)
4
3 A (2, 3) C (7, 2) A ( 4, 2)
3
2 A  (4, 2)
2
1
X` X 1
-8 -7 6 5 4 3 2 1O 1 2 3 4 5 6 7 8
C1 (–7, –2) 1
A1 (–2, –3) 2 A2 (2, –3) X` X
4 3 2 1 O 1 2 3 4 5
3 C2 (7, –2)  (2, 0)
1
4
5 Y`
B1 (–4, –5) B2 (4, –5) 28. The point P (3, 4) is reflected to P in the x-

 axis and O is the image of O (origin) in the
Y` line PP. Find : (i) the co-ordinates of P and
O. (ii) the length of segments PP and OO.
(iii) The quadrilateral formed by joining the points,
BCC2B2 is an isosceles trapezium and its area (iii) the perimeter of the quadrilateral POPO.

1 1 Sol. P is the image of P (3, 4) reflected in x-axis


 ( BB2  CC2 )× 3  (10 + 4) × 3 and O is the image of O the origin in the line
2 2
PP.
1
 × 14 × 3  21 sq. units Ans. (i)  Co-ordinates of P  are (3, – 4) and
2
co-ordinates of O reflected in PP are (6, 0)
P.Q. Use graph paper to answer the following
questions. (Take 2 cm = 1 unit on both axes) (ii) Length of PP = 8 units and OO = 6 units
(i) Plot the points A (–4, 2) and B (2, 4) (iii) Perimeter of POPO is
(ii) A is the image of A when reflected in the y-
4 × OP  4 × ( OQ ) 2  ( PQ ) 2  4 32  4 2
axis. Plot it on the graph paper and write the
coordinates of A.
(iii) B is the image of B when reflected in the Y
line AA. Write the coordinates of B. 5
(iv) Write the geometric name of the figure 4 P (3, 4)
ABAB.
3
(v) Name a line of symmetry of the figure formed.
2
Sol.
1
(i)The points A(–4, 2) and B(2, 4) have been O’ (6, 0)
X` X
plotted on the graph. 3 2 1 O 1 2 3 4 5 6
(ii) The coordinates of A are (4, 2). 1

(iii) The coordinates of B are (2, 0) 2

(iv) From the graph, AB = AB and AB = AB. 3


So ABAB is a kite. 4 P’ (3, –4)
5
Y`
258 Arun Deep's Understanding Math-10
 4 ( 9  16)  4 × 25 = 4 × 5 = 20 units Ans. MULTIPLE CHOICE QUESTIONS
29. Use a graph paper for this question. (Take 10 Choose the correct answer from the given four
small divisions = 1 unit on both axes). P and Q have oprtions (1 to 7) :
co-ordinates (0, 5) and (  2, 4). 1. The reflection of the point P (–2, 3) in the
(i) P is invariant when reflected in an axis. x-axis is
Name the axis. (a) (2, 3) (b) (2, –3)
(ii) Find the image of Q on reflection in the (c) (–2, –3) (d) (–2, 3)
axis found in (i). Sol. Reflection of the point P (–2, 3) in x-axis is
(iii) (0, k) on reflection in the origin is (–2, –3) [ Mx (x, y) = (x, –y)] (c)
invariant. Write the value of k. 2. The reflection of the point P (–2, 3) in the y-
(iv) Write the co-ordinates of the image of Q, axis is
obtained by reflecting it in the origin (a) (2, 3) (b) (2, –3)
followed by reflection in x-axis. (2005) (c) (–2, –3) (d) (0, 3)
Sol. Sol. The reflection of the point P (–2, 3) under
Y
reflection in y-axis (2, 3)
5 P (0, 5)
[ My (x, y) = (–x, y)] (a)
Q (–2, 4) 4 Q’ (2, 4)
3. If the image of the point P under reflection in
3 the x-axis is (–3, 2), then the coordiantes of
2 the point P are
1 (a) (3, 2) (b) (–3, –2)
(c) (3, –2) (d) (–3, 0)
X` X
6 5 4 3 2 1 O 1 2 3 4 5 6 Sol. The image of the point P under reflection in
1
the x-axis is (–3, 2),
2  Mx (x, y) = (–3, 2)
3  (x, –y) = (–3, 2)
4 Q  (–2,  4) i.e. x = –3 and y = –2
then the co-ordinates of the point P will be
5
Y`
(–3, –2) (b)
(i) Two points P (0, 5) and Q (–2, 4) are given 4. The reflection of the point P (1, –2) in the
As the abscissa of P is 0. line y = –1 is
  It is invarient when is reflected in y-axis. (a) (–3, –2) (b) (1, –4)
(ii) Let Q be the image of Q on reflection (c) (1, 4) (d) (1, 0)
in y-axis. Sol. We know that reflection of point (x, y) in the
 Co-ordinate of Q will be (2, 4) line y = a is P (x, –y + 2a)
(iii) (0, k) on reflection in the origin is The reflection of the point P (1, –2) in the
invarient. line y = –1 is (1, 0) (d)
 co-ordinates of image will be (0, 0). 5. The reflection of the point A (4, –1) in the
 k = 0 line x = 2 is
(iv) The reflection of Q in the origin is (a) (0, –1) (b) (8, –1)
the point Q and its co-ordinates will (c) (0, 1) (d) none of these
be (2, –4) and reflection of Q(2, –4) Sol. The reflection of A (4, –1) in the line x = 2
in x-axis is (2, 4) which is the point Q will be A (0, –1) (a)
259 Arun Deep's Understanding Math-10
6. The reflection of the point (–3, 0) in the origin (d, 5) after reflection in the origin. Find the
is the point values of a, b, c and d.
(a) (0, –3) (b) (0, 3) Sol. If the image of P (a, b) after reflected in the
x-axis be (a, – b) but it is given (– 2, c).
(c) (3, 0) (d) none of these
Sol. Reflection of the point (–3, 0) in origin will  a = – 2, c = – b
be (3, 0) [ M0 (x, y) = (–x, –y)] (c) If P is reflected in the origin, then its co-ordinates
will be (– a, – b), but it is given (d, 5)
7. Which of the following points is invariant
with respect to the line y = –2 ? –b=5b=–5
(a) (3, 2) (b) (3, –2) d = – a = – (– 2) = 2, c = – b = – (– 5) = 5
(c) (2, 3) (d) (–2, 3) Hence a = – 2, b = – 5, c = 5, d = 2 Ans.
Sol. The variant points are (3, –2) (b) 4. A (4, – 1), B (0, 7) and C (– 2, 5) are the
vertices of a triangle.  ABC is reflected in
CHAPTER TEST the y-axis and then reflected in the origin.
Find the co-ordinates of the final images of
1. The point P (4, – 7) on reflection in x- the vertices.
axis is map ped o nto P  . Then P  on Sol. A (4, – 1), B (0, 7) and C (– 2, 5) are the
reflection in the y-axis is mapped onto vertices of  ABC.
P  . Find the co-ordinates of P  and P  . After reflecting in y-axis, the co-ordinates
Write down a single transformation that
of points will be A (– 4, – 1), B (0, 7), C (2,
maps P onto P  . 5). Again reflecting in origin, the co-
Sol. P  is the image of P (4, – 7) reflected in ordinates of the images of the vertices will
x-axis be
 Co-ordinates of P  are (4, 7) A (4, 1), B  (0, – 7), C (– 2, – 5) Ans.

Again P  is the image of P  reflected in y-axis 5. The points A (4, – 11), B (5, 3), C (2, 15), and
D (1, 1) are the vertices of a parallelogram.
 Co-odinates of P are (– 4, 7) If the parallelogram is reflected in the y-axis
 Single transformation that maps P and and then in the origin, find the co-ordinates
of the final images. Check whether it remains
P  is in the origin.
a parallelogram. Write down a single
2. The point P (a, b) is first reflected in the transformation that brings the above
origin and then reflected in the y-axis to P . change.
If P  has co-ordinates (3, – 4), evaluate a, b. Sol. The points A (4, – 11), B (5, 3), C (2, 15) and
Sol. The co-ordinates of image of P(a, b) D (1, 1) are the vertices of a parallelogram.
reflected in origin are (– a, – b). After reflecting in y-axis, the images of these
points will be A (– 4, 11), B (– 5, 3), C (– 2,
Again the co-ordinates of P , image of the
15) and D (– 1, 1).
above point (– a, –b) reflected in the
y-axis are (a, – b). Again reflecting these points in origin, the
image of these points will be
But co-ordinates of P  are (3, – 4)
A (4, –11), B (5, – 3), C (2, – 15), D (1,
 a = 3 and – b = – 4  b = 4 – 1)
Hence a = 3, b = 4. Yes, the reflection of single transformation
3. A point P (a, b) becomes (– 2, c) after is in x-axis. Ans.
reflection in the x-axis, and P becomes
260 Arun Deep's Understanding Math-10
6. Use a graph paper for this question (take 2 (iv) Find the co-ordinates of A, the reflection
cm = 1 unit on both x and y axes). of A in the origin followed by reflection in
(i) Plot the following points: the y-axis.
A (0, 4), B (2, 3), C (1, 1) and D (2, 0). (v) Find the co-ordinates of B, the reflection
(ii) Reflect points B, C, D on y-axis and write of B in the x-axis followed by reflection in
down their coordinates. Name the images the origin.
as B, C, D respectively. Sol.  OAB is reflected in the origin O to  OAB,
(iii) Join points A, B, C, D, D, C, B and A in Co-ordinates of A = (– 3, – 4), B (0, – 5).
order, so as to form a closed figure. Write
down the equation of line of symmetry of  Co-ordinates of A will be (3, 4) and of B
the figure formed. (2017) will be (0, 5).
Sol. (i) On graph A (0, 4), B (2, 3), C (1, 1) and (ii) The diagram representing the given
D (2, 0) information has been drawn here.
(ii) B = (–2, 3), C = (–1, 1), D = (–2, 0) (iii) The figure in the diagram is a rectangle.
(iv) The co-ordinates of B, the reflection of B is
the x-axis are (0, – 5) and co-ordinates of B,
Y
A (0, 4) the reflection in origin of the point (0, – 5) will
4
be (0, 5).
B ( 2, 3) (v) The co-ordinates of the points, the reflecton
B (2, 3)
3 of A in the origin are (– 3, – 4) and co-ordinates
of A, the reflected in y-axis of the point (– 3,
– 4) are (3, – 4) Ans.
2

C  ( 1, 1) 1 C (1, 1)

D ( 2, 0) D (2, 0)
X X
2 1 O 1 2

1
Y

The equation of the line of symmetry is


x=0
7. The triangle OAB is reflected in the origin O
to triangle OAB. A and B have co-ordinates
(–3, – 4) and (0, – 5) respectively.
(i) Find the co-ordinates of A and B.
(ii) Draw a diagram to represent the given
information.
(iii) What kind of figure is the quadrilateral
ABAB ?
11
Section Formula
POINTS TO REMEMBER

1. To prove that a quadrilateral is a :


(i) rhombus, show that all sides are equal.
(ii) square : All sides are equal and both diagonals are equal.
(iii) Parallelogram : opposite sides are equal.
(iv) Rectangle : opposite sides are equal and diagonals are equal.
2. Section Formula :
If R, divides a line segment PQ whose co-ordinates are (x1, y1) and (x2, y2) in the ratio of m1 :
m2, then co-ordinates of R will be

m1 x2  m2 x1
x ,
m  m2

m1 y2  m2 y1
y
m1  m2

3. Mid-point Formula :
Here m1 = m2 or m1 : m2 = 1 : 1 then co-ordinates will be
x1  x2 y1  y2
, .
2 2
4. Co-ordinates of G, the centriod of a  ABC whose vertices are A (x1y1), B (x2, y2), C (x3, y3)

will be
FG x1  x2  x3 , y1  y2  y3 IJ .
H 3 3 K
Note : Centriod is the point where medians of a triangle intersect each other.

EXERCISE 11

1. Find the co-ordinates of the mid-point of the line segments joining the following pairs of points:
(i) (2, – 3), (– 6, 7) (ii) (5, – 11), (4, 3)
(iii) (a + 3, 5 b), (2 a – 1, 3 b + 4)
Sol. (i) Co-ordinates of the mid-point of (2, – 3), (– 6, 7) will be

FG x1  x2 , y1  y2 IJ or
H 2 2 K

261 Arun Deep's Understanding Math-10


262 Arun Deep's Understanding Math-10
FG 2  6 , 3  7 IJ or FG 4 , 4 IJ or (2,2) 14  9 5
   1
H 2 2 K H 2 2K 5 5
(ii) Mid-point of (5, – 11) and (4, 3)  Co-ordinates of P are (3, – 1) Ans.
2. P divides the distance between A (–2, 1)
F x  x2 , y1  y2 IJ
G 1 and B (1, 4) in the ratio of 2 : 1. Calculate
H 2 2 K the co-ordinates of the point P.
Sol. Points are A (–2, 1) and B (1, 4) and
F 5  4 11  3 IJ
or GH 2 , Let P (x, y) divides AB in the ratio of
2 K
m1 : m2 i.e. 2 :1
F 9 8 I F 9 I
or GH 2 , 2 JK or GH 2 ,  4JK
 Co-ordinates of P will be
m1 x2  m2 x1 2 × 1  1×( 2 )
x
(iii) Mid-point of (a + 3, 5 b) and m1  m2  21
(2 a – 1, 3 b + 4)
22 0
x  x2 y1  y2   0
 1 , 3 3
2 2
m1 y2  m2 y1
F a  3  2a  1 , 5 b  3b  4 IJ y
m1  m2
or G
H 2 2 K
2 × 4  1×1 8  1 9
   3
F 3a  2 8b  4 I
or GH 2 , 2 JK
21 3 3
 Co-ordinates of point P are (0, 3).
3. (i) Find the co-ordinates of the points of
F 3a  2
or GH 2 , ( 4 b  2 ) JK
I trisection of the line segment joining the
point (3, –3) and (6, 9).
P.Q. The co-ordinates of two points A and B (ii) The line segment joining the points (3, –4)
are (– 3, 3) and (12, – 7) respectively. P is and (1, 2) is trisected at the points P and Q.
a point on the line segment AB such that If the coordinates of P and Q are (p, –2)
AP : PB = 2 : 3. Find the co-ordinates of P.
5 
Sol. Points are A (– 3, 3), B (12, – 7) and  , q  respectively, find the values
3 
Let P ( x1 y1) be the point which divides
AB in the ratio of m1 : m2 i.e. 2 : 3 of p and q.
then co-ordinates of P will be Sol. (i) Let P (x1, y1) and Q (x2, y2) be the
points which trisect the line segment joining
m1 x2  m2 x1 2 × 12  3×( 3) the points A (3, – 3) and B (6, 9)
x 
m1  m2 23

24  9 15
  3 ...
5 5 P (x1, y1) divides AB in the ratio of 1 : 2
m1 y2  m2 y1 2 × ( 7 )  3 ( 3) m1 x2  m2 x1
y   x1  m1  m2
m1  m2 23
263 Arun Deep's Understanding Math-10
1× 6  2 × 3 6  6 12 Sol. The point P (x, y) divides the line segment
   4 joining the points A (3, 2) and B (5, 1) in the
1 2 3 3
ratio 1 : 2
m y  m2 y1 1× 9  2 ×( 3) mx2  nx1 1 5  2  3 5  6 11
y1  1 2   x= = = =
m1  m2 1 2 mn 1 2 3 3
96 3 my 2  ny1 11  2  2 1  4 5
  1 y= = = =
3 3 mn 1 2 3 3
 Co-ordinates of P are (4, 1).  11 5 
Thus coordinates of point P are  ,  .
Again ... Q (x 2, y2) divides the line segment  3 3
AB in the ratio of 2 : 1 P lies on the line 3x – 18y + k = 0
 It will satisfy it.
m1 x2  m2 x1
 x2  m1  m2  11  5
3   – 18   + k = 0
3 3
2 × 6  1× 3 12  3 15
   5  11 – 30 + k = 0  –19 + k = 0
2 1 3 3
k = 19
m1 y2  m2 y1 2 × 9  1 ( 3) P.Q. A point P divides the line segment joining the
y2 
m1  m2  2 1 points A (3, –5) and B (–4, 8) such that

18  3 15 AP k
  5 BP 1
= . If P lies on the line x + y = 0, then
3 3
 Co-ordinates of Q are (5, 5) find the value of k.
Sol. A point P divides the line segment joining the
(ii) Points P and Q trisect the line AB.
AP k
: : points A (3, –5), B (–4, 8) such that =
BP 1
A P Q B
(3, 4) (p, 2) (5/3, q ) (1, 2)  Ratio = AP : PB = k : 1
Let co-ordinates of P be (x, y), then
In other words, P divides it in the ratio 1 : 2 mx2  nx1 k  ( 4 )  1  3
x= =
and Q divides it in the ratio 2 : 1 mn k 1
mx2  nx1 1 1  2  3 1 6 7 4k  3
 p= = = = x=
mn 1 2 3 3 k 1

my 2  ny1 2  2  1  ( 4 ) 4  4 8k  5  my 2  ny1 
q= = = =0 and y =  y  
mn 2 1 2 k 1  mn 
Thus coordinates of point P are
7
 p= ,q=0
3  4 k  3 8 k  5 
 , 
4. The line segment joining the points A (3, 2)  k 1 k 1 
and B (5, 1) is divided at the point P in the This point lies on the line x + y = 0
ratio 1 : 2 and it lies on the line 3x – 18y + k
4k  3 8k  5
= 0. Find the value of k.  + =0
k 1 k 1
264 Arun Deep's Understanding Math-10
 –4k + 3 + 8k – 5 = 0 (i) Find the co-ordinates of P and P
2 1 (ii) Compute the distance P P
 4k – 2 = 0  4k = 2  k = = (iii) Find the middle point of the line segment
4 2
P P
 Required ratio be k : 1 i.e. 1 : 2.
(iv) On which co-ordinate axis does the middle
5. Find the coordinates of the point which is
point of the line segment P P lie ?
three-fourth of the way from A (3, 1) to
B (–2, 5). Sol. (i) Co-ordinates of P, the image of
P (3, – 5) when reflected in x-axis will be
Sol. Let P be the required point, then
(3, 5) and co-ordinates of P, the image
AP 3 of P (3, – 5)
= when reflected in y-axis will be (– 3, – 5)
AB 4
[ Mx (x, y) = (x, –y) and My (x, y) = (–x, y)]
B (2, 5) P (x, y) A (3, 1)
(ii) Length of P P ( 3  3) 2  ( 5  5) 2
and co-ordinates of A are (3, 1) and of B
are (–2, 5)  ( 6) 2  ( 10) 2  36  100
AP 3 AP 3  136  4 × 34  2 34 units
 = = =
AB 4 AP  PB 4 (iii) Let co-ordinates of middle point M be (x, y)
 4AP = 3AP + 3PB
x  x2 3  3 0
 4AP – 3AP = 3PB  x 1   0
2 2 2
AP = 3PB
y  y2 5  5 0
AP 3 y 1   0
 = 2 2 2
PB 1
 middle point is (0, 0).
 m1 = 3, m2 = 1
Let co-ordinates of P be (x, y) (iv) Middle point of PP be N (x1, y1)
m1 x2  m2 x1 3   2   1  3   x1 
3 3 0
 0
 x=
m1  m2 = 31 2 2
5  5 10
63 3 x2    5
= = 2 2
4 4
 Co-ordinates of middle point of PP are (0, – 5)
m1 y2  m2 y1 3  5  1 1 As x = 0, this point lies on y-axis Ans.
y= =
m1  m2 3 1 P.Q. Use graph paper for this question. Take
1 cm = 1 unit on both axes. Plot the points
15  1 16
= = =4 A(3, 0) and B(0, 4).
4 4
(i) Write down the co-ordinates of A 1, the
3  reflection of A in the y-axis.
 Co-ordinates of P will be  ,4  Ans.
(ii) Write down the co-ordinates of B1, the
 4 
reflection of B in the x-axis.
P.Q. Point P (3, –5) is reflected to P in the x-
axis. Also P on reflection in the y-axis is (iii) Assign the special name to quadrilateral
mapped as P. ABA1 B1.
265 Arun Deep's Understanding Math-10
(iv) If C in the mid point of AB, write down the centre is C. find the co-ordinates of the
co-ordinates of C1, the reflection of C in point C. (1990)
the origin. . .
Sol. . C is the centre of the circle and AB is the
(v) Assign the special name to the quadrilateral diameter
ABC1 B1.  C is the mid point of AB.
Sol. Two points A (3, 0) and B (0, 4) have been Let co-ordinates of C (x, y)
plotted on the graph.
3  5 1 4
 x , y
Y 2 2
4 B(0, 4)

3
A B(5, –4)
C
3 ,2
(–3, 1) C
2 2

1
A (, 0) 2 3
A(3, 0)  x , y
X`
O
X 2 2
3 2 1 1 2 3
3
1  x = 1, y 
2
FG 3 IJ
C1
–3 , –2
2
2  Co-ordinates of C are 1, 2 Ans.
H K
–3 7. The mid-points of the line segment joining the
points (3m, 6) and (–4, 3n) is (1, 2m – 1).
 B1(0, 4)
Find the value of m and n.

Y`
Sol. Let the mid-point of the segment joining
two points
(i) A1 is the reflection of A(3, 0) in the y-
A(3m, 6) and (–4, 3n) is P(1, 2m – 1)
axis.
 Its co-ordinates will be (– 3, 0). x1  x2 3m  4
 1= 
(ii) B1 is the reflection of B(0, 4) in the x-axis. 2 2
 co-ordinates of B1 will be (0, – 4)  3m – 4 = 2  3m = 2 + 4 = 6
(iii) The so formed figure ABA1B1 is a rhombus. 6 6  3n
 m= = 2 and 2m – 1 =
(iv) C is the mid point of AB 3 2
co-ordinates of C will be  4m – 2 = 6 + 3n
 4 × 2 – 2 = 6 + 3n  8 – 2 = 6 + 3n
30 0 4 3 
 ,  or  ,2   n=0
 2 2  2 
Hence m = 2, n = 0 Ans.
C, is the reflection of C in the origin
8. The co-ordinates of the mid-point of the
F 3 I
co-ordinates of C, will be GH 2 ,  2JK .
line segment PQ are (1, – 2). The co-
ordinates of P are (– 3, 2). Find the co-
(v) The name of quadrilateral ABC1B1 is a ordinates of Q.(1992)
trapezium because AB is parallel to B1C1. Sol. Let the co-ordinates of Q be (x, y).
6. The line segment joining A (– 3, 1) and
co-ordinates of P are (– 3, 2) and mid-
B (5, – 4) is a diameter of a circle whose
point of PQ are (1, – 2) then
266 Arun Deep's Understanding Math-10
point A (5, – 3) be A1 (x, y) in the point (– 1, 3)
P (, 2) M (1, –2) Q (x , y ) then the point (– 1, 3) will be the mid point of AA1.
3  x 5 x
1  – 3 + x = 2  1  5+x=–2
2 2
 x=2+3=5  x= –2–5=–7
2 y
and 2  2+y=–4 3  y
2 and 3  –3+y=6
2
 y=–4–2=–6
 y=6+3=9
 x = 5, y = – 6
 Co-ordinates of the image A, will be (– 7, 9).
Hence co-ordinates of Q are (5, – 6) Ans.
9. AB is a diameter of a circle with centre  5
C (–2, 5). If point A is (3, –7). 11. The line segment joining A   1,  and B
 3
Find : (i) the length of radius AC.
(a, 5) is divided in the ratio 1 : 3 at P, the point
(ii) the coordinates of B.
where the line segment AB intersects y-axis.
Sol. AC = 3  2 2   7  52 Calculate (i) the value of a (ii) the co-ordinates
of P. (1994)
[using distance formula]
Sol. Let P (x, y) divides the line segment joining
= 5  12
2 2 = 25  144
 5
the points A   1,  , B (a, 5) in the ratio 1 : 3
= 169 = 13 units  3

1:3

A(, 5/3) P (x, y) B (a, 5)


A B
(3,  7) C (x , y ) 1× a  3× ( 1) a3
(2, 5)  x 
1 3 4

1  a  3  (1) a  3 5  5 10 5
AB is diameter and C is mid point of AB y=    
1 3 4 4 4 2
Let co-ordinate of B are (x, y)
(i) ... AB intersects y-axis at P
3 x y7
 = –2 and =5 a3
2 2  x=0 0a–3=0
4
3 + x = –4 and y – 7 = 10
 a=3
x = –4 – 3 and y = 10 + 7
x = –7 and y = 17  5
(ii)  Co-ordinates of P are  0,  Ans.
 Co-ordiantes of B be (–7, 17).  2
10. Find the reflection (image) of the point 12. The point P (– 4, 1) divides the line segment
(5, – 3) in the point (– 1, 3). joining the points A (2, – 2) and B in the
Sol. Let the co-ordinates of the images of the ratio of 3 : 5. Find the point B.
267 Arun Deep's Understanding Math-10
Sol. Let the co-ordinates of B be (x, y) m1 ( 14 )  m2 × 2
Co-ordinates of A (2, – 2) and point P (– 4, 1)  4  m1  m2
divides AB in the ratio of 3 : 5
m1 : m2
3 × x  5×( 2 ) 3 x  10 P(2, –2) Q(–14, 6)
 4   R(–4, b)
35 8
 – 4 m1 – 4 m2 = – 14 m1 + 2 m2
and 3 x + 10 = – 32  3 x = – 32 – 10 = – 42
 – 4 m1 + 14 m1 = 2 m2 + 4 m2
42
 x  14  10 m1 = 6 m2
3
3× y  5×( 2 ) 3 y  10 m1 6 3
1 1=  m  10   m1 : m2 = 3 : 5
35 8 2 5
 3 y – 10 = 8  3 y = 8 + 10 = 18 Again
18
 y 6 m1 × 6  m2 ×( 2 ) 6 m1  2 m2
3 b 
m1  m2 m1  m2
 Co-ordinates of B be (– 14, 6) Ans.
13. (i) In what ratio does the point (5, 4) divide 6 × 3  2 × 5 18  10 8
the line segment joining the points (2, 1)  b   1
35 8 8
and (7, 6) ?
 b=1
(ii) In what ratio does the point (–4, b)
14. The line segment joining A (2, 3) and
divide the line segment joining the points P
B (6,  5) is intercepted by the x-axis at
(2,–2), Q (–14, 6) ? Hence find the value
the point K. Write the ordinate of the point
of b.
k. Hence, find the ratio in which K divides
Sol. (i) Let the ratio be m1 : m2 that the point AB. Also, find the coordinates of the point
(5, 4) divides the line segment joining the K.
points (2, 1), (7, 6).
Sol. Let the co-ordinates of K be (x, 0) as it
m1 × 7  m2 × 2 intersects x-axis.
 5 m1  m2 Let point K divides the line segment joining
m : n2 the points A (2, 3) and B (6, –5) in the ratio
A(2, 1) P(5, 4) B(7, 6) m 1 : m 2.

m1 y2  m2 y1
 0 m1  m2
 5 m1 + 5 m2 = 7 m1 + 2 m2
 5 m2 – 2 m2 = 7 m1 – 5 m1  3 m2 = 2 m1
m1 ×( 5)  m2 × 3
 0 m1  m2
m1 3
 m  2  m1 : m2 = 3 : 2
2  – 5 m1 + 3 m2 = 0  – 5 m1 = – 3 m2
(ii) The point (– 4, b) divides the line segment
m1 3
joining the points P (2, – 2) and Q (– 14, 6)  m  5  m1 : m2 = 3 : 5
in the ratio m1 : m2. 2
268 Arun Deep's Understanding Math-10
Now, By joining AB, we see
that O (0, 0) lies on AB
m1 x2  m2 x1 3  6  5  2
x=  Let O divides AB in the ratio m1 : m2
m1  m2 35
m1 x 2  m 2 x1 m1  8  m2  4
18  10 28 7  x= m1  m 2  0 = m1  m2
=  
8 8 2
 8m1 – 4m2 = 0
7 
 8m1 = 4m2
Co-ordinate of K are  2 , 0 Ans. m1 4 1
   m = =
2 8 2
15. If A = (–4, 3) and B = (8, –6), (i) find the  m1 : m2 = 1 : 2
length of AB. (ii) in what ratio is the line
 O, divides AB in the ratio 1 : 2.
joining AB, divided by the x-axis ?
(2008) 16. In what ratio does the line x – y – 2 = 0
divide the line segment joining the points
Sol. A = (–4, 3) and B = (8, –6) (3, –1) and (8, 9) ? Also find the coordinates
of the point of division.
 Length of AB = x2  x1 2   y 2  y1 2
Sol. Let the points be A (3, –1) and B (8, 9) and
let line x – y – 2 = 0 divides the line segment
Y joining the points A and B in the ratio
5 m1 : m2 at point P (x, y) then
4
A (4, 3) m1 x2  m2 x1 m1  8  m2  3
3
x= 
m1  m2 m1  m2
2
m1
1
m1 y2  m2 y1
X` X and y = m1  m2
4 3 2 1 O 1 2 3 4 5 6 7 8
1

2
m1  9  m2 (1) 9m1  m2
3
 
m2 m1  m2 m1  m1
4

5
m1: m2
A B
6 (3, –1) P(x, y) (8, 9)
B (8, 6)
Y` . .. The point P (x, y) lies on the line x – y – 2 = 0

= 8   4 2   6  32 
8m1  3m2

9m1  m2
20
m1  m2 m1  m2
= 8  42   6  32  8m1 + 3m2 – 9m1 + m2 – 2m1 – 2m2 = 0
 –3m1 + 2m2 = 0  3m1 = 2m2
= 122   92 = 144  81
m1 2
 m 3
= 225 = 15 2
269 Arun Deep's Understanding Math-10
(i) ... Ratio = m1 : m2 = 2 : 3 (iii) In what ratio does the y-axis divide the line
AB ?
2  8  3  3 16  9 25
 x=   5 Sol. (i) Let co-ordinate of P be (x, y) which
23 5 5
divides the line segment joining the points A
2  9  3  (1) 18  3 15 (– 4, 1) and B (17, 10) in the ratio of 1 : 2.
and y =   3
23 5 5
m1 x2  m2 x1
 x
(ii)  Co-ordinates of point P are (5, 3) m1  m2
17. Given a line segment AB joining the points
A (–4, 6) and B (8, –3). Find: 1:2
(i) the ratio in which AB is divided by the A(–4, 1) P(x, y) B(17, 10)
y-axis.
1× 17  2 ×( 4 ) 17  8 9
(ii) find the coordinates of the point of    3
1 2 3 3
intersection.
(iii) the length of AB. m y  m2 y1
y 1 2
Sol.(i) Let the y-axis divide AB in the ratio m : 1 m1  m2

So, 1× 10  2 × 1 10  2 12
   4
m  8  4 1 1 2 3 3
0=  8m – 4 = 0
m 1  Co-ordinates of P are (3, 4).
4 1 (ii) Distance of OP where O is the origin i.e.
 m =  m =
8 2 co-ordinates are (0, 0)
1
So, required ratio = : 1 or 1 : 2  Distance 
2 ( 3  0 ) 2  ( 4  0) 2
m P(0, y) 1
A
(–4, 6)
B  32  4 2  9  16  25  5 units
(8, –3)
(iii) Let y-axis divides AB in the ratio of m1 : m2
1   3  2  6 9 at P and let co-ordinates of P be (0, y)
(ii) Also, y = = =3
1 2 3
m x  m2 x1
So, coordinates of the point of intersection  0  1 2
m1  m2
be (0, 3).
m1 ×17  m2 ×( 4 )
(iii) AB = 8  42   3  62    m1  m2

= 144  81 =  17 m1 – 4 m2 = 0
225 = 15 units
 17 m1 = 4 m2
P.Q. (i) Write down the co-ordinates of the point
P that divides the line joining A (– 4, 1) and m1 4
B (17, 10) in the ratio 1 : 2.  m  17
2
(ii) Calculate the distance OP where O is the
 m1: m2 = 4 : 17
origin.
270 Arun Deep's Understanding Math-10
... OA also the mid point of second diagonal
18. Calculate the length of the median through
the vertex A of the triangle ABC with BD and let co-ordinates of D be (x, y)
vertices A (7, –3), B (5, 3) and C (3, –1) 5 1 x
   10 = 2 + 2 x
Sol. Let D (x, y) be the median of  ABC through 2 2
A to BC.  2 x = 10 – 2 = 8
 D will be the mid point of BC 8 0 y
 x 4 and 1  y=2
 Co-ordinates of D will be, 2 2
 Co-ordinates of D are (4, 2) Ans.
A(7, –3)
20. If the points A (–2, –1), B (1, 0), C (p, 3)
and D (1, q) from a parallelogram ABCD,
find the values of p and q.
Sol. A (–2, –1),
B (1, 0), C (p, 3) and D (1, q) are the vertices
of a parallelogram ABCD
B(5, 3) D C(3, –1)
D (1, q) C (p, 3)

5 3 8 31 2
x   4 and y   1
2 2 2 2
O
Co-ordinates of D are (4, 1)
A (2, 1) B (1, 0)
 Length of DA  ( 7  4 ) 2  ( 3  1) 2
Diagonal AC and BD
 ( 3) 2  ( 4 ) 2  9  16 bisect each other at O
 25  5 units. O is the mid point of AC as well as BD
Let co-ordinates of O be (x, y)
19. Three consecutive vertices of a
parallelogram ABCD are A (1, 2), B (1, 0) When O is mid-point of AC, then
and C (4, 0). Find the fourth vertex D. p2 3 1 2
 x= ,y= = =1
Sol. Let O in the mid-point of AC the diagonal 2 2 2
of ABCD Again when O is the mid-point of BD
 Co-ordinates of O will be 1 1 2 0q q
Then x = = = 1 and y = =
2 2 2 2
FG 1  4 , 2  0 IJ or FG 5 ,1IJ Now comparing, we get
H 2 2 K H2 K
p2
=1p–2=2p=2+2=4
D C 2
q
 p = 4 and =1q=2
2
Hence p = 4, q = 2
O
21. If two vertices of a parallelogram are (3, 2)
(–1, 0) and its diagonals meet at (2, –5), find
A B the other two vertices of the parallelogram.
271 Arun Deep's Understanding Math-10
D C  ( 1  5) 2  ( 6) 2  ( 4 ) 2  ( 6 ) 2

P  16  36  52
(2, 5)
BC  [5  ( 1)]2  [ 2  ( 2 ) 2 ]
A (3, 2) B (1, 0)  (5  1) 2  ( 2  2 ) 2
Sol. Two vertices of a ||gm ABCD are A (3, 2),  ( 6) 2  ( 4 ) 2
B (–1, 0) and point of intersection of its
..
diagonals is P (2, –5)  36  16  52 . AB = BC
P is mid-point of AC and BD.   ABC is an isosceles triangle.
Let co-ordinates of C be (x, y), then AC  ( 5  5) 2  ( 4  2 ) 2
x3  ( 10) 2  ( 2 ) 2 
2= x+3=4x=4–3=1 100  4  104
2
Now AC2 = AB2 + BC2
y2
and –5 =  y + 2 = –10  ( 104 ) 2  ( 52 ) 2  ( 52 ) 2
2
 104 = 52 + 52  104 = 104
 y = –10 – 2 = –12 . . . AC2 = AB2 + BC2
 Co-ordinates of C are (1, –12)   ABC is an isoceles right angled triangle.
Similarly we shall find the co-ordinates of D also Let the coordinates of D be (x, y) and
diagonals AC and BD of ABCD is a square,
x 1 then diagonals bisect each other at O.
2= x–1=4x=4+1=5
2 If ABCD is a square, then diagonals bisects
y0 each other at O.
–5 =  –10 = y . . . O is the mid point of AC
2
 Co-ordinates of D are (5, –10).  Co-ordinates of O will be

P.Q. Prove that the points A (–5, 4), B (–1, –2) FG 5  5 , 2  4 IJ or FG 0 , 6 IJ or (0, 3)
and C (5, 2) are the vertices of an isosceles H 1 2 K H 2 2K
... 0 is mid-point of BD also
right angled triangle. Find the co-ordinates
of D so that ABCD is a square. 1  x
 0 –1+x=0x=1
Sol. Points A (–5, 4), B (–1, –2) and C (5, 2) 2
are given. If these are vertices of an isosceles 2  y
triangle ABC then AB = BC. and 3  –2+y=6
2
 y=6+2=8
 Co-ordinates of D are (1, 8)
22. Find the third vertex of a triangle if its two
vertices are (– 1, 4) and (5, 2) and mid
point of one sides is (0, 3).
Sol. Let A (– 1, 4) and B (5, 2) be the two points
and let D (0, 3) be its the mid point of AC
AB  [ 1  ( 5)]2  ( 2  4 ) 2 and co-ordinates of C be (x, y).
272 Arun Deep's Understanding Math-10
x1  x2  x + x = 0
C(x, y) 0 1 2 ...(i)
2
1 y  y2
D  1  y1 + y2 = 1 ...(ii)
(0, 3) 2 2
1 x  x3
Again  2 = x2 + x3 = 1 ...(iii)
A (–1, 4) B (5, 2)
2 2
1 y  y3
and  2  y2 + y3 = 1 ...(iv)
x 1 2 2
 0 x–1=0x=1
2 1 x  x1
and  3  x3 + x1 = 1 ...(v)
y4 2 2
3 y+4=6y=6–4=2
2 y3  y1
 Co-ordinates of C will be (1, 2) 0  y3 + y1 = 0 ...(vi)
2
If we take mid-point D (0, 3) of BC, then
Adding (i), (iii) and (v) ; we have
5 x 2 (x1 + x2 + x3) = 0 + 1 + 1 = 2
0 x+5=0 x=–5
2  x1 + x2 + x3 = 1
2 y Now substracing (iii), (v) and (i)
and 3  2+y=6y=6–2=4
2 respectively, we get
 Co-ordination of C will be (–5, 4) x1 = 0, x2 = 0, x3 = 1
Hence co-ordinates of C, third vertx will Again Adding (ii), (iv) and (vi)
be (1, 2) or (– 5, 4) 2 (y1 + y2 + y3) = 1 + 1 + 0 = 2
23. Find the coordinates of the vertices of the
 y1 + y2 + y3 = 1
triangle the middle points of whose sides
Now subtracting (iv), (vi) and (ii)
FG 1
are 0 , 2 ,
IJ FG 1 , 1 IJ and FG 1 , 0IJ . respectively we get,
H K H 2 2K H 2 K y1 = 0, y2 = 1, y3 = 0
F 1I  Co-ordinates of A, B and C will be
Sol. Let ABC be a  in which D GH 0, 2 JK ,
(0, 0), (0, 1) and (1, 0) Ans.
24. Show by section formula that the points
F 1 1I F1 I
E GH 2 , 2 JK and F GH 2 , 0JK are the mid- (3, – 2), (5, 2) and (8, 8) are collinear.
Sol. Let the point (5, 2) divides the line joining
points of sides AB, BC and CA respectively.
the points (3, – 2) and (8, 8) in the ratio of
Let co-ordinates of A be (x1, y1), m 1: m 2
B (x2, y2) and C (x3, y3)
m1 × 8  m 2 × 3
 5
m1  m 2
 8 m1 + 3 m2 = 5 m1 + 5 m2
 8 m1 – 5 m1  5 m2 – 3 m2
m1 2
 3 m1 = 2 m2  m  3 ...(i)
2
8 m1  2 m 2
Again 2  m1  m 2
273 Arun Deep's Understanding Math-10
 8 m1 – 2 m2 = 2 m1 + 2 m2 Co-ordinaes of M will be
 8 m1 – 2 m1 = 2 m2 + 2 m2
m1 4 2 
FG 10  2 , 5  1 IJ or  FG 12 , 6 IJ or (6, 3)
 6 m1 = 4 m2  m  6  3 ...(ii) H 2 2 K H 2 2K
2
from (i) and (ii) it is clear that point (5, 2) Length of LM  ( 6  8) 2  ( 3  1) 2
lies on the line joining the points (3, – 2)
and (8, 8). Hence proved.  ( 2 ) 2  ( 2 ) 2  44  8
25. Find the value of p for which the points
 4 × 2  2 2 units ...(i)
(–5, 1), (1, p) and (4, –2) are collinear.
Sol. Let point A(–5, 1), B(1, p) and C(4, –2) Length of BC = ( 2  6) 2  [1  ( 3)] 2
are collinear and let point A(–5, 1) divides
BC in the ratio m1: m2  ( 4 ) 2  (1  3) 2  ( 4 ) 2  ( 4 ) 2
m x  m2 m1  16  16  32  16  2  4 2 units ...(ii)
 x 1 2
m1  m2 from (i) and (ii) ; we have
m1  4  m1  1 4m1  m2 1
 5   LM  BC.
m1  m2 m1  m2 2
 –5m1 – 5m2 = 4m1 + m2 P.Q. A (2, 5), B (–1, 2) and C (5, 8) are the
 –5m1 – 4m1 = m2 + 5m2 vertices of a triangle ABC. P and Q are
m1 6 2 points on AB and AC respectively such that
 –9m1 = 6m2  m  9  3 ...(i) AP : PB = AQ : QC = 1 : 2.
2
(i) Find the co-ordinates of P and Q.
m  (2)  m2  p 2m1  m2 p 1
and 1  1 
m1  m2 m1  m2 (ii) Show that PQ = BC.
3
 m1 + m2 = –2m1 + m2p  m1 + 2m1=m2 p – m2 Sol. A (2, 5), B (–1, 2) and C (5, 8) are the
vertices of ABC, P and Q are points on AB and
m1 p  1 AP AQ 1
3m1 = m2(p – 1)  
AC respectively such that PB  QC  2
...(ii)
m2 3
From (i) and (ii) ; we have A(2, 5)
p 1 2 1
  –3p + 3 = 6 1
3 3 P Q
3
 –3p = 6 – 3  –3p = 3  p =  1 2
3 2
 p = –1
P.Q. A (10, 5), B (6, – 3) and C (2, 1) are the B(–1, 2) C(5, 8)
vertices of triangle ABC. L is the mid point
of AB, M is the mid-point of AC. Write Let co-ordinates of P be (x1, y1) and of Q
down the co-ordinaes of L and M. Show be (x2, y2)
1
that LM  BC. (2003) P divides AB in the ratio 1 : 2
2
Sol. Co-ordinates of L will be m1 x2  m2 x1 1  (1)  2  2
 x1 = 
m1  m2 1 2

FG 10  6 , 5  3 IJ 1  4 3
H 2 2 K =  1
3 3
F 16 , 2 IJ or (8, 1)
or GH m1 y2  m2 y1 1  2  2  2 2  10 12
   4
2 2K y1 =
m1  m2 1 2 3 3
274 Arun Deep's Understanding Math-10
 Co-ordiantes of P will be (1, 4) (1, –1) and C (5, 1) (2006)
Similarly Q divides AC in the ratio 1 : 2 Sol. Co-ordinates of the centroid of a triangle
m1 x2  m2 x1 1 5  2  2 whose vertices are (x1, y1), (x2, y2) and
 x2 = =
m1  m2 1 2  x1  x2  x3 y1  y2  y3 
(x3, y3) are  , 
54 9  3 3 
= = =3
3 3  Co-ordinates of the centroid of the given triangle
m1 y 2  m2 y1 1 8  2  5  1  1  5 3 1 1 5 
and y2 = = are  ,  i.e.  , 1 Ans.
m1  m2 1 2  3 3  3 
8  10 18 28. Two vertices of a triangle are (3, – 5)
= = =6 and (– 7, 4). Find the third vertex given
3 3
that the centroid is (2, – 1).
 Co-ordinates of Q will be (3, 6)
Sol. Let the co-ordinates of third vertices be
(ii) Now length of BC = x2  x1 2   y2  y1 2 (x, y) and other two vertices are (3, – 5)
and (– 7, 4) and centroid = (2, – 1).
= 5  12  8  22 = 62  62 3 7  x x4
2  2
3 3
= 36  36 + 72 = 2  36 = 6 2
x – 4 = 6  x = 6 + 4  x = 10
and PQ = 1  32  4  62 5  4  y
and  – 1  –3=–1+y
3
=  2 
2
  2  = 4  4 = 8 = 2  4 = 2 2
2
y=–3+1=2
3 BC 1  Co-ordinates are (10, – 2) Ans.
6 2
= ×2 6 = = = BC 29. The vertices of a triangle are A (–5, 3), B
3 3 3 3
(p – 1) and C (6, q). Find the values of p
Hence proved. and q if the centroid of the triangle ABC is
26. The mid-point of the point (1, –1).
the line segment AB
Sol. The vertices of ABC are
shown in the adjoining
diagram is (4, – 3). Write A (–5, 3), B (p – 1), C (6, q)
down the co-ordinates of and centroid of ABC is O (1, –1)
A and B. co-ordinates of centroid of ABC will be
 5  p  6 3 1 q  1 p 2  q 
Sol. A lies on x-axis  ,    3 , 3 
and B on y-axis.  3 3   
Let co-ordinates of A be (x, 0) and of B be (0, y) But centroid is given (1, –1)
P (4, – 3) is the mid-point of AB  Comparing, we get
x0 1 p
4 x=8 =11+p=3
2 3
0 y  p=3–1=2
and 3  y=–6
2 2q
Co-ordinates of A will be (8, 0) and of B and = –1  2 + q = –3
3
will be (0, – 6) Ans.
27. Find the co-ordinates of the centroid of a  q = –3 – 2  q = –5
triangle whose vertices are A (–1, 3), B Hence p = 2, q = –5
275 Arun Deep's Understanding Math-10
MULTIPLE CHOICE QUESTIONS
a 
Sol. P  , 4  is mid-point of the line segment
Choose the correct answer from the given four 3 
options (1 to 12) : joining the points Q (–6, 5) and R (–2, 3)
1. The points A (9, 0), B (9, 6), C (–9, 6) and
D (–9, 0) are the vertices of a a 6  2 8
 = = = –4
3 2 2
(a) rectangle (b) square
(c) rhombus (d) trapezium a = –4 × 3  a = –12  Ans. (d)
Sol. Given vertices are ; 3. If the end points of a diameter of a circle are
A (–2, 3) and B (4, –5), then the coordinates
A (9, 0), B (9, 6), C (–9, 6), D (–9, 0)
of its centre are
AB2 = (x2 – x1)2 + (y2 – y1)2
(a) (2, –2) (b) (1, –1)
= (9 – 9)2 + (6 – 0)2 = 02 + 62
(c) (–1, 1) (d) (–2, 2)
= 02 + 36 = 36
Sol. End points of a diameter of a circle are
CD2 = (–9 + 9)2 + (6 – 0)2 (–2, 3) and B (4, –5) then co-ordinates of
= 02 + 62 = 0 + 36 = 36 the centre of the circle
BC2 = (9 + 9)2 + (6 – 6)2
= 182 + 02 = 324 + 0 = 324
AD2 = (9 + 9)2 + 02 A (–2, 3) B(4, –5)
C
= 182 + 02 = 324 + 0 = 324
AB = CD and BC = AD
But these are opposite sides of a rectangle  2 4 35 2 2
=  ,  or  , 
 ABCD is a rectangle. (a)  2 2  2 2 
P.Q. The mid-point of the line segment joining the = (1, –1)  Ans. (b)
points A (–2, 8) and B (–6, –4) is 4. If one end of a diameter of a circle is (2, 3)
(a) (–4, –6) (b) (2, 6) and the centre is (–2, 5), then the other end
(c) (–4, 2) (d) (4, 2) is
Sol. Mid-point of the line segment joining the (a) (–6, 7) (b) (6, –7)
points A (–2, 8), B (–6, –4) (c) (0, 8) (d) (0, 4)
Sol. One end of a diameter of a circle is (2, 3)
 2  6 8  4   8 4 
=  ,  or  ,  and centre is (–2, 5)
 2 2   2 2
Let (x, y) be the other end of the diameter
= (–4, 2)  Ans. (c)  C(–2, 5) be the centre of AB.
a  2 x
3. If P  , 4  is the mid-point of the line = –2  2 + x = –4
3  2
segment joining the points Q (–6, 5) and
R (–2, 3), then the value of a is
(a) –4 (b) –6 (2, 3)A B(x, y)
C(–2, 5)
(c) 12 (d) –12
276 Arun Deep's Understanding Math-10
 x = –4 – 2 = –6
Y
3 y A (0, 2y)
and = 5  3 + y = 10
2
 y = 10 – 3 = 7
 Co-ordinates of other end are (–6, 7) (a)
5. If the mid-point of the line segment joining
the points P (a, b – 2) and Q (–2, 4) is X’ O B(2x, 0) X
R (2, –3), then the values of a and b are Y’

(a) a = 4, b = –5 (b) a = 6, b = 8 Sol. In the given figure, the vertices of a OAB


(c) a = 6, b = –8 (d) a = –6, b = 8 are (0, 0), (0, 2y) and (2x, 0)
Sol. the mid-point of the line segment joining the The point which is equidistant from O, A and
points P (a, b – 2) and Q (–2, 4) is R (2, –3) B is the mid-point of AB.

a2  0  2x 2 y  0 
2= a–2=4  Co-ordinates are  , 
2  2 2 
 a=4+2=6 or (x, y) (a)
8. The fourth vertex D of a parallelogram ABCD
b24 b2 whose three vertices are A (–2, 3), B (6, 7)
–3 = =
2 2 and C (8, 3) is
 b + 2 = –6  b = –6 – 2 = –8 (a) (0, 1) (b) (0, –1)
 a = 6, b = –8  Ans. (c) (c) (–1, 0) (d) (1, 0)
6. The point which lies on the perpendicular Sol. ABCD is a ||gm whose vertices A (–2, 3),
bisector of the line segment joining the points B (6, 7) and C (8, 3). The fourth vertex D
A (–2, –5) and B (2, 5) is will be the point on which diagonals AC and
BD bisect each other at O.
(a) (0, 0) (b) (0, 2)
(c) (2, 0) (d) (–2, 0) D (x, y) C (8, 3)
Sol. the line segment joining the points A (–2, –5)
and B (2, –5), has mid-point

2 2 55
 ,  = (0, 0) O
 2 2 
 (0, 0) lies on the perpendicular bisector of
A ( 2, 3) B (6, 7)
AB.  Ans. (a)
7. The coordinates of the point which is
equidistant from the three vertices of AOB   2 8 3 3
 Co-ordinates of O are  ,  or
(shown in the given figure) are  2 2 
(a) (x, y) (b) (y, x) 6 6
 ,  or (3, 3)
x y  y x 2 2
(c)  ,  (d)  , 
2 2  2 2 Let co-ordinates of D be (x, y), then
277 Arun Deep's Understanding Math-10
x6 ratio 1 : 2 internally lies in the
3= 6=x+6x=6–6=0 (a) Ist quadrant (b) IInd quadrant
2
(c) IIIrd quadrant (d) IVth quadrant
y7 Sol. A point divides line segment joining the points
and 3 = y+7=6
2 A (7, –6) and B (3, 4) in the ratio 1 : 2
 y = 6 – 7 = –1 internally.
 Co-ordinates of D are (0, –1)  Ans. (b)
Y
P.Q. A line intersects the y-axis and x-axis at the
points P and Q respectively. If (2, –5) is the B (3, 4)
mid-point of PQ, then the coordinates of P
and Q are, respectively
(a) (0, –5) and (2, 0)
X X
(b) (0, 10) and (–4, 0) O
P (x, y)
(c) (0, 4) and (–10, 0)
(c) (0, –10) and (4, 0)
A (7, 6)
Sol. A line intersects y-axis at P and x-axis a Q.
R (2, –5) is the mid-point Y

Y Let (x, y) divides it in the ratio 1 : 2


mx2  nx1 1 3  2  7 3  14
 x= = =
mn 1 2 3
(2, 5) P 17 2
R = =5
X X 3 3
Q O
my 2  ny1 1  4  2  ( 6 )
y= =
mn 1 2
4  12 8
= =
Y 3 3
We see that x is positive and y is negative.
Let co-ordinates of P be (0, y) and of Q be
 It lies in the fourth quadrant.  Ans. (d)
(x, 0), then
10. The centroid of the triangle whose vertices
0 x are (3, –7), (–8, 6) and (5, 10) is
2= x=4
2 (a) (0, 9) (b) (0, 3)
(c) (1, 3) (d) (3, 3)
y0
and –5 =  y = –10 Sol. Centroid of the triangle whose vertices are
2
(3, –7), (–8, 6) and (5, 10) is
 Co-ordinates of P are (4, 0) and of Q are
(0, –10)  Ans. (d)  3  8  5  7  6  10   9
 ,  or  0, 
9. The points which divides the line segment  3 3   3
joining the points (7, –6) and (3, 4) in the or (0, 3)  Ans. (b)
278 Arun Deep's Understanding Math-10
CHAPTER TEST P.Q. A and B have co-ordinates (4, 3) and
1. The base BC of an equilateral triangle ABC (0, 1), Find
lies on y-axis. The coordinates of the point (i) the image A of A under reflection in the y-
C are (0, –3). If origin is the mid-point of axis.
the base BC, find the coordinates of the points (ii) the image of B of B under reflection in the
A and B. line AA.
Sol. Base BC of an equilateral ABC lies on (iii) the length of AB.
y-axis co-ordinates of point C are (0, –3),
origin (0, 0) is the mid-point of BC. Sol.

A
X X
O
(0, 0)
C
(0, 3)

Y

Let co-ordinates of B be (x, y)

x0 x
 0=  =0x=0
2 2
(i) Co-ordinates of A, the image of A (4, 3)
y 3 reflected in y-axis will be (– 4, 3).
=0y–3=0
2
(ii) Co-ordinates of B the image of B (0, 1)
 y=3 reflected in the line AA will be (0, 5).
 Co-ordinates of B are (0, 3) (iii) Length of AB
Again let co-ordinates of A be (x, 0) as it lies
on x-axis.  [ 0  ( 4 )] 2  (5  3) 2
AB = AC = BC = 6 units
 ( 4) 2  ( 2) 2  16  4
= ( x  0)  (0  3)
2 2
= 62  20  4 × 5  2 5 units
 x2 + (–3)2 = 62 2. Find the co-ordinates of the point that
 x2 + 9 = 36  x2 = 36 – 9 = 27 divides the line segment joining the points
P (5, –2) and Q (9, 6) internally in the ratio
 x = +3 3 of 3 : 1.
Sol. Let R be the point whose co-ordinates are
 Co-ordiantes of A will be (+3 3 , 0)
(x, y) which divides PQ in the ratio of 3:1.
279 Arun Deep's Understanding Math-10
m1 x 2  m 2 x 1 4. P and Q are the points on the line segment
 x joining the points A (3, – 1) and B (– 6, 5)
m1  m 2
such that AP = PQ = QB. Find the co-
ordinates of P and Q.
3:1
Sol. ... AP = PQ = QB
P(5, –2) R(x, y) Q(9, 6)
1 : 2 2 : 1
3× 9  1× 5 27  5 32 P Q
   8 A(3, –1) (–6, 5)B
31 4 4
 P divides AB in the ratio of 1 : 2 and Q
m y  m2 y1 3× 6  1×( 2 )
y 1 2  divides it in 2 : 1.
m1  m2 31
Let co-ordinates of P will be (x1, y1) and
18  2 16 of Q will be (x2, y2)
  4
4 4
m1 x 2  m 2 x 1 1×( 6)  2 ( 3)
 Co-ordinates of R will be (8, 4)  x1  
m1  m 2 1 2
3. Find the coordinates of the point P which is
three-fourth of the way from A (3, 1) to 6  6 0
  0
B (–2, 5). 3 3
Sol. Co-ordinates of A (3, 1) and B (–2, 5) m1 y 2  m 2 y 1
P lies on AB such that y1 
m1  m 2
3 3
AP = AB = (AP + PB) 1× 5  2 ( 1) 52 3
4 4    1
1 2 3 3
 AP = 3PB  AP : PB = 3 : 1  Co-ordinates of P will be (0, 1)
Let co-ordinates of P be (x, y) Again
mx2  nx1 3  (2)  1  3 m1 x 2  m 2 x 1
 x= = 2 ×( 6)  1 ( 3)
mn 3 1 x2  
m1  m 2 2 1
6  3 3
= = 12  3 9
4 4    3
3 3
3:1 m1 y 2  m 2 y 1 2 × 5  1 ( 1)
y2  
A(3, 1) P(x, y) B(–2, 5) m1  m 2 2 1
my 2  ny1 3  5  1  1 15  1 16 10  1 9
y=
mn
=
3 1
=
4
=
4
=4   3
3 3
 Co-ordinates of Q will be (– 3, 3) Ans.
3 
 Co-ordinates of P are  , 4 5. The centre of a circle is ( + 2,  – 5). Find
 4 
the value of  given that the circle passes
280 Arun Deep's Understanding Math-10
through the points (2, – 2) and (8, – 2).  q=4
Sol. Let A (2, – 2), B (8, – 2) and centre of the p4
circle be O ( + 2,  – 5). and 5   p + 4 = 10
2
B  p = 10 – 4 = 6
A  p=6
O ( + 2,  – 5) Hence p = 6, q = 4
7. The ends of a diameter of a circle have the
co-ordinates (3, 0) and (–5, 6). PQ is
. .. OA = OB = radii of the circle another diameter where Q has the co-
ordinates (–1, –2). Find the co-ordinates
OA  (2    2)2  (2    5) 2 of P and the radius of the circle.
Sol. Let AB be the diameter where co-ordinates
  2  (3   )2   2  (   3)2 ...(i) of A are (3, 0) and of B are (– 5, 6).
 Co-ordinates of its origin O will be
and OB =  8    2 2    2    5  2
F 3  5 , 0  6 I or F 2 , 6 I or (– 1, 3)
 (6   )2  (3   )2
H 2 2 K H 2 2K
Now PQ is another diameter in which
 (6   )2  (3   )2 ...(ii) co-ordinates of Q are (– 1, – 2).
Let co-ordinates of P be (x, y)
From (i) and (ii) ; we have
Then co-ordinates of centre O will be
2 2 2 2
  (  3)  (6   )  (3   )
FG 1  x ,  2  y IJ
Squaring both sides, we have H 2 2 K
2 + ( – 3)2 = (6 – )2 + (3 – )2
 2 = (6 – )2 P(x, y) B(–5, 6)
 2 = 36 – 12  + 2
O(–1, 3)
 2 – 2 + 12  = 36  12  = 36

36 A Q(–1, –2)
  3 (3, 0)
12
6. The mid-point of the line joining A (2, p)
1  x
and B (q, 4) is (3, 5). Calculate the   1
numerical values of p and q. 2

Sol. ... (3, 5) is the mid-point of A (2, p) and  –1+x=–2


B (q, 4).  x=–2+1=–1
2q 2  y
 3 2+q=6q=6–2=4 and 3
2 2
281 Arun Deep's Understanding Math-10
 –2+y=6 m1 x 2  m 2 x 1
 y=6+2=8  3 
m1  m 2
 Co-ordinates of P will be (– 1, 8)
Now length of radius OP m1 ( 2 )  m 2 ( 5)
 –3 
m1  m 2
 ( 1  1) 2  ( 8  3) 2

 ( 0) 2  (5) 2  0  25 1 2

A(–5, –4) P(–3, p) B(–2, 3)


 25  5 units Ans.
8. In what ratio does the point (–4, 6) divide 2 m1  5 m 2
 3 
the line segment joining the points A (–6, 10) m1  m 2
and B (3, –8) ?
 – 3 m1 – 3 m2 = – 2 m1 – 5 m2
Sol. Let the point (–4, 6) divides the line segment
joining the points A (–6, 10) and B (3, –8), in  – 3 m1 + 2 m1 = – 5 m2 + 3 m2
the ratio m : n  – m1 = – 2 m2  2 m2 = m1

mx2  nx1 m  3  n(6) m1 2


 –4 = =  m  1  m1 : m2 = 2 : 1
mn mn 2

3m  6n Again
–4 =
mn m1 y 2  m 2 y 1
p
 –4m – 4n = 3m – 6n m1  m 2
 –4n + 6n = 3m + 4m
2 × 3  1 × ( 4 ) 64 2
 7m = 2n   
2 1 3 3
P(–4, 6)
2
A(–6, 10) m:n B(3, –8) Hence p  Ans.
3
m 2 10. In what ratio is the line joining the points
 = (4, 2) and (3, – 5) divided by the x-axis?
n 7
Also find the co-ordinates of the point of
 Required ratio be 2 : 7.
division.
9. Find the ratio in which the point P (– 3, p) Sol. Let the point P which is on x-axis, divides
divides the line segment joining the points the line segment joining the points A (4, 2)
(–5, – 4) and (– 2, 3). Hence find the value and B (3, – 5) in the ratio of m1 : m2.
of p.
and let co-ordinates of P be (x, 0)
Sol. Let P (– 3, p) divides AB in the ratio of
m1 : m2 coordinates of A (– 5, – 4) and m1 y 2  m 2 y 1 m1 ( 5)  m 2 ( 2 )
 0 
B(– 2, 3). m1  m 2 m1  m 2
282 Arun Deep's Understanding Math-10
5 m 1  2 m 2  co-ordinates of P will be (2, 3) Ans.
  0  – 5 m + 2 m = 0 12. Determine the ratio in which the line
m1  m 2 1 2
2x + y – 4 = 0 divide the line segment joining
 – 5 m1 = – 2 m2  5 m1 = 2 m2 the points A (2, –2) and B (3, 7). Also find
m1 2 the co-ordinates of the point of division.
 m  5  m1 : m2 = 2 : 5 Sol. Points are given A(2, –2), B(3, 7) and let the
2
line 2x + y – 4 = 0 divides AB in the ratio
m1 x 2  m 2 x 1 2 ( 3)  5 ( 4 ) m1 : m2 at P and let co-ordinates of P be (x,y),
Again x  
m1  m 2 25 m x  m2 x1 m1  3  m2  2
then x  1 2 
6  20 26 m1  m2 m1  m2
 
7 7
FG 26 , 0IJ Ans. 3m1  2m2
 Co-ordinates of P will be 
H7 K m1  m2
11. If the abscissa of a point P is 2, find the
m1  7  m2 (2) 7 m1  2m2
ratio in which it divides the line segment and y  
joining the points (– 4, 3) and (6, 3). Hence, m1  m2 m1  m2
find the co-ordinates of P.  P lies on the line 2x + y – 4 = 0, then
Sol. Let co-ordinates of A be (–4, 3) and of
2(3m1  2m2 ) 7 m1  2m2
B (6, 3) and of P be (2, y)  40
Let the ratio in which the P divides AB be m1  m2 m1  m2
m1 : m2
 6m1  4m2  7 m1  2m2  4m1  4m2  0
A (4, 3) P (2, y) B (6, 3)  9m1  2 m2  0  9m1  2m2
m1 x2  m2 x1 m1 2
 x=
m1  m2  m  9 or m1 : m2  2 : 9
2
m1  6  m 2   4  2  3  2  9 6  18 24
 2=
m1  m 2  x  
29 11 11
6m1  4m2 2  7  2  9 14  18 4
 2= and y   
m1  m2 29 11 11
 2m1 + 2m2 = 6m1 – 4m2
 24 4 
 6m1 – 2m1 = 2m2 + 4m2  Co-ordinates of P will be  , 
 4m1 = 6m2  11 11 

m1 6 3 Hence P(4, 0) divides AA in the ratio 1 : 1.


 = = 13. ABCD is a parallelogram. If the co-
m2 4 2
ordinates of A, B and D are (10, – 6),
 m1 : m2 = 3 : 2 (2, –6) and (4, – 2) respectively, find the
m1 y2  m2 y1 3 3  2  3 co-ordinates of C.
 y= = Sol. Let the co-ordinates of C be (x, y) and other
m1  m2 3 2
three vertices of the given parallelogram
96 15 are A (10, – 6), B (2, – 6) and D (4, – 2)
= = =3 . . . ABCD is a parallelogram
5 5
283 Arun Deep's Understanding Math-10
Its diagonals bisect each other.  x1 = 2 – 2 = 0
Let AC and BD intersect each other at O. and –8 = –3 + y1  y1 = –8 + 3 = –5
 O is mid-points of BD  Co-ordinates of C are (0, –5)
 Co-ordinates of O will be Again M is mid-point of BD, then
FG 2  4 , 6  2 IJ or FG 6 , 8 IJ or (3, – 4)  1  x2  1  y2
H 2 2 K H2 2 K 1= , –4 =
2 2
Again = O is the mid-point of AC then  –1 + x2 = 2
10  x  x2 = 2 + 1 = 3
3  10 + x = 6
2 and –1 + y2 = –8  y2 = –8 + 1 = –7
 x = 6 – 10 = – 4  co-ordinates of D are (3, –7)
Now, length of AB
6  y
and 4 
2 = 2   12   3   12
 – 6 + y = – 8 y = – 8 + 6
 y=–2 = 2  12   3  12
Hence Co-ordinates of C will be (– 4, – 2).
14. ABCD is a parallelogram whose vertices A = 32   22 = 94 = 13
and B have co-ordinates (2, –3) and (–1, and length of BC
–1) respectively. If the diagonals of the
parallelogram meet at the point M(1, –4), = 0   12   5   12
find the co-ordinates of C and D. Hence,
find the perimeter of the parallelogram. = 12   5  12
Sol. ABCD is a || gm in which co-ordinates of
1   4  =
2
A are (2, –3) and B (–1, –1) = 1 16 = 17
D C  Perimeter of | | gm ABCD = 2 (AB + BC)
=2  13  17  Ans.
15. Given, O, (0, 0), P (1, 2), S (– 3, 0) P
divides OQ in the ratio of 2 : 3 and OPRS
M (1, 4)
is a parallelogram. Find :

B (1, 1)
A (2, 3)

Its diagonals AC and BD bisect each other


at M (1, –4)
 M is mid point of AC and BD
Let co-ordinates of C be (x1, y1) and of D
be (x2, y2) when M is mid point of AC then
2  x1  3  y1
 1= and –4 =
2 2 (i) the co-ordinates of Q.
 2 + x1 = 2 (ii) the co-ordinates of R.
284 Arun Deep's Understanding Math-10
(iii) the ratio in which RQ is divided by y-axis. m1 2×2 4
5  
Sol. (i) Let co-ordinates of Q be (x, y) and of  m1  2 m 2 
2 m 2 5 5
R (x, y).
Point P (1, 2) divides OQ in the ratio of 2 : 3  m1 : m2 = 4 : 5 Ans.
16. If A (5, –1), B (–3, –2) and C (–1, 8) are
m1 x 2  m 2 x 1 2 x   3× 0
 1 m1  m 2
 the vertices of a triangle ABC, find the
23 length of the median through A and the
co-ordinates of the centroid of triangle
2 x  0 5
  1  2 x = 5  x  ABC.
5 2
Sol. A (5, –1), B (–3, –2) and C (–1, 8) are the
m1 y 2  m 2 y 1 2 y   3× 0 vertices of ABC
and 2  
m1  m 2 23 D, E and F are the midpoints of sides BC,
2 y CA and AB
  2  2 y = 10  y = 5 A(5, –1)
5

 Co-ordinaes of Q will be
FG 5 , 5IJ
H2 K F E
... the diagonals of a parallelogram bisect G
each other
 In | | gm OPRS, diagonals OR and PS
bisect each other at M. B(–3, –2) D C(–1, 8)
. . . M is the mid-point of PS

 Co-ordinates of M will be D is the midpoint of BC


 Co-ordinates of D will be

FG 3  1 , 0  2 IJ or FG 2 , 2 IJ or (– 1, 1).
H 2 2 K H 2 2K  x1  x2 x1  x2 
,
 3  1 2  8 
,
. ..   or   or
(ii) M is the mid-point of OR also  2 2   2 2 
0  x 
 1   4 6 
2  ,  or (–2, 3)
 x = – 2  2 2
0  y  G is the centroid
and 1   y = 2
2  Co-ordinates of G will be
 Co-ordinates of R will be (– 2, 2)
(iii) RQ is dividing by y-axis in N  x1  x2  x3 y1  y2  y3 
 , 
Let the ratio in which N divides RQ in m1 : m2  2 3 
. . . N lies on y-axis

 its abscissia (x) = 0  5  3  1 1  2  8  1 5


or  ,  or  , 
5  3 3  3 3
m 2 x   m 2 x  m1 ×  m 2 ( 2 )
0  0 2
m1  m 2 m1  m 2
Length of AD = ( x2  x1 )2  ( y2  y1 )2

5 m1 = (2  5)2  (3  1) 2 = (7)2  (4)2


 2 m2
2 5
  0  m1  2 m 2  0
m1  m 2 2 = 49  16  65 units
12. Equation of a Straight Line
POINTS TO REMEMBER
1. Angle of inclination of a straight line :
The angle which a straight line makes with the positive direction of x-axis measured in anti clockwise
direction is called the inclination or angle of inclination and is generally denoted by .
2. Slope (or gradient) of a straight line :
If  ( 90) is the inclination of a line, then tan  is called its slope or gradient and usually
denoted by m.
Thus m = tan 
3. Intercepts made by a line on the axis i.e. with x-axis, it is called x-intercep t and with y-axis,
it is y-intercept and it is denoted by c.
4. Equation of a stright line :
(i) Equation of a line parallel to x-axis is y = b.
(ii) Equation of a line parallel to y-axis is x = a
(iii) Slope - intercept form : y = mx + c
yc
and here slope ( tan ) = m = 
x
5. Point (x1, y1) = ... Equation of a line passing through y – y1 = m (x – x1)
6. Equation of a straight line passing through two point (x1, y1) and (x2, y2) will be
y2 – y1 = m (x2 – x1)
y  y1
Hence Slope will be (m)  2
x2  x1
Slopes of parallel lines and perpendicular line
1. Slopes of parallel lines
Two lines are parallel if an only if their slopes are equal.
2. Two non vertical lines are perpendicular if and only if the product of their slopes is – 1 i.e.
1 1
m1m2 = – 1 or m1  or m2 
m2 m
1
3. Rule to find the reflection of a point in the line y = x : change the abscissa to ordinate and
ordinate to abscissa.

EXERCISE 12.1
2. Find the inclination of a line whose gradient
1. Find the slope of a line whose inclination is
is
(i) 45 (ii) 30
1
Sol. (i) Here  = 45° (i) 1 (ii) 3 (iii)
3
 Slope of line = tan 45 = 1
Sol. (i) Here m = tan  = 1   = 45
(ii) Here  = 30°
(ii) Here m = tan   3   = 60
1
 Slope of line = tan 30  1
3 (iii) Here m = tan    = 30°
3
285 Arun Deep's Understanding Math-10
286 Arun Deep's Understanding Math-10
3. Find the equation of a straight line parallel 2
to x-axis which is at a distance (ii) y = mx + c  y  x3
7
(i) 2 units above it (ii) 3 units below it.  7 y = – 2 x + 21  2 x + 7 y – 21 = 0
Sol.(i) A line which is parallel to x-axis is 2
[Here m = and c = 3]
y = a and it passes through the point (0, 2) 7
 a=2 (iii) y = mx + c  y  3 x   FG IJ
4
 y=2y–2=0 3H K
(ii) A line which is parallel to x-axis is 4
 y  3 x   3y  3 3 x  4
y = a and it passes through the point (0, –3) 3
 3 3 x  3y  4  0
 a = –3
 y =–3y+3=0 4
[Here m = 3 and c = ]
4. Find the equation of a straight line parallel 3
to y-axis which is at a distance of : (iv) Inclination = 30
(i) 3 units to the right (ii) 2 units to the 1
left.  slope  tan 30 
3
Sol. (i) The equation of line parallel to y-axis is
1
at a distance of 3 units to the right is x = 3  Equation y = mx + c  y  x2
 x–3=0 3
(ii) The equation of line parallel to y-axis at a  3y  x  2 3
distance of 2 units to the left is
 x  3y  2 3  0
x = – 2  x + 2 = 0 Ans.
7. Find the slope and y-intercept of the
5. Find the equation of a straight line parallel
following lines :
to y-axis and passing through the point
(– 3, 5).
(i) x – 2 y – 1 = 0 (ii) 4 x – 5 y– 9 =- 0
Sol. The equation of the line parallel to y-axis is
given by x = a ...(1) x y
(iii) 3 x +5 y + 7 = 0(iv)  1
and it passing through (– 3, 5) 3 4
(v) y – 3 = 0 (vi) x – 3 = 0
 from (1) ;  a = –3
Sol. We know that in the equation
Thus, x = – 3  x + 3 = 0 Ans.
y = mx + c, m is the slope and c is the y-
6. Find the equation of the a line whose intercept.
(i) slope = 3, y-intercept = – 5 (i) Given eqn. be,
2 x–2y–1=0x–1=2y
(ii) slope   , y-intercept = 3
7 1 1
 2y=x–1 y x
4 2 2
(iii) gradient  3 , y-intercept   1 1
3 Here slope  and y-intercept  
(iv) inclination = 30°, y-intercept = 2 2 2
(ii) Given eqn. be,
Sol. Equation of a line whose slope and y- 4x–5y–9=04x–9=5y
intercept is given is y = mx + c where m is 4 9
the slope and c is the y-intercept  5y=4x–9 y x
5 5
(i) y = mx + c  y = 3 x + (–5) 4 9
Here slope  and y-intercept 
 y=3x–5 5 5
[Here m = 3 and c = –5] (iii) 3x+5y+7=0
287 Arun Deep's Understanding Math-10
 5y=–3x–7
3 7
 y x
5 5
3 7
Here slope  and y-intercept 
5 5
x y
(iv)   1  4 x + 3 y = 12
3 4
4 12
 3 y = – 4 x + 12  y  x
3 3
4
 y x  4 on comparing with y = mx + c
3
4 Sol. Slope of the line y = x + 1 after comparing
Here slope  and y-intercept = 4
3 it with y = mx + c, m = 1
(v) y – 3 = 0  y = 3  y = 0. x + 3  tan  = 1
Here slope = 0 and y-intercept = 3   = 45°
(vi) x – 3 = 0 and slope of line y  3x 1
Here in this equation, slope cannot be
defined and does not meet y-axis. m  3  tan   3   = 60°
8. The equation of the line PQ is Now in  formed by the given two lines
3y–3x+7=0 and x-axis.
(i) Write down the slope of the line PQ. Ext. angle = Sum of interior opposite angle.
(ii) Calculate the angle that the line PQ  60° =  + 45°
makes with the positive direction of   = 60° – 45° = 15°.
x = axis. 10. Find the value of p, given that the line
Sol. Equation of line PQ is 3 y – 3 x + 7 = 0 y
 x  p passes through the point
Writing in form of y = mx + c 2
(– 4, 4) (1992).
3x 7
3y=3x–7 y  y
3 3 Sol. Given, equation of line is  x  p.
2
7 . . . It passes through the points (– 4, 4)
 y x
3
 It will satisfy the equation
(i) Here slope = 1
4
(ii)  Angle which makes PQ with x-axis  = – 4 – p 2 = – 4 – p
is Q. 2
But tan  = 1   = 45°.  p=–4–2p=–6
9. The given figure represents the line Hence p = – 6 Ans.
11. Given that (a, 2 a) lies on the line
y = x + 1 and y  3 x  1. Write down the
y
angles which the lines make with the  3 x  6 , find the values of a.
2
positive direction of the x-axis. Hence
Sol. ... the point (a, 2 a) lies on the line
determine .
288 Arun Deep's Understanding Math-10
y y – y1 = m (x – x1)
 3x  6
2 2
 this point will satisfy the equation  y–2  ( x  1)  5 y – 10 = 2 x + 2
5
2a  2 x – 5 y + 2 + 10 = 0
  3 (a )  6  a = 3 a – 6  2 x – 5 y + 12 = 0 Ans.
2
6 15. Find the equation of a straight line whose
–3a+a=–6 –2a=–6a inclination is 60° and which passes through
2
 a = 3 Ans. the point (0, – 3).
12. The graph of the equation y = mx + c passes Sol. The equation of line whose slope is m and
through the points (1, 4) are (– 2, – 5). passes through a given point (0, –3) is
Determine the values of m and c. y – y1 = m (x – x1)
Sol. Given, Equation of the line is y = mx + c Here m = tan 60°  3 and given point is
. . . it passes through the points (1, 4)
(0, – 3)
 4=m×1+c4=m+c  y + 3  3 (x – 0)  y + 3  3 x
 m+c=4 ... (i)
Again it passes through the point (– 2, – 5)  3 x  y  3  0 Ans.
 – 5 = m (– 2) + c  – 5 = – 2 m + c 16. Find the gradient of a line passing through
 2m–c=5 ...(ii) the following pairs of points.
On adding (i) and (ii) ; we get (i) (0, – 2), (3, 4) (ii) (3, – 7), (– 1, 8)
3 m = 9  m = 3 y  y1
Substituting the value of m in (i) ; we get Sol. We know that m  2
x2  x1
3 + c = 4  4 – 3 = 1
Hence m = 3, c = 1 Ans. 42 6
(i) m   2
13. Find the equation of the line passing 3 0 3
through the point (2, – 5) and making an  gradient = 2
intercept of – 3 on the y-axis. 87 15
Sol. ... The line intersects y-axis making an (ii) m  
1  3  4
intercept of – 3
 the co-ordinates of point of intersection will 15
 gradient   Ans.
be (0, – 3) 4
17. The coordinates of two points E and F are
y2  y1
Now slope of the line (m)  (0, 4) and (3, 7) respectively, Find :
x2  x1 (i) The gradient of EF
3  5 2 (ii) The equation of EF
  =–1 (iii) The coordinates of the point where the line
0  2 2
EF intersects the x-axis.
 Equation of the line (using one point form)
Sol. Co-ordintes of points E (0, 4) and F (3, 7)
will be given by
are given, then
y – y1 = m (x – x1)
 y – (– 5) = – 1 (x – 2) y2  y1 74
(i)  gradient (m)  x  x 
 y+5=–x+2x+y+5–2=0 2 1 3 0
 x + y + 3 = 0 Ans.
3
14. Find the equation of a st. line passing  1
2 3
through (– 1, 2) and whose slope is . (ii) Equation of line EF, i.e. line passes through
5
Sol. (using one point form) the point (3, 7) and having slope 1 is given
Equation of the line will be by
289 Arun Deep's Understanding Math-10
y – y1 = m (x – x1)  y – 7 = 1 (x – 3) Sol. A(a, 3), B(2, 1) and C(5, a) are collinear.
 y – 7 = x – 3  x – y – 3 + 7 = 0 if Slope of AB = Slope of BC
 x–y+4=0
(iii) Co-ordinates of point of interrection of EF 1 3 a 1 2 a 1
 =  =
and the x-axis will be y = 0, 2a 52 2a 3
Substitutes the value y in  –6 = (a – 1) (2 – a) (Cross-multipication)
x – y + 4 = 0  x – 0 + 4 = 0 (... y = 0)  –6 = 2a – a2 – 2 + a
 x=–4  –6 = 3a – a2 – 2 a2 – 3a + 2 – 6 = 0
Hence co-ordinates are (– 4, 0) Ans.  a2 – 3a – 4 = 0
18. Find the intercepts made by the line  a2 – 4a + a – 4 = 0
2 x – 3 y + 12 = 0 on the co-ordinate axis.
 a(a – 4) + (a – 4) = 0
Sol. Putting y = 0, we will get the intercept made
on x-axis in,  (a + 1) (a – 4) = 0  a = –1, or a = 4
2 x – 3 y + 12 = 0 Now ; a = –1
 2 x – 3 × 0 + 12 = 0 2 x – 0  12 = 0 ( does not satisfy the equation)
 2 x = – 12  x = – 6  a=4
and putting x = 0, we get the intercepts
a 1 4 1 3
made on y-axis in 2 x – 3 y + 12 = 0  Slope of BC =   =1=m
 2 × 0 – 3 y + 12 = 0 52 3 3
12 Equation of BC ; (y – 1) = 1(x – 2)
– 3 y = – 12  y   4 Ans.
3 y–1=x–2  x–y= –1+2
19. Find the equation of the line passing x – y = 1 Ans.
through the points P (5, 1) and Q (1, – 1).
21. Use a graph paper for this question.
Hence show that the points P, Q and R
(11, 4) are collinear. The graph of a linear equation in x and y, passes
Sol. The two given points are P (5, 17), through A (  1,  1) and B (2, 5). From your
Q (1, –1). graph, find the values of h and k, if the line
y2  y1 passes through (h, 4) and (½, k). (2005)
 Slope of the line (m)  x  x
2 1
Points (h,4) and  , k  lie on the line
1  1 2 1 1
   Sol.
1 5 4 2 2 
Equation of the line, using one point form passing through A(–1,–1) and B(2,5)
is given by
Y
y – y1 = m (x – x1)
1 6
y 1
( x  1)  2 y + 2 = x – 1 B(2,5)
2 5
(h ,4)
4
 x–2y–1–2=0x–2y–3=0 3
...(i) 2 ( 1 ,k)
2
If point R (11, 4) be on it, then it will satisfy 1
X' X
it. Now substituting the value of x and y in –5 –4 –3 –2–1 -11 2 3 4 5 6 7
(i)11 – 2 × 4 – 3 = 11 – 8 – 3 = 11 – 11 = 0 A(–1,–1)

 R satisfies it
Hence P, Q and R are collinear.
20. Find the value of ‘a’ for which the following
points A (a, 3), B (2, 1) and C (5, a) are
collinear. Hence find the equation of the line. Y'
290 Arun Deep's Understanding Math-10

3 y 2  y1
From the graph, we see that h =   Slope of AD = m = x  x
2 2 1

and k = 2 Ans. 5 1 6
= = = –6
22. ABCD is a parallelogram where A(x, 3 4 1
y),  Equation of AD, using one point form,
B (5, 8), C (4, 7) and D (2, –4). Find y – y1 = m (x – x1) A(3, 5)
(i) the coordinates of A y + 1 = –6(x – 4)
(ii) the equation of diagonal BD.
y + 1 = –6x + 24
Sol. (i) y + 6x = –1 + 24
6x + y = 23
D (2,  4) C (4, 7)

B(7, 8) D C(1, –10)

O 24. Find the equation of a line passing through


the point (– 2, 3) and having x-intercept 4
A (x , y ) B (5, 8) units. (2002)
Sol. Given, x-intercept of line = 4
 5  2 8  4  Co-ordinates of the point will be (4, 0)
Coordinates of O =  ,  = (3.5, 2)
2 2  Now slope of the line passing through the
In ||gm, diagonals of a ||gm bisect each other. points
(– 2, 3) and (4, 0)
x4 y7
 3.5 = and 2 = y2  y1 0  3 3 1
2 2  Slope (m)  x  x  4  2  6  
 x+4=7  y+7=4 2 1 2
 x=7–4=3  y = 4 – 7 = –3  Equation of the line using one point form
x = 3, y = –3 will be given by
y – y1 = m (x – x1)
Thus, the coordinates of A are (3, –3)
1
(ii) Equation of diagonal BD is given by  y – 0   ( x  4)  2 y = – x + 4
2
4  8  x + 2 y = 4 or x + 2 y – 4 = 0 Ans.
y–8 = (x – 5)
25 25. Find the equation of the line whose x-inter-
12 cept is 6 and y-intercept is –4.
 y–8= (x – 5) Sol. Given, x-intercept = 6
3
 The line will pass through the point (6, 0)
 y – 8 = 4x – 20  4x – y – 12 = 0
y-intercept = –4  c = –4
23. In ABC, A (3, 5), B (7, 8) and C (1, –10).  The line will pass through the point (0, –4).
Find the equation of the median through A.
y2  y1 4  0 4 2
Sol. Clearly AD is median Now m = x  x  0  6  6  3
2 1
 D is mid point of BC
2
 7  1 8  10   Equation of line having slope and
 D be  ,  3
 2 2 
y-intercept –4 given by
i.e. (4, –1) y = mx + c
291 Arun Deep's Understanding Math-10

2 2  y–x=3
 y= x + (–4) = x – 4  y = x + 3 Ans.
3 3
 3y = 2x – 12 27. Find the equation of the line passing
through the point (1, 4) and intersecting
 2x – 3y = 12
the line x – 2 y – 11 = 0 on the y-axis.
26. A(2, 5), B(–1, 2) and C(5, 8) are the
Sol. .. line x – 2 y – 11 = 0 passes through
.
vertices of a triangle ABC, ‘M’ is a point
y-axis
on AB such that AM : MB = 1 : 2. Find the
co-ordinates of ‘M’. Hence find the  x = 0,
equation of the line passing through the Now substituting the value of x in the
points C and M. (2018) equation x – 2 y – 11 = 0, we have
Sol. 11
 – 2 y – 11 = 0  – 2 y = 11  y  
2

FG
 Co-ordinates of point will be 0,  11 IJ
H2 K
Now slope of the line joining the points
FG
(1, 4) and 0, 
11 IJ
H 2 K
y  y1  11  4  19
AM : MB = 1 : 2 m 2 2 2 19
x2  x1  0  1  1  2
m2 x1  m1 x2
 x= and equation of the line using one point
m1  m2
form will be given by
2  2  1  (1) 3 y – y1 = m (x – x1)
 x=  1
1 2 3 11 19
 y  ( x  0)
2 2
m2 y1  m1 y2
y=  2 y + 11 = 19 x
m1  m2
 19 x – 2 y – 11 = 0 Ans.
2  5  1 2 12 28. Find the equation of the straight line
 y=  4 containing the point (3, 2) and making
1 2 3
positive equal intercepts on axes.
 coordinates of M are (1, 4). Sol. Let the line containing the point P (3, 2)
For line passing through C(5, 8) and M(1, 4). passes through x-axis at A (x, 0) and y-
axis at B (0, y)
y2  y1 4  8  4
m=   1 ... OA = OB (given)
x2  x1 1  5  4
 x=y
 y – y1 = m(x – x1) [using one point form] y2  y1
Now slope of the line (m)  x  x
 y – 8 = 1(x – 5) 2 1
 y–8=x–5 0  y x
  . .
= – 1 ( . x = y)
x0 x
292 Arun Deep's Understanding Math-10
 Equation of the line will be [using one point D (x , y ) C (3, 2)
form]

A (3, 6) B (5, 10)

 5 + x = 6 and 10 + y = 8
 x = 1 and y = –2
 Coordinate of D be (1, –2)
(ii) Length of diagonal BD
y – y1 = m (x – x1)  y – 2 = – 1 (x – 3) = (1  5) 2  (2  10) 2 = (4) 2  (12) 2
 y–2=–x+3x+y–2–3=0
 x + y – 5 = 0  x + y = 5 Ans. = 16  144 = 160 units = 4 10 units
29. Three vertices of a parallelogram ABCD (iii) Equation of the side joining A (3, 6) and
taken in order are A (3, 6), B (5, 10) D (1, –2) is given by
and C(3, 2).
x3 y6
(i) the coordinates of the fourth vertex D. =
3 1 62
(ii) length of diagonal BD.
(iii) equation of side AD of the parallelogram x3 y6
 =
ABCD. (2015) 2 8
Sol. Three vertices of a parallelogram ABCD taken  4(x – 3) = y – 6  4x – 12 = y – 6
in order are A (3, 6), B (5, 10) and C (3, 2)  4x – y = 6
(i) We need to find the co-ordinates of D Thus, the equation of the side joining A (3, 6)
We know that the diagonals of a and D (1, –2) is 4x – y = 6
parallelogram bisect each other 30. A and B are two points on the x-axis and y-
axis respectively. P (2, –3) is the mid point
Let (x, y) be the co-ordinates of D
of AB. Find the
33 6 2
 Mid-point of diagonal AC =  , 
 2 2 
i.e. (3, 4)
and, mid-point of diagonal
 5  x 10  y  A
BD =  ,  O
 2 2 
P (2, 3)
Thus, we have B
5 x 10  y
= 3 and =4
2 2
293 Arun Deep's Understanding Math-10
(i) the co-ordinates of A and B.
(ii) the slope of line AB.
(iii) the equation of line AB. (2010)
Sol. Point A and B are on x-axis and y-axis N(0, y)
respectively
Let co-ordinates of A be (x, 0) and of B be
P(3, 2)
(0, y)
P(2, –3) is the mid point of AB
M(x, 0)
x0 0 y
Then, 2 = and –3 =
2 2
 x = 4, y = –6
2  0  3 x 2  y  3 0
(i) Hence co-ordinates of A are (4, 0) and of B P (3, 2) = ,
are (0, –6) 23 23
3x 2 y
y2  y1 6  0 6 3  ,
(ii)  Slope of AB = x  y =   5 5
2 1 0  4 4 2 On comparing, we get
(iii) Equation of AB will be y – y1 = m(x – x1) 3x
3 3  5 15
3   x  5
 y – (–3) = (x – 2) ( P lies on it) 5 3 3
2
2y 2  5 10
3  2  y  5
 y+3= (x – 2)  2y + 6 = 3x – 6 5 2 2
2
(i) M(x, 0) = (5, 0), N(0, y) = (0, 5)
 3x – 2y = 6 + 6  3x – 2y = 12
y2  y1 50 5
(ii) Slope of MN = x  x =   1
2 1 0  5 5
32. The line through P(5, 3) intersects y-axis
at Q.
(i) Write the slope of the line.
A (ii) Write the equation of the line.
O
(iii) Find the coordinates of Q.
P (2, 3)
B Y
P(5, 3)

31. M and N are two points on the x-axis and


45º
y-axis respectively. P(3, 2) divides the line X X
O
segment MN in the ratio 2 : 3. Find :
(i) the coordinates of M and N Q
(ii) slope of the line MN.
Sol.
2:3 Y

M P(3, 2) N Sol. (i) Here  = 45º


(x, 0) (0, y) So, slope of the line = tan  = tan 45º = 1
294 Arun Deep's Understanding Math-10
(ii) Equation of the line through P and Q is 34. Find the equations of the diagonals of a
y – 3 = 1 (x – 5)  y – x + 2 = 0 rectangle whose sides are
(iii) Let the coordinates of Q be (0, y) x = – 1, x = 2, y = – 2, y = 6.
y 2  y1
Then m = x  x
2 1
3 y
 1=
50
 5 = 3 – y  y = –2
So, coordinates of Q are (0, –2)
33. (i) Write down the coordinates of the point
P that divides the line joining A(–4, 1) and
B(17, 10) in the ratio 1 : 2.
(ii) Calculate the distance OP, where O is the
origin.
(iii) In what ratio does the y-axis divide the line
AB ? Sol. The equations of sides of a rectangle whose
equations are x = – 1, x = 2, y = – 2, y = 6.
Sol. (i) Let co-ordinate of P be (x, y) which
These lines form a rectangle when they
divides the line segment joining the points A
(– 4, 1) and B (17, 10) in the ratio of 1 : 2. intersect at A, B, C, D respectively
 Co-ordinates of A, B, C and D will be
m1 x2  m2 x1
 x (–1, –2), (2, –2), (2, 6) and (–1, 6)
m1  m2 respectively.
1× 17  2 ×( 4 ) 17  8 9 AC and BD are its diagonals.
   3
1 2 3 3 (i) Slope of the diagonal AC
m y  m2 y1 1× 10  2 × 1 10  2
y 1 2   y  y1 6  2 8
m1  m2  2 
1 2 3 x2  x1 2 1  3
12  Equation of AC will be
 4
3
8
 Co-ordinates of P are (3, 4) y – y1 = m (x – x1) = y + 2  (x + 1)
3
(ii) Distance of OP where O is the origin i.e.  3y+6=8x+8
co-ordinates are (0, 0)  8 x – 3 y + 8 – 6 = 0  8 x – 3 y + 2 = 0
 Distance  ( 3  0 ) 2  ( 4  0) 2 y2  y1
(ii) Slope of BD 
 32  4 2  9  16  25  5 units x2  x2
(iii) Let y-axis divides AB in the ratio of m1 : m2 62 8
 
at P and let co-ordinates of P be (0, y) 1  2 3
m x  m2 x1 m1 ×17  m2 ×( 4 )  Equation BD will be
0 1 2   y – y1 = m (x – x1)
m1  m2 m1  m2
 17 m1 – 4 m2 = 0  17 m1 = 4 m2 8
 y+2  ( x  2)
3
m1 4
 m  17 m1: m2 = 4 : 17  3 y + 6 = – 8 x + 16
2  8 x + 3 y + 6 – 16 = 0  8 x + 3 y – 10 = 0
295 Arun Deep's Understanding Math-10
35. Find the equation of a straight line passing  These line are neither parallel nor perpendicular.
through the origin and through the point of 2. If 6 x + 5 y – 7 = 0 and 2 px + 5 y + 1 = 0
intersection of the lines 5 x + 7 y = 3 and are parallel lines, find the value of p.
2 x – 3 y = 7. Sol. In equation
Sol. Given lines are ; 6x+5y–7=0 5y=–6x+7
5x+7y=3 ...(i) 6 2
 y x
2x–3y=7 ...(ii) 5 5
Multiply (i) by 3 and (ii) by 7, we have 6
 Slope (m)   ...(i)
15 x + 21 y = 9 5
14 x – 21 y = 49 Again given, equation 2 px + 5 y + 1 = 0
On adding we get, 2 1
 5 y = – 2 px – 1  y px 
58 5 5
29 x = 58  x  2
29 2
 slope ( m)   p ...(ii)
Substituting the value of x in (i) ; we get 5
5 × 2 + 7 y = 3  10 + 7 y = 3 . . . lines are parallel  slopes are equal.

 7 y = 3 – 10 = – 7  y = – 1  m1 = m2
 Point of intersection of lines be (2, –1). 6 2p
Now slope of the line joining the points from (i) and (ii) ;  
5 5
(2, – 1) and the origin (0, 0)
6 5FG IJ
y  y1 0  1 1  p ×
H K  3 Ans.
m 2 5 2
x2  x1  0  2  2 3. Lines 2 x – by + 5 = 0 and ax + 3 y = 2 are
Equation of line using one point form given parallel, find the relation connecting a and
by b. (1991)
1 Sol. Given equation of line be,
y – y1 = m (x – x1)  y  0   ( x  0)
2 2 x – by + 5 = 0 ...(i)
 2 y = – x  x + 2 y = 0. Ans. 2 5
 – by = – 2 x – 5  y  
b b
EXERCISE 12.2 2
 Slope (m) of line (1) 
1. State which one of the following is true : b
and second equation of line be ax + 3 y = 2
The straight lines y = 3 x – 5 and
a 2
2 y = 4 x + 7 are :  3 y = – ax + 2  y   x 
(i) parallel (ii) perpendicular 3 3
(iii) neither parallel nor perpendicular. a
 slope (m2) of line (2)  
Sol. Slope of line y = 3 x – 5 = 3 3
... Lines are parallel.
and slope line 2 y = 4 x + 7
7 2 a
 y  2x = 2.  m1 = m2  
2 b 3
Since slope of line (ax + by + c = 0)  – ab = 6  ab = – 6 Ans.
4. If the straight lines 3x – 5y = 7 and
coeff. of x 4x + ay + 9 = 0 are perpendicular to one
=–
coeff.of y another, find the value of a. (2018)
... Slope of both the lines are neither equal Sol. Given lines are ; 3x – 5y = 7
nor their product is – 1. and 4x + ay + 9 = 0
296 Arun Deep's Understanding Math-10
Now 3x – 7 = 5y ay = –4x – 9 Slope of line 3x = y + 1 i.e.y = 3x – 1 be 3

3x 7 4x 9 1
 
... m1 × m2  
× 3  1
 y= y=– 3
5 5 a a
 These lines are perpendicular to each other
3 4 (ii) Co-ordinates of mid point of line joining
 m1 =  m2 = the points (– 2, 3) and (4, 1) will be
5 a
Since lines are perpendicular, FG 2  4 , 3  1IJ or FG 2 , 4 IJ or (1, 2)
 m1 × m2 = –1 H 2 2 K H 2 2K
3 4  12 If mid-point (1, 2) lies on the line
   1   1 3 x = y + 1 then it will satisfy it
5 a 5a
 –12 = –5a Now substituting the value of x and y in
3 x = y + 1  3 (1) = 2 + 1
12 2
 a=  2  2.4 Ans.  3 = 3 which is true
5 5
Hence the line 3 x = y + 1 bisects the line
5. If the lines 3 x + by + 5 = 0 and
joining the points (– 2, 3) and (4, 1). Ans.
ax – 5y + 7 = 0 are perpendicular to each
other, find the relation connecting a and b. 7. The line through A (–2, 3) and B (4, b) is
perpendicular to the line 2x – 4y = 5. Find the
Sol. In given equation 3 x + by + 5 = 0
value of b. (2012)
3 5
 by = – 3 x – 5  y   ×  Sol. Gradient (m1) of the line passing through
b 3
the points A (–2, 3) and B (4, b)
3
 Slope ( m1 ) 
b b3 b3
and in second equation ax – 5 y + 7 = 0 = =
42 6
a 7
 5 y = ax + 7  y   Gradient (m2) of the line 2x – 4y = 5
5 5
a x 5 1
 Slope ( m2 )  or y = – is .
5 2 2 2
. . . Lines are perpendicular to each other.
Since, the lines are perpendicular to each
3 a
 m 1m 2 = – 1  ×  1 other.
b 5  m1 × m2 = –1
3 a
 5 b  1  – 3 a = – 5 b b3 1
 × = –1  b – 3 = –12  b = –9
6 2
 3 a = 5 b Ans.
6. Is the line through (– 2, 3) and (4, 1) 8. If the lines 3 x + y = 4, x – a y + 7 = 0 and
perpendicular to the line 3 x = y + 1 ? bx + 2 y + 5 = 0 for the three consecutive
sides of a rectangle, find the value of
Does the line 3 x = y + 1 bisect the join of
a and b.
(– 2, 3) and (4, 1). (1993)
Sol. In the line 3 x + y = 4 ...(i)
Sol. Slope of the line passing through the points
 y=–3x+4
y2  y1  Slope (m1) = – 3
(– 2, 3) and (4, 1)  x  x
2 1 In the given line x – ay + 7 = 0 ...(ii)
1  3 2 1  ay = x + 7  y 
1
x
3
  
42 6 3 a a
297 Arun Deep's Understanding Math-10
1 and with y-intercept – 3 units.
 Slope ( m2 )  Sol. In the line 2 x + 5 y + 7 = 0
a
and in the 3rd given line bx + 2 y + 5 = 0 2 7
 5y=–2x–7 y x
...(iii) 5 5
b 5 2
 2 y = – bx – 5  y   x  Here slope ( m1 )  
2 2 5
b Let the slope of the line perpendicular to
 Slope ( m3 )  
2 the given line = m2
... These are the consecutive three sides of a 2
rectangle.  m1m2 = – 1   m2  1
5
 (i) and (ii) are perpendicular to each other
5 5
1  m2  1 × 
 m 1m 2 = – 1   3 ×  1 2 2
a . . . It makes y-intercept – 3 units
 –3=–aa=3  The line passes through the point (0, – 3).
and (i) and (iii) are parallel to each other.  Equation of the new line using one point
b form is given by
 m1 = m3   3 
2 y – y1 = m (x – x1)
 –b=–6b=6
5 5
Hence a = 3, b = 6 Ans.  y  ( 3)  ( x  0)  y  3  x
2 2
9. Find the equation of a line, which has the
y-intercept 4, and is parallel to the line  2 y + 6 = 5 x  5 x – 2 y – 6 = 0
2 x – 3 y – 7 = 0. Find the coordinates of 11. Find the equation of a st. line perpendicular
the point where it cuts the x-axis. (1998) to the line 3 x – 4 y + 12 = 0 and having
Sol. In the given line 2 x – 3 y – 7 = 0 same y-intercept as 2 x – y + 5 = 0.
Sol. In the given line 3 x – 4 y + 12 = 0
2 7
 3y=2x–7 y x 3
3 3  4 y = 3 x + 12  y  x  3
4
2
Hence slope ( m1 )  3
3 Here slope ( m1 ) 
4
 Equation of the line parallel to the given
Let the slope of the line perpendicular to
line will be
the given line be = m2
y – y1 = m (x – x1)
. . . it passes through (0, 4), then 3
 m 1m 2 = – 1  m  1
4 2
2
y–4  ( x  0)  3 y – 12 = 2 x 4
3  m2  
 2 x – 3 y + 12 = 0 ...(ii) 3
Now let it intersect x-axis at (x, 0) For y-intercept of the given equation
 y=0 2 x – y + 5 = 0 ; we have x = 0
Substituting the value of y in (ii) ; we get  2×0–y+5=0y=5
 The equation of the required line passing
2 x – 3 × 0 + 12 = 0  2 x = – 12
through (0, 5) will be given by
 x=–6
Thus, the coordinates of required point be 4
y – y1 = m (x – x1)  y  5  ( x  0)
(–6, 0). 3
10. Find the equation of a straight line  3 y – 15 = – 4 x
perpendicular to the line 2 x + 5 y + 7 = 0  4 x + 3 y – 15 = 0 Ans.
298 Arun Deep's Understanding Math-10
12. Find the equation of the line passing the given line be = m2
through (0, 4) and parallel to the line 3
 m 1m 2 = – 1  × m2  1
3 x + 5 y + 15 = 0. (1999) 8
Sol. In the given equation 3 x + 5 y + 15 = 0 8
m2 
3 3
 5 y = – 3 x – 15  y  x3 8
5  Equation of the line where slope is and
3
3 passes through the point (– 1, – 2) will be
How slope ( m1 )  
5 y – y1 = m (x – x1)
 Slope of the line parallel to the given line 8
 y – (– 2)  [ x  ( 1)]
3 3
 and passes through the point 8
5  y  2  ( x  1)
(0, 4) 3
 3y+6=8x+8
 Equation of the required line will be given by
 8x–3y+8–6=0
y – y1 = m (x – x1)
 8 x – 3 y + 2 = 0 Ans.
3 14. (i) The line 4 x – 3 y + 12 = 0 meets x-axis
 y–4  ( x  0)
5 at A. Wrie down the co-ordinates of A.
 5 y – 20 = – 3 x (ii) Detrmine the equation of the line passing
 3 x + 5 y – 20 = 0 Ans. through A and perpendicular to
P.Q. The equation of a line is y = 3 x – 5. Write 4 x – 3 y + 12 = 0. (1993)
down the slope of this line and the intercept Sol. (i) In the line 4 x – 3 y + 12 = 0 ...(i)
made by it on the y-axis. Hence or 4
otherwise, write down the equation of a 3 y = 4 x + 12  y  x  4
3
line which is parallel to the line and which 4
passes through the point (0, 5). Here slope ( m1 ) 
3
Sol. In the given line y = 3 x – 5 Let the slope of the line perpendicular to
Here slope (m1) = 3 the given line be = m2
For y-intercept ; 4
Substituting x = 0, we have y = – 5  m 1m 2 = – 1  × m2  1
3
 y-intercept = – 5 3
 m2  
The slope of the line parallel to the given 4
line will be 3 and passes through the point Let the point on x-axis be A (x, 0)
(0, 5).  Putting y = 0 in eqn. (i) ; we have
 Equation of the line will be given by 4 x – 3 × 0 + 12 = 0  4 x + 12 = 0
y – y1 = m (x – x1)  4 x = – 12  x = – 3
 y – 5 = 3 (x – 0)  y – 5 = 3 x  Co-ordinates of A will be (– 3, 0).
 3 x – y + 5 = 0  y = 3 x + 5 Ans. (ii) Equation of the line perpendicular to the
13. Write down the equation of the line given line passing through A will be.
perpendicular to 3 x + 8 y = 12 and passing 3
through the point (– 1, – 2). y – y1 = m (x – x1)  y – 0  ( x  3)
4
Sol. In the given line 3 x + 8 y = 12  4y=–3x–9
3 12  3 x + 4 y + 9 = 0 Ans.
 8 y = – 3 x + 12  y  x
8 8 15. Find the equation of the line that is parallel
3 to 2 x + 5 y – 7 = 0 and passes through the
Here slope ( m1 ) 
8 mid-point of the line segment joining the
Let the slope of the line perpendicular to
points (2, 7) and (– 4, 1).
299 Arun Deep's Understanding Math-10
Sol. The given line 2 x + 5 y – 7 = 0  2 x – 3 y – 7 = 0 Ans.
2 2 17. Find the equation of a straight line passing
 5y=–2x+7 y x
5 5 through the intersection of 2 x + 5 y – 4 = 0
2 with x-axis and parallel to the line
Here slope ( m1 )  
5 3 x – 7 y + 8 = 0.
 Slope of the line parallel to the given line Sol. Let the point of intersection of the line
2 2 x + 5 y – 4 = 0 and x-axis be (x, 0)
will be  .
5 Substituting the value of y in the equation
Co-ordinates of the mid point joining the 2 x + 5 × 0 – 4 = 0  2 x – 4 = 0
points (2, 7) and (– 4, 1) will be  2x=4
FG 2  4 , 7  1IJ or FG 2 , 8 IJ or (– 1, 4)  x 2
4
H 2 2 K H 2 2K 2
 Equation of the line using one point form  Co-ordinates of the point of intersection
will be given by will be (2, 0)
2 Now in the line 3 x – 7 y + 8 = 0
y – y1 = m (x – x1)  y  4  ( x  1)
5 3 8
 5 y – 20 = – 2 x – 2  7y=3x+8 y x
7 7
 2 x + 5 y – 20 + 2 = 0 3
 2 x + 5 y – 18 = 0 Ans. Slope ( m1 ) 
7
16. Find the equation of the line that is and the slope of the line parallel to the above
perpendicular to 3 x + 2 y – 8 = 0 and
3
passes through the mid point of the line line will be 
7
segment joining the points (5, – 2), (2, 2).
 Equation of the line passes through (2, 0)
Sol. In the given line 3 x + 2 y – 8 = 0
3 3
 2y=–3x+8 y x4 and having slope
7
will be given by
2
3
Here slope ( m1 )  3
2 y – y1 = m (x – x1)  y  0  ( x  2)
Co-ordinates of the mid point of the line 7
segment joining the points (5, – 2) and 7 y = 3 x – 6  3 x – 7 y – 6 = 0 Ans.
(2, 2) will be 18. The equation of a line is 3x + 4y – 7 = 0. Find :
(i) the slope of the line.
FG 5  2 , 2  2 IJ or FG 7 , 0IJ (ii) the equation of a line perpendicular to the
H 2 2 K H2 K given line and passing through the
and let the slope of the line perpendicular intersection of the lines x – y + 2 = 0 and
to the given line be = m2
3x + y – 10 = 0.
3
 m 1m 2 = – 1  m  1 Sol. (i) Given equation of the line be
2 2
2 3x + 4y – 7 = 0
 m2 
3  4y = 7 – 3x
 Equations of the line perpendicular to the
7 FG IJ 3 7
given line and passing through , 0 will  y= x
2 H K 4 4
be.
y – y1 = m (x – x1) [using one point form] Comparing it with y = mx + c

 y0
FG
2 x7 IJ
3y=2x–7  m=
3
3 H 2 K 4
300 Arun Deep's Understanding Math-10

3 3
 Slope of the line = y – y1 = m (x – x1)  y  2  ( x  1)
4 4
 4y+8 =–3x+3
(ii) Slope of the line perpendicular to the given  3x+4y+8–3=0 3x+4y+5=0
1  4  4 For finding the co-ordinates of the foot of
line will be m1 =      . the perpendicular we have to solve the
m  3  3
equations
Now given lines be, 4x–3y–5=0 ...(i)
x–y+2=0 ....(i) and 3 x + 4 y + 5 = 0 ...(ii)
Multiplying (i) by 4 and (ii) by 3, we get
3x + y – 10 = 0 ....(ii)
16 x – 12 y – 20 = 0
On adding both we get, 9 x + 12 y + 15 = 0
On adding we get
8
4x – 8 = 04x = 8 x = =2 25 x – 5 = 0  25 x = 5
4
5 1
From (i), we have  x 
25 5
2–y+2=0  4–y=0 y=4 Substituting the value of x in (i) ; we get
 Point of intersection of the two line be (2, 4) FG 1 IJ  3 y  5  0  4  3 y  5  0

Now equation of the line perpendicular to H 5K 5
the given line and passing through (2, 4) will 4 4  25 21
be y – y1 = m1 (x – x1)  3y  5  
5 5 5
4 21 7
 y–4= (x – 2)  3y – 12 = 4x – 8  y 
3 5 3 5
 4x – 3y + 8 + 12 = 0
 Co-ordinates of foot of  be
FG 1 , 7 IJ
 4x – 3y + 4 = 0 H5 5 K
19. Find the equation of the line perpendicular Ans.
from the point (1, – 2) on the line 20. Prove that the line through (0, 0) and
4 x – 3 y – 5 = 0. Also find the co-ordinates (2, 3) is parallel to the line through (2, –2)
of the foot of the perpendicular. and (6, 4).
Sol. In the equation 4 x – 3 y – 5 = 0, Sol. Slope of the line through (0, 0) and (2, 3)
4 5 y2  y1 3  0 3
 3y=4x–5 y
3
x
3 (m1) = x  x  2  0  2
2 1
4 and slope of the line through (2, –2) and
Slope ( m1 ) 
3 (6, 4)
Let the slope of the perpendicular = m2 42 6 3
(m2) =  
4 62 4 2
Since, m1 × m2 = – 1  × m2  1
3
3 3
 m2    m1 = m2 =
4 2
 Equation of the perpendicular whose slope  The lines are parallel to each other Ans.
3 21. Prove that the line through (– 2, 6) and
is  and drawn through the point (1, – 2).
4 (4, 8) is perpendicular to the line through
301 Arun Deep's Understanding Math-10
(8, 12) and (4, 24). and equation of the line using one point
Sol. Slope of the line through (– 2, 6) and form is given by
(4, 8) y – y1 = m (x – x1)
y  y1 8  6 2 1 7
( m1 )  2  y  3  ( x  1)
x2  x1  4  2  6  3 4
and slope of the line through (8, 12) and  4 y – 12 = 7 x + 7
(4, 24)  7 x – 4 y + 7 + 12 = 0
24  12 12  7 x – 4 y + 19 = 0 Ans.
and ( m2 )    3
48 4 24. A (–1, 3), B (4, 2), C (3, –2) are the vertices of a
1 triangle.
Here m1 × m2  × ( 3)  1
3 (i) Find the coordinates of the centroid G of
 Lines are perpendicular to each other. the triangle.
22. Show that the triangle formed by the points (ii) Find the equation of the line through G and
A (1, 3), B (3, – 1) and C (– 5, – 5) is a parallel to AC (2017)
right angled triangle by using slopes.
Sol. Given, A (–1, 3), B (4, 2), C (3, –2)
Sol. Slope (m1) of line by joining the points
(i) Coordinates of centroid G
y2  y1
A (1, 3) and B (3, – 1)  x  x  x1  x 2  x3 y1  y 2  y3 
=  
2 1 ,
1  3 4  3 3 
 m1    2
31 2
Slope (m2) of the line joinig the points  1 4  3 3  2  2   6 3
=  ,  = ,  =(2 , 1)
y2  y1  3 3  3 3
B (3, – 1) and C (– 5, – 5) 
x2  x1 So, the coordinates of centroid are (2, 1).
5  1 4 1
 m2    y 2  y1 2  3 5
5  3 8 2
(ii) Slope of AC = x  x = 3  (1) =
1 2 1 4
Here, m1 × m2  2 ×  1
2
5
 Lines AB and BC are perpendicular to each  Slope of the required line (m) =
4
other.
Let the equation of the line through G, be
Hence  ABC is a right angled triangle.
given by
Ans.
y – y1 = m(x – x1)
23. Find the equation of the line through the
point (– 1, 3) and parallel to the line joining 5
the points (0, – 2) and (4, 5).  y–1=  (x – 2)
4
Sol. Slope of the line joining the points (0, – 2)  4y – 4 = –5x + 10
y2  y1 5 2 7  5x + 4y – 14 = 0 which is the required line.
and (4, 5)  m1  x  x  
2 1 40 4 P.Q. The line through P (5, 3) intersects y-axis at Q.
 Slope of the line parallel to it passing (i) Write the slope of the line.
(ii) Write the equation of the line.
7
through (– 1, 3)  (iii) Find the coordinates of Q.
4
302 Arun Deep's Understanding Math-10
 m 1m 2 = – 1
Y
1
 × m2  1
12
( 12 )
 m2  1 ×  12
P (5, 3) 1
45°  Equation of the line passing through (0, – 3)
X X and having slope m2 = 12 is given by
O
y – y1 = m (x – x1)
 y + 3 = 12 (x – 0)
Y
 y + 3 = 12 x
Sol.  12 x – y – 3 = 0 Ans.
(i) Here  = 45° 26. The vertices of a ABC are A(3, 8),
B(–l, 2) and C(6, –6). Find :
So, slope of the line = tan  = tan 45° = 1
(ii) Equation of the line through P and Q is (i) Slope of BC.
y – 3 = 1 (x – 5) (ii) Equation of a line perpendicular to BC
 y–x+2=0 and passing through A.

(iii) Let the coordinates of Q be (0, y) Sol. (i) Let B(–1, 2) = (x1, y2)
and C(6, –6) = (x2, y2)
y2  y1
Then m = x  x
2 1 y2  y1 6  2 8
Slope of BC =  
x2  x1 6  1 7
3 y
 1= (ii) AE is perpendicular to BC and passing
50
through A
 5 = 3 – y  y = –2 7
 For AE, slope =
So, required coordinates of Q are (0, –2) 8
25. Find the equation of the line through 8
[ Slope of BC =  slope of its
(0, – 3) and perpendicular to the line joining 7
the points (– 3, 2) and (9, 1). 7
perpendicular AE = ]
8
Sol. The slope (m1) of the line joining the points
and A(x1, y1) = (3, 8)
(– 3, 2) and
Equation of AE is :
y2  y1 1  2 1
(9, 1)  x  x   y – y1 = m(x – x1)
2 1 9  3 12
7
Let slope of the line perpendicular to the  y–8= (x – 3)
8
line = m2
 8y – 64 = 7x – 21 i.e. 7x – 8y + 43 = 0 Ans.
303 Arun Deep's Understanding Math-10
(i) the median of the triangle through A
A(3, 8) (ii) the altitude of the triangle through B
Sol. (i) D is the mid-point of BC
 Co-ordinates of D will be

FG 3  1 , 3  5IJ or FG 2 , 8 IJ or (1, 4)
B (–1, 2) E C ( 6 , 6 )
H 2 2 K H 2 2K
27. The vertices of a triangle are A (10, 4),
B (4, – 9) and C (– 2, – 1). Find the
equation of the altitude through A.
The perpendicular drawn from a vertex of
a triangle to the opposite is called altitude.
Sol. Vertices of  ABC are A (10, 4), B (4, – 9)
and C (– 2, – 1) y  y1
 Slope of median AD ( m)  2
x2  x1
y2  y1
Slope of the line BC ( m1 )  x  x 44 8
   8
2 1 1 2 1
1  9 8 4 Then equation of AD will be
   y – y1 = m (x – x1)  y – 4 = – 8 (x – 1)
2  4 6 3
Let the slope of the altitude from A (10, 4)  y – 4 = – 8 x + 8  8 x + y – 4 – 8 = 0
to BC = m2  8 x + y – 12 = 0
(ii) BE is the altitude from B to AC
4
 m 1m 2 = – 1  × m2  1 y2  y1
3 5 4
 Slope of AC ( m1 )  x  x 
2 1 1  2
 m2  1  FG 3 IJ  3 9
H 4K 4 
3
 3
 Equation of the line through A(10, 4) and Let slope of BE = m2
3 But m1m2 = – 1  – 3 × m2 = – 1
having slope will be given by
4 1 1
m2  
y – y1 = m (x – x1) 3 3
 Equation of altitude BE will be given by
3
 y–4= (x – 10) y – y 1 = m (x – x1)
4
1
 4 y – 16 = 3 x – 30 i.e. y – 3 
(x – 3)
3
 3 x – 4 y + 16 – 30 = 0
 3 y – 9 = x – 3 x – 3y – 3 + 9 = 0
 3 x – 4 y – 14 = 0 Ans.
 x – 3 y + 6 = 0 Ans.
28. A (2, – 4), B (3, 3) and C (– 1, 5) are the
29. Find the equation of the right bisectors of
vertices of triangle ABC. Find the equation
the line segment joining the points (1, 2)
of
304 Arun Deep's Understanding Math-10
and (5, – 6).
1
Sol. Slope of the line joining the points (1, 2)  Slope of PQ = (m1 m2 = – 1)
2
and (5, – 6)
 Equation of PQ be given by
y  y1 6  2 8
m1  2 y – y1 = m (x – x1)
x2  x1  5  1  4  2
1
Let m2 be the right bisector of the line  y–3= (x – 4)
2
 m 1m 2 = – 1  – 2 × m 2 = – 1
1 1  2y – 6 = x – 4
m2    x – 2y + 6 – 4 = 0
2 2
mid point of the line segment joining (1, 2)  x – 2y + 2 = 0 ...(i)
and (5, – 6) will be ..
(iii) . Given point (–2, p) lies on eqn. (i) ; we
FG 1  5 , 2  6 IJ or FG 6 , 4 IJ or (3, – 2) have
H 2 2 K H2 2 K – 2 – 2p + 2 = 0
 Equation of line, the right bisector through  –2p + 0 = 0
(3, –2) will be given by
 –2p = 0
y – y1 = m (x – x1)
 p = 0 Ans.
1
 y + 2  ( x  3) 31. The points B (1, 3) and D (6, 8) are two
2
opposite vertices of a square ABCD. Find
 2y+4=x–3x–2y–3–4=0 the equation of the diagonal AC.
 x – 2 y – 7 = 0 Ans.
y2  y1
30. Points A and B have coordinates (7, –3) and Sol. Slope of BD ( m1 )  x  x
(1, 9) respectively. Find 2 1
83 5
(i) the slope of AB.   1
61 5
(ii) the equation of the perpendicular bisector . . . Diagonal AC is perpendicular bisector of
of the line segment AB. diagonal BD
(iii) the value of ‘p’ if (–2, p) lies on it.  Slope of AC = – 1 (... m1m2 = – 1)
Sol. Coordinates of A and B are (7, –3), and (1, 9) and co-ordinates of mid point of BD will
be
y 2  y1 9  (3)
(i)  slope (m) =  FG 1  6 , 3  8 IJ or FG 7 , 11IJ
x2  x1 1 7
H 2 2 K H2 2 K
9  3 12
= 
1 7  6
 2
 Equation of AC through
FG 7 , 11IJ and
H2 2 K
(ii) Let PQ is the perpendicular bisector of AB having slope –1 is given by
intersecting it at M. y – y1 = m (x – x1)
 Co-ordinates of M will be FG
11  1 x  7 IJ
 y H K
2 2
 x1  x2 y1  y2   7  1 3  9 
 ,  =  ,  11 7
 2 2   2 2   y  x 
2 2
 2 y – 11 = – 2 x + 7
8 6
i.e.  ,  or (4, 3).  2 x + 2 y – 11 – 7 = 0
2 2
305 Arun Deep's Understanding Math-10
 2 x + 2 y – 18 = 0 (i) In the given line x + 2y – 5 = 0
or x + y – 9 = 0 Ans. 1 5
32. ABCD is a rhombus. The co-ordinates of  2y = – x + 5  y   x
2 2
A and C are (3, 6) and (– 1, 2) respectively.
Write down the equation of BD. (2000) 1
... Slope (m1)  
Sol. Co-ordinates of A (3, 6), C (– 1, 2) 2
y2  y1 1
26  Slope of its parallel line  
 Slope of line AC ( m1 )  x  x  2
2 1  1 3
4 and equation of the parallel line
 1
4 y – y1 = m (x – x1)
 But line BD is the right bisector of AC. 1
 Slope of BD = –1 (... m1m2 = – 1)  y–3= (x + 2)  2y – 6 = – x – 2
2
and co-ordinates of mid point of AC wil be  x + 2y – 6 + 2 = 0  x + 2y – 4 = 0
(ii) ... Any line perpendicular to x – axis will be
FG 3  1 , 6  2 IJ or FG 2 , 8 IJ or (1, 4) parallel to y – axis.
H 2 2 K H 2 2K  Equation of the line will be x = a
 Equation of BD will be
It passes through (–2, 3)
y – y1 = m (x – x1)  y – 4 = – 1 (x – 1)
 a = –2
 y–4=–x+1x+y–4–1=0
Thus, x = – 2 x + 2 = 0 be the required
 x + y – 5 = 0 Ans. line. Ans.
P.Q. Find the equation of the line passing through P.Q. (i) Write down the co-ordinates of the point
the intersection of the lines P that divides the line joining A (– 4, 1) and
4x + 3y = 1 and 5x + 4y = 2 and B (17, 10) in the ratio 1 : 2.
(i) parallel to the line x + 2y – 5 = 0 (ii) Calculate the distance OP where O is the
(ii) perpendicular to the x–axis. origin
Sol. Given lines are ; (iii) In what ratio does the y-axis divides the
line AB ?
4x + 3y = 1 ...(i)
Sol. (i) Let the point P divides the line joining A
5x + 4y = 2 ...(ii)
(– 4, 1) and B (17, 10) in the ratio of 1:2.
Multiplying (i) by 4 and (ii) by 3
Let the co-ordinates of P will be (x, y)
16x + 12y = 4
15x + 12y = 6 m1 x2  m2 x1 1 × 17  2 × ( 4 )
 x 
m1  m2 1 2
– – –
On subtracting, x =–2 17  8 9
  3
Substituting the value of x in (i) ; we have 3 3
4 (– 2) + 3y = 1
m1 y2  m2 y1 1 × 10  2 ×1
 – 8 + 3y = 1  3y = 1 + 8 = 9 y 
m1  m2 1 2
9
 y 3 10  2 12
3   4
 Required point of intersection be (– 2, 3) 3 3
306 Arun Deep's Understanding Math-10
 Co-ordinates of P wil be (3, 4)  2x+y=4
(ii) O is the origin P (, ) lies on it.
...

 Distance between O and P  2+ =4 ...(i)


. ..
P is the image of P in the line
 ( x2  x1 ) 2  ( y2  y1 ) 2 x–2y–7=0
 the line bisects PP at M.
 ( 0  3) 2  ( 0  4 ) 2  ( 3) 2  ( 4 ) 2
or M is the mid-point of PP
 9  16  25  5 units.
(iii) Let y-axis divides AB in the ratio of m1, : m2  Co-ordinates of M will be
FG 1   , 2   IJ
H 2 2 K
m1 x2  m2 x1 ... M lies on the given line x – 2 y – 7 = 0
 x
m1  m2 1  2
FG IJ
 2 70
m ×17  m2 ×( 4 ) 2 H 2 K
 0 1 1 
m1  m2   (2  )  7  0
2
 17 m1 – 4 m2 = 0  17 m1 = 4 m2  1 +  – 4 – 2  – 14 = 0
m1 4   – 2  = 4 + 14 – 1 = 17 ...(ii)
   m1 : m2 i.e. 4 : 17 Ans.
m2 17   = 17 + 2 
33. Find the image of the point (1, 2) in the Substituting the value of  in (i) ; we have
line x – 2 y – 7 = 0 2 (17 + 2 ) +  = 4
Sol. Draw a perpendicular from the point P  34 + 4  +  = 4  5  = 4 – 34 = – 30
(1, 2) on the line x – 2 y – 7 = 0 30
   6
5
P(1, 2) Substituting the value of  in (i) ; we have
2  – 6 = 4  2  = 4 + 6 = 10
10
  5
2
M x – 2y – 7 = 0  Co-ordinates of P will be (5, – 6) Ans.
34. If the line x – 4 y – 6 = 0 is the
perpendicular bisector of the line segment
P’(, )
PQ and the co-ordinates of P are (1, 3),
find the co-ordinates of Q.
Let P is the image of P and let its Sol. Let the co-ordinates of Q be (, ) and let
co-ordinates are (, ) slope of line the line x – 4 y – 6 = 0 is the perpendicular
bisector of PQ and it intersects the line at M.
x–2y–7=0
 M is the mid point of PQ
1 7 1
 2 y = x – 7  y  x  is Now slope of line x – 4 y – 6 = 0
2 2 2
 Slope of PP = – 2 (... m1m2 = – 1)  4y=x–6 y
1 6
x  is
1
 Equation of PPis given by 4 4 4
y – y1 = m (x – x1)  y – 2 = – 2 (x – 1)  Slope of PQ = – 4(... m1m2 = – 1)
 y – 2 = – 2 x + 2  2 x + y = 2 + 2 and equation of line PQ
307 Arun Deep's Understanding Math-10
y – y1 = m (x – x1)  y – 3 = – 4 (x – 1)
 ( 3) 2  ( 0) 2  90  9 =3
 y – 3 = – 4 x + 4  4 x + y – 3 – 4 = 0
 4 x + y – 7 = 0  4 x + y = 7 AB  (3  p)2  ( 0  q ) 2

P(1, 3)  ( 3  p) 2  q 2

M x – 4y – 6 = 0

Q(,  )

Q (, ) lies on it.


... ... OA = AB (sides of a square)
 4+=7 ...(i)
 ( 3  p) 2  q 2 = 3
Now co-ordinates of M will be
(3 – p)2 + q2 = 9 (Squaring both sides)
FG 1   , 3   IJ 9 + p2 – 6 p + q2 = 9
H 2 2 K
...  p2 + q2 – 6 p = 0 ...(i)
M lies on the line x – 4 y – 6 = 0

1  3 FG IJ OB  ( p  0) 2  ( q  0) 2  p2  q 2
 4 60
2 2 H K But OB2 = OA2 + AB2
 1 +  – 4 (3 + ) – 12 = 0 [using pythagoras theorem]
 1 +  – 12 – 4  – 12 = 0 2

  – 4  = 24 – 1 = 23 ...(ii)  ( p 2  q 2 ) 2 = 32   (3  p ) 2  q 2 
 
Multiply (i) by 4 and (ii) by
 p2 + q2 = 9 + (3 – p)2 + q2
16  + 4  = 28
 p2 + q2 = 9 + 9 + p2 – 6 p + q2
 – 4  = 23
18
On adding we get,  6 p = 18  p  3
6
51
17  = 51  
3 Substituting the value of p in (i) ; we have
17
(3)2 + q2 – 6 (3) = 0  9 + q2 – 18 = 0
Substituting the value of  in (i) ; we have
 q2 – 9 = 0  q2 = 9  q = 3
4 × 3 +  = 7   = 7 – 12 = – 5
 p = 3, q = 3
 Co-ordinates of Q will be (3, – 5) Ans. ... AB parallel to y-axis
35. OABC is a square, O is the origin and the
 Equation AB will be x = 3
points A and B are (3, 0) and (p, q). If
OABC lies in the first quadrant, find the  x–3=0
values of p and q. Also write down the and equation of BC will be y = 3
equations of AB and BC. (... BC || x-axis)
Sol. OA   y–3=0
( 3  0) 2  ( 0  0 ) 2
308 Arun Deep's Understanding Math-10
MULTIPLE CHOICE QUESTIONS 5. The slope of the line passing through the
Choose the correct answer from the given four points (3, –2) and (3, –4) is
options (1 to 13) : (a) –2 (b) 0
1. The slope of a line parallel to y-axis is (c) 1 (d) not defined
(a) 0 (b) 1 Sol. The slope of the line passing through (3, –2)
(c) –1 (d) not defined and (3, –4)
Sol. Slope of a line parallel to y-axis is not defined.
y 2  y1 4  2 2
Since the inclination of line || to y-axis be 90° = x x = =
 Slope of line = tan 90° =  Ans. (b) 2 1 33 0
2. The slope of a line which makes an angle of = not defined =   Ans. (d)
30° with the positive direction of x-axis is
6. The inclination of the line y = 3 x – 5 is
1
(a) 1 (b) (a) 30° (b) 60° (c) 45° (d) 0°
3
Sol. The slope of the line y = 3 x – 5 is
1
(c) 3 (d)   m=
3 3 = tan   = 60°  Ans. (b)
Sol. Slope of a line which makes an angle of 30° 7. If the slope of the line passing through the
with positive direction of x-axis = tan 30° points (2, 5) and (k, 3) is 2, then the value of
1 k is
=  Ans. (b) (a) –2 (b) –1 (c) 1 (d) 2
3
3. The slope of the line passing through the Sol. Slope of the line passing through the points
points (0, –4) and (–6, 2) is (2, 5) and (k, 3) is 2, then
(a) 0 (b) 1 y 2  y1 35 2
(c) –1 (d) 6 m= x x 2= 2=
2 1 k2 k2
Sol. Slope of the line passing through the points
 2k – 4 = –2  2k = 4 – 2 = 2
(0, –4) and (–6, 2)
2
y 2  y1 24 6  k= =1  Ans. (c)
= x x = = = –1  Ans.(c) 2
2 1 60 6
8. The slope of a line parallel to the line passing
4. The slope of the line passing through the
through the points (0, 6) and (7, 3) is
points (3, –2) and (–7, –2) is
(a) 0 (b) 1 3 3 7 7
(a) (b)  (c) (d) 
7 7 3 3
1
(c)  (d) not defined Sol. Slope of the line parallel to the line passing
10
through (0, 6) and (7, 3).
Sol. Slope of the line passing through the points
(3, –2) and (–7, –2)  Slope of the given line

y 2  y1 2  2 0 y 2  y1 36 3
= = = = 0  Ans. (a) = x x = =  Ans. ((b)
x2  x1 73  10 2 1 70 7
309 Arun Deep's Understanding Math-10

3 4 4
Thus, slope of line || to given line =  (c) (d) 
7 3 3
[ m 1 = m 2] Sol. Given line be, 3x = 4y + 11
9. The slope of a line perpendicular to the line 3 11 3
passing through the points (2, 5) and (–3, 6)  4y = 3x – 11  y = x– =
4 4 4
is
1 1 3
(a)  (b)  Slope of given the =
5 5 4

(c) –5 (d) 5
 Slope of the line perpendicular to this line
Sol. Slope of the line joining the points (2, 5),
(–3, 6) 4
= ( m1 m2 = –1)  Ans. (d)
3
y 2  y1 65 1
= x x = = 12. If the lines 2x + 3y = 5 and kx – 6y = 7 are
2 1 3 2 5
parallel, then the value of k is
 Slope of the line perpendicular to this line = 5 (a) 4 (b) –4
 1 
 m1 m2  1 i.e.  5  1  Ans. (d) 1 1
 5  (c) (d) 
4 4
10. The slope of a line parallel to the line Sol. Lines 2x + 3y = 5 and kx – 6y = 7 are parallel
2x + 3y – 7 = 0 is  Slope of 2x + 3y = 5 = Slope of kx – 6y = 7
2 2  3y = –2x + 5
(a)  (b)
3 3 2 5
 y= x+
3 3 3 3
(c)  (d)
2 2 2
Sol. Slope of the line parallel to the line Slope of 2x + 3y = 5 is = m1 = .
3
2x + 3y – 7 = 0 and Slope of kx – 6y = 7  6y = kx – 7
= slope of the given line
k 7
Now 3y = –2x + 7  y= x–
6 6
2 x 7
 y= +  Ans. ((a) k
3 3  Slope m2 =
6
On comparing with y = mx + c
Since both lines are parallel.
2
 Slope of line || to given line = m =  m1 = m2
3
11. The slope of a line perpendicular to the line 2 k 2  6
 = k= = –4  Ans. (b)
3x = 4y + 11 is 3 6 3

3 3 13. If the line 3x – 4y + 7 = 0 and 2x + ky + 5 = 0


(a) (b)  are perpendicular to each other, then the
4 4
310 Arun Deep's Understanding Math-10
value of k is y = mx + c  3 x  ( 4 )
3 3
(a) (b)   y= 3 x – 4 Ans.
2 2
2. Write down the gradient and the intercept
2 2 on the y-axis of the line 3 y + 2 x = 12.
(c) (d)  Sol. Given line be, 3 y + 2 x = 12
3 3
 3 y = 12 – 2 x  3 y = – 2 x + 12
Sol. Given, lines are 3x – 4y + 7 = 0 ...(1)
and 2x + ky + 5 = 0 ...(2) 2
 y x4
3
 Product of their slopes = (m1 × m2) = –1
2
For the (1) ; 4y = 3x + 7  Slope  and y-intercept = 4
3
3 7
 y= x+ 3. If the equation of a line is y  3 x  1,
4 4
find its inclination.
3 Sol. Given line be,
 Slope (m1) =
4
y 3 x  1 ; comparing with y = mx + c
and for second line 2x + ky + 5 = 0
m = slope  3  tan   3
 ky = –2x – 5
  = 60° (... tan 60°  3 )
2 5 4. If the line y = mx + c passes through the
 y= x–
k k points (2, – 4) and (– 3, 1), determine the
values of m and c.
2
 Slope (m2) = Sol. The equation of given line y = mx + c
k
. . . it passes through (2, – 4) and (– 3, 1)
Since the given lines are perpendicular to each
Now substituting the value of these points
other.
–4=2m+c ...(i)
 m1m2 = –1
and 1=–3m+c ...(ii)
3 2 6 On subtracting we get,
 × = –1  = –1
4 k 4k 5
– 5 = 5 m  m  1
6 3 5
 –k =  k = Ans. (a) Substituting the value of m in (i)
4 2
– 4 = 2 (– 1) + c  – 4 = – 2 + c
CHAPTER TEST
c=–4+2=–2
1. Find the equation of a line whose inclination
 m = – 1, c = – 2
is 60° and y-intercept is – 4.
5. If the points (1, 4), (3, – 2) and (p, – 5) lie
Sol. Given, angle of inclination = 60° on a st. line, find the value of p.
 Slope of line = tan  = tan 60°  3 Sol. Let the points to be A (1, 4), B (3, – 2) and
 Equation of the line using slope intercept C (p, – 5) are collinear and let B (3, – 2)
form will be, given by divides AC in the ratio of m1 : m2.
311 Arun Deep's Understanding Math-10
x–2y=5 ...(ii)
m1 x2  m2 x1
 x Multiply (i) by 2 and (ii) by 1, we get
m1  m2
4 x + 2 y = 10
m p  m2 ×1
 3 1 x–2y=5
m1  m2
On adding ; we get,
3 m 1 + 3 m 2 = m 1p + m 2
15
 3 m 1 – m 1p = m 2 – 3 m 2 5 x = 15  x 3
5
 m1 (3 – p) = – 2 m2 Substituting the values of x in (i) ; we have
m1 2 2×3+y=56+y=5
 m  3 p ...(i)
2  y=5–6= –1
m1 ( 5)  m2 × 4  Co-ordinates of point of intersection be
and 2  m1  m2 (3, – 1).
 – 2 m1 – 2 m2 = – 5 m1 + 4 m2 3
 – 2 m1 + 5 m1 = 4 m2 + 2 m2 Given y-intercept of line be c =
7
 3 m1 = 6 m2 Thus eqn. of line be y = mx + c
m1 6
 m  3 2 ...(ii)  y = mx –
3
2 7
from (i) and (ii) ; we have
... the line passes through (3, – 1)
2
2 –2=6–2p
3 p 3
 1  m × 3 
8 7
 2p=6+2=8 p 4
2 3 4
 3 m  1  
6. Find the inclination of the line joining the 7 7
points P (4, 0) and Q (7, 3).
4 4
Sol. Slope of the line joining the points P (4, 0)  m 
7  3 21
and Q (7, 3)
 Required equation of line be
y  y1 3 0 3
 2 4
x2  x1  7  4  3  1
3
y x   21 y = – 4 x – 9
21 7
 tan  = 1   = 45° (... tan 45° = 1)  4 x + 21 y + 9 = 0 Ans.
where  be the inclination of the line. P.Q. If the point A is reflected in the y-axis, the
Hence inclination of line = 45° Ans. co-ordinates of its image A1, are (4, – 3),
7. Find the equation of the line passing (i) Find the co-ordinates of A
through the point of intersection of the lines (ii) Find the co-ordinates of A2, A3 the im-
2 x + y = 5 and x – 2 y = 5 and having y- ages of the points A, A1, respectively un-
3 der reflection in the line x = – 2
intercept equal to  .
7
Sol.
Sol. Equation of given lines are
(i) ... A is reflected in the y-axis and its image
2x+y=5 ...(i)
is A (4, – 3)
312 Arun Deep's Understanding Math-10
 Co-ordinates of A will be (– 4, – 3) 3 11
 y x ...(ii)
k k
Let slope of line (i) be m1 and of (ii) be m2
4 3
 m1  and m2  
3 k
. . . These lines are perpendicular to each other

4 FG IJ
3
 m 1m 2 = – 1  ×   1
3 H K
k
4
  1  – k = 4  k = – 4 Ans.
k
9. Write down the equation of the line parallel
to x – 2 y + 8 = 0 and passing through the
point (1, 2).
Sol. The equation of the line is
x–2y+8=02y=x+8
1
 y x4
2
1
 Slope of the line 
(ii) ... A2 is the image of A (– 4, – 3) in the line 2
x = – 2 which is parallel to y-axis  Slope of the line parallel to the given line
1
 AA2 is perpendicular to the line x = – 2 and passing through (1, 2) 
2
this line bisects AA2 at M.
 Equation of the line will be [using one point
Let co-ordinates of A2 be (, ) form] given by
 =0 and  = – 3 y – y1 = m (x – x1)
 Co-ordinates of A2 will be (0, – 3) 1
 y–2  ( x  1)  2 y – 4 = x – 1
Again x = – 2 is perpendicular bisector of 2
A1, A3 intersecting it at M  x–2y–1+4=0
 M is mid point of A1A3.  x – 2 y + 3 = 0 Ans.
 A3M = MA1 10. Write down the equation of the line passing
 Co-ordinates of A3 will be (– 8, – 3) Ans. through (– 3, 2) and perpendicular to the
line 3 y = 5 – x.
x y
8. If the lines  = 7 and 3 x + ky = 111 Sol. Equations of the given line be
3 4
are perpendicular to each other, find the 3 y = 5 – x  3 y = – x + 5
value of k. 1 5
 y x
Sol. Equation of lines are 3 3
x y 1
  7  4 x + 3 y = 84  Slope of the line  
3 4 3
4 and slope of the line perpendicular to it and
 3y = – 4x + 84 y  x  28 ...(i) passing through (– 3, 2) will be = 3
3
and 3 x + ky = 11  ky = – 3 x + 11 (... m1m2 = – 1)
313 Arun Deep's Understanding Math-10
 Equation of the line using one point form 12. The points A (7, 3) and C (0, – 4) are two
will be given by opposite vertices of a rhombus ABCD. Find
y – y1 = m (x – x1) the equation of the diagonal BD.
 y – 2 = 3 (x + 3) y2  y1
Sol. Slope of line AC = (m1)  x  x
 y–2=3x+9 2 1
 3x–y+9+2=0 4  3 7
  1
 3 x – y + 11 = 0 Ans. 07 7
11. Find the equation of the line perpendicular to
the line joining the points A (1, 2) and B (6, 7)
and passing through the point which divides
the line segment AB in the ratio 3 : 2.
Sol. Let slope of the line joining the points
A (1, 2) and B (6, 7) be m1
y  y1 72 5
 m1  2   1
x2  x1 6 1 5
Let m2 be the slope of the line perpendicular
to it then m1 × m2 = – 1  1 × m2 = – 1 ... Diagonals of a rhombus bisect each other
 m2 = – 1 at right angles
Let the point P (x, y) divides the line AB in  BD is perpendicular to AC
the ratio of 3 : 2  Slope of BD = – 1(... m1m2 = – 1)
3:2 and co-ordinates of O, the mid point of AC
A(1, 2) P(x, y) B(6, 7)
will be

m1x2  m2 x1 3 × 6  2 ×1 FG 7  0 , 3  4 IJ or FG 7 , 1IJ
 x m1  m2 
H 2 2 K H2 2 K
3 2


18  2

20
4  Equation of BD through GH ,
F 7 1IJ
5 5 2 2K

m1 y2  m2 y1 and having slope (–1) is given by


3×72 ×2
and y  m1  m2 
3 2 1 7FG IJ
y – y1 = m (x – x1)  y   1 x 
21  4 25
2 2 H K
  5
5 5 1 7
 y  x 
 Co-ordinates of P will be (4, 5) 2 2
Now equation of the line passing through  2y+1=–2x+7
P (4, 5) and having slope – 1 is given by  2x+2y+1–7=0
y – y1 = m (x – x1)  y – 5 = – 1 (x – 4)  2x+2y–6=0
 y – 5 = – x + 4  x + y – 5 – 4 = 0  x + y – 3 = 0 (Dividing by 2) Ans.
 x+y–9=0 13. A straight line passes through P (2, 1) and
314 Arun Deep's Understanding Math-10
cuts the axes in points A, B.  3 x + 2 y – 8 = 0 Ans.
If BP : PA = 3:1, find : 14. A straight line makes on the co-ordinates
axes positive intercepts whose sum is 7. If
the line passes through the point (– 3, 8),
find its equation.
Sol. Let the line make intercept a and b with
the x-axis and y-axis respectively then the
line passes through

(i) the co-ordinates of A and B


(ii) the equation of the line AB
Sol. A lies on x-axis and B lies on y-axis
Let co-ordinates of A be (x, 0) and B be
(0, y)
and P (2, 1) divides BA in the ratio 3 : 1. A (a, 0) and B (0, b)
m1x2  m2 x1 3× x 1× 0 but a + b = 7
 x m1  m2  2
31  b=7–a
3x 8 Now slope of the line
 2 3x=8 x
4 3 y  y1 b  0 b
 2  
m1 y2  m2 y1 3× 0 1× y x2  x1 0  a a
and y  m1  m2  1  31  Equation of the line through (a, 0) and
y b
 1  y = 4 having slope
4 a

 Co-ordinates of A will be
FG 8 , 0IJ and of B y – y1 = m (x – x1)
H3 K b
will be (0, 4)  y–0  ( x  a) ...(i)
a
y 2  y1 . . . the line (i) passes through the point (– 3, 8)
 Slope of the line AB 
x2  x1
b 7a
40 4 3 3  8–0  ( 3  a )   ( 3  a )
   4 ×   a a
8 8 8 2
0   8 a = (7 – a) (3 + a)
3 3
 Equation of AB using one point form will  8 a = 21 + 7 a – 3 a – a2
be given by  a2 + 8 a – 7 a + 3 a – 21 = 0
3  a2 + 4 a – 21 = 0
y – y1 = m (x – x1)  y – 1  ( x  2)
2  a2 + 7 a – 3 a – 21 = 0
 2y–2=–3x+6  a (a + 7) – 3 (a + 7) = 0
 3x+2y–2–6=0  (a + 7) (a – 3) = 0
315 Arun Deep's Understanding Math-10
Either a + 7 = 0, then a = – 7, which is not 3
possible as it is not positive  Slope of BD 
7
or a – 3 = 0, then a = 3
7
and b=7–3=4 and slope of AC   (... m1m2 = – 1)
3
 Equation of the line will be given by  Equation of AC will be
b
y–0  ( x  a) 7
a y – y1 = m (x – x1)  y  2  ( x  3)
3
4  3 y + 6 = – 7 x + 21
 y ( x  3)  3 y = – 4 x + 12
3  7 x + 3 y + 6 – 21 = 0
 4 x + 3 y – 12 = 0  7 x + 3 y – 15 = 0
 4 x + 3 y = 12 Ans.
Now we will find the co-ordinates of O,
15. If the co-ordinates of the vertex of a square the points of intersection of AC and BD
ABCD are (3, – 2) and the equation of
We will solve the equations
diagonal BD is 3x – 7y + 6 = 0, find the
equation of the diagonal AC. Also find the 3x–7y=–6 ...(i)
co-ordinates of the centre of the square. 7 x + 3 y = 15 ...(ii)
Sol. Co-ordinates of A are (3, – 2). Multiplying (i) by 3 and (ii) by 7, we get
9 x – 21 y = – 18
49 x + 21 y = 105
On adding we get
87 3
58 x = 87  x  
58 2
Substituting the value of x in (iii), we get

9
FG 3 IJ  21 y = – 18  27  21 y  18
H 2K 2

27 27  36 63
 21 y   18  
Diagonals AC and BD of the square ABCD 2 2 2
bisect each other at right angle at O.
63 3
 O is the mid-point of AC and BD y 
2 × 21 2
Equation of BD is 3 x – 7 y + 6 = 0
3 6  Co-ordinates of O will be
FG 3 , 3 IJ Ans.
 7 y = 3 x + 6  y 
7
x
7
H 2 2K
13
Similarity
POINTS TO REMEMBER :
1. Similar figures. Two figures are similar which have exactly the same shape but not necessar-
ily the same size.
2. Similar triangles. When two triangles are similar, their corresponding angles are equal and
corresponding sides are proportional.
3. Postulates of similarity of triangles.
(i) If two triangles have a pair of corresponding angles equal and the sides including them
proportional. Then the triangles are similar and this postulate is called SAS postulate.
(ii) If two triangles have two pairs of corresponding angles equal, the triangles are similar. This
postulate is called AA or AAA postulate.
(iii) If two triangles have their three pairs of corresponding sides proportional, the triangles are
similar and this postulate is called SSS postulate.
Note. (i) Symbol of similarity is used as ‘~’.
(ii) All squares, equilateral triangles and circles are similar.
4. Maps. The map of a plane figure and the actual figure are similar to one another. If the map of
m
a plane figure is drawn to the scale m : n then k the scale factor  .
n
 (i) Length of map = k × length of the actual figure
(ii) Area of map = k2 × area of the actual figure.
5. Models. Model of a plane or solid figure and the actual figure are similar to one another. If the
m
model of a figure is drawn to the scale m : n, then k, the scale factor 
n
 (i) Volume of the model = k3 × volume of the actual figure.
THEOREM 13.1
1. The ratio of the areas of the similar triangles is equal to the ratio of the squares of any two
corresponding sides.
Given : ABC ~ DEF
Area of ABC AB2 BC2 CA 2
To prove :   
Area of DEF DE 2 EF2 FD 2
Construction : Draw AL BC and DM  EF
Proof :  BC ~ DEF (given)
AB BC AC
   ...(i) and B  E
DE EF DF
Now in ABL and DEM,
B  E (proved)
L  M (each 90º)

316 Arun Deep's Understanding Math-10


317 Arun Deep's Understanding Math-10
AB AL
 ABL ~ DEM 
DE DM
1
BC×AL
area ( ABC) 2 BC AL BC BC BC2
Now   ×  ×  (from (i) and (ii))
area ( DEF) 1 EF×DM EF DM EF EF EF2
2
area ( ABC) AB2 BC 2 AC2
Now from (i)    Hence proved.
area ( DEF ) DE 2 EF 2 DF 2
Corollary. The ratio of the areas of the similar triangle is equal to the ratio of the squares of any
two corresponding altitudes.

EXERCISE 13.1
1. State which pairs of triangles in the figure Sol. (i) In ABC and PQR
given below are similar. Write the similarity
AB 3.2 4
rule used and also write the pairs of similar
triangles in symbolic form (all lengths of PQ = 4 = 5
sides are in cm):
AC 3.6 4
= =
PR 4.5 5
P
A
BC 3 5
4.5 QR = 5.4 = 9
3.6 4
3.2
All the sides are not proportional.
 The triangles are not similar.
B 3 C Q 5.4 R (ii) In DEF and LMN
E = N = 40°
(i) DE 4 2 EF 4.8 2
= = and = =
LN 2 1 MN 2.4 1
 DEF ~ LMN (SAS axiom)
P.Q. It is given that DEF ~ RPQ. Is it true to
say that D = R and F = P? Why?
Sol. DEF ~ RPQ
4 L
 D = R and F = Q not P
 No, F  P
F 2 2. If in two right triangles, one of the acute
40° angle of one triangle is equal to an acute angle
4.8
M 2.4 N of the other triangle, can you say that the
E
(ii)
two triangles are similar? Why?
Sol. In two right triangles, one of the acute angle
of the one triangle is equal to an acute angle
of the other triangle.
The triangles are similar. (AAA axiom)
318 Arun Deep's Understanding Math-10
P.Q. In the given figure, BD and CE intersect each other at the point P. Is PBC ~ PDE? Give reasons
for your answer.

B 12 cm
5 cm

P
6 cm
10 cm
C
D

Sol. In the given figure, two line segments intersect each other at P.
In BCP and DEP
BPC = DPE (Vertically opposite angles)

5 6  1
=  each  
10 12  2
 BCD ~ DEP (SAS axiom)
3. It is given that ABC ~ EDF such that AB = 5 cm, AC = 7 cm, DF = 15 cm and DE = 12 cm.
Find the lengths of the remaining sides of the triangles.
Sol. ABC ~ EDF
AB = 5 cm, AC = 7 cm, DF = 15 cm and DE = 12 cm
ABC ~ EDF
E
AB AC BC
 = = A
ED EF DF
7cm

5 7 BC
12c

 = =
5cm

12 EF 15
7 5 7  12 84
 =  EF = = = 16.8 cm B C D 15cm F
EF 12 5 5
5 BC
and =
12 15
5  15 25
 BC = = = 6.25 cm
12 4
4. (a) If ABC ~ DEF, AB = 4 cm, DE = 6 cm, EF = 9 cm and FD = 12 cm, then find the perimeter
of ABC.
(b) If ABC ~ PQR, Perimeter of ABC = 32 cm, perimeter of PQR = 48 cm and PR = 6 cm, then
find the length of AC.
319 Arun Deep's Understanding Math-10
Sol. (a) ABC ~ DEF
AB = 4 cm, DE = 6 cm, EF = 9 cm and FD = 12 cm

D
A

12
6c m

cm
4cm
B C E 9cm F

Perimeter of ABC
ABC ~ DEF
AB AC BC
 = =
DE DF EF
4 AC BC
 = =
6 12 9
AC 4 12  4 BC 4 BC 4
=  AC = = 8 cm and =  =
12 6 6 EF 6 9 6
9 4
 BC = = 6 cm
6
 Perimeter of ABC = AB + BC + AC
= 4 + 6 + 8 = 18 cm
(b) ABC ~ PQR
Given Perimeter of ABC = 32 cm and
Perimeter of PQR = 48 cm
Side PR = 6cm
ABC ~ PQR P
A
AB AC BC

6c

= =
PQ PR QR
m

Perimeter of ABC AC
 =
Perimeter of PQR PR B C Q R
32 AC
 =
48 6
32  6
 AC = = 4 cm
48
Hence, the length of AC = 4 cm.
5. Calculate the other sides of a triangle whose shortest side is 6 cm and which is similar to
320 Arun Deep's Understanding Math-10
a triangle whose sides are 4 cm, 7 cm and Sol. (a) In the given figure,
8 cm. AB || DE, AC = 3 cm, CE = 7.5 cm, BD = 14 cm
Sol. Let ABC ~ DEF in which shortest side of
A B
ABC is BC = 6 cm.
3 cm
In DEF, DE = 8 cm, EF = 4 cm C
and DF = 7 cm 14 cm
7.5 cm
D
A
D E
To calculate CB and DC
7cm
8c m

In ABC and CDE


ACB = DCE
(Vertically opposite angles)
B 6cm C E 4cm F
BAC = CED (Alternate agnles)
ABC ~ DEF
 ABC ~ CDE (AA axiom)
AB BC AC AB 6 AC
 = =  = = AC BC 3 BC
DE EF DF 8 4 7  =  =
CE CD 7.5 CD
AB 6 8 6
Now, =  AB = = 12 cm  BC × 7.5 = 3CD  BD  14 cm 
8 4 4  
 CB  14  DC
AC 6 76 21
=  AC = = = 10.5 cm Let BC = x, then CD = (14 – x) cm
7 4 4 2
x × 7.5 = 3 × (14 – x)
6. (a) In the figure given below, AB || DE, AC = 7.5x = 42 – 3x
3 cm, CE = 7.5 cm and BD = 14 cm.
7.5x + 3x = 42  10.5x = 42
Calculate CB and DC.
A B 42
x= =4
10.5
C
 BC = 4 cm and DC = 14 – 4 = 10 cm
(b) In the given figure, CA || BD
AB and CD intersect at O
D E
To prove :
(b) In the figure (2) given below, CA || BD, the
lines AB and CD meet at O. (i) ACO ~ BDO
(i) Prove that ACO ~ BDO. (ii) BD = 2.4 cm, OD = 4 cm
(ii) If BD = 2.4 cm, OD = 4 cm, OB = 3.2 cm OB = 3.2 cm, AC = 3.6 cm, then calculate
and AC = 3.6 cm, calculate OA and OC. OA and OC

A A
D D

O O

B B
C C
321 Arun Deep's Understanding Math-10
Proof : C = C (common)
(i) In ACO and BDO BAC = ADC (given)
AOC = BOD  ABC ~ ADC (AA axiom)
(Vertically opposite angles)
CA BC
A = B  =
DC CA
 ACO ~ BDO (AA axiom)
(Corresponding sides are proportional)
BD = 2.4 cm, OD = 4 cm, OB = 3.2 cm,
AC = 3.6 cm  CA × CA = DC × BC
ACO ~ BOD  CA2 = DC × BC
8. (a) In the figure (1) given below, AP = 2PB
AO CO AC
 = = and CP = 2PD.
OB OD BD (i) Prove that ACP is similar to BDP and
AO CO 3.6 AC || BD.
 = = (ii) If AC = 4.5 cm, calculate the length of BD.
3.2 4 2.4
AO 3.6 3.6  3.2 A
 =  AO = = 4.8 cm
3.2 2.4 2.4
D
CO 3.6 3.6  4
=  CO = = 6 cm
4 2.4 2.4 P
7. (a) In the figure (i) given below, P = RTS.
Prove that RPQ ~ RTS.
(b) In the figure (ii) given below,
B
ADC = BAC. Prove that CA2 = DC × BC. C

R A (b) In the figure (2) given below,


ADE = ACB.
T (i) Prove that s ABC and AED are similar.
(ii) If AE = 3 cm, BD = 1 cm and AB = 6 cm,
calculate AC.
S
A
P Q B D C
(i) (ii)
E
Sol. (a) In the given figure, P = RTS
To prove : RPQ ~ RTS D
Proof : In RPQ and RTS
R = R (common) B C
P = RTS (given)
 RPQ ~ RTS (AA axiom) (c) In the figure (3) given below, PQR =
(b) In the given figure, ADC = BAC PRS. Prove that triangles PQR and PRS
To prove : CA2 = DC × BC are similar. If PR = 8 cm, PS = 4 cm,
Proof : In ABC and ADC calculate PQ.
322 Arun Deep's Understanding Math-10
 ABC ~ AED (AA axiom)
P
BC AB AC
 = =
DE AE AD

S R BC 6 AC
= =
DE 3 5

 AD  AB  BD
 
  6  1  5 cm 
Q
6 AC 65
Sol. In the given figure, =  AC = = 10 cm
3 5 3
AP = 2PB, CP = 2PD
(c) In the given figure, PQR = PRS
To prove :
To prove :
(i) ACP ~ BDP and AC || BD
(ii) If AC = 4.5 cm, find length of BD (i) PQR ~ PRS
Proof : (ii) If PR = 8 cm, PS = 4 cm, find PQ

AP 2 P
(i) AP = 2PB  =
PB 1
8
4
CP 2
and CP = 2PD  = S
PD 1 R
and APC = BPD
(Vertically opposite angles)
 ACP ~ BDP (SAS axiom)
 CAP = PBD Q
But these are alternate angles
 AC || BD Proof : In PQR and PRS
AP AC 2 P = P (common)
(ii)  = = PQR = PRS (given)
PB BD 1
 PQR ~ PRS (AA axiom)
 AC = 2BD  2BD = 4.5 cm
4.5 PQ PR QR
 BD = = 2.25 cm  = =
2 PR PS SR
(b) In the given figure, ADE = ACB (Sides are proportional)
To prove : PQ 8 88
(i) ABC ~ AED =  PQ = = 16 cm
8 4 4
(ii) If AE = 3 cm, BD = 1 cm  PQ = 16 cm
and AB = 6 cm, find AC 9. In the given figure, ABC is a triangle in which
Proof : In ABC and AED AB = AC. P is a point on the side BC such
A = A (common) that PM  AB and PN  AC. Prove that
ACB = ADE (given) BM × NP = CN × MP.
323 Arun Deep's Understanding Math-10

A AB  BC  CA
= PQ  QR  RP

Perimeter of ΔABC
= Perimeter of ΔPQR

M N 11. In the given figure, ABCD is a trapezium in


which AB || DC. The diagonals AC and BD
B P C AO BO
intersect at O. Prove that = .
Sol. In the given figure, ABC in which AB = AC. OC OD
P is a point on BC such that PM  AB and
PN  AC A B
To prove : BM × NP = CN × MP
Proof : In ABC, AB = AC
 B = C (Angles opposite to equal sides) O
Now in BMP and CNP
M = N (each 90°) D C
 B = C (proved)
Using the above result, find the value(s) of x
 BMP ~ CNP (AA axiom)
if OA = 3x – 19, OB = x – 4, OC = x – 3 and
BM MP OD = 4.
 = (sides are proportional)
CN NP Sol. ABCD is a trapezium in which AB || DC
 BM × NP = CN × MP Diagonals AC and BD intersect each other at
(By cross multiplication) O.
Hence proved.
A B
10. Prove that the ratio of the perimeters of two
similar triangles is the same as the ratio of
their corresponding sides.
Sol. Given : ABC ~ PQR O
To prove : Ratio in their perimeters is the
same as the ratio in their corresponding sides. D C
P To prove :
A
AO BO
(i) =
OC OD
(ii) If OA = 3x – 19, OB = x – 4, OC = x – 3,
OD = 4
B C Q R Find the value of x
Proof :
Proof : ABC ~ PQR
(i) In AOB and COD
AB BC CA AOB = COD
 PQ = QR =
RP (vertically opposite angles)
324 Arun Deep's Understanding Math-10
OAB = OCD  AB × AE = AC × AD
 AOB ~ COD (By cross multiplication)
OA OB P.Q. In the given figure, DB  BC, DE  AB and
 = BE AC
OC OD
AC  BC. Prove that = .
(ii) OA = 3x – 19, OB = x – 4, OC = x – 3, DE BC
OD = 4 A
3 x  19 x4 D
then =
x3 4
By cross multiplication,
E
(x – 3) (x – 4) = 4(3x – 19)
x2 – 4x – 3x + 12 = 12x – 76 B C
 x2 – 7x – 12x + 12 + 76 = 0 Sol. In the given figure, DB  BC, DE  AB and
 x2 – 19x + 88 = 0 x2 – 8x – 11x + 88 = 0 AC  BC
 x(x – 8) – 11(x – 8) = 0 BE AC
(x – 8) (x – 11) = 0 To prove : =
DE BC
Either x – 8 = 0, then x = 8 Proof : In ABC and DEB
or x – 11 = 0, then x = 11 C = 90°
 x = 8, 11  A + ABC = 90° ...(i)
P.Q. In ABC, A is acute. BD and CE are and In DEB
perpendicular on AC and AB respectively. DBE + ABC = 90° ...(ii)
Prove that AB × AE = AC × AD. From (i),
Sol. In ABC, A is acute A = DBE
BD and CE are perpendiculars on AC and AB Now in ABC and DBE
respectively. C = E (each 90°)
A
 ABC ~ DBE (AA axiom)
AC BC AC BE
 =  =
BE DE BC DE
D P.Q. In the given figure, medians AD and BE of
E ABC meet at the point G, and DF is drawn
parallel to BE. Prove that
(i) EF = FC (ii) AG : GD = 2 : 1.
B C
A
To prove : AB × AE = AC × AD
Proof : In ADB and AEC
A = A (common)
E
ADB = AEC (each 90°)
G
 ADB ~ AEC (AA axiom) F

AB AD
 = B D C
AC AE
325 Arun Deep's Understanding Math-10
Sol. In the given figure,
F
AD and BE are the medians of ABC
intersecting each other at G
D
DF || BE is drawn
To prove : 3cm

7.5cm
E

4.5cm
(ii) EF = FC (ii) AG : GD = 2 : 1
Proof : y
(i) D is the mid-point of BC and DF || BE x
 F is the mid-point of CE A B C
1 1
 EF = FC = EC  EF = AE Sol. In the given figure,
2 2 AB || EF || CD
Now in AGE and ADF AB = 15 cm, EG = 5 cm, GC = 10 cm and
GE or BG || DF DC = 18 cm
 AGE ~ ADF Calculate :
(i) EF (ii) AC
AG AE 1 2 2 Proof : In EFG and CGD
 = = =1× =
GD EF 1 1 1 EGF = CGD
2 (vertically opposite angles)
 AG : GD = 2 : 1 FEG = GCD (alternate angles)
12. (a) In the figure given below, AB, EF and  EFG ~ CGD (AA axiom)
CD are parallel lines. Given that AB = 15 cm, EG EF
EG = 5 cm, GC = 10 cm and DC = 18 cm.  =
GC CD
Calculate
5 EF 5  18
(i) EF (ii) AC.  =  EF = = 9 cm
10 18 10
 EF = 9 cm
D
(ii) In ABC and EFC
EF || AB
A  ABC ~ EFC
AC AB AC 15
18cm

E  =  =
5 cm EC EF 5  10 9
15cm

10c AC 15 15  15
G m
 =  AC = = 25 cm
15 9 9
B F C  AC = 25 cm
(b) In the given figure, AF || BE || CD
(b) In the figure given below, AF, BE and CD AF = 7.5 cm, CD = 4.5 cm, ED = 3 cm
are parallel lines. Given that AF = 7.5 cm, BE = x and AE = y
CD = 4.5 cm, ED = 3 cm, BE = x and AE = To find the value of x and y
y. Find the values of x and y. In AEF and CED
326 Arun Deep's Understanding Math-10
AEF = CED (vertically opposite angles)
F = C (alternate angles)
 AEF ~ CED (AA axiom)
AF AE 7.5 y
 =  =
CD ED 4.5 3
7 .5  3
 y= = 5.0 cm
4 .5
Similarly in ACD, BE || CD
 ABE ~ ACD
EB AE
=
CD AD

x y x 5 5
 = y3  = =
CD 4.5 53 8

4 .5  5 22.5 225 45 13
x= = = = = 2 cm
8 8 10  8 16 16
13. In the given figure, A = 90° and AD  BC If BD = 2 cm and CD = 8 cm, find AD.
A

Sol. In ABC, we have A = 90°


Also, AD  BC
Now, B D C
In ABC, we have,
BAC = 90°
 BAD + DAC = 90° ...(i)
In ADC, we have,
ADC = 90°
So, DCA + DAC = 90° ...(ii)
From (i) and (ii), we have
BAD + DAC = DCA + DAC
 BAD = DCA ...(iii)
In BDA and ADC,
BDA = ADC (90° each)
BAD = DCA (proved above)
So, BDA ~ ADC (AA similarity)
BD AD AB
 = =
AD DC AC
(Corresponding sides of similar 's are proportional)
327 Arun Deep's Understanding Math-10

BD AD
 =
AD DC
 AD = BD × CD
2

 AD2 = 2 × 8 = 16
 AD = 4 cm
14. A 15 metres high tower casts a shadow of 24 metres long at a certain time and at the same time, a
telephone pole casts a shadow 16 metres long. Find the height of the telephone pole.
Sol. Height of a tower AB = 15 m
and its shadow BC = 24 m
D
At the same time and position
Let height of a telephone pole DE = x m
and its shadow EF = 16 m x
The time is same
 ABC ~ DEF F 16m E
AB CD
 = A
DE EF
15 24
 = 15m
x 16
15  16
 x= = 10
24 C 24 B
 Height of pole = 10 m
15. A street light bulb is fixed on a pole 6 m above the level of street. If a woman of height 1.5 m
casts a shadow of 3 m, find how far she is away from the base of the pole?
Sol. Height of height pole (AB) = 6 m
and height of a woman (DE) = 1.5 m
Here shadow EF = 3 m
Pole and woman are standing in the same line A

 AD || DF
 AFB ~ DFE D 6m
FB AB 3 x 6 4 1.5
 =  = =
EF DE 3 1 .5 1
F 3m E B
 3 + x = 12  x = 12 – 3 = 9 m
 Woman is 9 m away from the pole.

EXERCISE 13.2
1. (a) In the figure (i) given below if DE BC, AD = 3 cm, BD = 4 cm and BC = 5 cm.
Find (i) AE : EC (ii) DE.
(b) In the figure (ii) given below, PQ AC, AP = 4 cm, PB = 6 cm and BC = 8 cm. Find CQ and BQ.
(c) In the figure (iii) given below, if XY QR,
328 Arun Deep's Understanding Math-10
PX = 1 cm, QX = 3 cm, YR = 4.5 cm and QR PB = 6 cm and BC = 8 cm
= 9 cm, find PY and XY. To find CQ and BQ
A
C Now PQ AC (Given)
BQP = BCA (Alternate angles)
Q
D E Also, B = B (common)
  ABC ~  BPQ
C
B C A P B BQ BP PQ
(i) (ii)  = = Q
BC AB AC
P

BQ 6 PQ
 = =
X Y BC 64 AC
A P (ii)
B
BQ 6 PQ
 = =
BC 10 AC
Q R
(iii) BQ 6 PQ
 = = [  BC = 8 cm given]
Sol. (a) In the figure (i) A 8 10 AC
Given : DE BC, AD = 3 cm,
BD = 4 cm and BC = 5 cm. D E
BQ 6
Now, =
To find (i) AE : EC and (ii) DE 8 10

Since DE BC of  ABC 6 48
B C
 BQ = ×8= = 4.8 cm
10 10
AD AE AE AD
 =  =
BD EC EC BD Also, CQ = BC – BQ  CQ = (8 – 4.8) cm
 CQ = 3.2 cm
3
= [  AD = 3 cm BD = 4 cm] Hence, CQ = 3.2 cm and BQ = 4.8 cm Ans.
4
AE : EC = 3 : 4 Ans. (c) In the figure (iii)
Given : XY QR,
(ii) In  ADE and  ABC
PX = 1 cm, QX = 3 cm, YR = 4.5 cm and
D = B and E = C [  DE BC given] QR = 9 cm
  ADE ~  ABC To find : PY and XY
DE 3 Now, XY QR (Given)
DE AD P
 =  =
BC AB 5 3 4 PX PY
 =
[  AB = AD + BD = 3 cm + 4 cm] QX YR X Y

DE 3 3 5 1 PY
 =  DE =  =
5 7 7 3 4.5
Q R
15 1 4 .5  1
DE = = 2 cm Ans. (iii)
7 7  = PY  1.5 = PY
3
(b) In the figure (ii)
 PY = 1.5 cm Ans.
Given : PQ AC, AP = 4 cm,
329 Arun Deep's Understanding Math-10
Also, X = Q In ABC
and Y = R (XY QR given)  DE || BC
  PXY ~  PQR AB AC
 =
DB EC
XY PX
 = 2x 2x  3
QR PQ
 =
x3 x2
XY 1
 = [PQ = 1 + 3 = 4 cm] By cross multiplication,
9 1 3
2x(x – 2) = (2x + 3) (x – 3)
XY 1 9  2x2 – 4x = 2x2 – 6x + 3x – 9
 =  XY = = 2.25 cm Ans.
9 4 4  2x2 – 4x – 2x2 + 6x – 3x = –9
2. In the given figure, DE || BC.  –x = –9  x = 9
(i) If AD = x, DB = x – 2, AE = x + 2 and  x=9
EC = x – 1, find the value of x. 3. E and F are points on the sides PQ and PR
(ii) If DB = x – 3, AB = 2x, EC = x – 2 and respectively of a PQR. For each of the
AC = 2x + 3, find the value of x. following cases, state whether EF || QR:
(i) PE = 3.9 cm, EQ = 3 cm, PF = 8 cm and
A RF = 9 cm.
(ii) PQ = 1.28 cm, PR = 2.56 cm, PE = 0.18
cm and PF = 0.36 cm.
Sol. (i) In PQR, E and F are the points on the
D E sides PQ and PR respectively
PE = 3.9 cm, EQ = 3 cm, PF = 8 cm,
RF = 9 cm
Is EF || QR?

P
B C

Sol. In the given figure, DE || BC


(i) AD = x, DB = x – 2, AE = x + 2, EC = x – 1 F E
In ABC,
DE || BC
AD AE
 = Q R
DB EC
x x2 PE 3.9 39 13
 = (By cross multiplication) = = =
x2 x 1 EQ 3 30 10
x(x – 1) = (x – 2) (x + 2) PF 8
x2 – x = x2 – 4 =
FR 9
–x = –4  x = 4
(ii) DB = x – 3, AB = 2x PE PF
EQ 
EC = x – 2, AC = 2x + 3 FR
330 Arun Deep's Understanding Math-10
 EF is not parallel to QR PR PB  BR 46 10 2.5
(ii) PQ = 1.28 cm, PR = 2.56 cm, PE = 0.18 and = = = =
PB PB 4 4 1
cm, PF = 0.36 cm
PQ PR
P =
PA PB
 AB || QR
P.Q. (a) In figure (i) given below, DE || BC and
F E BD = CE. Prove that ABC is an isosceles
triangle.
(b) In figure (ii) givne below, AB || DE and BD
|| EF. Prove that DC2 = CF × AC.
Sol. (a) Given : In the figure,
Q R
DE || BC and BD = CE
PQ 1.28 128 64 To prove : ABC is an isosceles triangle
= = = Proof : In ABC, DE || BC
PE 0.18 18 9
PR 2.56 256 64 A
= = =
PF 0.36 36 9
PQ PR D E
=
PE PF
 EF || QR
4. A and B are respectively the points on the
sides PQ and PR of a triangle PQR such that
PQ = 12.5 cm, PA = 5 cm, BR = 6 cm and B C
(i)
PB = 4 cm. Is AB || QR? Give reasons for
your answer. AD AE
Sol. In PQR, A and B are points on the sides  =
DB EC
PQ and PR such that
But DB = EC (Given) ...(i)
PQ = 12.5 cm, PA = 5 cm, BR = 6 cm and
 AD = AE ...(ii)
PB = 4 cm
Adding (i) and (ii),
AD + DB = AE + EC
P
 AB = AC
 ABC is an isosceles triangle
(b) Given : In the given figure,
B A
AB || DE, BD || EF
To prove : DC2 = CF × AC
Proof : In ABC, DE || AB
DC CE
Q R  = ...(i)
CA CB
PQ 12.5 2.5 In CDE
Now = = EF || DB
PA 5 1
331 Arun Deep's Understanding Math-10

CF CE
= ...(ii)
CD CB

F
D E

A (ii) B
From (i) and (ii),
DC CF DC CF
=  =
CA CD AC DC
By cross multiplication,
DC2 = CF × AC
5. (a) In the figure (i) given below, CD || LA and DE || AC. Find the length of CL if BE = 4 cm and EC
= 2 cm.
AD AE
(b) In the given figure, D = E and = . Prove that BAC is an isosceles triangle.
BD EC

A A

D E

B E C L B C

(i) (ii)
Sol. (a) Given : CD || LA and DE || AC
Length of BE = 4 cm
Length of EC = 2 cm
Now, in BCA
DE || AC
BE BD
 =
BC BA
(Corallary of basic proportionality theorem)
332 Arun Deep's Understanding Math-10
 DE || BC
BE BD
 = AD = AE
BE  EC AB
 DB = EC [From (i)]
4 BD Adding, we get
 (4  2) = ...(i)
AB AD + DB = AE + EC
Now, In BLA  AB = AC
CD || LA  ABC is an isosceles triangle

BC BD 6. In the figure given below, A, B and C are


 = points on OP, OQ and OR respectively such
BL AB
that AB || PQ and AC || PR. show that BC ||
(Corallary of basic proportionality theorem) QR.
BC BD
 = P
BC  CL AB

6 BD A
= ...(ii)
6  CL AB
Combining eq. (i) and (ii), we get O

6 4 B C
=
6  CL 6 Q R
 6 × 6 = 4 × (6 + CL)
 24 + 4CL = 36 Sol. In the given figure, A, B, C are points on
 4CL = 36 – 24 OP, OQ and OR respectively and AB || PQ
and AC || PR
12 To prove : BC || QR
 CL = = 3 cm
4
Proof : In POQ,
 The length of CL = 3 cm AB || PQ
(b) Given : In the given figure, D = E
OA OB
AD AE  = BQ ...(i)
= AP
DB EC
Similarly in OPR
To prove : BAC is an isosceles triangle
AC || PR
Proof : In ADE,
OA OC
D = E (Given)  = ...(ii)
AP CR
 AD = AE (Sides opposite to equal angles)
From (i) and (ii),
In ABC,
OB OC
AD AE
= ...(i) BQ = CR
DB EC
333 Arun Deep's Understanding Math-10
Now in OQR (ii) Calculate LM : QR, given that BM : MC
OB = 1 : 2.
OC
BQ = CR 8. In the adjoining figure, AD is bisector of
BAC. If AB = 6 cm, AC = 4 cm and BD
 BC || QR
= 3 cm, find BC.
7. ABCD is a trapezium in which AB || DC and (a) (b)
its diagonals intersect each other at O. Using
Basic Proportionality theorem, prove that B
AO CO
= .
BO DO
Sol. Given : ABCD is a trapezium in which AB ||
DC
Its diagonals AC and BD intersect each other
at O
Sol. (a) Given : AB CR and LM QR.
D C Also BM : MC = 1 : 2
To Prove :
BM AL
(i) =
MC LQ
(ii) to calculate LM : QR
Proof : (i) In  ARQ
 LM QR
A B
AM AL
 = ....(i)
AO CO MR LQ
To prove : = Now, in  AMB and  MCR
BO DO
Proof : In OAB and OCD, AMB = CMR
AOB = COD (vertically opposite angles)
(Vertically opposite angles) MBA = MCR (Alternate angles)
OAB = OCD (Alternate angles) [  AB CR (given)]
and OBA = ODC (Alternate angles)   AMB and  MCR
 OAB ~ OCD
AM BM
 = ....(ii)
OA OB MR MC
 =
OC OD From (i) and (ii), we get
BM AL
AO CO =
 = (By alternendo) MC LQ
OB DO
(ii) From (2),
P.Q. (a) In the figure (1) given below, AB || CR
AM BM AM 1
and LM QR. =  = ....(iii)
MR MC MR 2
BM AL
(i) Prove that =  BM : MC 1: 2  BM  1 
MC LQ  MC 2 
334 Arun Deep's Understanding Math-10
 LM QR (given) 1 = 3 (corresponding angles) ....(iii)
AM LM From (i), (ii) and (iii), we get
 = ...(iv)
MR QR 3 = 4
AR QR  AC = AE ....(iv)
or, =
AM LM In  BCE , DA CE,
AM  MR QR BD AB
or, =  =
AM LM DC AE
MR QR BD AB
or, 1 + =  = (  From (4), AC = AE)
AM LM DC AC
2 QR 3 6
or, 1 + =  =  3 × 4 = 6 × DC
1 LM DC 4
 AM 1 MR 2  3 4
 From (iii ), MR  2  AM  1   DC = =2
  6
 BC = BD + DC = 3 cm + 2 cm = 5 cm Ans.
3 QR
or, =
1 LM EXERCISE 13.3
LM 3
or, = 1. Given that s ABC and PQR are similar.
QR 1
LM Find :
 = 1 : 3 Ans.
QR (i) The ratio of the area of  ABC to the area
(b)Given :  ABC , AD is (internal) bisector of of  PQR if their corresponding sides are
in the ratio 1:3.
BAC .
(ii) the ratio of their corresponding sides if area
AB = 6 cm, AC = 4 cm and 3 D = 3 cm
of  ABC : area of  PQR = 25 : 36.
To calculate : The value of BC.
Sol. (i) ...  ABC ~  PQR
Construction : Through C, draw a st. line CE
DA meeting BA produced in E area of  ABC BC2
 area of  PQR 
Now, in  ABC , QR 2
As AD is bisector of A , (By theorem 15·1)
But BC = QR = 1:3
E
area of  ABC (1) 2 1
A 3
 area of  PQR  
(3) 2 9
1 2
Hence area of  ABC : area of  PQR
4
=1:9
(ii) ..  ABC   PQR
.
B D C
area of  ABC BC2
1 = 2 ....(i) 
area of  PQR QR 2
 CE DE (By construction) and AC cuts
them, (By theorem 15·1)
But area of  ABC = area of  PQR
2 = 4 (Alternate angles) ....(ii)
Again CE DA and BE cuts them, = 25 : 36
335 Arun Deep's Understanding Math-10
Sol. Let ABC   DEF, AL and DM are their

BC2

25

BC FG IJ 2  FG 5 IJ 2 altitudes
QR 2 36 QR H K H 6K then area of  ABC = 36 cm2
BC 5 area of DEF = 25 cm2 and AL = 2·4 cm.
  Let DM = x
QR 6
Now ...  ABC   DEF
 BC : QR = 5:6
2.  ABC  DEF. If area of  ABC = 9 sq. area of  ABC AL2
 
cm., area of  DEF = 16 sq. cm and area of  DEF DM 2
BC = 2·1 cm., find the length of EF.
Sol. Let EF = x 36 ( 2. 4 ) 2
 
...  ABC   DEF, 25 x2
 36 x = (2·4)2 × 25
2
area ABC BC 2 9 BC2
   
area DEF EF 2 16 EF2 ( 2. 4 ) 2 × 25 576 × 25 16
 x2   
36 100 × 36 4
( 2.1) 2 9 2.1 3
    = 4 = (2)2
x2 16 x 4
 x = 2 cm
(Taking square root) Hence altitude of the other triangle
4 × 2.1 = 2 cm Ans.
 3 x = 4 × 2·1  x 
3 5. (a) In the figure, (i) given below, PB and
 x = 2·8 QA are perpendiculars to the line segment
Hence EF = 2·8 cm Ans. AB. If PO = 6 cm, QO = 9 cm and the
3.  ABC   DEF. If BC = 3 cm, EF = 4 cm area of  POB = 120 cm2, find the area
and area of  ABC = 54 sq. cm. Determine of  QOA. (2006)
the area of  DEF.
Sol. ...  ABC   DEF P

area of ABC BC2


 
area of DEF EF 2 A
O B
54 ( 3) 2
 
area of DEF ( 4 ) 2
Q
54 9
 
area of DEF 16 Sol. In  AOQ and  BOP, we have
54 × 16  OAQ =  OBP [Each = 90º]
 area of  DEF   6 ×16
9  AOQ =  BOP
= 96 cm. Ans. [Vertically opposite angles]
4. The area of two similar triangles are
  AOQ ~  BOP [A.A. similarity]
36 cm2 and 25 cm2. If an altitude of the
first triangle is 2·4 cm, find the Area of ΔAOQ OQ 2
corresponding altitude of the other triangle.  Area of ΔBOP =
PO 2
336 Arun Deep's Understanding Math-10
AB = 6.5 cm and OD = 2.8 cm.
P

m
A

2 .8 c
O B

C
10 cm O 5 cm
Q
A

[ Area's of similar triangles are

6.5
proportional to the squares of their

cm
corresponding sides]

Area of AOQ 9 2


B
=
120 6 2
To prove : (i)  OAB ~  OCD
Area of AOQ 81 (ii) Find CD and OB
 =
120 36 (iii) Find the ratio of areas of  OAB and
81 120  OCD.
 Area of  AOQ = = 9 × 30 cm2
36 Proof : In the  OAB and  OCD,
= 270 cm2 Ans.
(i)   AOB =  COD
(b) In the figure (ii) given below, AB || DC.
(Vertically opposite angles)
AO = 10 cm, OC = 5cm, AB = 6.5 cm
and OD = 2.8 cm.  OAB =  OCD (Alternate angles)
(i) Prove that  OAB ~  OCD.  OBA =  ODC (Alternate angles)
(ii) Find CD and OB.   OAB ~  OCD (AAA axiom)
(iii) Find the ratio of areas of  OAB and
OA OB AB
 OCD. (ii)   
OC OD CD
D 10 OB 6.5
  
5 2.8 CD
OB 10
C (a) 
O 2.8 5
A
10
 OB = × 2.8 = 5.6 cm
5
 OB = 5.6 cm
6.5 10
B (b) =
CD 5
Sol. Given : In the figure AB || CD 6.5 5
 CD =
AO = 10 cm, OC = 5 cm, 10
337 Arun Deep's Understanding Math-10

32.5 Sol. (a) In the figure,


CD = = 3.25 cm
10 DE || BC
(iii)   OAB ~  OCD (Proved)  D = B and E = C
(Corresponding angles)
ar ΔOAB
 Now in  ADE and ABC,
ar ΔOCD 
D = B, E = C (proved)
AB 2
6.5 2
A = A (Common)
= 2 =
CD 3.252   ADE   ABC (AAA postulate)

6.5  6.5  area of ADE


=
3.25  3.25 area of ABC

2 2 4
= = ( DE ) 2
1 1 
( BC) 2
ar(OAB) : ar(  OCD) = 4 : 1 Ans.
28
6. (a) In the figure (i) given below, DE ||  area of ABC
BC. If DE = 6 cm, BC = 9 cm and area
of  ADE = 28 sq. cm, find the area of 
( 6) 2 36
ABC.  
( 9) 2 81

28 × 81
 area of  ABC   63
36
 Area of  ABC = 63 cm2
(b) In the figure, DE || BC
 D = B and E = C
(Corresponding angles)
(b) In the figure (iii) given below, DE || BC Now in  ADE and  ABC
and AD : DB = 1 : 2, find the ratio of the D = B, E = C (proved)
areas of  ADE and trapezium DBCE.
 A = A (Common)
  ADE   ABC (AAA postulate)

AD 1
But 
DB 2

DB 2
 
AD 1
Adding 1 both sides
338 Arun Deep's Understanding Math-10

DB 2 AD + DB 2  1 AD DE
1 1     
AD 1 AD 1 AD + BD BC
AB 3 AD 1
    1
BD
AD 1 AB 3
2 DE
  ADE   ABC  1 = (... BC = 4·5)
BD  BD 4 .5
 area of ADE  AD 
2 1FG IJ 2

1 2
area of  ABC AB 2 3 HK 9
1
 area of  ABC = 9 area of  ADE BD DE
2  .
 area of trapezium DBCE 
3 45
BD
 area of ABC – area of ADE 2
= 9 area of ADE – area of ADE
= 8 area of ADE 1 2 DE
   
2 3 4 .5
area of ADE 1
 area of trapezium DBCE  8 1 DE
  
3 4 .5
 area of  ADE : area of trepezium DBCE
= 1 : 8 Ans. 4 .5
 DE = = 1·5 cm Ans.
7. In the given figure, DE || BC. 3
(i) Prove that ADE and ABC are similar. (iii) Area of ABC = 18 cm2
1 area of ΔADE DE 2
(ii) Given that AD  BD , calculate DE if  
2 area of ΔABC BC 2
BC = 4·5 cm.
(Area of similar triangles are proportional
to the square of their corresponding
sides)
2
area of ΔADE  DE 
 
18  BC 

2
area of ΔADE  AD 
  
(iii) If area of  ABC = 18cm2, find area of 18  AB 
trapezium DBCE.
2
Sol. (i) Given : In  ABC, DE || BC. area of ΔADE  1  1
   
To prove :  ADE ~  ABC 18 3 9
Proof : In  ADE and  ABC,
[proved in (ii)]
A = A (common)
ADE = ABC(corresponding angles) 1
 area of ADE = 18 × =2
ADE ~ ABC. (AA axiom) 9
(ii) .  ADE ~ ABC
. .
 area of tripezium DBCE
AD AB DE = area ABC – area ADE
  
AB AC BC = 18 – 2 = 16 cm2 Ans.
339 Arun Deep's Understanding Math-10
8. In the given figure, AB and DE are area of ΔABC AB2
perpendicular to BC. (iii) area of ΔDEC =
(i) Prove that ABC ~ DEC DE 2
(ii) If AB = 6 cm: DE = 4 cm and AC = 15 cm, 62 36 9
calculate CD. = 2 = 16 = =9:4
4 4
(iii) Find the ratio of the area of ABC : area of
9. In the adjoining figure, ABC is a triangle.
DEC.
AD 3
A DE is parallel to BC and = .
DB 2
AD DE
D (i) Determine the ratios , .
AB BC
(ii) Prove that  DEF is similar to  CBF..
EF
B C Hence, find .
E FB
Sol. (iii) What is the ratio of the areas of  DEF
(i) To prove : ABC ~ DEC and  CBF ? (2007)
In ABC and DEC A

A
D E

D F

B C

B C AD 3
E Sol. (i) = (Given)
DB 2
ABC = DEC = 90º DB 2
C = C (common)  =
AD 3
 ABC ~ DEC (by AA axiom)
DB 2
AC AB or +1= +1
(ii) = AD 3
CD DE
(Corresponding sides of similar triangles are DB  AD 23
or =
proportional) AD 3
15 6 AB 5 AD 3
CD
=
4 or =  =
AD 3 AB 5
15  4 In  ADE and  ABC
 CD =
6  ADE =  B (Corresponding  s)
CD = 10 cm  AED =  C (Corresponding  s)
340 Arun Deep's Understanding Math-10
 By AA similarity
 ADE ~  ABC
AD DE DE 3
 =  =
AB BC BC 5
(ii) In  DEF and  CBF

E
D 1 5 3 Sol.(i) In PMN and PQR,
F
6 (1) PMN = PQR (corresponding angles)

B
4 2
C
(2) PNM = PRQ (corresponding angles)
 PMN ~ PQR (angle-angle similarity)
1 = 2 (Alternate  s)
3 = 4 (Alternate  s) PM MN
  (CSST are proportional)
5 = 6 (Vertically opp.  s) PQ QR
  DEF ~  CBF
2 MN
EF DE 3  
= = 5 QR
FB BC 5
(iii) As the ratio of the area of two similar (ii) In OMN and ORQ,
triangles is equal to the ratio of the square (1) OMN = ORQ (int. alternate angles)
of any two corresponding sides.
(2) MON = ROQ (vertically opp. angles)
Area of ΔDFE DE 2  OMN ~ ORQ (by angle-angle test)
 Area of ΔBFC =
BC 2
Area of (OMN) MN 2
 DE 
2
3 9
2
(iii) 
=   =   = Area of (ORQ) RQ 2
 BC  5 25
Ratio of areas of similar triangles
10. In PQR, MN is parallel to QR and
PM 2 Area of ( OMN) 22 4
 .  = 2  = 4 : 25
MQ 3 Area of ( ORQ) 5 25
Ans.
MN 11. In ABC, AP : PB = 2 : 3. PO is parallel to
(i) Find .
QR BC and is extended to Q so that CQ is parallel
to BA. Find :
(ii) Prove that OMN and ORQ are
similar. (i) area APO : area ABC.
(iii) Find Area of OMN : Area of ORQ. (ii) area APO : area CQO. (2008)
341 Arun Deep's Understanding Math-10
A area (APO) : area (CQO) = 4 : 9
12. (a) In the figure (i) given below, ABCD
P O Q
is a trapezium in which AB | | DC and
AB = 2 CD. Determine the ratio of the
areas of  AOB and  COD.
B C
(b) In the figure (ii) given below, ABCD is a
Sol. In the figure, parallelogram. AM  DC and AN  CB. If
PQ || BC and PO is produced to Q such that AM = 6 cm, AN = 10 cm and the area of
CQ || BA parallelogram ABCD is 45 cm2, find
and AP : PB = 2 : 3.
(i) AB (ii) BC
A
(iii) area of  ADM : area of  ANB.
P O Q (c) In the figure (iii) given below, ABCD is a
parallelogram. E is a point on AB, CE
intersects the diagonal BD at O and
EF || BC. If AE : EB = 2 : 3, find
B C
(i) EF : AD
(i) Now in APO and ABC (ii) area of  BEF : area of  ABD
A = A (Common) (iii) area of  ABD : area of trap. AFED
APO = ABC (Corresponding angle) (iv) area of  FEO : area of  OBC.
 APO ~ ABC (AA axiom)
Areas of the similar triangle are proportional
to the square of their corresponding sides

area ΔAPO AP 2 AP 2
 area ΔABC = =
AB2 AP  PB2
22 4 4
=
2  3 2 =
52 =
25
area (APO) : area (ABC) = 4 : 25
(ii) In APO and CQO
AOP = COQ
(Vertically opposite angles) Sol. (a) In trapezium ABCD, AB || DC.
APQ = OQC (Alternate angles)  OAB = OCD [alternate angles]
 APQ ~ CQO
OBA = ODC
area ΔAPO AP AP 2 4
2 2 2
  AOB   COD
 = 2 = 2 = =
area ΔCQO CQ PB 32 9 area of  AOB AB2
 
( PB = CQ) area of COD CD 2
342 Arun Deep's Understanding Math-10

( 2 CD) 2 . . EF BE
 ( . AB = 2 CD)  
CD 2 AD AB
AE 2
4 CD 2 4 But 
 2
 EB 3
CD 1
AE 2
 area of  AOB : area of  COD = 4 : 1  1 1
EB 3
(b) In || gm ABCD, AM  DC and AN  CB
AE  EB 2  3
Now area of || gm ABCD = DC × AM or BC  
× AN EB 3
 DC × AM = BC × AN = area of || gm AB 5 BE 3
    
 DC × 6 = BC × 10 = 45 EB 3 AB 5
45 15 EF BE 3
(i)  DC   = 7·5 cm   
6 2 AD AB 5
 AB = 7·5 cm (... AB = DC)  EF : AD = 3 : 5
45 (ii) ...  BEF   ABD
(ii) and BC   4 .5 cm.
10
area of BEF ( EF ) 2
(iii) Now in  ADM and  ABN 
area of ABD ( AD) 2
D = B (opposite angles of a || gm)
M = N (each 90°) ( 3) 2 9
 
  ADM   ABN ( 5) 2 25
 area of  BEF : area of  ABD = 9 : 25
area of  ADM AD2
 
area of  ABN AB2 area of ABD 25
(iii)  (from (ii))
area of BEF 9
BC2 ( 4 .5) 2
  25 area of  BEF = 9 area of  ABD
AB2 ( 7.5) 2
 25 (area of  ABD – area of trap AEFD)
20 .25 2025 = 9 area of  ABD
 . 
56 25 5625
 25 area of  ABD – 25 area of AFED
81 9 = 9 area of  ABD
 
225 25
 25 area of trap AEFD = 25 area of
 area of  ADM : area of  ABN
 ABD – 9 area of  ABD
= 9 : 25
 25 area of AEFD = 16 area of  ABD
(c) In || gm ABCD, E is a point on AB,
area of ABD 25
CE intersects the diagonal BD at O  
area of trap AEFD 16
EF || BC and AE : EB = 2 : 3
In  ABD, EF || BC or AD  area of ABD : area of trap AEFD
= 25 : 16.
AB AD
(i)   (iv) In  FEO and  OBC
BE EF
343 Arun Deep's Understanding Math-10
EOF =BOC
1
(Vertically opposite angles) BP  QN
area ΔBPQ 2
F = OBC (alternate angles) =
area ΔCPQ 1
  FEO   OBC PC  QN
2
area of FEO EF 2
 
area of OBC BC 2 BP 1
= = (given)
PC 2
EF 2 9
  [form (i)]
AD2 25 1
(i) area BPQ = area CPQ
 area of  FEO : area of  OBC = 9 : 25 2
13. In the adjoining figure, ABCD is a 1
parallelogram. P is a point on BC such that = × 20 cm2 = 10 cm2
2
BP : PC = 1 : 2 and DP produced meets AB
produced at Q. (ii) Now in CDP and BQP,
If area of CPQ = 20 cm , find
2
CPD = QPB (Vertically opposite angles)
PDC = PQB (Alternate angles)
A B Q  CDP ~ BQP (AA axiom)

P
area CDP PC 2
 area BQP =
BP 2

D C area CDP 22


 =
(i) area of BPQ. area BQP 12
(ii) area CDP.
area CDP 4
(iii) area of || gm ABCD.  =
area BQP 1
Sol. In the figure, ABCD is a parallelogram. P is a
point on BC such that BP : PC = 1 : 2 and DP  area CDP = 4 × area BQP
is produced to meet ABC produced at Q.
= 4 × 10 = 40 cm2
Area of CPQ = 20 cm2
(iii) Now area of || gm ABCD = 2 area DCQ
N {DCQ and || gm ABCD are one the same
A B base and between the same parallels}
Q
= 2 area (DCP + CPQ)
P = 2 (40 + 20) cm2
= 2 × 60 cm2 = 120 cm2
D C 14. (a) In the figure (i) given below, DE || BC
and the ratio of the areas of  ADE and
Draw QN  CB (Produced)
trapezium DBCE is 4 : 5. Find the ratio of
344 Arun Deep's Understanding Math-10
DE : BC. area of trap DBCE + area of  ADE

area of  ADE

5 4 9
 
4 4
area of  ABC 9
 area of  ADE  4

area of  ADE 4
 area of  ABC  9
(b) In the figure (ii) given below, AB || DC and
AB = 2 DC. If AD = 3 cm, BC = 4 cm and ( DE ) 2 4
AD, BC produced meet at E, find Now from (i) 2

( BC) 9
(i) ED (ii) BE (iii) area of  EDC : area of
trapezium ABCD. (2)2

( 3) 2

DE 2
 
BC 3
 DE : BC = 2 : 3
(b) In the figure, DC || AB, AB = 2 DC,
AD = 3 cm, BC = 4 cm
In  EAB, DC || AB
Sol. (a) In  ABC, DE || BC
EA EB AB 2 DC  2
Now in  ABC and  ADE    
DA CB DC DC 1
A = A (common)
(i)  EA = 2, DA = 2 × 3 = 6
D = B and E = C
ED = EA – DA = 6 – 3 = 3 cm
(Corresponding angles)
  ADE   ABC EB 2
(ii)   EB = 2 CB = 2 × 4 = 8 cm
CB 1
area of ADE ( DE ) 2  BE = 8 cm
 area of ABC  ...(i)
( BC) 2
(iii) ... In  EAB, DC || AB
area of ADE 4   EDC   EAB
But 
area of trap DBCE 5
area of  EDC DC 2
 

area of trap DBCE

5 area of  ABE AB2
area of  ADE 4
DC 2
area of trap DBCE 
 1 ( 2 DC) 2
area of  ADE
DC 2 1 1
5   
 1 (Adding 1 both sides) 4 DC 2 4 4
4
345 Arun Deep's Understanding Math-10
 area of ABE = 4 area of  EDC.
 area of  EDC + area of trap ABCD = 4
area of  EDC
 area of trap ABCD = 4 area of  EDC –
area of  EDC
 area of trap ABCD = 3 area of  EDC
area of  EDC 1
 
area of trap ABCD 3
 area of  EDC : area of trap. ABCD = 1 : 3 PAB = PCD (alt. angles)
15. (a) In the figure given below, ABCD is a   APB   CPD (AA postulate)
trapezium in which DC is parallel to AB.
BP AB BP 9
If AB = 9 cm, DC = 6 cm and BD = 12    
cm., find PD CD 12  BP 6
(i) BP (ii) the ratio of areas of APB and  6 BP = 108 – 9 BP
DPC.  6 BP + 9 BP = 108
 15 BP = 108

108
 BP   7 .2 cm.
15
(ii) Again ...  APB   CPD
(b) In the figure given below, ABC = DAC area of  APB AB2
and AB = 8 cm, AC = 4 cm, AD = 5 cm.  
area of  CPD CD 2
(i) Prove that ACD is similar to BCA
(ii) Find BC and CD ( 9) 2 81 9
 2
 
(iii) Find the area of ACD : area of ABC. ( 6) 36 4
(b) In ACD and BCA
A
A
m

4c
8c

5 cm

B D C
B D C
Sol. (a) In trapezium ABCD, DC || AB
C = C (common)
AB = 9 cm, DC = 6 cm, BD = 12 cm
ABC = CAD
(i) In  APB and CPD
(Given)
APB = CPD
 ACD  BCA (by AA axiom)
(Vertically opposite angles)
346 Arun Deep's Understanding Math-10

AC CD AD 5 cm
= = T Q
P
BC CA AB 2c
m
S
4 CD 5
 = =
BC 4 8

4 5 R
 =
BC 8 (i) In PQR and PST

48 32 P = P (common)
BC = = = 6.4 cm PST = PQR = 90° (each)
5 5
 PQR ~ PST By (A.A. axiom)
CD 5
 =
4 8  Area of PQR PQ 2 52 25 
    
5 5  Area of PST PS2 22 4 
 CD = ×4= = 2.5cm
8 2
(iii)  ACD  BCA Area of PQR 25 25
 
Area of SRQT 25  4 21
area(ACB) AC 2
 area(BCA)  17. ABC is a right angled triangle with ABC
AB2
= 90°. D is any point on AB and DE is
perpendicular to AC. Prove that:
( 4) 2 16 1
 2   (i) ADE ~ ACB.
(8) 64 4
(ii) If AC = 13 cm, BC = 5 cm and
area ( Δ ACD) : area ( Δ ABC) = 1 : 4 AE = 4 cm. Find DE and AD.
16. In the given figure, PQR = PST =
A
90°, PQ = 5 cm and PS = 2 cm.
E
(i) Prove that PQR ~ PST.
(ii) Find Area of PQR : Area of quadrilateral D
SRQT.

T
P Q B C
Sol. (i) Consider ADE and ACB
S A = A (Common)
mB = mE = 90°
Thus by angle-angle similarity, ACB ~
ADE
R (ii) Consider ADE and ACB
Since they are similar triangles, the sides
Sol. Given PQ = 5 cm and PS = 2cm are proportional
347 Arun Deep's Understanding Math-10
Thus, we have,
1 10
= × 5 × 12 –
AE AD DE 2 3
= = ...(1)
AB AC BC
10
Consider ABC = 30 –
3
By applying Pythagoras Theorem, we have,
90  10 80
AB2 + BC2 = AC2 = = cm2
3 3
 AB2 + 52 = 132
Thus ratio of areas of ADE to quadrilateral
 AB2 + 25 = 169
 AB2 = 169 – 25 = 144 10
3 1
 AB = 12 cm BCED = 80 =
8
From equation (1), we have 3
4 AD DE 18. Two isosceles triangles have equal vertical
= =
12 13 5 angles and their areas are in the ratio
7 : 16. Find the ratio of their corresponding
1 AD height.
 =
3 13
Sol. In two isosceles s ABC and DEF
13
 AD = cm = 4.33 cm
3

4 DE
Also =
12 5

20 5
 DE = = cm = 1.67 cm
12 3
(iii) We need to find the area of ADE and
quadrialteral BCED
1
Area of ADE = × AE × DE
2
A = D (given)
1 5  B + C = E + F
= ×4×
2 3 But B = C and E = F
(opposite angles of equal sides)
10
= cm2  B = E and C = F
3
  ABC   DEF
Area of quad. BCED = Area of ABC –
Area of ADE area of  ABC AL2

area of  DEF DM 2
1 10
= × BC × AB – (Cor. of theorem 13·1)
2 3
348 Arun Deep's Understanding Math-10
(i) the distance of a diagonal of the plot in
AL2 7 AL 7
 2
   km.
DM 16 DM 4
(ii) the area of the plot in sq. km.
Hence AL : DM = 7 :4
1
19. On a map drawn to a scale of 1 : 250000, Sol. Scale factor (k ) 
50000
a triangular plot of land has the following
measurements : Measurements of plot ABCD on the map
AB = 3 cm, BC = 4 cm and ABC = 90°. are AB = 6 cm and BC = 8 cm.
Calculate
D C
(i) the actual length of AB in km.
(ii) the area of the plot in sq. km.
1 8cm
Sol. Scale factor k = 1 : 250000 
250000
Length on map,
A 6cm B
AB = 3 cm, BC = 4 cm
1
 Length of AB of Actual plot  (Length Diagonal AC  AB2  BC2
k
of AB on the map)
 (6) 2  (8) 2  36  64
= 250000 (3 cm)
250000 × 3  100  10 cm
 km  15  7 .5 km
100 ×1000 2
and area = AB × BC
1
(ii) Area of plot on the map  × AB × BC = 6 cm × 8 cm = 48 cm2
2
1 1
 × 3 × 4 = 6 cm2 (i) Now actual length of AC =
2 k
1 2 (length of AC on map)
 Area of actual plot  2 × 6 cm
k
50000 10
= (250000)2 × 6 cm2 = 50000 × 10 cm  km
100  1000
250000 × 250000 × 6
 km2 = 5 km.
100000 × 100000
25 75 (ii) Area of plot 
FG 1 IJ 2 (Area of plot on map)
 ×6  37.5 km2 Ans. H kK
4 2
20. On a map drawn to a scale of 1 : 50000, a = 50000 × 50000 (48 cm2)
rectangular plot of land, ABCD has the
50000  50000  48
following measurements AB = 6 cm and  km 2
100000  100000
BC = 8 cm.
Calculate : = 12 km2 Ans.
349 Arun Deep's Understanding Math-10
21. A map of a square plot of land is drawn to Sol.
a scale 1 : 25000. If the area of the plot in
the map is 72 cm2, find : Height of model 1
(i) Height of actual building =
30
(i) the actual area of the plot of land.
(ii) the length of the diagonal in the actual plot 80 1
of land. =
H 30
1  H = 2400 cm = 24 m
Sol. Let k =
25000 3
Volume of model  1 
1 1 (ii) =  
Volume of tank  30 
 k=  k2 =
25  103 625  106
V 1
 =
Area of the map 27 27000
= k2
Area of the actual plot of land
1
 V= m3 = 1000 cm3
1000
72 cm 2
 23. A model of a ship is made to a scale of
Area of the actual plot of land
1 : 200.
1 (i) If the length of the model is 4 m, find the
= length of the ship.
625  106
(ii) If the area of the deck of the ship is
Area of the actual plot of land = 72 × 625 160000 m2, find the area of the deck of the
× 106 cm2 = 45000 × 106 cm2 model.
= 45000 × 106 × 10–10 km2 = 4.5 km2 (iii) If the volume of the model is 200 litres, find
the volume of the ship in m3.
1
Area of square = × (Diagonal)2 (1000 litres = m3)
2
Sol. Scale = 1 : 200
 (Diagonal)2 = 2 × 45 km2 = 9 km2 (i) Length of a model of ship = 4 m
 Diagonal = 9 km
2
4  200
 Length of the ship = = 800 m
 Diagonal = 3 km 1
22. The model of a building is constructed with (ii) Area of deck of ship = 160000 m2
the scale factor 1 : 30.  Area of deck of the model
(i) If the height of the model is 80 cm, find the 2
 1  2
actual height of the building in metres. = 160000 ×   m
 200 
(ii) If the actual volume of a tank at the top of
the building is 27 m3, find the volume of the 1
tank on the top of the model. (2009) = 160000 × = 4 m2
40000
350 Arun Deep's Understanding Math-10
(iii) Volume of the model of the ship = 200 l value of x is
3 (a) 2.25 cm (b) 4 cm
 200  (c) 4.5 cm (d) 5.25 cm
Volume of ship = 200 ×   l
 1 
A 6 cm C
= 200 × 8000000 l
200  8000000 3
= m = 1600000 m3
1000

cm
4 .5
MULTIPLE CHOICE QUESTIONS
Choose the correct answer from the given B
four options (1 to 22): R
1. In the given figure, ABC ~ QPR. Then
R is

m
x

3c
(a) 60° (b) 50°
(c) 70° (d) 80°
A P
Q

70° Sol. In the given figure,


ABC ~ QPR
AC BC 6 4.5
50°  =  =
QR PR 3 x
B C
3  4 .5
R  x= = 2.25
6
 x = 2.25 cm (a)
3. In the given figure, two line segments AC
and BD intersect each other at the point P
Q such that PA = 6 cm, PB = 3 cm, PC = 2.5
cm, PD = 5 cm, APB = 50° and CDP =
30°. Then, PBA is equal to
(a) 50° (b) 30°
(c) 60° (d) 100°
P
Sol. In the given figure,
ABC ~ QPR A
D
 A = Q, B = P and C = R 6 cm
5 cm
But A = 70°, B = 50° 30°
P
 C = 180° – (70° + 50°) 50°
= 180° – 120° = 60° 2.5 cm
3 cm C
C = R
 R = C = 60° B
2. In the given figure, ABC ~ QPR. The
351 Arun Deep's Understanding Math-10
Sol. In the given figure two line segments AC DEF are
and BD intersect each other at P and (a) similar (b) congruent
PA = 6 cm, PB = 3 cm, PC = 2.5 cm, (c) both similar and congruent
PD = 5 cm, APB = 50° and CDP = 30° (d) neither similar nor congruent
In APB and CPD
A
AP 6 BP 3 6
= and = =
PD 5 CP 2.5 5
AP BP
= and APB = CPD
PD CP
(Vertically opposite angles) F E
 APB ~ CPD
 A = D = 30
and third PBA = 180° – (50° + 30°)
= 180° – 80° = 100° (d)
P.Q. If in two triangles ABC and PQR, B D C
AB BC CA
= = Sol. D, E and F are the mid-points of the sides
QR PR PQ , then BC, CA and AB of ABC then two triangles
(a) PQR ~ CAB (b) PQR ~ ABC ABC and DEF are similar. (a)
(c) CBA ~ PQR (d) BCA ~ PQR 5. The given figure, AB || DE. The length of
Sol. In two ABC and PQR CD is
(a) 2.5 cm (b) 2.7 cm
AB BC CA PQ QR RP
QR = = PQ  = = 10
PR CA AB BC (c) cm (d) 3.5 cm
Then PQR ~ CAB (a) 3
4. In triangles ABC and DEF, B = E, F =
B
C and AB = 3DE, then the two triangles
are 4.5
(a) congruent but not similar cm
5cm
(b) similar but not congruent
(c) neither congruent nor similar x E
(d) congruent as well as similar C
A 3cm
Sol. In ABC and DEF
B = E, F = C D
AB = 3DE
Sol. In the given figure, AB || DE
Two angles of the one triangles are equal to
 ABC ~ DCE
corresponding two angles of the other
But sides are not equal AB BC
 =
 Triangles are similar but not congruent. (b) DE CD
P.Q. In the given figure, if D, E and F are mid-
5 4.5 3  4.5
points of the sides BC, CA and AB  =  CD = = 2.7 cm (b)
respectively, then the two triangles ABC and 3 CD 5
352 Arun Deep's Understanding Math-10

AB BC Sol. In the given figure, DE || BC


P.Q. If in triangles ABC and DEF, = ,
DE FD AD AE 3 x
 =  =
then they will be similar when DB EC 4 3
(a) B = E (b) A = D
3 3 9
(c) B = D (d) A = F  x= = = 2.25 cm (b)
4 4
Sol. In two triangles ABC and DEF
8. In the given figure, PQ || CA and all lengths are
AB BC given in centimetres. The length of BC is
=
DE FD (a) 6.4 cm (b) 7.5 cm
They will be similar if their included angles (c) 8 cm (d) 9 cm
are equal
C
 B = D (c)
6. If PQR ~ ABC, PQ = 6 cm, AB = 8 cm Q
and perimeter of ABC is 36 cm, then
perimeter of PQR is 5
(a) 20.25 cm (b) 27 cm
(c) 48 cm (d) 64 cm
Sol. PQR ~ ABC
PQ = 6 cm, AB = 8 cm A 2.4 P 4 B
Perimeter of ABC = 36 cm Sol. In the given figure,
 Let perimeter of PQR be x cm PQ || CA
PQ Perimeter of ΔPQR Let BC = x
 = Perimeter of ΔABC
AB BQ BP 5 4
 =  =
6 x 6  36 QC PA x5 2.4
 = x= = 27 cm  4x – 20 = 12  4x = 12 + 20 = 32
8 36 8
Perimeter of PQR = 27 cm (b) 32
7. In the given figure, DE || BC and all  x= =8
4
measurements are in centimetres. The length
 BC = 8 cm (c)
of AE is
9. In the given figure, MN || QR. If PN = 3.6 cm,
(a) 2 cm (b) 2.25 cm
NR = 2.4 cm and PQ = 5 cm, then PM is
(c) 3.5 cm (d) 4 cm
A (a) 4 cm (b) 3.6 cm
(c) 2 cm (d) 3 cm
P
3

F E
M N
4
3

B C Q R
353 Arun Deep's Understanding Math-10
Sol. In the given figure, MN || QR
2  7.5 15
PN = 3.6 cm, NR = 2.4 cm and PQ = 5 cm  x= = =3
5 5
Let PM = x cm
 DE = 3 cm (b)
PM PN x 3.6 10. It is given that ABC ~ PQR with
 MQ =  =
NR 5 x 2.4
BC 1 area of PQR
x 36 QR = 3 , then area of ABC is equal to
 =  36(5 – x) = 24x
5 x 24 (a) 9 (b) 3
 180 – 36x = 24x
1 1
 180 = 24x + 36x  60x = 180 (c) (d)
3 9
180
 x= =3 Sol. ABC ~ PQR
60
 PM = 3 cm (d) BC 1
P.Q. D and E are respectively the points on the QR = 3
sides AB and AC of a ABC such that AD =
2 cm, BD = 3 cm, BC = 7.5 cm and DE || area of PQR QR 2
 area of ABC =
BC. Then the length of DE is BC 2
(a) 2.5 cm (b) 3 cm
(c) 5 cm (d) 6 cm (3) 2 9
= 2 = =9 (a)
Sol. D and E are the points on sides AB and AC (1) 1
of ABC, AD = 2 cm, BD = 3 cm,
11. If the areas of two similar triangles are in the
BC = 7.5 cm
ratio 4 : 9, then their corresponding sides are
A
in the ratio
(a) 9 : 4 (b) 3 : 2
2cm

(c) 2 : 3 (d) 16 : 81
D E Sol. Ratio in the areas of two similar triangle
x
=4:9
3cm

Ratio in their corresponding sides

B 7.5 C = 4 : 9 =2:3 (c)


DE || BC 12. If ABC ~ PQR, BC = 8 cm and
Let DE = x cm QR = 6 cm, then the ratio of the areas of
In ABC, DE || BC ABC and PQR is
 ADE ~ ABC (a) 8 : 6 (b) 3 : 4
(c) 9 : 16 (d) 16 : 9
AD DE
= Sol. ABC ~ PQR, BC = 8 cm, QR = 6 cm
AB BC

2 x 2 x area of ΔABC BC 2

23
=
7.5

5
=
7.5 area of ΔPQR = QR 2
354 Arun Deep's Understanding Math-10

82 64 16 BC2  9
2
BC 9
= 2 = = (d)  =    =
6 36 9 (6) 2  12  6 12
area of ΔABC 9 69 9
13. If ABC ~ QRP, area of ΔPQR = ,  BC = = = 4.5 cm (b)
4 12 2
AB = 18 cm and BC = 15 cm, then the length 15. In the given figure, DE || CA and D is a point
of PR is equal to on BD such that BD : DC = 2 : 1. The ratio
(a) 10 cm (b) 12 cm of area of ABC to area of BDE is
20 (a) 4 : 1 (b) 9 : 1
(c) cm (d) 8 cm
3 (c) 9 : 4 (d) 3 : 2
Sol. ABC ~ QRP
A
area of ΔABC 9
area of ΔPQR = 4 E

AB = 18 cm, BC = 15 cm2
A Q

B D C
18 Sol. In the given figure, DE || CA
D is a point on BC and BD : DC = 2 : 1
B 15 C R P  BD : BC = 2 : (2 + 1) = 2 : 3
 In ABC, DE || CA
Length of PR
 ABC ~ BDE
area of ΔABC 9 BC2
area of ΔPQR = = area of ΔABC BC 2 (3) 2 9
4 PR 2  area of ΔBDE = 2 = ( 2) 2 = 4
BD
9 15 2 3 15
 =  =  Ratio = 9 : 4 (c)
4 2 2 PR
PR 16. If ABC and BDE are two equilateral triangles
(Taking square root) such that D is mid-point of BC, then the ratio
15  2 of the areas of triangles ABC and BDE is
 PR = = 10 cm (a) (a) 2 : 1 (b) 1 : 2
3
14. If ABC ~ PQR, area of ABC = 81 cm2, (c) 1 : 4 (d) 4 : 1
area of PQR = 144 cm2 and QR = 6 cm, Sol. ABC and BDE are equilateral triangle
then length of BC is
A
(a) 4 cm (b) 4.5 cm
(c) 9 cm (d) 12 cm
E
Sol. ABC ~ PQR, area of ABC = 81 cm2 and
area of PQR = 144 cm2, QR = 6 cm, BC = ?
area of ΔABC BC 2 BC2 81
= 2 = 2 =
area of ΔPQR QR (6 ) 144 B D C
(given)  ABC ~ BDE
355 Arun Deep's Understanding Math-10
D is mid-point of BC 49 20
 E is mid-point of AB (a)
9
cm (b)
3
cm
1 (c) 15 cm (d) 22.5 cm
 DE || CA = CA
2 Sol. ABC ~ PQR
area of ABC = 54 cm2
area of ΔABC AC 2 AC 2 and of PQR = 24 cm2
area of ΔBDE = 2 = 2
ED 1  AD and PM are their median respectively
 AC 
2  PM = 10 cm
Let AD = x cm, then
AC2 4
= = =4:1 area of ΔABC AD 2 x2 54
1 1
AC2 area of ΔPQR = PM 2 = (10) 2 = 24
4
 Ratio = 4 : 1 (d) x2 2
54 9 3
17. The areas of two similar triangles are 81 cm2  = = =  
(10) 2 24 4 2
and 49 cm2 respectively. If an altitude of the
smaller triangle is 3.5 cm, then the x 3 3  10
corresponding altitude of the bigger triangle  = x= = 15 cm
10 2 2
is
 AD = 15 cm (c)
(a) 9 cm (b) 7 cm
(c) 6 cm (d) 4.5 cm CHAPTER TEST
Sol. Areas of two similar triangles are 81 cm2
1. In the given figure, 1 = 2 and 3 = 4.
and 49 cm2
Show that PT × QR = PR × ST.
Altitude of the smaller triangle is 3.5 cm
Sol. Given : In the given figure,
Let the altitude of bigger triangle is x, then
1 = 2 and 3 = 4
area of bigger triangle 81
area of smaller triangle = 49 P

(bigger altitude) 2
= 1 6 2
(smaller altitude) 2

81 x2 9 x
 = 2  = 3
49 (3.5) 7 3 .5 4
S R
9  3 .5 Q T
 x= = 4.5 cm
7
To prove : PT × QR = PR × ST
 Altitude of bigger triangle = 4.5 cm (d)
Proof : 1 = 2
18. Given ABC ~ PQR, area of ABC =
Adding 6 to both sides
54 cm2 and area of PQR = 24 cm2. If AD
and PM are medians of 's ABC and PQR 1 + 6 = 2 + 6
respectively, and length of PM is 10 cm, then SPT = QPR
length of AD is In PQR and PST
356 Arun Deep's Understanding Math-10
SPT = QPR (Proved)
3 = 4 (Given) A
 PQR ~ PST
PT ST
 = QR  PT × QR = PR × ST
PR F E
(By cross multiplication)
 PT × QR = PR × ST G
2. In the adjoining figure,
AB = AC. If PM  AB
and PN  AC, show that B C
PM × PC = PN × PB.
Sol. (a)Given : In following figure,
AED = ABC
Required : The values of x and y.
Sol. Given : In the given figure,
Now, in  ABC and  ADE
AB = AC, PM  AB and PN  AC
To prove : PM × PC = PN × PB AED = ABC (given)
Proof : In ABC, AB = AC A = A (common)
 B = C   ABC ~  ADE
Now in CPN and BPM, (By A.A. axiom of similarity)
N = M (Each 90°)
C = B (Proved) AD DE 3 y
 =  =
 CPN ~ BPM (AA axiom) AC BC 42 10

PC PN 3 y 10 3 30
 =  =  =y  y=
PB PM 6 10 6 6
 PC × PM = PN × PB  y=5 ....(1)
Hence PM × PC = PN × PB
AB BC 3 x 10
3. (a)In the figure (1) given below. AED = Also, =  =
AE DE 4 5
ABC. Find the values of x and y.
(b) In the figure (2) given below, medians BE (From (1), y = 5 cm)
and CF of a ABC meet at G. Prove that:  5(3 + x) = 40  15 + 5x = 40
(i) FGE ~ CGB (ii) BG = 2GE. 25
 5x = 40 – 15  5x = 25  x =
5
A
 x=5
3
4 Hence, value of x = 5 cm and y = 5cm Ans.
D
(b) Given —In ABC, BE and CF are the
y E
x
medians of sides AC and AB respectively
2
which intersect each at G.
10 To Prove— (i) FGE ~ CGB
B C (ii) BG = 2 GE
(1)
357 Arun Deep's Understanding Math-10
Proof— F and E are the mid points of (i) In  ABC, PQ || AC.
AC and AB respectively.
BQ BP PQ
 = =
1 BC AB AC
 FE || BC and FE = BC
2
PQ BP BP 3 3
= = = = =
FE 1 AC AB BP  AP 3+4 7
Or 
BC 2  PQ : AC = 3 : 7
Now in FGE and CGB, (ii) Now in  ARC and  PSQ.
FGE = BGC(vertically opposite angle) ARC = PSQ(Each 900)

FEG = GBC (Alternate angles) ACR = QPS(Alternate angles)


 FGE ~ CGB(AA postualte)  ARC ~ PSQ(AA Postualte)

GE FE 1 AC AR
   [From (i)]  =
PQ QS
BG BC 2
 BG = 2GE Q.E.D. AC 7
But = and QS = 6 cm
4. In the given figure, P is a point on AB such PQ 3
that PB : AP = 3 : 4 and PQ || AC.
AR
(i) Calculate PQ : AC.  =
6
(ii) If AR  CP, QS  CB and QS = 6 cm,
calculate the length of AR.  AR = × 6 = 14 cm Ans.

C 5. In a ABC, D and E are points on the sides


AB and AC respectively such that DE || BC.
If AD = 2.4 cm, AE = 3.2 cm, DE = 2 cm
and BC = 5 cm, find BD and CE.
R Q
Sol. Given : In ABC, D and E are the points on
S the sides AB and AC respectively
DE || BC
A P B AD = 2.4 cm, AE = 3.2 cm, DE = 2 cm, BC
= 5 cm
Sol. Given— In  ABC, P is a point on AB
such that AP : PB = 4:3 and PQ || AC is A
drawn meeting BC in Q. CP is joined and
QS  CP and AR  CP 2.4cm 3.2cm
To Find— D E
(i) Calculate the ratio between PQ : AC giving 2cm
reason.
(ii) In  ARC ARC= 900 and In  PQS,
PSQ = 900, if QS = 6 cm, calculate AR. B 5cm C
358 Arun Deep's Understanding Math-10
To find : BD and CE AE 3.3 3
Proof : In ABC and
EC
=
5.5
=
5
DE || BC
ΑD AE
AD AE DE =
 = = DB EC
AB AC BC
 DE || BC
2.4 3.2 2 7. If the areas of two similar triangles are 360
= = cm2 and 250 cm2 and if one side of the
AB AC 5
first triangle is 8 cm, find the length of the
2.4 2 2.4  5 corresponding side of the second triangle.
Now, =  AB = = 6 cm
AB 5 2 Sol. Let  ABC and  DEF are similar and area
of  ABC = 360 cm2
3.2 2 3. 2  5
and =  AC = = 8 cm
AC 5 2
 BD = AB – AD = 6 – 2.4 = 3.6 cm
and CE = AC – AE = 8 – 3.2 = 4.8 cm
6. In a ABC, D and E are points on the sides
AB and AC respectively such that AD = 5.7cm,
BD = 9.5cm, AE = 3.3cm and AC = 8.8cm.
Is DE || BC? Justify your answer.
Sol. In ABC, D and E are points on the sides
and area of  DEF = 250 cm2
AB and AC respectively
BC = 8 cm
AD = 5.7 cm, BD = 9.5 cm, AE = 3.3 cm
Let EF = x cm
and AC = 8.8 cm
area of  ABC BC 2
Now 
A area of  DEF EF 2
5 .7 c m
3 .3

360 (8) 2 64 360


   2 
8.8
cm

250 x 2 x 250
cm

D E
250 × 64 400
 x2  
9 .5 c m

360 9
400 20 2
B C  x  6
9 3 3

To prove : Whether DE || BC 2
 EF  6 cm Ans.
Proof : In ABC, 3
AD = 5.7 cm, BD = 9.5 cm, AE = 3.3 cm, 8. In the adjoining figure, D is a point on BC
AC = 8.8 cm such that ABD = CAD. If AB = 5 cm,
AC = 3 cm and AD = 4 cm, find
 EC = AC – AE = 8.8 cm – 3.3 cm = 5.5 cm
(i) BC
ΑD 5.7 3 (ii) DC
Now, = =
DB 9.5 5 (iii) area of  ACD : area of  BCA.
359 Arun Deep's Understanding Math-10

area ΔACD AD 2 4 16 2

area ΔABC
= 2 =
AB 52 = 25
ar(ACD) : ar(ABC) = 16 : 25 Ans.
9. In the adjoining figure the diagonals of a
parallelogram intersect at O. OE is drawn
parallel to CB to meet AB at E, find
area of AOE : area of ||gm ABCD.
Sol. In  ABC and  ACD
C = C (Common) D C

ABC = CAD (given) O


  ABC ~  ACD
AB BC AC
   A
E
B
AD AC DC
5 BC 3 Sol. In the figure
  
4 3 DC Diagonals of parallelogram ABCD are AC
and BD which intersect each other at O.
BC 5
(i)  OE is drawn parallel to CB to meet AB in
3 4
E.
3 × 5 15 To find :
 BC    3.75 cm
4 4 Area of AOE : area of parallelogram ABCD
... Diagonals of a ||gm bisect each other at O
3 5
(ii) 
DC 4  Four triangles in equal area are formed.
3 × 4 12 1
 DC    2. 4 cm  area of OAB  area of ||gm ABCD
5 5 4
(iii) In ACD and ABC ... O is mid-point of AC of ABC and DE ||
CB.
 CAD =  ABC (given)
 E is also mid-point of AB.
 ACD =  ACB (common)
 OE is the median of AOB
 ACD ~ ABC (AA axiom)
But median of a triangle bisects the triangle
A into two equal triangles in areas.
1
 Area of AOE  area of AOB
2

5 cm 3 cm 1 1
  area of ||gm ABCD
4 cm 2 4
1
 area of ||gm ABCD
B C 8
D
360 Arun Deep's Understanding Math-10

area of  AOE 1 (i) DO : OE = 2 : 1


 
area of ||gm ABCD 8 (ii) area of  OEC : area of  OAD = 1 : 4.

 area of AOE : area of ||gm ABCD Sol. Given : In || gm ABCD,


= 1 : 8. E is mid point of BC and DE meets the
diagonal AC at O and meet AB produced at
10. In the given figure, ABCD is a trapezium in
which AB || DC. If 2AB = 3DC, find the ratio F.
of the areas of AOB and COD. To prove : (i) DO : OE = 2 : 1
(ii) area of  OEC : area of  OAD = 1 : 4
D C
Proof : In  AOD and  EDC
AOD = EOC
O (vertically opposite angle)
OAD = OCB (alt. angles)
A B   AOD ~  EOC (AA postulate)
Sol. In the given figure, ABCD is trapezium in OA DO AD 2EC
which AB || DC and 2AB = 3DC    
OC OE EC EC
AB 3
 = 1 1
DC 2 ( EC  BC or EC  AD)
2 2
To find : area AOB : area of COD
Proof : In AOB and COD OA DO 2
  
OC OE 1
AOB = COD
(vertically opposite angles)  DO : OE = 2 : 1
OAB = OCD (alternate angles) (ii) . ..  AOD ~  OEC [proved in (i)]
 AOB ~ COD (AA axiom)
area of  OEC (OE) 2
 area of  AOD 
area AOB AB 2 (3) 2 9 (DO) 2
 = = 2 =
area COD DC 2 ( 2) 4
(1) 2 1
Ratio in the areas of AOB and COD = 9 : 4  2

(2) 4
11. In the adjoining figure, ABCD is a
parallelogram. E is mid-point of BC. DE  area of  OEC : area of  AOD = 1 : 4
meets the diagonal AC at O and meet AB P.Q. In PQR, MN is parallel to QR and
(produced) at F. Prove that
PM 2

MQ 3

MN
(i) Find .
QR
(ii) Prove that OMN and ORQ are similar.
361 Arun Deep's Understanding Math-10
(iii) Find Area of OMN : Area of ORQ. 12. A model of a ship is made to a scale of
1 : 250. Calculate :
(i) the length of the ship, if the length of model
is 1·6 m.
(ii) the area of the deck of the ship, if the area
of the deck of model is 2·4 m2.
(iii) the volume of the model, if the volume of
the ship is 1 km3.
Sol. Scale factor (k) of the model of the ship
1

250
Sol. (i) In PMN and PQR, (i) Length of model = 1·6 m
(1) PMN = PQR (corresponding angles)
1
 Length of ship  (length of model)
(2) PNM = PRQ (corresponding angles) k
 PMN ~ PQR (angle-angle test) = 250 (1·6 m) = 400·0 m.
(ii) Area of deck of ship on the model = 2·4
PM MN m2
  (CSST are proportional)
PQ QR
1
 Area of deck on ship  (Area of deck
2 MN k2
  on model) = (250)2 × 2·4 m2
5 QR
= 150000 m2
(ii) In OMN and ORQ, (iii) Volume of ship = 1 km3
(1) OMN = ORQ (int. alternate angles)  Volume of ship on the model = (k) 3
(2) MON = ROQ (vertically opp. angles) (Volume of actual ship)
 OMN ~ ORQ (by angle-angle test) 1 1 (1000) 3
 3
× 1 km 3  ×
( 250) ( 250) 3 1
Area of (OMN) MN 2
(iii)  = (4)3 = 64 m3 Ans.
Area of (ORQ) RQ 2
Ratio of areas of similar triangles

Area of ( OMN) 22 4
 = 2  Ans.
Area of ( ORQ) 5 25
14
LOCUS
POINTS TO REMEMBER

1. Locus. The locus of a point is the path traced out by the point moving under given geometric
condition or conditions. In other words we can say that. ‘The locus is a set of all those points
which satisfy the given geometrical condition or conditions.’
LOCUS IN SOME STANDARD CASES
1. The locus of a point, which is equidistant from two fixed
points, is the perpendicular bisector of the line segment joining
the two fixed points.

2. The locus of a point, which is equidistant from two


intersecting straight lines, consists of a pair of straight lines
which bisect the angles between the two given lines.

3. The locus of a point, which is equidistant from two parallel


straight lines, is a straight line parallel to the given lines and
mid-way between them.

4. The locus of a point, which is at a given distance from a


given straight line, consists of a pair of straight lines parallel
to the given line and at a given distance from it.

5. The locus of the centre of a wheel, which moves on a straight


horizontal road, is a straight line parallel to the road and at a
distance equal to the radius of the wheel.

362 Arun Deep's Understanding Math-10


363 Arun Deep's Understanding Math-10

6. The locus of a point, which is inside a circle and is equidistant


from two points on the circle, is the diameter of the circle
which is perpendicular to the chord of the circle joining the
given points.

7. The locus of the mid-points of all parallel chords of a circle is


the diameter of the circle which is perpendicular to the given
parallel chords.

8. The locus of a point (in a plane), which is at a given distance


r from a fixed point, is a circle with the fixed point as its
centre and radius r.

9. The locus of a point which is equidistant from two given


concentric circles of radii r1 and r2 is the circle of radius
r1  r2
concentric with the given circles. It lies mid-way
2
between them.

10. The locus of a point which is equidistant from a given circle


consists of a pair of circles concentric with the given circle.

11. If A, B are fixed points, then the locus of a point P such that
APB = 90° is the circle with AB as diameter.

12. The locus of the mid-points of all equal chords of a circle is the
circle concentric with the given circle and of radius equal to the
distance of equal chords from the centre of the given circle.
364 Arun Deep's Understanding Math-10
13. The locus of centres of circles touching a given line PQ at a
given point T on it is the straight line perpendicular to PQ at T.

THEOREM 14·1 Given. Two fixed points A and B. MQ is the


perpendicular bisector of AB and P is any point
The locus of a point which is equidistant on MQ.
from two fixed points, is the perpendicular bisector
of the straight line joining the two points.
Given : Two fixed points A and B, P is a
moving point such that AP = BP

To Prove : AP = BP
Construction : Join AP and BP
Proof : In  PAM and  PBM
To Prove : P lies on the perpendicular
AM = MB (M is mid points of AB)
bisector of the line segment AB
AMP = PMB (each 90º)
Construction. Join AB, Let M be the mid
point of AB. Join PM MP = MP (Common)

Proof. In  PAM and  PBM,   PAM   PBM


AP = BP (Given) (SAS axioms of congruency)
AM = BM (M is mid point of AB)  AP = PB (c.p.c.t.)
PM = PM (Common) Hence locus of a point which is
.. .
equidistant from two fixed points, is the
    PAM  PBM (SSS axiom of perpendicular bisector of the straight line joining
congruency) the two fixed points.
 AMP = PMB c.p.c.t.
THEOREM 14·2
But AMP + BMP = 180° (Linear pair)
 AMP = BMP = 90° The locus of a point which is equidistant
from two intersecting straight lines, consists of a
 MP is the perpendicular bisector of AB. pair of straight lines which bisect the angles
Hence P lies on the perpendicular bisector of AB. between the two given lines.
Conversely. Any point on the perpendicular Given. Two intersecting straight lines AB and
bisector of a line segment joining two fixed points CD intersecting each other at O. P is a moving point
is equidistant from the fixed points. such that PM  OB and PN  OD and PM = PN
365 Arun Deep's Understanding Math-10
Given. Two straight lines AB and CD
intersecting each other at O. P is a point on the
bisector of BOD. PM  OB and PN  OD
To Prove : MP = NP
Proof : In  OMP and  ONP
MOP = NOP
To Prove : P lies on the bisector of BOD . .. P lies on the bisector of DOB.
Construction : Join OP
OMP = ONP (each 90°)
Proof : In right  OMP and  ONP
Side PM = PN (given) OP = OP (Common)
Hyp. OP = OP (Common)   OMP  ONP
  OMP   ONP (AAS axiom of congruency)
(R.H.S axiom of congruency)
 MP = NP (c.p.c.t.)
 BOP = DOP (c.p.c.t.)
 P lies on the bisector of BOD. Similarly if P is a point on the bisector
Conversely. Any point on the bisector of an of AOD and MP  AO, NP  OD then
angle is equidistant from the arms of the angle.
MP = NP.
Hence, the locus of a point which is
equidistant from two intersecting straight lines,
consists of a pair of straight lines which bisect
the angles between the two given lines.

EXERCISE 14
1. A point moves such that its distance from
a fixed line AB is always the same. What is 2. A point P moves so that its perpendicular
the relation between AB and the path trav- distance from two given lines AB and CD
elled by P ? are equal. State the locus of the point P.
Sol. Let point P moves in such a way that it is Sol. (i) When two lines AB and CD are parallel,
at a fixed distance from the fixed line AB. then the locus of the point P which is
equidistant from AB and CD is a line (l) in
l
the mid way of AB and CD and parallel to
them.

m
l
 It is a set of two lines l and m parallel to AB
drawn on either side of it at equal distance
from it.
366 Arun Deep's Understanding Math-10
m The locus of P will be the circle whose
diameter is AB. We know that angle in a
semi-circle is always equal to 90°.
 APB = 90°
l (ii) AB is a fixed line and P is a point such that
APB = 60°. Then locus of P will be a
major segment of a circle whose AB is a
chord.

(ii) If AB and CD are intersecting lines, then


the locus of the point P will be a pair of the
straight lines l and m which bisect the angles
between the given lines AB and CD.
3. P is a fixed point and a point Q moves such
that the distance PQ is constant, what is the
locus of the path traced out by the point Q ?
Sol. P is a fixed point and Q is a moving point
such that it is always at an equidistant from
P.

5. Draw and describe the locus in each of the


following cases :
(i) The locus of points at a distance 2·5 cm
from a fixed line.
(ii) The locus of vertices of all isosceles
 P is the centre of the path of Q which is triangles having a common base.
a circle. The distance between P and Q is
(iii) The locus of points inside a circle and
the radius of the circle.
equidistant from two fixed points on the
Hence locus of point Q is a circle with P as circle.
centre.
(iv) The locus of centres of all circles passing
4. (i) AB is a fixed line. State the locus of the
through two fixed points.
point P so that APB = 90°.
(v) The locus of a point in rhombus ABCD
(ii) A, B are fixed points. State the locus of which is equidistant from AB and AD.
the point P so that APB = 60°.
(1998)
Sol. (i) AB is a fixed line and P is a point such
(vi) The locus of a point in the rhombus ABCD
that APB = 90°. which is equidistant from points A and C.
(1998)
Sol. (i)
367 Arun Deep's Understanding Math-10
1. Draw a given line AB. (iv) Let C1, C2, C3 ...... be the centres of circle
2. Draw lines of l and m parallel to AB at a which pass through the two fixed points A
distance of 2·5 cm. and B.
Lines l and m are the locus of point P which
is at a distance of 2·5 cm.
(ii)

 ABC is an isosceles triangle in which AB


= AC. Draw a line XY passing through these
From A, draw AD perpendicular to BC. AD centres C1, C2, C3.
is the locus of the point A the vertices of  Hence locus of centres of circles passing
ABC. through two points A and B is the
... In rt.  ABD and  ACD perpendicular bisector of line segment
Side AD = AD (Common) joining the two fixed points.
Hyp. AB = AC (given) (v) In rhombus ABCD,
  ABD   ACD (R.H.S. Axiom) Join AC.
 BD = DC (c.p.c.t.)
Hence locus of vertices of isosceles
triangles having common base is the
perpendicular bisector of BC.
(iii) (i) Draw a circle with centre O.

... AC is the diagonal of rhombus ABCD


 AC bisect A.
 Any point on AC, is the locus which is
equidistant from AB and AD.
(ii) Take points A and B on it and join (vi) ABCD is a rhombus. Join BD
them.
(iii) Draw a perpendicular bisector of AB
which passes from O and meets the circle
at C.
 CE the diameter, which is the locus of a
point inside the circle and equidistant from
two points A and B at the circle.
368 Arun Deep's Understanding Math-10
BD is the locus of a point in the rhombus From P, draw a line CD and another line
which is equidistant from A and C. EF from P parallel to AB.
Diagonal BD bisects the B and D. Thus CD and EF are the lines which are
 Any point on BD will be equidistant from A the locus of the point equidistant from AB.
and C. (iv) Take a point O and another point P such
6. Describe completely the locus of points in that OP = 5 cm. with centre O and radius
each of the following cases : equal to OP, draw a circle. Thus this circle
(i) mid-point of radii of a circle. is the locus of point P which is at a distance
(ii) centre of a ball, rolling along a straight line
of 5 cm from O, the given point.
on a level floor.
(iii) point in a plane equidistant from a given line.
(iv) point in a plane, at a constant distance of 5
cm from a fixed point (in the plane).
(v) centre of a circle of varying radius and
touching two arms of ABC.
(vi) centre of a circle of radius 2 cm and touching
a fixed circle of radius 3 cm with centre O.
Sol. (i) The locus of mid points of the radii of a (v) Draw the bisector BX of ABC. This
circle is another concentric circle with radius bisector of angle is the locus of the centre
is half of the radius of the given circle.
of a circle with different radii.
Any point on BX, is equidistant from the
arms BA and BC of the ABC.

(ii) AB is the straight line on the ground and


the ball is rolling on it

(vi) If the circle with 2 cm as radius touches


the given circle externally then the locus of
the centre of the circle will be a concentric
circle with radius 3 + 2 = 5 cm.
 locus of the centre of the ball is a line
parallel to the given line AB.
(iii) AB is the given line and P is a point in the plane.
369 Arun Deep's Understanding Math-10
If the circle with 2 cm as radius touches
the given circle with 3 cm as radius
internally, then the locus of the centre of
the circle will be a concentric circle with
radius 3 – 2 = 1 cm.

Steps of Construction :
(i) Take BC = 8 cm long line segment.
7. Using ruler and compasses construct : At B, draw a ray BX making an angle of
(i) a triangle ABC in which AB = 5.5 cm, 60° with BC.
BC = 3.4 cm and CA = 4.9 cm. Cut off BA = 7 cm. and joing AC.
(ii) the locus of points equidistant from A and (i) Draw the perpendicular bisector of BC.
C. (ii) Draw the angle bisector of B which
Sol. Steps : (i) Draw BC = 3.4 and mark the intersect the prependicular bisector of BC at
arcs of 5.5 and 4.9 cm from B and C. Join P.
A, B and C. P is the required point.
ABC is the required triangle. (iii) On measuring the length of BP = 4·6 cm
(ii) Draw  bisector of AC. (approx)
(iii) Draw an angle of 90° at AB at A which 9. A straight line AB is 8 cm long. Locate by
intersects  bisector at O. Draw circle construction the locus of a point which is:
taking O as centre and OA as radius.
(i) Equidistant from A and B.
A
(ii) Always 4 cm from the line AB.
(iii) Mark two points X and Y, which are 4 cm
from AB and equidistant from A and B.
Name the figure AXBY. (2008)
cm

O
5.5

4.9 cm Sol. Steps of construction —


l
X
B 3.4 cm C

8. Construct triangle ABC, with AB = 7 cm,


4 cm

BC = 8 cm and ABC = 60°. Locate by O


A 8 cm B
construction the point P such that :
4 cm

(i) P is equidistant from B and C and


(ii) P is equidistant from AB and BC
(iii) Measure and record the length of PB. Y
(2000)
370 Arun Deep's Understanding Math-10
(i) Draw a line segment AB = 8 cm. (iii) Join AC.
(ii) With the help of compasses and ruler, draw The ABC is the required triangle.
the perpendicular bisector l of AB which (b) Draw the bisector BY of ABC.
intersects AB at O. (c) Draw the perpendicular bisector of BC which
(iii) Then any point on l, is equidistant from A intersects BY at P.
and B.  P is the required point P which is equidistant
(iv) Cut off OX = OY = 4 cm. from BC and BA and also equidistant from B
The X and Y are the required locus, which is and C.
equidistant from AB and also from A and B. On measuring PB it is 3.4 cm (approx)
(v) Join AX, XB, BY and YA. 11. Construct a triangle ABC with AB = 5.5 cm,
The figures so formed AXBY, is the shape of AC = 6 cm and BAC = 105°. Hence :
a square because its diagonals are equal and (i) Construct the locus of points equidistant from
bisect each other at right angles. BA and BC.
10. Use ruler and compasses only for this (ii) Construct the locus of points equidistant
question. from B and C.
(i) Construct ABC, where AB = 3.5 cm, BC = (iii) Mark the point which satisfies the above two
6 cm and ABC = 60º. loci as P. Measure and write the length of PC.
(ii) Construct the locus of points inside the Sol. Steps of construction :
triangle which are equidistant from BA and Construct the triangle ABC with AB = 5.5 cm
BC. BAC = 105° C
(iii) Construct the locus of points inside the triangle and AC = 6 cm
which are equidistant from B and C. (i) Points which are
(iv) Mark the point P which is equidistant from

cm
equidistant from 6 cm

4.8
AB, BC and also equidistant from B and C. BA and BC lies
Measure and record the length of PB. on the bisector P
(2010) of ABC.
Sol. In ABC, AB = 3.5 cm, BC = 6 cm and (ii) Points
ABC = 60º equidistant from 105°
X
L B and C lies on B 5.5 cm A
A the perpendicular bisector of BC. Draw
perpendicular bisector of BC. The required point
Y P is the point of intersection of the bisector of
ABC and the perpendicular bisector of BC.
cm

P (iii) Required length of PC = 4.8 cm.


3.5

cm
3.4
12. Points A, B and C represent position of
60º three towers such that AB = 60 m, BC =
D
B 6 cm C 73m and CA = 52 m. Taking a scale of 10
m to 1 cm, make an accurate drawing of
ABC. Find by drawing, the location of a
M point which is equidistant from A, B and
C, and its actual distance from any of the
Steps of construction : towers.
(a) (i) Draw a line segment BC = 6 cm Sol. AB = 60 m = 6.0 cm, BC = 73 m = 7·3 cm
(ii) At, B draw a ray BX making an angle of 60º and CA = 52 m = 5·2 cm.
and cut off BA = 3.5 cm. (i) Draw a line segment BC = 7·3 cm
371 Arun Deep's Understanding Math-10
Hence there are four points which are
equidistant from AB and CD and 2 cm from
O, the point of intersection of AB and CD.
14. Without using set square or protractor,
construct the quadrilateral ABCD in which
BAD = 45°, AD = AB = 6 cm, BC = 3·6
cm and CD = 5 cm.
(i) Measure BCD.
(ii) Locate the point P on BD which is
equidistant from BC and CD. (1992)
Sol. (i) Take AB = 6 cm long
(ii) AT A, draw the angle of 45° and cut off
(ii) With centre B and radius 6 cm and with AD = 6 cm
centre C and radius 5·2 cm, draw two arcs
intersecting each other at A
(iii) Joining AB and AC.
(iv) Draw the perpendicular bisector of AB, BC
and CA respectively, which intersect each
other at point P. Join PB. P is equidistant from
A, B and C on measuring PB = 3·7 cm.
 Actual distance = 37 m.
13. Draw two intersecting lines to include an (iii) With centre D and radius 5 cm and with
angle of 30°. Use ruler and compasses to centre B, and radius 3·5 cm draw two arcs
locate points which are equidistant from intersecting each other at C.
these lines and also 2 cm away from their (iv) Join CD and CB and join BD
point of intersection. How many such ABCD is the required quadrilateral.
points exist ? (1990) (v) On measuring BCD = 65°.
Sol. (i) Two lines AB and CD intersect each (vi) Draw the bisector of  BCD which
other at O. intersects BD at P.
P is the required point which is equidistant
from CD and CB.
15. Without using set square or protractor,
construct rhombus ABCD with sides of
length 4 cm and diagonal AC of length 5
cm. Measure ABC. Find the point R on
AD such that RB = RC. Measure the length
of AR. (1990)

(ii) Draw the bisectors of BOD and AOD.


(iii) With centre O and radius equal to 2 cm.
marks points on the bisector of angles at
P, Q, R and S respectively. Sol. (i) Take AB = 4 cm
372 Arun Deep's Understanding Math-10
(ii) With centre A, draw an arc of 5 cm radius arc intersecting the angle bisector of BCA
and with B draw another arc of radius 4 at P and P.
cm intersecting each other at C.  P and P are two loci which satisfies the
(iii) Join AC and BD. given condition.
(iv) Again with centre A and C, draw two arcs of On measuring CP and CP
radius 4 cm intersecting each other on D. CP = 3·6 cm
(v) Join AD and CD. and CP = 1·1 cm.
ABCD is the required rhombus and on 17. By using ruler and compasses only,
measuring the ABC, it is 78°. construct an isosceles triangle ABC in
(vi) Draw perpendicular bisector of BC which BC = 5 cm, AB = AC and BAC =
intersecting AD at R. 90°. Locate the point P such that :
On measuring the length of AR, it is equal (i) P is equidistant from the sides BC and AC.
to 1·2 cm. (ii) P is equidistant from the points B and C.
16. Without using set-squares or protractor Sol. Steps of Construction :
construct :
(i) Triangle ABC, in which AB = 5·5 cm,
BC = 3·2 cm and CA = 4·8 cm.
(ii) Draw the locus of a point which moves so
that it is always 2·5 cm from B.
(iii) Draw the locus of a point which moves so
that it is equidistant from the sides BC and
CA.
(iv) Mark the point of intersection of the loci
with the letter P and measure PC. (1994)
Sol. Steps of Construction :
(i) Draw BC = 3·2 cm long.
(i) Take BC = 5·0 cm and bisect it at D.
(ii) With centre B and radius 5·5 cm and with
centre C and radius 4·8 cm draw arcs (ii) Taking BC as diameter, draw a semicircle.
intersecting each other at A. (iii) At D, draw a perpendicular intersecting the
circle at A.
(iv) Join AB and AC.
(v) Draw the angle bisector of C intersecting
the perpendicular at P.
P is the required point.
18. Using ruler and compass only, construct a
semicircle with diameter BC = 7 cm. Locate
a point P on the circumference of the
semicircle such that A is equidistant from
B and C. Complete the cyclic quadrilateral
ABCD, such that D is equidistant from AB
(iii) Join AB and AC. and BC. Measure ADC and write it down.
(iv) Draw the bisector of BCA. Sol. Steps of construction :
(v) With centre B and radius 2·5 cm, draw an (i) Draw BC = 7 cm.
373 Arun Deep's Understanding Math-10
(ii) Draw perpendicular bisector of BC which Steps of Construction :
meets BC at point O. (i) Draw AB = 6 cm
(iii) With O as centre and OB (= OC) as radius, (ii) At B, draw angle of 60° and cut off BC = 5 cm.
draw a semi-circle which meets (iii) Draw the angle bisector of B.
perpendicular bisector of BC at point A.
(iv) With centre A and radius 5 cm. draw an arc
(iv) Since, A lies on the perpendicular bisector which intersect the angle bisector of B at D.
of BC, A is equidistant from B and C.
(v) Join AD and DC.
(v) D is equidistant from AB and BC.
ABCD in the required quadrilateral. On
 D lies on bisector of angle ABC. measuring CD, it is 5·3 cm (approx).
So, draw bisector of angle ABC which 20. Construct an isosceles triangle ABC such
meets the semi-circle at point D. that AB = 6 cm, BC = AC = 4 cm. Bisect
Join AD and DC. C internally and mark a point P on this
 ABCD is the required cyclic quadrilateral. bisector such that CP = 5 cm. Find the
points Q and R which are 5 cm from P and
Also, ADC = 135°. Ans.
also 5 cm from the line AB (2001)
Sol. Steps of Construction :
A (i) Draw a line AB = 6 cm

B Q C

19. Using ruler and compasses only, construct


a quadrilateral ABCD in which AB = 6 cm,
BC = 5 cm, B = 60°, AD = 5 cm and D
is equidistant from AB and BC. Measure
CD. (ii) With centre A and B and radius 4 cm, draw
Sol. two arcs intersecting each other at C.
(iii) Join CA and CB
(iv) Draw the bisector of  C and cut off
CP = 5 cm.
(v) Draw a line XY parallel to AB at a distance
of 5 cm.
(vi) From P, draw arcs of radius 5 cm each
intersecting the line XY at Q and R.
Hence Q and R are the required points.
374 Arun Deep's Understanding Math-10
21. Use ruler and compasses only for this (v) Measure and record the length of CQ.
question. Draw a circle of radius 4 cm and (1998)
mark two chords AB and AC of the circle Sol. Steps of Construction :
of length 6 cm and 5 cm respectively. (i) Draw AB = 9 cm
(i) Construct the locus of points, inside the (ii) At B draw an angle of 60° and cut off BC
circle, that are equidistant from A and C. = 6 cm.
(iii) Join AC
Prove your construction.
(ii) Construct the locus of points, inside the
circle, that are equidistant from AB and AC.
(1995)
Sol. Steps of Construction :
(i) With centre O and radius 4 cm draw a circle.
(ii) Take a point A on this circle.

(iv) Draw perpendicular bisector of BC. All points


on it, will be equidistant from B and C.
(v) From A, draw a line XY parallel to BC.
(vi) Produce the perpendicular bisector of BC
to meet XY in Q.
(vii) Join QC and QB.
(iii) With centre A and radius 6 cm draw an arc
cutting the circle at B.  QBC will be the triangle equal in area to
(iv) Again with radius 5 cm, draw another arc  ABC because these are on the same base
cutting the circle at C. BC and between the same parallel lines.
(v) Join AB and AC. On measuring, the length of CQ is 8·2 cm
(vi) Draw the perpendicular bisector of AC. (approx.).
 Any point on it, will be equidistant from A CHAPTER TEST
and C.
(vii) Draw the angle bisector of A intersecting 1. Draw a straight line AB of length 8 cm. Draw
the perpendicular bisector of AC at P. the locus of all points which are equidistant
P is the required locus. from A and B. Prove your statement.
22. Ruler and compasses only may be used Sol. (i) Draw a line segment AB = 8 cm.
in this question. All construction lines and
arcs must be clearly shown, and be of
sufficient length and clarity to permit
assessment.
(i) Construct a triangle ABC, in which BC = 6
cm, AB = 9 cm. and ABC = 60°.
(ii) Construct the locus of all points, inside 
ABC, which are equidistant from B and C. (ii) Draw the perpendicular bisector of AB
(iii) Construct the locus of the vertices of the intersecting AB at D.
triangles with BC as base, which are equal  Every point P on it will be equidistant from
in area to  ABC. A and B.
(iv) Mark the point Q, in your construction, (iii) Take a point P on the perpendicular bisector.
which would make  QBC equal in area to (iv) Join PA and PB.
 ABC, and isosceles. Proof : In  PAD and  PBD
375 Arun Deep's Understanding Math-10
PD = PD (common) both the above conditions. What kind of
AD = BD (D is mid-point of AB) quadrilateral is PQRS ? Compute the area
PDA = PDB (each 90°) of the quadrilateral PQRS.
  PAD   PBD (SAS axiom of congruency) Sol.
 PA = PB (c.p.c.t.)
Similarly we can prove any other point on
the perpendicular bisector of AB, is
equidistant from A and B.
Hence Proved.
2. A point P is allowed to travel in space. State
the locus of P so that it always remains at
a constant distance from a fixed point C.
Sol. ... The point P is moving in the space and it
is at a constant distance from a fixed point
C. Steps of Construction :
 Its locus is a sphere. (i) Take a line AB = 12 cm
3. Draw a line segment AB of length 7 cm. (ii) Take M, the mid point of AB.
Construct the locus of a point P such that (iii) Draw straight lines CD and EF parallel to
area of triangle PAB is 14 cm2. AB at a distance of 3 cm.
Sol. Base of  PAB = 7 cm (iv) With centre M and radius 5 cm, draw areas
and its area = 14 cm2 which intersects CD at P and Q and EF at
R and S.
Area × 2 14 × 2
 Height   cm = 4 cm (v) Join QR and PS.
Base 7 PQRS is a rectangle where length
PQ = 8 cm.
 Area of rectangle PQRS = PQ × RS
= 8 × 6 = 48 cm2
5. AB and CD are two intersecting lines. Find
the position of a point which is at a distance
of 2 cm from AB and 1·6 cm from CD.
Sol. (i) AB and CD are the intersecting lines
which intersect each other at O.
Now draw a line XY parallel to AB at a
distance of 4 cm.
Now take any point P on XY
Join PA and PB
 area of  PAB = 14 cm.
Hence locus of P is the line XY which is O
parallel to AB at distance of 4 cm.
4. Draw a line segment AB of length 12 cm.
Mark M, the mid-point of AB. Draw and
describe the locus of a point which is
(i) at a distance of 3 cm from AB. (ii) Draw a line EF parallel to AB and GH parallel
(ii) at a distance of 5 cm from the point M. to CD intersecting each other at P
Mark the points P, Q, R, S which satisfy  P is the required point.
376 Arun Deep's Understanding Math-10
6. Two straight lines PQ and PK cross each Steps of Construction :
other at P at an angle of 75°. S is a stone (i) Take PR = 8 cm and draw the perpendicular
on the road PQ, 800 m from P towards Q. bisector of PR intersecting it at O.
By drawing a figure to scale 1 cm = 100 (ii) From O, out. off OS = OQ = 3 cm
m, locate the position of a flag staff X, (iii) Join PQ, QR, RS and SP.
which is equidistant from P and S, and is PQRS is a rhombus. Whose diagonal are
also equidistant from the road. PR and QS.
Sol. 1 cm = 100 cm (iv) PR is the bisector of SPQ.
 800 m = 8 cm. (v) Draw the perpendicular bisector of SR
Steps of Construction : intersecting PR at X
 X is equidistant from PQ and PS and
(i) Draw the lines PQ and PK intersecting each
also from S and R.
other at P making an angle of 75°.
On measuring length of XR = 3·2 cm (approx)
8. Without using set square or protractor,
construct the parallelogram ABCD in which
AB = 5·1 cm. the diagonal AC = 5·6 cm
and the diagonal BD = 7 cm. Locate the
point P on DC, which is equidistant from
AB and BC.
Sol.
(ii) Take a point S on PQ such that PS = 8 cm.
(iii) Draw the perpendicular bisector of PS.
(iv) Draw the angle bisector of KPS
intersecting the perpendicular bisector at X.
X is the required point which is equidistant
from P and S and also from PQ and PK.
7. Construct a rhombus PQRS whose
diagonals PR, QS are 8 cm and 6 cm
respectively. Find by construction a point
Steps of Construction :
X equidistant from PQ, PS and equidistant
(i) Take AB = 5·1 cm
from R, S. Measure XR.
Sol. 5 .6
(ii) At A, with readius = 2.8 cm and at B
2
7. 0
with radius  3.5 cm, draw two arcs
2
intersecting each other at O.
(iii) Join AO and produce it to C such that OC
= AD = 2·8 cm and join BO and produce it
to D such that BO = OD = 3·5 cm
(iv) Join BC, CD, DA
ABCD is a parallelogram.
(v) Draw the angle bisector of ABC
intersecting CD at P. P is the required point
which is equidistant from AB and BC.
377 Arun Deep's Understanding Math-10
9. By using ruler and compass only, (i) Draw a line segment AB = 6·5 cm.
construct a quadrilateral ABCD in which (ii) At A, draw a ray making an angle of 75°
AB = 6·5 cm, AD = 4cm and DAB = and cut off AD = 4 cm.
75°. C is equidistant from the sides AB (iii) Draw the bisector of DAB.
and AD, also C is equidistant from the (iv) Draw perpendicular bisector of AB
points A and B. intersecting the angle bisector at C.
Sol. Steps of Construction : (v) Join CB and CD.
ABCD is the required quadrilateral.

D
4cm

A 6.5cm B
15
Circle
POINTS TO REMEMBER
1. Circle : The locus of a point which moves in a plane so that its distance from a fixed point in that
plane, remains constant is called a circle. The fixed point is called the centre and the constant
distance is called the radius of the circle.
2. Parts of a circle : A circle has three parts : interior, exterior and circle itself.
3. Chord of a circle : Any line segment joining two points on a circle is called the chord of the
circle.
4. Diameter : A chord which passes through the centre of the circle is called diameter and diameter
= 2 × radius.
5. Secant of a circle : A line which intersects the circle is called a secant.
6. Tangent to a circle : A line which meets a circle in one and only one point is called a tangent to
the circle : Where the tangent touches the circle, is called the point of contact.
7. Direct and transverse common tangents :
(i) A common tangent to two circles is called a direct common tangent, if it touches the circles in
the same sides.
(ii) A common transverse tangent : A common tangent to two circles is called an inverse
common tangent if it touches the circles in opposite sides.
8. Arc of a circle : A part of a circle is called an arc of the circle.
(i) Semi circle : One half of the whole circle is called the semi circle. The whole arc of a circle
is called circumference of the circle.
(ii) An arc which is less than one half of the whole arc of a circle is called Minor arc and greater
than one half of the whole arc is called the Major arc.
9. Sector of a circle : The part of the plane region enclosed by an arc bounded by two radii is called
a sector of a circle. The sector which subtends an angle at the centre is called the angle of the arc
or sector of a circle.
10. Segment of a circle : A chord divides the circle into two parts and each part is called a segment
of a circle. The segment which is less then semi circle is called a minor segment and a segment
which is greater than semi circle is called a major segment.
11. Concentric circles : Two or more circles which have the same centre are called concentric
circles. Their radii are different.
12. Equal or congruent circles : Two or more circles which have equal radii but different centres
are called equal or congruent circles.
13. Circumscribed circle : Any circle which passes through the vertices of a polygon is called a
circumscribed circle and its centre is called a circumcentre.
14. Inscribed circle : Any circle which touches the sides of a polygon is called an inscribed circle
and its centre is called an incentre of the circle.
378 Arun Deep's Understanding Math-10
379 Arun Deep's Understanding Math-10
15. Cyclic quadrilateral :
Any quadrilateral whose vertices lie on the same circle, is called a cyclic quadrilateral.
16. Tangents to a circle or circles :
(i) One tangent can be drawn if the point is on the circle.
(ii) Two tangents can be drawn if the point is outside the circle.
(iii) If two circles touch internally, only one common tangent can be drawn.
(iv) If two circles touch externally, three common tangents can be drawn.
(v) If two circles intersect each other, two common tangents can be drawn.
(vi) If two circles donot intersect each other, four tangents can be drawn.
17. Axioms of equal arcs : In equal circles (or in the same circle) if two arcs subtend equal angles
at the centre, they are equal.
Conversely : In two equal circles (or in the same circle) if two arcs are equal, they subtend
equal angles at the centre or centres.

THEOREM 15.1
The straight line drawn from the centre of a circle to bisect a chord, which is not a diameter,
is perpendicular to the chord.
Given : A chord AB of a circle with centre O and OM bisects the chord AB i.e. AM = MB.
To Prove : OM  AB.
Construction : Join OA and OB.
Proof : In OAM and OBM
OM = OM (Common)
OA = OB (Radii of the same circle)
AM = MB (Given)
 OAM  OBM (SSS axioms of congruency)
 OMA = OMB (c.p.c.t.)
But OMA + OMB = 180º (linear pair)
 OMA = OMB = 90º
Hence OM  AB Q.E.D.

THEOREM 15.2
(Converse of theorem 15.1)
The perpendicular to a chord from the centre of a circle bisects the chord.
Given : A chord AB of a circle with centre O and OM  AB.
To Prove : AM = MB.
Construction : Join OA and OB.
Proof : In right OMA and OMB
Hyp. OA = OA (Radii of the same circle)
Side OM = OM (Common)
380 Arun Deep's Understanding Math-10
 OMA  OMB (RHS axiom congruency) A, B and C.
 AM = MB (c.p.c.t.) (Q.E.D.) Hence one and only one circle can be drawn
Corollary : The perpendicular bisector of a through three non-collinear points. (Q.E.D.)
chord of a circle passes through the centre. Corollary : Perpendicular bisectors of two
chords of a circle intersect each other at the
THEOREM 15.3
centre.
One and only one circle can be drawn through
three non-collinear points.
THEOREM 15.4
Equal chords of a circle are equidistant from
the centre.

Given : Three non-collinear points A, B and Given : Two chords AB and CD of a circle
C. with centre O, are equal i.e. AB = CD.
To Prove : One and only one circle can be To Prove : AB and CD are equidistant from
drawn passing through the points A, B and the centre O i.e. if OL  AB and OM  CD,
C. then OL = OM.
Construction : Join AB and BC. Construction : Join OA and OC.
Draw perpendicular bisectors of AB and BC Proof. ... OL  AB
interesting each other at O.
 L is mid point of AB.
Join OA, OB and OC.
Proof : ... O lies on the perpendicular bisector 1
 AL  AB
of AB 2
 OA = OB ...(i) Similarly OM  CD
Again O lies on the perpendicular bisector of 1
BC  CM  CD
2
 OB = OC ...(ii) . .. AB = CD (given)
From (i) and (ii)
 AL = CM
OA = OB = OC
Now in right OLA and OMC,
 O is equidistant from A, B and C.
Hyp. OA = OC
 One circle can be drawn taking O as centre
and OA as radius. (Radii of the same circle)
... Two lines intersect each other at only one Side AL = CM (proved)
point.  OLA  OMC
 A perpendicular bisector of AB and AC (RHS axiom of congruency)
intersect each other at only one point O.  OL = OM (c.p.c.t.) Q.E.D.
 There is only one circle passing through
381 Arun Deep's Understanding Math-10
THEOREM 15.5
(Converse of Theorem 15.4)
Chords of a circle that are equidistant from
the centre of the circle are equal.

Given : Two chords AB and CD are of a


circle OL  AB and OM  CD and OL =
OM.
To Prove : AB = CD.
Construction : Join OA and OC
Proof : In AOC,
Proof : In right OAL and OCM, . .. OA = OC (Radii of the same
Hyp. OA = OC
circle)
(Radii of the same circle)  OAC = ACO
Side OL = OM (given) (Angles opposite to equal sides)
 OAL  OCM
and Ext. AOP = OAC + ACO
(RHS axiom of congruency)
= 2 ACO ...(i)
 AL = CM
Similarly we can Prove that
1
But AL  AB (... OL  AB) BOP = 2 OCB ...(ii)
2
In figure (i) and (iii)
1
and CM  CD (... OM  CD) Adding (i) and (ii)
2
AOP + BOP = 2 ACO + 2 OCB
1 1
 AB  CD  AOP + BOP = 2 (ACO + OCB)
2 2
 AB = CD Q.E.D.  AOB = 2 ACB
THEOREM 15.6 In figure (ii)
The angle subtended by an arc of a circle Subtracting,
at the centre is double the angle subtended BOP – AOP = 2 OCB – 2 ACO
by it at any other point on the remaining  AOB = 2 (OCB – ACO)
part of the circle.  AOB = 2 ACB
Given : A circle with centre O and an arc AB Hence AOB = 2 ACB Q.E.D.
which subtends AOB at the centre and
THEOREM 15.7
ACB at any other point C on the remaining
part of the circle. Angles in the same segment of a circle
are equal.
To Prove : AOB = 2 ACB.
Given : A circle with centre O and ACB
Construction : Join CO and produce it to P. and ADB are in the same segment.
382 Arun Deep's Understanding Math-10
C and meets BD at E (in fig. (i)) or on
producing BD (fig. (ii)).
Proof : ACB = AEB
(Angles in the same segment of a circle)
But ACB = ADB (given)
 AEB = ADB
But AEB  ADB
To Prove : ACB = ADB. {Ext. angle of a triangle is greater than
Construction : Join OA and OB. its interior opposite angles}
Proof : Arc AB subtends AOB at the centre  Our supposition is wrong.
and ACB at the remaining part of the circle. Hence A, B and C and D are concyclic.
 AOB = 2 ACB ...(i) Q.E.D.
Similarly THEOREM 15.9
AOB = 2 ADB ...(ii) Angle in a semicircle is a right angle.
From (i) and (ii)
2 ACB = 2 ADB
 ACB = ADB Q.E.D.
THEOREM 15.8
If a line segment joining two points
subtends equal angles at two other points
lying on the same side of the line
containing the segment, than the four Given : A semi circle ACB with centre O and
points are concyclic. ACB is in the semi circle.
To Prove : ACB = 90º
Proof : Arc AB subtends AOB at the centre
and ACB in the remaining part of the circle.
 AOB = 2 ACB
 2 ACB = AOB
1
 ACB  AOB
2
But AOB = 180º
(... AOB is a straight line)
1
Given : AB is the line segment and C and D  ACB   180º  90º
are the two points lying on the same side of 2
the line AB and ACB = ADB. Hence ACB = 90º
Q.E.D.
To Prove : Points A, B, C and D are
concyclic. THEOREM 15.10
Construction : Let the points A, B, C and D (Converse of Theorem 15.9)
are not concyclic and let a circle passing If an arc of a circle subtends a right angle
through three non-collinear points A, B and at any point on the remaining part of the
383 Arun Deep's Understanding Math-10
circle, then the arc is a semi circle.  DOB = 2 DCB ...(i)
Similarly reflex DOB = 2 DAB ...(ii)
Adding (i) and (ii),
2 DCB + 2 DAB
= DOB + reflex DOB
 2 (DCB + DAB) = 360º
(Angles at a point)
360º
Given : A circle with centre O and arc AB  DCB + DAB   180º
2
subtends ACB on the remaining part of the
 A + C = 180º
circle such that ACB = 90º
Similarly we can Prove that
To Prove : Arc AB is a semi circle.
B + D = 180º. Q.E.D.
Construction : Join OA and OB.
THEOREM 15.12
Proof : Arc AB subtends AOB at the centre
O and ACB at the remaining part of the (Converse of Theorem 15.11)
circle. If a pair of opposite angles of a quadrilateral
 AOB = 2 ACB are supplementary, then the quadrilateral is a
cyclic.
But ACB = 90º (given)
 AOB = 2 × 90º = 180º
 AOB is a straight line.
Hence arc ACB is a semi circle.
Q.E.D.
THEOREM 15.11
The opposite angles of a cyclic quadrilateral
are supplementary.

Given : A quadrilateral ABCD in which


A + C = 180º
To Prove : ABCD is a cyclic quadrilateral.
Construction : Let ABCD is not a cyclic
quadrilateral and let a circle passing through
the points A, B and D meets BC in E (fig. (i))
Given : A cyclic quadrilateral ABCD in a on BC produced at E (fig. (ii)). Join DE.
circle with centre O. Proof : ... ABED is a cyclic quadrilateral.
To Prove : A + C = 180º, B + D =  A + BED = 180º ...(i)
180º.
(Opposite angles of a cyclic quad.)
Construction : Join OB and OD.
But A + C = 180º (given) ...(ii)
Proof : Arc DAB subtends DOB at the  From (i) and (ii)
centre and DCB at the remaining part of
the circle. BED = C
384 Arun Deep's Understanding Math-10
But C  BED as exterior angle of a triangle Proof : In OAB and OPQ.
cannot be equal to its interior opposite angle) OA = OP (Radii of equal circles)
 Our supposition is wrong. OB = OQ
Hence ABCD is a cyclic quadrilateral. Q.E.D. AB = PQ (given)
THEOREM 15.13  OAB  OPQ
The exterior angle of a cyclic quadrilateral (SSS axiom of congruency)
is equal to its interior opposite angle.
 AOB = PQQ (c.p.c.t.)
 arc AB = arc PQ
Q.E.D.
THEOREM 15.15
(Converse of Theorem 15.14)
In equal circles (in the same circle) if two
arcs are equal, then their chords are equal.
Given : ABCD is a cyclic quadrilateral whose
side AB is produced to E.
To Prove : CBE = D.
Proof : ABCD is a cyclic quadrilateral.
(given)
 ABC + D = 180º ...(i)
But ABC + CBE = 180º
(Linear pair) ...(ii) Given : Arc AB and arc PQ of two equal
circles with centre O and O respectively and
 from (i) and (ii)
arc AB = arc PQ.
ABC + D = ABC + CBE
To Prove : AB = PQ.
 D = CBE
Hence CBE = D Q.E.D. Construction : Join OA, OB, OP and OQ.
THEOREM 15.14 Proof : ... Arc AB = arc PQ (given)
In equal circles (or in the same circle),  AOB = POQ
equal chords cut off equal arcs. Now in OAB and OPQ,
OA = OP (Radii of the equal circles)
OB = OQ
AOB = POQ (Proved)
 OAB  OPQ
(SAS axiom of congruency)
 AB = PQ (c.p.c.d.) Q.E.D.
Given : AB and PQ are chords of two equal
THEOREM 15.16
circles with centre O and O respectively and
AB = PQ. The tangent at any point of a circle and
To Prove : Arc AB = Arc PQ the radius through the point are
perpendicular to each other.
Construction : Join OA, OB, OP and OQ.
385 Arun Deep's Understanding Math-10
Given : TPT is a tangent to the circle with (iii) APO = BPO
centre O and OP is the radius through the
point of contact P.

Proof : In right OAP and OBP


Hyp. OP = OP (Common)
To Prove : OP  TPT Side OA = OB (Radii of the same circle)
Construction : If OP is not perpendicular to  OAP  OBP (R.H.S. axiom of
TPT, draw OM  TPT to meet the circle at congruency)
Q.  PA = PB (c.p.c.t)
Proof : OP = OQ (Radii of the same circle) AOP = BOP (c.p.c.t)
... OM  TPT (construction) and APO = BPO (c.p.c.t)
 OMP = 90º Q.E.D.
 In right OPM, THEOREM 15.18
OP = OQ If two circles touch each other internally or
externally, then the point of contact lies on
or OQ < OM
the straight line joining their centres.
 OP < OM
 OP is the shortest from all other lines drawn
from O to TPT.
 Our supposition in wrong.
Hence OP  TPT Q.E.D.
THEOREM 15.17
If two tangents are drawn from an external
point to a circle, then
(i) the tangents are equal in length.
(ii) the tangents subtend equal angles at the
centre of the circle.
(iii) the tangents are equally inclined to the
line joining the point and the centre of the
circle.
Given : P is an external point to a circle with
centre O. PA and PB are the two tangents
drawn from P to the circle at A and B. OP,
OA and OB are joined.
To Prove : (i) PA = PB Given : Two circles with centres O and C,
touch each other at P internally or externally.
(ii) AOP = BOP
To Prove : P lies on the line joining O andC.
386 Arun Deep's Understanding Math-10
Construction : Join OP, CP and draw a CAP = BDP
common tangent PT to the two circles at (Angles in the same segment)
point P.  APC ~ BPD
Proof : ... OP is the radius and PT is the (AA axiom of similarity)
tangent
PA PC
 OP  PT  
PD PB
 OPT = 90º ...(i)
Similarly, CP is the radius and PT is the  PA · PB = PC · PD.
tangent (ii) In fig. (ii)
 CPT = 90º ...(ii)
In fig. (i) (circles touching externally)
Adding (i) and (ii)
OPT + CPT = 90º + 90º = 180º
 OPC is a straight line.
In fig. (ii) (circles touching internally)
OPT = CPT = 90º
In APC and BPD
 OP and CP coincide each other.
APC = BPD (Common)
 OPC is a straight line. CAB = BDP
Hence P lies on the line joining O and C. (Ext. angle of a cyclic quad. is equal
Q.E.D. to its int. opposite angle)
THEOREM 15.19  APC ~ BPD
If two chords of a circle intersect each other (AA axiom of similarity)
internally or externally, then the product of
PA PC
the lengths of segments are equal.  
PD PB
Given : Two chords AB and CD of a circle
intersect internally [fig. (i)] or externally [fig.  PA · PB = PC · PD.
(ii)] at P. Hence in both the cases,
To Prove : PA · PB = PC · PD PA · PB = PC · PD. Q.E.D.
Construction : Join AC and BD. Note. This property is known as rectangle
property of a circle.
The above theorem can be stated in this way
also : (If two chords of a circle intersect
internally or externally, then the rectangle
formed by the two segment of one chord is
equal to the rectangle formed by the two
segments of the other chord.)
THEOREM 15.20
Proof : In fig. (i) In APC and BPD If a chord and a tangent intersect externally,
APC = BPD then the product of the lengths of segments
(Vertically opposite angles) of the chord is equal to the square of the
387 Arun Deep's Understanding Math-10
length of the tangent from the point of contact
to the point of intersection.
[If a chord AB and a tangent from T to the
circle intersect each other at P outside the
circle, then proved that PT2 = PA . PB]

To Prove : (i) RSP = RPT


(ii) RQP = TPR.
Construction : Join PO and produce it to
meet the circle at A. Join AR.
Given : A chord AB of a circle with centre O
and a tangent to the circle at T meets the Proof : ARP = 90º (angle in a semicircle)
chord AB produced at P.  In APR,
To Prove : PA · PB = PT2 PAR + APR = 90º ...(i)
Construction : Draw OD  AB and joint ... OP is the radius and TPT is the tangent to

OB, OP and OT. the circle.


Proof : ... OD  AB  OP  TPT
 D is the mid-point of AB  OPR + RPT = 90º
or AD = DB ...(i)  APR + RPT = 90º ...(ii)
Now PA × PB = (DP + AD) (DP – DB) From (i) and (ii)
= (DP + DB) (DP – DB) (From (i)) PAR + APR = APR + RPT
2
= DP – DB 2  PAR = RPT (... APR is common)
= (OP2 – OD2) – DB2 (In ODP) But RSP = PQR
= OP – OD – DB = OP – (OD2 + DB2)
2 2 2 2 (Angles in the same segment)
= OP2 – OB2  RSP = RPT
(... OB = OT) (radii of the same circle) Now PQRS is a cyclic quadrilateral
= OP2 – OT2 = PT2 (in rt. OPT)  RSP + RQP = 180º
Hence PA . PB = PT 2 Q.E.D. But RPT + RPT = 180º (Linear pair)
THEOREM 15.21  RSP + RQP = RPT + RPT
If a line touches a circle and from the  RQP = RPT
point of contact, a chord is drawn, (... RSP = RPT proved)
theangles between the tangent and the Hence RQP = TPR Q.E.D.
chord are respectively equal to the angles Note : Converse of the above theorem is also
in the corresponding alternate segments. true. If a line is drawn through an end of a
Given : In a circle with centre O, TPT is a chord of a circle so that the angle formed
tangent to the circle at the point P and PR is with the chord is equal to the angle subtended
the chord. S and Q are the points in the by the chord in the alternate segment, then
alternate segments. the line is a tangent to the circle.
388 Arun Deep's Understanding Math-10
EXERCISE 15.1
1. Using the given information, find the value  32º + 45º + x = 180º
of x in each of the following figures :  77º + x = 180º
 x = 180º – 77º = 103º
(iii) BAD = BCD
(Angles in the same segment)

D
A x°

69° 31°
B C

But BAD = 20°


 BCD = 20°
... CEA = 90°
C
A 2 5° D  CED = 90°
D x°

35° P Now in CED,
70° B CED + BCD + CDE = 180°
120° B
C A  90° + 20° + x = 180°
 110° + x = 180°
Sol. (i) ... ADB and ACB are in the same
 x = 180° – 110° = 70°
segment.
(iv) In ABC
 ADB = ACB = 50º
Now in ADB, ABC + ACB + BAC = 180°
DAB + x + ADB = 180º ( Sum of angles of a triangle)
(Angles of a triangle)  69° + 31 ° + BAC = 180°
 42º + x + 50º = 180º  BAC = 180° – 100°
 x + 92º = 180º  BAC = 80°
 x = 180º – 92º = 88º Since BAC and BAD are in the same
segment.
(ii) ABD = ACD
 BAD = x° = 80°
(Angles in the same segment)
(v) Given CPB = 120°, ACP = 70°
But ACD = 32º
To find, x° i.e., PDB
 ABD = 32º
Now in ABD, Reflex CPB = BPO + CPA
ABD + ADB + DAB = 180º  120° = BPD + BPD
(Angles in a triangle) ( BPD = CPA are vertically opposite s)
389 Arun Deep's Understanding Math-10

120
 2BPD = 120°  BPD = = 60°
2
Also ACP and PBD are in the same
segment
 PBD = ACP = 70°
Now, in PBD
PBD + PDB + BPD = 180°
(Sum of all s in a triangle)
 70° + x° + 60° = 180°
 x = 180° – 130°
Sol. (i) ACB = ADB
 x = 50°
(Angles in the same segment of a circle)
(vi) DAB = BCD
( Angles in the same segment of the circle) But ADB = xº
 DAB = 25° ( BCD = 25° given)  ACB = xº
In DAP, Now in ABC,
Ex. CDA = DAP + DPA CAB + ABC + ACB = 180º
 x° = DAB + DPA
 40º + 90º + xº = 180º
 x° = 25° + 35°
(AC is the diameter)
 x° = 60°
2. If O is the centre of the circle, find the  130º + xº = 180º
value of x in each of the following figures  xº = 180º – 130º = 50º
(using the given information) :
(ii) ACD = ABD
(Angles in the same segment)
 ACD = xº (... ABD = xº)
Now in OAC,
OA = OC [Radii of the same circle]

 ACO = OAC
[Opposite angles of equal sides]
 xº = 62º.
390 Arun Deep's Understanding Math-10
(iii) AOB + AOC + BOC = 360º (Angles at a point)
 AOB + 80º + 130º = 360º
 AOB + 210º = 360º
 AOB = 360º – 210º = 150º
Now arc AB subtends AOB at the centre ACB at the remaining part of the circle.
 AOB = 2 ACB
1 1
 ACB  AOB   150º  75º
2 2
(iv) ABC + CBD = 180º (Linear pair)
 ABC + 75º = 180º
 ABC = 180º – 75º = 105º
Now arc AC subtends reflex AOC at the centre and ABC at the remaining part of the circle.
 Reflex AOC = 2 ABC
= 2 × 105º = 210º
(v) AOC + COB = 180º (Linear pair)
 135º + COB = 180º
 COB = 180º – 135º = 45º
Now arc BC subtends COB at the centre and CDB at the remaining part of the circle
 COB = 2 CDB
1
 CDB  COB
2
1 45º 1
  45º   22 º
2 2 2
(vi) Arc AD subtends AOD at the centre and ACD at the remaining part of the circle
 AOD = 2 ACB
1 1
 ACB  AOD   70º  35º
2 2
. .. CMO = 90º
 AMC = 90º (... AMC + CMO = 180º)
391 Arun Deep's Understanding Math-10
Now in ACM,
ACM + AMC + CAM = 180º (Angles in a triangle)
 35º + 90º + xº = 180º
 125º + xº = 180º
xº = 180º – 125º = 55º
3. (a) In the figure (i) given below, AD || BC. If ACB = 35°. Find the measurement of DBC.
(b) In the figure (ii) given below, it is given that O is the centre of the circle and AOC = 130º.
Find ABC.
A
B

35°

C
D
( i)

Sol. (a) Construction : Join AB


A = C = 35° [ Alt. angles]

A
B

35°

C
D
Also B = A = 35° [ Angles in same segment]
ABC = 35° Ans.
(b) AOC + reflex AOC = 360º
 130º + reflex AOC = 360º
 Reflex AOC = 360º – 130º = 230º
Now arc AC subtends reflex AOC at the centre and ABC at the remaining part of the circle.
 Reflex AOC = 2 ABC

1
 ABC = reflex AOC
2

1
= × 230° = 115°
2
392 Arun Deep's Understanding Math-10
4. (a) In the figure (i) given below, calculate the values of x and y.
(b) In the figure (ii) given below, O is the centre of the circle. Calculate the values of x and y.

Sol. (a) ABCD is a cyclic quadrilateral

 B + D = 180º
 y + 40º + 45º = 180º  y + 85º = 180º
 y = 180º – 85º = 95º
ACB = ADB
(Angles in the same segment)
 xº = 40º
(b) Arc ADC subtends AOC at the centre and ABC at the remaining part of the circle
 AOC = 2 ABC
1 1
 ABC  AOC   120º  60º
2 2
 xº = 60º
Again ABCD is a cyclic quadrilateral
 B + D = 180º  60º + yº = 180º
 y = 180º – 60º = 120º.
5. (a) In the figure (i) given below, M, A, B, N are points on a circle having centre O. AN and MB
cut at Y. If NYB = 50º and YNB = 20º, find MAN and the reflex angle MON.
(b) In the figrue (ii) given below, O is the centre of the circle. If AOB = 140° and OAC = 50°,
find
(i) ACB (ii) OBC (iii) OAB (iv) CBA
393 Arun Deep's Understanding Math-10
Now major arc AB subtends AOB at the
B centre and ACB is at the remaining part of
the circle.
 Reflex AOB = 2 ACB
O 140° C 1
 ACB = reflex AOB
50° 2
1
A = × 220° = 110°
2
(i) (ii) (ii) In quad. OACB,
Sol. (a) NYB = 50º, YNB = 20º. OAC + ACB + AOB + OBC = 360°
 50° + 110° + 140° + OBC = 360°
 300° + OBC = 360°
 OBC = 360° – 300°
 OBC = 60°
(iii) In OAB,
OA = OB ( radii of the same circle)
 OAB = OBA
In YNB, But OAB + OBA + AOB = 180°
NYB + YNB + YBN = 180º  OAB + OAB + 140° = 180°
 50º + 20º + YBN = 180º  2 OAB = 180° – 140° = 40°
 YBN + 70º = 180º 40
 YBN = 180º – 70º = 110º  OAB = = 20°
2
But MAN = YBN (iv) But OBC = 60°
(Angles in the same segment)  CBA = OBC – OBA
 MAN = 110º = 60° – 20° = 40°
Major arc MN subtends reflex MON at the 6. (a) In the figure (i) given below, O is the
centre and MAN at the remaining part of centre of the circle and PBA = 42°.
the circle. Calculate the value of PQB
 Reflx MON = 2 MAN = 2 × 110º = 220º (b) In the figure (ii) given below, AB is a
(b) (i) AOB + reflex AOB = 360° diameter of the circle whose centre is O.
(Angles at a point) Given that ECD = EDC = 32°, calculate
 140° + reflex AOB = 360° (i) CEF (ii) COF.
 Reflex AOB = 360° – 140° = 220° P
B F
C

42°
A B
O
140°

O C A B
O E

50°
D
Q
A (i) (ii)
394 Arun Deep's Understanding Math-10
Sol. (a) In APB, Sol. (a) (i) PRB = BAP
APB = 90° (Angle in a semi-circle) (Angles in the same segment of the circle)
But A + APB + ABP = 180°  PRB = 35º (... BAP = 35º given)
(Angles of a triangle) (ii) In PRQ,
 A + 90° + 42° = 180° Ext. APR = PRQ + PQR
 A + 132° = 180° = PRB + Q
 A = 180° – 132° = 48° = 35º + 25º = 60º
But A = PQB But APB = 90º (Angle in a semi circle)
(Angles in the same segment of a circle)  BPR = APB – APR
 PQB = 48° = 90º – 60º = 30º
(b) (i) In EDC,
(iii) APR = ABR
Ext. CEF = ECD + EDC
(Angles in the same segment of the circle)
= 32° + 32° = 64°
 60º = ABR
(Ext. angle of a triangle is equal to the sum
of its interior opposite angles) In PBQ
(ii) arc CF subtends COF at the centre and Ext. PBR = Q + BPQ
CDF at the remaining part of the circle = 25º + 90º = 115º Ans.
 COF = 2 CDF = 2 CDE (b) B = D
= 2 × 32° = 64° (Angles in the same segment)
7. (a) In the figure (i) given below, AB is a  D = 40º
diameter of the circle APBR. APQ and RBQ ACD = 90º (Angle in the semi circle)
are straight lines, A = 35º, Q = 25º. Find Now in ADC,
(i) PRB (ii) PBR (iii) BPR. ACD + D + DAC = 180º
(b) In the figure (ii) given below, it is given (Angle in a triangle)
that ABC = 40º and AD is a diameter of
 90º + 40º + DAC = 180º
the circle. Calculate DAC.
 130º + DAC = 180º
 DAC = 180º – 130º = 50º
8. (a) In the figure given below, P and Q are
centres of two circles intersecting at B and
C. ACD is a st. line. Calculate the numerical
value of x.

(i)

(b) In the figure given below, O is the


cimcumcentre of triangle ABC in which AC
= BC. Given that ACB = 56º, calculate
395 Arun Deep's Understanding Math-10
(i) CAB (ii) OAC. 124
 A   62º or CAB = 62º
2
... OC is the radius of the circle

 OC bisects ACB
1 1
 OCA  ACB   56º  28º
2 2
Now in OAC,
OA = OC (radii of the same circle)
Sol. (a) Arc AB subtends APB at the centre
 OAC = OCA = 28º
and ACB at the remaining part of the circle
9. (a) In the figure (i) given below, chord ED
1 1 is parallel to the diameter AC of the circle.
 ACB  APB   130º  65º
2 2 Given CBE = 65º, calculate DEC.
But ACB + BCD = 180º
(Linear pair)
 65º + BCD = 180º
 BCD = 180º – 65º = 115º
Major arc BD subtends reflex BQD at the
centre and BCD at the remaining part of
the circle
 Reflex BQD = 2 BCD
(b) In the figure (ii) given below, C is a
= 2 × 115º = 230º
point on the minor arc AB of the circle with
But reflex angle BQD + x = 360º centre O. Given ACB = pº, AOB = qº,
(Angles at a point) express q in terms of p. Calculate p if OACB
 230º + x = 360º is a parallelogram.
 x = 360º – 230º = 130º Sol. (a) CBE = CAE
(b) Join OC (Angle in the same segment of a circle)
. .. In ABC, AC = BC  CAE = 65º
 A = B AEC = 90º (Angle in a semi circle)
But A + B + C = 180º Now in AEC
 A + A + 56º = 180º AEC + CAE + ACE = 180º
 2 A = 180º – 56º = 124º (Angle of a triangle)
 90º + 65º + ACE = 180º
 155º + ACE = 180º
ACE = 180º – 155º = 25º
... AC || ED (given)
 ACE = DEC (alternate angles)
 DEC = 25º
396 Arun Deep's Understanding Math-10
(b) Major arc AB subtends reflex AOB at Sol. (a) (i) CED = 90º (Angle in semi-circle)
the centre and ACB at the remaining part  In CED
of the circle.
CED + CDE + DCE = 180º
 90º + CDE + 40º = 180º
 130º + CDE = 180º
 CDE = 180º – 130º = 50º

 reflex AOB = 2 ACB = 2 p ...(i)


But reflex AOB + q = 360º
 Reflex AOB = 360º – q ...(ii)
From (i) and (ii)
2 p = 360º – q
q = 360º – 2 p = 2 (180º – p) (ii) In OBD,
If OACB is a parallelogram, then Ext. AOD = OBE + ODB
p=q  p = 360º – 2p  AOD = OBE + CDE
360º  75º = OBE + 50º
 3p = 360º  p  120º  OBE = 75º – 50º = 25º
3
 q = p = 120º (b) Join BI and CI
10. (a) In the figure (i) given below, straight In ABC,
lines AB and CD pass through the centre O BAC + ABC + ACB = 180º
of a circle. If OCE = 40º and AOD = (Angles of a triangle)
75º, find the number of degrees in :
(i) CDE (ii) OBE.
(b) In the figure (ii) given below, I is the
incentre of ABC. AI produced meets the
circumcircle of ABC at D. Given that
ABC = 55º and ACB = 65º, calculate
(i) BCD (ii) CBD
(iii) DCI (iv) BIC.

A
 BAC + 55º + 65º = 180º
 BAC = 180º – 120º = 60º
... I is incentre

I  I lies on the bisectors of angle of the


55
O

65
O ABC.
B C
60º
 BAD = CAD   30º
D 2
397 Arun Deep's Understanding Math-10
But BCD = BAD = 30º 1
(Angles in the same segment)  BOD = BOC ...(i)
2
Similarly CBD = CAD = 30º
1
55º 1 But 2A = BOC  A = BOC ...(ii)
and IBC   27 º 2
2 2 From (i) and (ii),
65º 1  BOD = A
and ICB   32 º Hence proved.
2 2
12. In the given figure, AB and CD are equal
FG
1 1
 BIC  180º  27 º 32 º
IJ chords. AD and BC intersect at E. Prove that
H
2 2 K AE = CE and BE = DE.
= 180º – 60º = 120º. A C
Now ICD = ICB + BCD,
1 1
 32 º 30º  62 o E
2 2
11. O is the circumcentre of the triangle ABC
and D is mid-point of the base BC. Prove
that BOD = A. B D
Sol. In the given figure, O is the centre of
Sol. In the given figure, AB and CD are two equal
circumcentre of ABC. D is mid-point of
chords AD and BC intersect each other at E.
BC. BO, CO and OD are joined.
To prove : AE = CE and BE = DE
A Proof : In AEB and CED
AB = CD (given)
A = C (angles in the same segment)
B = D (angles in the same segment)
O
 AEB CED (ASA axiom)
 AE = CE and BE = DE (c.p.c.t.)
B D C PQ. If circles are drawn taking two sides of a
triangle as diameters, prove that the other
point of intersection of these circles lies on
To prove : BOD = A the third side.
Proof : Arc BC subtends BOC on the centre A
and A on the remaining part of the circle.
 BOC = 2A
In OBD and BCO
OD = OD (common)
BD = CD (D is mid-point of BC) B D C
OB = OC (radii of the same circle) Sol. In the given figure,
 OBD BCO (SSS axiom) Two circles are drawn taking two sides AB
 BOD = COD (c.p.c.t.) and AC of ABC, as diameters
398 Arun Deep's Understanding Math-10
To prove : The other point of intersection lie perpendiculars on a line PQ, such that BD
on BC meets the circle at E.
Construction : Join AD.

B D C
Proof : AB is the diameter of the circle
 ADB = 90° ...(i)
Similarly, AC is the diameter
To Prove : AC = ED.
 ADC = 90° ...(ii)
Adding (i) and (ii) Construction : Join AE.
ADB + ADC = 90° + 90° = 180° Proof : AEB = 90º
It forms a linear pair (angle in a semi-circle)
 BD and DC are on the same straight line But ED  PQ (given)
 D, the other point of intersection of the  AE || PQ
circles lie on BC.
But BD and AC are perpendicular to PQ.
Hence proved.
 AEDC is a rectangle.
13. (a) In the figure (i) given below, AB is a
diameter of a circle with centre O. AC and Hence AC = ED
BD are perpendiculars on a line PQ. BD (Opposite sides of a rectangle) Q.E.D.
meets the circle at E. Prove that AC = ED. (b) AB || CD and O is the centre of the circle
(b) In the figure (ii) given below, O is the centre where AB is diameter, ABC = 25º. Join
of a circle. Chord CD is parallel to the OC and OD.
diameter AB. If ABC = 25º, calculate
Arc AC subtends AOC at the centre and
CED.
ABC at the remaining part of the circle.

 AOC = 2 ABC = 2 × 25º = 50º


But OCD = AOC (alternate angles)
= 50º
Sol. (a) Given : AB is the diameter of circle
But ODC = OCD = 50º
with centre O. AC and BD are
399 Arun Deep's Understanding Math-10
in OCD, OC = OD)
(. . . 15. (a) In the figure (i) given below, P is the
In OCD, point of intersection of the chords BC and
AQ such that AB = AP. Prove that
COD + OCD + ODC = 180º
CP = CQ.
(Angles of a triangle)
 COD + 50º + 50º = 180º
 COD + 100º = 180º
 COD = 180º – 100º = 80º
Now arc CD subtends COD at the centre
and CED at the remaining point of the
circle.
 COD = 2 CED  80º = 2 CED
1
 CED  80º   40º Ans. (b) In the figure (i) given below, AB = AC
2 = CD, ADC = 38º. Calculate :
14. In the adjoining figure, O is the centre of (i) ABC (ii) BEC.
the given circle and OABC is a
parallelogram. BC is produced to meet the
circle at D.
Prove that ABC = 2 OAD.

Sol. (a) Given : Two chords AQ and BC


Sol. Given : In the figure, intersect each other at P inside the circle.
OABC is a | | gm and O is the centre of the AB and CQ are joined and AB = AP.
circle. BC is produced to meet the circle at To Prove : CP = CQ
D. Construction : Join AC.
To Prove : ABC = 2OAD. Proof : In ABP and CQP
Construction : Join AD.  B = Q
Proof : Arc AC subtends AOC at the (Angles in the same segment)
centre and ADC at the remaining part of
BAP = PCQ
the circle.
(Angles in the same segment)
AOC = 2 ADC
BPA = CPQ
But OAD = ADC (alternate angles)
... AOC = 2 OAD (Vertically opposite angles)
 ABP ~ CQP
But ABC = AOC
(AAA axioms of similarity)
(opposite angles of a | | gm)
 ABC = 2 OAD AB AP
 
Hence proved. CQ CP
400 Arun Deep's Understanding Math-10
But AB = AP (given) Proof : ... ACB and ADB are in the
 CQ = CP same segment of the circle
(b) AB = AC = CD, ADC = 38º ACB = ADB
In ACD, AC = CD (given) Similarly, ACP and ADP are in the
same segment
 CAD = ADC = 38º
ACP = ADP
and ext. ACB = CAD + ADC
= 38º + 38º = 76º 1
But ACP = ACB
In ABC, AB = AC 2
 ABC = ACB = 76º (... CP is the angle bisector)
and BAC = 180º – (ABC + ACB) and ACB = ADB (proved)
= 180º – (76º + 76º) = 180º – 152º = 28º 1
But BEC = BAC ADP = ADB
2
(Angles in the same segment)
Hence, DP bisects ADB
 BEC = 28º
Hence proved.
16. (a) In the figure (i) given below,
(b) Given : In the figure, ABC is inscribed
CP bisects ACB. Prove that DP bisects
in a circle. BD bisects ABC.
ADB.
(b) In the figure (ii) given below, BD
AB BE
bisects ABC. Prove that  .
BD BC

AB BE
To Prove : 
BD BC
Construction : Join CD.
Proof : In ABE and BCD,
Sol. (a) Given : In the figure, CP is the A = D
bisector of ACB meeting the circle at (Angles in the same segment)
P. PD is joined ABE = BDC
(... BD is the bisector of ABC)
ABE ~ BAC (AA axiom)
AB BE
 
BD BC
Hence proved.
17. (a) In the figure (ii) given below, chords
AB and CD of a circle intersect at E.
(i) Prove that triangles ADE and CBE are
Prove : DP bisects ADB similar.
401 Arun Deep's Understanding Math-10
(ii) Given DC = 12 cm, DE = 4 cm and Proof : (i) In ADE and CBE
AE = 16 cm, calculate the length of BE. D = B
{Angles in the same segment}
A = C {Angles in the same segment}
 ADE ~ CBE
(AA axiom of similarity)
(ii) DC = 12 cm, DE = 4 cm.
 EC = 12 – 4 = 8 cm.
AE = 16 cm.
... Chords AB and CD intersect each other

at E.
(b) In the figure (ii) given below, AB and CD
are two intersecting chords of a circle.  AE × EB = CE × ED
Name two triangles which are similar.  16 × EB = 8 × 4
Hence, calculate CP given that AP = 6cm,
84
PB = 4 cm, and CD = 14 cm (PC > PD).  EB  = 2 cm
16
(b) Now in APD and CPB,
DAB = DCB
CDA = CBA
{Angles in the same segment of the circle}

Sol. (a) Given : Two chords AB and CD


intersect each other at E inside the circle.

 APD ~ CPB


(AA axiom of similarity)
AP CP

PD PB
To Prove : (i) ADE ~ CBE  AP × PB = CP × PD
(ii) If DC = 12 cm, DE = 4 cm and  6 × 4 = x (14 – x)
AE = 16 cm, calculate the length of BE.  24 = 14 x – x2
402 Arun Deep's Understanding Math-10
 x2 – 14 x + 24 = 0  (5) = AD2 + (4)2  25 = AD2 + 16
2

 x2 – 12 x – 2 x + 24 = 0  AD2 = 25 – 16 = 9 = (3)2
 x (x – 12) – 2 (x – 12) = 0  AD = 3 cm
 (x – 12) (x – 2) = 0 Chords AE and BC of the circle intersect each
Either x – 12 = 0, other at D inside the circle
then x = 12  AD × DE = BD × DC
or x – 2 = 0,  3 × DE = 4 × 9
then x = 2 49 36
 DE = =
 CP = 12 cm or 2 cm 3 3
But CP > PD (given)  DE = 12 cm
 CP = 12 cm 19. (a) In the figure (i) given below, PR is
18. In the adjoining figure, AE and BC intersect a diameter of the circle, PQ = 7 cm,
each other at point D. If CDE = 90º, QR = 6 cm and RS = 2 cm. Calculate the
AB = 5 cm, BD = 4 cm and CD = 9 cm, find perimeter of the cyclic quadrilateral
DE. (2008) PQRS.
C (b) In the figure (ii) given below, the
diagonals of a cyclic q ua drilateral
ABCD interse ct i n P an d the are a
o f t h e t ri a n gl e A PB i s 2 4 cm 2 . If
AB = 8 cm and CD = 5 cm, calculate
the area of DPC.
A D E

B
Sol. In the figure, AE and BC intersect each other
at D. AB is joined.
C
9 cm

A 5
D cm E
4 cm
B
CDE = 90º, AB = 5 cm
BD = 4 cm
and CD = 9 cm
In right ADB,
AB2 = AD2 + BD2 (Pythagoras Theorem)
403 Arun Deep's Understanding Math-10
Sol. (a) PR is the diameter of the circle  APB ~ DPC
PQ = 7 cm, QR = 6 cm, RS = 2 cm. (AA axiom of similarity)

area of ΔAPB AB2


 
area of ΔDPC CD 2

24 (8) 2
 
area of ΔDPC (5) 2

24 64
 
area of ΔDPC 25
 area of DPC
In PQR, 24  25 75 3
   9 cm 2 Ans.
Q = 90º 64 8 8
(Angle in a semi-circle) 20. (a) In the figure (i) given below, QPX is
 PR2 PQ2
= + QR2 = (7)2 + (6)2 the bisector of YXZ of the triangle XYZ.
Prove that XY : XQ = XP : XZ.
= 49 + 36 = 85
(b) In the figure (ii) given below, chords
Again in PSR,
BA and DC of a circle meet at P. Prove
S = 90º that:
(Angle in a semi-circle) (i) PAD = PCB (ii) PA . PB = PC . PD.
 PR = PS + RS2
2 2

 85 = PS2 + (2)2  85 = PS2 + 4


 PS2 = 85 – 4 = 81 = (9)2
 PS = 9 cm
Now the perimeter of PQRS,
= 7 + 6 + 2 + 9 = 24 cm Ans.
(b) In APB and DPC

Sol. (a) Given : XYZ is inscribed in a circle.


Bisector of YXZ meets the circle at Q.
QY is joined.
To Prove : XY : XQ = XP : XZ

APB = DPC
(Vertically opposite angles)
ABP = DCP
(Angles in the same segment)
404 Arun Deep's Understanding Math-10
Proof : In XYQ and XPZ EXERCISE 15.2
Q = Z
1. If O is the centre of the circle, find the
(Angles in the same segment) value of x in each of the following figures
YXQ = PXZ (using the given information) :
(... XQ is the bisector of YXZ)
 XYQ ~ XPZ

XY XQ XY XP
   
XP XZ XQ XZ
 XY : XQ = XP : XZ Q.E.D.
(b) Given : Two chords BA and DC meet
each other at P outside the circle. AD and
BC are joined.
(i) (ii)

To Prove : (i) PAD = PCB


(ii) PA . PB = PC . PD
(iii)
Proof : PAD + DAB = PCB + BCD
Sol. (i) ABCD is a cyclic quadrilateral
(each 180º)
But DAB = BCD
(angle in the same segment)
 PAD = PCB
Now in PBC and PAD
PCB = PAD (proved)
P = P (common)
 PBC ~ PAD
 Ext. DCE = BAD
(AA axiom of similarity)
 BAD = xº
PC PB Now arc BD subtends BOD at the centre
 
PA PD and BAD at the remaining part of the
circle.
 PA . PB = PC . PD Q.E.D.
 BOD = 2 BAD = 2 x
405 Arun Deep's Understanding Math-10
 2 x = 150º  x = 75º centre of the circle. If AOC = 150°, find
(ii) BCD + DCE = 180º (i) ABC (ii) ADC.
(Linear pair)
B

O
150°
A C

(b) In the figure (i) given below, AC is a


diameter of the given circle and
 BCD + 80º = 180º
BCD = 75º. Calculate the size of
 BCD = 180º – 80º = 100º.
... Arc BAD subtends reflex BOD at the
(i) ABC (ii) EAF.
centre and BCD at the remaining part of
the circle.
 Reflex BOD = 2 BCD
 xº = 2 × 100º = 200º.
(iii) In ACB,

Sol. (a) Given, AOC = 150° and AD = CD


We know that, an angle subtend by an arc
of a circle at the centre is twice of the angle
subtend by the same arc at any point on
the remaining part of the circle.
CAB + ABC + ACB = 180º (i)  AOC = 2 × ABC
(Angles of a triangle)
AOC 150
But ACB = 90º (Angle in a semicircle) ABC =  = 75°
2 2
 25º + 90º + ABC = 180º
(ii) From figure, ABCD is a cyclic quadrilateral
 115º + ABC = 180º
 ABC + ADC = 180°
 ABC = 180º – 115º = 65º
... ABCD is a cyclic quadrilateral
[ Sum of opposite angles in a cyclic
quadrilateral is 180°]
 ABC + ADC = 180º  75° + ADC = 180°
(Opposite angles of a cyclic quadrilateral) ADC = 180° – 75°
 65º + xº = 180º  ADC = 105°
 xº = 180º – 65º = 115º (b) (i) ... AC is the diameter of the circle.
2. (a) In the figure (i) given below, O is the  ABC = 90º (Angle in a semi circle)
406 Arun Deep's Understanding Math-10
(ii) ... ABCD is a cyclic quadrilateral
A
 BAD + BCD = 180º
 BAD + 75º = 180º 32º D
(. . . BCD = 75º) 32º
 BAD = 180º – 75º = 105º
58º 90º
But EAF = BAD B C
(Vertically opposite angles) 148º

EAF = 105º E
3. (a) In the figure, (i) given below, if (ii) BEC = 180º – 32º = 148º
DBC = 58º and BD is a diameter of the (opp. angle of cyclic quadrilateral)
circle, calculate: (iii) BAC = BDC = 32º
(i) BDC (ii) BEC (iii) BAC (Angles in same segment)
(b) In the figure, AB || DC
A
BCE = 80º and BAC = 25º
D ABCD is a cyclic quadrilateral and DC is
produced to E

A B
B C 25º

E
80º
(b) In the figure (ii) given below, AB is parallel E
D C
to DC, BCE = 80º and BAC = 25º. Find :
(i) Ext. BCE = Interior A
(i) CAD (ii) CBD
 80º = BAC + CAD
(iii) ADC (2008)
 80º = 25º + CAD
 CAD = 80º – 25º = 55º
A B (ii) But CAD = CBD
25º
(Alternate angles)
 CBD = 55º
80º
E
(iii) BAC = BDC
D C (Angles in the same segments)
Sol. (a) DBC = 58º  BDC = 25º ( BAC = 25º)
BD is diameter Now AB || DC and BD is the transversal
 DCB = 90º (Angle in semi circle)  BDC = ABD (Alternate angles)
(i) In BDC ABD = 25º
BDC + DCB + CBD = 180º  ABC = ABD + CBD = 25º + 55º = 80º
BDC = 180º – 90º – 58º = 32º But ABC + ADC = 180º
407 Arun Deep's Understanding Math-10
(Opposite angles of a cyclic quadrilateral)  ABC = 180° – 80° = 100°
 80º + ADC = 180º In ADC,
 ADC = 180º – 80º = 100º DAC = 180° – ADC + ACD
4. (a) In the figure given below, ABCD is a cyclic = 180° – (80° + 52°) = 180° – 132° = 48°
quadrilateral. If ADC = 80° and ACD = But CBD = DAC
52°, find the values of ABC and CBD.
(Angles in the same segment)
 CBD = 48°
C
D
80° 52° (b) In the given figure, O is the centre of the
circle.

E
A B
51° 150°
(b) In the figure given below, O is the centre of
A C
the circle. AOE = 150°, DAO = 51°. O B
Calculate the sizes of BEC and EBC.

E AOE = 150°, DAO = 51°


To find BEC and EBC
A C ABED is a cyclic quadrilateral
O B
 Ext. BEC = DAB = 51°
AOE = 150°
 Ref. AOE = 360° – 150° = 210°
Sol. (a) In the given figure, ABCD is a cyclic Now arc ABE subtends AOE at the centre
quadrilateral ADC = 80° and ACD = 52°
and ADE at the remaining part of the circle.
To find the measure of ABC and CBD
1 1
 ADE = Ref. AOE = × 210° = 105°
C 2 2
D
80° 52°
But Ext. EBC = ADE = 105°
Hence BEC = 51° and EBC = 105°
5. (a) In the figure (i) given below, ABCD is a
parallelogram. A circle passes through A and
B D and cuts AB at E and DC at F. Given that
A
BEF = 80º, find ABC.
ABCD is a cyclic quadrilateral (b) In the figure (ii) given below, ABCD is
 ABC + ADC = 180° a cyclic trapezium in which AD is parallel
(sum of opposite angles = 180°) to BC and B = 70º, find :
 ABC + 80° = 180° (i) BAD (ii) BCD.
408 Arun Deep's Understanding Math-10
(i) QBC (ii) BCP.

Sol. (a) ADFE is a cyclic quadrilateral Sol. (a) (i) ABCD is a cyclic quadrilateral
 Ext. FEB = ADF
 ADF = 80º
... ABCD is a parallelogram

 B = D = ADF = 80º


or ABC = 80º
(b) ... In trapezium ABCD, AD || BC
(i)  B + A = 180º
 70º + A = 180º
 A = 180º – 70º = 110º
 BAD = 110º  A + C = 180º
(ii) ABCD is a cyclic quadrilateral  30º + p = 180º
 A + C = 180º  p = 180º – 30º = 150º
 110º + C = 180º (ii) Arc BD subtends BOD at the centre
and BAD at the remaining part of the circle
 C = 180º – 110º = 70º
 BOD = 2 BAD
 BCD = 70º
 q = 2 × 30° = 60°
6. (a) In the figure given below, O is the centre
of the circle. If BAD = 30º, find the values BAD and BED are in the same segment
of p, q and r. of the circle
 BAD = BED  30° = r
 r = 30°
(b) Join PQ
AQPD is a cyclic quadrilateral.

(b) In the figure given below, two circles


intersect at points P and Q. If A = 80º
and D = 84º, calculate  A + QPD = 180º
409 Arun Deep's Understanding Math-10
 80º + QPD = 180º  SRP = RPQ = 32º (alternate angles)
 QPD = 180º – 80º = 100º Now PRST is a cyclic quadrilateral,
and D + AQP = 180º  STP + SRP = 180º
 84º + AQP = 180º  STP + 32º = 180º
 AQP = 180º – 84º = 96º.  STP = 180º – 32º = 148º
Now PQBC is a cyclic quadrilateral,
(b) In the given figure,
 Ext. QPD = QBC
ACE = 43° and CAF = 62°
 QBC = 100º.
Now, in AEC,
and ext. AQP = BCP
ACE + CAE + AEC = 180°
 BCP = 96º
7. (a) In the figure given below, PQ is a  43° + 62° + AEC = 180°
diameter. Chord SR is parallel to PQ.Given  105° + AEC = 180°
PQR = 58º, calculate  AEC = 180° – 105° = 75°
(i) RPQ (ii) STP But ABD + AED = 180°
(T is a point on the minor arc SP)
(Sum of opposite angles of a cyclic
quadrilateral)
and AED = AEC
 a + 75° = 180°
 a = 180° – 75° = 105°
But EDF = BAE
(Angles in the alternate segment)
(b) In the figure given below, if ACE = 43°
and CAF = 62°, find the values of a, b  c = 62°
and c. (2007) Now in BAF
C a + 62° + b = 180°
 105° + 62° + b = 180°
43°

 167° + b = 180°
B
 b = 180° – 167° = 13°
a D Hence a = 105°,
c
b = 13°, c = 62°
62° b 8. (a) In the figure (i) given below, AB is a
F
A E diameter of the circle. If ADC = 120°,
find CAB.
Sol. (a) In PQR,
(b) In the figure (ii) given below, sides AB
PRQ = 90º (Angle in a semi circle) and DC of a cyclic quadrilateral ABCD are
and PQR = 58º produced to meet at E, the sides AD and
 RPQ = 90º – PQR = 90º – 58º = 32º BC are produced to meet at F. If x : y : z =
. .. SR || PQ (given) 3 : 4 : 5, find the values of x, y and z.
410 Arun Deep's Understanding Math-10
D C
120° E

A B

(i)

Now In DCF,
Ext. 2 = x + z ...(i)
and in CBE
Ext. 1 = x + y ...(ii)
Adding (i) and (ii)
x + y + x + z = 1 + 2
 2 x + y + z = 180º
Sol. (a) Construction : Join BC, and AC (... ABCD is a cyclic quad.) ...(iii)
then ABCD is a cyclic quadrilateral. But x : y : z = 3 : 4 : 5
x 3 4
D C    y x.
E
y 4 3
120°
x 3 5
A B and   z  x.
z 5 3
 Substituting the value of y and z in (iii)
4 5
In quadrilateral ABCD 2x  x  x  180º
3 3
ABC + ADC = 180°
 6 x + 4 x + 5 x = 180º × 3
( Sum of opposite s of a cyclic
quadrilateral)  15 x = 180º × 3
 ABC + 120° = 180°  ABC = 60° 180  3
 x  36º .
Now in ABC 15
ABC + ACB + CAB = 180° 4 4
 y x  36º   48º
( In ABC, ACB = 90°) 3 3
(angle in a semicircle)
5 5
 CAB + 60° + 90° = 180° and z x  36º   60º
3 3
 CAB = 30° Hence x = 36º, y = 48º and z = 60º
(b) DCF = BCE = x 9. (a) In the figure (i) given below, ABCD is a
(Vertically opposite angle) quadrilateral inscribed in a circle with centre
411 Arun Deep's Understanding Math-10
O. CD is produced to E. If ADE = 70º Also, AOC = 140°
and OBA = 45º, calculate (Angle subtended by an arc of a circle at
(i) OCA (ii) BAC the centre is double than the angle drawn
(b) In figure (ii) given below, ABF is a subtended by it at any point on the remaining
straight line and BE || DC. If DAB = 92º part of a circle)
and EBF = 20°, find : Now in OAC,
(i) BCD (ii) ADC. OAC + OCA + AOC = 180°
( Sum of all s of a triangle)
 OAC + OCA + 140° = 180°
 OAC + OAC + 140° = 180°
(OA = OC, radii of same circle implies that
angle opposite to equal sides are equal)
 2OAC = 40°  OAC = 20°
Similarly in OAB,
 OAB = OBA = 45°
Sol. (a) ABCD is a cyclic quadrilateral
( OA = OB, radii of same circle)
Now BAC = OAB + OAC
BAC = 45° + 20°
BAC = 65°
(b) ABCD is a cyclic quadrilateral
 BCD + DAB = 180º

Given, ABCD is a quadrilateral inscribed in


a circle with centre O. Also, CD is a chord
produced to E.
ADE = 70° and OBA = 45°
ADE + ADC = 180° (Linear pair)  BCD + 92º = 180º
 ADC = 180° – 70° = 110°  BCD = 180º – 92º = 88º.
Again, D + B = 180° Again Ext. CBF = ADC
(Opposite angles of a cyclic quadrilateral  ADC = CBF = CBE + EBF
are supplementary) = BCD + EBF
 110° + B = 180°
(... DC || BE and alternate angles are equal)
 B = 70°
= 88º + 20º = 108º
 OBA + OBC = 70°
10. (a) In the figure (ii) given below, PQRS is
 45° + OBC = 70° a cyclic quadrilateral in which PQ = QR
 OBC = 70° – 45° and RS is produced to T. If QPR = 52º,
 OBC = 25° calculate PST.
412 Arun Deep's Understanding Math-10

B
C
x

y O

D
o
50

(b) In the figure (ii) given below, O is the centre A


of the circle. If OAD = 50º, find the values
In OAD, OA = OD
of x and y.
(Radii of the same circle)
B  ODA = OAD = 50º.
C
x
Now Ext. BOD = ODA + OAD,
 y = 50º + 50º = 100º
y O 11. (a) In the figure (i) given below, O is the
centre of the circle. If COD = 40º and
D
o
CBE = 100º, then find :
50
(i) ADC (ii) DAC
A
(iii) ODA (iv) OCA.
Sol. (a) PQRS is a cyclic quadrilateral in which (b) In the figure (ii) given below, O is the
centre of the circle. If BAD = 75° and
PQ = QR
BC = CD, find :
(i) BOD (ii) BCD
(iii) BOC (iv) OBD (2009)

75°

O
B
 in PQR
QPR = QRP = 52º D
 PQR = 180º – (QPR + QRP)
C
= 180º – (52º + 52º) = 180º – 104º = 76º
Now in cyclic quad. PQRS, Sol. (a) (i) ... ABCD is a cyclic quadrilateral.
Ext. PST = PQR = 76º  Ext. CBE = ADC
(b) ABCD is a cyclic quadrilateral  ADC = 100º
 OAD + BCD = 180º (ii) Arc CD subtends COD at the centre
 50º + x = 180º and CAD at the remaining part of the circle
 x = 180º – 50º = 130º  COD = 2 CAD
413 Arun Deep's Understanding Math-10
1 1 (ii) In quadrilateral ABCD,
 CAD  COD   40º  20º BCD + BAD = 180°
2 2
(Sum of opposite s in quadrilateral)
 BCD = 180° – 75°
 BCD = 105°
1
(iii) BOC = BOD
2

1
BOC = × 150° = 75°
2
(iii) In COD, OC = OD (iv) In OBD
(radii of the same circle) OBD + ODC + BOD = 180°
 CDO = DCO (Sum of all s of a triangle is 180°)
But CDO + DCO + COD = 180º  OBD + ODC + 150° = 180°
 CDO + CDO + COD = 180º  OBD + OBD + 150° = 180°
 2 CDO + 40º = 180º ( BC = CD)
 2 CDO = 180º – 40º = 140º  2OBD = 30°
140º  OBD = 15°
 CDO   70º 12. In the given figure, O is the centre and AOE
2
is the diameter of the semicircle ABCDE. If
Now ODA = ADC – CDO, AB = BC and AEC = 50°, find :
= 100º – 70º = 30º (i) CBE (ii) CDE
(iv) OCA = OCD – ACD (iii) AOB.
= 70º – {180º – ADC – CAD} Prove that OB is parallel to EC.
= 70º – {180º – 100º – 20º} C
= 70º – {180º – 120º} = 70º – 60º = 10º. B D
(b) Given BAD = 75°, chord BC = chord CD
Construction : Join OC and BD A 50°
E
O
A Sol. In the given figure, O is the centre of the
75° semi-circle ABCDE and AOE is the diameter.
AB = BC, AEC = 50°
O C
B
B D
D
50°
C A E
O

(i) BOD = 2 × BAD To prove : OB || EC and find


(i) CBE, (ii) CDE and AOB
BOD = 2 × 75° = 150°
414 Arun Deep's Understanding Math-10
Proof : AECB is a cyclic quadrilateral quadrilateral. Prove that SQ = RS.
 AEC + ABC = 180°
 50° + ABE + EBC = 180°
 50° + 90° + EBC = 180°
 140° + EBC = 180°
( ABE is in a semi-circle)
 EBC = 180° – 140° = 40°
But BEDC is a cyclic quadrilateral
 EBC + CDE = 180°
40° + CDE = 180°
 CDE = 180° – 40° = 140°
Sol. (a) Given : Chord BC || ED, ABE and ACD
AB = BC
are straight lines.
1 To Prove : AED is an isosceles triangle.
 AEB = BEC = AEC
2
Proof :  BCDE is a cyclic quadrilateral.
1  Ext. ABC = D ...(i)
= × 50° = 25°
2
But BC || ED (given)
In OBE,
OB = OE (radii of the same circle)
 OBE = OEB = 25°
and Ext. AOB = OBE + OEB
= 25° + 25° = 50°
 AOB = 50°
AOB = OEC (each = 50°)
But these are corresponding angles
 OB || EC
13. (a) In the figure (i) given below, ED and
BC are two parallel chords of the circle and  ABC = E
ABE, ACD are two st. lines. Prove that AED (corresponding angle) ...(ii)
is an isosceles triangle.
From (i) and (ii)
D = E
In AED,
D = E (proved)
 AE = AD
(sides opposite to equal angles)
Hence AED is an isosceles triangle. Q.E.D.
(b) Given : In a circle, PQRS is a cyclic
(b) In the figure (ii) given below, SP is the quadrilateral. QP is produced to T such that
bisector of RPT and PQRS is a cyclic PS is the bisector of RPT.
415 Arun Deep's Understanding Math-10
at D and E.
To prove : DE || BC
Construction : Join DE.
AB = AC
 B = C (angles opposite to equal sides)
But BCED is a cyclic quadrilateral
 Ext. ADE = C
= B ( C = B)
To Prove : SQ = RS But these are corresponding angles
Proof : ... PQRS is a cyclic quadrilateral.  DE || BC
 Ext. TPS = SRQ ...(i) Hence proved.
and RPS = RQS ...(ii) 15. (a) Prove that a cyclic parallelogram is a
(Angles in the same segment) rectangle.
But TPS = RPS (b) Prove that a cyclic rhombus is a square.
(... PS is the bisector of TPR) Sol. (a) ABCD is a cyclic parallelogram.
 from (i) and (ii) D C
SRQ = RQS
Now in SQR,
SRQ = RQS proved.
A B
 SQ = RS
(sides opposite to equal angles) To prove : ABCD is a rectangle
Q.E.D. Proof : ABCD is a parallelogram
14. In the given figure, ABC is an isosceles  A = C and B = D
triangle in which AB = AC and circle passing (opposite angles are equal)
through B and C intersects sides AB and AC  ABCD is a cyclic quadrilateral
at points D and E. Prove that DE || BC.  A + C = 180°
 A + A = 180° ( A = C)
A
 2A = 180°
180
D E  A = = 90°
2
 C = A = 90°
Similarly, we can prove that
B = D = 90°
 ABCD is a rectangle
B C (b) Given : ABCD is a cyclic rhombus
To prove : ABCD is a square
Sol. In the given figure, ABC is an isosceles Proof : ABCD is a rhombus
triangle in which AB = AC. A circle passing  A = C and B = D
through B and C intersects sides AB and AC (opposite angles are equal)
416 Arun Deep's Understanding Math-10

D C C
2 cm

D
6c
m
A B

A O
But ABCD is a cyclic quadrialteral B 3 cm
 A + C = 180° and B + D = 180°
 A + A = 180° ( A = C) Proof :
 2A = 180° (i) In ODB and OAC
180 Ext. ODB = A
 A = = 90°
2 (Ext. angle of a cyclic quadrilateral is equal
 C = A = 90° to its interior opposite angle)
Similarly we can prove that Similarly, OBD = C
B = D = 90° O = O (common)
 ABCD is a square.  ODB ~ OAC (AAA axiom)
16. In the given figure, chords AB and CD of the OD OB 6 3
circle are produced to meet at O. Prove that (ii)  =  =
OA OC OA 62
triangles ODB and OAC are similar. Given
that CD = 2 cm, DO = 6 cm and BO = 3 cm, 6 3 68
 =  OA = = 16
area of quad. CABD OA 8 3
calculate AB. Also find area of ΔOAC .  AB = OA – OB = 16 – 3 = 13 cm
(iii) OAC ~ ODB
C
area ΔOAC OC 2 82 64
 area ΔODB = 2 = 2 = ...(i)
D OB 3 9
Subtracting from both sides, we get
area ΔOAC 64
O
area ΔODB – 1 = 9 – 1
A B

Sol. In the given figure, AB and CD are chords of area ΔOAC  area ΔODB 64  9
area ΔODB =
a circle. They are produced to meet at O. 9
To prove : (i) ODB ~ OAC
If CD = 2 cm, DO = 6 cm, and BO = 3 cm area quadrilateral CABD 55
area ΔOBD =
To find : AB and also area of the 9

Quad. ABCD area quadrilateral CABD 55


area of ΔOAC and area ΔOAC =
64
Construction : Join AC and BD [From (i)]
417 Arun Deep's Understanding Math-10
EXERCISE 15.3
1. Find the length of the tangent drawn to a  (13)2 = (CT)2 + (12)2
circle of radius 3 cm, from a point distant  169 = (CT)2 + 144
5 cm from the centre.  (CT)2 = 169 – 144 = 25 = (5)2
Sol. In a circle with centre O and radius 3 cm  CT = 5 cm.
and P is at a distance of 5 cm.
Hence radius of the circle = 5 cm. Ans.
3. The tangent to a circle of radius 6 cm from
an external point P, is of length 8 cm.
Calculate the distance of P from the nearest
point of the circle.

i.e. OT = 3 cm, OP = 5 cm
... OT is the radius of the circle

 OT  PT
Now in right OTP, by Pythagorus axiom,
OP2 = OT2 + PT2
 (5)2 = (3)2 + PT2
 PT2 = (5)2 – (3)2 = 25 – 9 = 16 = (4)2
 PT = 4 cm.
Sol. Radius of the circle = 6 cm
2. A point P is at a distance 13 cm from the
centre C of a circle and PT is a tangent to and length of tangent = 8 cm
the given circle. If PT = 12 cm, find the Let OP be the distance
radius of the circle. i.e. OA = 6 cm, AP = 8 cm
Sol. CT is the radius ... OA is the radius
CP = 13 cm and tangent PT = 12 cm.  OA  AP.
Now in right OAP,
OP2 = OA2 + AP2 (By Pythagorus axiom)
= (6)2 + (8)2 = 36 + 64 = 100 = (10)2
 OP = 10 cm. Ans.
4. Two concentric circles are of the radii 13
cm and 5 cm. Find the length of the chord
. .. CT is the radius and TP is the tangent of the outer circle which touches the inner
circle.
 CT  TP.
Sol. Two concentric circles with centre O and
Now in right CPT, OP and OB are the radii of the circles
(CP)2 = (CT)2 + (PT)2 respectively, then
(Pythagorus axiom) OP = 5 cm, OB = 13 cm.
418 Arun Deep's Understanding Math-10
(b) In figure (ii) given below, quadrilateral
ABCD is circumscribed, find x.

AB is the chord of the outer circle which


touches the inner circle at P.
... OP is the radius and APB is the tangent
Sol. (a) From A, AP and AQ are the tangents to
to the inner circle. the circle
 In right OPB, by Pythagorus axioms,  AQ = AP = 4 cm.
(OB)2 = (OP)2 + (PB)2
 (13)2 = (5)2 + (PB)2
 169 = 25 + (PB)2  (PB)2 = 169 – 25 = 144
 (PB)2 = 144 = (12)2
 PB = 12 cm.
But P is the mid-point of AB.
 AB = 2 × PB = 2 × 12 = 24 cm Ans.
But AC = 12 cm
5. Two circles of radii 5 cm and 2·8 cm touch
 CQ = 12 – 4 = 8 cm.
each other. Find the distance between their
From B, BP and BR are the tangents to the
centres if they touch :
circle
(i) externally (ii) internally.  BR = BP = 6 cm
Sol. Radii of the circles are 5 cm and 2·8 cm. Similarly, from C,
i.e. OP = 5 cm and CP = 2·8 cm. CQ and CR the tangents
 CR = CQ = 8 cm.
 x = BC = BR + CR = 6 cm + 8 cm = 14 cm Ans.
5cm
(b) From C, CR and CS are the tangents to
2.8cm
the circle

(i) When the circles touch externally, then


the distance between their centres = OC
= 5 + 2·8 = 7·8 cm.
(ii) When the circles touch internally, then
the distance between their centres = OC  CS = CR = 3 cm.
= 5·0 – 2·8 = 2·2 cm Ans. But BC = 7 cm.
6. (a) In figure (i) given below, triangle ABC  BS = BC – CS = 7 – 3 = 4 cm.
is circumscribed, find x.
419 Arun Deep's Understanding Math-10
Now from B, BP and BS are the tangents  SD = OS = 10 cm.
to the circle
 BP = BS = 4 cm
and from A, AP and AQ are the tangents to the
circle.
 AP = AQ = 5 cm.
x = AB = AP + BP = 5 cm + 4 cm
= 9 cm Ans.
7. (a) In figure (i) given below, quadrilateral
ABCD is circumscribed ; find the perimeter Now from D, DR and DS are the tangents
of quadrilateral ABCD.
to the circle.
(b) In figure (ii) given below, quadrilateral
 DR = DS = 10 cm.
ABCD is circumscribed and AD  DC ;
find x if radius of incircle is 10 cm. From B, BP and BQ are tangents to the
circle.
 BQ = BP = 27 cm.
 CQ = CB – BQ = 38 – 27 = 11 cm.
Now from C, CQ and CR are the tangents
to the circle.
 CR = CQ = 11 cm.
 DC = x = DR + CR
= 10 + 11 = 21 cm. Ans.

Sol. (a) From A, AP and AS are the tangents to 8. (a) In the figure (i) given below, O is the
the circle centre of the circle and AB is a tangent at
B. If AB = 15 cm andAC = 7.5 cm, find the
 AS = AP = 6
radius of the circle.
From B, BP and BQ are the tangents
(b) In the figure (ii) given below, from an
 BQ = BP = 5
external point P, tangents PA and PB are
From C, CQ and CR are the tangents
drawn to a circle. CE is a tangent to the
 CR = CQ = 3 circle at D. If AP = 15 cm, find the
and from D, DS and DR are the tangents. perimeter of the triangle PEC.
 DS = DR = 4
 Perimeter of the quad. ABCD
B A E
= (6 + 5 + 5 + 3 + 3 + 4 + 4 + 6) cm. = 36 cm. P
D
(b) In the circle with centre O, radius OS
= 10 cm, ADC = 90º D A
O C C

PB = 27 cm, BC = 38 cm. B

. .. OS is radius and AD is the tangent. (ii)


(i)
 OS  AD.
420 Arun Deep's Understanding Math-10
Sol. Join OB PE + EA + CB + PC
OBA = 90º (Radius through the point of [... ED = EA and CB = CD]
contact is perpendicular to the tangent) = AP + PB = 15 + 15
= 30 cm
B
15cm
9. (a) If a, b, c are the sides of a right triangle
r where c is the hypotenuse, prove that the
radius r of the circle which touches the
Or C 7.5cm A
sides of the triangle is given by r =
abc
.
2
 OB2 = OA2 – AB2 (b) In the given figure, PB is a tangent to a
 r2 = (r + 7.5)2 – 152 circle with centre O at B. AB is a chord of
length 24 cm at a distance of 5 cm from
 r2 = r2 + 56.25 + 15r – 225
the centre. If the length of the tangent is 20
 15r = 168.75 cm, find the length of OP.
 r = 11.25
Hence, radius of the circle = 11.25 cm P

(ii) In the figure, PA and PB are the tangents


drawn from P to the circle. O

A M B
A E P

C
Sol. (a) Let the circle touch the sides BC, CA
B
and AB of the right triangle ABC at the points
D, E and F respectively, where BC = a, CA
= b and AB = c (as showing in the given
figure).
CE is a tangent at D.
AP = 15 cm A
.. .
PA and PB are the tangents to the circle
 AP = BP = 15 cm
Similarly EA and ED are tangents c

 EA = ED b
F
Similarly, BC = CD
O
Now perimeter of  PEC. E

= PE + EC + PC
= PE + ED + CD + PC B a D C
421 Arun Deep's Understanding Math-10
As the lengths of tangents drawn from an  OP2 = 400 + 169
external point to a circle are equal,
 OP = 569 cm
AE = AF, BD = BF and CD = DE
10. Three circles of radii 2 cm, 3 cm and 4 cm
OD  BC and OE  CA
touch each other externally. Find the
( tangents is  to radius) perimeter of the triangle obtained on joining
 ODCE is a square of side r the centres of these circles.
 DC = CE = r Sol. Three circles with centres A, B and C touch
 AF = AE = AC – EC = b – r and each other externally at P, Q and R
BF = BD = BC – DC = a – r respectively and the radii of these circles
Now, AB = AF + BF are 2 cm, 3 cm and 4 cm.

 c = (b – r) + (a – r)
 2r = a + b – c

abc
 r= .
2
(b) Construction : Join OB.
OC = 5 cm (given)
and AB = 24 cm

24
So, BC = cm = 12 cm
2
( The perendicular from the centre to the By joining their centres ABC is formed in
chord bisects the chord) which.

In BOC, by Pythagoras Theorem  AB = 2 + 3 = 5 cm


OB2 = OC2 + BC2 BC = 3 + 4 = 7 cm
 OB2 = 52 + 122 and CA = 4 + 2 = 6 cm.
 OB2 = 25 + 144  Perimeter of the ABC
 OB2 = 169 = AB + BC + CA
 OB = 13 cm = 5 cm + 7 cm + 6 cm = 18 cm Ans.
Now, in POB 11. (a) In the figure (i) given below, the sides
PB = 20 cm, OB = 20 cm and PBO = 90° of the quadrilateral touch the circle. Prove
that AB + CD = BC + DA.
( Tangent to a circle at a point is
perpendicular to the radius through the point (b) In the figure (i) given below, ABC is triangle
of contact) with AB = 10cm, BC = 8cm and AC = 6cm
(not drawn to scale). Three circles are
So, by Pythagoras Theorem
drawn touching each other with vertices
OP2 = BP2 + OB2 A, B and C as their centres. Find the radii
 OP2 = 202 + 132 of the three circles.
422 Arun Deep's Understanding Math-10
BC = 8 cm (given)
BR + RC = 8 cm
 BP + RC = 8 ....(ii)
Again, AC = 6 cm
AQ + QC = 6 cm
PA + RC = 6 cm ....(iii)
(i) + (ii) + (iii) we have
Sol. (a) Given : Sides of quadrilateral ABCD
2 (BP + PA + RC) = 24 cm
touch the circle at P, Q, R and S
respectively.  BP + PA + RC = 12 cm ....(iv)
(iv) – (i) we have
RC = 12 – 10 = 2 cm
(iv) – (ii) we have
PA = 12 – 8 = 4 cm
(iv) – (iii) we have
To Prove : AB + CD = BC + DA BP = 12 – 6 = 6 cm
Proof : Tangents from A, AP and AS are to Radii : BP = 6 cm, PA = 4 cm, RC = 2 cm
the circle. 12. (a) In the figure (i) PQ = 24 cm, QR = 7cm
 AP = AS and PQR = 90°. Find the radius of the
inscribed circle PQR.
Similarly PB = BQ
CR = CQ and DR = DS. P
Adding them, we get
AP + PB + CR + DR
= AS + BQ + CQ + DS
 AP + PB + CR + DR
O x cm
= BQ + CQ + AS + SD
 AB + CD = BC + DA. Q.E.D.
Q R
(b) AB = 10 cm, BC = 8 cm, AC = 6 cm
BP + PA = 10 cm ....(i) (b) In the figure (ii) given below, two concentric
circles with centre O are of radii 5 cm and
3 cm. From an external point P, tangents PA
C and PB are drawn to these circles.
If AP = 12 cm, find BP.
R Q

B P A
423 Arun Deep's Understanding Math-10
Sol. (a) In the figure, a circle is inscribed in the
triangle PQR which touches the sides. O is
centre of the circle.
PQ = 24 cm, QR = 7 cm PQR = 90°
OM is joined.
Join OL

P
and radius of inner circle = 3 cm
 OA = 5 cm
OB = 3 cm
and AP = 12 cm
. .. OA is radius and AP is the tangent.
O
L
x cm
 OA  AP
Q M 7cm R  In right OAP
OP2 = OA2 + AP2 = (5)2 + (12)2
OL and OM are the radii of the circle
= 25 + 144 = 169 = (13)2
 OL  PQ and OM  BC
 OP = 13
and PQR = 90°
Similarly in right OBP
 QLOM is a square
OP2 = OB2 + BP2
 OL = OM = QL = QM
 (13)2 = (3)2 + BP2
Now in PQR O = 90°
 169 = 9 + BP2
 PQ2 = PQ2 + QR2 (Pythagoras theorem)
 BP2 = 169 – 9 = 160
= (24)2 + (7)2 = 576 + 49 = 625 = (25)2
 BP  160  16  10
 PR = 25 cm
Let = OB = x, then  4 10 cm Ans.
QM = QL = x 13. (a) In the figure (i) given below, AB = 8
cm and M is mid-point of AB. Semi-circles
PL and PN are the tangents to the circle.
are drawn on AB, AM and MB as diameters.
PL = PN A circle with centre C touches all three semi-
 PQ – LQ = PR – RN circles as shown, find its radius.
 24 – x = 25 – PM ( RN = RM)
 24 – x = 25 – (QR – QM)
 24 – x = 25 – (7 – x)  24 – x = 25 – 7 +x

6
 24 – 25 + 7 = 2x  2x = 6  x = = 3cm
2
(b) In the figure (ii) given below, equal
(b) Radius of outer circle = 5 cm circles with centres O and O touch each
424 Arun Deep's Understanding Math-10
other at X. OO is produced to meet a circle
O at A. AC is tangent to the circle whose
centre is O. OD is perpendicular to AC.
Find the value of :
AO  area of ADO 
(i) (ii) .
AO area of ACO
In ADO and ACO.
A = A (Common)
D = C (each 90º)
 ADO ~ ACO
(AA axiom of similarity)
AO  r 1
(i)   
Sol. (a) Let x be the radius of the circle with AO 3r 3
centre C and radii of each equal (ii) ... ADO ~ ACO
area of ADO  ( AO  ) 2
 
area of ACO ( AO ) 2
2 2

FG AO IJ  FG 1 IJ (from (i))
H AO K H 3K
1
 Ans.
4 9
semi circles   2 cm
2 14. The length of the direct common tangent
CP = x + 2 and CM = 4 – x to two circles of radii 12 cm and 4 cm is
15 cm. Calculate the distance between their
 In right PCM centres.
(PC)2 = (PM)2 + (CM)2 Sol. Let R and r be the radii of the circles with
 (x + 2)2 = (2)2 + (4 – x)2 centre A and B respectively and Let TT be
 x2 + 4 x + 4 = 4 + 16 – 8 x + x2 their common tangent.
 4 x + 8 x + 4 – 4 – 16 = 0  (TT)2 = (AB)2 – (R – r)2
16 4 1  (15)2 = (AB)2 – (12 – 4)2
 12 x = 16  x    1 cm
12 3 3  225 = AB2 – (8)2
1  (AB)2 = 225 + 64 = 289 = (17)2
 Radius  1 cm Ans.
3  AB = 17 cm.
(b) Two equal circles with centre O and O
touch each other externally at X. OO is
joined and produced to meet the circle at
A. AC is the tangent to the circle with centre
O. OD is  AC. Join OC.
... OC is radius and AC is tangent, then

OC  AC.
Let radius of each equal circle = r. Hence distance between their centres
= 17 cm Ans.
425 Arun Deep's Understanding Math-10
15. Calculate the length of a direct common  PB = 10 cm.
tangent to two circles of radii 3 cm and Now in PAB and BCQ,
8 cm with their centres 13 cm apart.
 A = C (each 90º)
Sol. Let A and B be the centres of the circles
ABP = CBQ
whose radii are 8 cm and 3 cm and let TT
length of their common tangent and AB = (vertically opposite angles)
13 cm.  PAB ~ BCQ
(AA axiom of similarity)
AP PB 6 10
   
CQ BQ 3 BQ
10  3
 BQ   5 cm
6
 PQ = PB + BQ = 10 + 5 = 15 cm Ans.
Now (TT)2 = (AB)2 – (R – r)2 17. Two circles with centres A, B are of radii 6
= (13)2 – (8 – 3)2 = 169 – (5)2 cm and 3 cm respectively. If AB = 15 cm,
= 169 – 25 = 144 = (12)2 find the length of a transverse common
tangent to these circles.
 TT = 12 cm.
Sol. AB = 15 cm.
Hence length of common tangent
Radius of the circle with centre A = 6 cm
= 12 cm Ans.
and radius of second circle with radius
16. In the given figure, AC is a transverse
B = 3 cm.
common tangent to two circles with
centres P and Q and of radii 6 cm and 3 cm
respectively. Given that AB = 8 cm,
calculate PQ.

Let AP = x, then PB = 15 – x
In ATP and SBP
T = S (each 90º)
Sol. AC is a transverse common tangent to the APT = BPS
two circles with centre P and Q and of radii
(vertically opposite angles)
6 cm and 3 cm respectively AB = 8 cm.
Join AP and CQ.  ATP ~ SBP
(AA axiom of similarity)
AT AP 6 x
  
BS PB 3 15  x
x 6
   3 x = 90 – 6 x
15  x 3
In right PAB,  3 x + 6 x = 90  9 x = 90
PB2 = PA2 + AB2 = (6)2 + (8)2 90
= 36 + 64 = 100 = (10)2  x  10
9
426 Arun Deep's Understanding Math-10
 AP = 10 cm, PB = 15 – 10 = 5 cm
Now in right ATP,
AP2 = AT2 + TP2
 (10)2 = (6)2 + TP2  100 = 36 + TP2 TP2 = 100 – 36 = 64  TP2 = (8)2
 TP = 8 cm.
Similarly in right PSB
PB2 = BS2 + PS2
 (5)2 = (3)2 + PS2  25 = 9 + PS2  PS2 = 25 – 9 = 16 = (4)2
 PS = 4 cm.
Hence TS = TP + PS = 8 + 4 = 12 cm.
18. (a) In the figure (i) given below, PA and PB are tangents at a points A and B respectively of a
circle with centre O. Q and R are points on the circle. If APB = 70°, find
(i) AOB (ii) AQB (iii) ARB
(b) In the figure (ii) given below, two circles touch internally at P from an external point Q on the
common tangent at P, two tangents QA and QB are drawn to the two circles. Prove that QA =
QB.

R
P 70° O Q

Sol. (a) To find : (i) AOB, (ii) AQB, (iii) ARB


Given : PA and PB are tangents at the points A and B respectively of a circle with centre O and
OA and OB are radii on it.
APB = 70°
Construction : Join AB
(i) Since, AP and BP are tangents to the circle and OA and OB are radii on it.
 OA  OP and OB  BP
 AQB = 180° – 70° = 110°
(ii) Arc AB subtends AOB at the centre and AOB at the remaining part of the circle.
 AQB = 2AOB
1 1
 AQB = AOB  AQB = × 110° = 55°
2 2
(iii) Now in OAB
OA = OB (Radii of the same circle)
 OAB = OBA
427 Arun Deep's Understanding Math-10

1 1 OAB + 90° + ADC = 180°


= (180  110)   70  35 OAB + 90° + 50° = 180°
2 2
OAB + 140° = 180°
We know, AOB = 110°
OAB + 180° – 140° = 40°
Reflex AOB = 360° – 110° = 250°
( Angle at centre of a circle = double the
1 angle at the remaining part of circle)
 ARB = of reflex AOB
2  ABC = 2 × 40° = 80°
( Angle at the centre of a circle = double 20. In the given figure, tangents PQ and PR
the angle at the remaining part of the circle) are drawn from an external point P to a
1 circle such that RPQ = 30°. A chord RS
= × 250° = 120° is drawn parallel to the tangent PQ. Find
2
RQS.
(b) In the fig. (ii)
Two circles touch each other internally at S R
P. From a point Q, outside, common
tangents QP, QA and QB are drawn to the
two circles. O

To prove : QA = QB.
Proof : From Q, QA and QP are the 30°
P
Q
tangents to the outer circle.
Sol. In the given figure,
 QP = QA ...(i)
PQ and PR are tangents to the circle with
Similarly, from Q, QB and QP are the centre O drawn from P
tangents to the inner circle. RPQ = 30°
 QP = QB ...(ii) Chord RS || PQ is drawn
From (i) and (ii) To find RQS
QA = QB PQ = PR (tangents to the circle)
Hence proved.  PRQ = PQR
19. In the given figure, AD is a diameter of a But RPQ = 30°
circle with centre O and AB is tangent at A. 180  30 150
C is a point on the circle such that DC  PRQ = PQR = = = 75°
2 2
produced intersects the tangent at B. If
SR || QP
ABC = 50°, find AOC.
 SRQ = PQR = 75°
D and RSQ = QRS
( QR = QS)
= 75°
O C Now in QRS,
RQS = 180° – (RSQ + QRS)
50° = 180° – (75° + 75°) = 180° – 150° = 30°
B
A
21. (a) In the figure (i) given below, PQ is a
Sol. Given AB is tangent to the circle at A and tangent to the circle A, DB is a diameter,
OA is radius, OA  AB ADB = 30° and CBD = 60°, calculate
In ABD (i) QAB (ii) PAD (iii) CDB.
428 Arun Deep's Understanding Math-10
(iii) In right BCD
BCD = 90º (Angle is a semi-circle)
CBD = 60º (given)
 CDB = 180º – (90º + 60º)
= 180º – 150º = 30º Ans.
(b) ABCD is a cyclic quadrilateral

(b) In the figure (ii) given below, ABCD is


a cyclic quadrilateral. The tangent to the
circle at B meets DC produced at F. If
EAB = 85º and BFC = 50º, find CAB.

 Ext. EAB = BCD


 BCD = 85º
But BCD + BCF = 180º
(Linear pair)
Sol. (a) PQ is tangent and AD is chord  85º + BCF = 180º
(i)  QAB = BDA = 30º  BCF = 180º – 85 = 95º
(Angles in the alternate segment) Now in BCF,
(ii) In ADB, BCF + BFC + CBF = 180º
DAB = 90º (Angle in a semi-circle)  95º + 50º + CBF = 180º
 145º + CBF = 180º
 CBF = 180º – 145º = 35º
. .. BF is a tangent and BC is the chord.
 CAB = CBF
(Angles in the alternate segment)
 CAB = 35º Ans.
22. (a) In the figure (i) given below, O is the
and ADB = 30º centre of the circle and SP is a tangent. If
 ABD = 180º – (90º + 30º) SRT = 65°, find the value of x, y and z.
= 180º – 120º = 60º (2015)
But PAD = ABD (b) In the figure (ii) given below, O is the
centre of the circle. PS and PT are tangents
(Angles in the alternate segment) and SPT = 84º. Calculate the sizes of the
= 60º (Proved) angles TOS and TQS.
429 Arun Deep's Understanding Math-10
T meet at C. If ACO = 30º, find
x (i) BCO (ii)AOB (iii) APB
Q
O y z P
65° R
A
S
30°
O C
Sol. Consider the following figure :
TS  SP, P
B
TSR = OSP = 90°
In TSR,
TSR + TRS + RTS = 180° Sol - (i) BCO = ACO = 30º
 90° + 65° + x = 180° ( C is the intersecting point of tangent AC
 x = 180° – 90° – 65°  x = 25° y = 2x and BC)
[Angle subtended at the centre is double (ii) OAC = OBC = 90º
that of the angle subtended by the arc at  AOC = BOC = 180º – (90º + 30º)
the same centre]
( sum of the three angles of a  is 180º)
 y = 2 × 25°  y = 50º
= 60º
In OSP,
 AOB = AOC + BOC
OSP + SPO + POS = 180°
= 60º + 60º = 120º
 90º + z + 50° = 180º  z = 180º – 140º
 z = 40º 1 120º
(iii) APB = AOB = = 60º
Hence x = 25°, y = 50° and z = 40° 2 2
. .
(b) . TP and SP are the [ angle subtended at the remaining part of
tangents and OS and the circle is half the subtended at the centre]
OT are the radii 24. (a) In the figure (i) given below, O is the
centre of the circle. The tangent at B and D
 OS  SP and OT  TP
meet at P. If AB is parallel to CD and ABC
 OSP = OTP = 90º = 55º, find : (i) BOD (ii) BPD.
SPT = 84º (b)In the figure (ii) given below, O is the centre
But SOT + OTP + TPS + OSP = 360º of the circle. AB is a diameter, TPT is a
 SOT + 90º + 84º + 90º = 360º tangent to the circle at P. If BPT = 30º,
calculate :
 SOT + 264º = 360º SOT = 360º – 264º
(i) APT (ii) BOP.
 SOT= 96º
and reflex SOT = 360º – 96º = 264º
Now major arc ST subtends reflex SOT at the
centre andTQS at the remaining part of the circle.
1 1
 TQS = reflex SOT   264º  132º .
2 2
Hence TOS = 96º and TQS = 132º Ans.
23. In the given figure, O is the centre of the
circle. Tangents to the circle at A and B
430 Arun Deep's Understanding Math-10
. ..
Sol. (a) AB || CD (ii) TPT is tangent and PB is the chord.
(i)  ABC = BCD (Alternate angles)  BAP = BPT
 BCD = 55º (Angles in the alternate segment)
= 30º
Now arc BP subtends BOP at the centre
and BAP at the remaining part of the circle.
 BOP = 2 BAP = 2 × 30º = 60º Ans.
25. In the adjoining figure, ABCD is a cyclic
quadrilateral.

Now arc BD subtends BOD at the centre


and BCD at the remaining part of the
circle.
 BOD = 2 BCD = 2 × 55º = 110º
OB is radius and BP is tangent
 OB  BP
Similarly, OD  DP The line PQ is the tangent to the circle
Now in quad. OBPD, at A. If CAQ : CAP = 1 : 2, AB
bisects CAQ and AD bisects CAP,
BOD + ODP + OBP + BPD = 360º
then find the measure of the angles of
(Angles of quadrilateral) the cyclic quadrilateral. Also prove that
 110º + 90º + 90º + BPD = 360º BD is the diameter to the circle.
 290º + BPD = 360º Sol. ABCD is a cyclic quadrilateral. PAQ is
 BPD = 360º – 290º = 70º the tangent to the circle at A. CAD :
(b) TPT is the tangent. CAP = 1 : 2. AB and AD are the
b isectors o f CAQ and CAP
(i) AB is diameter respectively
 APB = 90º To prove :
(Angle in a semi-circle) (i) BD is the diameter of the circle
and BPT = 30º (ii) Find the measure of the angles of the
 APT = 180º – (90º + 30º) cyclic quadrilateral.
= 180º – 120º = 60º Proof. (i) ... AB and AD are the bisectors of
CAQ and CAP respectively and
CAQ + CAB = 180° (linear pair)
CAB = CAD = 90°
BAD = 90°
Hence, BD is at diameter of the circle.
(ii) CAQ : CAP = 1 : 2
Let CAQ = x and CAP = 2x
431 Arun Deep's Understanding Math-10
 x + 2x = 180° 1
  RPQ = × 120º  60º Ans.
180 2
 3x = 180°  x   60
3
... CAQ = 60° and CAP = 120° 27. (a) In the figure (i) given below, AB is a
diameter. The tangent at C meets AB
PAQ is the tangent and AC is the chord produced at Q, CAB = 34º. Find
of the circle
(i) CBA (ii) CQA (2006)
ADC = CAQ = 60°
Similarly ABC = CAP = 120°
Hence A = 90°, B = 120°,
C = 90° and D = 60° Ans.
26. In a triangle ABC, the incircle (centre O)
touches BC, CA and AB at P, Q and R
respectively. Calculate (i) QOR (ii) QPR
given that A = 60º.
Sol. ... OQ and OR are the radii and AC and AB (b) In the figure (ii) given below, AP and BP
are tangents. are tangents to the circle with centre O.
Given APB = 60º, calculate.
 OQ  AC and OR  AB
(i) AOB (ii) OAB
Now in quad. AROQ
(iii) ACB.
A = 60º, ORA = 90º and OQA = 90º
 ROQ = 360º – (A + ORA
+ OQA)
= 360º – (60º + 90º + 90º)
= 360º – 240º = 120º

Sol. (a) AB is the diameter.

Now arc RQ subtends ROQ at the centre


and RPQ at the remaining part of the
circle.  ACB = 90º and CAB = 30º
 ROQ = 2 RPQ  In ABC,
ACB + CAB + CBA = 180º
1
 RPQ = ROQ  90º + 30º + CBA = 180º
2
432 Arun Deep's Understanding Math-10
(Angles of a triangle) centre of the circumcircle of triangle XYZ.
 CBA = 180º – 124º = 56º Tangents at X and Y intersect at T. Given
XTY = 80º and XOZ = 140º, calculate
and Ext. CBQ = 180º – 56º = 124º the value of ZXY. (1994)
. .. CQ is tangent and CB is chord. Sol. (a) Join OY, OX and OY are the radius of
 BCQ = CAB = 34º the circle and XT and YT are the tangents
to the circle.
(Angles in the alternate segments)
Now in BCQ,
BCQ + CBQ + CQB = 180º
(Angles of a triangle)
 34º + 124º + CQB = 180º
 158º + CQB = 180º
 CQB = 180º – 158º = 22º.
Hence CQA = 22°  OX  XT and OY  YT
(b) (i) AP and BP are the tangents to the circle and XTY = 80º
and OA and OB are radii on it.  XOY = 180º – XTY
 OA  AP and OB  BP = 180º – 80º = 100º
Now ZOY = 360º – (XOZ + XOY)
 AOB = 180º – 60º = 120º
= 360º – (140º + 100º)
(ii) Now in OAB,
= 360º – 240º = 120º
But arc ZY subtends ZOY at the centre
and ZXY at the remaining part of the
circle.
1
 ZXY = ZOY
2
1
= ×120º  60º Ans.
2
OA = OB (radii of the same circle) (b) In the figure (ii) given below, O is the centre
 OAB = OBA of the circle and PT is the tangent to the
circle at P. Given QPT = 30º, calculate
1 1

(180º 120º )  × 60º  30º . (i) PRQ(ii) POQ.
2 2
(iii) Arc AB subtends AOB at the centre and
ACB at the remaining part of the circle.
 AOB = 2 ACB
1
 ACB = AOB
2
1
 ACB = × 120º  60º [From (i)]
2
28. (a) In the figure (i) given below, O is the
433 Arun Deep's Understanding Math-10
Sol. (b) (i) PT is tangent and OP is radius,
then OP  PT.

 AP.PB = CP.PD
(i) When AP = 6 cm, PB = 4 cm, PD = 3 cm,
 OPT = 90º then
But QPT = 30º  6 × 4 = CP × 3
 OPQ = 90º – 30º = 60º 
6×4
CP =  8 cm .
. .. OP = OQ 3
(ii) When AB = 12 cm, AP = 2 cm, PC = 5 cm
(radii of the same circle)
 OQP = OPQ = 60º
Hence in OPQ, POQ = 60º
(ii) Take a point A on the circumference and
join AP and AQ.
Now arc PQ subtends POQ at the centre
and PAQ on the circumference of the
circle.
PB = AB – AP = 12 – 2 = 10 cm
1 1
 PAQ = POQ = × 60º  30º . .. AP × PB = CP × PD
2 2
 2 × 10 = 5 × PD
Now APRQ is a cyclic quadrilateral
 PAQ + PRQ = 180º 2 ×10
 PD =  4 cm .
5
 30º + PRQ = 180º
(iii) When AP = 5 cm, PB = 6 cm, CD = 13 cm.
 PRQ = 180º – 30º = 150º Ans.
Let CP = x, then PD = CD – CP
29. Two chords AB, CD of a circle intersect
internally at a point P. If or PD = 13 – x
(i) AP = 6 cm, PB = 4 cm and PD = 3 cm,
find PC.
(ii) AB = 12 cm, AP = 2 cm, PC = 5 cm, find
PD.
(iii) AP = 5 cm, PB = 6 cm and CD = 13 cm,
find CP.
Sol. In a circle, two chords AB and CD intersect
each other at P internally.
434 Arun Deep's Understanding Math-10
 AP × PB = CP × PD 2
PT = (5 + 7.8) × 5 = 12.8 × 5
 5 × 6 = x (13 – x)  30 = 13 x – x2 PT2 = 64  PT = 8 cm
 x2 – 13 x + 30 = 0  x2 – 10 x – 3 x + 30 = 0 In OTP
 x (x – 10) – 3 (x – 10) = 0  (x – 10) (x – 3) = 0 PT2 + OT2 = OP2
Either x – 10 = 0, then x = 10 82 + x2 = (x + 4)2  64 + x2 = x2 + 16 + 8x
or x – 3 = 0, then x = 3
48
 CP = 10 cm or 3 cm Ans. 64 – 16 = 8x  48 = 8  x = = 6 cm
8
30. (a) In the figure (i) given below, PT is a
T
tangent to the circle. Find TP if AT = 16
cm and AB = 12 cm.
B 4cm
A P
O
5cm
D
7.8cm
C

 Radius = 6 cm
AB = 2 × 6 = 12 cm
(b) In the figure given below, diameter AB and
31. PAB is a secant and PT is tangent to a circle.
chord CD of a circle meet at P. PT is a If
tangent to the circle at T. CD = 7.8 cm, PD
(i) PT = 8 cm and PA = 5 cm, find the length
= 5 cm, PB = 4 cm. Find:
of AB.
(i) AB. (ii)the length of tangent PT.
(ii) PA = 4·5 cm and AB = 13·5 cm, find the
length of PT.
T
Sol. ... PT is the tangent and PAB is the secant
of the circle.
B P
A

D
C

Sol. (a) PT is the tangent to the circle and AT is


a secant.
 PT2 = TA × TB
Now TA = 16 cm, AB = 12 cm  PT2 = PA.PB
 TB = AT – AB = 16 – 12 = 4 cm (i) PT = 8 cm, PA = 5 cm
PT2 = 16 + 4 = 64 = (8)2  (8)2 = 5 × PB  64 = 5 PB
 PT = 8 cm or TP = 8 cm Ans. 64
 PB =  12 .8 cm
(b) PT is tangent and PDC is secant out to the 5
circle  AB = PB – PA = 12·8 – 5·0
 PT2 = PC × PD = 7·8 cm Ans.
435 Arun Deep's Understanding Math-10
(ii) PT2 = PA × PB 33. (a) In the figure (i) given below, PAB is a
secant and PT is tangent to a circle.
If PA : AB = 1 : 3 and PT = 6 cm, find
the length of PB.
(b) In the figure (ii) given below, ABC is an
isosceles triangle in which AB = AC and
Q is mid-point of AC. If APB is a secant
and AC is tangent to the circle at Q,
prove that AB = 4 AP.

But PA = 4·5 cm, AB = 13·5 cm


 PB = PA + AB = 4·5 + 13·5 = 18 cm
Now PT2 = 4·5 × 18 = 81 = (9)2
 PT = 9 cm.
32. In the adjoining figure, CBA is a secant and
CD is tangent to the circle. If AB = 7 cm
and BC = 9 cm, then
(i) Prove that ACD ~ DCB.
(ii) Find the length of CD (2009)

A B C

D Sol. (a) In the figure (i),


PAB is a secant and PT is the tangent to
the circle.
Sol. In DACD and DDCB  PT2 = PA × PB
C = C (common) But PT = 6 cm, and PA : AB = 1 : 3
CAD = CDB Let PA = x, then AB = 3 x
[Angle between chord and tangent is equal  PB = PA + AB = x + 3 x = 4 x
to angle made by chord in alternate segment.] Now PT2 = PA × PB
 ACD ~ DCB  (6)2 = x (4 x)
 36 = 4 x2
AC DC
 
DC BC 36
 x2   9  ( 3) 2
4
 DC 2 = AC × BC
 x=3
= 16 × 9 = 144
 PB = 4 x = 4 × 3 = 12 cm Ans.
 DC = 12 cm
436 Arun Deep's Understanding Math-10
(b) In the figure (ii) Subtracting (ii) from (i)
ABC is an isosceles triangles in which PA – PB = PC – PD
AB = AC.  AB = CD Q.E.D.
Q is mid-point of AC. 35. (a) In the figure (i) given below, AT is
APB is the secant and AC is the tangent to tangent to a circle at A. If BAT = 45º and
the circle at Q. BAC = 65º, find ABC.
To prove : AB = 4 AP
Proof : AQ is the tangent and APB is the
secant.
 AQ2 = AP × AB
2
1 
 AC   AP  AB
 2 
2
1 
  AB   AP  AB (b) In the figure (ii) given below, A, B and C
 2 
are three points on a circle. The tangent at
( AB = AC) C meets BA produced at T. Given that
AB2 ATC = 36º and ACT = 48º, calculate
 = AP × AB  AB2 = 4 AP × AB the angle subtended by AB at the centre of
4
the circle. (2001)
AB2
 = 4 AP
AB
 AB = 4 AP Hence proved.
34. Two chords AB, CD of a circle intersect
externally at a point P. If PA = PC, Prove
that AB = CD.
Sol. Given : Two chords AB and CD intersect
each other at P outside the circle. PA = PC.

Sol. (a) AT is the tangent to the circle at A and


AB is the chord of the circle.

To Prove : AB = CD.
Proof : Chords AB and CD intersect each
other at P outside the circle.
 PA × PB = PC × PD
But PA = PC (given) ...(i)
 PB = PD ...(ii)
437 Arun Deep's Understanding Math-10
 ACB = BAT Sol. Given : ABC is an isosceles triangle with
(Angles in the alternate segment) AB = AC. AT is the tangent to the
circumcircle at A.
= 45º
Now in ABC,
ABC + BAC + ACB = 180º
(Angles of a triangle)
 ABC + 65º + 45º = 180º
 ABC + 110º = 180º
 ABC = 180º – 110º = 70º.
(b) Join OA, OB and CB. In ATC,
Ext. CAB = ATC + TCA To Prove : AT | | BC.
= 36º + 48º = 84º Proof : In ABC,
But TCA = ABC AB = AC (given)
(Angles in the alternate segment)
 C = B
 ABC = 48º
(Angles opposite to equal sides)
But AT is the tangent and AC is the chord.
 TAC = B
(Angles in the alternate segment)
But B = C (proved)
 TAC = C
But in ABC, But these are alternate angles
ABC + BAC + ACB = 180º
 AT | | BC Q.E.D.
(Angles of a triangle)
37. If the sides of a rectangle touch a circle,
 48º + 84º + ACB = 180º
prove that the rectangle is a square.
 132º + ACB = 180º
Sol. Given : A circle touches the sides AB, BC,
 ACB = 180º – 132º
CD and DA of a rectangle ABCD at P, Q, R
ACB = 48º and S respectively.
... Arc AB subtends AOB at the centre

and ACB at the remaining part of the


circle.
 AOB = 2 ACB = 2 × 48º = 96º Ans.
36. In the adjoining
figure ABC is isosceles
with AB = AC. Prove that
the tangent at A to the
circumcircle of ABC is
parallel to BC. To Prove : ABCD is a square.
438 Arun Deep's Understanding Math-10
Proof : Tangents from a point to the circle Construction : Join AB.
are equal. Proof : PT is tangent and PA is chord.
 AP = AS
 APT = ABP
Similarly BP = BQ
(Angles in the alternate segments) ...(i)
CR = CQ and DR = DS
But BDCA is a cyclic quadrilateral
Adding, we get
 Ext. ABP = ACD ...(ii)
AP + BP + CR + DR = AS + BQ + CQ
From (i) and (ii)
+ DS
APT = ACD
 AP + BP + CR + DR = AS + DS + BQ
But these are alternate angles
+ CQ
 AB + CD = AD + BC  CD | | PT Q.E.D.
But AB = CD and AD = BC (b) In the figure (ii) given below, two circles
(opposite sides of a rectangle) with centres C, C intersect at A, B and the
point C lies on the circle with centre C.
 AB + AB = BC + BC
PQ is a tangent to the circle with centre C
 2 AB = 2 BC  AB = BC at A. Prove that AC bisects PAB.
 AB = BC = CD = DA
Hence ABCD is a square. Q.E.D.
38. (a) In the figure (i) given below, two circles
intersect at A, B. From a point P on one of
these circles, two line segments PAC and
PBD are drawn, intersecting the other circle
at C and D respectively. Prove that CD is
parallel to the tangent at P.

Sol. Given : Two circles with centres C and C


intersect each other at A and B. PQ is a
tangent to the circle with centre C at A.

Sol. Given : Two circles intersect each other


at A and B. From a point P on one circle
PAC and PBD are drawn. From P, PT is a
tangent drawn. CD is joined.

To Prove : AC bisects PAB.


Construction : Join AB and CP.
Proof : In ACB,
AC = BC (radii of the same circle)
To Prove : PT | | CD.
 BAC = ABC ...(i)
439 Arun Deep's Understanding Math-10
PAQ is tangent and AC is the chord of the 1
circle  × 116º  58º Ans.
2
 PAC = ABC ...(ii) (b) In the figure (ii) given below, O and O are
From (i) and (ii) centres of two circles touching each other
externally at the point P. The common
BAC = PAC tangent at P meets a direct common tangent
Hence AC is the bisector of PAB AB at M. Prove that :
Q.E.D. (i) M bisects AB (ii) APB = 90º.
39. (a) In the figure (i) given below, AB is a
chord of the circle with centre O, BT is
tangent to the circle. If OAB = 32º, find
the values of x and y.

Sol. Given : Two circles with centre O and O


touch each other at P externally. From P, a
common tangent is drawn and meets the
common direct tangent AB at M.
Sol. AB is a chord of circle with centre O. BT
is a tangent to the circle and OAB = 32º.
. .. In OAB,
OA = OB (radii of the same circle)

To Prove : (i) M bisects AB i.e. AM = MB.


(ii) APB = 90º.
Proof : (i) From M, MA and MP are the
tangents.
 OAB = OBA  MA = MP ...(i)
 x = 32º Similarly MB = MP ...(ii)
and AOB = 180º – (x + 32º) From (i) and (ii)
= 180º – 64º = 116º MA = MB
Now arc AB subtends AOB at the centre or M is the mid-point of AB.
and ACB at the remaining part of the (ii) . .. MA = MP
circle.  MAP = MPA ...(i)
 AOB = 2 ACB Similarly MP = MB
1  MPB = MBP ...(ii)
 ACB = AOB
2 Adding (i) and (ii),
440 Arun Deep's Understanding Math-10
Ext. TCQ = TAC ....(ii) In OAB,
 From (i) and (ii) OAB = 40°
But OBA = OAB = 40°
TCQ = ATP ...(iii)
( OA = OB radii of the same circle)
(ii) ATP = ACT ...(iv)
 AOB = 180° – (40° + 40°)
(Angles in the alternate segments)
= 180° – 80° = 100°
From (iii) and (iv)
But arc AB subtends AOB at the centre
 TCQ = ACT
and ACB at the remaining part of the
Hence CT is the bisector of ACQ. circle
(iii) Arc AT subtends AOT at the centre and ACT
at the remaining part of the circle. 1 1
 ACB = AOB = × 100° = 50°
2 2
 AOT = 2 ACT
(a)
= ACT + ACT
2. ABCD is a cyclic quadrilateral such that
= ACT + TCQ
AB is a diameter of the circle
(... TCQ = ACT) circumscribing it and ADC = 140°,
= ACQ Q.E.D. then BAC is equal to
MAP + MPB = MPA + MBP (a) 80° (b) 50°
 APB = MAP + MBP (c) 40° (d) 30°
But APB + MAP + MBP = 180º Sol. ABCD is a cyclic quadrilateral, AB is the
(Angles of a triangle) diameter of the circle circumscribing it
 APB + APB = 180º ADC = 140°, BAC =
 2 APB = 180º Join AC
180º
 APB =  90º Q.E.D. D C
2
140°
MULTIPLE CHOICE QUESTIONS
A B
1. In the given figure, O is the centre of the circle.
If OAB = 40°, then ACB is equal to
(a) 50° (b) 40°
(c) 60° (d) 70°
C
 ADC + ABC = 180°
(opposite angles of the cyclic
O
quadrilateral)
A
40° 140° + ABC = 180°
B  ABC = 180° – 140° = 40°
Now in ABC,
Sol. In the given figure, O is the centre of the circle.
441 Arun Deep's Understanding Math-10
ACB = 90° (angle in a semi-circle)  A = B
 BAC = 90° – ABC But A + B = 90°
= 90° – 40° = 50° (b) 90
 A = B = = 45°
P.Q. In the given figure, O is the centre of the 2
circle. If ABC = 20°, then AOC is equal  CAB = 45° (d)
to 3. In the given figure, O is the centre of the
(a) 20° (b) 40° circle. If BAO = 60°, then ADC is equal
(c) 60° (d) 10° to
(a) 30° (b) 45°
(c) 60° (d) 120°
O
A
B
20°
A
C
B C
O

Sol. In the given figure,


Arc AC subtends AOC at the centre and D
ABC at the remaining part of the circle.
Sol. In the given figure, O is the centre of the
 AOC = 2ABC circle BAO = 60°
 2 × 20° = 40° (b)
A
P.Q. In the given figure, AB is a diameter of the
circle. If AC = BC, then CAB is equal to 60°
(a) 30° (b) 60°
B C
O
(c) 90° (d) 45°

C
D

In ABO, OA = OB
A B
(radii of the same circle)
 ABO = BAO = 60°
Ext. AOC = BAO + ABO
Sol. In the given figure, AB is the diameter of the = 60° + 60° = 120°
circle and AC = BC Now arc AC subtends AOC at the centre
 ACB = 90° (angle in a semi-circle) and ADC at the remaining part of the circle
 AC = BC  AOC = 2ADC  2ADC = 120°
442 Arun Deep's Understanding Math-10

120  CAO = 105° – 45° = 60° (d)


 ADC = = 60° (c) 4. In the given figure, O is the centre of a circle.
2
If the length of chord PQ is equal to the radius
P.Q. In the given figure, O is the centre of the
of the circle, then PRQ is
circle. If AOB = 90° and ABC = 30°,
(a) 60° (b) 45°
then CAO is equal to
(a) 30° (b) 45° (c) 30° (d) 15°
(c) 90° (d) 60°

Q
O

C O

P
Q
A B
Sol. In the given figure, O is the centre of the
Sol. In the given figure, O is the centre of the circle
circle
Chord PQ = radius of the circle
 OPQ is an equilateral triangle
 POQ = 60°
Arc PQ subtends POQ at the centre and
C O PRQ at the remaining part of the circle
90° 1 1
 PRQ = POQ = × 60° = 30° (c)
30°
2 2
A B 5. In the given figure, if O is the centre of the
circle then the valeu of x is
In AOB,
(a) 18° (b) 20°
AOB = 90°, ABC = 30°
(c) 24° (d) 36°
In AOB, AOB = 90°
OA = OB (radii of the same circle)
90 D C
 OAB = OBA = = 45°
2
O
Arc AB subtends AOB at the centre and 2x
ACB at the remaining part of the circle
3x
1 1 B
 ACB = AOB = × 90° = 45°
2 2
A
Now in ACB, ABC = 30°, ACB = 45°
 BAC = 180° – (30° + 45°) Sol. In the given figure, O is the centre of the
= 180° – 75° = 105° circle.
But OAB = 45° Join OA.
443 Arun Deep's Understanding Math-10
Now in right OPQ
OQ2 = OP2 + PQ2
D C (25)2 = OP2 + (24)2
 OP2 = 252 – 242 = 625 – 576
O 2x
 OP2 = 49 = (7)2  OP = 7 cm
3x
 Radius of the circle = 7 cm (a)
B 6. From a point which is at a distance of 13 cm
from the centre O of a circle of radius 5 cm,
A the pair of tangents PQ and PR to the circle
ADB = ACB = 2x are drawn. Then the area of the quadrilateral
PQOR is
(Angles in the same segment)
(a) 60 cm2 (b) 65 cm2
arc AB subtends AOB at the centre and 2
ADB at the remaining part of the circle (c) 30 cm (d) 32.5 cm2
 AOB = 2ADB = 2 × 2x = 4x Sol. Let point P is 13 cm from O, the centre of
the circle
In OAB,
Radius of the circle (OQ) = 5 cm
OAB = OBA = 3x(OA = OB)
PQ and PR are tangents from P to the circle
Sum of angles of OAB = 180°
Join OQ and OR
 3x + 3x + 4x = 180°
 10x = 180°
180 Q
 x= = 18°
10
 x = 18° (a) 5cm
P.Q. From a point Q, the length of the tangent to Q
O 13cm
a circle is 24 cm and the distance of Q from
the centre is 25 cm. The radius of the circle
is
(a) 7 cm (b) 12 cm R

(c) 15 cm (d) 24.5 cm


Sol. From Q, length of tangent PQ to the circle
PQ is tangent and OQ is the radius
= 24 cm
 PQ2 = OP2 – OQ2 = 132 – 52 = 169 – 25
and QO = 25 cm
= 144 = (12)2
P  PQ = 12 cm
24 1
cm
Now area of OPQ = PQ × OQ
2
O Q
25cm
1 
 base  height 
2 

1
= × 12 × 5 = 30 cm2
PQ is tangent and OP is radius 2
 OP  PQ  area of quad. PQOR = 2 × 30 = 60 cm2 (a)
444 Arun Deep's Understanding Math-10
P.Q. If angle between two radii of a circle is 130°, POR = 55°
the angle between the tangents at the ends OR is radius and PR is tangent
of the radii is  OR  PR
(a) 90° (b) 50°
 In OPR
(c) 70° (d) 40°
OPR = 90° – 55° = 35°
Sol. Angles between two radii OA and OB = 130°
 QPR = 2 × OPR = 2 × 35° = 70° (c)
From A and B, tangents are drawn which
meet at P P.Q. If tangents PA and PB from an exterior point
P to a circle with centre O are inclined to
A each other at an angle of 80°, then POA is
equal to
(a) 50° (b) 60°
(c) 70° (d) 100°
O 130° P
Sol. Length of tangents PA and PB to the circle
from a point P outside the circle with centre
O, and inclined an angle of 80°
B

OA radius and AP is tangent to the circle


A
 OAP = 90°
Similarly OBP = 90°
 AOB + APB = 180°
 130° + APB = 180° O P
80°
 APB = 180° – 130° = 50° (b)
7. In the given figure, PQ and PR are
tangents from P to a circle with centre O. If
B
POR = 55°, then QPR is
(a) 35° (b) 55°
(c) 70° (d) 80°
OA is radius and AP is tangent
1
Q  OAP = 90° and OPA = APB
2

1
= × 80° = 40°
P 2
O 55°
 POA = 90° – 40° = 50° (a)
8. In the given figure, PA and PB are tangents
R from point P to a circle with centre O. If the
radius of the circle is 5 cm and PA  PB,
then the length OP is equal to
Sol. In the given figure, PQ and PR are the (a) 5 cm (b) 10 cm
tangents to the circle from a point P outside (c) 7.5 cm (d) 5 2 cm
it
445 Arun Deep's Understanding Math-10

X
A
C

P O
5cm O E
A B
8cm
B

Sol. In the given figure, PA and PB are tangents D


Y
to the circle with centre O. Radius of the
circle is 5 cm, PA  PB.
In right OEC,
OE = 8 – 5 = 3 cm
A OC = 5 cm
5c

 CE = OC 2  OE 2 = 5 2  32
m

P O
= 25  9 = 16 = 4 cm
B  CD = 2 × CE = 2 × 4 = 8 cm (d)
10. If radii of two concentric circles are 4 cm
OA is radius and PA is tangent to the circle and 5 cm, then the length of each chord of
 OA  PA one circle which is tangent to the other is
APB = 90° ( PA  PB) (a) 3 cm (b) 6 cm
(c) 9 cm (d) 1 cm
1
 APO = 90° × = 45° Sol. Radii of two concentric circles are 4 cm and
2 5 cm
 AOP = 90° – 45° = 45° AB is chord of the bigger circle which is
i.e., OA = AP = 5 cm tangent to the smaller circle at C.
 OP = Join OA, OC
OA 2  PA 2 = 52  52
= 25  25 = 50 = 2  25
= 2  5 cm = 5 2 cm (d)
9. At one end A of a diameter AB of a circle of
radius 5 cm, tangent XAY is drawn to the O
circle. The length of the chord CD parallel
to XY and at a distance 8 cm from A is
(a) 4 cm (b) 5 cm
(c) 6 cm (d) 8 cm A B
C
Sol. AB is the diameter of a circle with radius
5 cm
At A, XAY is a tangent to the circle CD || OC = 4 cm, OA = 5 cm
XAY at a distance of 8 cm from A and OC  ACB
Join OC  In right OAC
446 Arun Deep's Understanding Math-10
OA2 = OC2 +AC2  = + 52 42 AC2 Sol. In the given figure, O is the centre of the
 25 = 16 + AC2  AC2 = 25 – 16 = 9 = (3)2 circle. PR is tangent and PQ is chord
 AC = 3 cm RPQ = 50°
 Length of chord AB = 2 × AC OP is radius and PR is tangent to the circle
= 2 × 3 = 6 cm (b)  OP  PR
P.Q. In the given figure, AB is a chord of the circle But OPQ + RPQ = 90°
such that ACB = 50°. If AT is tangent to  OPQ + 50° = 90°
the circle at the point A, then BAT is equal  OPQ = 90° – 50° = 40°
to OP = OQ (radii of the same circle)
(a) 65° (b) 60°  OQP = OPQ = 40°
(c) 50° (d) 40° and POQ = 180° – (OPQ + OQP)
= 180° – (40° + 40°)
C
= 180° – 80° = 100° (a)
50°
12. In the given figure, PA and PB are tangents
to a circle with centre O. If APB = 50°,
then OAB is equal to
B (a) 25° (b) 30°
(c) 40° (d) 50°

A T
A
Sol. In the given figure, AB is a chord of the circle
such that ACB = 50°
AT is tangent to the circle at A
50° O
AT is tangent and AB is chord
 ACB = BAT = 50°
(Angles in the alternate segments) (c)
11. In the given figure, O is the centre of a circle B
and PQ is a chord. If the tangent PR at P
makes an angle of 50° with PQ, then POQ
is Sol. In the given figure, PA and PB are tangents
(a) 100° (b) 80° to the circle with centre O. APB = 50°
(c) 90° (d) 75° But AOB + APB = 180°
 AOB + 50° = 180°
P R  AOB = 180° – 50° = 130°
50° In OAB,
OA = OB (radii of the same circle)
 OAB = OBA
O Q But OAB + OBA = 180° – AOB
= 180° – 130° = 50°
50
 OAB = = 25° (a)
2
447 Arun Deep's Understanding Math-10
12. In the given figure, sides BC, CA and AB of respectively. If PC = 5 cm, AR = 4 cm and
ABC touch a circle at point D, E and F RB = 6 cm, then the perimeter of ABC is
respectively. If BD = 4 cm, DC = 3 cm and (a) 60 cm (b) 45 cm
CA = 8 cm, then the length of side AB is (c) 30 cm (d) 15 cm
(a) 12 cm (b) 11 cm
(c) 10 cm (d) 9 cm A

A
R Q

F E B P C

Sol. In the given figure, sides BC, CA and AB of


ABC touch a circle at P, Q and R
respectively
B D C PC = 5 cm, AR = 4 cm, RB = 6 cm

Sol. In the given figure, sides BC, CA and AB of A


ABC touch a circle at D, E and F

m
4c
respectively.
R Q
BD = 4 cm, DC = 3 cm and CA = 8 cm
m
6c

B P 5cm C
 AR and AQ are tangents to the circles
 AQ = AR = 4 cm
8c

F E
Similarly CQ = CP = 5 cm
m

and BP = BR = 6 cm
Now AB = AR + BR = 4 + 6 = 10 cm
BC = BP + CP = 6 + 5 = 11 cm
AC = AQ + CQ = 4 + 5 = 9 cm
B 4cm D 3cm C
 Perimeter of the ABC = AB + BC + CA
BD and BF are tangents to the circle = 10 + 11 + 9 = 30 cm (c)
 BF = BD = 4 cm 15. PQ is a tangent to a circle at point P. Centre
Similarly, CD = CE = 3 cm of circle is O. If OPQ is an isosceles
 AE = AC – CE = 8 – 3 = 5 cm triangle, then QOP is equal to
and AF = AE = 5 cm (a) 30° (b) 60°
Now AB = AF + BF = 5 + 4 = 9 cm (d) (c) 45° (d) 90°
14. In the given figure, sides BC, CA and AB of Sol. PQ is tangent to the circle at point P centre
ABC touch a circle at the points P, Q and R of the circle is O.
448 Arun Deep's Understanding Math-10
centre O. If C is a point on the circle and
APB = 40°, then ACB is equal to
(a) 80° (b) 70°
O (c) 90° (d) 140°

A
P Q

OPQ is an isosceles triangle


OP = PQ C O 40° P
OP is radius and PQ is tangent to the circle
 OP  PQ i.e., OPQ = 90°
OP = PQ ( OPQ is an isosceles triangle) B
90
 QOP = PQO = = 45° (c)
2
Sol. In the given figure, PA and PB are tangents
P.Q. In the given figure, PT is a tangent at T to to the circle at A and B respectively C is a
the circle with centre O. If TPO = 25°, point on the circle and APB = 40°
then the value of x is
But APB + AOB = 180°
(a) 25° (b) 65°
 40° + AOB = 180°
(c) 115° (d) 90°
 AOB = 180° – 40° = 140°
Now arc AB subtends AOB at the centre
T and ACB is on the remaining part of the
circle
O 25° 1 1
P  ACB = AOB = × 140° = 70° (b)
x 2 2
17. In the given figure, two circles touch each
other at A. BC and AP are common tangents
to these circles. If BP = 3.8 cm, then the
Sol. In the given figure, PT is the tangent at T to lenght of BC is equal to
the circle with centre O. TPO = 25° (a) 7.6 cm (b) 1.9 cm
OT is radius and TP is the tangent (c) 11.4 cm (d) 5.7 cm
 OT  TP
 In OPT B P C
TOP + OPT = 90°
 TOP + 25° = 90°
A
TOP = 90° – 25° = 65°
But TOP + x = 180° (Linear pair)
65° + x = 180°  x = 180° – 65° = 115°
 x = 115° (c) Sol. In the given figure, two circles touch each
16. In the given figure, PA and PB are tangents other at A. BC and AP are common tangents
at points A and B respectively to a circle with to these circles. BP = 3.8 cm
449 Arun Deep's Understanding Math-10
PB and PA are the tangents to the first circle Sol. In the given figure, PQR is a tangent at Q to
 PB = PA = 3.8 cm a circle.
Similarly PC and PA are tangents to the Chord AB || PR and BQR = 70°
second circle BQ is chord and PQR is a tangent
 PA = PC = 3.8 cm  BQR = A
BC = PB + PC = 3.8 + 3.8 = 7.6 cm (a) (Angles in the alternate segments)
18. In the given figure, if sides PQ, QR, RS and AB || PQR
SP of a quadrilateral PQRS touch a circle at  BQR = B (alternate angles)
points A, B, C and D respectively, then  A = B = 70°
PD + BQ is equal to  AQB + A + B = 180°
(a) PQ (b) QR (Angles of a triangle)
(c) PS (d) SR  AQB + 70° + 70° = 180°
 AQB + 140° = 180°
P
 AQB = 180° – 140° = 40° (b)
D
S 20. Two chords AB and CD of a circle intersect
A externally at a point P. If PC = 15 cm,
C CD = 7 cm and AP = 12 cm, then AB is
Q (a) 2 cm (b) 4 cm
B (c) 6 cm (d) none of these
R

Sol. In the given figure, sides PQ, QR, RS and C


SP of a quadrilateral PQRS touch a circle at D
the points A, B, C and D respectively.
P
PD and PA are the tangents to the circle
 PA = PD ...(i) B
A
Similarly QA and QB are the tangents
 QA = QB ...(ii) Sol. In the given figure, two chords AB and CD
of a circle intersect externally at P.
Now PD + BQ = PA + QA = PQ (a)
PC = 15 cm, CD = 7 cm, AP = 12 cm
[From (i) and (ii)]
Join AC and BD
19. In the given figure, PQR is a tangent at Q to
a circle. If AB is a chord parallel to PR and C
7cm
BQR = 70°, then AQB is equal to D
15 c m
(a) 20° (b) 40°
P
(b) 35° (d) 45°
B
A 12cm
A
B

In APC and BPD


P = P (common)
70° A = BDP
P Q R {Ext. of a cyclic quad. is equal to its interior
opposite angles}
450 Arun Deep's Understanding Math-10
 APC ~ BPD (AA axiom) In OQP, OQP = OPQ = 40º

PA PC  POQ = 180º – 40º – 40º = 100º


=
PD PB
 TOP = x
PA.PB = PC.PD
= 180º – ( POQ + ROQ)
12.PB = 15 × 8 (PD = 15 – 7 = 8)
= 180º – (100º + 20º) = 180º – 120º
15  8
PB = = 10 = 60º Ans.
12
2. (a) In the figure (i) given below, triangle
 AB = AP – PB = 12 – 10 = 2 cm (a)
ABC is equilateral. Find BDC and BEC.

CHAPTER TEST (b) In the figure (ii) given below, AB is a


diameter of a circle with centre O. OD is
1. In the figure given below, ‘O’ is the centre perpendicular to AB and C is a point on the
of the circle. If QR = OP and  ORP = 20º, arc DB. Find BAD and ACD.
find the value of ‘x’ giving reasons.
(2018)

Sol.In the figure,


(1) QR = OP (given)
Sol.
(2) But OP = OQ are radii of same circle
(a) ... ABC is an equilateral triangle.
 QR = OQ from st (1), (2)
 Each angle = 60º
 ROQ = 20º
 A = 60º
OQP = QOR + ORQ
But A = D
Interior angle
(Angles in the same segment)
 OQP = 20º + 20º = 40º
451 Arun Deep's Understanding Math-10
 D = 60º or BAD = 45º
Now ABEC is a cyclic quadrilateral, (ii) Arc AD subtends AOD at the centre and
 A + E = 180º ACD at the remaining part of the circle.
 AOD = 2 ACD
 90º = 2 ACD (... OD  AB)

90º
 ACD =  45º
2

3. (a) In the given figure AC is a tangent to the


circle with centre O. If ADB = 55°, find x and
y. Give reasons for your answers.

 60º + E = 180º
 E = 180º – 60º B

 E = 120º
Hence BDC = 60o and BEC = 120o
(b) AB is diameter of circle with centre O. O
y° E
OD  AB and C is a point on arc DB.
x° C
55°
D
A

(b) In the figure given below, AB is the diameter


of the semi-circle ABCDE with centre O.
If AE = ED and BCD = 140º, find AED
and EBD. Also Prove that OE is parallel
(i) In AOD, AOD = 90º
to BD.
and OA = OD (radii of the semi-circle)
 OAD = ODA
But OAD + ODA = 90º
 OAD + OAD = 90º
 2 OAD = 90º

90º Sol. (a) For y :


 OAD =  45º
2 BAD = OAD = 90°
452 Arun Deep's Understanding Math-10
[Angle between radius and tangent Chord AE = ED (given)
at the point of contact]
 DBE = EBA
In BAD,
But DBE + EBA = 50º
B + 90° + 55° = 180°
 B = 35°  DBE + DBE = 50º
AOE = 2B = 2 × 35°  2 DBE = 50º
[Angle at centre is twice the angle at  DBE = 25º or EBD = 25º
remaining circumference]
In OEB, OE = OB
 y° = 70° i.e. y = 70
(radii of the same circle)
For x :
In OAC,  OEB = EBO = DBE
x° + y° + A = 180° But these are alternate angles.
 x° + 70° + 90° = 180  OE | | BD Q.E.D.
i.e. x = 20 4. (a) In the figure (i) given below, O is the
(b) AB is the diameter of semi-circle ABCDE centre of the circle.
with centre O. AE = ED and BCD = 140º.
Prove that AOC = 2 (ACB + BAC).
In cyclic quadrilateral EBCD.
(b) In the figure (ii) given below, O is the centre
of the circle. Prove that x + y = z.

(i) BCD + BED = 180º


 140º + BED = 180º
 BED = 180º – 140º = 40º
But AEB = 90º
(Angles in a semi-circle)
 AED = AEB + BED
Sol. (a) Given : O is the centre of the circle.
= 90º + 40º = 130º.
To Prove : AOC = 2 (ACB + BAC).
(ii) Now in cyclic quadrilateral AEDB, Proof : In ABC,
AED + DBA = 180º ACB + BAC + ABC = 180º
 130º + DBA = 180º (Angles of a triangle)
 ABC = 180º– (ACB + BAC)...(i)
 DBA = 180º – 130º = 50º
453 Arun Deep's Understanding Math-10
In the circle, arc AC subtends AOC at
the centre and ABC at the remaining part
of the circle.
Reflex AOC = 2 ABC ...(ii)
Reflex AOC = 2 [180º – (ACB + BAC)]
Reflex AOC = 360º – 2 (ACB + BAC)
But AOC = 360º – reflex AOC

Similarly in ABD
Ext. BDC = x + ABD
= x + EBD ...(iii)
Substituting the value of (ii) and (iii) in (i)

= 360 – [360º – 2 (ACB + BAC) BOC = y – EBD + x + EBD = x + y

= 360º – 360º + 2 (ACB + BAC)  z=x+y Q.E.D.

= 2 (ACB + BAC) 5. (a) In the figure (i) given below, AB is


diameter of a circle. If DC is parallel to AB
Hence AOC = 2 (ACB + BAC) and CAB = 25º, find :
Q.E.D.
(i) ADC
(b) Given : In the figure, O is the centre of the
(ii) DAC.
circle.
(b) In the figure sides AB and DC of a cyclic
To Prove : x + y = z.
quadrilateral are produced to meet at a
Proof : Arc BC subtends AOB at the point P and the sides AD and BC are
centre and BEC at the remaining part of produced to meet at a point Q. If ADC
the circle. = 75° and BPC = 50°, find BAD and
 BOC = 2 BEC CQD.

But BEC = BDC


(Angles in the same segment)
 BOC = BEC + BDC ...(i)
In ABD, Ext. y = (EBD + BEC)
BEC = y – EBD ...(ii)
454 Arun Deep's Understanding Math-10
 CAD = 180° – 125° = 55°
Q
Since ABCD is a cyclic quadrilateral
 ADC + ABC = 180°

D  75° + ABC = 180°


 ABC = 180° – 75° = 105°
75°
C In AQB, CQD + BAD + ABC =
180°
50°  CQD + 55° + 105° = 180°
A P
B CQD = 180° – 760° = 20°
6. (a) In the figure (i) given below, ABCD is a
cyclic quadrilateral. If AB = CD, Prove that
Sol. (a) AB is diameter and DC | | AB,
AD = BC.
CAB = 25º. Join AD, BD.
(b) In the figure (ii) given below, ABC is an
isosceles triangle with AB = AC. If ABC
= 50°, find BDC and BEC.

C A D

BAC = BDC D
(Angles in the same segment) 50°
B C
But ADB = 90º A
B
(Angles in a semi-circle) E
 ADC = ADB + BAC = 90º + 25º (i) (ii)
= 115º
Sol. (a) Given : ABDC is a cyclic quadrilateral
DC | | AB (given) AB = CD.
 CAB = ACD (alternate angles) To Prove : AD = BC.
 ACD = 25º
Now in ACD,
DAC + ADC + ACD = 180º
(Angles of a triangle)
 DAC + 115º + 25º = 180º
 DAC + 140º = 180º
 DAC = 180º – 140º = 40º Ans. Construction : Join AD and BC.
(b) Since the sum of angles of PAD be 180° Proof : In ABD and CBD
 ADC + BAD + BPC = 180° AB = CD (given)
 75° + BAD + 50° = 180° BD = BD (common)
455 Arun Deep's Understanding Math-10
BAD = BCD Now in right OTP
(Angles in the same segment) OP2 = OT2 + TP2
 ABC  CBD  (13)2 = OT2 + (12)2
(SSA axiom of congruency)  169 = OT2 + 144
 BC = AD Q.E.D.  OT2 = 169 – 144 = 25 = (5)2
(b) Given : ABC is an isosceles triangle and  OT = 5 cm
ABC = 50° The nearest point is A from P to cut circle
In ABC, an isosceles triangle over OA = radius of the circle = 5 cm.
 ACB = ABC  AP = OP – OA = 13 – 5 = 8 cm Ans.
( opp. s of an isosceles s) 8. Two circles touch each other internally.
 ACB = 50° Prove that the tangents drawn to the two
Also, in ABC circles from any point on the common
tangent are equal in length.
 ABC + ACB + BAC = 180°
Sol. Given : Two circles with centre O and O
(Sum of an isosceles triangle is 180°)
touch each other internally at P.
 50° + 50° + BAC = 180°
 BAC = 180° – 100°
 BAC = 80°
Also BDC = BAC
BDC = 80°
(Angles in the same segment)
Now ABEC is a cyclic quadrilateral
 A + E = 180°
 80° + E = 180°  E = 180° – 80° PT is any point on the common tangent of
 E = 100° circles at P. From T, TA and TB are the
tangents drawn to two circles.
Hence BDC = 80° and BEC = 100°
7. A point P is 13 cm from the centre of a To Prove : TA = TB.
circle. The length of the tangent drawn from Proof : From T, TA and TP are the tangents
P to the circle is 12 cm. Find the distance to the first circle.
of P from the nearest point of the circle.  TA = TP ...(i)
Sol. Join OT, OP = 13 cm and TP = 12 cm Similarly, from T, TB and TP are the
tangents to the second circle.
 TB = TP ...(ii)
From (i) and (ii)
TA = TB Q.E.D.
9. From a point outside a circle with centre
O, tangents PA and PB are drawn. Prove
. ..
OT is the radius that
 OT  TP (i) AOP = BOP.
456 Arun Deep's Understanding Math-10
(ii) OP is the perpendicular bisector of the 10. (a) The figure given below shows two
chord AB. circles with centres A, B and a transverse
Sol. Given : From a point P, outside the circle common tangent to these circles meet the
with centre O. PA and PB are the tangents straight line AB in C. Prove that :
to the circle, OA, OB and OP are joined. AP : BQ = PC : CQ.

A B
C
Q

(b) In the figure (ii) given below, PQ is a


To Prove : (i) AOP = BOP tangent to the circle with centre O and AB
(ii) OP is the perpendicular bisector of the is a diameter of the circle. If QA is parallel
chord AB. to PO, prove that PB is tangent to the circle.
Construction : Join AB which intersects
OP at M. Q A
Proof : In right OAP and OBP
Hyp. OP = OP (common) P O
Side OA = OB (radii of the same circle)
B
 OAP  OBP
(R.H.S. Axiom of congruency) Sol. (a) Given : Two circles with centres A and
 AOP = BOP (C.P.C.T.) B and a transverse common tangent to these
circles meet AB at C.
and APO = BPO (C.P.C.T.)
Now in APM and BPM,
PM = PM (common)
APM = BPM (proved)
AP = BP
(tangents from P to the circle)
 APM  BPM
(SAS axiom of congruency)
 AM = BM (C.P.C.T.) To Prove : AP : BQ = PC : CQ.
and AMP = BMP Proof : In APC and BQC
But AMP + BMP = 180º PCA = QCB
 AMP = BMP = 90º (vertically opposite angles)
 OP is perpendicular bisector of AB at M. APC = BQC (each 90º)
Q.E.D.  APC  BQC
457 Arun Deep's Understanding Math-10
AP PC N
   AP : BQ = PC : CQ 12c
BQ CQ m
17cm A
P
Q.E.D. B 13cm Q
15cm
(b) Given : In the figure, O is the centre of
the circle. AB is diameter. PQ is the tangent M
and QA || PO Sol. In the given figure, two chords with centre
A and B touch externally. PM is a tangent to
Q A the circle with centre A and QN is tangent to
the circle with centre B. PM = 15 cm, QN =
12 cm, PA = 17 cm, QB = 13 cm. We have
P O to find AB.
B N
12c
m
17cm A R
To prove : PB is tangent to the circle P
B 13cm Q
Construction : Join OQ 15cm
Proof : In OAQ M
OQ = OA (Radii of the same circle) AM is radius and PM is tangent
 OQA = OAQ  AM  PM
QA || PQ Similarly, BN  NQ
 OAQ = POB (Corresponding angles) Now in right APM,
AP2 = AM2 + PM2
and OQA = QOP (Alternate angles)
 172 = AM2 + 152
But QAO = OQA (Proved)
 AM2 = 172 – 152
 POB = QOB = 289 – 225 = 64 = (8)2
Now, in OPQ and OBP  AM = 8 cm
OP = OP (Common) Similarly in right BNQ
OQ = OB (Radii of the same circle) BQ2 = BN2 + NQ2
BOP = POB (Proved) 132 = BN2 + 122
 OPQ  OBP (SAS axiom)  169 = BN2 + 144
 OQP = OBP (c.p.c.t.) BN2 = 169 – 144 = 25 = (5)2
 BN = 5 cm
But OQP = 90°
Now AB = AM + BN
( PQ is tangent and OQ is the radius)
(AR = AM and BR = BN)
 OBP = 90°
= 8 + 5 = 13 cm
 PB is the tangent of the circle. 12. Two chords AB, CD of a circle intersect
11. In the figure given below, two circles with externally at a point P. If
centres A and B touch externally. PM is a
PB = 7 cm, AB = 9 cm and PD = 6 cm,
tangent to the circle with centre A and QN is
find CD.
a tangent to the circle with centre B. If PM
= 15 cm, QN = 12 cm, PA = 17 cm and QB Sol. ... AB and CD are two chords of a circle
= 13 cm, then find the distance between the which intersect each other at P, outside the
centres A and B of the circles. circle.
458 Arun Deep's Understanding Math-10

A
9 cm
B 7 cm
P
D 6 cm
C

 PA × PB = PC × PD.
PB = 7 cm, AB = 9 cm, PD = 6 cm
Sol. Given : (a) AB is chord of a circle with
AP = AB + BP = 9 + 7 = 16 cm
centre O and PT is tangent and CD is the
 PA × PB = PC × PD diameter of the circle which meet at P.
 16 × 7 = PC × 6 AP = 16 cm, AB = 12 cm, OP = 2 cm
16×7 56  PB = PA – AB = 16 – 12 = 4 cm
PC =  cm
6 3 . .. ABP is a secant and PT is tangent.
 CD = PC – PD  PT2 = PA × PB
56 38 2
 6  12 cm
3 3 3
13. (a) In the figure (i) given below, chord AB
and diameter CD of a circle with centre O
meet at P. PT is tangent to the circle at T.
If AP = 16 cm, AB = 12 cm and DP = 2
cm, find the length of PT and the radius of
the circle.

= 16 × 4 = 64 = (8)2
 PT = 8 cm
Again PT2 = PD × PC

8×8
 (8)2 = 2 × PC  PC =
2
 PC = 32 cm
 CD = PC – PD = 32 – 2 = 30 cm
(b) In the figure (ii) given below, chord AB
and diameter CD of a circle meet at P. If 30
Radius of the circle =  15 cm .
AB = 8 cm, BP = 6 cm and PD = 4 cm, 2
find the radius of the circle. Also find the
(b) Chord AB and diameter CD intersect
length of the tangent drawn from P to the
each other at P outside the circle. AB = 8 cm,
circle.
BP = 6 cm, PD = 4 cm.
459 Arun Deep's Understanding Math-10

PT is the tangent to the circle drawn from


BX = 5 cm, OX = 10 cm and radius of the
P.
. ..
circle = 6 cm
Two chords AB and CD intersect each
other at P outside the circle.  XP = XO + OP = 10 + 6 = 16 cm
PA = AB + PB = 8 + 6 = 14 cm XQ = XO – OQ = 10 – 6 = 4 cm
 PA × PB = PC × PD  XB.XA = XP.XQ
 14 × 6 = PC × 4  5.XA = 16 × 4

14 × 6 84 16 × 4 64
 PC =   21 cm  XA =  cm
4 4 5 5
 CD = PC – PD = 21 – 4 = 17 cm 4
XA = 12 cm
17 5
 Radius of the circle   8 .5 cm .
2
4 4
(ii) Now PT is the tangent and ABP is secant.  AB = XA – XB = 12  5  7 cm
5 5
 PT2 = PA × PB = 14 × 6 = 84
= 7·8 cm
PT = 84  4 × 21  2 21 cm Now . .. XT is the tangent of the circle.
14. In the figure given below, chord AB and  XT2 = XP.XQ = 16 × 4 = 64 = (8)2
diameter PQ of a circle with centre O meet
XT = 8 cm
at X. If BX = 5 cm, OX = 10 cm and the
radius of the circle is 6 cm, compute the 15. (a) In the figure (i) given below, CBP =
length of AB. Also find the length of tangent 40º, CPB = qº and DAB = pº. Obtain an
drawn from X to the circle. equation connecting p and q. If AC and BD
meet at Q so that AQD = 2 qº and the
points C, P, B and Q are concyclic, find the
values of p and q.
(b) In the figure (ii) given below, AC is a
diameter of the circle with centre O. If
CD | | BE, AOB = 130º and ACE = 20º,
find :
(i) BEC (ii) ACB
Sol. Chord AB and diameter PQ meet at X on
producing outside the circle. (iii) BCD (iv) CED.
460 Arun Deep's Understanding Math-10
 130º + BOC = 180º
D  BOC = 180º – 130º = 50º

C
E
O

A B

Sol. (a) (i) Given : ABCD is a cyclic


quadrilateral.
 Ext. PBC = ADC (i) Now arc BC subtends BOC at the centre
 40º = ADC and BEC at the remaining part of the circle.

1 1
 BEC = BOC = × 50º  25º .
2 2
(ii) Similarly arc AB subtends AOB at the
centre and ACE at the remaining part of
the circle

1 1
 ACB = AOB = × 130º  65º .
2 2
In ADP,
p + q + ADP = 180º (iii) ... CD | | EB
p + q = 180º – ADP = 180º – ADC  ECD = CEB (alternate angles)
= 180º – 40º = 140º = 25º
 p + q = 140º.  BCD = ACB + ACE + ECD
(ii) ... C, P, B, Q are concyclic = 65º + 20º + 25º = 110º.
 CPB + CQB = 180º (iv) . .. EBCD is a cyclic quadrilateral
 q + 2 q = 180º (... CQB = DQA)
 CED + BCD = 180º
 3 q = 180º
 CED + BEC + BCD = 180º
180º
 q  60º  CED + 25º + 110º = 180º
3
 CED + 135º = 180º
But p + q = 140º
 CED = 180º – 135º = 45º
 p + 60º = 140º
 p = 140º – 60º = 80º PQ. (a) In the figure (i) given below, APC, AQB
Hence p = 80º, q = 60º Ans. and BPD are straight lines.
(b) AOB = 130º (i) Prove that ADB + ACB = 180º.
But AOB + BOC = 180º (ii) If a circle can be drawn through A, B,
(Linear pair) C and D, Prove that it has AB as diameter.
461 Arun Deep's Understanding Math-10
 ADP + PCB = 180º
 ADB + ACB = 180º
(ii) If A, B, C and D are concyclic then
ADB = ACB
(Angles in the same segment)
But ADB + ACB = 180º
(b) In the figure (ii) given below, AQB is a (proved in (i))
straight line. Sides AC and BC of ABC  ADB = ACB = 90º
cut the circles at E and D respectively. Prove But these are angles on one side of AB.
that the points C, E, P and D are concyclic.
 AB is the diameter of the circle.
Q.E.D.
(b) Given : AQB is a straight line. Sides AC
and BC of ABC cut the circles at E and D
respectively.

Sol. (a) Given : In the figure, APC, AQB and


BPD are straight lines.
To Prove : C, E, P, D are concyclic.
Construction : Join PE, PD and PQ.
Proof : In cyclic quad. AQPE,
A + EPQ = 180º
 EPQ = 180º – A ...(i)
Similarly PQBD is cyclic quadrilateral
 QPD = 180º – B
To Prove : (i) ADB + ACB = 180º But EPD + EPQ + QPD = 360º
(ii) If a circle is drawn through A, B, C and (angles at a point)
D, then AB is a diameter.  EPD + 180º – A + 180º – B = 360º
Construction : Join PQ.  EPD = A + B
Proof : In cyclic quad. AQPD. Adding C both sides,
ADP + AQB = 180º EPD + C = A + B + C = 180º
But AQB = PCB  EPDC is a cyclic quadrilateral.
(Ext. angle of a cyclic quad. is equal Hence E, P, D and C are concyclic.
to its interior opposite angle) Q.E.D.
462 Arun Deep's Understanding Math-10
16. In the figure (i) given below, chords AB,  AEB = 30º
BC and CD of a circle with centre O are Arc AB subtends AOB at the centre and
equal. If BCD = 120º, find AEB at the remaining part of the circle.
(i) BDC (ii) BEC
 AOB = 2 AEB = 2 × 30º = 60º
(iii) AEB (ii) AOB.
Hence Prove that OAB is equilateral. Now in AOB
OA = OB (radii of the same circle)
 OAB = OBA
But OAB + OBA + AOB = 180º
 OAB + OAB + 60º = 180º
 2 OAB = 180º – 60º = 120º
 OAB = 60º
 OAB = OBA = AOB = 60º
Hence OAB is an equilateral triangle.
Q.E.D.
Sol. In BCD, BC = CD P.Q. In the figure given below, AB is a diameter
 CBD = CDB of a circle with centre O. The chord BC of
But BCD + CBD + CDB = 180º the circle is parallel to the radius OD and
( Angles of a triangle) the lines OC and BD meet at E. Prove that
(i) CED = 3 CBD (ii) CD = DA.
Sol. Given : In a circle with centre O, AB is
diameter and chord BC | | radius OD, OC
and BD intersect each other at E.

 120º + CBD + CBD = 180º


 120º + 2 CBD = 180º
 2 CBD = 180º – 120º = 60º
To Prove : (i) CED = 3 CBD
 CBD = 30º and CDB = 30º
(ii) CD = DA.
BEC = CDB
(Angles in the same segment) Construction : CD, DA are joined.
 BEC = 30º Proof : Arc CD subtends COD at the
... centre and CBD at the remaining part of
CB = AB
the circle.
 BEC = AEB
 COD = 2 CBD ...(i)
(equal chords subtend equal angles)
463 Arun Deep's Understanding Math-10
BC | | OD
 CBD = BDO ...(ii)
In DOE,
Ext. BEO = EDO + EOD
= BDO + COD
= CBD + 2 CBD
From (i) and (ii)
= 3 CBD Sol.(a) AB and XY are diameters of a circle with
But CED = BEO centre O.
(vertically opposite angles) APX = 30º.
 CED = 3 CBD To find :
(ii) In DBO, (i) AOX (ii) APY
OD = OB (iii) BPY (iv) OAX
(radii of the same circle) (i) arc AX subtends AOX at the centre
. ..

and APX at the remaining point of the


 OBD = BDO
circle.
= CBD (from (ii))
 AOX = 2 APX
 ABD = CBD
= 2 × 30º = 60º
 AD = CD . ..
(ii) XY is the diameter
(... Equal chords subtend equal angles)
 XPY = 90º
Q.E.D.
(Angle in a semi-circle)
17. (a) In the adjoining figure, (i) given below AB
 APY = XPY – APX
and XY are diameters of a circle with centre O.
If APX = 30º, find = 90º – 30º = 60º
(i) AOX (ii) APY (iii) APB = 90º (Angle in a semi-circle)
(iii) BPY (iii) OAX.  BPY = APB – APY
= 90º – 60º = 30º
(iv) In AOX,
OA = OX (radii of the same circle)
 OAX = OXA
But AOX + OAX + OXA = 180º
(Angles of a triangle)
(b) In the figure (ii) given below, AP and
BP are tangents to the circle with centre O.  60º + OAX + OAX = 180º
If CBP = 25º and CAP = 40º, find :  2 OAX = 180º – 60º = 120º
(i) ADB (ii) AOB 120º
 OAX =  60º Ans.
(iii) ACB (iv) APB. 2
464 Arun Deep's Understanding Math-10
(b) Join CD.  ADB = CDA + CDB
= 40º + 25º = 65º.
(ii) arc ACB subtends AOB at the centre and
ADB at the remaining part of circle.
 AOB = 2 ADB = 2 × 65º = 130º.
(iii) ACBD is a cyclic quadrilateral
 ACB + ADB = 180º
 ACB + 65º = 180º
 ACB = 180º – 65º = 115º
(i) CDB = CBP (iv) AOB + APB = 180º
(Angles in the alternate segments)  130º + APB = 180º
 CDB = 25º ...(i)  APB = 180º – 130º = 50º
Similarly, CDA = CAP = 40º
(Angles in the alternate segments)
16
CONSTRUCTIONS
EXERCISE 16.1
1. Use a ruler and compass only in this each tangent.
question.
Sol. Steps of Construction :
(i) Draw a circle, centre O and radius 4 cm.
(i) Take a line segment AB =6 cm.
(ii) Mark a point P such that OP = 7 cm.
(ii) Draw its perpendicular bisector bisecting
Construct the two tangents to the circle
from P. Measure and record the length of it at O.
one of the tangents. (iii) With centre O and radius OB, draw a cirlce.
Sol. Steps of Construction : (iv) Produce AB to P such that OP = 5 cm.
(i) Draw a circle with centre O and radius 4 (v) Draw its perpendicular bisector intersecting
cm. it at M.
(ii) Take a point P such that OP = 7 cm.
(vi) With centre M and radius OM, draw a circle
(iii) Bisect OB at M. which intersects the given circle at T and
(iv) With centre M and diameter OP, draw S.
another circle intersecting the given circle
(vii) Join OT, OS, TP and SP.
at A and B.
PT and PS are the required tangents to the
(v) Join PA and PB.
given circle.
PA and PB is a pair of tangents to the circle.
On measuring, each tangent is 4 cm long
On measuring PA, it is equal to 5·5 cm.
i.e. PT = PS = 4 cm.

4c
m

m
3c

O 5 cm
A M P
6 cm B

2. Draw a line AB = 6 cm. Construct a circle


with AB as diameter. Mark a point P at a S
distance of 5 cm from the mid-point of AB.
Construct two tangents from P to the circle
with AB as diameter. Measure the length of 3. Construct a tangent to a circle of radius 4
465 Arun Deep's Understanding Math-10
466 Arun Deep's Understanding Math-10
cm from a point on the concentric circle of (v) Join PA, PB, QC and QD.
radius 6 cm and measure its length. Also PA, PB and QC and QD are the required
verify the measurement by actual calculation. tangents.
Sol. Steps of construction :
(i) Take a point O.
(ii) With centre O and radii 4 cm and 6 cm, draw C A
two concentric circles.
(iii) Join OA and take its mid-point M.
(iv) With centre M and radius MA, draw an other Q P
7 cm N O 3 cm M 7 cm
circle which intersects the first circle at P
and Q.
(v) Join AP and AQ. D B
AP and AQ are the required tangents to the
first circle from the point A.
5. Draw a line segment AB of length 8 cm.
Taking A as centre, draw a circle of radius 4
cm and taking B as centre, draw another
circle of radius 3 cm. Construct tangents to
P each circle from the centre of the other circle.
Sol. Steps of construction :
(i) Draw a line segment AB = 8 cm.
O M
A (ii) With centre A and radius 4 cm and with centre
B and radius 3 cm, draw circles.
(iii) Draw a third circle AB as diameter which
Q intersects the given two circles at C and D
and P and Q respectively.
(iv) Join AC and AD, BP and BQ.
(v) AC and AD, BP and BQ are the required
4. Draw a circle of radius 3 cm. Take two points tangents.
P and Q on one of its extended diameter each
at a distance of 7 cm from its centre. Draw
P
tangents to the circle from these two points C
P and Q.
Sol. Steps of construction :
(i) Take a point O and with centre O, and radius3 A B
4 cm 8cm 3 cm
cm, draw a circle.
(ii) Produce its diameter both sides and cut off
OP = OQ = 7 cm. D
(iii) Take mid-points of OP and OQ as M and N Q
respectively.
(iv) With centres M and N and OP and OQ as
diameters, draw circles which intersect the
given circle at A, B and C and D respectively.
467 Arun Deep's Understanding Math-10
EXERCISE 16.2  This is the required circle.

1. Draw an equilateral triangle of side 4 cm. X


Draw its circumcircle. C

O
90
A 4cm B
Sol. Steps of Construction :
(i) Draw a line segment BC = 4 cm
(ii) With centres B and C, draw two arcs of radius
3. Construct a triangle with sides 3 cm, 4 cm
4 cm which intersect each other at A.
and 5 cm. Draw its circumcircle and measure
(iii) Join AB and AC its radius.
 ABC is equilateral triangle. Sol. Steps of Construction
(iv) Draw the right bisector of BC and AC
(i) Draw a line segment BC = 4 cm.
intersecting each other at O.
(ii) With centre B and radius 3 cm and with centre
(v) Join OA, OB and OC.
C and radius 5 cm draw two arcs which
(vi) With centre O, and radius equal to OB or OC intersect each other at A.
or OA, draw a circle which will pass through
A, B and C. (iii) Join AB and AC.
(iv) Draw the perpendicular bisector of sides BC
This is the required circumcircle of  ABC.
and AC which intersect each other at O.
2. Using a ruler and a pair of compasses only,
(v) Join OB.
construct: (i) a triangle ABC given AB = 4cm,
BC = 6 cm and BC = 90°. (ii) a circle (vi) With centre O and radius OB, draw a circle
which passes through the points A, B and C which will pass through A, B and C.
and mark its centre as O. (2008)
Sol. Steps of Construction :
(i) Draw a line segment AB = 4cm
(ii) At B, draw a ray BX making an angle of 90o
and cut off BC = 6 cm.
(iii) Join AC.
(iv) Draw the perpendicular bisectors of sides
AB and AC intersecting each other at O.
(v) With centre O, and radius equal to OB or OA
or OC, draw a circle which passes through
A, B and C.
468 Arun Deep's Understanding Math-10
(vii) On measuring the radius OB = 2·5 cm.
4. Using ruler and compasses only :
(i) Construc a triangle ABC with the following data:
Base AB = 6 cm, AC = 5·2 cm and CAB = 60°.
(ii) In the same diagram, draw a circle which passes
through the points A, B and C. and mark its centre O.
Sol. Steps of Construction :
(i) Draw a line segment AB = 6 cm.
(ii) At A, draw a ray making an angle of 60°.
(iii) With centre B and radius 5·2 cm. draw an This is the required incircle. Measure the radius
arc which intersects the ray at C. ID which is 1·5 cm (approx)
(iv) Join BC. C 6.(i) Contruct a triangle ABC with BC = 6.4 cm,
CA = 5.8 cm and  ABC = 60º. Draw its incircle.
Measure and record the radius of the incircle.
(ii) Construct a ABC with BC = 6.5 cm,
O AB = 5.5 cm, AC = 5 cm. Construct the incircle
5.2

of the triangle. Measure and record the radius of


cm

60° the incircle. (2014)


A 6cm B Steps of Construction :
(i) Draw a line segment BC = 6.4 cm
(ii) Construct  DBC = 60º at B.
(iii) With C as centre and radius CA = 5.8 cm.
(v) Draw the perpendicular bisector of AB and Draw an arc cutting BD at A.
BC intersecting each other at O. (iv)Join AC. Then ABC is the required triangle.
(vi) With O as a centre and OA as a radius draw (v)Draw the angle bisectors of  B and  C
a circle which touches the Δ ABC at A, B and C. which intersect each other at O.
5. Using ruler and compasses only, draw an (vi)Draw OE  BC, intersecting BC in E.
equilateral triangle of side 5 cm and draw its (vii)With O as centre and D
inscribed circle. Measure the radius of the circle. OE as radius draw the A

Sol. Steps of Construction : required incircle.


(i) Draw a line segment BC = 5 cm Measure the radius
(ii) With centre B and C and radius 5 cm, draw OE which is = 1.5 cm.
two arcs intersecting each other at A.
(iii) Join AB and AC.
5.8

O
cm

(iv) Draw the angle bisectors of B and C


intersecting each other at I.
(v) From I, draw a perpendicualr ID on BC. E
B C
6.4 cm
(vi) With centre I and radius ID, draw a circle
which touches the sides of the triangle internally.
469 Arun Deep's Understanding Math-10
A

5.5
cm

cm
5.5

7. Using ruler and compasses only, construct


a triangle ABC in which BC = 4 cm, ACB
B 6.5cm N C
= 45° and the perpendicular from A on BC
is 2·5 cm. Draw the circumcircle of triangle
ABC and measure its radius.
Steps of construction : Steps of Construction
1. Draw a line BC = 6.5 cm. (i) Draw a line segment BC = 4 cm.
2. With centre B and C draw arcs AB = 5.5 cm (ii) At B, draw a perpendicular and cut BE =
and AC = 5 cm 2·5 cm.
3. Join AB and AC, ABC is the required triangle.
4. Draw the angle bisetors of B and C. Let these
bisectors meet at O.
5. Taking O as centre. Draw a incircle which
touches all the sides of the Δ ABC.
6. From O draw a perpendicular to side BC
which cut at N.
7. Measure ON which is required radius of the
incircle.
ON = 1.5 cm
P.Q. The bisectors of angles A and B of a scalene
triangle ABC meet at O. (iii) From E, draw a line EF parallel to BC.
(i) What is the point O called ? (iv) From C, draw a ray making an angle of
(ii) OR and OQ are drawn perpendicular to AB 45° which intersects EF at A.
and CA respectively. What is the relation (v) Join AB.
between OR aned OQ ? (vi) Draw the perpendicular bisectors of sides
(iii) What is the relation between ACO and BC and AC intersecting each other at O.
BCO ? (vii) Join OB, OC and OA.
Sol. (i) The point O where the angle bisectors
meet is called the incentre of the triangle. (viii) With centre O and radius OB or OC or OA
draw a circle which will pass through A, B
(ii) The perpendiculars drawn from O to AB
and C.
and CA are equal i.e. OR and OQ.
(iii) ACO = BCO This circle is circumcircle of  ABC. On
... OC will bisect the C. measuring its radius OB = 2 cm.
470 Arun Deep's Understanding Math-10
8. Using ruler and compasses only, construct Steps of construction :
a ABC such that BC = 5 cm, AB = 6.5 (i) Draw a line segment AB = 4 cm.
cm and ABC = 120º (ii) AT A and B draw rays making an angle of
(i) Construct the circumcircle of ABC. 120° each and cut off AF = BC = 4 cm.
(ii) Construct a cyclic quadrilateral ABCD, such (iii) At F and C, draw rays making angle of 120°
that D is equidistant from AB and BC. each and cut off EF = CD = 4 cm.
Sol. (i) Steps of construction : (iv) Join ED.
(a) Draw a line segment BC = 5 cm. ABCDEF is the required hexagon.
(b) Construct ABC = 120°. (v) Draw perpendicular bisectors of sides AB
(c) Cut BA = 6.5 cm and BC intersecting each other at O.
(d) Join A to C.
(vi) With centre O and radius equal OA or OB
(e) Construct perpendicular bisectors of AB and
draw a circle which passes through the
BC, intersecting at O. Join AO
(f) Taking O as centre, and OA as radius draw a vertices of the hexagon.
circle, passing through A, B and C. This is the required circumcircle of hexagon
(ii) (a) Draw the bisector of ABC such that ABCDEF.
it touches the circle at point D. 10. Draw a regular hexagon of side 4 cm and
(b) Join A to D and C to D. construct its incircle.
(c) ABCD is required cyclic quadrilateral. Sol. Steps of constructions :

9. Construct a regular hexagon of side 4 cm.


Construct a circle circumscribing the
hexagon.
Sol.
(i) Draw a regular hexagon ABCDEF of side 4
cm.
(ii) Draw the angle bisectors of A and B
which intersect each other at O.
(iii) Draw OL  AB.
(iv) With centre O and radius OB, draw a circle
which touches the sides of the hexagon.
This is the required incircle.
471 Arun Deep's Understanding Math-10
CHAPTER TEST (iii) With AP as diameter draw a circle with
centre O.
1. Draw a circle of radius 3 cm. Mark its centre
as C and mark a point P such that (iv) Draw bisector of OQ which intersect OQ
CP = 7 cm. Using ruler and compasses at M.
only, Construct two tangents from P to the (v) With centre M and draw a circle with radius
circle. MQ which intersects the first circle at T
Sol. Steps of Construction : and S.
(i) Draw a circle with centre C and radius (vi) Join QT and QS.
3 cm. QT and QS are the tangents to the first
(ii) Mark a point P such that CP = 7 cm. circle.
(iii) With CP as diameter, draw a circle
intersecting the given circle at T and S.
(iv) Join PT and PS. T
(v) Draw a tangent at Q to the circle given.
Which intersects PT at D.
(vi) Draw the angle bisector of PDQ O 7 cm
A Q
intersecting CP at E. 4 cm P M

(vii) With centre E and radius EQ, draw a circle.


It will touch the tangent T and PS and the
given circle at Q. S
This is the required circle.

3. Using ruler and compasses only, construct


a triangle ABC having given c = 6 cm,
b = 7 cm and A = 30°. Measure side a.
Draw carefully the circumcircle of the
triangle.
Sol. Steps of Construction :
(i) Draw a line segment AC = 7 cm.
(ii) At C, draw a ray CX making an angle of
30º
2. Draw a line AQ = 7 cm. Mark a point P on (iii) With centre A and radius 6 cm draw a arc
AQ such that AP = 4 cm. Using ruler and which intersects the ray CX at B.
compasses only, construct : (iv) Join BA.
(i) a circle with AP as diameter. (v) Draw perpendicular bisectors of AB and
(ii) two tangents to the above circle from the AC intersecting each other at O.
point Q. (vi) With centre O and radius OA or OB or OC,
Sol. Steps of construction : draw a circle which will pass through A, B
(i) Draw a line segment AQ = 7 cm. and C.
(ii) From AQ, cut off AP = 4 cm. This is the required circumcircle of  ABC.
472 Arun Deep's Understanding Math-10
BC = 7 cm, AB = 5 cm and ABC = 45º.
(ii) Draw the inscribed circle to ABC drawn
in part (i).
Sol. Steps of construction :
(i) Draw a line segment BC = 7 cm.
(ii) At B, draw a ray BX making an angle of
45º and cut off BA = 5 cm.
(iii) Join AC.
(iv) Draw the angle bisectors of B and C
intersecting each other at I.
(v) From I, draw a perpendicular ID on BC.
4. Using ruler and compasses only, construct (vi) With centre I and radius ID, draw a circle
an equilateral triangle of height 4 cm and which touches the sides of ABC at D, E
draw its circumcircle. and F respectively.
Sol. Steps of Construction : This is the required inscribed circle.
(i) Draw a line XY and take a point D on it.
X
(ii) At D, draw a perpendicular and cut off
DA = 4 cm. A

cm
(iii) From A, draw rays making an angle of 30°

5
F
on each side of AD meeting the line XY at E
B and C. I
(iv) Now draw perpendicular bisector of AC
intersecting AD at O. 45º D
B C
(v) With centre O and radius OA or OB or OC
7 cm
draw a circle which will pass through A, B
and C. This is the required circumcircle of
 ABC.
6. Draw a triangle ABC, given that BC = 4cm,
C = 75° and that radius of circumcircle
of  ABC is 3 cm.
Sol. Steps of Construction :
(i) Draw a line segment BC = 4 cm
(ii) Draw the perpendicular bisector of BC.
(iii) From B draw an arc of 3 cm radius which
intersects the perpendicular bisector at O.
(iv) Draw a ray CX making an angle of 75°.
(v) With centre O and radius 3 cm draw a circle
which intersects the ray CX at A.
5. Using ruler and compasses only :
(vi) Join AB.
(i) Construct a triangle ABC with the following
data :  ABC is the required triangle.
473 Arun Deep's Understanding Math-10
(b) At B, draw a ray BX making an angle of
90° and cut off BA = 5 cm.
(c) Join AC.
(d) Draw the perpendicular bisector of BC.
(e) From A with 5 cm radius draw arc which
intersects the perpendicular bisector of BC
at P and P.
 There are two points.
(f) Draw the angle bisectors of B and C
intersecting at O.
(g) From O, draw OD  BC.
(h) With centre O and radius OD, draw a circle
which will touch the sides AB and BC. This
7. Draw a regular hexagon of side 3.5 cm
is the required circle.
construct its circumcircle and measure its
radius.
Sol. Steps of construction :
(i) Draw a regular hexagon ABCDEF whose
each side is 3.5 cm.
(ii) Draw the perpendicular bisector of AB and

5cm
BC which intersect each other at O.
(iii) Join OA and OB.
(iv) With centre O and radius OA or OB, draw a

circle which passes through A, B, C, D, E and P.
Then this is the required circumcircle.
E D

(ii) Steps of Construction.


(a) Draw a line segment BC = 6 cm.
(b) At B, draw a ray BX making an angle of
O 90° and cut off BA = 5 cm.
F C
(c) Join AC.
3.5 cm (d) Draw the angle bisectors of B and C
mc

which intersect each other at I.


3.5

(e) From I draw a perpendicular ID on BC.


(f) With centre I and radius ID draw a circle
A 3.5cm B which touches
the sides of A
traingle A, B, C
8. Construct a triangle ABC with the following data : at D, E and F.
AB = 5 cm, BC = 6 cm and ABC = 90°.
(i) Find a point P which is equidistant from B F
and C and is 5 cm from A. How many such
points are there ? E
I
(ii) Construct the inscribed circle of ABC
drawn above. B D C
Sol. Steps of Construction :
(a) Draw a line segment BC = 6 cm.
17
Mensuration
POINTS TO REMEMBER

1. Circumference of a circle = 2r


2. Area of a circle = r2
3. Area of a circular ring =  (R2 – r2).
4. Circumference and area of circumscribed circle of an equilateral triangle.
If a be the side of the triangle and h be the height, then
2 3
Radius (R)  h, h  a
3 2
4
Circumference = 2  R  h
3

2 4 2
Area  R  h
9
5. Circumference and area of inscribed circle of an equilateral triangle where side of triangle = a,
3 1
and h is the height, then h  a , radius r  h
2 3
2 1
Circumference = 2 r  h , Area  r 2  h 2
3 9
6. Circumference and area of circumscribed and inscribed circles of a regular hexagon. Let ‘a’ be
the side of a regular hexagon and R, r be the radii of the circumscribed and inscribed circles
respectively then :
3
R = a and r  a
2
Circumference of circumscribed circle = 2R = 2a.
Circumference of inscribed circle = 2 r = 3 a.
Area of circumscribed circle = R2 = a2.
3 2 22
Area of inscribed circle  r 2  a . Note :   or 3·14.
4 7
7. (i) Curved surface area of a cylinder = 2rh
(ii) Total surface area = 2rh + 2r2 = 2r (h + r)
(iii) Volume = r2h
(iv) Hollow cylinder

474
475 Arun Deep's Understanding Math-10
Let R and r be the radii of outer cylinder and inner cylinder then :
(i) Thickness = R – r
(ii) Area of cross section =  (R2 – r2)
(iii) Outer curved surface area = 2 Rh
(iv) Inner curved surface area = 2 rh
(v) Total surface area
= 2 Rh + 2 rh + 2  (R2 – r2)
= 2  (Rh + rh + R2 – r2)
(vi) Volume of material used
=  R2h – 2  r2h =  h (R2 – r2)
8. (i) Surface area of cone =  rl

where l  r 2  h 2
(ii) Total surface area =  rl +  r2
=  r ( l + r)
1 2
(iii) Volume  r h
3
9. (i) Surface area of a sphere = 4 r2
4 3
(ii) Volume of sphere  r
3

10. Spherical shell (R and r are the radii of outer and inner shell)
(i) Thickness of shell = R – r
4
(ii) Volume   (R3  r 3 )
3

11. Hemisphere :
(i) Surface area (Curved surface) = 2  r2
(ii) Total surface area = 3  r2
2 3
(iii) Volume  r
3
EXERCISE 17.1
22 Sol. Given, radius of the cylinder (r) = 5 cm
Take  = , unless stated otherwise. and Height (h) = 10 cm
7
1. Find the total surface area of a solid cylinder  Total surface area = 2r (h + r)
of radius 5 cm and height 10 cm. Leave your = 2 × 5(10 + 5) cm2
answer in terms of . = 10 × 15 = 150 cm2
476 Arun Deep's Understanding Math-10
2. An electric geyser is cylindrical in shape, 4. In the given figure, a retangular tin foil of
having a diameter of 35 cm and height 1.2m. size 22 cm by 16 cm is wrapped around to
Neglecting the thickness of its walls, calculate form a cylinder of height 16 cm. Find the
(i) its outer lateral surface area, volume of the cylinder.
(ii) its capacity in litres.
Sol. Given diameter of cylindrical geyser = 35 cm
35

16 cm
 Radius (r) of geyser = cm
2
and Height of cylindrical geyser = 1.2 m = 120 cm
22 cm
(i) Outer lateral surface area = 2rh
22 35 Sol. Length of rectangular tin foil (l) = 22 cm
=2× × × 120 cm2 = 13200 cm2
7 2 and breadth (b) = 16 cm
(ii) Capacity = r2h By folding lengthwise,
22 35 35 22
= × × × 120 cm3 = 115500 cm3 radius of the cylinder (r) =
7 2 2 2π
115500 11 7 7
= litres = 115.5 litres = =
1000 22 2
[ 1 l = 1000 cm3] and height (h) = 16 cm
3. A school provides milk to the students daily  Volume of the cylinder so formed = r2h
in cylindrical glasses of diameter 7 cm. If
22 7 7
the glass is filled with milk upto a height of = × × × 16 = 616 cm3
12 cm, find how many litres of milk is needed 7 2 2
to serve 1600 students. 5. (i) How many cubic metres of soil must be
Sol. Given, number of students = 1600 dug out to make a well 20 metres deep and 2
Diameter of cylindrical glasses = 7 cm metres in diameter?
(ii) If the inner curved surface of the well in
7 part (i) above is to be plastered at the rate of
 Radius (r) = cm
2 `50 per m2, find the cost of plastering.
Height of milk filled in it (h) = 12 cm Sol. (i) Given, depth of well (h) = 20 m
 Volume of one glass = r2h and diameter of well = 2 m
22 7 7
= × × × 12 cm3 = 462 cm3 2m
7 2 2
Volume of milk for 1600 students
 Total quantity of milk needed
= 462 × 1600 cm3
20 m

462  1600
= litres
1000
7392
= = 739.2 litres
10
477 Arun Deep's Understanding Math-10

2 Sol. (i) Given, volume of a cylinder = 448 cm3


 Radius = =1m Height (h) = 7 cm
2
 Volume of earth dug out = r2h Volume 448π
 Radius (r) = =
22 πh π7
= × 1 × 1 × 20 m3
7
= 64 = 8 cm
440 6
= = 62 m3 (a) Now lateral surface area = 2rh
7 7
= 2 × 8 × 7 cm2 = 112 cm2
(ii) Inner curved surface area = 2rh
(b) Total surface area = 2r(h + r)
22 880 2 = 2 × 8(7 + 8)
=2× × 1 × 20 = m
7 7 = 16 × 15 = 240 cm2
Given, rate of plastering = `50 per m2 (ii) Given, height of a wooden pole (h) = 7 m
880 and Diameter of pole = 20 cm
 Total cost of plastering = × 50
7
20
44000  Radius of pole (r) = = 10 cm
2
=` = `6285.70
7
10
6. A roadroller (in the shape of a cylinder) has a = m
diameter 0.7 m and its width is 1.2 m. Find 100
the least number of revolutions that the roller  Volume = r2h
must make in order to level a playground of 22 10  10 22 3
size 120 m by 44 m. = × × 7 m3 = m
7 100  100 100
Sol. Given, diameter of a road roller = 0.7 m
Given weight of wood used = 225 kg per m3
0.7
 Radius (r) = = 0.35 m 22
2
 Total weight of pole = × 225 kg
and width (h) = 1.2 m 100
 Curved surface area = 2rh
99
22 = = 49.5 kg
=2× × 0.35 × 1.2 m2 2
7
8. The circumference of the base of a
= 44 × 0.05 × 1.2 = 2.64 m2
Now, area of playground = 120 × 44 cylindrical vessel is 132 cm and its height
= 5280 m2 is 25 cm. Find the (2018)
(i) radius of the cylinder
5280
 Required number of revolutions =
2.64 22
= 2000
(ii) volume of the cylinder (use  = )
7
7. (i) If the volume of a cylinder of height 7 cm
Sol. (i) Given, Circumference = 132 cm
is 448  cm3, find its lateral surface area and
total surface area.
and height = 25 cm.
(ii) A wooden pole is 7 m high and 20 cm in C = 2r
diameter. Find its weight if the wood weighs 22
225 kg per m3.  2 × × r = 132
7
478 Arun Deep's Understanding Math-10
and Curved surface area of cylinder
132  7
 r= = 1000 cm2
2  22
Area 1000
 r = 21 cm (i)  Height (h) = 2 π r =
2  3.14  10
(ii) Volume of the cylinder = r2h
1000  100 5000
22 = = = 15.9 cm
=  21 21 25 = 34650 cm3 Ans. 2  314  10 314
7 (ii) Volume of cylinder = r2h
9. The area of the curved surface of a cylinder = 3.14 × 10 × 10 × 15.9 cm3
is 4400 cm2, and the circumference of its
314  10  10  159 49916
base is 110 cm. Find = =
100  10 10
(i) the height of the cylinder.
= 4992.6 cm 3
(ii) the volume of the cylinder.
11. The barrel of a fountain pen, cylindrical in
Sol. Given area of curved surface of a cylinder
shape, is 7 cm long and 5 mm in diameter. A
= 4400 cm2 full barrel of ink in the pen will be used up
and Circumference of base = 110 cm when writing 310 words on an average. How
many words would use up a bottle of ink
Circumference 110  7
 Radius = = cm containing one-fifth of a litre?
2π 2  22
Answer correct to the nearest 100 words.
35 Sol. Height of cylindrical barrel of a pen (h)
= cm = 7 cm
2
Diameter of barrel = 5 mm
(i)  Now height of cylinder
5 5 1
Curved surface area  Radius (r) = mm = cm = cm
= 2 2  10 4
2π r
 Volume of the ink used in it = r h
2

4400  7  2 22 1 1 11 3
= = 40 cm = ×7× × cm3 = cm
2  22  35 7 4 4 8
(ii) Volume of the cylinder = r2h 1
Given, Ink in the bottle = l = 200 cm3
22 35 35 5
= × × × 40 cm3
7 2 2 Volume = 200 cm3
= 38500 cm3  Number of words written by this ink
10. A cylinder has a diameter of 20 cm. The area 200  8 496000
of curved surface is 1000 cm2. Find = × 310 =
11 11
(i) the height of the cylinder correct to one
= 45090.9 = 45100 words
decimal place.
12. Find the ratio between the total surface area
(ii) the volume of the cylinder correct to one
of a cylinder to its curved surface area given
decimal place. (Take  = 3.14)
that its height and radius are 7.5 cm and
Sol. Given diameter of a cylinder = 20 cm
3.5 cm.
20 Sol. Given radius of a cylinder (r) = 3.5 cm
 Radius (r) = = 10 cm
2
479 Arun Deep's Understanding Math-10
and height (h) = 7.5 cm Total surface area = 1628 cm2
 Total surface area = 2r(r + h) Let r be radius and h be height of cylinder
= 2 × 3.5(3.5 + 7.5) cm2 then r + h = 37
= 7 × 11 = 77 cm2 and 2r(r + h) = 1628
and curved surface area = 2rh 1628
= 2 × 3.5 × 7.5 cm2 2r × 37 = 1628  2r =
37
= 7 7.5 cm2
 Required ratio = 77 : 7 × 7.5 = 11 : 7.5 2  22 1628 1628 7
 r= r= × = 7 cm
7 37 37 44
15
= 11 : = 22 : 15  Radius of cylinder = 7 cm
2
and height of cylinder = 37 – 7 = 30 cm
13. The radius of the base of a right circular
 Volume of cylinder = r2h
cylinder is halved and the height is doubled.
What is the ratio of the volume of the new 22
cylinder to that of the original cylinder? = × 7 × 7 × 30 cm3
7
Sol. Let radius of the base of a right circular = 4620 cm3
cylinder = r
(ii) Total surface area of a cylinder = 352 cm2
and height (h) = h
Height of cylinder = 10 cm
 Volume = r2h
Let radius r of cylinder, then
r
Radius of new cylinder = 2  22
2 2r(h + r) = 352  r(10 + r) = 352
7
and height of new cylinder = 2h
352  7
r
2  r(10 + r) = = 56
 Volume of new cylinder    × 2h 44
2 10r + r2 – 56 = 0
 r2 + 10r – 56 = 0  r2 + 14r – 4r – 56 = 0
r2 π r 2h
=× × 2h =  r(r + 14) – 4(r + 14) = 0
4 2
 (r + 14) (r – 4) = 0
Ratio between the two cylinder (new + old)
Either r + 14 = 0, then r = –14 which is not
π r 2h possible being negative.
= : r2h
2 or r – 4 = 0, then r = 4
 Diameter of cylinder = 2r = 2 × 4 = 8 cm
1
= :1=1:2 15. The ratio between the curved surface and
2 the total surface of a cylinder is 1 : 2. Find
14. (i) The sum of the radius and the height of a the volume of the cylinder, given that its total
cylinder is 37 cm and the total surface area surface area is 616 cm2.
of the cylinder is 1628 cm2. Find the height Sol. Ratio between curved surface area and total
and the volume of the cylinder. surface area = 1 : 2
(ii) The total surface area of a cylinder is 352
Total surface area = 616 cm2
cm2. If its height is 10 cm, then find the
diameter of the base. 616  1
Sol. Sum of radius and height of a cylinder = 37 cm  Curved surface area = = 308 cm2
2
480 Arun Deep's Understanding Math-10
 2rh = 308 = 2r = 30
308 308  7 30 15 15  7 105
 rh = = = 49  Radius = = cm = = cm
2π 2  22 2π π 22 22
and 2r2 = 308 and height = 18 cm

308  7 22 105 105


 r2 = = 49 = (7)2  Volume = r12h1 = × × × 18
2  22 7 22 22
 r = 7 cm 15  105  9
= cm3
49 11
and h = = 7 cm and by rolling breadthwise circumference
7
 Volume of cylinder = r2h = 18 cm
18 18  7 63 3
22  Radius (r2) = = = cm
= × 7 × 7 × 7 = 1078 cm3 2π 2  22 22
7
and h2 = 30 cm
16. Two cylindrical jars contain the same amount
Volume = r22h
of milk. If their diameters are in the ratio
3 : 4, find the ratio of their heights. 22 63 63
= × × × 30
Sol. Given volume of two cylinders is same 7 22 22
Diameter of both cylinder are in the ratio
9  63  15
=3:4 = cm3
11
3x
Let radius of the first = 15  105  9 9  63  15
2  Required ratio = :
11 11
4x = 105 : 63
and second = = 2x
2 = 15 : 9 i.e. 5 : 3
Let h1 and h2 be the height of the two 18. A cylindrical tube open at both ends is made
Then, r2h1 = r2h2 of metal. The internal diameter of the tube is
11.2 cm and its length is 21 cm. The metal
3x 3x thickness is 0.4 cm. Calculate the volume of
× × × h1 =  × 2x × 2x × h2
2 2 the metal.
Sol. Given,
h1 π  2x  2x  2  2 16
 h = = Internal diameter of a metal tube = 11.2 cm
2 π  3 x  3 x 9
11.2
 Ratio in their heights = 16 : 9 and internal radius of tube (r) =
2
17. A rectangular sheet of tin foil of size
30 cm × 18 cm can be rolled to form a = 5.6 cm
cylinder in two ways along length and along Length of tube (h) = 21 cm
breadth. Find the ratio of volumes of the two and Thickness of metal = 0.4 cm
cylinders thus formed.  External radius of tube (R) = 5.6 + 0.4
Sol. Given size of the sheet be = 30 cm × 18 cm = 6.0 cm
(i) By rolling lengthwise,  Required volume of metal used
The circumference of the cylinder
481 Arun Deep's Understanding Math-10

22 44
= h(R2 – r2) = × 21[62 – 5.62] = 2032.8 + (4.84 – 4)
7 7
= 66(6 + 5.6) (6 – 5.6) cm3 44
= 2032.8 + × 0.84
= 66 × 11.6 × 0.4 cm3 = 306.24 cm3 7
P.Q. The given figure shows a metal pipe 77 cm = 2032.8 + 5.28 = 2038.08 cm2
long. The inner diameter of a cross-section 19. A lead pencil consists of a cylinder of wood
is 4 cm and the outer one is 4.4 cm. Find its with a solid cylinder of graphite filled in the
interior. The diameter of the pencil is 7 mm
(i) inner curved surface area and the diameter of the graphite is 1 mm. If
(ii) outer curved surface area the length of the pencil is 14 cm, find the
(iii) total surface area. volume of the wood and that of the graphite.
Sol. Given diameter of the pencil = 7 mm
7 7
 Radius (R) = mm = cm
2 20
[ 1 cm = 10 mm]
Diameter of graphite (lead) = 1 mm
1 1
 Radius (r) = = cm
2 20
Length of pencil (h) = 14 cm
Sol. In the given figure,  Volume of graphite = r2h
Length of metal pipe (h) = 77 cm 22 1 1 11
= × × × 14 = cm3
Inner diameter = 4 cm 7 20 20 100
4 = 0.11 cm3
 Radius (r) = = 2 cm and volume of wood = R2h – r2h
2
22  7  2  1  2 
4.4      
and outer radius (R) = = 2.2 cm = h[R2 – r 2] = × 14
2 7  20   20  
(i) Inner curved area = 2rh
 49 1  48
= 44    = 44 × cm3
22  400 400  400
=2× × 2 × 77 cm2 = 968 cm3
7
528
(ii) Outer curved area = 2Rh = = 5.28 cm3
100
22 P.Q. A soft drink is available in two packs—
=2× × 2.2 × 77 cm3 = 1064.8 cm2
7 (i) a tin can with a rectangular base of length
(iii) Total surface area = Inner area + Outer area 5 cm and width 4 cm, having a height of
+ Area of two rings 15 cm and
= 2rh + 2Rh + 2R2 – r2) (ii) a plastic cylinder with circular base of
diameter 7 cm and height 10 cm. Which
22 container has greater capacity and by how
= 968 + 1064.8 + 2 × (2.22 – 22)
7 much?
482 Arun Deep's Understanding Math-10
Sol. (i) Base of the tin of rectangular base  Total weight = 184800 × 8 gm
= 5 cm × 4 cm = 1478400 gm = 1478.4 kg
Height of rectangular base = 15 cm
1
Volume of rectangular base = lbh [ 1 gm = kg]
1000
= 5 × 4 × 15 = 300 cm3
(ii) Base diameter of cylindrical plastic cylinder EXERCISE 17.2
= 7 cm 22
Take  = , unless stated otherwise.
7 7
 Radius of rectangular can (r) = cm
2 1. Find the curved surface area of a right circular
and height (h) = 10 cm cone whose slant height is 10 cm and base
radius is 7 cm.
22 7 7
 Volume = r2h = × × × 10 = 385 cm3 Sol. Slant height of a cone (l) = 10 cm
7 2 2
and radius of the base = 7 cm
It is clear that cylindrical container has greater
 Curved surface area = rl
capacity and by 385 – 300 = 85 cm3 more.
20. A cylindrical roller made of iron is 2 m long. 22
Its inner diameter is 35 cm and the thickness = × 7 × 10 = 220 cm2
7
is 7 cm all round. Find the weight of the 2. Diameter of the base of a cone is 10.5 cm
roller in kg, if 1 cm3 of iron weighs 8 g. and slant height is 10 cm. Find its curved
Sol. Given length of cylindrical roller (h) = 2 m surface area.
= 200 cm Sol. The diameter of the base of a cone = 10.5cm
Diameter of cylindrical roller = 35 cm
10.5
35  Its radius (r) = = 5.25 cm
 Inner radius = cm 2
2
and slant height (l) = 10 cm
and Thickness of cylindrical roller = 7 cm  Curved surface area = rl
35 22
 Outer radius of cylindrical roller = +7= = × 5.25 × 10 cm2 = 165.0 cm2
2 7
49 3. Curved surface area of a cone is 308 cm2
cm
2 and its slant height is 14 cm. Find
and inner height = 200 cm (i) radius of the base
 Volume of the iron in roller (ii) total surface area of the cone.
Sol. Curved surface area of a cone = 308 cm2
22  49  2  35  2 
Slant height = 14 cm
= h[R2 – r2] = × 200  2    2  
7      Curved surface area
(i) Radius (r) =
πl
4400  84 14  4400
= 2  2 = × 42 × 7 cm3 308  7
7   7 = cm = 7 cm
22  14
= 184800 cm3
(ii) Total surface area = Curved surface area +
Weight of 1 cm3 = 8 gm base area
483 Arun Deep's Understanding Math-10
4. Find the volume of the right circular cone with
22
= 308 + r2 = 308 + × 7 × 7 cm2 (i) radius 6 cm and height 7 cm
7
(ii) radius 3.5 cm and height 12 cm.
= 308 + 154 = 462 cm2
Sol. (i) Given radius of cone (r) = 6 cm
P.Q. Write whether the following statements are
and height (h) = 7 cm
true or false. Justify your answer.
(i) If the radius of a right circular cone is halved 1 2
 Volume = r h
and its height is doubled, the volume will 3
remain unchanged.
(ii) A cylinder and a right circular cone are having 1 22
= × × 6 × 6 × 7 cm3 = 264 cm3
the same base radius and same height. The 3 7
volume of the cylinder is three times the (ii) Given radius of a cone (r) = 3.5 cm
volume of the cone. and height of cone (h) = 12 cm
(iii) In a right circular cone, height, radius and
1 2
slant height are always the sides of a right  Volume = r h
triangle. 3
Sol. (i) If the radius of a right circular cone is 1 22
halved and its height is doubled, then the = × × 3.5 × 3.5 × 12 cm3 = 154 cm3
3 7
volume will remain unchanged — It is wrong
5. Find the capacity in litres of a conical vessel
πr 2 h with
as volume of old cone =
3 (i) radius 7 cm, slant height 25 cm
2
(ii) height 12 cm, slant height 13 cm
1 r Sol. (i) Radius = 7 cm
and volume of new cone =    × 2h
3 2 and slant height (l) = 25 cm
1 r2 1 1 2   h= l2  r2 = 25 2  7 2
  × 2h = ×  r h
3 4 2 3 
1
= 625  49 = 576 = 24 cm
= Volume of old cone
2 1 2
 Volume of conical vessel = r h
(ii) A cylinder and a right circular cone are having 3
the same base radius and same height the
volume of the cylinder is three times the 1 22
= × × 7 × 7 × 24 cm3 = 1232 cm3
volume of cone — It is true as 3 7
1 2 1232
Volume of cylinder = r2h = 3 × r h  Capacity of conical vessel = l = 1.232 l
3 1000
= 3(volume of cone) (ii) Height (h) of conical vessel = 12 cm
(iii) In a right circular cone, height, radius and and slant height of conical vessel = 13 cm
slant height are always the sides of a right
triangle — It is true as in a cone and in a  Radius = l 2  h2 = 13 2  12 2
right angled triangle.
Hypotenuse (slant × height) = r2 + h2 and = 169  144 = 25 = 5 cm
cone is formed by revolving the right triangle
1 2
about the perpendicular.  Volume of conical vessel = r h
3
484 Arun Deep's Understanding Math-10

1 22 2200 9. The slant height and base diameter of a


= × × 5 × 5 × 12 = cm3 conical tomb are 25 m and 14 m respectively.
3 7 7
Find the cost of white washing its curved
2200 11 surface area at the rate of `210 per 100 m2.
= litre = litres
7  1000 35 Sol. Slant height of conical tomb (l) = 25 m
6. A conical pit of top diameter 3.5 m is 12 m and base diameter = 14 m
deep. What is its capacity in kilolitres? 14
Sol. Given diameter of top of conical pit = 3.5 m  Radius (r) = =7m
2
3 .5  Curved surface area = rl
 Radius (r) = = 1.75 m
2 22
and depth (h) = 12 m = × 7 × 25 = 550 m2
7
1 2 Given rate of white washing = `210 per 100 m2
Capacity of conical pit = r h
3 550  210
 Total cost of white washing =
1 22 100
= × × 1.75 × 1.75 × 12 m = 38.5 m3 = `1155
3 7
= 38.5 kilolitres 10. A conical tent is 10 m high and the radius of
its base is 24 m. Find :
7. If the volume of a right circular cone of height
9 cm is 48 cm3, find the diameter of its (i) slant height of the tent.
base. (ii) cost of the canvas required to make the tent,
Sol. Volume of a right circular cone = 48 cm3 if the cost of 1 m2 canvas is `70.
Height (h) = 9 cm Sol. Given height of a conical tent (h) = 10 m
and radius (r) = 24 m
Volume 48π  3
 Radius (r) = = (i)  Slant height (l) = r 2  h2
1
πh 1 π  9
3 = 24 2  10 2 = 576  100
= 16 = 4 cm = 676 = 26 m
 Diameter of cone = 2 × r = 2 × 4 = 8 cm (ii) Curved surface area = rl
8. The height of a cone is 15 cm. If its volume 22 13728 2
is 1570 cm3, find the radius of the base. = × 24 × 26 m2 = m
7 7
(Use  = 3.14)
Given cost of 1 m2 canvas used = `70
Sol. Height of cone (h) = 15 cm
 Total cost of canvas required
Volume of cone = 1570 cm3
13728
=` × 70 = `137280
Volume 1570  3 7
 Radius of cone = =
1
πh 3.14  15 11. A Jocker's cap is in the form of a right
3 circular cone of base radius 7 cm and height
24 cm. Find the area of the cloth required to
= 100 = 10 cm make 10 such caps.
485 Arun Deep's Understanding Math-10
Sol. Base radius of a conical cap = 7 cm 1 2
and height (h) = 24 cm  Volume of first cone = r h
3
 Slant height (l) = r 2  h2 1 1
= (2y)2 × 5x = 20xy2
3 3
= 7 2  24 2 = 49  576
and height of the second cone = 2x
= 625 cm = 25 cm and radius of second cone = 5y
 Area of cloth used to make it = rl 1
 Volume of second cone = (5y)2 × 2x
22 3
= × 7 × 25 cm2 = 550 cm2
7 1
=  × 50xy
Thus, required cloth used to make 10 such 3
caps = 550 × 10 = 5500 cm2
1 1
12. (a) The ratio of the base radii of two right  Ratio in their volumes = 20xy : 50xy
circular cones of the same height is 3 : 4. 3 3
Find the ratio of their volumes. =2:5
(b) The ratio of the heights of two right circular (c) Let the height of bigger cone = h
cones is 5 : 2 and that of their base radii is and radius of bigger cone = r
2 : 5. Find the ratio of their volumes.
1 2
(c) The height and the radius of the base of a  Volume of bigger cone = r h
3
right circular cone is half the corresponding
height and radius of another bigger cone. h
Find : and height of the smaller cone =
2
(i) the ratio of their volumes.
r
(ii) the ratio of their lateral surface areas. and radius =
2
Sol. (i) The ratio in base radii of two right circular
cones of the same height = 3 : 4 2
1 r  h
Let h be the height and radius of first cone  Volume of smaller cone =     
3 2  2
= 3x and
Radius of second cone = 4x 1 r2 h  2
 Volume of first cone = r2h = × × = r h
3 4 2 24
=  × 3x × 3x × h = 9hx2
 2 1 2
and volume of second cone and ratio in their values : r h : r h
24 3
=  × 4x × 4x × h
= 16hx2 1
= :1=1:8
 Ratio in their volumes = 9hx2 : 16hx2 8
= 9 : 16 13. Find what length of canvas 2 m in width is
(b) The ratio of the heights of two cones = 5 : 2 required to make a conical tent 20 m in
and ratio in their radii = 2 : 5 diameter and 42 m in slant height allowing
10% for folds and the stitching. Also find
Let height of first cone = 5x
the cost of the canvas at the rate of `80 per
and radius of first cone = 2y metre.
486 Arun Deep's Understanding Math-10
Sol. Diameter of the base of the conical tent = 20 m (i) the slant height of the cone.
20 (ii) total surface area of the cone.
 Radius (r) = = 10 m Sol. Volume of a circular cone = 9856 cm3
2
Given area of the base = 616 cm2
and slant height (h) = 42 m
 Area of canvas used = rl Area of base
 Radius of cone (r) =
22 π
= × 10 × 42 m2 = 1320 m2
7
616  7
Width of cloth = 2 m = cm = 28 7 cm
22
1320
 Length of canvas cloth = = 660 m
2 = 196 = (14) 2 = 14 cm
Cloth used in stiching and folding
= 10% of 660 m 1 2
Now, volume of cone = r h
3
660  16
= = 66 m = 9856 cm3 (given)
100
 Total length of cloth required = 660 m + 66 m 1 22
 × × 14 × 14h = 9856 cm3
= 726 m 3 7
Given rate of cloth = `80 per metre
9856  3  7
 Total cost of canvas = `726 × 80 = `58080  h= = 48 cm
22  14  14
14. The perimeter of the base of a cone is 44 cm
and the slant height is 25 cm. Find the volume (i)  Slant height (l) = r 2  h2
and the curved surface of the cone.
Sol. Perimeter of the base of a cone = 44 cm = P = 14 2  48 2
P 44  7
 Radius of cone (r) = = = 7 cm = 196  2304 = 2500 = 50 cm
2π 2  22
(ii) Total surface area of cone = rl + r2
and slant height (l) = 25 cm
= r(l + r)
and height (h) = l2  r2 = 25 2  7 2 22
= × 14(50 + 14) cm2
= 625  49 = 576 = 24 cm 7
= 44(64) = 2816 cm2
1
(i)  Volume of cone = r2h 16. A right triangle with sides 6 cm, 8 cm and
3 10 cm is revolved about the side 8 cm. Find
1 22 the volume and the curved surface of the
= × × 7 × 7 × 24 cm2 = 1232 cm3 cone so formed. (Take  = 3.14)
3 7
(ii) and curved surface area = rl Sol. Side of a right triangle are 6 cm and 8 cm
It is revolved around 8 cm side then it
22 generates a cone.
= × 7 × 25 cm2 = 550 cm2
7  Radius of cone (r) = 6 cm
15. The volume of a right circular cone is Height of cone (h) = 8 cm
9856 cm 3 and the area of its base is Slant height of cone (l) = 10 cm
616 cm2. Find
487 Arun Deep's Understanding Math-10
1 2
Volume of given cone = R H
3
1 2
and volume of small cone = r h
8 cm 3

10
cm
1 2 1 1
 r h = × R2H
3 27 3

r2 1 r2 10
 2
h = × 30  2
h = ...(i)
6 cm R 27 R 9

r h h r h
1 But = =  = ...(ii)
(i) Volume of cone = r2h R H 30 R 30
3
From (i) and (ii) ; we have
1 2
= (3.14) × 6 × 6 × 8 cm3 = 301.44 cm3  h  10 h2  h 10
3   h=  =
 30  9 900 9
(ii) Curved surface area of cone = rl
= 3.14 × 6 × 10 cm2 = 188.4 cm2 10  900
h3 = = 1000 = (10)3
17. The height of a cone is 30 cm. A small cone 9
is cut off at the top by a plane parallel to its  h = 10 cm
1  A line parallel to base at a distance of
base. If its volume be of the volume of 30 – 10 = 20 cm is drawn.
27
the given cone, at what height above the base 18. A semi-circular lamina of radius 35 cm is
is the section cut? folded so that the two bounding radii are
joined together to form a cone. Find :
Sol. Height of a cone (H) = 30 cm
(i) the radius of the cone.
A small cone of radius r and height is cut off (ii) the (lateral) surface area of the cone.
from the top of the cone given
Sol. Radius of a semi-circular lamina = 35 cm
By folding it a cone is formed whose slant
height (l) = r = 35
h and half circumference = circumference of
the top of the cone
r
30 cm

H 22
= r = × 35 cm = 110 cm
7

1 r1
 Volume of small cone = of the Volume
27
A O 35 cm B
of the given cone
488 Arun Deep's Understanding Math-10

110 22
(i)  New radii of the cone = = 4 × 7 × 10.5 × 10.5 × 10.5

110  7 = 1386 cm2


=
2  22 4 3
and volume of sphere = r
3
35
= cm = 17.5 cm
2 4 22
(ii) Lateral surface area = rl = 3  7  10.5  10.5 10.5

22 35 = 4851 cm3
= × × 35 = 1925 cm2
7 2 3. A shot-put is a metallic sphere of radius
4.9 cm. If the density of the metal is 7.8 g
EXERCISE 17.3 per cm3, find the mass of the shot-put.
Sol. Given, radius of the metallic shot-put = 4.9 cm
22
Take  = , unless stated otherwise. 4 3
7
 Volume of shot-put = r
1. Find the surface area and volume of a 3
sphere of radius 14 cm. 4 22
= × × 4.9 × 4.9 × 4.9 cm3
Sol. Radius of sphere (r) = 14 cm 3 7
Surface area of sphere = 4r2 1479.016
= = 493 cm3
22 3
=4× × 14 × 14 cm2 Also given, density of metal = 7.8 g per cm3
7
Total mass of shot-put = 493 × 7.8 g
= 2964 cm2
= 3845.4 g
4 3 = 3.845 kg = 3.85 kg (approx)
and volume of sphere = r
3 P.Q. Find the surface area of a sphere of diameter:
21 cm
4 22
= × × 14 2 × 14 × 14 Sol. Diameter of sphere = 21 cm
3 7
21
34496  Radius of sphere (r) = cm
= cm3 2
3
 Surface area of sphere = 4r2
Diameter of sphere (d) = 21 cm
22 21 21 2
2. Find the surface area and volume of a =4× × × cm = 1386 cm2
7 2 2
sphere of diameter 21 cm.
4. Find the diameter of a sphere whose surface
21 area is 154 cm2.
Sol. Radius of sphere (r) = cm = 10.5 cm Sol. Surface area of a sphere = 154 cm2
2
Surface area of sphere = 4r2 Surface area
 Radius of sphere (r) =

489 Arun Deep's Understanding Math-10
7. The radius of a spherical balloon increases
154  7 from 7 cm to 14 cm as air is pumped into it.
= cm
4  22 Find the ratio of the surface areas of the
balloon in two cases.
49 7 Sol. Original radius of balloon = 7 cm
= = cm
4 2 Radius after filling air in it = 14 cm
Surface area of balloon, the original position
7
 Diameter of sphere = 2r = 2 × cm = 7 cm = 4r2
2
5. Find : 22
=4× × 7 × 7 = 616 cm2
(i) the curved surface area. 7
(ii) the total surface area of a hemisphere of Surface area after filling air = 4r2
radius 21 cm. 22
Sol. Given radius of a hemisphere = 21 cm =4× × 14 × 14 cm2 = 2464 cm2
7
(i)  Curved surface area of hemisphere = 2r2 Ratio between the two position
22 = 616 : 2464 = 1 : 4
=2× × 21 × 21 cm2 = 2772 cm2 8. A sphere and a cube have the same surface.
7
Show that the ratio of the volume of the
(ii) Total surface area of hemisphere = 3r2
sphere to that of the cube is 6 : π.
22
=3× × 21 × 21 cm2 = 4158 cm2 Sol. Let the edge of a cube = a
7
 Surface area = 6a2
6. A hemispherical brass bowl has inner-diameter and surface area of sphere = 6a2
10.5 cm. Find the cost of tin-plating it on the
inside at the rate of `16 per 100 cm2. 6a 2 3
Sol. Inner diameter of hemispherical bowl  Radius (r) = = a
4π 2π
= 10.5 cm and volume of cube = a3
10.5 4 3
Radius (r) = = 5.25 cm and volume of sphere = r
2 3
 Inner curved surface area = 2r2
3
22 10.5 10.5 4  3 
=2× × × cm2 =  a
7 2 2 3  2π 

693 2 4 3 3a 3 2 3a 3
= cm = 173.25 cm2 = × =
4 3 2 2π  π 2 π
Rate of tin-plating = `16 per 100 cm2
2 3a 3
693  16  Required ratio = : a3
 Total cost of tin-plating = ` 2 π
4  100
= 2 3 : π = 6 : π
2772
=` = `27.72 9. (a) If the ratio of the radii of two sphere is
100
3 : 7, find :
490 Arun Deep's Understanding Math-10
(i) the ratio of their volumes.
(5) 2 25
(ii) the ratio of their surface areas. = 2 =
( 4) 16
(b) If the ratio of the volumes of the two sphere
is 125 : 64, find the ratio of their surface  Ratio of their surface area be 25 : 16.
areas. P.Q. A cube of side 4 cm contains a sphere
Sol. (a) Ratio in radii of two spheres = 3 : 7 touching its sides. Find the volume of the
Let radius of the first sphere = 3x gap in between.
and radius of the second sphere = 7x Sol. Given side of a cube = 4 cm
Volume of cube = (side)3 = 4 × 4 × 4
4
(i)  Volume of the first sphere = (3x)3 = 64 cm3
3
Diameter of sphere contained by this cube is
4 d = 4 cm
and of second sphere = (7x)3
3 4
 Radius (r) = = 2 cm
4 4 2
 Required ratio = 27x3 : 343x3
3 3 4 3
 Volume of sphere = r
= 27 : 343 3
(ii) Surface area of the first sphere = 4r2 4 32
= 4(3x)2 = ×2×2×2=  cm3
3 3
and of second sphere = 4(7x)2
32 22 3 704 3
 Ratio =4(3x)2 : 4(7x)2 = × cm = cm
3 7 21
= 9x2 : 49x2 = 9 : 49
 Volume of space between them
(b) Let the volume of sphere be V1 and V2
According to question, we have 704
= 64 –
21
V1 125
 V = 1344  704 640 3
2 64 = = cm
21 21
4 3 = 30.48 cm3 (approx)
πr1
3 125
 = 10. Find the volume of a sphere whose surface
4 3 64 area is 154 cm2.
πr2
3
Sol. Given surface area of a sphere = 154 cm2
3 Also surface area of sphere = 4r2
πr13 125  r1  5
3

 =    =  
πr23 64  r2  4 154 154  7
 Radius (r) = = cm
 r1 = 5 and r2 = 4 4π 4  22
Now, total surface area = 4r2
According to question, we have 49 7
= = cm
4 2
4πr12 ( r1 ) 2
Required ratio = = 4 3
4πr22 (r2 ) 2 Now volume of sphere = r
3
491 Arun Deep's Understanding Math-10

4 22 7 7 7 539 3 5
= × × × × cm3 = cm = 89 cm3
3 7 2 2 2 6 6

539 2 P.Q. The water for a factory is stored in a


= = 179 cm3 hemispherical tank whose internal diameter
3 3
is 14 m. The tank contains 50 kilolitres of
2 water. Water is pumped into the tank to fill
11. If the volume of a sphere is 179 cm3, find to its capacity. Find the volume of water
3
pumped into the tank.
its radius and the surface area.
Sol. Internal diameter of a hemispherical tank (r)
2 = 14 m
Sol. Given volume of a sphere = 179 cm3
3
14
539 Radius of the tank = =7m
2
= cm3
3 Water stored in it = 50 kilolitres of water
4 3 2 3 2 22
Also volume of sphere = πr  Volume = r = × × 7 × 7 × 7 m3
3 3 3 7
1 = 718.66 m3 or 718.66 kilolitre water
 539 3  3  More water to be pumped in
 Radius =   
 3 4π 
= 718.66 – 50 = 668.66 kilolitres
1 1 13. The surface area of a solid sphere is
 539  3  7  3  343  3 1256 cm2. It is cut into two hemispheres.
=   =  
 3  4  22   8  Find the total surface area and the volume of
a hemisphere. Take  = 3.14.
1 Sol. Surface area of a solid sphere = 1256 cm2
 7  3  3 7
=    = cm = 3.5 cm By cutting it into two hemisphere,
 2   2 Curved surface area of each hemisphere
 Surface area of sphere = 4r2 1256
= = 628 cm2
22 7 7 2
=4× × × cm2 = 154 cm2
7 2 2
Surface area
12. A hemispherical bowl has a radius of 3.5 cm. Now radius of sphere =

What would be the volume of water it would
contain?
1256 314  100
Sol. Radius of a hemispherical bowl (r) = 3.5 cm = cm =
4  3.14 314
7
= cm =
2 100 = 10 cm
2 3 Now total surface area of hemisphere = 3r2
 Volume of water in it = r = 3 × 3.14 × 10 × 10 cm2 = 942 cm2
3
2 22 7 7 7 2 3
= × × × × cm3 and volume of hemisphere = r
3 7 2 2 2 3
492 Arun Deep's Understanding Math-10

2 1 3 2 3
= (3014) × (10 × 10 × 10) cm3 = r : r : r3 = 1 : 2 : 3
3 3 3
 It is true.
2 6280 1
= × 3140 = cm3 = 2093 cm3
3 3 3 EXERCISE 17.4
P.Q. Write whether the following statements are
true or false. Justify your answer : 22
Take  = , unless stated otherwise.
(i) The volume of a sphere is equal to two-third 7
of the volume of a cylinder whose height
1. The adjoining figure shows a cuboidal block
and diameter are equal to the diameter of the
of wood through which a circular
sphere.
cylinderical hole of the biggest size is drilled.
(ii) The volume of the largest right circular cone Find the volume of the wood left in the block.
that can be fitted in a cube whose edge is 2r
equals the volume of a hemisphere of radius r.
(iii) A cone, a hemisphere and a cylinder stand
on equal bases and have the same height.
The ratio of their volumes is 1 : 2 : 3.
Sol. (i) The volume of a sphere is equal to the
two third of the volume of a cylinder whose
height and diameter are equal to the diameter
of the sphere.

4 3 2 Sol. Diameter of the biggest hole = 30 cm.


True, as volume of sphere = r = r3)
3 3 30
 Radius (r)   15 cm
and volume of cylinder = r2 (2r) = 2r3 2
and height (h) = 70 cm.
(ii) The volume of the longest right circular cone
that can be filled in a cube whose edge is 2r  Volume of the hole made = r2h
equal to the volume of a hemisphere of
22
radius r is True.  × 15 × 15 × 70 cm3 = 49500 cm3
7
Since volume of cone where height 2r
Total volume of the log = 70 × 30 × 30 cm3
1 2 2
= r h = r3 = Volume of hemisphere = 63000 cm3
3 3
 Volume of the wood left
(iii) A cone, a hemisphere and a cylinder stand
on equal bases and have the same height. = (63000 – 49500) cm3
The ratio of their volumes is 1 : 2 : 3 = 13500 cm3
Clearly ratio of these volumes 2. The given figure shows a solid trophy made
of shining glass. If one cubic centimetre of
1 2 2
= r × r : r3 : r2.r glass costs `0·75, find the cost of the glass
3 3
for making the trophy.
493 Arun Deep's Understanding Math-10

14
 Radius (r) = = 7 cm
28 cm 2
and height = 14 cm
1 2
28 cm  Volume of cone = r h
3

1 22
= × × 7 × 7 × 14
28 cm 3 7
28 cm 2156 2
= = 718 cm3
3 3
Sol. Given edge of cubical part = 28 cm
 Volume of the remaining portion
and radius of cylinderical part (r)
28 2 1
=  14 cm = 2744 – 718 = 2025 cm3
2 3 3
Height of cylinderical part (h) = 28 cm 4. A cone of maximum volume is carved out of
 Total volume of the trophy = (Edge)3 + r2h a block of wood of size 20 cm × 10 cm ×
10 cm. Find the volume of the remaining
22 wood.
= (28)3 + × 14 × 14 × 28 cm3
7 Sol. Size of wooden block
= (21952 + 17248) cm3 = 39200 cm3 = 20 cm × 10 cm × 10 cm
Given rate of cost of glass = `0·75 per cm3  Maximum diameter of the cone = 10 cm
 Required total cost = `39200 × 0·75 and height (h) = 20 cm
= `29400
3. From a cube of edge 14 cm, a cone of 10
 Radius (r) = = 5 cm
maximum size is carved out. Find the 2
volume of the remaining material. Now volume of block = 20 × 10 × 10
Sol. Given edge of a cube = 14 cm = 2000 cm3
 Volume = (side)3 = (14)3
1 2
= 14 × 14 × 14 cm3 = 2744 cm3 and volume of cone = r h
3
Now diameter of the cone cut out from it
= 14 cm 1 22 11000 3
= × × 5 × 5 × 20 cm3 = cm
3 7 21
 Volume of the remaining portion
14cm

11000
= 2000 –
21

h 42000  11000 31000


= = = 1476.19 cm3
21 21

2
= 1476 cm3 (approx.)
14cm 3
494 Arun Deep's Understanding Math-10
5. 16 glass spheres each of radius 2 cm are and depth (h) = 1.4 cm
packed in a cubiodal box of internal  Volume of cuboid = l × b × h
dimensions 16 cm × 8 cm × 8 cm and then = 15 × 10 × 3.5 cm3 = 525 cm2
the box is filled with water. Find the volume
and volume 4 conical depressions
of the water filled in the box.
Sol. Radius of each glass sphere = 2 cm 1 2 4 22
=4× r h = × × (0.5)2 × 1.4 cm2
3 3 7
4 3
 Volume of each glass sphere = r
3 88
= × 0.25 × 1.4 cm3
4 22 21
= × × 2 × 2 × 2 cm3
3 7 30.8
= cm2 = 1.467 cm3
704 21
= cm3
21  Volume of wood in the stand
704 = 525 – 1.467 = 523.533 cm3
Volume of 16 glass spheres = × 16 cm3 6. A cuboidal block of side 7 cm is surmounted
21
by a hemisphere. What is the greatest
11264 3 diameter that the hemisphere can have? Also
= cm = 536.38 cm3
21 find the surface area of the solid.
Volume of box = 16 × 8 × 8 = 1024 cm3 Sol. Given side of cuboidal block = 7 cm
 Volume of remaining space for water Diameter of hemisphere = 7 cm
= 1024 – 536.38 = 487.62 cm3
7
P.Q. A pen stand made of wood is in the shape of and radius (r) = cm
2
a cuboid with four conical depressions to
hold pens. The dimensions of the cuboid are
15 cm by 10 cm by 3.5 cm. The radius of
each of the depression is 0.5 cm and the
depth is 1.4 cm. Find the volume of the wood
in the entire stand, correct to 2 decimal places.

 Surface area of the total solid = 5a2 + 2r2


22 7 7
= 5(7)2 + 2 × × × cm2
7 2 2
= 245 + 77 = 322 cm2
7. A wooden article was made by scooping out
Sol. Dimensions of cuboid = 15 cm × 10 cm ×
a hemisphere from each end of a solid
3.5 cm
cylinder (as shown in the given figure). If
and radius of each conical depression (r)
the height of the cylinder is 10 cm and its
= 0.5 cm
495 Arun Deep's Understanding Math-10
base is of radius 3.5 cm, find the total surface Give your answer correct to the nearest
area of the article.
22
whole number. Take   .
7
Sol. Height of cylinder = 7 cm
Radius (hemisphere, cone and cylinder)
= 3 cm
22
Height of cone = 3 cm  
7
3 cm

Sol. Given height of the cylinder = 10 cm


7 cm
and radius of the base = 3.5 cm
Total surface area of the article
= Curved surface area of cylinder + 2 ×
Curved surface area of hemisphere
= 2rh + 2 × 2r2 3 cm

22 22
=2× × 3.5 × 10 + 2 × 2 × × 3.5
7 7
Volume of remaining solid = Volume of
× 3.5 cm2
cylinder – Volume of cone
= 220 + 154 = 374 cm2
– Volume of hemisphere
8. A hemispherical and conical hole are
scooped out of a solid wooden cylinder. (i) Volume of cylinder = r 2 h   32  7
 .63
1 2
(ii) Volume of cone = r h
3
1
  32  3  9. 
3
2
(iii) Volume of hemisphere = .  (3)3  18. 
3
 Required volume =  (63 – 9 – 18)
=  (36)
22
Find the volume of the remaining solid   36  113.14
7
where the measurements are as follows:
 Volume of remaining = 113 cm3
The height of the solid cylinder is 7 cm,
9. A toy is in the form of a cone of radius 3.5
radius of each of hemisphere, cone and
cm mounted on a hemisphere of same radius.
cylinder is 3 cm. Height of cone is 3 cm.
496 Arun Deep's Understanding Math-10
If the total height of the toy is 15.5 cm, find
the total surface area of the toy.
Sol. Total height of the toy = 15.5 cm
Radius of the base of conical part (r) = 3.5cm

15.5cm  Slant height (l)  r 2  h22


3.5cm
 ( 12 ) 2  ( 5) 2
= 144  25  169 = 13 m
Now surface area of tent = rl + 2 rh
 Height of conical part (h) = 15.5 – 3.5 =  r (l + 2 h)
= 12 cm 22
 × 12 (13  2 × 11) m2
 Slant height (l) = 7
r 2  h2
22
 × 12 × 35 m2 = 1320 m2
= (3.5) 2  12 2 = 12.25  144 7
11. An exhibition tent is in the form of a cylinder
= 156.25 cm = 12.5 cm surmounted by a cone. The height of the tent
Total surface area of the toy = rl + 2r2 above the ground is 85 m and the height of
= r(l + 2r) the cylindrical part is 50 m. If the diameter
of the base is 168 m, find the quantity of
22 canvas required to make the tent. Allow 20%
= × 3.5(12.5 + 2 × 3.5) cm2 extra for folds and stitching. Give your
7
answer to the nearest m2.
= 11(12.5 + 7) cm2 = 11 × 19.5 cm2
Sol. Total height of the tent = 85 m
= 214.5 cm2 Height of cylindrical part (h1) = 50 m
10. A circus tent is in the shape of a cylinder
surmounted by a cone. The diameter of the
cylindrical portion is 24 m and its height is
11 m. If the vertex of the cone is 16 m above
the ground, find the area of the canvas used
to make the tent.
Sol. Radius of base of cylindrical portion of tent
24
(r)   12 m
2
 Height of conical part (h2) = 85 – 50 = 35 m
Height of cylindrical portion (h1) = 11 m
Diameter of the base = 168 m
Height of conical part (h2) = 16 – 11 = 5 m
168
Radius of conical part = 12 m  Radius   84m
2
497 Arun Deep's Understanding Math-10
 Volume of remaining part
Slant height (l)  r 2  h22
  r 2 h1 
1 2 FG1
 r h2   r 2 h1  h2
IJ
 ( 84 ) 2  ( 35) 2  7056  1225 3 H3 K
 8281  91m 22 FG 1 IJ
 × 7 × 7 30  × 24
Now surface area of the tent 7 H 3 K
=  r l + 2  r h =  r (l + 2 h1) = 154 (30 – 8) cm3 = 154 × 22 = 3388 cm3
22 Also, slant height of conical cavity
 × 84 ( 91  2 × 50)
7
l  r22  h22  ( 7 ) 2  ( 24 ) 2
22
 × 84 ×191 m 2 = 50424 m2
7  49  576  625  25cm
Extra canvas for fold and stiching @ 20%  Surface area of the remaining part
20 = 2  r1h1 +  r12 +  r2 l =  r (2 h1 + r + l)
 50424 ×  10084 .8 m 2
100 22
 × 7 ( 2 × 30  7  25)
 Total canvas required = 50424 + 10084·8 m2 7
= 60508·8 m2 = 60509 m2 = 22 (60 + 7 + 25) = 22 × 92 = 2024 cm2
12. From a solid cylinder of height 30 cm and 13. The given figure, shows a hemisphere of
radius 7 cm, a conical cavity of height radius 5 cm surmounted by a right circular
24 cm and of base radius 7 cm is drilled out. cone of base radius 5 cm. Find the volume
Find the volume and the total surface of the of the solid if the height of the cone is 7 cm.
remaining solid.
Give your answer correct to two places of
Sol. Radius of solid cylinder (r1) = 7 cm decimal.
Sol. Height of the conical part (h) = 7 cm
and radius of the base (r) = 5 cm

7cm

5cm

Height (h1) = 30 cm
Radius of cone (r2) = 7 cm
and height (h2) = 24 cm
1 2 2
 Volume of cylinder =  r2 h1  Total volume = r h + r2
3 3
1 2
and volume of cone   r h2 1 2 
3 = r2  h  r 
[ r1 = r2 = r] 3 3 
498 Arun Deep's Understanding Math-10

22 1 2  77 539 539 6 14
 h h ×  m
= × 5 × 5   7   5 cm3 6 9 9 77 3
7 3 3 
 h = 4·67 m
550  7 10  550 17 3  Required height of cone = 4·67 m
=    = × cm
7 3 3  7 3
Also, slant height of cone l  r 2  h 2
9350
= cm3 = 445.238 cm3
21 
FG 7 IJ 2  FG 14 IJ 2
= 445.24 cm3 (approx.) H 2K H 3 K
14. A buoy is made in the form of a hemisphere 49 196 441  784
surmounted by a right cone whose circular   =
4 9 36
base coincides with the plane surface of
the hemisphere. The radius of the base of 1225 35
2   m
the cone is 3·5 metres and its volume is 36 6
3
of the hemisphere. Calculate the height of  Required surface area of the buoy
the cone and the surface area of the buoy = 2  r2 +  r l
correct to 2 places of decimal. 22 7 7 22 7 35
 2× × ×  × ×
7 7 2 2 7 2 6
Sol. Given radius of base of hemisphere  m
2 77 385 462  385 2
   m
1 6 6
847
  141.17 m 2
6
P.Q. A circular hall (big room) has a
hemispherical roof. The greatest height is
equal to the inner diameter. Find the area of
the floor, given that the capacity of the hall
is 48510 m3.
2 Sol. Let h be the greatest height
 Volume of hemisphere  r3
3 and r be the radius of the base
2 22 7 7 7 3 539 3 Then 2 r = h + r  h = r
 × × × × m  m
3 7 2 2 2 6
539 2 539 3
 Volume of cone  ×  m
6 3 9
Let required height of the cone = h
1 2 539
 r h 
3 9
1 22 7 7 539
 × × × h 2
3 7 2 2 9  Volume of the hall  r3  r2 h
3
499 Arun Deep's Understanding Math-10
2 880 21
  r 3   r 2 ×( r )  r3 = ×
3 21 110
2 5 88
 r3  r3   r3 r3 = = 8 = (2)3
3 3 11
5 3  r=2m
  r  48510 (given)
3 Thus required height of the building = 2r
=2×2=4m
5 22 3
 × r  48510 16. A rocket is in the form of a right circular
3 7 cylinder closed at the lower end and
48510 × 3 × 7 surmounted by a cone with the same radius
 r3  as that of the cylinder. The diameter and the
5× 22
height of the cylinder are 6 cm and 12 cm
3 46305 respectively. If the slant height of the conical
 r  = 9261 = (21)3
5 portion is 5 cm, find the total surface area
 r = 21 m and the volume of the rocket. (Use  = 3.14).
 Required area of floor =  r2 Sol. Given, height of the cylindrical part (h) =
12 cm
22
× 21× 21  1386 m 2 Diameter of cylindrical part = 6 cm
7
6
15. A building is in the form of a cylinder  Radius of cylindrical part (r) = = 3 cm
surmounted by a hemisphere valted dome and 2
Slant height of the conical part (l) = 5 cm
9 3
contains 41 m of air. If the internal
21 5cm
diameter of dome is equal to its total height
above the floor, find the height of the building. 6cm

19 3
Sol. Volume of air in dome = 41 m 12cm
21
880 3
= m
21
Let radius of the dome = r m
Then height (h) = r m (i)  Total surface area of the rocket so formed
= rl + 2rh + r2
2 3
 Volume of building = r2h + r = r(l + 2h + r)
3
= 3.14 × 3[5 + 2 × 12 + 3] cm2
2 3 = 9.42 × [5 + 24 + 3] cm2
= r2 × r + r
3 = 9.42 × 32 = 301.44 cm2
2 3 5 3 1 2
= r3 + r = r (ii) Volume of rocket = r h1 + r2h
3 3 3

5 22 3 880 1 
 × r = = r2  h1  h 
3 7 21  3 
500 Arun Deep's Understanding Math-10

1 2  22  4  12  14 
= 3.14 × 3 × 3  l  r  12 cm3
2
 V=  49 
3  7  3 
1 2   30 
= 28.26 ×  5  3  12 cm3
2

 3  V = 22 × 7   = 1540 cm3 Ans.


 3
1  18. A solid is in the form of a right circular
= 28.26 ×  16  12 cm3 cylinder with a hemisphere at one end and a
3 
cone at the other end. Their common
4  40 3 diameter is 3.5 cm and the height of the
= 28.26 ×   12  = 28.26 × cm cylindrical and conical portions are 10 cm
3  3
and 6 cm respectively. Find the volume of
= 9.42 × 40 = 376.80 = 376.8 cm3 the solid. (Take  = 3.14)
17. The adjoining figure represents a solid Sol. Given, Diameter = 3.5 cm
consisting of a right circular cylinder with a
3 .5
hemisphere at one end and a cone at the  Radius of solid (r) = = 1.75 cm
2
other. Their common radius is 7 cm. The
height of the cylinder and cone are each of Height of cylindrical part (h1) = 10 cm
4 cm. Find the volume of the solid. and height of conical part (h2) = 6 cm

6cm
3.5cm

10cm

Sol. Volume of solid = Volume of cone + Volume


of cylinder + Volume of hemisphere

1 2 2  Total volume of the so formed solid


V= πr h  πr 2 h  πr 3
3 3 1 2 2
= r h1 + r2h + r3
1 2  3 3
V = πr   h  h 
2
r
3 3  1 2 
= r2  h1  h  r 
3 3 
1 2 
V = π7  4  4   7
2

3 3  1 2 
= 3.14 × 1.75 × 1.75   6  10   1.75 cm3
 3 3 
22 4 14 
V=  49  4  
7 3 3  3 .5 
= 9.61625 2  10  cm3
 3 
501 Arun Deep's Understanding Math-10
= 9.61625[12 + 1.167] cm3 110
= 9.61625 × 13.167 cm3  × 49  770 cm 2
7
= 126.617 cm3 20. The given figure shows a model of a solid
= 126.61 cm3 consisting of a cylinder surmounted by a
19. A toy is in the shape of a right circular cylinder hemisphere at one end. If the model is drawn
with a hemisphere on one end and a cone on to a scale of 1 : 200, find
the other. The height and radius of the (i) the total surface area of the solid in  m2.
cylindrical part are 13 cm and 5 cm
(ii) the volume of the solid in  litres.
respectively. The radii of the hemispherical
and conical parts are the same as that of the
cylindrical part. Calculate the surface area 3 cm
of the toy if the height of the conical part is
12 cm.
Sol. Given, Height of cylindrical part = 13 cm

8 cm
Radius of cylindrical part = 5 cm
Radius of cone (r) = 5 cm
Height of cone (h) = 12 cm

Sol. (i) In the given figure,


Height of cylindrical portion (h) = 8 cm
Radius (r) = 3 cm
Scale = 1 : 200
Total surface area = 2r2 + 2rh
= 2r[r + h] cm2
= 2 ×  × 3[3 + 8] cm2
= 6 × 11 = 66 cm2

(200) 2
 Surface area of the solid = 66 × cm2
1
= 66 × 40000 cm2

 slant height (l)  r 2  h2 66  40000


=  = 264 m2
100  100
 (5) 2  (12 ) 2 (ii) and volume in  litres (of model)
 25  144  169  13cm 2 3
= r + r2h
Now surface area of the toy 3
=  r l + 2  r h + 2  r2=  r (l + 2 h + 2 r)
2 
22 = πr 2  r  h  cm3
 × 5[13  2 ×13  2 × 5]cm 2 3 
7
22 

110
(13  26  10) cm 2 = π  (3)   3  8  cm3
3 
7
502 Arun Deep's Understanding Math-10
= 9(2 + 8) = 90 cm3 Take  = 3.1
Scale = 1 : 200 Sol. (i) For sphere, radius (r) = 6 cm
 Capacity = 90 × (200)3 cm3
and for solid cylinder, height (h) = 32 cm
= 90 × 8000000 cm3
= 720000000 cm3 Let radius of the cylinder be R cm

720000000π Clearly, volume of sphere = Volume of


= = 720 m3 cylinder
100  100  100
= 720 × 1000 litres = 720000 litres 4 3 4
 r = R2h    63   R 2  32
3 3
EXERCISE 17.5
288
1. The diameter of a metallic sphere is 6 cm.  288  32R 2 i.e. R2 =  9 i.e. R = 3cm
The sphere is melted and drawn into a wire 32
of uniform cross-section. If the length of  Radius of the cylinder = 3cm
the wire is 36 m, find its radius.
(ii) Curved surface area of the cylinder
Sol. Given diameter of metallic sphere = 6 cm
= 2rh
6
 Radius (r) = = 3 cm  2  3.1 3 32 cm 2  595.2 cm 2
2
3. A solid metallic hemisphere of radius 8 cm is
4 3 melted and recasted into right circular cone
Volume of sphere = r
3 of base radius 6 cm. Determine the height of
the cone.
4
= ×  × (3)3 cm3 Sol. Radius of a solid hemisphere (r) = 8 cm
3
2 3
4  Volume of solid hemisphere = r
=  × 3 × 3 × 3 cm3 = 36 cm3 3
3
 Volume of wire = 36 cm3 2
= (8)3 cm3
Given length of wire (h) = 36 m 3
Let r be the radius of the wire 2 1024
= × 512 =  cm3
 r2h = 36 3 3
 r2 × 36 × 100 = 36
1024
2  Volume of right circular cone =  cm3
1  1  3
 r2 = =   Given radius of cone (r) = 6 cm
100  10 
1 Volume
i.e. r = cm = 1 mm  Height of cone (h) =
10 1 2
πr
3
2. A solid metallic sphere of radius 6 cm
is melted and made into a solid cylinder 1024π  3 256 4
= = cm = 28 cm
of height 32 cm. Find the: 3  1 π  6  6 9 9
(i) radius of the cylinder 4. A rectangular water tank of base 11 m × 6 m
(ii) curved surface area of the cylinder contains water upto a height of 5 m. if the
503 Arun Deep's Understanding Math-10
water in the tank is transferred to a cylindrical 22 × 20 × x = 11
tank of radius 3.5 m, find the height of the
11 1 1
water level in the tank.  x= m= m= × 100
22  20 40 40
Sol. Base of a water tank = 11 m × 6 m
Height of water level in it (h) = 5 m 5
 Volume of water = 11 × 6 × 5 = 330 m3 = cm = 2.5 cm
2
 Volume of water in the cylindrical tank  Required rainfall be 2.5 cm
= 330 m3 5. The volume of a cone is the same as that of
7 the cylinder whose height is 9 cm and
Given radius of its base = 3.5 m = m diameter 40 cm. Find the radius of the base
2
of the cone if its height is 108 cm.
Volume Sol. Given diameter of a cylinder = 40 cm
 Height of water level =
πr 2
40
 Radius (r) = = 20 cm
330  7  2  2 2
= m
22  7  7 Height of cylinder (h) = 9 cm
60 4  Volume of cylinder = r2h
=
7
m=8 m
7 =  × 20 × 20 × 9 cm3
= 3600 cm3
P.Q. The rain water from a roof of dimensions
22 m × 20 m drains into a cylindrical vessel Now volume of cone = 3600 cm3
having diameter of base 2 m and height Given height of cone = 108 cm = H
3.5 m. If the rain water collected from the Let R be the radius of cone
roof just fill the cylindrical vessel, then find 1 2
the rainfall in cm. Then volume of cone = r H
3
Sol. Dimensions of roof = 22 m × 20 m
Let the required rainfall be x m
 Volume of water = 22 × 20 × x m3 Volume  3
 Radius (R) =
Thus, volume of water in cylinder = 22 × 20 1 π H
× x m3
Diameter of its base = 2 m 3600π  3
= = 100 = 10 cm
2 π  108
 Radius = =1m
2 6. Eight metallic spheres, each of radius 2 cm,
7 are melted and cast into a single sphere.
and height of water level = 3.5 m = m Calculate the radius of the new (single)
2
sphere.
 Volume of water = r2h Sol. Radius of each metallic sphere (r) = 2 cm
22 7 4 3
= × 1 × 1 × m3 = 11 m3  Volume of one sphere = r
7 2 3
 Volume of water in cylinder = Volume of
water 4
= × 23 cm3
3
504 Arun Deep's Understanding Math-10

32 97.755  21
=  cm3  h=
3 4.8  4.8  22
32  h = 4.05 cm
and volume of 8 such spheres =  × 8cm3 8. Two spheres of the same metal weigh 1 kg
3
and 7 kg. The radius of the smaller sphere is
256 3 cm. The two spheres are melted to form a
=  cm3
3 single big sphere. Find the diameter of the
big sphere.
256
Now volume of a single sphere =  cm3 Sol. Weight of first sphere = 1 kg
3
and weight of second sphere = 7 kg
4 3 Radius of smaller sphere = 3 cm
Since volume of sphere = πr
3 Let r be the radius of larger sphere
4 3
Volume Now volume of smaller sphere = r
3
 Radius of new sphere = 3 4
π
3 4
= (3)3 = 36 cm3
3
1
 256π 3 3 1 4
=    cm = (64) 3 = 4 cm and volume of larger sphere = R3
 3 4 π  3
Weight of smaller is 1 kg and of bigger is 7 kg
7. A hemispherical bowl of diameter 7.2 cm is
filled completely with chocolate sauce. This Weight of both sphere = 1 + 7 = 8 kg
sauce is poured into an inverted cone of 4
radius 4.8 cm. Find the height of the cone.  36 : R3 = 1 : 8
3
7.2 36 π  3
Sol. Radius of hemispherical bowl = = 3.6 cm 1
2  3 =
4 πR 8
2 3 27 1
Volume of hemispherical bowl = r   R3 = 27 × 8 = (3 × 2)3
3 3 =
R 8
2 22  R = 3 × 2 = 6 cm
= × × 3.6 × 3.6 × 3.6 = 97.755 cm3  Diameter of big sphere = 2 × 6 = 12 cm
3 7
9. A hollow copper pipe of inner diameter 6 cm
 Volume of chocolate sauce = 97.755 cm3
and outer diameter 10 cm is melted and
It is to poured in cone of radius = 4.8 cm changed into a solid circular cylinder of the
same height as that of the pipe. Find the
1 2
Volume of inverted cone = r h diameter of the solid cylinder.
3 Sol. Given inner diameter of a hollow pipe = 6 cm
1 22 and outer diameter = 10 cm
97.755 cm3 = × × 4.8 × 4.8 × h
3 7 6
 Inner radius (r) = = 3 cm
2
505 Arun Deep's Understanding Math-10

10 radius of the cone correct to one decimal


and outer radius (R) = = 5 cm place.
2
Sol. Internal radius of a hollow metallic cylindrical
Let height of the pipe = h cm tube (r) = 3 cm
Let R' be the radius of solid cylinder and height (h) = 21 cm
 Volume of pipe = (R2 – r2)h
1
= (52 – 32)h Thickness of metal =
2
cm = 0.5 cm
 h(25 – 9) = R'2h
 External radius (R) = 3 + 0.5 = 3.5 cm
16 = R'2  Volume of metallic tube = h(R2 – r2)
 R' = 16 = 4 =  × 21[(3.5)2 – 3.02] cm3
= 21[12.25 – 9] = 21 × 3.25 cm3
 Required diameter of solid cylinder
= 21 × 3.25 cm3
= 4 × 2 = 8 cm
Now volume of circular cone
P.Q. A solid sphere of radius 6 cm is melted into a = 21 × 3.25 cm3
hollow cylinder of uniform thickness. If the
Height of cone = 7 cm
external radius of the base of the cylinder is
4 cm and height is 72 cm, find the uniform 1 2
Since volume of cone = πr h
thickness of the cylinder. 3
Sol. Radius of a solid sphere (r) = 6 cm
1

4 3 4  2
 Volume = r = (6)3 cm3  Volume 
3 3  Radius of cone (r') =  
 1 πh 
 
4  3 
= × 216 = 288 cm3
3
1
 Volume of hollow cylinder = 288 cm3  21  3.25π  3  2 1
=   = (29.25) 2 = 5.4 cm
External radius (R) = 4 cm  π7 
Height (h) = 72 cm
10. A hollow sphere of internal and external
Let internal radius = r, then diameters 4 cm and 8 cm respectively, is
Volume = h(R2 – r2) melted into a cone of base diameter 8 cm.
 288 =  × 72(42 – r2) Find the height of the cone. (2002)
 4 = 16 – r2  r2 = 16 – 4 = 12 Sol. Internal diameter of a hollow sphere = 4 cm
and external diameter = 8 cm
i.e. r = 12 = 2 3 = 2(1.732) = 3.464 cm  Internal radius (r) = 2 cm
 Required thickness of cylinder = R – r = 4 – and external radius (R) = 4 cm
3.464 = 0.536 cm Volume of hollow sphere
P.Q. A hollow metallic cylindrical tube has an 4 4
internal radius of 3 cm and height 21 cm. = (R3 – r3) = (43 – 23) cm3
3 3
The thickness of the metal of the tube is
1 4 56  4 224
cm. The tube is melted and cast into a = (64 – 8) = =  cm3
2 3 3 3
right circular cone of height 7 cm. Find the Diameter of cone = 8 cm
506 Arun Deep's Understanding Math-10
Radius = 4 cm = r'  55h = 198
Let the height of the cone be h cm 198π
 Volume of the cone = Volume of the metal  h= = 3.6 m
55π
1 2 224 Hence required height of embankment = 3.6 m
r' h =  cm3
3 3 12. A cylindrical can of internal diameter 21 cm
contains water. A solid sphere whose diameter
224  π  3 is 10.5 cm is lowered into the cylindrical can.
 h= = 14 cm
3 π  4  4 The sphere is completely immersed in water.
 Required the height of the cone is 14 cm Calculate the rise in water level, assuming
that no water overflows.
11. A well with inner diameter 6 m is dug 22 m
deep. Soil taken out of it has been spread Sol. Internal diameter of cylindrical can = 21 cm
evenly all round it to a width of 5 m to form 21
an embankment. Find the height of the  Radius (R) = cm
2
embankment. Diameter of a solid sphere = 10.5 cm
Sol. Given inner diameter of a well = 6 m
10.5
Depth (h) = 22 m  Radius (r) = = 5.25 cm
2
6
 Radius (r) = =3m 4 3
2  Volume of sphere = r
3
 Volume of earth dug out = r2h
=  × 3 × 3 × 22 m3 = 198 m3 4
= (5.25)3 cm3
Width of an embankment = 5 m 3
Inner radius (r) = 3 m Let rise of water in cylindrical can = h
and outer radius (R) = 3 + 5 = 8 m 4
 R2h = (5.25)3
3
21 21 4
6m
h  × h= × 5.25 × 5.25 × 5.25
5m 2 2 3
21 21 4 21 21 21
 × h= × × ×
2 2 3 4 4 4
4 21 21 21 2 2
 h= × × × × ×
3 4 4 4 21 21
22m
7
 h= m = 1.75 m
4
13. There is water to a height of 14 cm in a
cylindrical glass jar of radius 8 cm. Inside
Let height of embankment = h m the water there is a sphere of diameter 12
 Volume of embankment = (R2 – r2) × h cm completely immersed. By what height will
the water go down when the sphere is
=  × h(82 – 32) m3
removed?
= h × 55 m3 Sol. Radius of the cylindrical jar (R) = 8 cm
507 Arun Deep's Understanding Math-10
Height of water level (h) = 14 cm 16.8
Volume of water = R2h  Radius (r) = = 8.4 cm
2
=  × 8 × 8 × 14 cm3 = 896 cm3
Volume of water filled in it
Diameter of sphere = 12 cm
1 2 1
12 = r h =  × 8.4 × 8.4 × 20 cm3
 Radius (r) = = 6 cm 3 3
2
1 22
4 = × × 8.4 × 8.4 × 20 cm3
Volume of sphere = r3 3 7
3
= 1478.4 cm3
4 1 1
=  × 6 × 6 × 6 cm3  % volume of water = 1478.4 ×
3 3 3
= 288 cm3 = 492.8 cm3
By immersing the sphere in the cylinder water  Volume of two equal solid cones = 492.8 cm3
rose up = 288 cm3
492.8
 Let height of water rose = h' cm and volume of one such cone =
2
  × 8 × 8 × h = 288
= 246.4 cm3
288π 9 15. A solid metallic circular cylinder of radius
h= = cm
π 88 2 14 cm and height 12 cm is melted and recast
9 into small cubes of edge 2 cm. How many
 Required height upto water rise = such cubes can be made from the solid
2
cylinder?
= 4.5 cm Sol. Radius of a solid metallic cylindrical (r)
14. A vessel in the form of an inverted cone is = 14 cm
filled with water to the brim. Its height is 20 and height (h) = 12 cm
cm and diameter is 16.8 cm. Two equal solid
Volume of cylinder = r2h
cones are dropped in it so that they are fully
submerged. As a result, one third of the water 22
in the original cone overflows. What is the = × 14 × 14 × 12 cm3
7
volume of each of the solid cone submerged?
= 7392 cm3
(2002)
Volume of one cube = (2 cm)3 = 8 cm3
Sol. Given height of conical vessal (h) = 20 cm
 Required number of cube so formed
and diameter of conical vessal = 16.8 cm
Total volume of cylinder
=
16.8cm Volume of one cone
7392
= = 924
8
20cm
20cm

16. How many shots each having diameter 3 cm


can be made from a cuboidal lead solid of
dimensions 9 cm × 11 cm × 12 cm?
Sol. Diameter of a shot = 3 cm
508 Arun Deep's Understanding Math-10

3 P.Q. Find the number of metallic circular discs


 Radius (r) = cm with 1.5 cm base diameter and height 0.2
2
cm to be melted to form a circular cylinder
4 3 of height 10 cm and diameter 4.5 cm.
Volume of one shot = r
3 1.5
Sol. Radius of the circular disc (r) = cm
3 2
4 22 3 88 27 3
= × ×   cm3 = × cm = 0.75 cm
3 7  
2 21 8
Height of circular disc (h) = 0.2 cm
Dimensions of a cuboidal lead Radius of cylinder (R) = 2.25 cm
= 9 cm × 11 cm × 12 cm
Height of cylinder (H) = 10 cm
 Volume of cuboidal lead = 9 × 11 × 12
Now,
= 1188 cm3
Number of metallic circular disc
 Number of shots to be made
Volume of cylinder
Volume of cuboidal lead =
= Volume of each shot Volume of each circular disc

πR 2 H R 2H
1188 1188  21  8 = = 2
=
88 27
=
88  27
= 84 shots πr 2 h r h

21 8
(2.25)2  (10) 2.25  2.25  10
P.Q. How many spherical lead shots of diameter = =
2
(0.75)  (0.2) 0.75  0.75  0.2
4 cm can be made out of a solid cube of lead
whose edge measures 44 cm?
225  225  10  100  100  10
Sol. Diameter of lead shot = 4 cm = = 450 shots
75  75  2  100  100
4
 Radius (r) = = 2 cm 17. A solid metal cylinder of radius 14 cm and
2 height 21 cm is melted down and recast into
4 3 spheres of radius 3.5 cm. Calculate the
and volume of one lead shot = r number of spheres that can be made.
3
Sol. Radius of a solid metallic cylinder (r) = 14 cm
4 22 704 3 and height (h) = 21 cm
= × × 2 × 2 × 2 cm3 = cm
3 7 21 Volume of cylinder = r2h
Edge (side) of a solid cube = 44 cm
22
 Volume = (a)3 = 44 × 44 × 44 cm3 = × 14 × 14 × 21 cm3 = 12936 cm3
7
Number of lead shots to be made
Total volume of cube 7
= Radius of sphere (r1) = 3.5 cm = cm
Volume of each lead shot 2
4
44  44  44 44  44  44  21  Volume of one sphere = r 3
= = 3 1
704 704
21 4 22 7 7 7 539 3
= × × × × cm3 = cm
= 2541 shots 3 7 2 2 2 3
509 Arun Deep's Understanding Math-10

12936  3 4
 Number of sphere so formed = =  × 6 × 6 × 6 cm3
539 3
= 72 spheres = 288 cm3
18. A metallic sphere of radius 10.5 cm is melted  Number of cones required
and then recast into small cones, each of
radius 3.5 cm and height 3 cm. Find the Volume of sphere
= Volume of one cone
number of cones thus obtained. (2005)
Sol. Given,
288π
Radius of a metallic sphere (r) = 10.5 cm =

4 3 = 72 cones
 Volume of sphere = r
3 20. A vessel is in the form of an inverted cone.
4 Its height is 11 cm and the radius of its top,
= ×  × 10.5 × 10.5 × 10.5 = 1543.5 cm3 which is open, is 2.5 cm. It is filled with
3
water upto the rim. When some lead shots,
Radius of each cone (r1) = 3.5 cm each of which is a sphere of radius 0.25 cm,
and height (h) = 3 cm
2
1 are dropped into the vessel, of the water
 Volume of one cone = r12h 5
3 flows out. Find the number of lead shots
1 dropped into the vessel. (2003)
=  × 3.5 × 3.5 × 3 cm3 = 12.25 cm3 Sol. Radius of the top of the inverted conical vessel
3
(R) = 2.5 cm
 Number of cones so formed
and height (h) = 11 cm
Volume of sphere
= 1 2
Volume of each cone  Volume of the water in the vessel = R h
3
1543.5π 1
=
12.25π
= 126 cones = (2.5)2 × 11 cm2
3
19. A certain number of metallic cones each of
11
radius 2 cm and height 3 cm are melted and =  × 6.25 cm3
recast in a solid sphere of radius 6 cm. Find 3
the number of cones. (2016) 1
Sol. Radius of each cone (r) = 2 cm Radius of spherical shot = 0.25 cm = cm
4
and height (h) = 3 cm
Volume of water flows out
1
 Volume of one cone = r2h 2 11 137.5
3 = of  × 6.25 cm3 =  cm3
5 3 15
1
=  × 2 × 2 × 3 = 4 cm3 4 3
3 and volume of one shot = r
3
Radius of a solid sphere (R) = 6 cm
4 1 1 1
4 = × × × cm3
 Volume of solid sphere = r3 3 4 4 4
3
510 Arun Deep's Understanding Math-10

π Sol. Surface area of a solid metallic sphere


= cm3 = 1256 cm2
48
137.5π  48 Surface area
 Number of shots required = (i)  Radius (r) =
15  π 4π
= 440 shots 1256 314  100
21. The surface area of a solid metallic sphere is = cm =
4  3.14 314
616 cm2. It is melted and recast into smaller
spheres of diameter 3.5 cm. How many such = 100 = 10 cm
spheres can be obtained? (2007) Radius of a solid cone (r1) = 2.5 cm
Sol. Surface area of a metallic sphere = 616 cm2 and height (h) = 8 cm
Since surface area of sphere = 4R2
1
 Volume = r 2h
Surface area 3 1
 Radius (R) =

1 157
= (3.14) × 2.5 × 2.5 × 8 cm3 = cm3
616  7 3 3
= cm = 49 = 7 cm
4  22 4 3
(ii) Volume of solid sphere = r
3
4 3
 Volume of larger sphere = r 4
3
= × 3.14 × 10 × 10 × 10 cm3
3
4 22 4312
= × × 7 × 7 × 7 cm3 = cm3 12560
3 7 3
= cm3
Diameter of smaller sphere = 3.5 cm 3
3 .5 7  Number of cones formed
 Radius (r) = = cm
2 4 Volume of sphere
=
4 3 Volume of cone
Volume of smaller sphere = r
3 12560 157
= ÷
4 22 7 7 7 539 3 3 3
= × × × × cm3 = cm
3 7 4 4 4 24
12560 3
4312 539 = × = 80 cones
3 157
 Number of smaller spheres = ÷
3 24 P.Q. Water is flowing at the rate of 15 km/h
4312 24 through a pipe of diameter 14 cm into a
= × = 64 spheres cuboid pond which is 50 m long and 44 m
3 539
wide. In what time will the level of water in
22. The surface area of a solid metallic sphere is the pond rise by 21 cm?
1256 cm2. It is melted and recast into solid
Sol. Speed of water flow = 15 km/h
right circular cones of radius 2.5 cm and
height 8 cm. Calculate : Diameter of pipe = 14 cm
(i) the radius of the solid sphere. 14 7
 Radius (R) = = 7 cm = m
(ii) the number of cones recast. (Use  = 3.14). 2 100
511 Arun Deep's Understanding Math-10
and dimension of a cuboid pond = r(2h + r)
= 50 m × 44 m
22
Level of water in the pond = 21 cm = × 3.5(2 × 7 + 3.5) cm2
7
21
= m = 11(14 + 3.5) cm2 = 11 × 17.5 cm2
100
= 192.5 cm2
 Volume of water in the pond
4 3
21 3 (ii) Volume of sphere = r
= 50 × 44 × m = 462 m2 3
100
 Length of flow of water in the pipe 4 22
= × × 3.5 × 3.5 × 3.5 cm3
3 7
462 462  7  100  100
= 2 =
πr 22  7  7 88 539 3
= × 6.125 = cm = 179.67 cm3
30000 3 3
= 30000 m = = 30 km and volume of can = r2h
1000
Distance 30 22
 Time taken = = × 3.5 × 3.5 × 7 cm3 = 269.5 cm3
Speed = 15 = 2 hours 7
23. A cylindrical can whose base is horizontal  Actual water in the can = 269.5 – 179.67
and of radius 3.5 cm contains sufficient water = 89.83 cm
so that when a sphere is placed in the can,
the water just covers the sphere. Given that 89.83  7
 Height of water level =
the sphere just fits into the can, calculate : 22  3.5  3.5
(i) the total surface area of the can in contact = 2.32 cm
with water when the sphere is in it.
(ii) the depth of the water in the can before the 1 7
=2 = cm
sphere was put into the can. Given your 3 3
answer as proper fractions.
Sol. Radius of a cylindrical can = 3.5 cm MULTIPLE CHOICE QUESTIONS
 Radius of the sphere = 3.5 cm Choose the correct answer from the given four
and height of water level in the can options (1 to 21) :
= 3.5 × 2 = 7 cm 1. If two cylinders of same lateral surface have
their radii in the ratio 4 : 9, then the ratio of
their heights is
(a) 2:3 (b) 3 : 2
(c) 4:9 (d) 9 : 4
Sol. Ratio in two cylinder having same lateral
surface area and their radii is 4 : 9
Let r1 be the radius of the first and r2 be the
second cylinder and h1, h2 and their heights
Let r1 = 4x and r2 = 9x
(i)  Total surface area of the can in touch with
water = 2rh + r2  2r1h1 = 2r2h2
512 Arun Deep's Understanding Math-10
 24x × h1 = 2 ×  × 9xh2
2
r r
h1 =  × 2l +   
9x 2 2
 h = =9:4
2 4x

 Ratio in their heights = 9 : 4 Ans. (d) πr 2


= rl +
4
2. The radii of two cylinders are in the ratio
2 : 3 and their heights are in the ratio 5 : 3.
The ratio of their volumes is  r
= r  l   Ans. (b)
 4
(a) 10 : 17 (b) 20 : 27
(c) 17 : 27 (d) 20 : 37 P.Q. In a cylinder, if radius is doubled and height
is halved then its curved surface area will be
Sol. Radii of two cylinder are in the ratio = 2 : 3
(a) halved (b) doubled
Let radius (r1) = 2x
(c) same (d) four times
and radius (r2) = 3x
Sol. Let radius of a cylinder = r
Ratio in their height = 5 : 3
and height = h
Let height of the first cylinder = 5y
Then curved surface area of cylinder = 2rh
and height of second = 3y
Now if radius is doubled and height is halved,
Now, volume of the first cylinder then curved surface area of cylinder
= r12h = (2x)2 × 5h = 20x2y
r
and volume of second = (3x)2 × 3y = 2 × 2h = 2rh
2
=  × 27x2y which is same  Ans. (c)
 Required ratio = 20x2y : 27x2y P.Q. If the diameter of the base of a cone is
= 20 : 27 Ans. (b) 12 cm and height is 20 cm, then its volume
is
3. The total surface area of a cone whose radius
(a) 240 cm3 (b) 480 cm3
r
is
2
and slant height 2l is (c) 720 cm3 (d) 960 cm3
Sol. Given diameter of the base of a cone = 12 cm
 r
(a) 2r (l + r) (b) r  l   12
 4  Radius (r) = = 6 cm
2
(c) r (l + r) (d) 2rl
and height of cone (h) = 20 cm
r
Sol. Radius of a cone = and slant height = 2l 1 2 1
2 Volume = r h =  × 6 × 6 × 20 cm3
3 3
Total surface area of a cone
= 240 cm3 Ans. (a)
= rl + r2
513 Arun Deep's Understanding Math-10
4. If the diameter of the base of cone is 10 cm 5. If the diameter of a sphere is 16 cm, then its
and its height is 12 cm, then its curved surface area is
surface area is (a) 64 cm2 (b) 256 cm2
(a) 60 cm2 (b) 65 cm2 (c) 192 cm2 (d) 256 cm2
(c) 90 cm 2 (d) 120 cm2 Sol. Diameter of a sphere = 16 cm
Sol. Given diameter of the base of a cone = 10 cm 16
 Radius (r) = = 8 cm
10 2
 Radius (r) = = 5 cm
2 Surface area of sphere = 4r2
and height (h) = 12 cm = 4 × 8 × 8 cm2
= 256 cm2 Ans. (b)
 l= r 2  h2 = 5 2  12 2 6. If the radius of a hemisphere is 5 cm, then
its volume is
= 25  144 = 169 = 13 cm
Curved surface area of cone 250 500
(a)  cm3 (b)  cm3
= rl =  × 5 × 13 = 65 cm2 Ans. (b) 3 3
P.Q. If the diameter of the base of a cone is 125
12 cm and height is 20 cm, then its volume (c) 75 cm3 (d)  cm3
3
is
Sol. Radius of a hemisphere (r) = 5 cm
(a) 240 cm3 (b) 480 cm3
(c) 720 cm 3 (d) 960 cm3 2 3
Volume of hemisphere = r
Sol. Given diameter of the base of a cone = 12 cm 3

12 2
 Radius (r) = = 6 cm = (5)3 cm3
2 3
and height of cone (h) = 20 cm 250
=  cm3 (a)
1 2 1 3
Volume = r h =  × 6 × 6 × 20 cm3 7. If the ratio of the diameters of the two
3 3
= 240 cm3 Ans. (a) spheres is 3 : 5, then the ratio of their surface
areas is
P.Q. If the radius of a sphere is 2r, then its volume
will be (a) 3 : 5 (b) 5 : 3
(c) 27 : 125 (d) 9 : 25
4 3 Sol. Ratio in the diameters of two spheres = 3 : 5
(a) r (b) 4r3
3 Let radius of the first sphere = 3x cm
8 πr 3 32πr 3 and radius of the second sphere = 5x cm
(c) (d)  Ratio in their surface area
3 3
= 4(3x)2 : 4(5x)2  9x2 : 25x2
Sol. Given radius of a sphere = 2r
= 9 : 25 Ans. (d)
4 3 4 P.Q. The radius of a hemispherical balloon
Volume of sphere = r =  × (2r)3
3 3 increases from 6 cm to 12 cm as air is being
pumped into it. The ratio of the surface areas
4 32πr 3
=  × 8r3 = Ans. (d) of the balloon in the two cases is
3 3
(a) 1 : 4 (b) 1 : 3
514 Arun Deep's Understanding Math-10
(c) 2 : 3 (d) 2 : 1 another then the surface area of the new solid
Sol. Radius of balloon (hemispherical) in the will same or not same i.e. can't say.
original position = 6 cm Ans. (d)
and in increased position = 12 cm P.Q. If a marble of radius 2.1 cm is put into a
 Ratio in their surface areas cylindrical cup full of water of radius 5 cm
= 4(6)2 : 4(12)2 and height 6 cm, then the volume of water
that flows out of the cylindrical cup is
= 62 : 122 = 36 : 144
(a) 38.8 cm3 (b) 55.4 cm3
=1:4 Ans. (a)
(c) 19.4 cm3 (d) 471.4 cm3
P.Q. The shape of a gilli, in the the game of gilli-
danda, is a combination of Sol. Given radius of a marble = 2.1 cm
4 3 3
 Volume of marble = r cm
3

4 22
= × × 2.1 × 2.1 × 2.1 cm3
(a) two cylinders 3 7
(b) a cone and a cylinders = 38.88 cm3 = 38.8 cm3 (a)
(c) two cones and a cylinder 10. The volume of the largest right circular cone
(d) two cylinders and a cone that can be carved out from a cube of edge
4.2 cm is
Sol. The shape of a gilli is the combination of
(a) 9.7 cm3 (b) 77.6 cm3
two cones and a cylinder (as shown in the
figure). Ans. (c) (c) 58.2 cm3 (d) 19.4 cm3
8. If two solid hemisphere of same base radius Sol. Given edge of cube = 4.2 cm
r are joined together along their bases, then 4.2
the curved surface of this new solid is Radius of largest cone cut out =
2
(a) 4r2 (b) 6r2 = 2.1 cm
(c) 3r2 (d) 8r2 and height = 4.2 cm
Sol. Radius of two solid hemisphere = r
1 2
These are joined together along the bases  Volume of cone = r h
3
 Curved surface area = 2r2 × 2 = 4r2 (a)
P.Q. During conversion of a solid from one shape 1 22
to another, the volume of the new shape will = × × 2.1 × 2.1 × 4.2 cm3
3 7
(a) increase (b) decrease = 19.404 = 19.4 cm3 (d)
(c) remain unaltered (d) be doubled 11. The volume of the greatest sphere cut off
Sol. During the conversion of a solid into another, from a circular cylindrical wood of base
the volume of the new shaper will be same. radius 1 cm and height 6 cm is
i.e. remain unaltered Ans. (c)
4
9. If a solid of one shape is converted to another, (a) 288  cm3 (b)  cm3
then the surface area of the new solid 3
(a) remains same (b) increases (c) 6  cm3 (d) 4  cm3
(c) decreases (d) can't say Sol. Given radius of cylinder (r) = 1 cm
Sol. If a solid of one shape is conversed into Height (h) = 6 cm
515 Arun Deep's Understanding Math-10
The largest sphere that can be cut off from 1 3 2 3
the cylinder of radius 1 cm = r : r : r3
3 3
4 3 4
 Volume of sphere = r = (1)3 1 2
3 3 = : :1=1:2:3 Ans. (d)
3 3
4 14. If a sphere and a cube have equal surface
=  cm3 Ans. (b)
3 areas, then the ratio of the diameter of the
12. The volumes of two spheres are in the ratio sphere to the edge of the cube is
64 : 27. The ratio of their surface areas is (a) 1 : 2 (b) 2 : 1
(a) 3 : 4 (b) 4 : 3 (c) π : 6 (d) 6 : π
(c) 9 : 16 (d) 16 : 9 Sol. A sphere and a cube have equal surface area
Sol. Given ratio in volumes of two spheres = Let a be the edge of a cube and r be the
64 : 27 radius of the sphere, then
4r2 = 6a2  (2r)2 = 6a2 ( d = 2r)
4 4π r13 64
= = 3 =
3 3 r2 27 d2 6 d 6
 2 =  =
a π a π
3 3
 r1  4
   =    Required ratio = d : a = 6 :
π
 r2  3
 Ans. (d)
r1 4 15. A solid piece of iron in the form of a cuboid
 r = of dimensions 49 cm × 33 cm × 24 cm is
2 3
moulded to form a sphere. The radius of the
 Ratio in their surface area sphere is
(a) 21 cm (b) 23 cm
4πr12 r12 2
4 16
= = =   = (c) 25 cm (d) 19 cm
4πr22 r22 3 9 Sol. Dimension of a cuboid = 49 cm × 33 cm ×
 Required ratio be 16 : 9. Ans. (d) 24 cm
13. If a cone, a hemisphere and a cylinder have  Volume of a cuboid = 49 × 33 × 24 cm3
equal bases and have same height, then the  Volume of sphere = Volume of a cuboid
ratio of their volumes is  Volume of a sphere = 49 × 33 × 24 cm3
(a) 1 : 3 : 2 (b) 2 : 3 : 1 4 3
Since volume of sphere = πr
(c) 2 : 1 : 3 (d) 1 : 2 : 3 3
Sol. If a cone, a hemisphere and a cylinder have
1
equal bases = r (say)  3
and height = h in each case and r = h  Volume 
 Radius (r) =  
1 2 2 3  4π 
 
Ratio in their volumes = r h : r : r2h  3 
3 3
1
1 2
= r2r : r3 : r2r  49  33  24  3  7  3
3 3 =  
 4  22 
516 Arun Deep's Understanding Math-10
1  Volume of cylinder = r2h
= (49  7  3  3  3) 3
15 15
=× × × 10 cm3
1 2 2
= (7  7  7  3  3  3) 3
1125π
= 7 × 3 = 21 cm (a) = cm3
2
16. If a solid right circular cone of height 24 cm
and base radius 6 cm is melted and recast in 1125π
 Volume of sphere (R) = cm3
the shape of a sphere, then the radius of the 2
sphere is
4 3
(a) 4 cm (b) 6 cm Since, Volume of Sphere = R
3
(c) 8 cm (d) 12 cm
Sol. Height of a circular cone (h) = 24 cm 1
and radius (r) = 6 cm  3
 Volume 
1 2  Radius of sphere (R) =  
 Volume of a cone = r h  4π 
3  
 3 
1
=  × 6 × 6 × 24 cm3 1 1
3  1125π  3  3  3375  3
=   =  
Volume of sphere = Volume of a cone  2  4π   8 
1
Now volume of sphere =  × 36 × 24 cm3 3 3375
3
3 1125
Let R be in radius of sphere 3 375
4 1 5 125
Then R3 =  × 36 × 24 5 25
3 3 5 5
36  24 1
4R3 = 36 × 24  r3 =
4
1
R3 = 3 × 3 × 3 × 2 × 2 × 2 = 3 3 × 2 3  53  33  3 53
 R = 3 × 2 = 6 cm  Ans. (b) =  3
 =
 cm
 2  2
17. If a solid circular cylinder of iron whose
diameter is 15 cm and height 10 cm is melted 15
and recasted into a sphere, then the radius = = 7.5 cm  Ans. (c)
2
of the sphere is
18. The number of balls of radius 1 cm that can
(a) 15 cm (b) 10 cm
be made from a sphere of radius 10 cm is
(c) 7.5 cm (d) 5 cm
(a) 100 (b) 1000
Sol. Diameter of a cylinder = 15 cm
(c) 10000 (d) 100000
15 Sol. Radius of sphere (R) = 10 cm
 Radius (r) = cm
2 4 4
and height (h) = 10 cm  Volume of sphere = R3 = (10)3 cm3
3 3
517 Arun Deep's Understanding Math-10

4 icecream cones of radius 2 cm and height 7


=  × 1000 cm3 cm upto its brim. The number of children
3
who will get the icecream cones is
and radius of one ball (r) = 1 cm (a) 163 (b) 263
4 4 (c) 363 (d) 463
 Volume of one ball = (1)3 cm3 =  cm3
3 3 Sol. Edge of a cubical icecream brick = 22 cm
 Volume = a3 = (22)3 = 10648 cm3
4π  1000  3
 Number of balls = 1000 (b) Radius (r) of icecream cone (r) = 2 cm
3 4  π
and height (h) = 7 cm
19. A metallic spherical shell of internal and
external diameters 4 cm and 8 cm, 1 2
 Volume of one cone = r h
respectively is melted and recast into the 3
form of a cone of base diameter 8 cm. The
height of the cone is 1 22 88
= × × 2 × 2 × 7 cm3 = cm3
(a) 12 cm (b) 14 cm 3 7 3
(c) 15 cm (d) 18 cm 10648  3
Sol. Internal diameter of metallic shell = 4 cm  Number of cones = = 363 (c)
88
and external diameter = 8 cm 21. Twelve solid spheres of the same size are
4 made by melting a solid metallic cylinder of
 Internal radius (r) = = 2 cm base diameter 2 cm and height 16 cm. The
2
diameter of each sphere is
8 (a) 4 cm (b) 3 cm
and external radius (R) = = 4 cm
2 (c) 2 cm (d) 6 cm
Sol. Diameter of cylinder = 2 cm
4
Volume of metal used = (R3 – r3)
3 2
 Radius (r) = = 1 cm
2
4 4
= (43 – 23) cm3 = × (64 – 8) cm3 and height (h) = 16 cm
3 3
 Volume = r2h
4
=  × 56 cm3 22 352 3
3 = × 1 × 1 × 16 = cm
7 7
Diameter of cone = 8 cm
 Volume of 12 solid spheres so formed
8
 Radius of cone = = 4 cm 352
2 = cm3
7
Volume 4π  56  3
 Height = = 352 352 3
1 2 3  1 π  4  4  Volume of each sphere = = cm
πr 7  12 84
3
1
= 14 cm (b)
 352  3  7  3
20. A cubical icecream brick of edge 22 cm is to  Radius of each sphere =  
 84  4  22 
be distributed among some children by filling
518 Arun Deep's Understanding Math-10
1 23. In the given figure, the bottom of the glass
= 13 = 1 cm has a hemispherical raised portion. If the glass
 Diameter = 2 × 1 = 2 cm (c) is filled with orange juice, the quantity of juice
which a person will get is
22. A hollow cube of internal edge 22 cm is filled
with spherical marbles of diameter 0.5 cm (a) 135  cm3 (b) 117  cm3
(c) 99  cm3 (d) 36  cm3
1
and it is assumed that space of the cube
8
remains unfilled. Then the number of marbles
that the cube can accommodate is
(a) 142296 (b) 142396
(c) 142496 (d) 142596

15 cm
Sol. Internal edge of a hollow cube = 22 cm
Volume = (side)3 = (22)3
= 22 × 22 × 22 cm3 = 10648 cm3
Diameter of spherical marble = 0.5 cm
1
= cm
2
6 cm
1 1 1
 Radius = × = cm 6
2 2 4 Sol. Radius of base of cylinder (r) = cm
2
4 3 = 3 cm
 Volume = r
3 and height (h) = 15 cm
4 22 1 1 1 2 3
= × × × × cm3  Volume of the glass = r2h – r
3 7 4 4 4 3

11  2 
= cm3 = r  rh  r 2 
168  3 

1 3  2 
Space left unfilled = 10648 ×
8
cm =  × 3  3  15   9  = 3(45 – 6) cm3
 3 
= 1331 cm3
= 3 × 39 = 117 cm3 (b)
 Remaining volume for marbles
= 10648 – 1331 = 9317 cm3 CHAPTER TEST
 Number of marble to accommodate 1. A cylindrical container is to be made of tin
11 9317  168 sheet. The height of the container is 1 m and
= 9317 ÷ = its diameter is 70 cm. If the container is open
168 11
at the top and the tin sheet costs `300 per
= 142296 (a) m2, find the cost of the tin for making the
container.
519 Arun Deep's Understanding Math-10
Sol. Height of container opened at the top (h)  Volume of wooden wasted = 5880 – 4620
= 1 m = 100 cm = 1260 cm3
and diameter of container = 70 cm 3. Find the volume and the total surface area of
a cone having slant height 17 cm and base
70
 Radius (r) = = 35 cm diameter 30 cm. Take  = 3.14.
2
Sol. Slant height of a cone (l) = 17 cm
 Total surface area = 2rh + r2 Diameter of base = 30 cm
[ container is open]
30
= r(2h + r)  Radius (r) = = 15 cm
2
22
= × 35(2 × 100 + 35) cm2  Height (h) =
7 l2  r2 = 17 2  15 2 cm
= 110(200 + 35) = 110 × 235 cm2 = 289  225 = 64 = 8 cm
110  235 2 517 2
= m = m 1 3
100  100 200 Now volume of cone = r h
3
517 2
 Area of sheet required = m 1
200 = (3.14) × 15 × 15 × 8 cm3 = 1884 cm3
3
Given cost of 1 m2 sheet = `300
and total surface area of cone = rl + r2
517 = r(l + r) = 3.14 × 15 × (17 + 15) cm2
 Total cost of sphere = × 300
200 = 3.14 × 15 × 32 cm2 = 1507.2 cm2
1551 4. Find the volume of a cone given that its height
=`
2
= `775.50 is 8 cm and the area of base 156 cm2.
Sol. Height of a cone = 8 cm
2. A cylinder of maximum volume is cut out
from a wooden cuboid of length 30 cm and Area of base = 156 cm
cross-section of square of side 14 cm. Find 1
the volume of the cylinder and the volume  Volume of cone = × area of base × height
3
of wood wasted.
Sol. Dimensions of the wooden cuboid 1 1248
= × 156 × 8 = cm3
= 30 cm × 14 cm × 14 cm 3 3
 Volume = 30 × 14 × 14 = 5880 cm3 = 416 cm3
Largest size of cylinder cut out of the 5. The circumference of the edge of a
wooden cuboid will be of diameter = 14 cm hemispherical bowl is 132 cm. Find the
and height = 30 cm capacity of the bowl.
Sol. Circumference of the edge of bowl
14
 Radius of cylinder = = 7 cm = 132 cm
2
 Radius of hemispherical bowl = r
 Volume of cylinder = r2h
132 132 × 7
22    21 cm
= × 7 × 7 × 30 cm3 = 4620 cm3 2 2 × 22
7
520 Arun Deep's Understanding Math-10
2
Thus, volume of the bowl  r3
3
2 22
 × × ( 21) 3 cm 3

10.2cm
3 7
2 22
 × × 9261 cm 3
3 7 4.2cm
= 19404 cm3
1
6. The volume of a hemisphere is 2425 cm2.
2
Find the curved surface area.  Height of conical part = 10.2 – 4.2 = 6 cm
Now volume of the toy
1
Sol. Given volume of a hemisphere = 2425 cm3
2 1 2 2
= r h + r3
3 3
4851 3
= cm
2 1 2
= r (h + 2r)
Let radius = r, then 3
2 3 4851 1 22
r = = × (4.2)2 (6 + 2 × 4.2)
3 2 3 7
2 22 4851
 × × r3 = 22
3 7 2 = × 4.2 × 4.2 × (6 + 8.4)
21
3
4851  3  7 9261  21  22
 r3 = = =   = × 4.2 × 4.2 × 14.4 cm2
2  2  22 8  2  21
21 = 266.112 cm3
 r= cm P.Q. A medicine capsule is in the shape of a
2
 Curved surface area = 2r2 cylinder of diameter 0.5 cm with two
hemispheres stuck to each of its ends. The
22 21 21 2 length of the entire capsule is 2 cm. Find the
=2× × × cm
7 2 2 capacity of the capsule.
= 693 cm2 Sol. Diameter of cylindrical part = 0.5 cm
7. A solid wooden toy is in the shape of a right Total length of the capsule = 2 cm
circular cone mounted on a hemisphere. If
the radius of the hemisphere is 4.2 cm and
the total height of the toy is 10.2 cm, find 0.5 cm
the volume of the toy.
Sol. A wooden solid toy is of a shape of right 2 cm
circular cone mounted on a hemisphere.
0.5
Radius of hemisphere (r) = 4.2 cm  Radius (r) = = 0.25 cm
Total height = 10.2 cm 2
521 Arun Deep's Understanding Math-10
and length of cylindrical part = 2 – 2 × 0.25
4 22 7 × 7 × 7 22 7 7
= 2 – 0.5 = 1.5 cm  × ×  × × ×12 cm 3
3 7 2×2×2 7 2 2
2 3
 Volume of the capsule = 2 × r + r2h 539 539  1386
3   462 
3 3
4 3
= r + r2h 1925 3
3  cm
3
4 
= r2  r  h  2
 641 cm 3
3  3

22 4 
= × 0.25 × 0.25   0.25  1.5 
7  3 

22 4 1 
= × 0.0625    1.5  cm3
7 3 4 

22 1 1 3
= ×    cm3
7 16 3 2

22 11 121 3
= × = cm
112 6 336
= 0.360 cm3 = 0.36 cm3
8. A solid is in the form of a cylinder with
hemispherical ends. The total height of the and Total surface area of the solid
solid is 19 cm and the diameter of the cylinder = 2 × 2  r2 + 2  r h
is 7 cm. Find the volume and the total surface = 4  r2 + 2  r h
area of the solid.
= 2  r (2 r + h)
7
Sol. Given radius of cylinder  cm
2 22 7 FG
7 IJ
7  2× × 2 ×  12  22 ( 7  12 )
and height of cylinder  19  2 × cm 7 2 H
2 K
2
= 19 – 7 = 12 cm = 22 × 19 cm2 = 418 cm2
7 9. The radius and height of a right circular cone
and radius of hemisphere  cm are in the ratio 5 : 12. If its volume is
2
2512 cm3, find its slant height.
 Total volume of the solid
(Take  = 3·14).
2
 2×  r 3   r 2 h Sol. Let radius of cone (r) = 5 x
3
then height (h) = 12 x


4 22 7
× ×
FG IJ 3  22 × FG 7 IJ 2 ×12 cm3  Volume of right circular cone =
1
 r2 h
3 7 2 H K 7 H 2K 3
522 Arun Deep's Understanding Math-10
1 .

3
( 3 14 ) ( 5 x ) 2 × 12 x

FG 2 x IJ 2 h =  x 2 h
Also given volume of cone = 2512 cm3
H2K
 Ratio between the two volumes of two sides
1
 ( 3 .14 ) 25x2 × 12x = 2512 3 2 2
3 = πx h :  x h
4
1 .
 × 3 14 × 300 x 3  2512 3
3  :1=3:4
4
2512 × 3
x3  11. A solid cone of base radius 9 cm and height
3 .14 × 300 10 cm is lowered into a cylindrical jar of
radius 10 cm, which contains water sufficient
2512 × 3 × 100 to submerge the cone completely. Find the
 = 8 = (2)3
314 × 300 rise in water level in the jar.
 x=2 Sol. Radius of the cone (r) = 9 cm
 Radius of cone (r) = 5 × 2 = 10 cm Height of the cone (h) = 10 cm
and height (h) = 12 × 2 = 24 cm Volume of water filled in cone

1 2 1
 Required slant height (l)  r 2  h2   r h   ( 9 ) 2 × 10 cm 3
3 3
 (10) 2  ( 24 ) 2
810
   270  cm 3
 100  576  676 = 26 cm 3

10. A cone and a cylinder are of the same height. Now radius of the cylindrical jar = 10 cm
If diameters of their bases are in the ratio Let h be the height of water in the jar
3 : 2, find the ratio of their volumes.   r2 h = 270 
Sol. Let height of cone and cylinder = h  (10)2 h = 270 
Diameter of the base of cone = 3 x  100  h = 270 
Diameter of base of cylinder = 2 x
270 
 h  2 .7 cm
1 2 100 
 Volume of cone   ( r1 ) h
3
12. An iron pillar has some part in the form of a
right circular cylinder and the remaining in

1

FG IJ 2 × h
3x
the form of a right circular cone. The radius
3 2 H K of the base of each of cone and cylinder is
8 cm. The cylindrical part is 240 cm high
1 9 2 3
  x h = πx 2 h and the conical part is 36 cm high. Find the
3 4 4
weight of the pillar if one cu. cm of iron
and volume of cylinder =  r2 h weighs 7·8 grams.
523 Arun Deep's Understanding Math-10
Sol. Radius of the base of cone = 8 cm Height of cylindrical part = 5 m
Total height of the tent = 21 m
 Height of conical portion = 21 – 5 = 16 m

16m

5m
5m
and radius of cylinder = 8 cm
Height of cylindrical part (h1) = 240 cm
and height of conical part (h2) = 36 cm
63m
Volume of the iron pillar
 Slant height of the conical portion

1 2
 r h2   r 2 h1   r 2
FG
1
h2  h1
IJ
3 3H K = r 2  h2 = 63 2  16 2


22
×8×8
1 LM OP
× 36  240 cm 3 = 3969  256 = 4225 m = 65 m
7 3 N Q  Surface area of the tent
LM
1408 36 OP 1408 = 2rh + rl
  240 cm 3  [ 252 ] cm 3
7 N
3 Q 7 22
= r (2h + l) = × 63 (2 × 5 + 65)
1408 7
 × 252  1408 × 36  50688 cm 3
7 = 198 × (10 + 65) = 198 × 75 m2
Weight of 1 cm3 = 7·8 gm = 14850 m2
 Total weight of the pillar cost of one 1 sq. m cloth = `36
= 50688 × 7·8 gm = 395366·4 gm
 Total cost = Rs. 14850 × 36 = `534600
= 395·3664 kg
14. The entire surface of a solid cone of base
13. A circus tent is made of canvas and is in the
radius 3 cm and height 4 cm is equal to the
form of right circular cylinder and a right
entire surface of a solid right circular cylinder
circular cone above it. The diameter and
of diameter 4 cm. Find the ratio of their
height of the cylindrical part of the tent are
(i) curved surfaces (ii) volumes.
126 m and 5 m respectively. The total height
of the tent is 21 m. Find the total cost of the Sol. Radius of the base of a cone (r) = 3 cm
tent if the canvas used costs `36 per square Height (h) = 4 cm
metre.
l= r 2  h2 = 32  4 2
Sol. Diameter of the cylindrical part = 126 m

126 = 9  16 = 25 = 5 cm
 Radius (r) = = 63 m
2  Total surface area of cone = rl + r2
524 Arun Deep's Understanding Math-10

22 1 2
= r(l + r) = × 3(5 + 3) cm2  Volume = r h
7 3

66 528 2 1
= ×8= cm =  × (2.1)2 × (8.4) cm3
7 7 3
Diameter of cylinder = 4 cm =  × 4.41 × 2.8 cm3 = 12.348 cm3
 Volume of sphere = 12.348 cm3
4
 Radius (r1) = = 2 cm
2 1
 3
528 2  Volume 
Total surface area = cm Radius =  
7  4π 
Let h be the height, then  3 
528 1
 2r1h1 + 2r2 = 1
7 12.348π  3  3 3
= 
  = (9.261)
 4 π 
528
 2r(h1 + r) =
7 1
= (2.1  2.1  2.1) 3 = 2.1 cm
22 528
 2× × 2(h1 + 2) = P.Q. How many lead shots each of diameter 4.2cm
7 7
can be obtained from a solid rectangular lead
528 7 piece with dimensions 66 cm, 42 cm and 21
 h1 + 2 = × cm.
7 2  22  2
 h1 + 2 = 6 Sol. Dimensions of a solid rectangular lead piece
h1 = 6 – 2 = 4 cm = 66 cm × 42 cm × 21 cm
 Volume = 66 × 42 × 21 cm3
(i) Ratio between curved surface of cone and
cylinder Diameter of a lead shot = 4.2 cm
= rl : 2r1h1 4.2
 Radius (r) = = 2.1 cm
=×3×5:2××2×4 2
= 15 : 16
4 3
(ii) Ratio between their volumes and volume = r
3
1 2
= r h : r12h1 4 22
3 = × × 2.1 × 2.1 × 2.1 cm3
3 7
1 = 88 × 0.1 × 4.41 = 38.808 cm3
= ×3×3×4:×2×2×4
3
66  42  21
=3:4 Number of shots =
38.808
15. A cone is 8.4 cm high and the radius of its
base is 2.1 cm. It is melted and recast into a 66  42  21  1000
= = 1500
sphere. Find the radius of the sphere. 38808
Sol. Radius of base of a cone (r) = 2.1 cm 16. Find the least number of coins of diameter
and height (h) = 8.4 cm 2.5 cm and height 3 mm which are to be
525 Arun Deep's Understanding Math-10
melted to form a solid cylinder of radius 1024
3 cm and height 5 cm.  12  h  
3
Sol. Radius of a cylinder (r) = 3 cm
1024  256
Height (h) = 5 cm  h   28 .44 cm
 Volume = r2h =  × 3 × 3 × 5 = 45 cm2 3× 12  9
Diameter of a coins = 2.5 cm 18. A vessel in the form of a hemispherical bowl
is full of water. The contents are emptied
2.5 into a cylinder. The internal radii of the bowl
 Radius (r1) = = 1.25 cm
2 and cylinder are respectively 6 cm and 4 cm.
Find the height of the water in the cylinder.
3
and height (h1) = 3 mm = cm Sol. Radius of hemispherical bowl = 6 cm
10
 Volume of the water in the bowl
 Volume of a coin = r12h1
2 2
  r 3   ( 6) 3 cm 3 = 144  cm3
3 3 3
=  × 1.25 × 1.25 × cm3
10  Volume of water in the cylinder = 144  cm3
= 0.46875 cm3 Radius of the cylinder = 4 cm
 Number of coins required Let h be the height of water
  r2 h = 144 
45π 45
= = = 96 coins  (4)2 h = 144  16 h = 144
0.46875π 0.46875
144
17. A hemisphere of lead of radius 8 cm is cast  h 9
16
into a right circular cone of base radius
6 cm. Determine the height of the cone Hence height of water in the cylinder = 9 cm
correct to 2 places of decimal. P.Q. The diameter of a metallic sphere is 42 cm.
It is metled and drawn into a cylindrical wire
Sol. Radius of hemisphere = 8 cm
of 28 cm diameter. Find the length of the
2 wire.
 Volume   r 3 cm 3
3 Sol. Diameter of sphere = 42 cm
2 2 42
  (8) 3 cm 3   512 cm 3  Radius of sphere   21cm
3 3 2
4

1024
 cm 3  Volume of the sphere  r3
3 3
4
1024   ( 21) 3 cm 3 = 12348  cm3
 Volume of right circular cone   cm 3 3
3
Now volume of the wire drawn
Radius = 6 cm
= 12348  cm3
Let h be the height of the cone
and diameter = 28 cm
1 2 1024
 r h   28
3 3  Radius   14 cm
2
1 1024 Let length of wire = h cm
  ( 6) 2 h  
3 3  Volume of wire =  r2 h
526 Arun Deep's Understanding Math-10
= (14)2
h = 196  h
cm2 cm2 4
 196  h = 12348   ×  ×( 6) 3 cm 3 = 288  cm3
3
12348  12348  Volume of hollow cylinder = 288  cm3
h   63 cm External radius of cylinder (R) = 5 cm
196  196
and height (h) = 32 cm
19. A sphere of diameter 6 cm is dropped into a
Let r be the inner radius
right circular cylindrical vessel partly filled
 Volume =  (R2 – r2) h
with water. The diameter of the cylindrical
vessel is 12 cm. If the sphere is completely   (R2 – r2) h = 288 
submerged in water, by how much will the 2 288
[(5)2 – r2] × 32 = 288  25  r 
level of water rise in the cylindrical vessel? 32
6 25 – r = 9  r = 25 – 9 = 16 = (4)
2 2 2
Sol. Radius of sphere   3 cm  r=4
2
 Thickness of hollow cylinder = R – r
= 5 – 4 = 1 cm
P.Q. A solid is in the form of a right circular cone
mounted on a hemisphere. The radius of the
hemisphere is 3·5 cm and the height of the
cone is 4 cm. The solid is placed in a cylindrical
vessel, full of water, in such a way that the
whole solid is submerged in water. If the
radius of the cylindrical vessel is 5 cm and its
12
Radius of cylinder   6 cm height is 10·5 cm, find the volume of water
2 left in the cylindrical vessel.
Let height of water raised = h cm Sol. Radius of hemisphere (r) = 3·5 cm
4 3 Height of cone (h1) = 4 cm
Now volume of sphere   r
3
4
  ( 3) 3 cm 3 = 36  cm3
3
and volume of water in the cylinder
= 36  cm3
  r h = 36   (6)2 h = 36
2

 36 h = 36  h = 1
 Height of raised water = 1 cm
20. A solid sphere of radius 6 cm is metled into a
hollow cylinder of uniform thickness. If the
external radius of the base of the cylinder is
5 cm and its height is 32 cm, find the uniform
thickness of the cylinder. Radius of cylindrical vessel = 5 cm
Sol. Radius of solid sphere = 6 cm and height = 10·5 cm
4 2 1
 Volume of solid sphere  r3 Volume of solid  r3  r2 h
3 3 3
527 Arun Deep's Understanding Math-10
1 2 Height of cylinder = 10·5 cm
  r ( 2 r  h1 )
3  Volume of cylinder which is full of water

1 22 22
 × ×( 3.5) 2 [ 2 × 3 .5  4 ]cm 2  r2 h  × 5× 5×10 .5 cm 3
3 7 7
= 22 × 25 × 1·5 cm3 = 825 cm3
1 22 12 .25
 × × [ 7  4 ] cm 2  Volume of water left in the cylinder
3 7 1
847
1 22 1225 847  825   825  141.97 cm 3
=    11  cm3 6
3 7 100 6
= 683·83 cm3
Radius of cylinder = 5 cm
18
TRIGONOMETRIC IDENTITIES
POINTS TO REMEMBER

Perpendicular PM
1. (i) sin  = 
Hypotenuse OP
Base OM
(ii) cos  = 
Hypotenuse OP
Perpendicular PM
(iii) tan  = 
Base OM
Base OM
(iv) cot  = 
Perpendicular PM
Hypotenuse OP
(v) sec  = 
Base OM
Hypotenuse OP
(vi) cosec  =  .
Perpendicular PM
1 1
2. (i) sin  = and cosec  =  sin  × cosec  = 1
cosec  sin 
1 1
(ii) cos   and sec    cos  × sec  = 1
sec  cos 

1 1
(iii) tan   and cot    tan  × cot  = 1
cot  tan 

sin  cos 
(iv) tan   and cot   .
cos  sin 
3. (i) sin2  + cos2  = 1 (ii) sin2  = 1 – cos2 
(iii) cos2  = 1 – sin2  (iv) 1 + tan2  = sec2 
(v) 1 + cot2  = cosec2  (vi) tan2  = sec2  – 1
(vii) cot2  = cosec2  – 1 (viii) sec2  – tan2  = 1
(ix) cosec2  – cot2  = 1.
4. (i) sin (90º – ) = cos , cos (90º – ) = sin 
(ii) tan (90º – ) = cot , cot (90º – ) = tan 
(iii) sec (90º – ) = cosec , cosec (90º – ) = sec 

528 Arun Deep's Understanding Math-10


529 Arun Deep's Understanding Math-10
EXERCISE 18
3 17
1. If A is an acute angle and sin A = , find all Sol. Given, sec A = (A is an acute angle)
5 8
other trigonometric ratios of angle A (using In right ABC
trigonometric identities).
AC 17
3 sec A = =
Sol. Given, sin A = AB 8
5
 AC = 17, AB = 8
In ABC, B = 90° Using Pythagoras Theorem, we have
AC = 5 and BC = 3
BC = AC 2  AB2 = 17 2  8 2
C
= 289  64 = 225 = 15

C
5
3

17
15
A B

 AB = AC 2  BC 2 = 5 2  32 A 8 B
= 25  9 = 16 = 4
BC 15
AB 4 Now sin A = =
Now, cos  = = AC 17
AC 5
1 8
BC 3  cos A = sec A =
tan  = = 17
AB 4
BC 15
1 4 tan A = =
cot  = tan θ = AB 8
3
1 8
1 5 cot A = tan A =
15
sec  = =
cos θ 4
1 17
1 5 and cosec A = =
sin A 15
and cosec  = sin θ =
3 P.Q. Express the ratios cos A, tan A and sec A in
terms of sin A.
17
2. If A is an acute angle and sec A = , find all
8 Sol. Since, cos A = 1  sin 2 A
other trigonometric ratios of angle A (using
[ cos2 + sin2 = 1]
trigonometric identities).
530 Arun Deep's Understanding Math-10
sin A sin A 3. If 12 cosec  = 13, find the value of
 tan A = =
cos A 1  sin A 2 2sinθ  3cosθ
.
4sinθ  9cosθ
1 1
and sec A = = 13
cos A 1  sin 2 A Sol. Given 12 cosec  = 13  cosec  =
12
1 In right ABC, A = 
P.Q. If tan A = , find all other trigonometric
3 AC 13
cosec  = =
ratios of angle A. BC 12
1 C
Sol. Given, tan A =
3
In right ABC, we have
BC 1 13
12
tan A = =
AB 3
 BC = 1, AB = 3
A 5 B
 AC = AB 2  BC 2 = ( 3 ) 2  (1) 2
 AC = 13, BC = 12
= 3 1 = 4 =2
 AB = AC 2  BC 2 = 13 2  12 2
C
= 169  144 = 25 = 5
BC 12
Now sin  = =
2 AC 13
1
AB 5
cos  = =
AC 13

A 3 B 12 5
2   3
BC 1 2sinθ  3cosθ 13 13
 sin A = = Now =
AC 2 4sinθ  9cosθ 12
4  9
5
13 13
AB 3
cos A = = 24 15 9
AC 2 
13 13 13 9 13
1 = = = × =3
48 45 3 13 3
cot A = tan A = 3 
13 13 13
1 2 Without using trigonometric tables, evaluate
sec A = cos A = the following (6 to 10) :
3
1 2 tan 36
and cosec A = = =2 4. (i) cos2 26° + cos 64° sin 26° + cot 54
sin A 1
531 Arun Deep's Understanding Math-10

sec 17 tan 68


(ii) cosec 73 + cot 22 + cos2 44° + cos2 46°

tan 36
Sol. (i) cos2 26° + cos 64° sin 26° + cot 54

tan 36
cos2 26° + cos (90° – 26°) sin 26° + cot (90  36)

tan 36
= cos2 26° + sin2 26° + tan 36 = 1 + 1 = 2

[ cos (90° – ) = sin  and cot (90° – ) = tan , sin2  + cos2  = 1]


sec 17 tan 68
(ii) cosec 73 + cot 22 + cos2 44° + cos2 46°

sec (90  73) tan (90  22)


= cosec (73) + cot 22 + cos2 (90° – 46°) + cos2 46°

cosec 73 cot 22


= cosec 73 + cot 22 + sin2 46° + cos2 46° ( sin2  + cos2  = 1)

=1+1+1=3 [ sec (90° – ) = cosec  and tan (90° – ) = cot ]


sin 65 cos 32
5. (i) cos 25 + sin 58 – sin 28° sec 62° + cosec2 30° (2015)

sec 29
(ii) + 2 cot 8° cot 17° cot 45° cot 73° cot 82° – 3(sin2 38° + sin2 52°).
cosec 61

sin 65 cos 32


Sol. (i) cos 25 + sin 58 – sin 28° sec 62° + cosec2 30°

sin 65 cos 32


= cos (90  65) + sin (90  32) – sin 28° × sec (90° – 28°) + (2)2

sin 65 cos 32


= + – sin 28° × cosec 28° + 4
sin 65 cos 32

=1+1–1+4=6–1=5 sin (90  θ)  cosθ, cos(90  θ)  sin θ and sec (90° – θ) = cosec θ
sec 29
(ii) cosec 61 + 2 cot 8° cot 17° cot 45° cot 73° cot 82° – 3(sin2 38° + sin2 52°)

sec 29
= cosec (90  29) + 2 cot 8° cot 82° cot 17° cot 73° cot 45° – 3(sin2 38° + sin2 (90° – 38°)
532 Arun Deep's Understanding Math-10
sec 29
= sec 29 + 2 cot 8° cot (90° – 8°) cot 17° cot (90° – 17°) cot 45° – 3(sin2 38° + cos2 38°)

= 1 + 2 cot 8° tan 8° cot 17° cot 17° cot 45° – 3(1)


{sec (90° – ) = cosec , cot  = tan (90° – ) and sin2  + cos2  = 1}
=1+2×1×1×1–3 (cot 45° = 1)
=1+2–3=0
sin 35 cos 55  cos 35 sin 55
6. (i) cosec 2 10  tan 2 80 (2010)

(ii) sin2 34° + sin2 56° + 2 tan 18° tan 72° – cot2 30° (2014)
sin 35 cos 55  cos 35 sin 55
Sol. (i) cosec 2 10  tan 2 80

sin 35 cos (90  35)  cos 35 sin (90  35)
= cosec 2 10  tan 2 (90  10)

sin 35 sin 35  cos 35 cos 35


= cosec 2 10  cot 2 10

sin 2 35  cos 2 35 1  sin 2 θ  cos 2 θ  1 


= = =1  
cosec 10  cot 10
2 2
1  cosec 2 θ  cot 2 θ  1
(ii) sin2 34° + sin2 56° + 2 tan 18° tan 72° – cot2 30°
= sin2 34° + sin2 (90° – 34°) + 2 tan 18° tan (90° – 18°) – cot2 30°
= sin2 34° + cos2 34° + 2 tan 18° cot 18° – cot2 30°
1
= 1 + 2 × 1 – ( 3)2 { cot = and sin (90° – ) = cos  and tan (90° – ) = cot }
tan θ
=1+2–3=0
2 2
 tan 25   cot 25 
7. (i)   +   + 2 tan 18° tan 45° tan 72°
 cosec 65   sec 65 
(ii) (cos2 25° + cos2 65°) + cosec  sec (90° – ) – cot  tan (90° – ).
2 2
 tan 25   cot 25 
Sol. (i)   +   + 2 tan 18° tan 45° tan 72°
 cosec 65   sec 65 
2 2
 tan 25   cot 25 
=   +   + 2 tan 18° tan (90° – 18°) tan 45°
 cosec (90  25)   sec (90  25) 
2 2
 tan 25   cot 25 
=   +   + 2 tan 18° cot 18° tan 45°
 sec 25   cosec 25 
2
 sin 25  cos 25   cos 25  sin 25 
=   +   + 2 × 1 × 1
 cos 25  1   sin 25  1 

 sin 2 θ  cos 2 θ  1
= sin2 25° + cos2 25° + 2  
 tan θ cot θ  1 
=1+2=3
(ii) (cos2 25° + cos2 65°) + cosec  sec (90° – ) – cot  tan (90° – )
= [cos2 25° + cos2 (90° – 25°)] + cosec  cosec  – cot  · cot 
= (cos2 25° + sin2 25°) + (cosec2  – cot2 )
=1+1=2
cos 28 cosec 62
P.Q. (i) 2(sec2 35° – cot2 55°) – tan 18 tan 36 tan 30 tan 54 tan 72

cosec 2 (90  θ)  tan 2 θ 2tan 2 30 sec 2 52 sin 2 38


(ii) – .
2(cos 2 48  cos 2 42) cosec 2 70  tan 2 20

cos 28 cosec 62


Sol. (i) 2(sec2 35° – cot2 55°) – tan 18 tan 36 tan 30 tan 54 tan 72

cos 28 cosec (90  28)


= 2[sec2 35° – cot2 (90° – 35°)] – tan 18 tan (90  18) tan 36 tan (90  36) tan 30

cos 28 sec 28


= 2[sec2 35° – tan2 35°] – tan 18 cot 18 tan 36 cot 36 tan 30

 sec 2 θ  tan 2 θ  1
 
= 2(1) –
1
=2–
3
=2–  tan θ cot θ  1 
1 3  cos θ sec θ  1 
11 1
 
3
cosec 2 (90  θ)  tan 2 θ 2(tan 2 30 sec 2 52 sin 2 38)
(ii) –
2(cos 2 48  cos 2 42) cosec 2 70  tan 2 20

sec 2 θ  tan 2 θ 2[tan 2 30 sec 2 52 sin 2 (90  52)


= –
2(cos 2 48  cos 2 (90  48) cosec 2 70  tan 2 (90  70)

 1  2 
2  sec 2 52 cos 2 52
1  3  
= –
2[cos 2 48  sin 2 48] cosec 2 70  cot 2 70
534 Arun Deep's Understanding Math-10

1  1
1 2  1 = cos 2 A × cos2 A = 1 = R.H.S.
= – 3 
2 1 1
9. (i) tan A + cot A = sec A cosec A
(ii) (1 – cos A) (1 + sec A) = tan A sin A.
 sin 2 θ  cos 2 θ  1 
  Sol. (i) tan A + cot A = sec A cosec A
 sec θ  tan θ  1 
1 2 2 2
= –   L.H.S. = tan A + cot A
2 3
 cosec θ  cot θ  1
2 2

sin A cos A sin 2 A  cos 2 A


3 4 1 = cos A + sin A =
sin A cos A
= =
6 6
Prove that following (8 to 26) identities, 1
= = cosec A sec A
where the angles involved are acute angles sin A cos A
for which the trigonometric ratios as
= sec A cosec A = R.H.S.
defined:
8. (i) (sec A + tan A) (1 – sin A) = cos A (ii) (1 – cos A) (1 + sec A) = tan A sin A

(ii) (1 + tan2 A) (1 – sin A) (1 + sin A) = 1. L.H.S. = (1 – cos A) (1 + sec A)

Sol. (i) (sec A + tan A) (1 – sin A) = cos A  1 


L.H.S. = (sec A + tan A) (1 – sin A) = (1 – cos A) 1  
 cos A 
 1 sin A 
=    (1 – sin A) (cos A  1)
 cos A cos A  = (1 – cos A) cos A
(1  sin A)  (1  sin A) 1  sin 2 A
= cos A =
cos A (1  cos A) (1  cos A)
=
cos A
cos 2 A
= = cos A = R.H.S.
cos A 1  cos 2 A sin 2 A
= =
( 1 – sin2 A = cos2 A) cos A cos A
(ii) (1 + tan2 A)(1 – sin A) (1 + sin A) = 1
L.H.S. = (1 + tan2 A) (1 – sin A) (1 + sin A) sin 2 A sin A
= = sin A × cos A
 sin 2 A  cos A

=  1   (1 – sin2 A)
2  { 1 – cos2 A = sin2 A}
 cos A 
= tan A sin A = R.H.S.
cos 2 A  sin 2 A
= × cos2 A 10. (i) cot2 A – cos2 A = cot2 A cos2 A
cos 2 A

 1  sin 2 A  cos 2 A  tan 2 A


(ii) 1 +  secA
  1  sec A
 sin 2 A  cos 2 A  1
535 Arun Deep's Understanding Math-10

1  secA sin 2 A = 1 + cos A


(iii) 
secA 1  cos A
(1 + cos A) (1 – cos A)
=
sin A 1 – cos A
(iv)  cos ec A + cot A
1  cos A
1 – cos 2 A sin 2 A
Sol. (i) cot2 A – cos2 A = cot2 A cos2 A = 
1 – cos A 1  cos A
L.H.S. = cot2 A – cos2 A
= R.H.S.
cos 2 A 2
=  cos A
sin 2 A sin A
(iv)  cos ec A + cot A
1  cos A
cos 2 A – cos 2A sin 2 A
= 1 cos A
sin 2 A L.H.S. = cosec A + cotA = 
sin A sin A

cos 2 A (1 – sin 2 A)
= 1  cos A (1  cos A)(1– cosA)
sin 2 A = 
sin A sin A(1 – cosA)
= cot2A.cos2A = L.H.S.
1 – cos 2 A
tan 2 A =
(ii) 1 +  secA sin A (1 – cosA)
1  sec A

tan 2 A 1  sin A  tan 2 A sin 2 A


L.H.S. = 1 +  =
1  secA 1  secA sin A (1 – cosA)

1  secA + sec2A – 1 sin A


= = 1 – cosA = L.H.S.
1  sec A
[ 1 + tan2 A = sin2A]
sin A 1  cos A
sec A (1 + sec A) 11. (i) 
=  sec A = R.H.S. 1 + cos A sin A
1  sec A
sin A
1  secA sin 2 A Sol. L. H.S. =
(iii)  1 + cos A
secA 1  cos A

sin A (1  cos A)
1 1  cos A 
1 (1+ cos A) (1  cos A)
1  sec A cos A  cos A
L.H.S. = 
sec A 1 1
[Multiplying and dividing by (1 – cos A)]
cos A cos A
536 Arun Deep's Understanding Math-10

sin A (1  cos A) sin A (1  cos A) 1


1
 
1  cos 2 A sin 2 A sec A  1 cos A
Sol. L. H.S.= 
sec A  1 1
(... 1 – cos2 A = sin2 A) 1
cosA
1  cos A
  R. H.S. 1  cos A
sin A
cos A 1  cos A cos A
  ×
1  tan 2 A 1  cos A cos A 1 + cos A
(ii)  tan 2 A
cot 2 A  1 cos A
sin 2 A
1 1  cos A
1  tan 2 A cos 2 A   R. H.S.
Sol. L. H.S.=  1 + cos A
cot 2 A  1 cos 2 A
1
sin 2 A tan 2θ 1  cosθ
(ii) Prove that =
cos 2 A  sin 2 A secθ  12 1  cosθ
cos 2 A cos 2 A  sin 2 A
  tan 2θ
cos 2 A  sin 2 A cos 2 A tan 2 θ
Ans. LHS = =
sin 2 A secθ  12 sec 2θ  1  2secθ
sin 2 A sin 2 θ
×
cos2 A  sin 2 A cos 2 θ sin 2 θ cos 2 θ
= = ×
sin 2 A 1 2 cos 2 θ 1  cos 2 θ  2cosθ
  tan 2 A = R. H.S. 2
1
2
cos A cos θ cosθ

sinA sin 2 θ 1  cos θ


2
1  cosθ 1  cosθ 
(iii) = cosecA – cotA
1  cosA
(2008) =
1  cosθ  2 =
1  cosθ  2 =
1  cosθ 2
Ans. R.H.S. = cosecA – cotA 1  cosθ
= = R.H.S. Proved.
1 cosA 1  cosA 1  cosθ
= – = (iii) (1 + tanA)2 + (1 – tanA)2 = 2sec2A
sinA sinA sinA
Ans. L.H.S. = (1 + tanA)2 + (1 – tanA)2
1  cosA  1  cosA  = 1 + 2 tanA + tan2A + 1 – 2 tanA + tan2A
sinA 1  cosA 
= = 2 + 2 tan2A = 2 (1 + tan2A)
= 2 sec2A ( 1 + tan2A = sec2A)
{Multiplying and dividing by 1 + cosA}
= R.H.S.
1  cos 2 A sin 2 A (iv) sec2 A + cosec2 A = sec2 A cosec2 A
=
sinA1  cosA 
=
sinA1  cosA  Sol. L.H.S. = sec2 A + cosec2 A
1 1 sin 2 A + cos 2 A
{ 1 – cos2A = sin2A}   
2
cos A sin A 2
sin 2 A + cos 2 A
sinA 1
1  cosA
= L.H.S.   sec 2 A cosec 2 A = R.H.S.
sin A cos2 A
2

sec A  1 1  cos A 1  sin A cos A


12. (i)  (2007) 13. (i)   2 sec A
sec A  1 1 + cos A cos A 1 + sin A
537 Arun Deep's Understanding Math-10
1 + sin A

cos A
 cosec A
LM cosec A + 1 + cosec A  1 OP
Sol. L. H.S.=
cos A 1 + sin A N (cosec A  1) (cosec A + 1) Q
cosec A ×2 cosec A 2 cosec 2 A
(1  sin A) (1 + sin A) + cos 2 A  
 cosec 2 A  1 cot 2 A
cos A (1 + sin A)
2 ×sin 2 A 2
1  sin A + sin A + sin 2 A + cos 2 A  
 sin 2 A ×cos 2
A cos 2 A
cos A (1 + sin A)
=2 sec2 A = R.H.S.
1  2 sin A + 1 2  2 sin A 2 cos2 A  1
 
cos A (1 + sin A) cos A (1 + sin A) (ii) cot A – tan A 
sin A  cos A
2 (1  sin A) 2 Sol. L.H.S. = cot A – tan A
   2 sec A
cos A (1 + sin A) cos A cos A sin A cos 2 A  sin 2 A
  
= R.H.S. sin A cosA sin A cos A
tan A tan A cos 2 A  (1  cos 2 A)
(ii)   2 cosec A 
sec   1 sec A + 1 sin A cos A
cos 2 A  1  cos 2 A
tan A tan A 
Sol. L. H.S.=  sin A cos A
sec   1 sec A + 1
2 cos2 A  1
  R. H.S.
 tan A
FG 1  1 IJ sin A cos A
H sec A  1 sec A + 1K cot A  1

cot A
(iii)
2 sec 2 A 1 + tan A
 tan A G
F sec A + 1 + sec A  1 IJ cos A
H (sec A  1) (sec A + 1) K cot A  1 sin A
1

tan A ×2 sec A 2 sec A tan A 2 sec A Sol. L. H.S.= 2  sec 2 A 1
   2
sec 2 A 1 tan 2 A tan A cos 2 A
cosA  sin A
2 ×1×cos A
 sin A
cos A ×sin A 
2 cos2 A  1
2 cos2 A
= = 2 cosec A = R.H.S. cos A  sin A cos2 A
sin A
 ×
sin A 2 cos2 A  1
cosec A cosec A
14. (i)   2 sec 2 A cos2 A (cosA  sin A)
cosec A  1 cosec A + 1 
sin A (2 cos2 A  1)
cosec A cosec A
Sol. L. H.S.   cos2 A (cos A  sin A)
cosec A  1 cosec A  1 
sin A [2 cos 2 A  ( sin 2 A + cos2 A)]

 cosec A
LM 1  1 OP 
cos 2 A (cosA  sin A)
N cosec A  1 cosec A + 1 Q sin A [2 cos 2 A  sin 2 A  cos 2 A]
538 Arun Deep's Understanding Math-10

cos2 A (cosA  sin A) cos  sin 2 


 Sol. L. H.S.= 
sin A (cos2 A  sin 2 A) 1  tan  cos   sin 

cos 2 A (cos A  sin A) cos  sin 2 


  
sin  cos   sin 
sin A (cos A  sin A) (cos A  sin A) 1
cos 
cos 2 A
 cos  sin 2 
sin A (cos A  sin A)  
cos   sin  cos   sin 
cos A cos 
cot A sin A cos2  sin 2 
R. H.S.    
1 + tan A sin A
1 cos   sin  cos   sin 
cos A
cos 2   sin 2 

cos A cos   sin 
sin A
 ( cos  + sin  ) (cos   sin  )
cos A + sin A 
cos   sin 
cos A
= cos  + sin  = R.H.S.
cos A cos A 16. (i) cosec4  – cosec2  = cot4  + cot2 
 ×
sin A cos A + sin A (ii) 2 sec2  – sec4  – 2 cosec2  + cosec4  =
cot4  – tan4 .
cos 2 A Sol. (i) cosec4  – cosec2  = cot4  + cot2 

sin A (cos A + sin A) L.H.S. = cosec4  – cosec2 
= cosec2  (cosec2  – 1)
 L.H.S. = R.H.S.
= cosec2  cot2 
15. (i) tan2  – sin2  = tan2  sin2  {cosec2  – 1 = cot2 )
Ans. L.H.S. = tan2  – sin2  = (cot2  + 1) cot2 
= cot4  + cot2 
sin 2 
  sin 2  = R.H.S.
cos 2  (ii) 2 sec2  – sec4  – 2 cosec2  + cosec4 
= cot4  – tan4 
sin 2   sin 2  cos 2 
 L.H.S. = 2 sec  – sec  – 2 cosec2  +
2 4
cos 2  cosec4 
= 2(tan 2  + 1) – (tan 2  + 1)2 – 2(1 +
sin 2  (1  cos 2  ) sin 2  cot2 ) + (1 + cot2 )2
  sin 2  ×
cos 2  cos 2 
 sec 2 θ  tan 2 θ  1 
sin2  × tan2 = tan2  sin2  = R.H.S.  
 cosec 2 θ  1  cot 2 θ 
cos  sin 2  = 2 tan2  + 2 – (tan4  + 2 tan2  + 1) – 2 –
(ii)  = cos  + sin 
1  tan  cos   sin  2 cot2  + (1 + 2 cot2  + cot4 )
539 Arun Deep's Understanding Math-10
=2 tan2 +2– –2
tan4 tan2 –1–2–2 cot2  + 1 + 2 cot2  + cot4 
= cot4  – tan4  = R.H.S.

1  cos θ  sin 2 θ
17. (i) = cot 
sin θ(1  cos θ)

tan 3 θ  1
(ii) = sec2  + tan .
tan θ  1

1  cos θ  sin 2 θ
Sol. (i) = cot 
sin θ(1  cos θ)

1  cos θ  sin 2 θ cos θ  cos 2 θ


L.H.S. = = { 1 – sin2  = cos2 }
sin θ(1  cos θ) sin θ (1  cos θ)

cos θ (1  cos θ) cos θ


= sin θ (1  cos θ) = sin θ = cot 

= R.H.S.

tan 3 θ  1
(ii) = sec2  + tan 
tan θ  1

(tan θ  1)
L.H.S. = tan θ  1 (tan2  + tan  + 1) { a3 – b3 = (a – b) (a2 + ab + b2)}

= tan2  + tan  + 1 = tan2  + 1 + tan 


= sec2  + tan  { sec2  = tan2  + 1}
= R.H.S.

1  cosec A cos 2 A
18. (i) =
cosec A 1  sin A

1  cos A sin A
(ii) = 1  cos A (2013)
1  cos A

1  cosec A cos 2 A
Sol. (i) cosec A =
1  sin A

1
1
1  cosec A sin A sin A  1 sin A
L.H.S. = cosec A = 1 = sin A ×
1
sin A

= sin A + 1
540 Arun Deep's Understanding Math-10

cos 2 A 1  sin 2 A (1  sin A) 2 (1  sin A) 2


R.H.S. = = = =
1  sin A 1  sin A 1  sin 2 A cos 2 A

(1  sin A) (1  sin A) 1  sin A 1 sin A


= 1  sin A = 1 + sin A = sin A + 1 = = +
cos A cos A cos A
 L.H.S. = R.H.S. = sec A + tan A = tan A + sec A = R.H.S.

1  cos A 1  cos A
(ii) = cosec A – cot A (ii) = cosec A – cot A
1  cos A 1  cos A

1  cos A 1  cos A
L.H.S. =
L.H.S. =
1  cos A 1  cos A

Rationalising the denominator Rationalising the denominator,

(1  cos A) (1  cos A)
(1  cos A) (1  cos A) =
= (1  cos A) (1  cos A)
(1  cos A) (1  cos A)

(1  cos A) 2 (1  cos A) 2 1  cos A


(1  cos A) 2 (1  cos A) 2 = = = sin A
= = 1  cos 2 A 2
sin A
1  cos 2 A sin 2 A
1 cos A
1  cos A 1 cos A = –
sin A sin A
= cosec A – cot A = R.H.S.
= sin A = sin A  sin A

= cosec A – cot A = R.H.S. sec A  1 sec A  1


20. (i) + = 2 cosec A
sec A  1 sec A  1
1  sin A
19. (i) = tan A + sec A cos A cot A
1  sin A (ii) 1  sin A = 1 + cosec A.
1  cos A
(ii) = cosec A – cot A. sec A  1 sec A  1
1  cos A Sol. (i) + = 2 cosec A
sec A  1 sec A  1

1  sin A sec A  1 sec A  1


Sol. (i) = tan A + sec A
1  sin A L.H.S. =
sec A  1
+
sec A  1

1  sin A sec A  1 sec A  1


L.H.S. =
1  sin A =
sec A  1
+
sec A  1
Rationalising the denominator,
sec A  1  sec A  1 2 sec A
(1  sin A) (1  sin A) = (sec A  1) (sec A  1) = sec 2 A  1
=
(1  sin A) (1  sin A) { sec2 A – 1 = tan2 A}
541 Arun Deep's Understanding Math-10

=
2 sec A 2 sec A
= tan A 2
LM cos A  sin A OP
2
tan A N cos A sin A cos A sin A Q
2  cos A 2 L 1  1 OP
= cos A  sin A = sin A  2M

= 2 cosec A = R.H.S.
N sin A cos A Q
= 2 (cosec A + sec A)
cos A cot A
(ii) 1  sin A = 1 + cosec A = 2 (sec A + cosec A) = R.H.S.
(ii) sec4 A – tan4 A = 1 + 2 tan2 A
cos A cot A cos A cos A L.H.S. = sec4 A – tan4 A
L.H.S. = 1  sin A = sin A (1  sin A) = (sec2 A – tan2 A) (sec2 A + tan2 A)
= (1 + tan2 A – tan2 A) (1 + tan2 A + tan2 A)
 cos A  { sec2 A = tan2 A + 1}
cos A  
 sin A  = 1(1 + 2 tan2 A) = 1 + 2 tan2 A = R.H.S.
22. (i) cosec6 A – cot6 A = 3 cot2 A cosec2 A + 1
cos 2 A 1  sin 2 A
=
sin A (1  sin A)
=
sin A (1  sin A) Sol. L.H.S. = cosec6 A – cot6 A
{ cos2 A = 1 – sin2 A} = (cosec2 A)3 – (cot2 A)3
= (cosec2  – cot2 A)3
(1  sin A) (1  sin A) 1  sin A
= = sin A + 3 cosec2 A cot2 A (cosec2 A – cot2 A)
sin A (1  sin A)
= (1)3 + 3 cosec2 A cot2 A × 1
1 sin A = 1 + 3 cot2 A cosec2 A
= sin A + sin A = cosec A + 1
= 3 cot2 A cosec2 A + 1 = R.H.S.
= 1 + cosec A = R.H.S. (ii) sec6 A – tan6 A = 1 + 3 tan2 A + 3 tan4 A
1  tan A 1  cot A Sol. L.H.S. = sec6 A – tan6 A
21. (i) 
sin A cos A = (sec2 A)3 – (tan2 A)3
= 2 (sec A + cosec A) = (sec2 A – tan2 A)3
(ii) sec4 A– tan4 A = 1 + 2 tan2 A. + 3 sec2 A tan2 A (sec2 A – tan2 A)
1  tan A 1  cot A = (1)3 + 3 sec2 A tan2 A × 1
Sol. (i) L. H.S.= 
sin A cos A = 1 + 3 sec2 A tan2 A
sin A cos A = 1 + 3 [(1 + tan2 A) (tan2 A)]
1 1
cos A sin A = 1 + 3 [tan2 A + tan4 A]
 
sin A cos A = 1 + 3 tan2 A + 3 tan4 A = R.H.S.

cos A + sin A sin A + cos A cot  + cosec   1 1  cos 


  23. (i) 
cos A  sin A cos A  sin A cot   cosec  + 1 sin 

LM cos A + sin A OP cot  + cosec   1


2 Sol. L. H.S.=
cot   cosec   1
N cos A sin A Q
542 Arun Deep's Understanding Math-10

cos  1 sin  sin 


 1 Sol. L.H.S.  cos   sin   cos   1
sin  sin 
 sin  sin 
cos  1
 1
sin  sin 
sin 2 

cos   1  sin  sin  1  cos 
 ×
sin  cos   1  sin 
1  cos 2  (1  cos  ) (1  cos  )
 
cos   1  sin  1  cos  1  cos 

cos   1  sin 
 1  cos
cos   (1  sin  )
 sin 
cos   (1  sin  ) R.H.S.  2 
cot   cosec 
[cos   (1  sin  )][cos   (1  sin  )]
 sin 
[cos   (1  sin )][cos   (1  sin  )] 2 
cos  1

[ cos  + (1  sin  )] 2 sin  sin 

cos 2   (1  sin  ) 2
sin  sin 2 
( cos  + 1  sin  ) 2  2 2 
 cos   1 cos   1
cos 2   (1  sin 2   2 sin  ) sin 

cos 2   sin 2   1  2 cos   2 sin   2 sin  cos  2 cos   2  sin 2 



cos 2   1  sin 2   2 sin  
cos   1
1  1  2 cos   2 sin   2 sin  cos 
 2 cos   2  (1  cos 2  )
1  sin 2   1  sin 2   2 sin  
cos   1
2  2 cos   2 sin   2 sin  cos 
 2 (cos   1)  (1  cos  ) (1  cos  )
2 sin   2 sin 2  
cos   1
2 (1  cos  )  2 sin  (1  cos  )
 2 (cos   1)  (cos   1) (cos   1)
2 sin  (1  sin  ) 
cos   1
(1  cos  ) 2 (1  sin  )
 (cos   1) ( 2  cos   1)
2 sin  (1  sin  )  1  cos 
cos   1
1  cos   L.H.S. = R.H.S.
  R. H.S.
sin  24. (i) (sin  + cos ) (sec  + cosec )
sin  sin  = 2 + sec  cosec 
(ii) 2
cot   cosec  cot   cosec  Sol. L.H.S. = (sin  + cos ) (sec  + cosec )
543 Arun Deep's Understanding Math-10

FG 1  1 IJ cos 2  sin 2 
 ( sin  + cos  )  × (tan   cot  )
H cos  sin  K sin  cos 


(sin   cos  ) (sin   cos  )
 sin  cos 
FG sin   cos  IJ
sin  cos  H cos  sin  K
sin 2 θ  sin θ cos θ  sin θ cos θ  cos 2 θ sin 2   cos 2 
 sin  cos 
=
sin θ cos θ sin  cos 
= 1 = R.H.S.
1  2 sin θ cos θ 1 2 sin θ cos θ
= = + sin 3A + cos3 A sin 3A  cos 3 A
sin θ cos θ sin θ cos θ sin θ cos θ 25. (i) + 2
sin A + cos A sin A  cos A
= cosec  sec  + 2 Sol. L.H.S.
= 2 + sec  cosec .
(sin A  cos A) (sin 2 A  sin A cos A  cos2 A)
(ii) (cosec A – sin A) (sec A – cos A) sec2 A 
(sin A + cos A)
= tan A
Sol. L.H.S. (sin A  cos A) (sin 2 A + sin A cos A + cos 2 A)

(cosec A – sin A) (sec A – cos A) sec2 A (sin A  cos A)
= (1 – sin A cos A) + (1 + sin A cos A)
 1  1  1 [ sin2 A + cos2 A = 1}
=   sin A    cos A  2
 sin A   cos A  cos A = 1 – sin A cos A + 1 + sin A cos A = 2 = R.H.S.
tan 2 A cot 2 A
 1  sin 2 A   1  cos 2 A  1 (ii)  1
=  sin A   
 cos A  cos 2 A 1 + tan 2 A 1 + cot 2 A
   
tan 2 A cot 2 A
Sol. L. H.S.= 
cos 2 A sin 2 A 1 sin A 1 + tan 2 A 1 + cot 2 A
· · 2 = cos A = tan A
sin A cos A cos A 1
2
R.H.S. =
tan A
 tan 2 A
1 + tan 2 A 1+
1
(iii) (cosec  – sin ) (sec  – cos )
tan 2 A
(tan  + cot ) = 1
Sol. L.H.S. = (cosec  – sin ) 1
tan 2
A 2
(sec  – cos ) (tan  + cot )   tan A
1 + tan 2 A tan 2 A  1


FG 1  sin IJ FG 1  cos IJ tan 2 A
H sin  K H cos  K tan 2 A tan 2 A
 
(tan  + cot ) 1  tan 2 A tan 2 A (tan 2 A  1)
1  sin 2  1  cos2  tan 2 A 1
  (tan   cot  )  
sin  cos  1  tan A 1  tan 2 A
2
544 Arun Deep's Understanding Math-10

1  tan 2 A  sin A
  1  R. H.S.   tan A
1 + tan 2 A cos A
 L.H.S. = R.H.S.
1 1
26. (i)  (ii) (sin A + sec A)2 + (cos A + cosec A)2
sec A + tan A cos A
= (1 + sec A cosec A)2
1 1 L.H.S. = (sin A + sec A)2 + (cos A + cosec A)2
 
cos A sec A  tan A = sin2 A + sec2 A + 2sin A sec A + cos2 A +
(ii) (sin A + sec A)2 + (cos A + cosec A)2 = (1 + cosec2 A + 2cos A cosec A
sec A cosec A)2 = (sin2 A + cos2 A) + (sec2 A + cosec2 A) +

tan A  sin A sec A  1 1 1


(iii) tan A  sin A = sec A  1 2sin A × cos A + 2 × cos A × sin A

1 1  1 1  2sin 2 A  2cos 2 A
Sol. (i) L. H.S.=  =1+  2
 2  +
sec A + tan A cos A  cos A sin A  sin A cos A

1 1  sin 2 A  cos 2 A 
  2[sin 2 A  cos 2 A]
1

sin A cos A =1+  2 2  +
cos A cos A  cos A sin A  sin A cos A

cos A 1 1 2
  =1+ 2 2
cos A sin A
+
sin A cos A
1 + sin A cos A
[ sin2  + cos2  + 1]
cos 2 A  1  sin A  sin 2  sin A
  2
cos A (1 + sin A) cos A (1 + sin A)  1 
= 1  
 sin A (1 + sin A)  cos A sin A 
 = – tan A
cos A (1 + sin A) [ (a + b)2 = a2 + (b)2 + 2ab)]
= (1 + cosec A sec A)2
1 1
R. H.S.=  = R.H.S.
cos A sec A  tan A
tan A  sin A sec A  1
1 1 (iii) tan A  sin A = sec A  1
= 
cos A 1 sin A

cos A cos A sin A
 sin A
1 cos A tan A  sin A cos A
  L.H.S. = tan A  sin A = sin A
cos A 1  sin A  sin A
cos A
1  sin A  cos 2 A sin 2 A  sin A
 
cos A (1  sin A) cos A (1  sin A) sin A  sin A cos A
cos A
 sin A + sin 2 A  sin A (1  sin A) = sin A  sin A cos A
 
cos A (1  sin A) cos A (1  sin A) cos A
545 Arun Deep's Understanding Math-10

sin A (1  cos A) 1  cos A


= sin A (1  cos A) = 1  cos A

Dividing each term by cos A, we have

1
1
cos A sec A  1
= 1 = sec A  1 = R.H.S.
1
cos A

27. If sin  + cos  = 2 sin (90° – ), show that cot  = 2 + 1.


Sol. Given, sin  + cos  = 2 sin (90° – )
sin  + cos  = 2 cos 
Dividing by sin , we have
1 + cot  = 2 cot 
 1= 2 cot  – cot 
 1 = ( 2 – 1) cot 

1 1  ( 2  1)
 cot  = = (Rationalising the denominator)
2 1 ( 2  1) ( 2  1)

( 2  1) 2 1 2 1
= = =
( 2 )  (1)
2 2
2 1 1
= 2 + 1 = R.H.S.
28. If 7 sin2  + 3 cos2  = 4, 0° <  < 90°, then find the value of .
Sol. Given, 7 sin2  + 3 cos2  = 4 <  < 90°
 3 sin2  + 3 cos2  + 4 sin2  = 4
 3(sin2  + cos2 ) = 4 – 4 sin2 
3
 3 × 1 = 4(1 – sin2 )  = cos2 
4

3
 cos  = = cos 30°
2
  = 30°
29. If sec  + tan  = m and sec  – tan  = n, prove that mn = 1.
Sol. sec  + tan  = m and sec  – tan  = n
 mn = (sec  + tan ) (sec  – tan )
= sec2  – tan2  = 1 (sec2  – tan2  = 1)
Hence proved.
546 Arun Deep's Understanding Math-10
34. If x = a sec  + b tan  and y = a tan  + b sec , prove that x2 – y2 = a2 – b2.
Sol. x = a sec  + b tan  and y = a tan  + b sec 
To prove : x2 – y2 = a2 – b2
x2 – y2 = (a sec  + b tan )2 – (a tan  + b sec )2
= (a2 sec2  + b2 tan2  + 2ab sec  tan ) – (a2 tan2  + b2 sec2  + 2ab sec  tan )
= a2 sec2  + b2 tan2  + 2ab sec  tan  – a2 tan2  – b2 sec2  – 2ab sec  tan 
= a2 (sec2  – tan2 ) – b2 (sec2  – tan2 )
= a2 × 1 – b2 × 1 {sec2  – tan2  = 1}
= a2 – b2
Hence proved.
35. If x = h + a cos  and y = k + a sin , prove that (x – h) + (y – k) = a .
2 2 2

Sol. x = h + a cos  and y = k + a sin 


To prove : (x – h)2 + (y – k)2 = a2
(x – h) = a cos  ...(1)
(y – k) = a sin  ...(2)
Squaring and adding, eqn. (1) and (2) ; we have
(x – h)2 + (y – k)2 = a2 cos2  + a2 sin2 
= a2 (sin2  + cos2 )
= a2 × 1 = a2 { sin2  + cos2  = 1)
MULTIPLE CHOICE QUESTIONS

Choose the correct answer from the given four


1  cos 2 θ sin 2 θ  1
options (1 to 12) : = ×
cos 2 θ sin 2 θ
1
1. cot2  – sin 2 θ is equal to  sin 2 θ cos 2 θ
= = –1 Ans. (a)
sin 2 θ cos 2 θ
(a) 1 (b) –1
( sin2  + cos2  = 1)
(c) sin2  (d) sec2 
tan 2 θ
1 cos 2 θ 1 3. is equal to
Sol. cot  – sin 2 θ =
2
2 – sin 2 θ 1  tan 2 θ
sin θ
(a) 2 sin2  (b) 2 cos2 
cos 2 θ  1  sin 2 θ (c) sin2  (d) cos2 
= = –1 Ans. (b)
sin 2 θ sin 2 θ sin 2 θ sin 2 θ
2. (sec2  – 1) (1 – cosec2 ) is equal to cos 2 θ cos 2 θ
tan 2 θ
(a) –1 (b) 1 Sol. = sin 2 θ = cos 2 θ  sin 2 θ
1  tan 2 θ 1
(c) 0 (d) 2 cos 2 θ cos 2 θ
Sol. (sec2  – 1) (1 – cosec2 )

 1   1  sin 2 θ cos 2 θ
=  1 1   = 2 ×
2  sin 2 θ  cos θ sin θ  cos 2 θ
2
 cos θ   
( sin2  + cos2  = 1)
547 Arun Deep's Understanding Math-10

sin 2 θ 1 sin 2 A
= = sin2  Ans. (c) = 2 ×
1 cos A 1
4. (cos  + sin )2 + (cos  – sin )2 is equal to
sin 2 A
(a) –2 (b) 0 = = tan2 A Ans. (d)
(c) 1 (d) 2 cos 2 A
Sol. (cos  + sin )2 + (cos  – sin )2 7. If sec  – tan  = k, then the value of sec  +
= cos2  + sin2  + 2 sin  cos  + cos2  + tan  is
sin2  – 2 sin  cos  1
= 2(sin2  + cos2 ) = 2 × 1 = 2 Ans. (d) (a) 1 – (b) 1 – k
k
( sin2  + cos2  = 1)
1
5. (sec A + tan A ) (1 – sin A) is equal to (c) 1 + k (d)
(a) sec A (b) sin A k
(c) cosec A (d) cos A Sol. Given, sec  – tan  = k
Sol. (sec A + tan A ) (1 – sin A) 1 sin θ
cos θ – cos θ = k
 1 sin A 
=  cos A  cos A  (1 – sin A)
  1  sin θ
cos θ = k
1  sin A
= cos A × 1 – sin A Squaring both sides, we get
2
(1  sin A) (1  sin A)  1  sin θ  (1  sin θ) 2
  = (k)2  = k2
= cos A  cos θ  cos 2 θ

1  sin 2 A cos 2 A (1  sin θ) 2


= = = cos A  Ans. (d) = = k2
cos A cos A 1  sin 2 θ

1  tan 2 A (1  sin θ) 2
6. is equal to  = k2
1  cot 2 A (1  sin θ) (1  sin θ)
(a) sec2 A (b) –1 1  sin θ
(c) cot2 A (d) tan2 A = 1  sin θ = k2

sin 2 A 1  sin θ 1 (1  sin θ)


1 1
1  tan 2 A cos 2 A  1  sin θ = 2  =
k 1  sin θ k
Sol. =
1  cot 2 A cos 2 A
1 Multiplying and dividing by (1 + sin )
sin 2 A
 (1  sin θ) 2  1
=   =
cos A  sin A
2 2
1  1  sin θ 
2
k
cos 2 A cos 2 A
= =
sin 2 A  cos 2 A 1  (1  sin θ) 2  1
2
sin A sin 2 A   2  =
 cos θ  k
548 Arun Deep's Understanding Math-10

1  sin θ 1 sin θ Sol. 3 tan2


26° – 3 cosec2 64°
1 1
= = + = = 3 tan2 26° – 3 cosec2 (90° – 26°)
cos θ k cos θ cos θ k
= 3 tan2 26° – 3 sec2 26°
1 = 3(tan2 26° – sec2 26°) = 3 × (–1) = –3
sec  + tan  = (d)
k { sec2  – tan2  = 1}
Alter : Given sec  – tan  = k  Ans. (c)
sec2 – tan2 = 1  (sec  + tan ) sin (90  θ)sin θ
1 P.Q. The value of tan θ – 1 is
(sec  – tan ) = 1 sec  + tan  =
k (a) –cot  (b) –sin2 
P.Q. Which of the following is true for all values (c) –cos2  (d) –cosec2 
of  (0° <  < 90°): sin (90  θ)sin θ cos θ sin θ
(a) cos2  – sin2  = 1 Sol. tan θ –1=
sin θ
–1
(b) cosec2  – sec2  = 1 cos θ
(c) sec2  – tan2  = 1
(d) cot2  – tan2  = 1 sin θ cos θ  cos θ
= sin θ – 1 = cos2  – 1
Sol. sec2  – tan2  = 1 is true for all values of
 as it is an identity. (0° <  < 90°) (c) = –(1 – cos2 ) = –sin2  (b)
8. If  is an acute angle of a right triangle, then
the value of CHAPTER TEST
sin  cos (90° – ) + cos  sin (90° – ) is
1. (i) If  is an acute angle and cosec  = 5,
(a) 0 (b) 2 sin  cos 
(c) 1 (d) 2 sin2  find the value of cot  – cos .
Sol. sin  cos (90° – ) + cos  sin (90° – ) = 8
sin  sin  + cos  cos  (ii) If  is an acute angle and tan  = , find
15
 sin (90  θ)  cos θ  the value of sec  + cosec .
 
 cos (90  θ)  sin θ  Sol. (i) Given, is an acute angle.
= sin2  + cos2  = 1 (c) s.t. cosec  = 5
9. The value of cos 65° sin 25° + sin 65°
cos 25° is 1
 sin  =
(a) 0 (b) 1 (c) 2 (d) 4 5
Sol. cos 65° sin 25° + sin 65° cos 25°
2
= cos (90° – 25°) sin 25° + sin (90° – 25°)  1 
cos 25°  cos  = + 1  sin θ =
2 1   
 5
= sin 25°·sin 25° + cos 25°·cos 25° = sin2
25° + cos2 25° 1 5 1 4 2
( sin2  + cos2  = 1) = 1 = = =
5 5 5 5
=1 (b)
10. 2 2
The value of 3 tan 26° – 3 cosec 64° is cos θ
Now cot  – cos  = – cos 
(a) 0 (b) 3 (c) –3 (d) –1 sin θ
549 Arun Deep's Understanding Math-10

2
5 2 2 2  1  2( 5  1)
= 1 – = – = 2 1   =
5 1 5  5 5
5

8 BC 8 C
(ii) Given tan  = ; In the figure, tan  = =
15 AB 15
 BC = 8, AB = 15
8
 AC = AB 2  BC 2 = 52  82 = 225  64 = 289 = 17
AC 17 AC 17
 sec  = = and cosec  = =
AB 15 BC 8
A 15 B
17 17 136  255 391 31
 sec  + cosec  = + = = =3
15 8 120 120 120
2. Evaluate the following :
 cos 2 20  cos 2 70 
(i) 2 ×   – tan 45° + tan 13° tan 23° tan 30° tan 67° tan 77°

 sin 2
25  sin 2
65 

sin 2 22  sin 2 68


(ii) + sin2 63° + cos 63° sin 27°
cos 2 22  cos 2 68

sec 29
(iii) + 2 cot 8° cot 17° cot 45° cot 73° cot 82° – 3(sin2 38° + sin2 52°)
cos ec61

 cos 2 20  cos 2 70 


Sol. (i) 2 ×   – tan 45° + tan 13° tan 23° tan 30° tan 67° tan 77°

 sin 2
25  sin 2
65 

 cos 2 (90  70)  cos 2 70 


= 2 2  – tan 45° + tan 13° tan 77° tan 23° tan 67° tan 30°
 sin 25  sin (90  25) 
2

 sin 2 70  cos 2 70  1


= 2 2  – 1 + tan 13° tan (90° – 13°) tan 23° tan (90° – 23°) ×
 sin 25   cos 2
25  3

1 1
= 2   – 1 + tan 13 cot 13° tan 23° cot 23° × { tan (90° – ) = cot }
1 3

1 1 1 3 1 ( 3  1) 3 3 3
=2–1+1×1× =2–1+ =1+ = = =
3 3 3 3 3 3 3

sin 2 22  sin 2 68


(ii) + sin2 63° + cos 63° sin 27°
cos 2 22  cos 2 68
550 Arun Deep's Understanding Math-10

sin 2 22  sin 2 (90  22)


= + sin2 63° + cos 63° sin (90° – 63°)
cos 2 22  cos 2 (90  22)

sin 2 22  cos 2 22 1


= + sin2 63° + cos 63° × cos 63° = + (sin2 63° + cos2 63°) = 1 + 1 = 2
cos 22  sin 22
2 2
1
[ sin (–) = cos
sec 29
(iii) + 2 cot 8° cot 17° cot 45° cot 73° cot 82° – 3(sin2 38° + sin2 52°)
cos ec61
sec 29
= cos ec(90°  29) + 2 cot 8° × cot (90 – 8°) cot 17° × cot (90° – 17°) cot 45° – 3(sin2 38° + sin2 52°)

sec 29
= + 2 cot 8° × tan 8° × cot 17° tan 17° × 1 – 3(sin2 38° + sin2 52°)
s ec29
=1+2×1×1×1–3×1=1+2–3=0
4 2 x
3. If (sec2 59° – cot2 31°) – sin 90° + 3 tan2 56° tan2 34° = , then find the value of x.
3 3 3
4 2 x
Sol. Given (sec2 59° – cot2 31°) – sin 90° + 3 tan2 56° tan2 34° =
3 3 3
4 2 x
 [sec2 59° – cot2 (90° – 59°)] – sin 90° + 3 tan2 56° tan2 (90° – 56°) =
3 3 3
4 2 x
 [sec2 59° – tan2 59°] – × 1 + 3tan2 56° cot2 56° =
3 3 3
[ tan (–) = cot and cot (–) = tan ]
4 2 x 4 2 x 429 x 11 x
 ×1– +3×1=  – +3=  =  =
3 3 3 3 3 3 3 3 3 3
11 3
 x= = 111  x = 11
3
Prove the following (4 to 11) identities, where the angles involved are acute angles for which the
trigonometric ratios are defined :
cos A cos A cos A cos A
4. (i)   2 sec A (ii)   2 tan A
1  sin A 1 + sin A cosec A + 1 cosec A  1

Sol. L.H.S.=
cos A

cos A  cos A 1 LM 
1 OP  cosA LM 1 + sin A + 1  sin A OP
1  sin A 1 + sin A 1  sin A 1  sin A
N Q N (1  sin A) (1 + sin A) Q
 cosA
LM 2 OP  2 cos A 
2
 2 sec A = R.H.S.
N 1  sin2A Q cos2A cos A

(ii) L.H.S.=
cos A

cos A  cosA
1LM 
1 OP
cosec A + 1 cosec A  1 cosec A + 1 cosec A  1
N Q
551 Arun Deep's Understanding Math-10

 cos A
LM cosec A  1  cosec A + 1 OP  sin 2 θ  cos 2 θ 
 
N (cosec A + 1) (cosec A  1) Q  sin θ cos θ 
 
cos A [2 cosec A] 2 cos A
 2
 (cos 2 θ  sin 2 θ) 1
cosec A  1 sin A (cot 2 A) = × sin θ cos θ
sin θ cos θ
2 cot A 2
 2
  2 tan A = R.H.S. sin 2 cos 2 θ
cot A cot A
= = 1 = R.H.S.
sin 2 θ cos 2 θ
(cos   sin  ) (1  tan  )
(iii)  sec  5. (i) sin2  + cos4  = cos2  + sin4 
2 cos2   1
Sol. L.H.S. = sin2  + cos4 
FG sin  IJ = (1 – cos2 ) + cos4 
(cos   sin  ) 1  = 1 – cos2  + cos4 
H cos  K
Sol. L.H.S.= = 1 – cos2  (1 – cos2 )
2 cos2   1
= 1 – cos2 .sin2 
(cos   sin  ) (cos   sin  ) R.H.S. = cos2  + sin4 

cos  ( 2 cos2   1) = 1 – sin2  + sin4 
= 1 – sin2  (1 – sin2 )
cos2   sin 2  cos2   (1  cos2  ) = 1 – sin2  cos2 
 
cos  ( 2 cos2   1) cos  ( 2 cos2   1)  L.H.S. = R.H.S.
cot  cosec  + 1
cos2   1  cos2  2 cos2   1 (ii)   2 sec 
  cosec  + 1 cot 
cos  ( 2 cos2   1) cos  ( 2 cos2   1)
cot  cosec  + 1
1 Sol. L.H.S.= 
  sec   R.H.S. cosec  + 1 cot 
cos 
P.Q. (cosec  – sin ) (sec  – cos ) cos  1
1
(tan  + cot ) = 1. sin  sin 
 
Sol. (cosec  – sin ) (sec  – cos ) (tan  + 1 cos 
1
cot ) = 1 sin  sin 
L.H.S. = (cosec  – sin ) (sec  – cos )
(tan  + cot ) cos  1  sin 
 1   1  sin  sin 
 
=   sin θ    cos θ  1  sin  cos 
 sin θ   cos θ 
sin  sin 
 sin θ cos θ 
   cos  sin  1  sin  sin 
 cos θ sin θ   ×  ×
sin  1  sin  sin  cos 
 1  sin 2 θ   1  cos 2 θ 
cos  1  sin 
=  sin θ   cos θ   
   1  sin  cos 
552 Arun Deep's Understanding Math-10

cos 2   1  sin 2   2 sin  


2 (sin A + cos A)

(1  sin  ) cos  sin A + cos2 A + 2 sin A cos A  1
2

1  1  2 sin  2  2 sin  2 (sin A + cos A)


  
(1  sin ) cos  (1  sin  ) cos  1  2 sin A cos A  1
2 (1  sin  ) 2 2 (sin A + cos A)
  
cos  (1  sin  ) cos  2 sin A cos A
= 2 sec  = R.H.S.
6. (i) sec4 A (1 – sin4 A) – 2 tan2 A = 1 sin A + cos A sin A
 
Sol. L.H.S. sin A cos A sin A cos A
1 cos A
= (1 + sin2 A) (1 – sin2 A) – 2 tan2 A 
4
cos A sin A cos A
[ a2 – b2 = (a + b) (a – b)]
1 1
 
(1 + sin 2 A) cos 2 A sin 2 A cos A sin A
= 2
cos 4 A cos2 A = sec A + cosec A = R.H.S.

1  sin 2 A 2 sin 2 A sin 3 θ  cos 3 θ


  7. (i) + sin  cos  = 1
cos 2 A cos 2 A sin θ  cos θ

1  sin 2 A  2 sin 2 A (ii) (sec A – tan A)2 (1 + sin A) = 1 – sin A.



cos 2 A sin 3 θ  cos 3 θ
Sol. (i) + sin  cos  = 1
sin θ  cos θ
1  sin 2 A cos 2 A
  1
cos 2 A cos 2 A
sin 3 θ  cos3 θ
( 1 – sin2 A = cos2A) LHS = + sin  cos 
sin θ  cos θ
= R.H.S.
1 1 (sin θ  cos θ) (sin 2 θ  sin θ cos θ  cos 2 θ)
(ii)  =
sin A  cos A + 1 sin A + cos A  1 (sin θ  cos θ)
= sec A + cosec A + sin  cos 
1 = +
sin2 cos2  – sin  cos  + sin  cos 
Sol. L. H.S. =
sin A  cos A + 1 = 1 = R.H.S.
(ii) (sec A – tan A)2 (1 + sin A) = 1 – sin A
1

sin A + cos A  1 2
 1 sin A 
L.H.S. =    (1 + sin A)
sin A + cos A  1  sin A + cos A + 1  cos A cos A 

(sin A + cos A + 1) (sin A + cos A  1)
2
 1  sin A 

2 (sin A + cos A) =   (1 + sin A)
(sin A + cos A) 2  (1) 2  cos A 
553 Arun Deep's Understanding Math-10

(1  sin A) 2 (cos A  sin A) (cos A  sin A)


= (1 + sin A) = (cos A  sin A)
cos 2 A
= cos A + sin A
(1  sin A) (1  sin A)
2 = sin A + cos A = R.H.S.
= (ii) (sec A – cosec A) (1 + tan A + cot A) = tan A
1  sin 2 A
sec A – cot A cosec A
L.H.S. = (sec A – cosec A) (1 + tan A +
(1  sin A) 2 (1  sin A)
= cot A)
(1  sin A) (1  sin A)
 1 1   sin A cos A 
= 1 – sin A = R.H.S. =    1   
 cos A sin A   cos A sin A 
cos A sin 2 A
8. (i) 1  tan A – = sin A + cos A sin A  cos A
cos A  sin A = sin A cos A
(ii) (sec A – cosec A) (1 + tan A + cot A) = tan A
sec A – cot A cosec A sin A cos A  sin 2 A  cos 2 A
×
sin A cos A
tan 2 θ cosec 2 θ
(iii) + sin A  cos A sin A cos A  1
tan 2 θ  1 sec 2 θ  cosec 2 θ
= sin A cos A × sin A cos A
1
= . (sin A  cos A) (sin A cos A  1)
sin θ  cos 2 θ
2
= sin 2 A cos 2 A
cos A sin 2 A R.H.S. = tan A sec A – cot A cosec A
Sol. (i) 1  tan A – = sin A + cos A
cos A  sin A sin A 1 cos A 1
= cos A · cos A – sin A · sin A
cos A sin 2 A
L.H.S. = 1  tan A – sin A cos A
cos A  sin A
= 2 –
cos A sin 2 A
cos A
sin 2 A sin 3 A  cos 3 A
= 1  sin A – =
cos A cos A  sin A sin 2 A cos 2 A

(sin A  cos A) (sin 2 A  cos 2 A  sin Acos A)


cos A  cos A sin 2 A =
= cos A  sin A – sin 2 A cos 2 A
cos A  sin A
(sin A  cos A) (1  sin A cos A)
cos 2 A sin 2 A =
sin 2 A cos 2 A
= –
cos A  sin A cos A  sin A
(sin A  cos A) (sin A cos A  1)
=
cos A  sin A
2 2
sin 2 A cos 2 A
=
cos A  sin A  L.H.S. = R.H.S.
554 Arun Deep's Understanding Math-10

tan 2 θ cosec 2 θ 1
(iii) + = sin 2 θ  cos 2 θ
tan θ  1
2
sec θ  cosec θ
2 2

tan 2 θ cosec 2 θ
L.H.S. = +
tan 2 θ  1 sec 2 θ  cosec 2 θ

sin 2 θ 1 1
2 2
2
cos θ 2
sin θ sin θ cos θ sin 2 θ
= + = × +
sin 2 θ 1 1 2
cos θ sin θ  cos θ
2 2
sin θ  cos 2 θ
2
1 
cos 2 θ cos 2 θ sin 2 θ sin 2 θ cos 2 θ

sin 2 θ sin 2 θ cos 2 θ


= +
sin 2 θ  cos 2 θ sin 2 θ (sin 2 θ  cos 2 θ)

sin 2 θ cos 2 θ
= +
sin θ  cos θ
2 2
sin θ  cos 2 θ
2

sin 2 θ  cos 2 θ 1
= = sin 2 θ  cos 2 θ
sin θ  cos θ
2 2

= R.H.S.

sin A  cos A sin A  cos A 2 2 2sec 2 A


9. sin A  cos A + sin A  cos A = = = .
sin 2 A  cos 2 A 1  2cos 2 A tan 2 A  1

sin A  cos A sin A  cos A 2 2 2sec 2 A


Sol. sin A  cos A + sin A  cos A = = =
sin A  cos A
2 2
1  2cos A
2
tan 2 A  1

sin A  cos A sin A  cos A


L.H.S. = sin A  cos A + sin A  cos A

(sin A  cos A) 2  (sin A  cos A) 2


=
sin 2 A  cos 2 A

2(sin 2 A  cos 2 A)
=
sin 2 A  cos 2 A

2 1 2
= =
sin A  cos A
2 2
sin A  cos 2 A
2

= R.H.S.
Now, we have
555 Arun Deep's Understanding Math-10
2 = 1 – (sin2  + cos2 )2 = 1 – 1 = 0
=
sin A  cos 2 A
2
= R.H.S.
11. If cot  + cos  = m, cot  – cos  = n,
2 2 then prove that (m2 – n2) = 16 mn.
= 1  cos 2 A  cos 2 A = 1  2cos 2 A Sol. cot  + cos  = m ...(i)
= R.H.S. cot  – cos  = n ...(ii)
Now, we have Adding (i) and (ii), we get
mn
2 2 cot  = m + n  cot  
= sin 2 A  cos 2 A 2
2
 tan   ...(iii)
2 mn
= sin 2 A  (1  sin 2 A)
Subtracting (ii) from (i),

2 2 mn
= = 2 cos  = m – n  cos  
sin A  1  sin A
2 2 2
2sin A  1 2
2
2  sec   ...(iv)
mn
2 cos 2 A Squaring and subtracting, (iii) and (iv) ;
= =
sin 2 A  cos 2 A sin 2 A cos 2 A

cos 2 A cos 2 A sec 2   tan 2  
FG 2 IJ 2  FG 2 IJ 2
H m  nK H m  nK
[Divide numerator and deno. by cos2A]
4 4
2sec 2 A 1 
= ( m  n) 2 ( m  n) 2
tan 2 A  1
10. 2 (sin6  + cos6 ) – 3 (sin4  + cos4 )  4
LM 1  1 OP  1
+1=0 MN (m  n) 2 (m  n) 2 PQ
Ans. L.H.S. = 2 (sin6  + cos6 )
– 3 (sin4  + cos4 ) + 1
 4M
L (m  n) 2  (m  n) 2 OP  1
= 2 [(sin ) + (cos2 )3]
2 3
MN (m  n) 2 (m  n) 2 PQ
– 3 [sin4  + cos4 ] + 1
= 2 [(sin2  + cos2 ) 4 ( 4 mn )
 1
(sin4  + cos4  – sin2  cos2 )] (m2  n 2 ) 2
– 3 [sin4  + cos4 ] + 1
16 mn
= 2 (sin4  + cos4  – sin2  cos2 )  1
(m  n 2 ) 2
2
– 3 (sin4  + cos4 ) + 1
 (m2 – n2)2 = 16 mn.
= 2 sin4  + 2 cos4  – 2 sin2  cos2 
P.Q. If sec  + tan  = p, prove that
– 3 sin4  – 3 cos4  + 1
= – 2 sin2  cos2  – sin4  – cos4  + 1 p2 1
sin  = .
= 1 – [sin4  + cos4  + 2 sin2  cos2 ] p2  1
556 Arun Deep's Understanding Math-10

p2 1 m2 n2
Sol. sec  + tan  = p, to prove that sin  = 2   1
p 1 sin 2 A tan 2 A

1 sin θ m2 n 2 cos1 A
 cos θ + cos θ = p   1
sin 2 A sin 2 A
1  sin θ  m2 – n2 cos2 A = sin2 A
 cos θ = p  m2 – 1 = –cos2 A + n2 cos2
 m2 – 1 = –cos2 A + n2 cos2 A
On squaring, we have
 m2 – 1 = (n2 – 1) cos2 A
(1  sin θ) 2 (1  sin θ) 2
= p 2  = p2 m2  1
cos 2 θ 1  sin 2 θ  cos2 A =
n2  1
(1  sin θ) 2 1
 = p2 P.Q. If sec A = x + , then prove that sec A +
(1  sin θ) (1  sin θ) 4x
1  sin θ p2 1
 1  sin θ = tan A = 2x or
2x
.
1
Applying componendo and dividendo, 1
Sol. Given, sec A = x +
4x
1  sin θ  1  sin θ p 1 2

1  sin θ  1  sin θ = p 2  1 1
To prove : sec A + tan A = 2x or
2x
2 p2  1 1 p2  1
 = 2  = 2 2
2sin θ p 1 sin θ p 1  1 
tan A = 
sec A 1 =
2  x   1
 4x 
p2 1
 sin  =
p2  1 1 1
=  x2  2
 1
P.Q. If tan A = n tan B and sin A = m sin B, prove 16 x 2

m2  1 1 1  1 
that cos2A = . =  x2   = x  
n2  1 16 x 2
2  4x 
sin A tan A
Sol. Given, m = , n 1 1
sin B tan B  sec A + tan A = x + +x– = 2x
4x 4x
1 m m 1 1 1
  cosec B =
sin B sin A sin A or x + –x+ =
4x 4x 2x
1 n n Hence proved.
  cot B =
tan B tan A tan A 12. When 0° <  < 90°, solve the following
equations:
Now, cosec2 B – cot2 B = 1
557 Arun Deep's Understanding Math-10
(i) 2 cos2  + sin  – 2 = 0 Given sec2  – 2 tan  = 0
(iii)
(ii) 3 cos  = 2 sin2   1 + tan2  – 2 tan  = 0
(iii) sec2  – 2 tan  = 0  tan2  – 2 tan  + 1 = 0
(iv) tan2  = 3 (sec  – 1).  (tan  – 1)2 = 0  tan  – 1 = 0
Sol. 0° <  < 90°  tan  = 1 = tan 45°
(i) Given 2 cos2  + sin  – 2 = 0   = 45°
 2 (1 – sin2 ) + sin  – 2 = 0 (iv)Given tan2  = 3 (sec  – 1)
 2 – 2 sin2  + sin  – 2 = 0  (sec2  – 1) = 3 (sec  – 1)
 –2 sin2  + sin  = 0  sec2  – 1 = 3 sec  – 3
 sin  (–2 sin  + 1) = 0  sec2  – 1 – 3 sec  + 3 = 0
Either sin  = 0, then  = 0  sec2  – 3 sec  + 2 = 0
1  sec2  – sec  – 2 sec  + 2 = 0
or –2 sin  + 1 = 0, then sin  =  sec  (sec  – 1) – 2 (sec  – 1) = 0
2
 (sec  – 1) (sec  – 2) = 0
  = 30°
Either sec  – 1 = 0, sec  = 1 = sec 0°
Hence  = 0° or 30°
  = 0°, not possible (  > 0°)
(ii) Given 3 cos  = 2 sin2 
or sec  – 2 = 0  sec  = 2
 3 cos  = 2 (1 – cos2 )
 sec  = 2 = sec 60°
 3 cos  = 2 – 2 cos2 
  = 60°
 2 cos2  + 3 cos  – 2 = 0
 2 cos2  + 4 cos  – cos  – 2 = 0
 2 cos  (cos  + 2) – 1( cos  + 2) = 0
(cos  + 2) (2 cos  – 1) = 0
Either cos  + 2 = 0  cos  = –2 which is
not possible.
–1 < cos  < 1 or 2 cos  – 1 = 0
1
 cos  = = cos 60°
2
  = 60°
19
TRIGONOMETRICAL TABLES
POINTS TO REMEMBER

1. In the tables, values of different angles in degrees and minutes upto 90º of natural sines, natural
cosines and natural tangents are given.
2. These tables consists of three parts :
(i) A column on extreme left containing degrees from 0º to 90º.
(ii) Ten columns at the in verticals of 6 headed by 0, 12, 18, 24, 30, 36, 42, 48, 54
(Note 1º = 60).
(iii) Five columns which are called mean difference columns for 1, 2, 3, 4 and 5 in the angle.
Note. Mean difference is added in case of sines and tangents and is subtracted in case of cosines.
EXERCISE 19
1. Find the value of : (iv) sin 23º 56
(i) sin 35º 22 (ii) sin 71º 31 Using the table of natural sines, we see 23º
(iii) sin 65º 20 (iv) sin 23º 56. in the horizontal line and for 54, we see in
vertical column, the value is 0·4051 and
Sol. (i) sin 35º 22 for 56 – 54 = 2 in the mean difference. It
Using the table of natural sines, we see 35º is 5.
in the horizontal line and for 18, in the ver-  sin 23º 56 = 0·4051 + 5 = 0·4056 Ans.
tical column, the value is 0·5779. Now read 2. Find the value of the following :
22 – 18 = 4 in the difference column, the
(i) cos 62º 27 (ii) cos 3º 11
value is 10. Adding 10 in 0·5779 + 10
= 0·5789, we find sin 35º 22 = 0·5789. (iii) cos 86º 40 (iv) cos 45º 58.
Sol. (i) cos 62º 27
(ii) sin 71º 31
From the table of natural consines, we see
Using the table of natural sines, we see 71º 62º in the horizontal line and 24 in the ver-
in the horizontal line and for 30 in the ver- tical column, the value is ·4633 and 27 –
tical column, the value is 0·9483 and for 24 = 3 in the mean difference. Its value is
31 – 30 = 1, we see in the mean differ- 8.
ence column, the value is 1.  cos 62º 27 = 0·4633 – 8 = 0·4625 Ans.
 sin 71º 31 = 0·9483 + 1 = 0·9484. (ii) cos 3º 11
(iii) sin 65º 20 From the table of natural cosines, we see
Using the table of natural sines, we see 65º 3º in the horizontal line and 6 in the vertical
in the horizontal line and for 18 in the ver- column, its value is 0·9985 and 11 – 6 = 5
tical column, the value is ·9085 and for 20 in the mean difference, its value is 1.
– 18 = 2, we see in the mean difference  cos 3º 11 = 0·9985 – 1 = 0·9984 Ans.
column. We find 2. (iii) cos 86º 40
 sin 65º 20 = 0·9085 + 2 = 0·9087 Ans. From the table of natural cosines, we see

558 Arun Deep's Understanding Math-10


559 Arun Deep's Understanding Math-10
86º in the horizontal line and 36 in the ver- From the table of natural sines, we look
tical column, its value is 0·0593 and for 40 for the value ( ·5789), which must be very
– 36 = 4 in the mean difference, it is 12. close to it, we find the value ·5779 in the
 cos 86º 40 = 0·0593 – 12 = 0·0581 Ans. column 35º 18 and in mean difference, we
(iv) cos 45º 58 see ·5789 – ·5779 = ·0010 in the column of
4.
From the table of natural cosines, we see
45º in the horizontal column and 54 in the   = 35º 18 + 4 = 35º 22 Ans.
vertical column, its value is 0·6959 and for (ii) sin  = ·9484
58 – 54 = 4, in the mean difference, it is From the table of natural sines, we look
8. for the value ( ·9484) which must be very
 cos 45º 58 = 0·6959 – 8 = 0·6951 Ans. close to it, we find the value ·9483 in the
3. Find the value of the following : column 71º 30 and in the mean differences,
we see ·9484 – 9483 = ·0001, in the col-
(i) tan 15º 2 (ii) tan 53º 14
umn of 1.
(iii) tan 82º 18 (iv) tan 6º 9.
  = 71º 30 + 1 = 71º 31 Ans.
Sol. (i) tan 15º 2
(iii) sin  = ·2357
From the table of natural tangents, we see
From the table of natural sines, we look
15º in the horizontal line, its value is 0·2679
and for 2, in the mean difference, it is 6. for the value ( ·2357) which must be very
close to it, we find the value ·2351 in the
 tan 15º 2 = 0·2679 + 6 = 0·2685. column 13º 36 and in the mean difference,
(ii) tan 53º 14 we see ·2357 – 2351 = ·0006, in the col-
From the table of natural tangents, we see umn of 2.
53º in the horizontal line and 12 in the ver-   = 13º 36 + 2 = 13º 38 Ans.
tical column, its value is 1·3367 and 14 –
(iv) sin  = ·6371
12 = 2 in the mean difference, it is 16.
From the table of natural sines, we look
 tan 53º 14 = 1·3367 + 16 = 1·3383 Ans.
for the value ( ·6371) which must be very
(iii) tan 82º 18 close to it, we find the value ·6361 in the
From the table of natural tangents, we see column 39º 30 and in the mean difference,
82º in the horizontal line and 18 in the ver- we see ·6371 – ·6361 = ·0010 in the col-
tical column, its value is 7·3962. umn of 4.
 tan 82º 18 = 7·3962.   = 39º 30 + 4 = 39º 34 Ans.
(iv) tan 6º 9 5. Use the tables to find the acute angle ,
From the table of natural tangents, we see given that :
6º in the horizontal line and 6 in the vertical (i) cos  = ·4625 (ii) cos  = ·9906
column, its value is ·1069 and 9 – 6 = 3, (iii) cos  = ·6951 (iv) cos  = ·3412.
in the mean difference, it is 9.
Sol. (i) cos  = ·4625
 tan 6º 9 = ·1069 + 9 = ·1078.
4. Use tables to find the acute angle , given
that : From the table of natural cosines, we look
for the value ( ·4625) which must be very
(i) sin  = ·5789 (ii) sin  = ·9484
close to it, we find the value ·4617 in the
(iii) sin  = ·2357 (iv) sin  = ·6371. column of 62º 30 and in the mean differ-
Sol. (i) sin  = ·5789 ence, we see ·4625 – ·4617 = ·0008 which
560 Arun Deep's Understanding Math-10
is in column of 3. the column of 60º 6 and in the mean dif-
  = 62º 30 – 3 = 62º 27. ference 1·7451 + 1·7391 = 0·0060 which
is in the column of 5.
(ii) cos  = ·9906
From the table of cosines, we look for the   = 60º 6 + 5 = 60º 11 Ans.
value ( ·9906) which must be very close (iii) tan  = 3·1749
to it, we find the value of ·9905 in the col- From the tables of natural tangents, we look
umn of 7º 54 and in mean difference, we for the value of ( 3·1749) which must be
see ·9906 – 9905 = ·0001 which is in col- very close to it, we find the value 3·1716 in
umn of 3. the column of 72º 30 and in the mean dif-
  = 7º 54 – 3 = 7º 51 Ans. ference 3·1749 – 3·1716 = 0·0033 which
(iii) cos  ·6951 is in the column of 1.
From the tables of cosines, we look for the   = 72º 30 + 1 = 72º 31 Ans.
value ( 6951) which must be very close (iv) tan  = ·9347
to it, we find the value ·6947 in the column From the tables of natural tangents, we look
of 46º and in mean difference, ·6951 – ·6947 for the value of ( ·9347 which must be
= 0·0004 which in the column of 2. very close to it, we find the value ·9325 in
  = 46º – 2 = 45º 58 Ans. the column of 43º and in the mean differ-
(iv) cos  = ·3412 ence ·9347 – ·9325 = 0·0022 which is in
From thetable of cosines, we look for the the column of 4.
value of (  ·3412) which must be very   = 43º + 4 = 43º 4 Ans.
close to it, we find the value ·3404 in the 7. Using trigonometric table, find the measure
column of 70º 6 and in the mean differ- of the angle A when sin A = 0·1822.
ence, ·3412 – 3404 = ·0008 which is in the Sol. sin A = 0·1822
column of 3.
From the tables of natural sines, we look
  = 70º 6 – 3 = 70º 3 Ans. for the value ( ·1822) which must be very
6. Use table to find the acute angle , given close to it, we find the value ·1822 in the
that : column 10º 30.
(i) tan  = ·2685 (ii) tan  = 1·7451  A = 10º 30 Ans.
(iii) tan  = 3·1749 (iv) tan  = 8. Using tables, find the value of 2 sin  – cos
·9347.  when (i)  = 35º (ii) tan  = ·2679.
Sol. (i) tan  = ·2685 Sol. (i)  = 35º
From the table of natural tangent, we look  2 sin  – cos  = 2 sin 35º – cos 35º
for the value of ( ·2685) which must be
= 2 × ·5736 – ·8192
very close to it, we find the value ·2679 in
the column of 15º and in the mean differ- (From the tables)
ence, ·2685 – ·2679 which is in the col- = 1·1472 – ·8192 = 0·3280.
umn of 2. (ii) tan  = ·2679
  = 15º + 2 = 15º 2 Ans. From the tables of natural tangents, we look
(ii) tan  = 1·7451 for the value of  ·2679, we find the value
From the tables of natural tangents, we look of the column 15º.
for the value of ( 1·7451) which must be   = 15º
very close to it, we find the value 1·7391 in Now, 2 sin  – cos  = 2 sin 15º – cos 15º
561 Arun Deep's Understanding Math-10
= 2 (·2588) – ·9659 = ·5136 – ·9659 2. Use tables to find the acute angle , given
= – 0·4483 Ans. that
9. If sin xº = 0·67, find value of (i) sin  = 0·5766 (ii) cos  = 0·2495
(i) cos xº (ii) cos xº + tan xº. (iii) tan  = 2·4523.
Sol. sin xº = 0·67 Sol. Using table, we find that
From the table of natural sines, we look (i) sin  = 0·5766 = 0·5764 + 0·0002
for the value of ( 0·67) which must be = sin (35º 12 + 1) = sin 35º 13
very close to it, we find the value ·6691 in   = 35º 13Ans.
the column 42º and in the mean difference, (ii) cos  = 0·2495 = 0·2487 + 0·0008
the value of 0·6700 – 0·6691 = 0·0009
= cos (75º 36 – 3) = cos 75º 33
which is in the column 4.
  = 75º 33Ans.
  = 42º + 4 = 42º 4
(iii) tan  = 2·4523 = 2·4504 + 0·0019
Now
= tan (67º 48 + 1)
(i) cos xº = cos 42º 4 = ·7431 – ·0008
= tan 67º 49 Ans.
= 0·7423 Ans.
3. If  is acute and cos  = 0·53, find the
(ii) cos xº + tan xº = cos 42º 4 + tan 42º 4 value of tan .
= 0·7423 + ·9025
Sol. From the table, we find that
= 1·6448 Ans.
cos  = 0·53 = ·5299 + ·0001 = cos 58º
10. If  is acute and cos  = ·7258, find the
  = 58º
value of (i)  (ii) 2 tan  – sin .
and tan 58º = 1·6003 Ans.
Sol. cos  = ·7258
4. Find the value of :
From the table of cosines, we look for the
value of ( ·7258) which must be very close sin 22º 11 + cos 57º 20 – 2 tan 9º 9.
to it, we find the value ·7254 in the column Sol. Using the tables, we find that
of 43º 30 and in the mean differences the sin 22º 11 = 0·3762 + 0·0014 = 0·3776
value of ·7258 – ·7254 = 0·0004 which in cos 57º 20 = 0·5402 – 0·0005 = 0·5397
the column of 2.
tan 9º 9 = 0·1602 + 0·0009 = 0·1611
(i)   = 43º 30 – 2 = 43º 28.
 sin 22º 11 + cos 57º 20 – 2 tan 9º 9
(ii) 2 tan  – sin 
= 0·3376 + 0·5397 – 0·1611 × 2
= 2 tan 43º 28 – sin 43º 28
= 0·3776 + 0·5397 – 0·3222
= 2 (·9479) – ·6879 = 0·9173 – 0·3222 = ·5951.
= 1·8958 – ·6879 = 1·2079 Ans. 5. If  is acute and sin  = 0·7547, find the
value of : (i)  (ii) cos  (iii) 2 cos  – 3
CHAPTER TEST tan .
1. Using trigonometrical tables, find the val- Sol. Using the tables, we find that
ues of : (i) sin 48º 52 (ii) cos 37º 34 (i) sin  = 0·7547 = sin 49º  = 49º.
(iii) tan 18º 21. (ii) cos  = cos 49º = 0·6561.
Sol. Using tables, we find that (iii) tan  = tan 49º = 1·1504
(i) sin 48º 52 = ·7524 + ·0008 = ·7532  2 cos  – 3 tan 
(ii) cos 37º 34 = ·7934 – ·0007 = ·7927 = 2 × 0·6561 – 3 × 1·1504
(iii) tan 18º 21 = ·3307 + ·0010 = ·3317. = 1·3122 – 3·4512 = – 2·1390
20
HEIGHTS AND DISTANCES
POINTS TO REMEMBER

1. Angle of Elevation.
When we see an object upward, then the angle formed by the line of sight and horizontal line is
called an angle of elevation.
2. Angle of Depression.
When we see an object downward, then the angle formed by the line of sight and horizontal line
is called an angle of depression.

3. For solving the questions on height and distance, we use T-ratios of known angles.
Note : Take 2 = 1·414, 3 = 1·732
4. Values of some angles :
 0° 30° 45° 60° 90°

1 1 3
sin  0 2
1
2 2

3 1 1
cos  1 2
0
2 2

1
tan  0 3
1 3 Not Defined

Note : For other angles, we use tables


EXERCISE 20
1. An electric pole is 10 metres high. If its  AB = 10 m, OB = 10 3 m
shadow is 10 3 metres in length, find the and  is the angle of elevation of the sun.
elevation of the sun. AB 10 1
Sol. Let AB be the pole and  tan  = OB = 
10 3 3
OB is its shadow. 1
 = 30° Ans. ( tan 30° = 3 )
543 Arun Deep's Understanding Math-10
563 Arun Deep's Understanding Math-10

AB x
2. The angle of elevation of the top of a tower  tan  = CB  tan 60° = 1  5
from a point on the ground and at a distance
x
of 150 m from its foot is 30°. Find the  3 = 1  5
height of the tower correct to one place of
decimal.  x= 3 × 1·5 = 1·732 × 1·5
Sol. Let BC be the tower and  x = 2·596 = 3·0
A is the point on the ground such that  Height of the wall = 3 m (approx.) Ans.
A= 30° and AC = 150 m 4. What is the angle of elevation of the sun
when the length of the shadow of a vertical
pole is equal to its height.
Sol. Let AB be the pole and CB be its shadow
and  is the angle of elevation of the sun.
Let AB = xm, then BC = xm
AB x
 tan  = CB  x  1
Let height of the tower = x m.
BC x
 tan  = AC  tan 30° = 150
1 x 150
   x= 3
3 150
150  3 150 3
 x=   50 3 m
3 3 3
= 50 (1·732) = 86·600 m  = 45° ( tan 45° = 1)
= 86·6 m Ans.  Angle of elevation = 45° Ans.
3. A ladder is placed against a wall such that P.Q. A river is 60 m wide. A tree of unknown
it just reaches the top of the wall. The foot height is on one bank. The angle of
of the ladder is 1·5 metres away from the elevation of the top of the tree from the
wall and the ladder is inclined at an angle point exactly opposite to the foot of the
of 60° with the ground. Find the height of tree on the other bank is 30°. Find the
the wall. height of the tree.
Sol. Let AB be the wall and AC be the ladder Sol. Let AB be the tree and BC is the width of
whose foot C is 1·5 m away from B the river and C is the point exactly opposite
Let AB = x m and angle of inclination is to B on the other bank and angle of
60° elevation is 30°.
564 Arun Deep's Understanding Math-10
Sol. Let AB be cliff whose height is 92 m and
C is buoy making depression angle of 20°.
x X A
20°

Let height of the tree AB = xm


92m
and width of river BC = 60 m
AB x 20°
 tan  = CB  tan 30° = 60 C x B

1 x 60 60 3  ACB = 20°
   x =  x = 
3 60 3 3 3 Let, CB = x m.
In right ABC,
60 3
x=  20 3 = 20 (1·732)
3 BC x
= 34·640 = 34·64 m cot =   cot 20° =
AB 92
 Height of the tree = 34·64 m Ans.  x = 92 cot 20°
5. From a point P on level ground, the angle = 92 × 2·7475 m
of elevation of the top of a tower is 30°. If = 252·7700 m
the tower is 100 m high, how far is P from Distance of buoy from foot of hill
the foot of the tower ? = 252.77m
Sol. Let AB be the tower and P is at a distance 7. A boy is flying a kite with a string of length
of x m from B, the foot of the tower. 100 m. If the string is tight and the angle
While height of the tower AB = 100 m and of elevation of the kite is 26°32, find the
angle of elevation = 30° height of the kite correct to one decimal
AB 100 place. (ignore the height of the boy).
 tan  = PB  tan 30° = x Sol. Let AB be the height of the kite A and AC is
1 100 the string and angle of elevation of the kite
   x = 100 3 is 26°32
3 x

x Let AB = x m and AC = 100 m


 x = 100 (1·732) = 173·2 m AB x
 Distance of P from the foot of the tower  sin = AC  sin 26°32 = 100
= 173·2 m Ans. x
 0·4467 = 100  x = 100 × 0·4467
6. From the top of a cliff 92 m high, the angle
of depression of a buoy is 20º. Calculate to  x = 44·67 = 44·7 m
the nearest metre, the distance of the buoy Height of the kite = 44·7 m Ans.
from the foot of the cliff. (2005)
565 Arun Deep's Understanding Math-10
8. An electric pole is 10 m high A steel wire 1 x 200 200 2
tied to the top of the pole is affixed at a   x =  x = 
2 200 2 2 2
point on the ground to keep the pole
upright. If the wire makes an angle of 45° 200 (1  414)
 x= 2 = 100 (1·414)
with the horizontal through the foot of the
pole, find the length of the wire. = 141·4 m
Sol. Let AB be the pole and AC be the wire which  Width of the river = 141·4 m Ans.
makes an angle of 45° with the ground. 9. A vertical tower is 20 m high. A man
Height of the pole AB = 10 m and let length standing at some distance from the tower
of wire AC = x m knows that the cosine of the angle of
elevation of the top of the tower is 0·53.
How far is he standing from the foot of
the tower ? (2001)
Sol. Let AB be the tower and let a man C stands
at a distance from the foot of the tower
=xm
and cos  = 0·53

AB 10
 sin  = AC  sin 45° = x
1 10
   x = 10 2 = 10 (1·414)
2 x
 x = 14·14
 Length of wire = 14·14 m Ans. Height of the tower AB = 20 m
 cos  = 0·53
P.Q. A bridge across a river makes an angle of
45° with the river bank. If the length of  = 58° (from the tables)
the bridge across the river is 200 metres,
what is the breadth of the river. AB 20
Now tan  = CB  tan 58° = x

20 20
 1·6003 = x  x = 1  6003

 x = 12·49 = 12·5 m
 Height of the tower = 12·5 m Ans.
10. The upper part of a tree broken by wind,
falls to the ground without being detached.
Sol. Let AB be the width of river = x m The top of the broken part touches the
ground at an angle of 38°30 at a point 6 m
Length of the bridge AC = 200 m
from the foot of the tree. Calculate.
and angle with the river bank = 45°
(i) the height at which the tree is broken.
AB x
 sin  = AC  sin 45° = 200 (ii) the original height of the tree correct to
two decimal places.
566 Arun Deep's Understanding Math-10
Sol. Let TR be the total height of the tree and 4  7724
TP is the broken part which touches the  0·6225 = y
ground at the distance of 6 m from the
foot of the tree making an angle of 38°30 4  7724
with the ground.  y = 0  6225  7  6665
Let PR = x and TR = x + y  Height of the tree = 4·7724 + 7·6665
 PQ = PT = y
= 12·4389
In right PQR,
= 12·44 m
PR
tan  = QR and height of the tree at which it is broken
= 4·77 m Ans.
x
 tan 38°30 = 6 11. An observer 1·5 m tall is 20·5 metres away
from a tower 22 metres high. Determine
x the angle of elevation of the top of the tower
 6 = 0·7954
from the eye of the observer.
Sol. In the figure, AB is tower and CD is
observer.  is the angle of observation from
C to A.

20.5cm

22m

C E
20.5cm
1.5m

1.5m

D 20.5cm B
x
 0. 7954
6
 x = 0·7954 × 6 = 4·7724  AB = 22 m, CD = 1·5 m
PR and BD distance = 20.5 m
and sin  = PQ
From C draw CE || DB, then

x  AE = 22 – 1.5 = 20·5 m
 sin 38°30 = y
and CE = DB = 20·5 m
567 Arun Deep's Understanding Math-10

AE 20·5
 tan  = = = 1 = tan 45°
CE 20·5
 = 45°
12. The angle of elevation from a point P of
the top of a tower QR, 50 m high is 60º
and that of the tower PT from a point Q is
30º. Find the height of the tower PT, correct
to the nearest metre.

Sol. Let CB be the cliff and AC be the pillar and


D be the boat which is 300 m away from
the foot of the cliff i.e. BD = 300 m.
Angles of elevation of the top and foot of
the pillar are 55°40 and 54°20
respectively.
Let CB = x and AC = y
Sol. In PQR,
In right CBD,
RQ 50 CB
tan 60º =  3 tan  = BD
PQ PQ

50 x
 PQ =  tan 54°20 = 300
3

In PTQ, x
 1·3933 = 300

 x = 300 × 1·3933
PT 1 PT
tan 30º =  
PQ 3 50  x = 417·99 m
3 Again in right ABD
1 50 50
 PT =   AB x  y
3 3 3 tan 55°40 = BD  300
PT = 16.67 = 17 m Ans.
x y
P.Q. From a boat 300 metres away from a  300 = 1·4641
vertical cliff, the angles of elevation of the
top and the foot of a vertical concrete pillar  x + y = 1·4641 × 300 = 439·23
at the edge of the cliff are 55°40 and  y = 439·23 – 417·99 = 21·24 m
54°20 respectively. Find the height of the
pillar correct to the nearest metre.  Height of the pillar = 21·24 m = 21 m
568 Arun Deep's Understanding Math-10
13. From a point P on the ground, the angle of
10  h
elevation of the top of a 10 m tall building  3 = 10 3
and a helicopter hovering over the top of
the building are 30º and 60º respectively.
 10 3 × 3 = 10 + h
Find the height of the helicopter above the
ground.  30 = 10 + h
Sol. Let AB be the building and H is the  h = 30 – 10 = 20
helicopter hovering over it. P is a point  Height of the helicopter from the ground
on the ground, then angle of elevation of
= 10 + 20 = 30 m Ans.
top of building and helicopter are 30º and
14. An aeroplane when flying at a height of
60º
3125 m from the ground passes vertically
below another plane at an instant when the
angles of elevation of the two planes from
h the same point on the ground are 30° and
60° respectively. Find the distance between
the two planes at the instant.
Sol. Let the distance between the two planes
=hm
Given that, AD = 3125 m and ACB = 60°
and ACD = 30°
°
°

B
hm

AB = 10 m
Let PA = x m and BH = h m D
Now, in right ABP
3125 m

P
tan =
B 60°
30°

AB 10 A C
 tan 30º = =
PA x
Now, in ACD
1 10
 = AD
3 x tan 30° =
AC
 x = 10 3 m
1 3125
Similarly in right APH,  =
3 AC
AH 10  h
tan60º = =  AC = 3125 3 ...(i)
PA x
569 Arun Deep's Understanding Math-10
and In ABC h
 3 [h  x]
AB AD  DB h  20
tan 60° =  3 =
AC AC  3 ( h  20)  h  3h  h  20 3
3125  h 3125  h 20 3 3 1
  AC = ...(ii)  h ( 3  1)  20 3  h  
AC 3 3 1 3 1
From equation (i) and equation (ii), we get = 47.32 m = 47 m.
3125  h 16. The shadow of a vertical tower on a level
 = 3125 3 ground increases by 10 m when the altitude
3
of the sun changes from 45º to 30º. Find
 h = (3125 3 × 3 ) – 3125 the height of the tower, correct to two
 h = 3125 × 3 – 3125  h = 9375 – 3125 decimal places. (2006)
 h = 6250 m Sol. In the figure, AB is the tower, BD and BC
Hence, the distance between two planes is are the shadow of the tower in two
6250 m. situations. Let BD = x m and AB = h m
15. A man observes the angle of elevation of In  ABD,
the top of the tower to be 45°. He walks
towards it in a horizontal line through its h h
base. On covering 20 m the angle of tan 45º =  1=  h = x .....(i)
x x
elevation changes to 60°. Find the height
of the tower correct to 2 significant In  ABC, A
figures. (2019)
Sol. In ABD
AB h
tan  =  tan 45° = hm
BD x
30° 45°
A C 10m D x B
h 1 h
tan 30º =  
x  10 3 x  10

 h 3 = h + 10 [From (i)]
h
 h  
3  1 = 10

10 10  3 1  10 3  1
60° 45°
h =
3 1
=
 3 1  3 1  =
2
B x – 20 D = 5 (1.73 + 1) = 5 × 2.73 = 13.65
x
Hence, height of the tower is 13.65 m
h 17. From the top of a hill, the angles of
 1=  h=x
x depression of two consecutive kilometer
In ABC, stones, due east are found to be 30º and
h 45º respectively. Find the distance of two
tan 60° = stones from the foot of the hill.
x  20
570 Arun Deep's Understanding Math-10
Sol.Let A and B be the position of two consecutive AB
kilometer stones. Then BC = . ...(i)
3
AB = 1 km = 1000 m In right angled ABD :
Let the distance BC = x m
AB 1 AB
 Distance AC = (1000 + x) m D tan 30º =  =
30°
BD 3 BC  60
 BC + 60 = 3 AB
Hill  BC = 3 AB – 60
From equation (i) and (ii) we have
30° 45° AB
A 1km B x C
3
= 3 AB – 60  AB = 3AB – 60 3
(1000 + x)m
In right  d  BCD,  3AB – AB = 60 × 1.732
CD CD 60  1.732
= tan 45º  = 1  CD = BC = x  AB = = 51.96 m
BC BC 2
19.At a point on level ground, the angle of elevation
In right  d  ACD, of a vertical tower is found to be such that its
DC x 1 5
tangent is . On walking 192 m towards the tower,,
= tan 30º  = 12
AC x 1000 3
3
 3x = x + 1000   
3  1 x = 1000
the tangent of the angle is found to be . Find the
height of the tower.
4
(1990)
1000 1000 3 1 Sol. Let TR be the tower and P is the point on the
 x=  x= ×
3 1 3 1 3 1 ground such that tan  =
5

 
12
1000 3  1 1000 3  1   3
=
3 1
= = 500 (1.73 + 1) and tan  = 4
2
= 500 × 2.73 = 1365 m. PQ = 192 m
 Distance of first stone from the foot of the hill =
1365 m and distance of the second stone from the
foot of the hill = 1000 m + 1365 m = 2365 m. Let TR = x and QR = y
18. A man observes the angles of elevation of the Now in right TQR
top of a building to be 30º. He walks towards it TR x 3 x
in a horizontal line through its base. On covering tan  = QR  y  4  y
60 m the angle of elevation changes to 60º. Find
the height of the building correct to the nearest 4
metre.  y= 3 x ...(i)
Sol- AB is a building CD = 60 m (given) Again in right TPR,
In ABC: A
TR 5 x
AB tan  = PR  12  y  192
tan 60º =
BC
5
AB  x = (y + 192) 12 ...(ii)
 3 = BC From (i) and (ii)
60º 30º 4 5
B C 6m D
x = ( 3 x + 192) 12
571 Arun Deep's Understanding Math-10
5 5 Calculate the distance between the vehicles.
x = 9 x  80  x – x  80 (1994)
9
4 80  9 Sol. Height of the church CH. Let A and B are
 9 x  80  x=  180 two vehicles which make the angle of
4
 Height of the tower = 180 m Ans. depressi on f rom C are x° and y°
respectively.
20. In the figure, not drawn to scale, TF is a
tower. The elevation of T from A is x°
2
where tan x = 5 and AF = 200 m. The
elevation of T from B, where AB = 80 m,
is y°. Calculate :
(i) The height of the tower TF.
(ii) The angle y, correct to the nearest degree.
(1997)
Let AH = x and BH = y
In right CBH,
CH 96 1 96
tan x° =   
AH y 4 y
 y = 96 × 4 = 384 m
Again in right CAH,
Sol. Let height of the tower TF = x CH 96
tan y° = 
2 AH x
tan x  , AF = 200 m, AB = 80 m 1 96
5    x = 96 × 7 = 672 m
(i) In right ATF, 7 x
 AB = x – y = 672 – 384 = 288 m Ans.
TF 2 x
tan x° =   21. In the adjoining figure, not drawn to the
AF 5 200
scale, AB is a tower and two objects C and
2×200 400
 x = =  80 m D are located on the ground, on the same
5 5 side of AB. When observed from the top A
 Height of tower = 80 m of the tower, their angles of depression are
(ii) In right TBF 45° and 60°. Find the distance between
TF 80 80 2 the two objects. If the height of the tower
tan y = BF  200  80  120  3 is 300 m. Give your answer to the nearest
metre. (1998)
= 0·6667
y = 33°41 = 34° Ans.
P.Q. From the top of a church spire 96 m high,
the angles of depression of two vehicles
on a road, at the same level as the base of
the spire and on the same side of it are x°
1 1
and y°, where tan x° = 4 and tan y° = 7 .
572 Arun Deep's Understanding Math-10
Sol. Let CB = x PL = QR = 140 m
and DB = y  TL = (x – 60) m
AB = 300 m Now in right TPL,
In right ACB, TL x  60
tan  = PL  tan 30° = 140
AB 300
tan  = CB  tan 45° = x 1 x  60

300 3 140
 1 = x  x = 300 m
140 140 3 140 3
In right ADB  x – 60 = x =  
3 3 3 3
AB 300
tan 60° = DB  y 140 3 140 (1 732)
x= + 60 =  60
3 3
300 300 300 3 300 3 = 80·83 + 60 = 140·83
3
y y 3 y= 3  3 3
 Height of first tower = 140·83 m Ans.
 y = 100 (1·732) = 173·2 m 23. As observed from the top of a 80 m tall
 CD = x – y = 300 – 173·2 lighthouse, the angles of depression of two
= 126·8 = 127 m ships on the same side of the lighthouse in
 Distance between two objects horizontal line with its base are 30º and 40º
respectively. Find the distance between the
= 127 m Ans. two ships. Give your answer correct to
22. The horizontal distance between two the nearest metre.
towers is 140 m. The angle of elevation of Sol. Let AB be the lighthouse and C and D be
the top of the first tower when seen from the two ships.
the top of the second tower is 30°. If the
A
height of the second tower is 60 m, find
the height of the first tower.
Sol. Let the height of first tower TR = x
80m
height of second tower PQ = 60 m
Distance between the two towers QR
o
= 140 m 30 40
o

D C B
Then, in ADB
AB
tan 30º =
BD
1 80
 =  BD  80 3 ...(i)
3 BD
AB 80
In ACB, tan 40º = =
BC BC
80 80
Draw PL | | QR, then 0.84 =  BC = = 95.25
BC 0.84
LR = PQ = 60 m
573 Arun Deep's Understanding Math-10

From (i), BD = 80 3 = 80 × 1.73 = 138.4 25. From two points A and B on the same side
of a building, the angles of elevation of the
DC = BD – BC = 138.4 – 95.25 = 43.15
top of the building are 30º and 60º
Hence, distance between the two ships is respectively. If the height of the building is
= 43.15 Ans. 10 m, find the distance between A and B
24. The angle of elevation of a pillar from a correct to two decimal places.
point A on the ground is 45° and from a
point B diametrically opposite to A and on 10
Sol. In DBC, tan 60º =
the other side of the pillar is 60°. Find the BC
height of the pillar, given that the distance
between A and B is 15 m. 10 10
 3 = BC  BC = 3
Sol. Let CD be the pillar and let CD = x
Angles of elevation of points A and B are 10
45° and 60° respectively. In DAC, tan 30º =
BC  AB

10 m
y

AB = 15 m
Let AD = y 30º 60º
A C
B
then DB = 15 – y
Now in right CAD,
1 10 1  10 
CD x  = 10    AB  = 10
tan  = AD  tan 45° = y 3  AB 3  3 
3
x
 1 = y  x=y ...(i)
10
In right CDB,  AB = 10 3 –
3
x x
tan 60° = 15  y  3 30  10 20 20 3
15  y = = =
3 3 3
 x= 3 (15 – y) ...(ii)
 x= 3 (15 – x)  x = 15 3 – 3x 20  1.732
= = 20 × .577 = 11.540 m
3
 x+ 3 x = 15 3
26. The angles of depression of two ships A
15 3
 x (1 + 3 ) = 15 3  x = 1  3 and B as observed from the top of a light
house 60 m high are 60° and 45°
15 (1  732) 25  98 respectively. If the two ships are on the
 x = 1  1 732  2  732 = 9·51 opposite sides of the light house, find the
 Height of the pillar = 9·51 m Ans. distance between the two ships. Give your
574 Arun Deep's Understanding Math-10
answer correct to the nearest whole 250
number. (2017) 1= { tan45º=1}
x
Sol. Let AD be the height of the light house
 CD = 60 m  x = 250 m
Let AD = x m, BD = y m
O
C

250 m
60 m
45º 60º
60° 45° A x M y B
A x D y B

 In ACD, 250
In OMB, tan 60º = y
CD
tan 60° =
AD
250 250 250
60  3 = y  y = =
60 3 1.73
 3 = x x= 3
y = 144.34
60 3 60 3  Width of river = x + y = 250 + 144.34
 x= × = = 394.34 m
3 3 3
= 20 × 1.732 = 34.64 m 28. From a tower 126 m high, the angles of
In BCD, depression of two rocks which are in a
horizontal line through the base of the tower
CD
tan 45° = are 16° and 12°20. Find the distance
BD
between the rocks if they are on
60 (i) the same side of the tower
 1 = y  y = 60 m
(ii) the opposite sides of the tower.
 Distance between two ships = x + y Sol. Let CD be the tower and CD = 126 m
= 34.64 + 60 = 94.64 m
Let A and B be the two rocks on the same
= 95 m (correct to nearest whole number)
line and angles of depression are 16° and
27. An aeroplane at an altitude of 250 m
12°20 respectively.
observes the angle of depression of two
boats on the opposite banks of a river to
be 45º and 60º respectively. Find the width
of the river. Write the answer correct to
the nearest whole number. (2014)

Sol. In OMA
OM
tan 45º =
AM
575 Arun Deep's Understanding Math-10
Let AB = x, CD = 1·8 m
EB = CD = 1·8 m, then AE = x – 1·8
Shadow FD = 5·4 m
Now in right ACE
AE x  1 8
tan  = CE  3  6 ...(i)
Again in right CFD,
In right CAD,
CD 1 8 1
CD 126 tan  = FD  5  4  3 ...(ii)
tan  = AD  tan 16° = x
From (i) and (ii)
126 126
 0·2867 = x  x = 0  2867 = 439·48 x  1 8 1
  3x – 5·4 = 3·6
3 6 3
In right CBD
126
 3x = 3·6 + 5·4 = 9·0
tan 12°20 = y 90
x=  3 0
126 3
0·2186 = y  Height of lamp post = 3m Ans.
126 P.Q. In the figure given alongside, from the top
 y= = 576·40 of a building AB, 60 metres high, the angles
0·2186
of depression of the top and bottom of a
(i) In first case (on the same side of the tower) vertical lamp post CD are observed to be
AB = BD – AD = y – x = 576·40 – 439·48 30° and 60° respectively. Find :
= 136·92m (i) the horizontal distance between AB and CD.
(ii) In second case (on the opposite side of the (ii) the height of the lamp post. (1992)
tower)
AB = BD + AD = y + x
= 576·40 + 439·48 = 1015·88 m Ans.
29. A man 1·8 m high stands at a distance of
3·6 m from a lamp post and casts a shadow
of 5·4 m on the ground. Find the height of
the lamp post.
Sol. AB is the lamp post. CD is the height of
man. BD is the distance of man from the
foot of the lamp and FD is the shadow of
man. Sol. AB is the building and AB = 60 m
CE | | DB. CD is the lamp post
Let CD = x
and angles of the
depression from the top
of the building to the
top and bottom of the
lamp post are 30° and
60° respectively.
576 Arun Deep's Understanding Math-10
Draw CE || DB Let AB = h and BD = x
Let BD = CE = y, From C, draw CE || DB then
EB = CD = x CE = DB = x and EB = CD = 8 m
 AE = 60 – x
and AE = AB – EB = (h – 8)
Now in right ABD,
Now in right ADB,
AB 60
tan  = BD  tan 60° = y AB h h
tan  =  tan 45º = 1 = x=h
60 DB x x
 3
y Similarly in right ACE,
60 AE h8
 y= 3 tan 30º = =
CE x
60 3 60 3
  1 h8
3 3 3 = 20 (1·732)
 = h= 3h–8 3
= 34·640 = 34·64 m 3 h
Now in right AEC  3h–h=8 3 h  3  1 = 8 3

8 3  3  1
AE AE 60  x
tan 30°= EC  BD  y 8 3
1 (60  x)
 h=
3 1
=
 3  1 3  1
 3  60 3

 60 = 3 (60 – x)  60 = 180 – 3x =



83 3
=

83  1.732
3 1 2
 3x = 180 – 60 = 120
= 4 (4.732) = 18.928 = 18.93 m
 x = 40
 and x = h = 18.93m
Hence (i) the distance between AB and CD =
34·64 m and (ii) height of the lamp post = 40 m  Height of multi-storeyed building = 18.93 m
30. The angles of depression of the top and and distance between the two buildings
the bottom of an 8 m tall building from the top = 18.93 m
of a multi-storeyed building are 30º and 45º 31. A pole of height 5 m is fixed on the top of a
respectively. Find the height of the multi-storeyed tower. The angle of elevation of the top of the
building and the distance between the two pole as observed from a point A on the ground
buildings, correct to two decimal places. is 60º and the angle of depression of the point
Sol.Let AB be the X A A from the top of the tower is 45º. Find the
30º
45º
multi-storeyed building and height of the tower. (Take 3 = 1.732).
P
CD be the building Sol. Let QR be the tower and PQ
The angles of depression h-8
be the pole on it
in from A, to C 30º Angle of elevation from P
F x E
and D are 30º and h to a point A is PAR = 60º X Q
8m 8m 45º
45º respectively and angle of depression 60º
45º
ACE = 30º and D x B from Q to A = 45º h
ADB = 45º (alternate angles) QAR = 45º (alternate angle)
CD = 8 m PQ = 5 m, 60º 45º
A R
577 Arun Deep's Understanding Math-10
Let QR = h m, then PQ = (5 + h)   PRL =  QPR = 30o
Now in right QAR, (Alternate angles)
QR h Let LR = x and TR = h
tan  =  tan 45º =  TQ = TR – QR = (h – 20) m
AR AR
Now in right  PRL,
h
 1=  AR = h
AR PL
tan  =
Again in right PAR LR

PR 5h 20 1

20
tan 60º = = tan 30o =  x x = 20 3 m
AR h x 3

5h Similarly in  PQT,,


 3 =  3h=5+h
h h  20
TQ h  20
tan 60o =  3  3
 h  3  1 = 5  h (1.732 – 1) = 5 PQ x 20 3

5 5000  20 3  3 = h – 20  20 × 3 = h – 20
0.732 h = 5  h = = h = 6.83
0.732 732 h = 60 + 20 = 80
 Height of tower = 6.83 m  Height of tower = 80m Ans.
32. A vertical pole and a vertical tower are on 33. From the top of a building 20 m high, the
the same level ground. From the top of the angle of elevation of the top of a monument
pole the angle of elevation of the top of the is 45° and the angle of depression of its foot
tower is 60o and the angle of depression of is 15°. Find the height of the monument.
the foot of the tower is 30o. Find the height Sol. Let AB be the building and AB = 20 m. and
of the tower if the height of the pole is 20 let let CD be the monument and let CD = x
m.
The distance between the building and the
Sol. Let TR is tower and PL is the pole on the monument be y,
same level, ground PL = 20m
From P, draw PQ || LR
then  TPQ = 60o and  QPR = 30o

h–20
O
60 Q
P
30
O
x h
Now in right BCD,
20m 20m
CD x
tan  = BD  tan 45° = y
O
30
L x R
578 Arun Deep's Understanding Math-10
x 34. In the adjoining figure, the shadow of a
1= y x=y ...(i) vertical tower on the level ground increases
Again in right ABD, by 10 m, when the altitude of the sun
changes from 45° to 30°. Find the height
AB 20 20
tan 15°    of the tower and give your answer, correct
BD y x
1
20 20 to of a metre. (2002)
10
 0·2679 = x  x = 0  2679 = 74·65 m
 Height of the monument = 74·64 m Ans.
P.Q. The angle of elevation of the top of an
unfinished tower at a point distant 120 m
from its base is 45°. How much higher
must the tower be raised so that its angle
of elevation at the same point may be 60°?
Sol. Let AB be the unfinished tower and AB = [Remark. Altitude of the sun means angle
120 m and angle of elevation = 45° of elevation of the sun.]
Let x be higher raised so that the angle of Sol. Let TR be the tower and TR = h ;
elevation becomes 60°. Let BR = x, AB = 10 m
Angles of elevation from the top of the
tower at A and B are 30° and 45°
respectively.

Now in right TAR


Let BC = y
TR h
In right ABC, tan  = AR  tan 30° = 10 + x
AB AB 120
tan  = CB  tan 45° = CB  y 1 h 10  x
   h = ...(i)
3 10  x 3
120
 1 = y  y = 120 m and in right TBR
Now in right DBC, TR h h
tan 45° = BR  x  1 = x
DB 120  x  x=h ...(ii)
tan 60° = CB  3
120
from (i) and (ii)
 120 3 = 120 + x  x = 120 3 – 120
10  h
h=  3 h = 10 + h
 x = 120 ( 3 – 1) = 120 (1·732 – 1·000) 3
= 120 × ·732 = 87·84  3 h – h = 10  (1·732 – 1) h = 10
 Raised tower = 87·84 m Ans.
579 Arun Deep's Understanding Math-10
 0·732 h = 10  EB = h
10  ED = EB + BD = h + 2 km
 h= = 13·66
0·732 Similarly in right CED
 Height of the tower = 13·7 m Ans.
CD
35. An aircraft is flying at a constant height tan 30° =
ED
with a speed of 360 km/h. From a point
on the ground, the angle of elevation of 1 h
the aircraft at an instant was observed to  =
3 h2
be 45°. After 20 seconds, the angle of
elevation was observed to b e 30°.
 3h=h+2
Determine the height at which the aircraft
is flying (use 3 = 1·732) 1·732h – h = 2  0·732h = 2
 0.732h = 2000 ( 2 km = 2000 m)
Sol. Speed of aircraft = 360 km/h

360  20 2000
Distance covered in 20 seconds = h=
60  60 0.732
= 2 km 2000  1000
E is the fixed point on the ground = = 2732 m (approx)
732
and CD is the position of AB in height of
aircraft MULTIPLE CHOICE QUESTIONS
Choose the correct answer from the given four
A 2 km C options (1 to 9):

1. If a kite is flying at a height of 40 3 metres


from the level ground, attached to a string
h h inclined at 60° to the horizontal, then the
45° length of the string is

30° (a) 80 m (b) 60 3 m


E B 2 km D
(c) 80 3 m (d) 120 m
Let AB = CD = h km
Sol. Let K is kite
Now in right AEB
Height of KT = 40 3 m
AB
tan  = Angle of elevation of string at the ground
EB
= 60°
h
tan 45° = Let length of string AK = x m
EB
KT 40 3
h Now sin 60° = =
 1= AK x
EB
580 Arun Deep's Understanding Math-10

3 40 3 1 75
= =
2 x 3 x

 x = 75 3 m (c)
K
3. A ladder 14 m long rests against a wall. If
the foot of the ladder is 7 m from the wall,
40 3 m
x then the angle of elevation is
(a) 15° (b) 30°
60° (c) 45° (d) 60°
A T Sol. Length of a ladder AB = 14 m

40 3  2 A
x= = 80 m
3

 Length of string = 80 m (a)


2. If the angle of depression of an object from

cm
a 75 m high tower is 30°, then the distance

14
of the object from the tower is

(a) 25 3 m (b) 50 3 m

(c) 75 3 m (d) 150 m B 7cm C

Sol. Height tower AB = 75 m Foot of the ladder is 7 m from the wall  is


C is an object on the ground and angle of the angle of elevation
depression from A is 30°.
BC
 cos  =
AB
A
7 1
= = = cos 60°
14 2
75m

  = 60° (d)

30° 4. If a pole 6 m high casts shadow 2 3 m long


C B on the ground, then the sun's elevation is
(a) 60° (b) 45°
Let BC = x m
(c) 30° (d) 90°
AB 75 Sol. Height of pole AB = 6 m
Now tan 30° = =
BC x
and its shadow BC = 2 3 m
581 Arun Deep's Understanding Math-10

AB h
A tan  = =
BC 3h

1
= = tan 30°
3
  = 30°

6m
Angle of elevation = 30° (b)
6. In ABC, A = 30° and B = 90°. If AC =
8 cm, then its area is
C 2 3 B (a) 16 3 cm2 (b) 16 m2

(c) 8 3 cm 2 (d) 6 3 cm 2
AB 6 3
tan  = = = Sol. In ABC, A = 30°, B = 90°
BC 2 3 3
AC = 8 cm
3 3
= C
3 3

3 3 m
= 8c
3

= 3 = tan 60° 30°


  = 60° A B
 Angle of elevation = 60° (a)
5. If the length of the shadow of a tower is BC
 sin 30° =
AC
3 times that of its height, then the angle
of elevation of the sun is 1 BC
 =
(a) 15° (b) 30° 2 8
(c) 45° (d) 60°
8
Sol. Let height of a tower AB = h m BC = = 4 cm
2
Then its shadow BC = 3hm
AB
cos 30° =
A AC

3 AB
 =
2 8
h
8 3
 AB = = 4 3 cm
2

1
C 3h B Now area ABC = AB × BC
2
582 Arun Deep's Understanding Math-10

1
= × 4 3 × 4 cm2 A
2

= 8 3 cm2 (c)
h
P.Q. In the given figure, if the angle of elevation B
x
is 60° and the distance AB = 10 3 m, then
the height of the tower is 30°
A’ 10m C
(a) 20 3 cm (b) 10 m
Sol. From the figure, AC is the height of tree and
(c) 30 m (d) 30 3 m from B, it was broken
 AB = AC
C Angle of elevation = 30°
AC = 10 m
Let AC = h m
and AB = x m
Tower
BC = h – x m

A C
cos  =
60° A B
A B
10
cos 30° =
x
Sol. In the given figure,

A = 60°, AB = 10 3 m 3 10
=
2 x
Let BC = h

h h 2  10
 tan 60° =   x=
10 3 3 = 10 3 3

 h = 10 3 × 20
3 = 10 × 3 = 30 m (c) = m (b)
3
P.Q. The top of a broken tree has its top touching
the ground (shown in the given figure) at a CHAPTER TEST
distance of 10 m from the bottom. If the
angle made by the broken part with ground 1. The angle of elevation of the top of a tower
is 30°, then the length of the broken part is from a point A (on the ground) is 30°. On
walking 50 m towards the tower, the angle
20
(a) 10 3 m (b) m of elevation is found to be 60°. Calculate
3 (i) the height of the tower (correct to one
decimal place).
(c) 20 m (d) 20 3 m
583 Arun Deep's Understanding Math-10
(ii) the distance of the tower from A. and B are 60° and 45° respectively.
Sol. Let TR be AC = 3000 m
the tower Let AB = x
and A is a  BC = 3000 – x
point on the Let PC = y
ground and Now in right APC,
angle of
elevation of AC 3000
tan  = PC  tan 60° = y
the top of tower = 30°
AB = 50 m 3000 3000
 3
and from B, the angle of elevation is 60°. y  y= 3
...(i)
Let TR = h and AR = x Again in right BPC,
 BR = x – 50
BC 3000  x
Now in right ATR, tan 45° = PC  1 = y
TR h
tan  = AR  tan 30° = x  y = 3000 – x
3000
1

h  = 3000 – x [from (i)]
 3 x
 x = 3h ...(i) 3

and in right BTR, 3000 3000  3


 x = 3000 – 3
3000 – 3 3
TR h h
tan 60° = BR  x  50  3
x  50 = 3000 – 1000 (1·732) = 3000 – 1732 = 1268
 Distance between the two planes = 1268 m
 h = 3 (x – 50) ...(ii) 3. A 7m long flagstaff is fixed on the top of a
from (i) and (ii) tower. From a point on the ground, the
h= 3 ( 3 h – 50)  h = 3h – 50 3 angles of elevation of the top and bottom
of the flagstaff are 45º and 36º respectively.
2h = 50 3  h = 25 3 Find the height of the tower correct to one
h = 25 (1·732) = 43·3 place of demical.
Substituting the value of h in (i) Sol. Let TR be the tower
x = 3 × 25 3 = 25 × 3 = 75 and PT is the flag on
 Height of tower = 43·3 m it such that PT = 7m
and distance of A from the foot of the tower Let TR = h and AR =
= 75 m Ans. x
2. An aeroplane 3000 m high passes vertically Angles of elevation
above another aeroplane at an instant when from P and T are 45°
the angles of elevation of the two aeroplanes and 36° respectively.
from the same point on the ground are 60° Now in right PAR,
and 45° respectively. PR 7+h
Find the vertical tan  = AR  tan 45° = x
distance between the
two planes. 7+h
 1= x
Sol. Let A and B are two
aeroplanes and P is a  x=7+h ...(i)
point on the ground Again in right TAR
such that angles of TR h
elevations from A tan 36° = AR  x
584 Arun Deep's Understanding Math-10
h h = 36·24
 0·7265 = x
 Height of tower = 36·24 m Ans.
 h = x (0·7265) ...(ii) 5. A boy 1·54 m tall can just see the sun over
From (i) and (ii) a wall 3·64 m high which is 2·1 m away
h = (7 + h) (0·7265) from him. Find the angle of elevation of
the sun.
 h = 7 × 0·7265 + ·7265 h
Sol. Let AB be the boy and CD be the wall which
 h – ·7265 h = 7 × 0·7265
is at a distance of 2·1 m
 (1 – ·7265) h = 7 × 0·7265
 0·2735 h = 7 × 0·7265
7  0  7265 7  7265
h= 
2735 2735
= 18·59
 Height of tower = 18·59 = 18·6 m Ans.
4. A boy 1·6 m tall is 20 m away from a tower
and observes that the angle of elevation of
the top of the tower is 60°. Find the height
of the tower.
Sol. Let AB be the boy and TR be the tower
 AB = 1·6 m
Let TR = h Then AB = 1·54 m, CD = 3·64 m
from A, show AE | | BR BD = 2·1 m
 ER = AB = 1·6 m Draw AE | | BD, then
TE = h – 1·6 ED = 1·54 m,
AE = BR = 20 m CE = 3·64 – 1·54 = 2·1 m
and AE = BD = 2·1 m,
Now in right CAE,
CE 2.1 21
tan     =1
AE 2.1 21
 = 45° ( tan 45° = 1)
 Angle of elevation of the sun = 45º Ans.
P.Q. In the adjoining figure, the angle of
Now in right TAE elevation of the top P of a vertical tower
from a point X is 60° ; at a point Y, 40 m
TE h  1 6
tan  = AE  tan 60° = 20 vertically above X, the angle of elevation is
45°. Find
h  1 6
 3  h – 1·6 = 20 3 (i) the height of the tower PQ
20
(ii) the distance XQ
h = 20 3 + 1·6 = 20 (1·732) + 1·6
(Give your answer to the nearest metre)
h = 34·640 + 1·6 = 34·64 + 1·60
585 Arun Deep's Understanding Math-10
 ·732 h = 69·280
69  280 69280
h=   94  64
732 732
 Height of the tower = 94·64 m = 95 m
and distance XQ = h – y = 95 – 40

(from (ii))
= 55 m Ans.
6. An aeroplane is flying horizontally 1 km
Sol. Let PQ be the tower and let PQ = h above the ground is observed at an elevation
and XQ = YR = y of 60°. After 10 seconds, its elevation is
XY = 40 m observed to be 30°. Find the speed of the
aeroplane in km/hr.
 PR = h – 40
Sol. A and D are the two positions of aeroplane ;
AB is the height and P is the point
 AB = 1 km, Let AD = x and PB = y
and angles of elevation from A and D at
point P are 60° and 30° respectively.

Now in right PXQ,


PQ
tan = XQ

h h Draw DC  PB produced.
 tan 60° = y  3
y  DC = AB = 1 km.
h Now in right APB,
 y= 3
...(i)
AB 1
Again in right PYR, tan  = PB  tan 60° = y

PR h  40 h  40 1 1
 1=  3
y  y = 3
tan 45° = YR = y y ...(i)
 y = h – 40 ...(ii) Again in right DPC
From (i) and (ii) DC 1 1 1
tan 30° = PC  x  y  3  x  y
h
h – 40 = 3
 3 h – 40 3 = h
x+y= 3 ...(ii)
 3 h – h = 40 3 From (i) and (ii)
 (1·732 – 1) h 1
= 40 (1·732) x+ = 3
3
586 Arun Deep's Understanding Math-10
1 31 2 Again in right ADE,
 x= 3 –
3
= 3  3
DE 16 1 16
2 × 3 2 (1. 732 ) 3. 464 tan 30° = AD  x  3  x
 x   km
3× 3 3 3
 x = 16 3 ...(ii)
Thus this distance covered in 10 seconds
From (i) and (ii)
 Speed of aeroplane (in km/hr)
3  464 60  60 3464 3600 h – 16 = 16 3  h = 16 3 + 16
=   
3 10 3  1000 10
 h = 16 ( 3 + 1) = 16 (1·732 + 1)
3464  36 3464  12 41568 = 16 × 2·732 = 43·712 = 43·71 m
=  
300 100 100
and x = h – 16 = 43·71 – 16 = 27·71
= 415·68 km/hr. Ans.
 Distance of cliff = 27·71 m
7. A man on the deck of a ship is 16 m above
the water level. He observes that the angle and height of cliff = 43·71 m Ans.
of elevation of the top of a cliff is 45° and 8. There is a small island in between a river
the angle of depression of the base is 30°. 100 metres wide. A tall tree stands on the
Calculate the distance of the cliff from the island. P and Q are points directly opposite
ship and the height of the cliff. to each other on the two banks, and in the
Sol. Let A is the man on the deck of a ship B line with the tree. If the angles of elevation
and CE is the cliff. of the top of the tree from P and Q are 30°
 AB = 16 m and angle of elevation from the and 45° respectively, find the height of the
top of the cliff in 45° and angle of tree.
depression at the base of the cliff is 30°. Sol. The width of river (PQ) = 100 m.
Let CE = h, AD = x, then B is the island and AB is the tree on it.
CD = h – 16, AD = BE = x
Now in right CAD

Angles of elevation from A to P and Q are


30° and 45° respectively.
Let AB = h and PB = x, then
BQ = 100 – x
Now in right APB,

CD h  16 AB h
tan  = AD  tan 45° = tan  = PB  tan 30° = x
x
h  16 1 h
 1= x  x = h – 16 ...(i)    x= 3h ...(i)
3 x
587 Arun Deep's Understanding Math-10
Again in right ABQ, Let BC = h, PQ = 20 = CA, AR = (h + 20) m
AB h h and CR = h + 20 + 20 = h + 40 m
tan 45° = BQ  100  x  1 = 100  x Let PC = QA = x
 h = 100 – x ...(i) Now in right PCB
From (i) and (ii) BC 1 h
tan 30° =  = x= 3hm
h = 100 – 3h  h + 3h = 100 PC 3 x
...(i)
100
 (1 + 1·732) h = 100  h = 2  732 Similarly in right PCR

100  1000 100000 CR h  40


 h= = 2732  36  6 tan 60° =  3 =
2732 PC x
 Height of the tree = 36·6 m Ans. h  40
9. A man standing on the deck of the ship which = 3 [From (i)]
3h
is 20 m above the sea-level, observes the
angle of elevation of a bird as 30° and the h + 40 = 3 × 3 h = 3h
angle of depression of its reflection in the
sea as 60°. Find the height of the bird. 40
3h – h = 40  2h = 40  h = = 20
Sol. Let P is the man standing on the deck of a 2
ship which is 20 m above the sea level and B From sea level height of the brid
is the bird.
= 20 m + h = 20 + 20 = 40 m Ans.
Now angle of elevation of the bird from P
= 30°
and angle of depression from P to the shadow
of the bird in the sea

30°
P x C
60°
20m
20m

Q x A
h+20m

60°
S R
21
Measures of Central Tendency
POINTS TO REMEMBER

1. Measures of Central Tendency (Average)


It can be in three forms of measures (i) Mean (ii) Median (iii) Mode
2. Mean or Arithmetic Mean
(i) Mean of ungrouped data
Let x1, x2, x3, x4,..........xn be n variates
n
x1  x2  x3  x4.......... xn 1
then Mean = n or x
n  xi
i 1

when is x symbole of mean and i is the index number and  (sigma) is the sum.
(ii) Mean of grouped data (Direct Method)
Let x1, x2, x3......xn are variates or observations and f1, f2, f3........fn are their frequencies
respectively,

then Mean =
f1x1  f2 x2  f3x3  ........... f x xn
or x
 fi xi
f1  f2  f3 ........... fn
 fi
This method is called the Direct Method.

(iii) Mean of grouped data (Short Cut Method) x A


 fi di
 fi
Where A is assumed mean, xi is the class mark, di is the deviation i.e.
di = xi – A, fi is the frequency.
(iv) Mean of grouped data (Step Deviation Method)

Mean = A + h ×
 f i ui
 fi
Where A = Assumed mean h = Class interval or width of class
xi  A
ui = h This method is called step deviation method.
3. Median. It is the central value of a given data arranged in ascending or decending order.
(i) If x1, x2, x3............xn are arranged in any order, then
n1
Median = th term if n is odd.
2

588 Arun Deep's Understanding Math-10


589 Arun Deep's Understanding Math-10

or mean of
FG n IJ and FG n  1IJ th terms if n is even.
H 2K H 2 K
4. Quartiles. Quartiles are the values of a statistical data which divides the whole set of observations
into four equal parts.
(i) Lower or first quartile. If the variates are arranged in ascending order then observations lying
midway between the lower extreme and the median is called the lower or the first quartile and is
denoted by Q1.
(ii) Upper or third quartile. If the variates are arranged in ascending order, then the observations
lying midway between the median and upper extreme is called the upper or third quartile and is
denoted by Q3.
If there are n observations or variates x1, x2, x3..........xn in ascending order, then
n1 n
Lower quartile (Q1) = th observations if n is odd. = th observation if n is even.
4 4

3 ( n  1) 3n
Upper quartile (Q3) = th observation if n is odd = th observation if n is even.
4 4
Note. Middle quartile (Q2) is the median.
5. Measures of Dispersion.
Dispersion means the spread of data. It indicates the extent to which the individual measures
differ from an average. There are many ways in which the despersion can be measured. They are
– range, interquartile range and semi-interquartile range etc.
Range
The difference between the maximum and minimum values of a variable is called its range.
6. Interquartile-range.
The difference between the upper quartile (Q3) and the lower quartile (Q1) is called the interquartile-
range.
Q3  Q1
Thus, interquartile-range = Q3 – Q1 and semi-interquartile range = 2 .
Remarks :
1. Deciles. The deciles divide the whole set of variates (arranged in ascending order) into ten equal
parts. These are denoted by D1, D2, D3,.........,D9..
2. Percentiles. The percentiles divide the whole set of variates (arranged in ascending order) into
hundred equal parts. These are denoted by P1, P2, P3,.........,P99.
7. Estimation of median and quartiles from ogives
In a continuous frequency distribution, the median and the quartiles (lower and upper) can be
estimated from the ogive of the given (continuous) frequency distribution.
Procedure.
(i) Construct cumulative frequency table. Let n be the sum of frequencies.
(ii) Draw ogive for the given distribution.
590 Arun Deep's Understanding Math-10
(iii) To find the median. EXERCISE 21.1
Locate a point along y-axis representing 1. (a) Calculate the arithmetic mean of 5·7,
n 1 6·6, 7·2, 9·3, 6·2.
frequency equal to (a) if n is odd
2 (b) The weights (in kg) of 8 new born babies
are 3, 3.2, 3.4, 3.5, 4, 3.6, 4.1, 3.2. Find
LM FG n
1 n
and (b) 2 2  2  1
IJ OP if n is even. the mean weight of the babies.
N H KQ Sol. (a) Sum of 5 observations
Through this point, draw a horizontal line = 5·7 + 6·6 + 7·2 + 9·3 + 6·2 = 35·0
to meet the ogive, and through this point of Sum of five observations
the ogive, draw a vertical line to meet the x- Mean =
No. of observation
axis at the point M (say).
35  0
The variate at the point M is the required  Mean = 7
5
median :
(b) Weights of 8 babies (in kg) are
The class in which the median lies is called
the median class. 3, 3.2, 3.4, 3.5, 4, 3.6, 4.1, 3.2
(iv) To find the lower quartile.  Total weights of 8 babies
= 3 + 3.2 + 3.4 + 3.5 + 4 + 3.6 + 4.1 + 3.2
Locate a point along y-axis representing
= 28.0 kg
n 1
frequency equal to (a) if n is odd  xi
4
 Mean weight =
n
n
and (b) if n is even.
4 28.0
= (Here n = 8)
Through this point, draw a horizontal line 8
to meet the ogive, and through this point of = 3.5 kg
the ogive, draw a vertical line to meet the x- 2. The marks obtained by 15 students in a class
axis at the point N (say). test are 12, 14, 07, 09, 23, 11, 08, 13, 11,
The variate at the point N is the required 19, 16, 24, 17, 03, 20 find
lower quartile. (i) the mean of their marks.
(v) To find the upper quartile. (ii) the mean of their marks when the marks of
Locate a point along y-axis representing each student are increased by 4.
3 (n  1) (iii) the mean of their marks when 2 marks are
frequency equal to (a) if n is odd deducted from the marks of each student.
4
(iv) the mean of their marks when the marks of
3n each student are doubled.
and (b) if n is even.
4 Sol. Sum of marks of 15 students.
Now proceed as in 4 to find the required
 xi = 12 +14 + 07 + 09 + 23 + 11 + 08 +
upper quartile.
13 + 11 + 19 + 16 + 24 + 17 + 03 + 20 = 207
Remark. If, in a problem, frequency
Here n = 15
distribution is discontinuous, first convert
it into continuous distribution and then find  xi 207
median and quartiles as explained above. (i) Mean = = 15 = 13·8
n
591 Arun Deep's Understanding Math-10
(ii) By increasing 4 marks in each student (b) In a class test, the mean of marks scored
then increased marks = 15 × 4 = 60 by a class of 40 students was calculated
New sum = 207 + 60 = 267 as 18.2. Later on, it was detected that
marks of one student was wrongly
267
 New mean = 15  17  8 copied as 21 instead of 29. Find the
correct mean.
(iii) By deducting 2 marks from each students,
Sol. (a) Mean age of 33 students = 13 years
then total deduction = 15 × 2 = 30
New sum = 207 – 30 = 177  Total age = 13 × 33 = 429 years
After leaving one girl, the mean of 32
177
New mean = 15  11  8 15 207
students  12  years.
(iv) The marks being doubled of each student 16 16
then the new sum = 207 × 2 = 414 207
 Total age of 32 students   32
414 16
 New mean = 15  27  6
= 414 years
3.(a) The mean of the numbers 6, y, 7, x, 14 is Hence, the age of the girl = 429 – 414
8. Express y in terms of x.
= 15 years Ans.
(b) The mean of 9 variates is 11. If eight of
them are 7, 12, 9, 14, 21, 3, 8 and 15 find (b) Mean of marks obtained by 40 students
the 9th variate. = 18.2
Sol. (a) Sum of numbers = 6 + y + 7 + x + 14  Total marks obtained by than = 18.2 × 40
= 27 + x + y ...(i) = 728
But mean of 5 numbers = 8 Difference of marks copited wrongly = 29 – 21
 Sum = 8 × 5 = 40 ...(ii) =8
From (i) and (ii)  Actual total marks = 728 + 8 = 736
27 + x + y = 40
736
 x + y = 40 – 27 = 13 New mean = = 18.4 Ans.
40
 y = 13 – x Ans.
5. Find the mean of 25 given numbers when
 xi  xi
(b) Mean of 9 variates = 11 =  the mean of 10 of them is 13 and the mean
n 9
of the remaining numbers is 18.
 Total sum =  xi = 11 × 9 = 99 Sol. Mean of 10 numbers = 13
But sum of 8 of these variates  Sum = 13 × 10 = 130
= 7 + 12 + 9 + 14 + 21 + 3 + 8 + 15
and mean of remaining 15 numbers = 18
= 89
 9th variate = 99 – 89 = 10 Ans.  their sum = 18 × 15 = 270
4. (a) The mean age of 33 students of a class  Total sum of 25 numbers = 130 + 270
is 13 years. If one girl leaves the class,
= 400
15
the mean becomes 12 years. What 400
16 Mean of 25 numbers = 25  16 Ans.
is the age of the girl ?
592 Arun Deep's Understanding Math-10

6. Find the mean of the following distribution:


Number 5 10 15 20 25 30 35
Frequency 1 2 5 6 3 2 1
Sol. x f f.x
5 1 5
10 2 20
15 5 75  f x  390  19  5

20
25
6
3
120
75
Mean =
 f 20 Ans.

30 2 60
35 1 35
 f = 20  fx = 390
7. The contents of 100 match boxes were checked to determine the number of matches they
contained.
No. of matches 35 36 37 38 39 40 41
No. of boxes 6 10 18 25 21 12 8
(i) Calculate, correct to one decimal place, the mean number of matches per box.
(ii) Determine how many extra matches would have to be added to the total contents of the 100
boxes to bring the mean upto exactly 39 matches. (1997)
Sol. No. of matches No. of boxes
(x) (f) f.x
35 6 210
36 10 360
37 18 666
38 25 950
39 21 819
40 12 480
41 8 328
 f = 100  fx = 3813

(i) Mean =
 f x  3813  38  13  38  1
 f 100
(ii) New Mean = 39
 Total sum = 39 × 100 = 3900
 New matches to be added = 3900 – 3813 = 87 Ans.
593 Arun Deep's Understanding Math-10
P.Q. Calculate the mean for the following distribution :
Pocket money (in Rs.) 60 70 80 90 100 110 120
No. of students 2 6 13 22 24 10 3

Sol. Pocket money (in Rs.) No. of students


(x) (f) f.x
60 2 120
70 6 420
80 13 1040
90 22 1980
100 24 2400
110 10 1100
120 3 360
 f = 80  fx = 7420

Mean 
 f x  7420  Rs. 92. 75 Ans.
 f 80
P.Q. Six coins were tossed 1000 times, and at each toss the number of heads were counted and the
results were recorded as under :
No. of heads 6 5 4 3 2 1 0
No. of tosses 20 25 160 283 338 140 34

Calculate the mean for this distribution.

Sol. No. of heads No. of tosses


(x) (f) f.x
6 20 120
5 25 125
4 160 640
3 283 849
2 338 676
1 140 140
0 34 0
 f = 1000  f = 2550

Mean =
 f x  2550  2  55 Ans.
 f 1000
594 Arun Deep's Understanding Math-10
8. Find the mean for the following distribution by short cut method :

Numbers 60 61 62 63 64 65 66
Cumulative frequency 8 18 33 40 49 55 60

Sol. Numbers Cumulative frequency frequency


(x) c.f. f f.x
60 8 8 480
61 18 10 610
62 33 15 930
63 40 7 441
64 49 9 576
65 55 6 390
66 60 5 330
 f = 60  fx = 3757

Mean =
 f x  3757  62  616  62  62 Ans.
 f 60
9.
Catagory A B C D E F G
Wages (in Rs.) per day 50 60 70 80 90 100 110
No. of workers 2 4 8 12 10 6 8
(i) Calculate the mean wage correct to the nearest rupee (1995)
(ii) If the number of workers in each catagory is doubled, what would be the new mean wage ?
Sol. Catagory Wages (in Rs.) No. of workers f.x
x f
A 50 2 100
B 60 4 240
C 70 8 560
D 80 12 960
E 90 10 900
F 100 6 600
G 110 8 880
 f = 50  fx = 4240

(i) Mean =
 f x  4240  84  80  85
 f 50
595 Arun Deep's Understanding Math-10
(ii) If the workers are doubled, then
Total number of workers = 50 × 2 = 100
Total wage will also be doubled
 Total wages = 4240 × 2 = 8480
8480
 New mean =  84  80  85 Ans.
100
10. If the mean of the following distribution is 7.5, find the missing frequency 'f '.
Variate 5 6 7 8 9 10 11 12
Frequency 20 17 f 10 8 6 7 6
Sol. Variate (x) Frequency (f) fx

5 20 100
6 17 102
7 f 7f
8 10 80
9 8 72
10 6 60
11 7 77
12 6 72
 f =74 + f  fx = 563 + 7f

 fx
 Mean =  f

563  7 f 8  10
 7.5 = 74  f  555 + 7.5f = 563 + 7f  0.5f = 8  f=  f = 16
5
 missing frequency (f) = 16 Ans.
P.Q. Find the value of the missing variate for the following distribution whose mean is 10
Variate (xi) 5 7 9 11 – 15 20
Frequency ( fi ) 4 4 4 7 3 2 1
Sol. Let missing variate be x, then
Variate Frequency
(x) (f) f.x
5 4 20
7 4 28
9 4 36
11 7 77
x 3 3x
15 2 30
20 1 20
 f = 25  fx = 211 + 3x
596 Arun Deep's Understanding Math-10

 f x 211  3 x
Mean = 
f 25
But mean = 10 (given)

211  3x
  10
25
 211 + 3x = 250
 3x = 250 – 211 = 39

39
 x= = 13
3
 Missing variate = 13 Ans.

11. Marks obtained by 40 students in a short assessment are given below, where a and b are two
missing data.
Marks 5 6 7 8 9
No. of Students 6 a 16 13 b

If the mean of the distribution is 7.2, find a and b.

Sol. Marks (x) No. students (f) fx


5 6 30
6 a 6a
7 16 112
8 13 104
9 b 9b
 f = 35+a+b=40  f x = 246+6a+9b

Since, 35 + a + b = 40
 a + b = 40 – 35
 a + b = 5  b = 5 – a ...(i)
  fx 
 Also, mean = f

246  6a  9b
 7.2 =
40
 246 + 6a + 9b = 288  6a + 9b = 42
 2a + 3b = 14 ...(ii)
 2a + 3 (5 – a) = 14
597 Arun Deep's Understanding Math-10
 2a + 15 – 3a = 14
 a = 1
Since, a + b = 5  1 + b = 5
b=4
Solving (i) and (ii), we get a = 1, b = 4
P.Q. Find the mean of the following distribution :-
Class Interval 0–10 10–20 20–30 30–40 40–50
Frequency 10 6 8 12 5 (2007)
Sol. Class Class mark Frequency fi x i
interval xi fi

0–10 5 10 50
10–20 15 6 90
20–30 25 8 200
30–40 35 12 420
40–50 45 5 225

 fi = 41  fixi = 985

 f i xi 985
 Mean = 
 fi 41 = 24.02 (approx.)
12. Calculate the mean of the following distribution: (2015)

Class Interval 0–10 10–20 20–30 30–40 40–50 50-60

Frequency 8 5 12 35 24 16

Sol. Consider the following distribution :

Class interval Frequency Class mark fi xi


fi xi
0–10 8 5 40
10–20 5 15 75
20–30 12 25 300
30–40 35 35 1225
40–50 24 45 1080
50-60 16 55 880

 fi = 100  fi xi = 3600
598 Arun Deep's Understanding Math-10
 fi xi 3600
Mean =   36
 fi 100
13. Calculate the mean of the following distribution using step deviation method :
Marks 0–10 10–20 20–30 30–40 40–50 50–60
Number of students 10 9 25 30 16 10

di
Sol. Marks Mid values No. of students di = xi – A ti = ; f it i
c
(xi) (f i ) c = 10
0–10 5 10 –20 –2 –20
10–20 15 9 –10 –1 –9
20–30 25=A 25 0 0 0
30–40 35 30 10 1 30
40–50 45 16 20 2 32
50–60 55 10 30 3 30
 fi  100  fi ti  63

Let A = 25 and c = 10
 fi ti 63
 Mean = A +  c  25   10  25  6.3  31.3
 fi 100
14. The data on the number of patients attending a hospital in a month are given below. Find the
average (mean) number of patients attending the hospital in a month by using the shortcut
method.
Take the assumed mean as 45. Give your answer correct to 2 decimal places.
Number of patients 10-20 20-30 30-40 40-50 50-60 60-70
Number of Days 5 2 7 9 2 5
Sol. A = 45
No. of Patients No. of day x d=x–A fd
10 – 20 5 15 – 30 – 150
20 – 30 2 25 – 20 – 40
30 – 40 7 35 – 10 – 70
40 – 50 9 45 0 0
50 – 60 2 55 10 20
60 – 70 5 65 20 100
f = 30 f = – 140

fd (140)
Mean = A +  45   45  4.67  40.33
f 30
599 Arun Deep's Understanding Math-10
15. The following table gives the wages of workers in a factory :
Wages in ` No. of Workers
45—50 5
50—55 8
55—60 30
60—65 25
65—70 14
70—75 12
75—80 6
Calculate the mean by the short cut method.
Sol. Wages in ` No. of Workers Mid. mark d = (x – A) f×d
(f) x A = 625
45—50 5 47.5 –15 –75
50—55 8 52.5 –10 –80
55—60 30 57.5 –5 –150
60—65 25 62.5 0 0
65—70 14 67.5 5 70
70—75 12 72.5 10 120
75—80 6 77.5 15 90
f = 100 fd = –25
 fd 25
Mean = A + = 62.5 + = 62.50 – .25 = 62.25
f 100
16. Calculate the mean of the distribution given below using the short cut method.
Marks 11-20 21-30 31-40 41-50 51-60 61-70 71-80
No. of students 2 6 10 12 9 7 4
xi  A
Sol. Marks Frequency (f) Mid Value (x) di = xi – A ui = f × di
10
A = 45.5
11-20 2 15.5 –30 –3 –60
21-30 6 25.5 –20 –2 –120
31-40 10 35.5 –10 –1 –100
41-50 12 45.5 = A 0 0 0
51-60 9 55.5 10 1 90
61-70 7 65.5 20 2 140
71-80 4 75.5 30 3 120
f = 50 fdi = 70
A = 45.5
 fd i 70
Mean = A + = 45.5 + = 45.5 + 1.4 = 46.9
f 50
600 Arun Deep's Understanding Math-10
17. A class teacher has the following absentee record of 40 students of a class for the whole term.
Find the mean number of days a students was absent.
No. of days 0-6 6-10 10-14 14-20 20-28 28-38 38-40
No. of students 11 10 7 4 4 3 1

Sol. No. of days Mid value No. of students f×x


(x) (f)
0-6 3 11 33
6-10 8 10 80
10-14 12 7 84
14-20 17 4 68
20-28 24 4 96
28-38 33 3 99
38-40 39 1 39
Total f = 40 fx  499

f x 499
Mean =  f = = 12·475
40
P.Q. The mean of the following distribution is 23.4. Find the value of p.

Class Intervals 0–8 8–16 16–24 24–32 32–40 40–48

Frequency 5 3 10 p 4 2

Sol. Class Frequency Class mark f×x


intervals (f)

0–8 5 4 20
8–16 3 12 36
16–24 10 20 200
24–32 p 28 448 fx
Mean =  f
32–40 4 36 144
40–48 2 44 88
Total f  24  p f x  488  28 p

488  28 p
 23.4 =  23.4 (24 + p) = 488 + 27p 561.23.4 p = 488 + 28p
24  p
73.6 736
 28p – 23.4p = 561.6 – 488  4.6p = 73.6 p =   16 Hence p = 16 Ans.
4.6 46
601 Arun Deep's Understanding Math-10
18. If the mean of the following distribution is 24, find the value of ‘a’. (2018)

Marks No. of students

0–10 7

10–20 a

20–30 8

30–40 10

40–50 5

(7  5)  (a 15)  (8  25)  (10  35)  (5  45)


Sol. Mean =
7  a  8  10  5
[ No. of students × Mid value of Marks]

35  15a  200  350  225


 = 24
30  a

810 + 15a = 24(30 + a)  810 + 15a = 720 + 24a


 90 = 9a  a = 10 Ans.
P.Q. The mean of the following distribution is 50 and the sum of all the frequencies is 120. Find the
values of p and q.

Class intervals 0-20 20-40 40-60 60-80 80-100

Frequency 17 p 32 q 19

Sol. Mean = 50, Total number of frequency = 120

Class intervals frequency (fi) Class mark (xi) fi × x i

0-20 17 10 170
20-40 p 30 30p
40-60 32 50 1600
60-80 q 70 70q
80-100 19 90 1710

 f i  68+p+q  fi x i  3480 + 30p + 70q


602 Arun Deep's Understanding Math-10
 fx
Mean =
 f and 68 + p + q = 120

3480  30 p  70 q
 p + q = 120 – 68 = 52 and 50 =
120
 6000= 3480 + 30p + 70q
 30p + 70q = 6000 – 3480 = 2520
 3p + 7q = 252 ....(i)
and p + q = 52 ....(ii)
Multiplying (i) by 1 and (ii) by 7 and subtracting (i) from (ii)
7p + 7r = 364
3p + 7q = 252
– – –
4p = 112

 112
 p= = 28  q = 52 – p = 52 – 28 = 24
4
Hence p = 28, q = 24 Ans.
19. The mean of the following frequency distribution is 57.6 and the sum of all the frequencies is
50. Find the values of p and q.

Class 0–20 20–40 40–60 60–80 80–100 100–120


Frequency 7 p 12 q 8 5

Sol. Mean = 57.6


and sum of all frequencies = 50

Class Frequency (f) Class mark (x) f×x

0–20 7 10 70
20–40 p 30 30p
40–60 12 50 600
60–80 q 70 70q
80–100 8 90 720
100–120 5 110 550

 f = 32+p+q=50  f x  1940+30p+70q

32 + p + q = 50 p + q = 50 – 32
 p + q = 18 ....(i)
 f x
Mean =
f
603 Arun Deep's Understanding Math-10

1940  30 p  70q
 57.6 =  2880 = 1940 + 30p + 70q
50
 30p + 70q = 2880 – 1940 = 940
 3p + 7q = 94 ....(ii)
Multiplying (i) by 7 and (ii) by 1 and subtracting (ii) from (i)
7p + 7q = 126
3p + 7q = 94
– – –
4p = 32

32
 p= =8
4
But p + q = 18  q = 18 – p = 18 – 8 = 10
Hence p = 8, q = 10 Ans.

20. The following table gives the life time in days of 100 electricity tubes of a certain make :
Life time in days No. of tubes
less than 50 8
less than 100 23
less than 150 55
less than 200 81
less than 250 93
less than 300 100

Find the mean life time of electricity tubes.

Sol. Life time (in days) c.f. Frequency Class Mark xA fu
u
h
(Class intervals) (f) (x) h = 50

0–50 8 8 25 –3 –24
50–100 23 15 75 –2 –30
100–150 55 32 125 –1 –32
150–200 81 26 175 0 0
200–250 93 12 225 1 12
250–300 100 7 275 2 14

Total  f = 100  f x = –60


604 Arun Deep's Understanding Math-10
Let assumed mean (A) = 175 and h = 50
By step deviation method, we have

 Mean = A + h ×
 fu  175  50  60 = 175 – 30 = 145 days Ans.
f 100

21. Using the information given in the adjoining histogram, calculate the mean correct to one decimal
place.
Sol. From the histogram given, we represent the information in the following table :

Class interval Frequency Class Mark f (x)


(f) (x)
20–30 3 25 75
30–40 5 35 175
40–50 12 45 540
50–60 9 55 495
60–70 4 65 260
 f = 33  f x = 1545

Mean
 fx  1545  46  81  46  8 Ans.
 f 33
EXERCISE 21.2
1. A student scored the following marks in 11 questions of a question paper :
3, 4, 7, 2, 5, 6, 1, 8, 2, 5, 7 Find the median marks.
Sol. Arranging in the ascending order, 1, 2, 2, 3, 4, 5, 5, 6, 7, 7, 8
Here, n = 11 i.e. odd,
605 Arun Deep's Understanding Math-10
n  1 11  1 12
 The middle term =   = 6th term
2 2 2
 Median = 5 Ans.
2. For the following set of numbers, find the median : 10, 75, 3, 81, 17, 27, 4, 48, 12, 47, 9, 15.
Sol. Arranging the given numbers in ascending order : 3, 4, 9, 10, 12, 15, 17, 27, 47, 48, 75, 81.
Here n = 12
... No. of numbers is even.

n n  
 Median = Mean of  th    1 th  terms
 2 2  

15  17 32
= Mean of 6th and 7th term    16 Ans.
2 2
3. Calculate the mean and the median of the numbers : 2, 1, 0, 3, 1, 2, 3, 4, 3, 5
Sol. Writing in ascending order 0, 1, 1, 2, 2, 3, 3, 3, 4, 5
Here, n = 10 which is even

1 1
 Mean = ( xi ) = 10 (0 + 1 + 1 + 2 + 2 + 3 + 3 + 3 + 4 + 5)
n
1
= 10 (24) = 2·4

and Median =
LM
1 n FG IJ
th term 
n
 1 th term
OP
N
2 2 H 2 K Q
1 L 10 F 10  1IJ th termOP
= M th term  GH K
2 N2 2 Q
1 1
= 2 (5th term + 6th term) = 2 (2 + 3)

5
= 2 = 2·5 Ans.
4. The median of the observations 11, 12, 14, (x – 2), (x + 4), (x + 9), 32, 38, 47 arranged in
ascending order is 24. Find the value of x and hence find the mean.
Sol. Observation are : 11, 12, 14, (x – 2), (x + 4), (x + 9), 32, 38, 47
n=9
th
 9 1
 Median =   term
 2 
i.e. 5th term = x + 4
606 Arun Deep's Understanding Math-10
 Median = x + 4
24 = x + 4  x = 24 – 4
 x = 20
Now Observation are : 11, 12, 14, (20 – 2), (20 + 4), (20 + 9), 32, 38, 47
i.e. 11, 12, 14, 18, 24, 29, 32, 38, 47
11  12  14  18  24  29  32  38  47 225
 Mean =   25
9 9
5. The mean of the numbers 1, 7, 5, 3, 4, 4, is m. The numbers 3, 2, 4, 2, 3, 3, p have mean m – 1
and median q. Find (i) p (ii) q (iii) the mean of p and q.
Sol. (i) Mean of 1, 7, 5, 3, 4, 4 is m. Here n = 6
1  7  5  3  4  4 24
 m=  4
6 6
Mean of 3, 2, 4, 2, 3, 3, p is m – 1
32  4  2  33 p 17  p 17  p
m–1= 7  4–1=  =3
7 7
 17 + p = 21  p = 21 – 17 = 4
(ii) Now median of 3, 2, 4, 2, 3, 3, 4
Writing them in ascending order 2, 2, 3, 3, 3, 4, 4
Here, n = 7 which is odd
n1 7 1
 Median = th term = 2 = 4th term = 3
2
  q = 3
1 1 7
(iii) Mean of p and q = 2 (4  3)  2  7  2 = 3·5 Ans.
6. Find the median for the following distribution :
Wages per day (in rupees) 38 45 48 55 62 65
No. of workers 14 8 7 10 6 2
Sol. Writing the distribution in cumulative frequency table :
Wages per day No. of workers c.f.
(in Rs.) (f)

38 14 14
45 8 22
48 7 29
55 10 39
62 6 45
65 2 47

n  f  47
607 Arun Deep's Understanding Math-10
Here, n = 47 which is odd
n1 47  1
 Median = th term = 2 = 24th term = 48
2
( Here all the observations form 23 to 29 are equal to 48)
 Median = Rs. 48 Ans.
P.Q. Find the median for the following distribution.
Marks 35 45 50 64 70 72
No. of students 3 5 8 10 5 5
Sol. Writing the distribution in cumulative frequency table :
Marks No. of students ( f ) c.f.
35 3 3
45 5 8
50 8 16
64 10 26
70 5 31
72 5 36
f  n  36
Here, n = 36 which is even

 Median =
1 nLM
th term 
n FG IJ
 1 th term =
1 36 OP LM
th term 
36 FG
 1 th term
IJ OP
2 2 N 2 H K 2 2 Q N 2 H K Q
1 1 1
= 2 (18th term + 19th term) = 2 (64 + 64) = 2 × 128 = 64
( Here all the observations from 17 to 26 all are equal to 64)
 Median = 64 Ans.
7. Marks obtained by 70 students are given below :
Marks 20 70 50 60 75 90 40
No. of students 8 12 18 6 9 5 12
Calculate the median marks.
Sol. Arranging the variates in ascending order and in c.f. table.
Marks No. of students ( f ) c.f.
20 8 8
40 12 20
50 18 38
60 6 44
70 12 56
75 9 65
90 5 70
f  n  70
608 Arun Deep's Understanding Math-10
Here, n = 70 which is even

 Median =
1 nLM
th term 
n FG IJ
 1 th term =
1 70 OP
th term 
LM
70 FG
 1 th term
IJ OP
2 2 N 2 H K 2 2 Q N
2 H K Q
1 1 1
= 2 (35th term + 36th term) = 2 (50 + 50) = 2  100  50 Ans.
( Here all the observations from 21 to 38 all are equal to 50)
8. Calculate the mean and the median for the following distribution :
Number 5 10 15 20 25 30 35
Frequency 1 2 5 6 3 2 1

Sol. Writing the distribution in c.f. table :


Number Frequency c.f. fx
(x) (f)
5 1 1 5
10 2 3 20
15 5 8 75
20 6 14 120
25 3 17 75
30 2 19 60
35 1 20 35
Total f  20 f  390

(i) Mean =
 fx  390  19  5 (ii) Here n = 20, which is even
 f 20
 Median =
LM
1 n
th term 
nFG IJ
 1 th term =
OP
1 20
th term 
LM
20 FG
 1 th term
IJ OP
2 2N 2 H K 2 2 Q N
2 H K Q
1 1 1
= 2 (10th term + 11th term) = 2 (20 + 20) = 2  40  20 Ans.
( Here observations from 9 to 14 all are equal to 20)
9. The daily wages (in rupees) of 19 workers are
41, 21, 38, 27, 31, 45, 23, 26, 29, 30, 28, 25, 35, 42, 47, 53, 29, 31, 35.
Find (i) the median (ii) lower quartile (iii) upper quartile range.
Sol. Arranging the observations in ascending order
21, 23, 25, 26, 27, 28, 29, 29, 30, 31, 31, 35, 35, 38, 41, 42, 45, 47, 53
Here n = 19 which is odd.
n1 19  1 20
(i)  Median = th term = 2  2 = 10th term = 31
2
609 Arun Deep's Understanding Math-10
n  1 19  1 20
(ii) Lower quartile (Q1) =   = 5th term = 27
4 4 4

(iii) Upper quartile (Q3) = 3


FG n  1IJ  3 FG 19  1IJ  3  5 = 15th term = 41
H 4 K H 4 K
(iv) Interquartile range = Q3 – Q1 = 41 – 27 = 14
10. From the following frequency distribution, find :
(i) the median (ii) lower quartile (iii) upper quartile (iv) inter quartile range
Variate 15 18 20 22 25 27 30

Frequency 4 6 8 9 7 8 6
Sol. Writing frequency distribution in c.f. table :

Variates Frequency (f) c.f.

15 4 4
18 6 10
20 8 18
22 9 27
25 7 34
27 8 42
30 6 48
 f  n = 48
Here, n = 48 which is even

(i) Median =
LM
1 n
th term 
n FG
 1 th term =
IJ
1 48 OP
th term 
LM
48
 1 th term
FG IJ OP
2 2N 2 H K
2 2 Q 2N H K Q
1 1 1
= 2 (24th term + 25th term) = 2 (22 + 22) =  44  22
2
( Observations from 19 to 27 are all equal to 22)
n 48
(ii) Lower quartile (Q1) = th term = 4 = 12th term = 20
4
3n 3  48
(iii) Upper quartile (Q3) = th term = 4 = 36th term = 27
4
(iv) Inter quartile range = upper quartile – lower quartile = 27 – 20 = 7
11. For the following frequency distribution, find :
(i) the median (ii) lower quartile (iii) upper quartile
Variate 25 31 34 40 45 48 50 60
Frequency 3 8 10 15 10 9 6 2
610 Arun Deep's Understanding Math-10
Sol. Writing the distribution in cumulative frequency (c.f.) table :

Variate Frequency (f) c.f.


25 3 3
31 8 11
34 10 21
40 15 36
45 10 46
48 9 55
50 6 61
60 2 63
 f  n  63

Here, n = 63 which is odd.

63  1 64
(i)  Median = 2 th or 2 = 32th term = 40

Observations from 22 to 36 all are equal to 40.

n 1 63  1
(ii) Lower quartile (Q1)  term  = 16th term = 34
4 4

3n  1 3×(63  1) 192
(iii) Upper quartile (Q3)   th  th = 48th term = 48 Ans.
4 4 4

EXERCISE 21.3
1. Find the mode of the following sets of numbers ;
(i) 5, 7, 6, 8, 9, 0, 6, 8, 1, 8 (ii) 9, 0, 2, 8, 5, 3, 5, 4, 1, 5, 2, 7
(iii) 3, 2, 0, 1, 2, 3, 5, 3 (1990)
Sol.
(i) The number 8 occurs maximum times
 Mode = 8
(ii) The number 5 occurs maximum times
 Mode = 5
(iii) The number 3, occurs maximum times
 Mode = 3
611 Arun Deep's Understanding Math-10
2. Find the mean, median and mode of the following distribution : (2009)
8, 10, 7, 6, 10, 11, 6, 13, 10

8  10  7  6  10  11  6  13  10 81
Sol.  Mean = = =9
9 9
Given nos. in ascending order are as follows:
6, 6, 7, 8, 10, 10, 10, 11, 13

n 1 9 1
Median = th term = = 5th term = 10
2 2
Mode = 10 (having highest frequency 3 times)
3. Calculate the mean, the median and the mode of the following numbers :
3, 1, 5, 6, 3, 4, 5, 3, 7, 2
Sol. Arranging in ascending order 1, 2, 3, 3, 3, 4, 5, 5, 6, 7

(i) Mean =
 xi = 1  2  3  3  3  4  5  5  6  7 39
= 10  3  9
n 10
(ii) Here n = 10 which is even
1 10 LM 10 FG IJ1 OP 1 7
 Median = 2 2 th term + 2  1 th term = 2 (5th + 6th) terms = 2 (3 + 4) = 2 = 3·5
N H K Q
(iii) Here 3 occurs maximum times
 Mode = 3
4. The marks of 10 students of a class in an examination arranged in ascending order is as
follows: 13, 35, 43, 46, x, x + 4, 55, 61, 71, 80
If the median marks is 48, find the value of x. Hence, find the mode of the given data. (2017)
Sol. Given marks are 13, 35, 43, 46, x, x + 4, 55, 61, 71, 80
 n = 10 (even), median = 48

1  n  
th th
  observation +   1 observation 
n
Median =
2  2  2  

5th observation + 6th observation xx4


= =
2 2

2x  4
 48 =
2

2( x  2)
 48 = x = 48 – 2 = 46
2
Now, put the value of x in 6th observation i.e. x + 4
612 Arun Deep's Understanding Math-10
 x + 4 = 46 + 4 = 50
 The numbers are : 13, 35, 43, 46, 46, 50, 55, 61, 71, 80
Since 46 has highest frequency
 Mode = 46
5. Find the mode and median of the following frequency distribution :
x 10 11 12 13 14 15
f 1 4 7 5 9 3
Sol. x f Cumulative frequency
10 1 1
11 4 5
12 7 12
13 5 17
14 9 26
15 3 29
 f  n  29
Here n = 29 (odd)

 29  1 
Median = Value of   th observation = Value of 15th observation = 13
 2 
Since, the frequency corresponding to 14 is maximum, so 14 is the mode.
6. In a class of 40 students, marks obtained by the students in a class test (out of 10) are given
below :

Marks 1 2 3 4 5 6 7 8 9 10

Number of students 1 2 3 3 6 10 5 4 3 3

Calculate the following for the given distribution : (i) median (ii) mode
Sol. Marks No. of students
(x) (f) c.f.
1 1 1
2 2 3
3 3 6
4 3 9
5 6 15
6 10 25
613 Arun Deep's Understanding Math-10

7 5 30
8 4 34
9 3 37
10 3 40
40
n = 40 which is even.

 40 
(i) Median =   th term = 20th term = 6
2

(ii) Mode = 6 (with highest frequency) = 6


7. The marks obtained by 30 students in a class assessment of 5 marks is given below:

Marks 0 1 2 3 4 5

No. of students 1 3 6 10 5 5

Calculate the mean, median and mode of the above distribution.

Sol.

Marks 0 1 2 3 4 5

No. of students 1 3 6 10 5 5

Cumulative Frequency 1 4 10 20 25 30

 fx 0  1  1  3  2  6  3  10  4  5  5  5
Mean =  f =
1  3  6  10  5  5

90
= =3
30
 3 is the mean
There are a total of 30 observations in the data
th th
 n n 
The median is the arithmetic mean of   and   1 observation in case of even number
 2 2 
of observations
th th
 30   30 
= Arithmetic mean of   and   1
 2   2 
= Arithmetic mean of 15th and 16th observation will be the median
614 Arun Deep's Understanding Math-10
33
Median = =3
3
Frequency is highest for the observation xi = 3
Mode = 3
8. The distribution given below shows the marks obtained by 25 students in an aptitude test. Find the
mean, median and mode of the distribution. (4)
Marks obtained 5 6 7 8 9 10
No. of students 3 9 6 4 2 1

Sol-

Marks obtained (xi) No. of students (fi) fixi


5 3 15
6 9 54
7 6 42
8 4 32
9 2 18
10 1 10
Total fi = 25 fixi = 171
 f i xi 171
Mean x =
 f i = 25
x = 6.84
Marks obtained Frequency Cumulative frequency
5 3 3
6 9 12
7 6 18
8 4 22
9 2 24
10 1 25
Total Total = 25
Number of terms = 25 (odd)
 25  1   26 
Median =   th term =   th term = 13th term
 2   2 
 Median = 7
Mode = Marks with maximum frequency is 6
 Mode = 6
P.Q. At a shooting competition, the scores of a competitor were as given below :
Score 0 1 2 3 4 5
No. of shots 0 3 6 4 7 5
(i) What was his modal score ?
(ii) What was his median score ?
(iii) What was his total score ?
615 Arun Deep's Understanding Math-10
(iv) What was his mean score ?
Sol. Writing the given distribution in cumulative frequency distribution :
Score No. of shots c.f. f.x.
(x) (f)
0 0 0 0
1 3 3 3
2 6 9 12
3 4 13 12
4 7 20 28
5 5 25 25
Total  f = 25  fx = 80
(i) Model score is 4 as it occurs in maximum times i.e. 7
 Mode = 4
(ii) Here n = 25 which is an odd number
25  1
 Median = = 13th term
2
Hence Median = 3
(iii) Total scores = 80

(iv) Mean =
 f x  80  3  2 Ans.
 f 25
P.Q. (i) Using step-deviation method, calculate the mean marks of the following distribution.
(ii) State the modal class.
Class Interval 50-55 55-60 60-65 65-70 70-75 75-80 80-85 85-90
Frequency 5 20 10 10 9 6 12 8
Sol-

C.I. xi fi Assumed mean fiui


A = 67.5
xi  A
ui = ,h=5
h
50-55 52.5 5 –3 –15
55-60 57.5 20 –2 –40
60-65 62.5 10 –1 –10
65-70 67.5 10 0 0
70-75 72.5 9 1 9
75-80 77.5 6 2 12
80-85 82.5 12 3 36
85-90 87.5 8 4 32

Total  f = 80  fi xi = 24
616 Arun Deep's Understanding Math-10

 fi ui 24
(i) Mean = A +  f  h = 67.5 + × 5 = 67.5 + 1.5 = 69
i 80
(ii) Modal class = 55 – 60
9. The following table gives the weekly wages (in Rs.) of workers in a factory :
Weekly wages 50–55 55–60 60–65 65–70 70–75 75–80 80–85 85–90
(in Rs.)
No. of workers 5 20 10 10 9 6 12 8
Calculate :
(i) the mean. (ii) the modal class
(iii) the number of workers getting weekly wages below Rs. 80.
(iv) the number of workers getting Rs. 65 or more but less than Rs. 85 as weekly wages.
(2002)
Sol. Representing the given distribution in cumulative frequency distribution

Weekly wages No. of workers Class marks c.f. f.x.


(f) (x)

50–55 5 52·5 5 262·5


55–60 20 57·5 25 1150·0
60–65 10 62·5 35 625·0
65–70 10 67·5 45 675·0
70–75 9 72·5 54 652·5
75–80 6 77·5 60 465·0
80–85 12 82·5 72 990·0
85–90 8 87·5 80 700·0

Total  f  80  f x 5520.0

(i) Mean =
 f x  5520  69
 f 80
(ii) Modal class :
frequency of class 55–60 is maximum i.e. 20
 Class 55–60 is the modal class.
(iii) No. of workers getting weekly wages below Rs. 80 = 60
(iv) No. of workers getting Rs. 65 or more but less than 85 as weekly wages = 72 – 35 = 37 Ans.
617 Arun Deep's Understanding Math-10
EXERCISE 21.4
1. Draw a histogram from the following frequency distribution and find the mode from the graph:
Class 0-5 5-10 10-15 15-20 20-25 25-30
Frequency 2 5 18 14 8 5
Sol. Now present the Height on x-axis and No. of students (frequency) on y-axis and draw a histogram
as shown. In the histogram join AB and CD intersecting at M. From M, draw MN to x-axis. N
shows the mode.
Y
Hence mode = 174 cm Ans. 18

16
Class 0-5 5-10 10-15 15-20 20-25 25-30
14
Frequency 2 5 18 14 8 5
12

Frequency
Mode = 14 10

X
O 5 10 15 20 25 30
Class

P.Q. Find the modal height of the following distribution by drawing a histogram :
Height (in cm) 140–150 150–160 160–170 170–180 180–190
No. of students 7 6 4 10 2
Sol.
Height (in cm) No. of students

140–150 7
150–160 6
160–170 4
170–180 10
180–190 2
618 Arun Deep's Understanding Math-10
Now present the Height on x-axis and No. of
students (frequency) on y-axis and draw a
histogram as shown. In the histogram join
AB and CD intersecting at M. From M, draw
MN to x-axis. N shows the mode.
Hence mode = 174 cm Ans.

2. A Mathematics aptitude test of 50 students was recorded as follows :

Marks 50-60 60-70 70-80 80-90 90-100


No. of Students 4 8 14 19 5

Draw a histogram for the above data using a graph paper and locate the mode. (2011)
Sol.

20

Now present the Height on x-axis and No. of


15
students (frequency) on y-axis and draw a
histogram as shown. In the histogram join AB
No. of Students

10 and CD intersecting at M. From M, draw MN


to x-axis. N shows the mode.
5 Hence mode = 174 cm Ans.

82.5
X
O 50 60 70 80 90 100

Marks

Hence, the required mode is 82.5.


3. Draw a histogram and estimate the mode for the following frequency distribution :

Classes 0–10 10–20 20–30 30–40 40–50 50–60


Frequency 2 8 10 5 4 3 (2003)
619 Arun Deep's Understanding Math-10
Sol. Classes Frequency

0–10 2
10–20 8
20–30 10
30–40 5
40–50 4
50–60 3
Representing classes on x-axis and frequency
on y-axis, we draw a histogram as shown.
In the histogram, join AB and CD intersecting
at M. From M, draw ML  x-axis. L shows
the mode.
Hence mode = 23
4. Using a graph paper, draw a histogram for the given distribution showing the number of runs
scored by 50 batsmen. Estimate the mode of the data :
Runs scored No. of batsmen
3000-4000 4
4000-5000 18
5000-6000 9
6000-7000 6
Y
7000-8000 7
18 Scale : x-axis : 2 cm = 1000 runs
8000-9000 2 y-axis : 2 cm = 2 batsmen

9000-10000 4 16

Sol. Mode = 4600 runs 14

Mode = 4600
12
Number of batsmen

10

X
O 3000 4000 5000 6000 7000 8000 9000 10000

Runs scored
620 Arun Deep's Understanding Math-10
5. Use a graph paper for this question. The daily pocket expenses of 200 students in a school are
given below:
Pocket expenses Number of students
(in ` ) (frequency)
0-5 10
5-10 14
10-15 28
15-20 42
20-25 50
25-30 30
30-35 14
35-40 12
Y
Draw a histogram representing the above 50
A B
distribution and estimate the mode from the graph.
Sol. Mark the upper corners of the highest rectangle P
45
and the corners of the adjacent rectangles as D
A, B, C, D as shown. Join AC and BD to intersect
40
at P. Draw PM x-axis. Then abscissa of M is 21,
which is the required mode.
Hence, mode = 21 Ans. 35

C
30
No. of students

25

20

15

10

X
O 5 10 15 20M 25 30 35 40 45
Pocket expenses in `
6. Draw a histogram for the following distribution :
Wt. in kg 40–44 45–49 50–54 55–59 60–64 65–69
No. of students 2 8 12 10 6 4
Hence estimate the modal weight.
621 Arun Deep's Understanding Math-10
Sol. We write the given distribution in continous form :
Wt. in kg. No. of students
39·5–44·5 2
44·5–49·5 8
49·5–54·5 12
54·5–59·5 10
59·5–64·5 6
64·5–69·5 4
Representing the weight (in kg) on x-axis and
No. of students on y-axis. We draw a
histogram as shown. Now join AB and CD
intersecting each other at M. From M, draw
ML perpendicular to x-axis. L is the mode
which is 51·5 kg.
7. Find the mode of the following distribution by drawing a histogram
Mid value 12 18 24 30 36 42 48
Frequency 20 12 8 24 16 8 12
Also state the modal class.
Sol. Mid value Class Frequency
12 9–15 20
18 15–21 12
24 21–27 8
30 27–33 24
36 33–39 16
42 39–45 8
48 45–51 12
Representing class on x-axis and frequency on
y-axis, we draw a histogram as shown. Join AB
and CD intersecting each other at M. From M,
draw ML perpendicular to x-axis. L shows the
mode which is 30·5 and class is 27–33. Ans.

EXERCISE 21.5
1. Draw an ogive for the following frequency distribution:
Height (in cm) 150-160 160-170 170-180 180-190 190-200
No. of students 8 3 4 10 2
622 Arun Deep's Understanding Math-10
Sol. Height (in cm) No. of students c.f.
(f)
150-160 8 8
160-170 3 11
170-180 4 15
180-190 10 25
190-200 2 27
Plot the points (150, 0), (160, 8), (170, 11), (180, 15), (190, 25) and (200, 27) on the graph and
join them with free hand. We get an ogive as shown:

30
(200, 27)

25 (190, 25)
Height (in cm)

20

15 (180, 15)

(170, 11)
10
(160, 8)

X
O 150 160 170 180 190 200
Number of students

2. Draw an ogive for the following data:


Class intervals 1-10 11-20 21-30 31-40 41-50 51-60
Frequency 3 5 8 7 6 2

Sol. Class intervals Frequency c.f.


0·5-10·5 3 3
10·5-20·5 5 8
20·5-30·5 8 16
30·5-40·5 7 23
40·5-50·5 6 29
50·5-60·5 2 31
623 Arun Deep's Understanding Math-10
Plot the points (0.5, 0), (10·5, 3), (20·5, 8), (30·5, 16), (40·5, 23), (50·5, 29), (60·5, 31) on the
graph and join then with free hand, we get an ogive as shown:

Y
32 (60.5, 31)
(50.5, 29)
28

24 (40.5, 23)

20
Frequency

(30.5, 16)
16

12

8 (20.5, 8)

X
O 20.5 30.5 40.5 50.5 60.5
Class intervals

3. Draw a cumulative frequency curve for the following data:

Marks obtained 24-29 29-34 34-39 39-44 44-49 49-54 54-59


No. of students 1 2 5 6 4 3 2

Sol. Marks obtained No. of students c.f.


(f)

24-29 1 1
29-34 2 3
34-39 5 8
39-44 6 14
44-49 4 18
49-54 3 21
54-59 2 23

Plot the points (24, 0), (29, 1), (34, 3), (39, 8), (44, 14), (49, 18), (54, 21) and (59, 23) on the
graph and join them with free hand to get an ogive as shown:
624 Arun Deep's Understanding Math-10

24 (49, 23)
(54, 21)
20
(49, 18)

No. of students 16
(44, 14)
12

8 (39, 8)

4 (34, 3)
(29, 1)
(24, 0)
X
O 24 29 34 39 44 49 54 59
Marks obtained

EXERCISE 21.6
1. Using the data given below construct the cumulative frequency table and draw the ogive. From
the ogive determine the median.

Marks 0–10 10–20 20–30 30–40 40–50 50–60 60–70 70–80


No.of students 3 8 12 14 10 6 5 2

Sol. Representing the given data in cumulative frequency distributions :


625 Arun Deep's Understanding Math-10

Marks No. of students (f) c.f.

0–10 3 3
10–20 8 11
20–30 12 23
30–40 14 37
40–50 10 47
50–60 6 53
60–70 5 58
70–80 2 60

Taking points (10, 3), (20, 11), (30, 23), (40, 37), (50, 47), (60, 53), (70, 58) and (80, 60) on the
graph. Now join them in free hand to form an ogive as shown.
Here n = 60 which is even

 Median =
LM
1 60 FG IJ
th term 
60
 1 th term
OP
N
2 2 H K
2 Q
1 L 60 F 60 I O
= M G  1J th termsP
2 N2 H2 K Q
1
= 2 (30th term + 31th term) = 30·5 observation
Now take a point A (30·5) on y-axis and from A, draw a line parallel to x-axis meeting the curve
at P and from P, draw a perpendicular to x-axis meeting is at Q.
 Q is the median which is 35.
P.Q. The following table shows the distribution of the heights of a group of a factory workers.

Height (cm) 150–155 155–160 160–165 165–170 170–175 175–180 180–185


No. of workers 6 12 18 20 13 8 6

(i) Determine the cumulative frequencies.


(ii) Draw the cumulative frequency curve on a graph paper.
Use 2 cm = 5 cm height on one axis and 2 cm = 10 workers on the other.
(iii) From your graph, write down the median height in cm. (2000)
Sol. Representing the distribution in cumulative
frequency distribution :
626 Arun Deep's Understanding Math-10

Height (cm) No. of workers ( f ) c.f.

150–155 6 6
155–160 12 18
160–165 18 36
165–170 20 56
170–175 13 69
175–180 8 77
180–185 6 83
 f = 83

Here, n = 83 which is even.


Now taking points (155, 6), (160, 18),
(165, 36), (170, 56), (175, 69), (180, 77)
and (185, 83) on the graph.
Now join them with free hand to form the
ogive or cumulative frequency curve as shown.
Here n = 83 which is odd
n1
 Median = th observation
2
83  1
= 2 = 42th observation
Take a point A (42) on y-axis and from A,
draw a horizontal line parallel to x-axis meeting
the curve at P. From P draw a line perpendicular
on x-axis which meets it at Q.
 Q is the median which is 166·5 cm. Ans.
P.Q. Use graph paper for this question.
The following table shows the weights in gm of a sample of 100 potatoes taken from a large
consignment :

Weight (gm) 50–60 60–70 70–80 80–90 90–100 100–110 110–120 120–130

Frequency 8 10 12 16 18 14 12 10

(i) Calculate the cumulative frequencies.


(ii) Draw the cumulative frequency curve and from it determine the median weight of the potatoes.
(1996)
627 Arun Deep's Understanding Math-10
Sol. Representing the given data in cumulative frequency table :
Weight (gm) Frequency c.f.
50–60 8 8
60–70 10 18
70–80 12 30
80–90 16 46
90–100 18 64
100–110 14 78
110–120 12 90
120–130 10 100

Now plot the points (50, 0), (60, 8), (70, 18), (80, 30), (90, 46), (100, 64), (110, 78), (120, 90),
(130, 100) on the graph and join them in free hand to form an ogive as shown
Here n =100 which is even.

Median =
LM
1 n
th term 
n FG
 1 th term
IJ OP
H K 100 (130, 100)
2 2N 2 Q
90 (120, 90)

=
LM
1 100
th term 
100 FG
 1 th term
IJ OP 80 (110, 78)
2 2 N 2 H K Q 70
c.f.

(100, 64)
60
1 50.5
= 2 (50 + 51) = 50·5 A
50 P
(90, 46)
Now take a point A (50·5) on the y-axis 40
and from A draw a line parallel to x-axis 30 (80, 30)
meeting the curve at P. From P, draw (70, 18)
a perpendicular on x-axis meeting it at Q. 20
(60, 8)
10
Q is the median which is = 93 gm. (50, 0) Q
O 50 60 70 80 90 100 110 120 130 X
WEIGHTS (in gm)
P.Q. Attempt this question on graph paper.
Age (yrs.) 5–15 15–25 25–35 35–45 45–55 55–65 65–75
No. of casualties 6 10 15 13 24 8 7
due to accidents

(i) Construct the `less than' cumulative frequency curve for the above data, using 2 cm = 10 years,
on one axis and 2 cm = 10 casualties on the other.
(ii) From your graph determine (1) the median and (2) the upper quartile. (1995)
628 Arun Deep's Understanding Math-10
Sol. Representing the given data in less than cumulative frequency.
Age No. of Casualties Cumulative Frequency
less than 15 6 6
less than 25 10 16
less than 35 15 31
less than 45 13 44
less than 55 24 68
less than 65 8 76
less than 75 7 83
Now plot the points (5, 0), (15, 6), (25, 16), (35, 31), (45, 44), (55, 68), (65, 76) and (75, 83) on
the graph and join these points in free hand to form a cumulative frequency curve (ogive) as shown.
Here n = 83, which is odd. Y
100
n  1 83  1 84
(i) Median =    42 90
2 2 2 (75, 83)
80 (65, 76)
Now we take point A (42) on y-axis and from
70 (55, 68)
A, draw a line parallel to x-axis meeting the H
curve at P and from P, draw a perpendicular 60 L
c.f.

to x-axis meeting it at Q. 50
42 A (45, 44)
Q is the median which is = 43 40 P
(35, 31)
3( n  1) 3×( 83  1) 252 30
(ii) Upper quartile    = 63
4 4 4 20
(25, 16)
Take a point B 63 on y-axis and from B, draw 10
(5, 0) (15, 6) Q M
a parallel line to x-axis meeting the curve at
O 5 15 25 35 45 55 65 75 X
L. From L, draw a perpendicular to x-axis 43 52
meeting it at M which is 52. AGE IN YEARS

 Upper quartile = 52 years


2. The weight of 50 workers is given below :

Weight in Kg 50-60 60-70 70-80 80-90 90-100 100-110 110-120


No. of Workers 4 7 11 14 6 5 3

Draw an ogive of the given distribution using a graph sheet. Take 2 cm = 10 kg on one axis
and 2 cm = 5 workers along the other axis. Use a graph to estimate the following:
(i) the upper and lower quartiles.
(ii) if weighing 95 kg and above is considered overweight find the number of workers who are
overweight.
629 Arun Deep's Understanding Math-10
Sol. The cumulative frequency table of the given distribution table is as follows :
Weight in km Number of workers Cumulative frequency
50–60 4 4
60–70 7 11
70–80 11 22
80–90 14 36
90–100 6 42
100–110 5 47
110–120 3 50

The ogive is as follows :

Plot the points (50, 0), (60, 4), (70, 11), (80, 22), (90, 36), (100, 42), (110, 47), (120, 50)
Join these points by using free hand drawing. The required ogive is drawn on the graph paper.
Here n = number of workers = 50
(i) To find upper quartile :
Let A be the point on y-axis representing frequency
630 Arun Deep's Understanding Math-10

3n 3  50 the ogive at E. Through E draw a vertical


= = 37.5 line to meet the x-axis at F. The abscissa of
4 4
the point F represent 72 kg.
Through A, draw a horizontal line to meet  The lower quartile = 72 kg
the ogive at B. Through B draw a vertical
(ii) On the graph point G represents 95 kg.
line to meet the x-axis at C. The abscissa of
Through G, draw a vertical line to meet the
the point C represents 92.5 kg.
ogive at H. Through H, draw a horizontal
 The upper quartile = 92.5 kg line to meet y-axis at 1. The ordinate of the
To find the lower quartile: point 1 represents 40 workers on y-axis
Let D be the point on y-axis representing  The number of workers who are 95 kg and
n 50 above
frequency = = = 12.5 = Total number of workers – number of
4 4
Through D, draw a horizontal line to meet workers of weight less than 95 kg
= 50 – 40 = 10

3. The table shows the distribution of scores obtained by 160 shooters in a shooting competition. Use
a graph sheet and draw an ogive for the distribution.
(Take 2 cm = 10 scores on the x-axis and 2 cm = 20 shooters on the y-axis)

Scores 0-10 10-20 20-30 30-40 40-50 50-60 60-70 70-80 80-90 90-100
No. of shooters 9 13 20 26 30 22 15 10 8 7

Use your graph to estimate the following:


(i) The median.
(ii) The inter quartile range.
(iii) The number of shooters who obtained a score of more than 85%. (2016)

Sol. Scores Number of shooters c.f.

0-10 9 9
10-20 13 22
20-30 20 42
30-40 26 68
40-50 30 98
50-60 22 120
60-70 15 135
70-80 10 145
80-90 8 153
90-100 7 160
N =  f = 160
Plot the points (0, 0), (10, 9), (20, 22), (30, 42), (40, 68), (50, 98), (60, 120), (70, 135), (80,
145), (90, 153), (100, 160) on the graph and join them with free hand to get an ogive as shown:
631 Arun Deep's Understanding Math-10

Y
165 (100, 160)

Q
150 (90, 153)
149 (80, 145) P

135 (70, 135)

120 (60, 120)

105
(50, 98)
No. of Students

90

80 A
75
(40, 68)

60

45 (30, 42)
40 C

30
(20, 22)
15
(10, 9)
B
X` X
10 20 30 31 40 44 50 60 70 8085%90 100

Y` Scores

Here n = 160
n 160
 = = 80
2 2
Median : Take a point 80 on y-axis and through it, draw a line parallel to x-axis which meets the
curve at A.
Through A, draw a perpendicular on x-axis which meet it at B.
B is median which is 44.
(ii) Inter quartile range (Q1)
n 160
= = 40
4 4
632
Arun Deep's Understanding Math-10
From a point 40 on y-axis, draw a line parallel Scores : From 85 on x-axis, draw a
to x-axis which meet the curve at C and from perpendicular to it meeting the curve at P.
C draw a line perpendicular to it which meet From P, draw a line parallel to x-axis meeting
it at D. which is 31. y-axis at Q.
 Inter quartile range is 31. Q is the required point which is 89.
(iii) Number of shooter who get move than 85%.  Number of shooter getting more than 85%
scores = 160 – 149 = 11.
4. The daily wages of 80 workers in a project are given below:
Wages (in `) 400-450 450-500 500-550 550-600 600-650 650-700 70-750
No. of workers 2 6 12 18 24 13 5
Use a graph paper to draw an ogive for the above distribution. (Use a scale of 2 cm = `50 on x-
axis and 2 cm = 10 workers on y-axis). Use your ogive to estimate:
(i) the median wage of the workers.
(ii) the lower quartile wage of the workers.
(iii) the number of workers who earn more than `625 daily. (2017)
Sol. Wages in (`) No. of workers Cumulative frequency
400-450 2 2
450-500 6 8
500-550 12 20
550-600 18 38
600-650 24 62
650-700 13 75
700-750 5 80
Y
 Number of workers = 80
80
th
n
(i) Median =   term = 40th term
2 70

Through mark 40 on the y-axis, 60


draw a horizontal line which
meets the curve at point A. 50 C
No. of workers

40 A

30

20

10

B
X
O 400 450 500 550 600 625 650 700 750
Daily wages
633 Arun Deep's Understanding Math-10
Through point A, on the curve draw a vertical line which meets the x-axis at point B.
The value of point B on the x-axis is the median, which is 604.
th
 80 
(ii) Lower quartile (Q1) =   term = 20th term = 550
 4 
(iii) Through mark 625 on x-axis, draw a vertical line which meets the graph at point C.
Then through point C, draw a horizontal line which meets the y-axis at the mark of 50.
Thus, number of workers that earn more than `625 daily = 80 – 50 = 30
5. Marks obtained by 200 students in an examination are given below :
Marks 0–10 10–20 20–30 30–40 40–50 50–60 60–70 70–80 80–90 90–100
No. of students 5 11 10 20 28 37 40 29 14 6
Draw an ogive for the given distribution taking 2 cm = 10 marks on one axis and 2 cm = 20
students on the other axis. Using the graph, determine
(i) The median marks.
(ii) The number of students who failed if minimum marks required to pass is 40.
(iii) If scoring 85 and more marks is considered as grade one, find the number of students who
secured grade one in the examination.
Sol.

Marks f c.f.
Y
0-10 5 5 (100, 200)
200 (90, 194)
10-20 11 16
(80, 180)
20-30 10 26 180

30-40 20 46
160
40-50 28 74 (70, 151)

50-60 37 111 140

60-70 40 151
No. of students

120
(60, 111)
70-80 29 180
100
80-90 14 194
90-100 6 200 80 (50, 74)

N = 200 60
(40, 46)
(i) Median is 57. 40
(ii) 44 students failed. (30, 26)
(20, 16)
20
(iii) No. of students who secured (10, 5)
grade one = 200 – 188 = 12. X
O 10 20 30 40 50 60 70 80 90 100
Marks
634 Arun Deep's Understanding Math-10
6. The monthly income of a group of 320 employees in a company is given below

Monthly Income No. of Employees

6000-7000 20
7000-8000 45
8000-9000 65
9000-10000 95
10000-11000 60
11000-12000 30
12000-13000 5
Draw an ogive of the given distribution on a graph sheet taking 2 cm = Rs. 1000 on one axis and 2 cm =
50 employees on the other axis. From the graph determine
(i) the median wage.
(ii) the number of employees whose income is below Rs. 8500.
(iii) If the salary of a senior employee is above Rs. 11500, find the number of senior employees in the
company.,
(iv) the upper quartile. (2010)
Sol. Monthly Income No. of Employees (f) c.f.

6000-7000 20 20
7000-8000 45 65
8000-9000 65 130
9000-10000 95 225
10000-11000 60 285
11000-12000 30 315
12000-13000 5 320

Now plot the points (7000, 20), (8000, 65), (9000, 130), (10000, 225), (11000, 285), (12000, 315) and (13000,
320) on the graph and join them in order with free hand to get an ogive as shown in the figure
(i) Total number of employees = 320 Y

N 320 400
 = = 160
2 2 350
(13000, 320)
From 160 on y-axis, draw a line parallel 300 (11000, 285) (12000, 315)
to x-axis meeting the curve at P. From P, draw a
No. of Employees

250
perpendicular on x-axis meeting it at M A B
(10000, 225)
M is the median which is 9300 200 Q
P
150
(9000, 130)
(ii) From 8500 on x-axis, draw a perpendicular
100 Q
which meets the curve at Q. From Q, draw a line parallel P
(8000, 65)
to x-axis meeting y-axis at N. Which is 98 50
(7000, 20)
(iii) From 11500 on the x-axis, draw a line M B
X
O 6000 7000 8000 9000 1000011000 1200013000
perpendicular to x-axis meeting the curve at R. From
Monthly Wages
R, draw a line parallel to x-axis meeting y-axis at L. Which is 300.
635 Arun Deep's Understanding Math-10
 No. of employees getting more than Rs. 11500 = 320 – 300 = 20
3 N 320  3
(iv) Upper Quartile (Q1) = = = 240.
4 4
From 240 on y-axis, draw a line perpendicular on x-axis which meets the curve at S. From S, draw
a perpendicular on x-axis meeting it at T, which is 10250.
Hence, Q3 = 10250.
7. Use graph paper for this question.
A survey regarding height (in cm) of 60 boys belonging to Class 10 of a school was conducted.
The following data was recorded :

Height in cm No. of boys

135–140 4

140–145 8

145–150 20

150–155 14

155–160 7

160–165 6

165–170 1

Taking 2 cm = height of 10 cm along one axis and 2 cm = 10 boys along the other axis, draw an
ogive of the above distribution. Use the graph to estimate the following :
(i) the median (ii) lower quartile
(iii) if height above 158 cm is considered tall, find the number of boys in the class who are tall.
Sol.
636 Arun Deep's Understanding Math-10

Height in cm No. of boys C.F.

130–135 – –

135–140 4 4

140–145 8 12

145–150 20 32

150–155 14 46

155–160 7 53

160–165 6 59

165–170 1 60

th th
n  60 
(i) Median =   term =   = 30th term = 149 (approx.)
2  2 

th th
n  60 
(ii) Lower quartile =   =   = 15th term = 146 cm
4  4 

(iii) 60 – 50 = 10 boys (approx.)


8. The marks obtained by 100 students in a Mathematics test are given below :

Marks 0-10 10-20 20-30 30-40 40-50 50-60 60-70 70-80 80-90 90-100

No. of students 3 7 12 17 23 14 9 6 5 4

Draw an ogive for the given distribution on a graph sheet.


Use a scale of 2 cm = 10 units on both axis).
Use the ogive to estimate the:
(i) median.
(ii) lower quartile.
(iii) number of students who obtained more than 85% marks in the test.
(iv) number of students who did not pass in the test if the pass percentage was 35.
Sol. We represent the given data in cumulative frequency table as given below :
637 Arun Deep's Understanding Math-10

Marks No. of student (f) Cumulative Frequency

0-10 3 3
10-20 7 10
20-30 12 22
30-40 17 39
40-50 23 62
50-60 14 76
60-70 9 85
70-80 6 91
80-90 5 96
90-100 4 100
N = 100 Y
100
100
Median = = 50th term = 45 90
2
 Median = 45 80
(ii) Lower quartile : (Q1)
70
100
Number of students

N = 100  = 25th term = 32


4 60

 Q 1 = 32 50
(iii) No. of students with 85%
or less = 70 40
 More than 85% marks 35
30
= 100 – 70 = 30 25
Q
(iv) Number of students who did 20
not pass = 38
10
32
X` X
O 10 20 3840 60 70 80 90 100
30 50
Y` Marks
9. The marks obtained by 120 students in a Mathematics test are given below :
Marks 0–10 10–20 20–30 30–40 40–50 50–60 60–70 70–80 80–90 90–100
No. of students 5 9 16 22 26 18 11 6 4 3
Draw an ogive for the given distribution on a graph sheet. Use a suitable scale for ogive to
estimate the following : (i) the median
(ii) the number of students who obtained more than 75% marks in the test.
(iii) the number of students who did not pass in the test if the pass percentage was 40. (2002)
(iv) the lower quartile
Sol. We represent the given data in cumulative frequency table as given below :
638 Arun Deep's Understanding Math-10
Marks No. of students Cumulative Frequency
0–10 5 5
10–20 9 14
20–30 16 30
30–40 22 52
40–50 26 78
50–60 18 96
60–70 11 107
70–80 6 113
80–90 4 117
90–100 3 120
Now we plot the points (10, 5), (20, 14), (30, 30), (40, 52), (50, 78), (60, 96), (70, 107), (80,
113), (90, 117) and (100, 120) on the graph and join the points in free hand to form an ogive as
shown.
Here n = 120 which is an even number

(i)  Median =
LM
1 120

FG
120
1
IJ OP
2 2N 2 H KQ
1
= 2 (60 + 61) = 60·5
Now take a point A (60·5) on y-axis
and from A draw a line parallel to x-
axis meeting the curve in P and from
P, draw a perpendicular to x-axis
meeting it at Q.
 Q is the median which is 43·00 (approx.)
(ii) Take a point C (75) on x-axis and from C draw a line perpendicular to it meeting the curve at R.
From R, draw a line parallel to x-axis meeting y-axis at S.
 S shows 110 students getting below 75% and 120 – 110 = 10 students getting more than 75%
marks.
(iii) Pass percentage is 40%
Now take a point D (40) on x-axis and from D draw a line perpendicular to x-axis meeting the
curve at E and from E, draw a line parallel to x-axis meeting the y-axis at F.
 F shows 52
 No of students who could not get 40% and failed in the examination are 52. Ans.
n 120
(iv) Lower quartile =   30
4 4
Now take a point B (30) on y-axis and from B, draw a line parallel to x-axis meeting the curve in
L and from L draw a perpendicular to x-axis meeting it at M.
M is the lower quartile which is 30.
639 Arun Deep's Understanding Math-10
10. The following distribution represents the height of 160 students of a school.
Height (in cm) 140-145 145-150 150-155 155-160 160-165 165-170 170-175 175-180
No. of Students 12 20 30 38 24 16 12 8

Draw an ogive for the given distribution plot the point (140, 0), (145, 12), (150, 32), (155,
taking 2 cm = 5 cm of height on one axis 62), (160, 100), (165, 124), (170, 140),
and 2 cm = 20 students on the other axis. (175, 152) and (180, 160). Join these points
Using the graph, determine : by a free hand curve to get the ogive.
(i)The median height. (ii)The inter quartile range. N
(i) Here N = 160  = 80
(iii) The number of students whose height is 2
above 172 cm. On the graph paper take a point A on the y-axis
Sol. The cumulative frequency table may be representing 80. Through A draw horizontal
prepared as follows : line meeting the ogive at B. From B, draw BC
Height No. of Cumulative  x-axis, meeting the x-axis at C.
(in cm) Students frequency The abscissa of C is 157.5
140 - 145 12 12 So, median = 157.5 cm
145-150 20 32 (ii) Proceeding in the same way as we have done
150-155 30 62 in above, we have, Q1 = 152 and Q3 = 164
155-160 38 100 So, inter quartile range = Q3 – Q1 = 164 –
160-165 24 124 152 = 12 cm
165-170 16 140 (iii) From the ogive, we see that the number of
170-175 12 152 students whose height is less than 172 is 145.
175-180 8 160 No. of students whose height is above 172
Now, we take height along x-axis and cm = 160 – 145 = 15
number of students along the y-axis. Now,
640 Arun Deep's Understanding Math-10
11. 100 pupils in a school have heights as tabulated below :
Height in cm 121–130 131–140 141–150 151–160 161–170 171–180

No. of pupils 12 16 30 20 14 8

Draw the ogive for the above data and from it determine the median (use graph paper).
Sol. Representing the given data in cumulative frequency table (in continous distribution) :

Height in cm No. of pupils c.f.


120·5–130·5 12 12
130·5–140·5 16 28
140·5–150·5 30 58
150·5–160·5 20 78
160·5–170·5 14 92
170·5–180·5 8 100

 Here, n = 100 which is an even number

1 n n LM FG
 Median = 2 2  2  1
IJ OP 
LM
1 100

FG
100
1
IJ OP = 1 101
(50 + 51) = 2  50  5
N H KQ N
2 2 2H KQ 2
Now plot points (130·5, 12), (140·5, 28), (150·5, 58), (160·5, 78), (170·5, 92) and (180·5, 100)
on the graph and join them in free hand to form an ogive as shown.
Now take a point A (50·5) on y-axis and from A draw a line parallel to x-axis meeting the curve at
P and from P, draw a line perpendicular to x-axis meeting it at Q.
 Q (147·5) is the median.
641 Arun Deep's Understanding Math-10
MULTIPLE CHOICE QUESTIONS
Choose the correct answer from the given four options (1 to 16):
1. If the classes of a frequency distribution are 1-10, 11-20, 21-30, ..., 51-60, then the size of each
class is
(a) 9 (b) 10 (c) 11 (d) 5·5
Sol. In the classes 1-10, 11-20, 21-30, ..., 51-60, the size of each class is 10. (b)
2. If the classes of a frequency distribution are 1-10, 11-20, 21-30, ..., 61-70, then the upper limit of
the class 11-20 is
(a) 20 (b) 21 (c) 19·5 (d) 20·5
Sol. In the classes of distribution, 1-10, 11-20, 21-30, ..., 61-70, upper limit of 11-20 is 20·5 as the
classes after adjustment are 0·5-10·5, 10·5-20·5, 20·5-30·5, ... (d)
3. In a grouped frequency distribution, the mid-values of the classes are used to measure which of
the following central tendency?
(a) median (b) mode (c) mean (d) all of these
Sol. In a grouped frequency distribution, the mid-values of the classes are used to measure Mean (c)

 fidi
4. In the formula: x = a +  f for finding the mean of the grouped data, d is are deviations from
i

a (assumed mean) of
(a) lower limits of the classes (b) upper limits of the classes
(c) mid-points of the classes (d) frequencies of the classes

 fidi
Sol. The formula x = a +  f is the finding of mean of the grouped data, d is are mid-points of the
i

classes. (c)
P.Q. If the class marks of a continuous frequency distribution are 22, 30, 38, 46, 54, 62, then the class
corresponding to the class mark 46 is
(a) 41·5-49·5 (b) 42-50 (c) 41-49 (d) 41-50
Sol. The class marks of distribution are 22, 30, 38, 46, 54, 62, then classes corresponding to these
class marks 46 is
46 – 4 = 42, 46 + 4 = 50
(Class intervals is 8 as 30 – 22 = 8, 38 – 30 = 8)
i.e., 42-50 (b)
642 Arun Deep's Understanding Math-10

  f i ui 
P.Q. In the formula: x = a + c   , for finding the mean of grouped frequency distribution, u =

  fi  i

yi  a yi  a a  yi
(a) (b) c(yi – a) (c) (d)
c c c
  f i ui 
Sol. In x = a + c   , for finding the mean of grouped frequency, 'u' is yi  a .
 f  (c)
 i  c
P.Q. While computing mean of grouped data, we assumed that the frequencies are
(a) evenly distributed over all the classes (b) centred at the class marks of the classes
(c) centred at the upper limits of the classes (d) centred at the lower limits of the classes
Sol. For computing mean of grouped data, we assumed that frequencies are centred at class marks of
the classes. (b)
5. Construction of a cumulative frequency distribution table is useful in determining the
(a) mean (b) median (c) mode (d) all the three measures
Sol. Construction of a cumulative frequency distribution table is used for determining the median. (b)
P.Q. The times, in seconds, taken by 150 athletes to run a 110 m hurdle race are tabulated below:
Class 13·8-14 14-14·2 14·2-14·4 14·4-14·6 14·6-14·8 14·8-15
Frequency 2 4 5 71 48 20
The number of athletes who completed the race in less than 14·6 seconds is
(a) 11 (b) 71 (c) 82 (d) 130
Sol. Time taken in seconds by 150 athletes to run a 110 m hurdle race as given in the sum,
the number of athletes who completed the race in less then 14·6 second is 2 + 4 + 5 + 71 = 82
athletes. (c)
6. Consider the following frequency distribution:
Class 0-5 6-11 12-17 18-23 24-29
Frequency 13 10 15 8 11
The upper limit of the median class is
(a) 17 (b) 17·5 (c) 18 (d) 18·5
Sol. From the given frequency upper limit of median class is 17·5
as total frequencies 13 + 10 + 15 + 8 + 11 = 57
57  1 58
 = = 29
2 2
and 13 + 10 + 15 = 28 where class is 12-17
But actual class will be 11·5-17·5
 Upper limit is 17·5 (b)
P.Q. Daily wages of a factory workers are recorded as:
Daily wages (in `) 131-136 137-142 143-148 149-154 155-160
No. of workers 5 27 20 18 12
643 Arun Deep's Understanding Math-10
The lower limit of the modal class is
(a) `137 (b) `143 (c) `136·5 (d) `142·5
Sol. In the daily wages of workers of a factory are 131-136, 137-142, 142-148, ...
which are not proper class
So, proper class will be 130·5-136·5, 136·5-142·5, 142·5-148·5, ...
 Lower limit of model class is 136·5 as 136·5-142·5 is the modal class. (c)
7. For the following distribution:
Class 0-5 5-10 10-15 15-20 20-25
Frequency 10 15 12 20 9
The sum of lower limits of the median class and modal class is
(a) 15 (b) 25 (c) 30 (d) 35
Sol. From the given distribution
Sum of frequencies = 10 + 15 + 12 + 20 + 9 = 66
66
and median is = 33
2
 Median class will be 10-15 and modal class is 15-20
 Sum of lower limits = 10 + 15 = 25 (b)
P.Q. Consider the following data:
Class 65-85 85-105 105-125 125-145 145-165 165-185 185-205
Frequency 4 5 13 20 14 7 4
The difference of the upper limit of the median class and the lower limit of the modal class is
(a) 0 (b) 19 (c) 20 (d) 38
Sol. From the given data
Total frequencies = 4 + 5 + 13 + 20 + 14 + 7 + 4 = 67
67  1
 Median class = 34 which is (4 + 5 + 13 + 20) 125-145 and modal class is 125-145
2
 Difference of upper limit of median class and the lower limit of the modal class
= 145 – 125 = 20 (c)
8. An ogive curve is used to determine
(a) range (b) mean (c) mode (d) median
Sol. An ogive curve is used to find median. (d)

CHAPTER TEST
1. The mean of 20 members is 18. If 3 is added to each of the first ten numbers, find the mean of
new set of 20 numbers.
Sol. Mean of 20 numbers = 18
 Total number = 18 × 20 = 360
644 Arun Deep's Understanding Math-10
By adding 3 to first 10 numbers,
The new sum will be = 360 + 3 × 10 = 360 + 30 = 390
390
 New Mean = 20  19  5 Ans.

2. The average height of 30 students is 150 cm. It was detected later that one value of 165 cm was
wrongly copied as 135 cm for computation of mean. Find the correct mean.
Sol. In first case,
Average height of 30 students = 150 cm
 Total height = 150 × 30 = 4500 cm
Difference in copying the number = 165 – 135 = 30 cm
 Correct sum = 4500 + 30 = 4530 cm
4530
 Correct mean =  151 cm Ans.
30
P.Q. Arun scored 36 marks in English, 44 marks in Civics, 75 marks in Mathematics and x marks in
Science. If he has scored an average of 50 marks, find x.
Sol. Marks in English = 36
Marks in Civics = 44
Marks in Mathematics = 75
Marks in Science = x
Total marks in 4 subjects = 36+ 44 + 75 + x = 155 + x
155  x
 average marks = 4
But average marks = 50 (given)
155  x
 4 = 50  155 + x = 200
 x = 200 – 155 = 45 Ans.
3. There are 50 students in a class of which 40 are boys and the rest girls. The average weight of
the students in the class in 44 kg and average weight of the girls is 40 kg. Find the average weight
of boys.
Sol. Total students of a class = 50
No. of boys = 40
 No. of girls = 50 – 40 = 10
Average weight of 50 students = 44 kg
 Total weight = 44 × 50 = 2200 kg
Average weight of 10 girls = 40 kg
 Total weight of girls = 40 × 10 = 400 kg
645 Arun Deep's Understanding Math-10
Then total weight of 40 boys = 2200 – 400 = 1800 kg
1800
 Average weight of boys =  45 kg
40
P.Q. The contents of 50 boxes of matches were counted giving the following results :
No. of matches 41 42 43 44 45 46
No. of boxes 5 8 13 12 7 5
Calculate the mean number of matches per box.
Sol. No. of matches No. of boxes f.x.
(x) (f)

41 5 205
42 8 336
43 13 559
44 12 528
45 7 315
46 5 230
Total  f = 25  fx  374

Mean 
 fx  2173  43. 46 Ans.
 f 50
4. The heights of 50 children were measured (correct to the nearest cm) giving the following
results :
Height (in cm) 65 66 67 68 69 70 71 72 73
No. of children 1 4 5 7 11 10 6 4 2
Calculate the mean height for this distribution correct to one place of decimal.

Sol. Height (in cm.) No. of children f (x)


(x) (f)

65 1 65
66 4 264
67 5 335
68 7 476
69 11 759
70 10 700
71 6 426
72 4 288
73 2 146
 f = 50  fx  3459
646 Arun Deep's Understanding Math-10

Mean =
 fx  3459  69  18  69  2 Ans.
 f 50
P.Q. Find the value of p for the following distribution whose mean is 20·6 :
Variate (xi) 10 15 20 25 35
Frequency ( fi ) 3 10 p 7 5

Sol. Variate (xi) Frequency ( fi ) fix i


10 3 30
15 10 150
20 p 20p
25 7 175
35 5 175
Total  f = 25 + p  f i xi  530 + 20p
f i xi 530  20 p
Mean = f  25  p
i
But Mean = 20·6 (given)
530  20 p
 25  p = 20·6  530 + 20p = 515 + 20·6p  20·6p – 20p = 530 – 515
6 15  10
 0·6p = 15  p  15  p=  25 Ans.
10 6
5. Find the value of p if the mean of the following distribution is 18.
Variate (x) 13 15 17 19 20 + p 23
Frequency ( f ) 8 2 3 4 5p 6

Sol. Variate (x) Frequency ( f ) fx


13 8 104
15 2 30
17 3 51
19 4 76
20 + p 5p 100p + 5p2
23 6 138
Total  f = 23 + 5p  fx  399 + 100p + 5p2

Mean =
 fx  399  100p  5p2
f 23  5p

But mean = 18 (given)


399  100 p  5p2 18
 23  5p = 1  399 + 100p + 5p2 = 414 + 90p
647 Arun Deep's Understanding Math-10
 5p2 + 100p + 399 – 90p – 414 = 0  5p2 +
10p – 15 = 0
 p2 + 2p – 3 = 0  p2 + 3p – p – 3 = 0
 p (p + 3) – 1 (p + 3) = 0  (p + 3) (p – 1) = 0
Either p + 3= 0, then p = – 3, but it is not possible as it is negative
or p – 1 = 0, then p = 1 Ans.
6. Find the mean age in years from the frequency distribution given below:
Age in years 25-29 30-34 35-39 40-44 45-49 50-54 55-59
No. of persons 4 14 22 16 6 5 3
Sol. Arranging the classes in proper form
Class age Mid value No. of persons fi × xi
(in years) (xi) (f i )
24.5-29.5 27 4 108
29.5-34.5 32 14 448
34.5-39.5 37 22 814
39.5-44.5 42 16 672
44.5-49.5 47 6 282
49.5-54.5 52 5 260
54.5-59.5 57 3 171
 fi = 70  fi xi = 2755

 f i xi 2755
 Mean =  f = = 39.357 = 39.36 years
i 70
P.Q. Calculate the Arithmetic mean, correct to one decimal place, for the following frequency distribution :
Marks 10–20 20–30 30–40 40–50 50–60 60–70 70–80 80–90 90–100
Students 02 04 05 16 20 10 06 08 04
Sol. Marks Students Class Mark fi xi
(f i ) (xi)
10–20 02 15 30
20–30 04 25 100
30–40 05 35 175
40–50 16 45 720
50–60 20 55 1100
60–70 10 65 650
70–80 06 75 450
80–90 08 85 680
90–100 04 95 380
 fi = 75  fi xi = 4285
648 Arun Deep's Understanding Math-10
 f i xi 4285
 Mean =  f  75  57  133  57  1 Ans.
i

7. The mean of the following frequency distribution is 62.8. Find the value of p.
Class 0-20 20-40 40-60 60-80 80-100 100-120
Frequency 5 8 p 12 7 8
Sol. Mean = 62.8
Class Frequency (fi) Class mark(x) fi × xi
0-20 5 10 50
20-40 8 30 240
40-60 p 50 50p
60-80 12 70 840
80-100 7 90 630
100-120 8 110 880
 fi = 40 + p  fi xi = 2640 + 50p

 f i xi 2640  50 p
Mean =  f  62.8 =
i 40  p  (40 + p) × 62.8 = 2640 + 50p
2512.0 + 62.8p = 2640 + 50p  62.8p – 50p = 2640 – 2512  12.8p = 128

128 128 10
 p= = = 10
12.8 128
Hence p = 10 Ans.
8. The daily expenditure of 100 families are given below. Calculate f1 and f2, if the mean daily
expenditure is `188.
Expenditure (in `) 140-160 160-180 180-200 200-220 220-240
No. of families 5 25 f1 f2 5
Sol. Mean = 188, No. of families = 100
Expenditure Mid value No. of persons fi × xi
(xi) (f i )
140-160 150 5 750
160-180 170 25 4250
180-200 190 f1 190f 1
200-220 210 f2 210f 2
220-240 230 5 1150
 fi = 35 + f1 + f2  fi xi =6150 + 190f1
= 100 + 210f2
649 Arun Deep's Understanding Math-10
35 + f1 + f2 = 100
 f1 + f2 = 100 – 35 = 65 ...(i)
 f1 = 65 – f2
6150  190 f1  210 f 2
and = 188
100
 190f1 + 210f2 = 18800 – 6150 = 12650
 190(65 – f2) + 200f2 = 12650
 12350 – 190f2 + 210f2 = 12650
 20f2 = 12650 – 12350
 20f2 = 300
300
 f2 = = 15
2
 f1 = 65 – 15 = 50
f1 = 50, f2 = 15
P.Q. The measures of the diameter of the heads of 150 screw is given in the following table. If the
mean diameter of the heads of the screws is 51.2 mm, find the values of p and q.
Diameter (in mm) 32-36 37-41 42-46 47-51 52-56 57-61 62-66
No. of screws 15 17 p 25 q 20 30
Sol. Mean = 51.2
No. of screws = 150
Diameter (in mm) No. of screws (fi) Class mark (xi) fi × xi
32-36 15 34 510
37-41 17 39 663
42-46 p 44 44p
47-51 25 49 1225
52-56 q 54 54q
57-61 20 59 1180
62-66 30 64 1920
 fi = 107+p+q  fi xi = 5498+44p+54q
107 + p + q = 150
p + q = 150 – 107 = 43 ....(i)
 f i xi 5498  44 p  54q
Mean =  = 51.2
 fi 150
 5498 + 44p + 54q = 7680
44p + 54q = 7680 – 5498  44p + 54q = 2182
 22p + 27q = 1091 ....(ii)
Multiplying (i) by 27 and (ii) and by 1
650 Arun Deep's Understanding Math-10
27p + 27q = 1161 ...(iii)
22p + 27q = 1091 ...(iv)
Subtracting, (iv) from (iii) we get
5p = 70

70
p= = 14
5
But p + q = 43
 q = 43 – p = 43 – 14 = 29
Hence p = 14, q = 29 Ans.
9. The median of the following numbers, arranged in ascending order is 25. Find x
11, 13, 15, 19, x + 2, x + 4, 30, 35, 39, 46
Sol. Here, n = 10, which is even

 Median =
LM
1 n
th 
n FG IJ OP
 1 th term =
1 10
th 
LM
10 FG
 1 th term
IJ OP
2 2N 2 H K Q 2 2 2N H K Q
1 1 2x  6
= 2 (5th + 6th) term = 2 (x + 2 + x + 4) = x3
2
But median is given = 25
 x + 3 = 25  x = 25 – 3 = 22 Ans.
10. If the median of 5, 9, 11, 3, 4, x, 8 is 6, find the value of x.
Sol. Arranging in ascending order, 3, 4, 5, x, 8, 9, 11,
Here n = 7 which is odd.
n1 7 1
 Median = th term = 2 = 4th term = x
2
 But median = 6
 x=6
P.Q. Find the median of : 17, 26, 60, 45, 33, 32, 29, 34, 56
If 26 is replaced by 62, find the new median.
Sol. Arranging the given data in ascending order : 17, 26, 29, 32, 33, 34, 45, 56, 60
Here n = 9 which is odd
n1 9  1 10
 Median = th term = 2  2 = 5th term = 33
2
(ii) If 26 is replaced by 62, their the order will be 17, 29, 32, 33, 34, 45, 56, 60, 62
Here 5th term is 34
 Median = 34 Ans.
11. The marks scored by 16 students in a class test are :
3, 6, 8, 13, 15, 5, 21, 23, 17, 10, 9, 1, 20, 21, 18, 12
651 Arun Deep's Understanding Math-10
Find (i) the median (ii) lower quartile (iii) upper quartile (iv) inter quartile range.
Sol. Arranging the given data in ascending order :
1, 3, 5, 6, 8, 9, 10, 12, 13, 15, 17, 18, 20, 21, 21, 23
Here n = 16 which is even.

(i)  Median =
LM
1 16
th 
16 FG
 1 th term
IJ OP
2 2 N 2 H K Q
1 12  13 25
= 2 (8th + 9th) term    12. 5
2 2
1 16
(ii) Lower quartile = n = 4th term = 6
4 4

3 3
(iii) Upper quartile = n   16 = 12th term = 18 Ans.
4 4
(iv) Inter quartile range = Q3 – Q1 = 18 – 6 = 12
PQ. Find the median and mode for the set of numbers : 2, 2, 3, 5, 5, 5, 6, 8, 9
Sol. Here n = 9 which is odd.
n1 9  1 10
 Median = th =  = 5th term = 5
2 2 2
Here 5 occur the maximum times
 Mode = 5 Ans.
12. Calculate the mean, the median and the mode of the following distribution :
Age in years 12 13 14 15 16 17 18
No. of students 2 3 5 6 4 3 2
Sol.
Age (in years) No. of students c.f. fi (xi)
(xi) ( fi )
12 2 2 24
13 3 5 39
14 5 10 70
15 6 16 90
16 4 20 64
17 3 23 51
18 2 25 36
Total 25 374
652 Arun Deep's Understanding Math-10

(i) Mean =
 fx  374  14  96
 f 25
(ii) Here n = 25 which is odd.
n1 25  1 26
 Median = th term = 2  2 = 13th term = 15
2
(iii) Here 15 occurs most i.e. in 6 times
 Mode = 15 Ans.
13. The daily wages of 30 employees in an establishment are distributed as follows :
Daily wages (in `) 0–10 10–20 20–30 30–40 40–50 50–60
No. of employees 1 8 10 5 4 2
Estimate the modal daily wages for this distribution by a graphical method.

Sol.
Daily wages (in `) No. of employees

0–10 1
10–20 8
20–30 10
30–40 5
40–50 4
50–60 2

Taking daily wages on x-axis and No. of employees on y-axis and draw a histogram as shown.
Join AB and CD intersecting each other at M.
From M draw ML perpendicular to x-axis, L is the mode
653 Arun Deep's Understanding Math-10

 Mode = `23 Ans.


PQ. Using the data given below, construct the cumulative frequency table and draw the ogive. From
the ogive, estimate : (i) the median (ii) the interquartile range.
Marks 0–10 10–20 20–30 30–40 40–50 50–60 60–70 70–80
Frequency 3 8 12 14 10 6 5 2
Also state the median class.

Sol. Marks Frequency Cumulative Frequency

0–10 3 3
10–20 8 11
20–30 12 23
30–40 14 37
40–50 10 47
50–60 6 53
60–70 5 58
70–80 2 60

Now plot the points (10, 3), (20, 11), (30, 23),
(40, 37), (50, 47), (60, 53), (70, 58), (80, 60)
on the graph and join them in free hand to form
an ogive as shown.
Here n = 60 which is an even number

(i)  Median =
1 nLM
th 
n FG IJ OP
 1 th term =
1 60
th 
60 LM FG
 1 th term
IJ OP
2 2 N 2 H K Q2 2 2 N H K Q
1
= 2 (30 + 31) = 30·5 th
Now take a point A (30·5) on y-axis. From A draw a line parallel to x-axis meeting the curve at P
and from P, draw a perpendicular to x-axis meeting it in Q. Q is the median which is 35 and
median class is 30–40.
n 60
(ii) Lower quartile =   15
4 4
3 3
Upper quartile = n   60  45
4 4
Now take points B (15) and C (45) on y-axis and from B and C draw lines parallel to x-axis
meeting the curve at L and M respectively. From L and M, draw lines perpendicular to x-axis
meeting it at E and F respectively. E and F are lower and upper quartile which are 22·3 and 47.
 Interquartile range = Q3 – Q1 = 47·0 – 22·3 = 24·7
654 Arun Deep's Understanding Math-10
14. Draw a cumulative frequency curve for the following data :

Marks obtained 0–10 10–20 20–30 30–40 40–50


No. of students 8 10 22 40 20

Hence determine :
(i) the median
(ii) the pass marks if 85% of the students pass.
(iii) the marks which 45% of the students exceed.

Sol. Marks Obtained No.of Students c.f.

0–10 8 8
10–20 10 18
20–30 22 40
30–40 40 80
40–50 20 100

Now plot points (10, 8), (20, 18), (30, 40), (40, 80) and (50, 100) on the graph and join them in
free hand to form an ogive.
Here n = 100 which is even

(i)  Median =
LM
1 n
th 
nFG IJ OP
 1 th term
N
2 2 2 H K Q
=
1 100 LMth 
100 FG
 1 th term
IJ OP
2 2 N 2 H K Q
1 1
= 2 (50th + 51th) = 2 × 101 = 50·5
Now take a point A (50·5) on y-axis and from
A, draw a line parallel to x-axis meeting the
curve at P and from P, draw a perpendicular
to x-axis meeting it at Q. Q is the median
which is 32·5.
(i) If 85% students pass, the pass marks will be
18.
(ii) Marks which 45% of the students exceeds
= 34 marks.
22
Probability
Points to Remember :
Number of favourable outcomes
1. Probability : Probability is a measure of =
Number of possible outcomes
uncertainty probability can be defined in
two ways; So, the empirical probability depends upon
(i) Empirical (or experimental or statistical) the number of trials undertaken and the
probability. number of times the event occurs in these
(ii) Classical (or theoretical on mathematical) trials.
probability. 7. Random experiment : An experiment is
2. Experiment : An action which results in called random if it has more than one
some well defined out comes is called an possible outcome and it is not possible to
experiment. tell the outcome in advance. For example:
3. Trial : An action performed once which tossing a coin, tossing of two coins,
results in some well defined outcomes is throwing a die or drawing a card from a
called a trial. pack of 52 (playing) cards etc.
4. Event : The possible outcomes of an 8. Classical probability : An assumption that
experiment are called events. For example all the experiments have equally likely
when a die is thrown, all the six outcomes outcomes leads to the following definition
the number 1, 2, 3, 4, 5, 6 are events. of classical or theoretical probability.
5. Ocurrence of an event : When the i.e.,
outcome of an experiment satisfies the
Number of outcomes favourable to E
condition mentioned in the event, then we P (E) = Total number of possible outcomesof the experiment
say that event has occurred. For example,
in the experiment of throwing a die, an Note : Here 0 < P (E) < 1
event E, may be taken as getting an even (i) When probability is sure then event is 1.
number. If the die comes up with any of (ii) Where event is impossible, then probability
the numbers 2, 4 or 6, we say that event E
is zero (0).
has accurred otherwise, if the die comes
up with 1, 3 or 5, we say that event E has 9. Complementary Event : If E is an event,
not occurred. then the event ‘not E’ is complementary
Note : Here the word event is used it will event of E and it is denoted by E or EC
mean that the event has occurred. It is true that for an event E,
6. Empirical probability : The definition of
P ( E ) = 1 – P (E) or P ( E ) = 1 – P ( E )
empirical probability as under. Let E be an
event, then empirical probability written as or P (E) + P ( E ) = 1
P (E) is defined as
i.e. sum of the probability of all the
P (E) = elementary events of an experiment is 1.
Number of trials in which E has occurred Note :
Total number of trials (i) In a pack or deck of 52 playing cards, they
655 Arun Deep's Understanding Math-10
656 Arun Deep's Understanding Math-10
are divided into 4 suits of 13 cards each. Sol. Number of total screws = 600
i.e. spades (), hearts (), diamonds ( )
and clubs (). 1
Rusted screws = of 600 = 60
(ii) Spades and clubs are of black colour and 10
hearts and diamonds are of red colour.  Good screws = 600 – 60 = 540
The cards in each suit, are ace, king, queen, Probability of a good screw
jack, 10, 9, 8, 7, 6, 5, 4, 3 and 2
(iii) Kings, queen and Jacks are called face Number of favourable outcome
cards. P (E) = Number of possible outcome
2. When two coins are tossed simultaneously
then possible out comes can be 22 i.e. 4 540 9
which are (H, H), (H, T), (T, H), (T, T) = = Ans.
600 10
where H is denoted for head and T is for
Tail of the coin. P.Q. A bag contains a red ball, a blue ball and a
yellow ball, all the balls being of the same
3. When two dice are thrown simultaneously,
size. Anjali takes out a ball from the bag
thus number of even can be 6 2 = 36
without looking into it. What is the
because each die has 1 to 6 number on its
faces. Then the possible outcome can be. probability that she takes out (i) yellow ball?
(ii) red ball ? (iii) blue ball ?
1 2 3 4 5 6
Sol. Number of balls in the bag = 3.
1 (1, 1) (1, 2) (1, 3) (1, 4) (1, 5) (1, 6)
2 (2, 1) (2, 2) (2, 3) (2, 4) (2, 5) (2, 6) 
(i) Probability of yellow ball =
3 (3, 1) (3, 2) (3, 3) (3, 4) (3, 5) (3, 6) 3
4 (4, 1) (4, 2) (4, 3) (4, 4) (4, 5) (4, 6)

5 (5, 1) (5, 2) (5, 3) (5, 4) (5, 5) (5, 6) (ii) Probability of red ball =
3
6 (6, 1) (6, 2) (6, 3) (6, 4) (6, 5) (6, 6)

Note : (iii) Probability of blue ball =
3
(i) The outcomes (1, 1), (2, 2), (3, 3), (4, 4),
(5, 5), (6, 6) are called doublets. 2. In a lottery, there are 5 prized tickets and
(ii) The pair (1, 2) and (2, 1) are different 995 blank tickets. A person buys a lottery
outcomes. ticket. Find the probability of his winning
a prize.
Sol. Number of prized tickets = 5
EXERCISE 22
Number of blank tickets = 995
1. A box contains 600 screws, one-tenth are
rusted. One screw is taken out at random Total number of tickets = 5 + 995 = 1000
from this box. Find the probability that it Probability of prized ticket
is a good screw.
657 Arun Deep's Understanding Math-10
Sania winning the match is 0.69. What is
Number of favourable outcome
P (E) = the probability of Sonali winning ?
Number of possible outcome
Sol. Probability of Sania’s winning the game
= 0.69
5 1
= = Ans. Let P (E) be the probability of Sania’s
1000 200
3. 12 defective pens are accidentally mixed
winning the game and P E be the
with 132 good ones. It is not possible to probability of Sania’s losing the game or
just look at a pen and tell whether or not it probability of Sonali, winning the game
is defective. One pen is taken out at random
from this lot. Determine the probability that

 P (E) + P E = 1

0.69 + P E  = 1
the pen taken out is a good one.

Sol. Number of defective pens = 12
Number of good pens = 132  P E  = 1 – 0.69 = 0.31
 Total number of pens = 12 + 132 = 144 Hence probability of Sonali’s winning the
Probability of good pen = game = 0.31 Ans.
5. A bag contains 3 red balls and 5 black balls.
Number of favourable outcome
P (E) = A ball is drawn at random from in bag.
Number of possible outcome What is the probability that the ball drawn
is
132 11
= = Ans. (i) red ? (ii) not red ?
144 12
Sol. Number of red balls = 3
5 Number of black balls = 5
P.Q. If the probability of winning a game is ,
11  Total balls = 3 + 5 = 8
what is the probability of losing ? Let P (E) be the probability of red balls,
5 then P ( E ) will be the probability of not
Sol. Probability of winning game = red balls.
11

5 
P (E) + P E = 1
 P (E) = (i) But P (E) =
11
We know that P (E) + P ( E ) = 1 Number of favourable outcome 3
where P (E) is the probability of losing the Number of possible outcome = 8
game.
(ii) P ( E ) = 1 – P (E)
5
 + P (E) = 1 3 83 5
11 =1– = = Ans.
8 8 8
5 11 5 6 P.Q. There are 40 students in Class X of a
 P (E) = 1 – = = Ans.
11 11 11 school of which 25 are girls and the others
4. Two players, Sania and Sonali play a tennis are boys. The class teacher has to select
match. It is known that the probability of one student as a class representative. She
658 Arun Deep's Understanding Math-10
writes the name of each student on a No. of constant = 25 – 5 = 21
separate card, the cards being identical.
Then she puts cards in a bag and stirs them Number of favourable outcome
 P (E) =
thoroughly. She then draws one card from Number of possible outcome
the bag. What is the probability that the
name written on the card is the name of 21
(i) a girl ? (ii) a boy ? = Ans.
26
Sol. Number of total students = 40 P.Q. A bag contains 5 black, 7 red and 3 white
Number of girls = 25 balls. A ball is drawn at random from the
Number of boys = 40 – 25 = 15 bag, find the probability that the ball drawn
(i) Probability of a girl is :
(i) red
Number of favourable outcome
P (E) = (ii) black or white
Number of possible outcome
(iii) not black.
25 5 Sol. In a bag,
= = Number of black balls = 5
40 8
(ii) Probability of a boy Number of red balls = 7
and number of white balls = 3
Number of favourable outcome
P (E) = Total number of balls in the bag
Number of possible outcome = 5 + 7 + 3 = 15
(i) Probability of red balls
15 3
= = Ans.
40 8 Number of favourable outcome
P (E) =
6. A letter is chosen from the word Number of possible outcome
‘TRIANGLE’. What is the probability that
it is a vowel ? 7
=
Sol. There are three vowels : I, A, E 15
 Number of letters in the word (ii) Probability of black or white balls
‘TRIANGLE’ = 8.
 Probability of vowel Number of favourable outcome
P (E) = Number of possible outcome
Number of favourable outcome
P (E) = Number of possible outcome 53 8
= =
15 15
3
= Ans. (iii) Probability of not black balls
8
7. A letter of English alphabet is chosen at Number of favourable outcome
random. Determine the probability that the P (E) =
Number of possible outcome
letter is a consonant.
Sol. No. of English alphabet = 26 7  3 10 2
= = = Ans.
No. of vowel = 5 15 15 3
659 Arun Deep's Understanding Math-10
8. A box contains 7 blue, 8 white and 5 black (iii) not green
marbles. If a marble is drawn at random (iv) neither white nor black.
from the box, what is the probability that it Sol. In a bag,
will be Number of red balls = 6
(i) black? (ii) blue or black? Number of white balls = 8
Number of green balls = 5
(iii) not black? (iv) green?
and number of black balls = 3
Sol. Total number of marbles in the box
 Total number of balls in the bag
= 7 + 8 + 5 = 20 = 6 + 8 + 5 + 3 = 22
Since, a marble is drawn at random from (i) Probability of white balls
the box
Number of favourable outcome
(i) Probability (of a black Marble) P (E) = Number of possible outcome
Number of favourable outcome
P (E) = 8 4
Number of possible outcome = =
22 11
5 1 (ii) Probability of red or black balls
= =
20 4 Number of favourable outcome
P (E) =
(ii) Probability (of a blue or black marble) Number of possible outcome

Number of favourable outcome 63 9


P (E) = = =
Number of possible outcome 22 22
(iii) Probability of not green balls i.e. having
75 12 3 red, white and black balls.
= = =
20 20 5 Number of favourable outcome
(iii) Probability (of not black marble) P (E) =
Number of possible outcome
= 1 – P (of black 1)
683 17
1 4 1 3 = =
=1– = = 22 22
4 4 4 (iv) Probability of neither white nor black balls
(iv) P (of a green marble) = 0 i.e. red and green balls
( Since, a box does not contain a green Number of favourable outcome
marble, so the probability of green marble P (E) =
Number of possible outcome
will be zero)
9. A bag contains 6 red balls, 8 white balls, 5 65 11 1
= = = Ans.
green balls and 3 black balls. One ball is 22 22 2
drawn at random from the bag. Find the P.Q. A piggy bank contains hundred 50 p coins,
probability that the ball is : fifty `1 coins, twenty `2 coins and ten `5
(i) white coins. It is equally likely that one of the coins
will fall down when the bank is turned upside
(ii) red or black
down, what is the probability that the coin
660 Arun Deep's Understanding Math-10
(i) will be a 50 p coin?
25  65 90 9
(ii) will not be `5 coin? = = =
100 100 10
Sol. In a piggy bank, there are
(iii) Probability of persons having blue or black
100, 50 p coin eyes
50, `1 coin
20, `2 coin Number of favourable outcome
P (E) = Number of possible outcome
10, `5 coin
Total coins = 100 + 50 + 20 + 10 = 180
10  65 75 3
One coin is drawn at random = = =
100 100 4
 Probability of
(iv) Probability of persons having neither blue
100 5 nor brown eyes i.e. having only black eyes
(i) 50 p coins = =
180 9 Number of favourable outcome
(ii) Will not be `5 coins P (E) =
Number of possible outcome
= 100 + 50 + 20 = 170
65 13
170 17 = = Ans.
 Probability = = Ans. 100 20
180 18
10. A carton consists of 100 shirts of which
P.Q. If 65% of the population have black eyes,
88 are good, 8 have minor defects and 4
25% have brown eyes and the remaining have major defects. Peter, a trader, will only
have blue eyes, what is the probability that accept the shirts which are good, but Salim,
a person selected at random has another trader, will only reject the shirts
(i) blue eyes (ii) brown or black eyes (iii) which have major defects. One shirts is
blue or black eyes (iv) neither blue nor drawn at random from the carton. What is
brown eyes ? the probability that
Sol. Population of people having black eyes (i) it is acceptable to Peter ?
= 65% (ii) it is acceptable to Salim ?
Population of persons having brown eyes Sol. In a carton, there the 100 shirts. Among
= 25 % these number of shirts which are good
Population of persons having blue eyes = 88
= 100 – (65 + 25) = 100 – 90 = 10% number of shirts which have minor defect
=8
(i) Probability of people having blue eyes
number of shirt which have major defect
Number of favourable outcome =4
P (E) = Number of possible outcome Total number of shirts = 88 + 8 + 4 = 100
Peter accepts only good shirts i.e. 88
10 1 Salim rejects only shirts which have major
= = defect i.e. 4
100 10
(i) Probability of good shirts which are
(ii) Probability of brown or black eyed persons
acceptable to Peter
Number of favourable outcome Number of favourable outcome
P (E) = P (E) =
Number of possible outcome Number of possible outcome
661 Arun Deep's Understanding Math-10

88 22 3 1
= = = =
100 25 6 2
(ii) A number less than 5 are 1, 2, 3, 4
(ii) Probability of shirts acceptable to Salim
 Probability of a number less than 5 is
Number of favourable outcome Number of favourable outcome
P (E) = P (E) =
Number of possible outcome Number of possible outcome
4 2
88  8 96 24 = =
= = = Ans. 6 3
100 100 25 (iii) A number greater than 5 is 6
11. A die is thrown once. What is the probability  Probability of a number greater than 5 is
that the Number of favourable outcome
(i) number is even P (E) = Number of possible outcome
(ii) number is greater than 2 ? 1
Sol. Dice is thrown once =
6
Gample space = {1, 2, 3, 4, 5, 6} (iv) Prime number is 2, 3, 5
(i) No. of ways in favour = 3  Probability of a prime number is
( Even numbers are 2, 4, 6)
Total ways = 6 Number of favourable outcome
P (E) = Number of possible outcome
3 1
 Probability = 
6 2 3 1
= =
(ii) No. of ways in favour = 4 6 2
(Numbers greater than 2 are 3, 4, 5, 6) (v) Number less than 7 are 1, 2, 3, 4, 5, 6
Total ways = 6  Probability of getting a number less than 7
4 2 Number of favourable outcome 6
 Probability =  = = =1
6 3 Number of possible outcome 6
12. In a single throw of a die, find the (vi) A number divisible by 3 is 3, 6
probability of getting :  Probability of a number divisible by 3 is
(i) an odd number Number of favourable outcome
(ii) a number less than 5 P (E) =
Number of possible outcome
(iii) a number greater than 5
(iv) a prime number 2 1
(v) a number less than 7 = = .
6 3
(vi) a number divisible by 3 (vii) Numbers between 3 and 6 is 4, 5
(vii) a number between 3 and 6 Probability of a number between 3 and 6 is
(viii) a number divisible by 2 or 3. Number of favourable outcome
Sol. A die is thrown and on its faces, numbers  P (E) =
1 to 6 are written. Number of possible outcome
 Total numbers of possible outcomes = 6 2 1
(i) Probability of an odd number. = =
6 3
 odd number are 1, 3 and 5 (viii) Numbers divisible by 2 or 3 are 2, 4 or 3, 6
Number of favourable outcome Probability of number divisible by 2 or 3 is
 P (E) = Number of favourable outcome
Number of possible outcome
P (E) =
Number of possible outcome
662 Arun Deep's Understanding Math-10

2 1 Sol. On the face of a game, numbers 1 to 8 are


=  Ans. shown.
6 3
 Possible outcomes = 8
13. A die has 6 faces marked by the given
numbers as shown below: (i) Probability of number 8 will be

1 2 3 1 2 3 Number of favourable outcome 1


P (E) = Number of possible outcome = 8
The die is thrown once. What is the
probability of getting (ii) Odd number are 1, 3, 5, 7
(i) a positive integer. Probability of a number which is an odd will be
(ii) an integer greater than –3. Number of favourable outcome 4 1
(iii) the smallest integer ? P(E)= Number of possible outcome = 8 = 2
Sol. Total outcomes n(S)= 6
(iii) A number greater than 2 are 3, 4, 5, 6, 7, 8
(i) a positive integer = (1, 2, 3) which are 6
No. of favourables n(E) = 3 Probability of number greater than 2 will be
n(E) 3 1 Number of favourable outcome
Probability = n(S) = = P (E) =
6 2 Number of possible outcome
(ii) Integer greater than –3
= (1, 2, 3, –1, –2) 6 3
= =
No. of favourables n(E) = 5 8 4
(iv) A number less than 9 is 8.
n(E) 5
Probability = n(S) =  Probability of a number less than 9 will be
6
8
(iii) Smallest integer = –3  P (E) =
8
No. of favourables n(E) = 1
15. Find the probability that the month of
n(E) 1 January may have 5 Mondays in
Probability = n(S) =
6 (i) a leap year
14. A game of chance consists of spinning an (ii) a non-leap year.
arrow which comes to rest pointing at one Sol. In January, there are 31 days and in an
of the numbers 1, 2, 3, 4, 5, 6, 7, 8 (shown ordinary year, there are 365 days but in a
in the adjoining figure) and these are equally leap year, there are 366 days.
likely outcomes. What is the probability that (i) In January of an ordinary year, there are
it will point at 31 day i.e. 4 weeks and 3 days.
(i) 8 ? (ii) an odd number ?
3
(iii) a number greater than 2 ?  Probability of Monday will be =
7
(iv) a number less than 9 ? 8 1
7 2 (ii) In January of a leap year, there are 31 days
i.e. 4 weeks and 3 days.
6 3
5
3
4  Probability of Monday will be = Ans.
7
663 Arun Deep's Understanding Math-10
16. Find the probability that the month of February may have 5 Wednesdays in
(i) a leap year (ii) a non-leap year.
Sol. In the month of February, there are 29 days in a leap year while 28 days in a non-leap year.
(i) In a leap year, there are 4 complete weeks and 1 day
1
 Probability of Wednesday = P (E) =
7
and in a non leap year, there are 4 complete weeks and 0 days
0
 Probability of wednesday P (E) = =0
7
17. Sixteen cards are labelled as a, b, c, ..., m, n, o, p. They are put in a box and shuffled. A boy is
asked to draw a card from the box. What is the probability that the card drawn is:
(i) a vowel (ii) a consonant
(iii) none of the letters of the word median.
Sol. Here, sample space (S) = {a, b, c, d, e, f, g, h, i, j, k, l, m, n, o, p}
 n(S) = 16
(i) Vowels (V) = {a, e, i, o}
 n(V) = 4
n (V) 4 1
 P (a vowel) = n (S) = =
16 4
(ii) Consonants (C) = {b, c, d, f, g, h, j, k, l, m, n, p}
 n(C) = 12
n (C) 12 3
 P (a consonant) = n (S) = =
16 4
(iii) None of the letters of the word MEDIAN (N) = {b, c, f, g, h, j, k, l, o, p}
 n(N) = 10
n ( N) 10 5
 P(N) = n (S) = =
16 8
18. An integer is chosen between 0 and 100. What is the probability that it is
(i) divisible by 7? (ii) not divisible by 7?
Sol. Intergers between 0 and 100 = 99
(i) Number divisible by 7 are
7, 14, 21, 28, 35, 42, 49, 56, 63, 70, 77, 84, 91, 98 = 14
14
 Probability =
99
(ii) Not divisible by 7 are 99 – 14 = 85
85
 Probability =
99
19. Cards marked with numbers 1, 2, 3, 4, ...... 20 are well shuffled and a card is drawn at random.
What is the probability that the number on the card is
664 Arun Deep's Understanding Math-10
(i) a prime number (ii) divisible by 3 (iii) a perfect square ? (2010)
Sol. Number cards is drawn from 1 to 20 = 20
One card is drawn at random
 No. of total (possible) events = 20
(i) The card has a prime number
 The prime number from 1 to 20 are 2, 3, 5, 7, 11, 13, 17, 19
 Actual No. of events = 8

No. of actual events 8 2


 P (E) = = =
No. of total events 20 5
(ii) Numbers divisible by 3 are 3, 6, 9, 12, 15, 18
 No. of actual events = 6
6 3
 P (E) = =
20 10
(iii) Numbers which are perfect squares = 1, 4, 9, 16 = 4
4 1
P (E) = =
20 5
20. There are 25 discs numbered 1 to 25. They are put in a closed box and shaken thoroughly. A
disc is drawn at random from the box. Find the probability that number on a disc is:
(i) an odd number (ii) divisible by 2 and 3 both.
(iii) a number less than 16.
Sol. Total outcomes = 1, 2, 3, 4, 5, 6,.......... 25.
(i) Odd numbers = 1, 3, 5, 7, 9, 11, 13, 15, 17, 19, 21, 23, 25
n (odd numbers) = 13
13
P (odd number) =
25
(ii) Numbers divisible by 2 and 3 both = 6, 12, 18, 24
 n=4
4
P (E) =
25
(iii) Numbers less than 16 = 1, 2, 3, 4,........ 15,
 n = 15
15 3
P (E) = 
25 5
21. A box contains 15 cards numbered 1, 2, 3, ......, 15 which are mixed thoroughly. A card is
drawn from the box at random. Find the probability that the number on the card is :
(i) odd
665 Arun Deep's Understanding Math-10
(ii)prime 2, 3, 4, 6, 8, 9, 10, 12, 14, 15 which are
(iii)divisible by 3 10 in numbers
(iv) divisible by 3 and 2 both  Probability of number divisible by 3 or 2
will be
(v) divisible by 3 or 2
(vi) a perfect square number. Number of favourable outcome
Sol. Number of cards in a box = 15 numbered P (E) =
Number of possible outcome
1 to 15
(i) Odd numbers are 1, 3, 5, 7, 9, 11, 13, 15 10 2
= = Ans.
 Number of odd numbers = 8 15 3
 Probability of odd numbers will be (vi) Perfect squares number are 1, 4, 9 i.e., 3
number
Number of favourable outcome
P (E) = 3 1
Number of possible outcome  P (E) = = Ans.
15 5
8 22. A box contains 19 balls bearing numbers
= 1, 2, 3, ....., 19. A ball is drawn at random
15
from the box. Find the probability that the
(ii) Prime number are 2, 3, 5, 7, 11, 13
number on the ball is :
 Number of primes is 6
(i) a prime number
 Probability of prime number will be
(ii) divisible by 3 or 5
Number of favourable outcome (iii) neither divisible by 5 nor by 10
P (E) =
Number of possible outcome (iv) an even number.
Sol. In a box, number of balls = 19 with
6 2 numbered 1 to 19. A ball is drawn
= =
15 5  Number of possible outcomes = 19
(iii) Numbers divisible by 3 are 3, 6, 9, 12, 15 (i) Prime number = 2, 3, 5, 7, 11, 13, 17, 19
which are 5 in numbers which are 8 in number.
 Probability of number divisible by 3 will be  Probability of prime number will be
Number of favourable outcome Number of favourable outcome
P (E) = P (E) =
Number of possible outcome Number of possible outcome

5 1 8
= = =
15 3 19
(iv) Divisible by 3 and 2 both are 6, 12 (ii) Divisible by 3 or 5 are 3, 5, 6, 9, 10, 12,
which are 2 in numbers. 15, 18 which are 8 in number
Probability of number divisible by 3 and 2  Probability ofr number divisible by 3 or 5
2 will be
both will be =
15 Number of favourable outcome
(v) Numbers divisible by 3 or 2 are P (E) =
Number of possible outcome
666 Arun Deep's Understanding Math-10
(iii) Number which are multiple of 6 = {6, 12, 18}
No. of favourable cards = 3
Number of favourable outcome 3
Hence, probability of getting card, which is multiple of 6 =
Number of possible outcome = 10
(iv) Odd number = 0 (No. odd card)
No. of favourable cards = 0
Number of favourable outcome 0
Hence, probability of getting an odd number card = Number of possible outcome = 10 = 0
23. Cards marked with numbers 13, 14, 15, ..., 60 are placed in a box and mixed thoroughly. One card
is drawn at random from the box. Find the probability that the number on the card drawn is
(i) divisible by 5 (ii) a perfect square number.
Sol. Number of card bearing numbers 13, 14, 15, ... 60 = 48
One card is drawn at random.
(i) Card divisible by 5 are 15, 20, 25, 30, 35, 40, 45, 50, 55, 60 = 10
10 5
 Probability = =
48 24
(ii) A perfect square = 16, 25, 36, 49 = 4
4 1
 Probability = =
48 12
24. Tickets numbered 3, 5, 7, 9, ....., 29 are placed in a box and mixed thoroughly. One ticket is
drawn at random from the box. Find the probability that the number on the ticket is
(i) a prime number (ii) a number less than 16 (iii) a number divisible by 3.
Sol. In a box there are 14 tickets with number 3, 5, 7, 9, 11, 13, 15, 17, 19, 21, 23, 25, 27, 29
 Number of possible outcomes = 14
(i) Prime numbers are 3, 5, 7, 11, 13, 17, 19, 23, 29 which are 9 in number
 Probability of prime will be
Number of favourable outcome 9
P (E) =
Number of possible outcome = 14
(ii) Number less than 16 are 3, 5, 7, 9, 11, 13, 15 which are 7 in numbers,
 Probability of number less than 16 will be
Number of favourable outcome 7 1
P (E) = 
Number of possible outcome = 14 2 Ans.
(iii) Numbers divisible by 3 are 3, 9, 15, 21, 27
which are 5 in number
 Probability of number divisible by 3 will be
Number of favourable outcome 5
P (E) =
Number of possible outcome = 14 Ans.
25. A box contains 90 discs which are numbered from 1 to 90. If one disc is drawn at random from
the box, find the probability that it bears
667 Arun Deep's Understanding Math-10
(i) a two-digit number
Number of favourable outcomes
(ii) a perfect square number P(E) =
Number of possible outcome
(iii) a number divisible by 5.
(iv) a prime number less than 30. 10 1
=  Ans.
Sol. There are 90 discs in a box containing 100 10
numbered from 1 to 90. PQ. Cards marked with numbers 2 to 101 are
 Number of possible outcomes = 90 placed in a box and mixed thoroughly. One
card is drawn at random from this box.
(i) Two digit numbers are 10 to 90 which are
Find the probability that the number on the
81 in numbers.
card is
 Probability of two digit number will be (i) an even number
Number of favourable outcomes (ii) a number less thatn 14
P (E) = Number of possible outcomes (iii) a number which is a perfect square
Sol. Number of cards with numbered from 2
81 9 to 101 are placed in a box
= =
90 10  Number of possible outcomes = 100
(ii) Perfect squares are 1, 4, 9, 16, 25, 36, 49, one card is drawn
64, 81 which are 9 in numbers. (i) Even numbers are 2, 4, 6, 8, 10, 12, 14,
16, .........., 96, 96, 100
 Probability of square will be
which are 50 in numbers.
Number of favourable outcome Probability of even number will be
P (E) =
Number of possible outcome
Number of favourable outcome
P (E) =
Number of possible outcome
9 1
= =
90 10 50 1
(iii) Number divisible by 5 are 5, 10, 15, 20, = =
100 2
25, 30, 35, 40, 45, 50, 55, 60, 65, 70, 75,
(ii) Numbers less than 14 are 2, 3, 4, 5, 6, 7,
80, 85, 90 which are 18 in numbers.
8, 9, 10, 11, 12, 13
 Probability of number divisible by 5 will be which are 12 in numbers
Number of favourable outcome  Probability of number less than 14 will be
P (E) =
Number of possible outcome Number of favourable outcome
P (E) = Number of possible outcome
18 1
= = Ans.
90 5 12 3
= =
(iv) Prime numbers less than 30 are 2, 3, 5, 7, 100 25
11, 13, 17, 19, 23, 29 (iii) Perfect square are 4, 9, 16, 25, 36, 49, 64,
which are 10 in numbers 81, 100 which are 9 in numbers
Probability of prime numbers, less than 30  Probability of perfect square number will
will be be
668 Arun Deep's Understanding Math-10

Number of favourable outcome x 6


P (E) =  =2×
Number of possible outcome x6 x6

9 x 12
=  =  x = 12
100 x6 x6
26. A bag contains 15 balls of which some are  Number of balls = x + 6 = 12 + 6 = 18 Ans.
white and others are red. If the probability 28. A bag contains 24 balls of which x are red,
of drawing a red ball is twice that of a white 2x are white and 3x are blue. A blue is
ball, find the number of white balls in the selected at random. Find the probability tht
bag. it is
Sol. In a bag, there are 15 balls. (i) white (ii) not red.
Some are white and others are red. Sol. In a bag, there are 24 balls
Probability of red ball = 2 probability of Since, there are x balls red, 2x balls white
white ball and 3x balls blue
Let number of white balls = x  x + 2x + 3x = 24
Then, number of red balls = 15 – x 24
 6x = 24  x = =4
15  x x 6
 2× = Hence, there shall be 4 red balls,
15 15
2 (15 – x) = x  30 – 2x = x (2 × 4) = 8 white balls
 30 = x + 2x  3x = 30 and (3 × 4) = 12 blue balls
8 1
30 (i) P (white) = =
 x= = 10 24 3
3
 Number of red balls = 10 8  12 20 5
(ii) P (not red) = = =
and Number of white balls = 15 – 10 = 5 Ans. 24 24 6
27. A bag contains 6 red balls and some blue ( P (not red) means probability of either
balls. If the probability of drawing a blue white or blue)
ball is twice that of a red ball, find the 29. A card is drawn from a well-shuffled pack
number of balls in the bag. of 52 cards. Find the probability of getting:
Sol. In a bag, there are 6 red balls, and some (i) ‘2’ of spades
blue balls (ii) a jack
Probability of blue ball = 2 probability of (iii) a king of red colour
red ball
(iv) a card of diamond
Let number of blue balls = x (v) a king or a queen
and number of red balls = 6 (vi) a non-face card
 Total balls = x + 6 (vii) a black face card
Probability of a blue ball = 2 (viii) a black card
(Probability of red ball) (ix) a non-ace
669 Arun Deep's Understanding Math-10
(x) non-face card of black colour
8 2
(xi) neither a spade nor a jack = =
52 13
(xii) neither a heart nor a red king
(vi) Non-face cards are = 52 – 3 × 4 = 52 – 12
Sol. In a playing card, there are 52 cards
= 40
 Number of possible outcome = 52
 Probability of non-face card will be
(i) Probability of ‘2’ of spade will be
Number of favourable outcome
Number of favourable outcome P (E) =
P (E) = Number of possible outcome Number of possible outcome

1 40 10
= = =
52 52 13
(ii) There are 4 jack card (vii) Black face cards are = 2 × 3 = 6
 Probability of jack will be  Probability of black face card will be

Number of favourable outcome Number of favourable outcome


P (E) = P (E) =
Number of possible outcome Number of possible outcome

4 1 6 3
= = = =
52 13 52 26
(iii) King of red colour are 2 in number (viii) No. of black cards = 13 × 2 = 26
 Probability of red colour king will be  Probability of black card will be
Number of favourable outcome Number of favourable outcome
P (E) = P (E) =
Number of possible outcome Number of possible outcome

2 1 26 1
= = = =
52 26 52 2
(iv) Cards of diamonds are 13 in number
(ix) Non-ace cards are 12 × 4 = 48
 Probability of diamonds card will be
 Probability of non-ace card will be
Number of favourable outcome
P (E) = Number of favourable outcome
Number of possible outcome P (E) =
Number of possible outcome
13 1
= = 48 12
52 4 = =
52 13
(v) Number of kings and queens = 4 + 4 = 8
 Probability of king or queen will be (x) Non-face card of black colours are 10 × 2
= 20
Number of favourable outcome  Probability of non-face card of black colour
P (E) =
Number of possible outcome will be
670 Arun Deep's Understanding Math-10

Number of favourable outcome 3


P (E) =  Probability =
Number of possible outcome 49
(iii) Probability of being a black card
20 5
= = 23
52 13 = (26 – 3 = 23) =
49
(xi) Number of card which are neither a spade
nor a jack = 13 × 3 – 3 = 39 – 3 = 36 13
(iv) Probability of being a heart =
Probability of card which is neither a spade 49
nor a jack will be (v) Probability of being a spade = (13 – 3 = 10)

Number of favourable outcome 10


P (E) = =
Number of possible outcome 49
(vi) Probability of being 9 of black colour (which
36 9 2
= = are 2) =
52 13 49
(xii) Number of cards which are neither a heart 31. From a pack of 52 cards, a black jack, a red
nor a red king = 3 × 13 = 39 – 1 = 38 queen and two black kings fell down. A card
 Probability of card which is neither a heart was then drawn from the remaining pack at
nor a red king will be random. Find the probability that the card
drawn is
Number of favourable outcome (i) a black card (ii) a king
P (E) =
Number of possible outcome (iii) a red queen.
Sol. In a pack of 52 cards, a black jack, a red
38 19 queen, two black being feel down.
= = Ans.
52 26 Then number of total out comes
30. All the three face cards of spades are removed = 52 – (1 + 1 + 2) = 48
from a well-shuffled pack of 52 cards. A card (i) Probability of a black card (which are 26 – 3
is then drawn at random from the remaining
pack. Find the probability of getting 23
= 23) =
(i) a black face card (ii) a queen 48
(iii) a black card (iv) a heart 2
(v) a spade (vi) '9' of black colour (ii) Probability of a being (4 – 2 = 2) =
48
Sol. In a pack of 52 cards
All the three face cards of spade are = 3 1
=
Number of remaining cards = 52 – 3 = 49 24
One card is drawn at random (iii) Probability of a red queen = (2 – 1 = 1)
(i) Probability of a black face card which are 1
=
3 48
=6–3=3=
49 32. Two coins are tossed once. Find the
(ii) Probability of being a queen which are probability of getting :
4–1=3 (i) 2 heads (ii) at least one tail.
671 Arun Deep's Understanding Math-10
Sol. Total possible outcomes are : HH, HT, TT, TH, i.e., 4
(i) Favourable outcomes are HH, i.e., 1
So, P(2 heads)

no. of favourable outcomes 1


= total no. of possible outcomes =
4
(ii) Favourable outcomes are HT, TT, TH, i.e., 3

3
So, P (at least one tail) =
4
33. Two different coins are tossed simultaneously. Find the probability of getting :
(i) two tails (ii) one tail
(iii) no tail (iv) atmost one tail.
Sol. Two different coins are tossed simultaneously
 Number of possible outcomes = (2)2 = 4
Number of event having two tails = 1 i.e. (T, T)
(i) Probability of two tails will be

Number of favourable outcome 1


P (E) =
Number of possible outcome = 4

(ii) Number of events having one tail


= 2 i.e. (TH) and (HT)
 Probability of one tail will be

Number of favourable outcome 2 1


P (E) =
Number of possible outcome = 4 = 2

(iii) Number of events having no tail = 1 i.e. (HH)


 Probability of having no tail will be

Number of favourable outcome 1


P (E) =
Number of possible outcome = 4

(iv) Atmost one tail


Number of events having at the most one tail = 3 i.e. (TH), (HT, (TT)
 Probability of at the most one tail will be

Number of favourable outcome 3


P (E) = Number of possible outcome = 4 Ans.
672 Arun Deep's Understanding Math-10
34. Two different dice are thrown simultaneously. Find the probability of getting:
(i) a number greater than 3 on each dice (ii) an odd number on both dice.
Sol. When two different dice are thrown simultaneously, then the sample space S of the random
experiment =
{(1, 1), (1, 2), (1, 3), (1, 4), (1, 5), (1, 6)
(2, 1), (2, 2), (2, 3), (2, 4), (2, 5), (2, 6)
(3, 1), (3, 2), (3, 3), (3, 4), (3, 5), (3, 6)
(4, 1), (4, 2), (4, 3), (4, 4), (4, 5), (4, 6)
(5, 1), (5, 2), (5, 3), (5, 4), (5, 5), (5, 6)
(6, 1), (6, 2), (6, 3), (6, 4), (6, 5), (6, 6)}
It consists of 36 equally likely outcomes.
(i) Let E be the event of 'a number greater than 3 on each dice'.
 E = {(4, 4), (4, 5), (4, 6), (5, 4), (5, 5), (5, 6), (6, 4), (6, 5), (6, 6)}
No. of favourable outcomes (E) = 9

9 1
 P (number greater than 3 on each dice) = =
36 4
(ii) Let E be the event of 'an odd number on both dice'.
 E = {(1, 1), (1, 3), (1, 5), (3, 1), (3, 3), (3, 5), (5, 1), (5, 3), (5, 5)}
No. of favourable outcomes (E) = 9

9 1
 P (Odd on both dices) = =
36 4
35. Two different dice are thrown at the same time. Find the probability of getting :
(i) a doublet (ii) a sum of 8
(iii) sum divisble by 5 (iv) sum of atleast 11.
Sol. Two different dice are thrown at the same time
 Possible outcomes will be (6)2 i.e. 36
(i) Number of events which doublet = 6 i.e. (1, 1), (2, 2) (3, 3), (4, 4), (5, 5) and (6, 6)
 Probability of doublets will be

Number of favourable outcome 6 1


P (E) = Number of possible outcome = =
36 6
(ii) Number of event in which the sum is 8 are (2, 6), (3, 5), (4, 4), (5, 3), (6, 2) = 5
 Probability of a sum of 8 will be

Number of favourable outcome 5


P (E) = Number of possible outcome =
36
673 Arun Deep's Understanding Math-10
(iii) Number of event when sum is divisible by selecting an even number is
5 are (1, 4), (4, 1), (2, 3), (3, 2), (4, 6),
1 1
(5, 5) = 7 in numbers (a) (b)
2 4
 Probability of sum divisible by 5 will be
4 2
Number of favourable outcome (c) (d)
P (E) = Number of possible outcome 9 5
Sol. One digit prime numbers are 2, 3, 5, 7 = 4
7  Probability of an even prime number (i.e., 2)
= 1
36 = (b)
(iv) Sum of atleast 11, will be in following events 4
(5, 6), (6, 5), (6, 6) 4. Out of vowels, of the English alphabet, one
 Probability of sum of atleast 11 will be letter is selected at random. The probability
of selecting 'e' is
Number of favourable outcome
P (E) = 1 5
Number of possible outcome (a) (b)
26 26

3 1 1 1
= = Ans. (c) (d)
36 12 4 5
Sol. Vowels of English alphabet are
MULTIPLE CHOICE QUESTIONS a, e, i, o, u = 5
Choose the correct answer from the given four One letter is selected at random.
options (1 to 28): 1
1. Which of the following cannot be the The probability of selecting 'e' = (d)
5
probability of an event?
5. When a die is thrown, the probability of
2 getting an odd number less than 3 is
(a) 0.7 (b)
3
1 1
(c) –1.5 (d) 15% (a) (b)
6 3
Sol. –1.5 (negative) can not be a probability as a
probability is possible 0 to 1. (c) 1
2. If the probability of an event is p, then the (c) (d) 0
2
probability of its complementary event will Sol. A die is thrown
be
Total number of events = 6
(a) p–1 (b) p
Odd number less than 3 is 1 = 1
1
(c) 1 – p (d) 1 – 1
p  Probability = (a)
6
Sol. Complementary of p is 1 – p 6. A fair die is thrown once. The probability of
 Probability of complementary even of p is getting an even prime number is
1 – p. (c)
3. Out of one digit prime numbers, one number 1 2
(a) (b)
is selected at random. The probability of 6 3
674 Arun Deep's Understanding Math-10

1 1 1 1
(c) (d) (a) (b)
3 2 3 4
Sol. A fair die is thrown once 1 1
 Total number of outcomes = 6 (c) (d)
5 6
Prime numbers = 2, 3, 5 and even prime is 2
Sol. A die having 6 faces bearing letters
Probability of getting an even prime number
A, B, C, D, A, C
1  Total number of outcomes = 4
= (a)
6 Probability of getting C, (2)
6. A fair die is thrown once. The probability of 2 1
getting a composite number is = = (a)
6 3
1 1 8. If a letter is chosen at random from the letters
(a) (b)
3 6 of English alphabet, then the probability that
it is a letter of the word 'DELHI' is
2
(c) (d) 0 1 1
3 (a) (b)
Sol. A fair die is thrown once 5 26
 Total number of outcomes = 6 5 21
Composite numbers are 4, 6 = 2 (c) (d)
26 26
2 1 Sol. Total number of English alphabets = 26
 Probability = = (a) Letter of Delhi = D, E, L, H, I = 5
6 3
7. If a fair dice is rolled once, then the 5
probability of getting an even number or a  Probability = (c)
26
number greater than 4 is P.Q. A card is drawn from a well-shuffled pack
1 1 of 52 playing cards. The event E is that the
(a) (b) card drawn is not a face card. The number
2 3
of outcomes favourable to the event E is
5 2 (a) 51 (b) 40
(c) (d)
6 3 (c) 36 (d) 12
Sol. A fair dice is thrown once. Sol. Number of playing cards = 52
 Total number of outcomes = 6 Probability of a card which is not a face card
Even numbers or a number greater than 4 = (52 – 12) = 40
= 2, 4, 5, 6 = 4  Number of possible events = 40 (b)
P.Q. A card is drawn from a deck of 52 cards.
4 2
 Probability = = (d) The event E is that card is not an ace of
6 3 hearts. The number of outcomes favourable
P.Q. Rashmi has a die whose six faces show the to E is
letters as given below : (a) 4 (b) 13
A B C D A C (c) 48 (d) 51
If she throws the die once, then the Sol. Total number of cards = 52
probability of getting C is Number of ace of heart = 1
675 Arun Deep's Understanding Math-10
Balance 52 – 1 = 51 10. If a card is drawn from a well-shuffled pack
 Number of possible events = 51 (d) of 52 playing cards, then the probability of
P.Q. If one card is drawn from a well-shuffled this card being a king or a jack is
pack of 52 cards, the probability of getting 1 1
an ace is (a) (b)
26 13
1 4
(a) (b) 2 4
52 13 (c) (d)
13 13
2 1 Sol. Total number of cards = 52
(c) (d)
13 13 Number of a king or a jack = 4 + 4 = 8
Sol. Total number of cards = 52 8 2
Number of aces = 4  Probability = = (c)
52 13
4 11. The probability that a non-leap year selected
Probability of card being an ace =
52 at random has 53 Sundays is.
1 1 2
= (d) (a) (b)
13 365 365
P.Q. A card is selected at random from a well- 2 1
shuffled deck of 52 cards. The probability (c) (d)
7 7
of its being a face card is
Sol. Number of a non-leap year = 365
3 4 Number of Sundays = 53
(a) (b)
13 13 In a leap year, there are 52 weeks or 364
6 9 days
(c) (d)  One days is left
13 13
Now we have to find the probability of a
Sol. Total number of cards = 52
Sunday out of remaining 1 day
No. of face cards = 3 × 4 = 12
1
12 3  Probability = (d)
 Probability of face card = = (a) 7
52 13
12. A bag contains 3 red balls, 5 white balls and
9. A card is selected at random from a pack of 7 black balls. The probability that a ball drawn
52 cards. The probability of its being a red from the bag at random will be neither red
face card is nor black is
3 3 1 1
(a) (b) (a) (b)
26 13 5 3
2 1 7 8
(c) (d) (c) (d)
13 2 15 15
Sol. Total number of card = 52 Sol. In a bag, there are
No. of red face card = 3 × 2 = 6 3 red balls + 5 white balls + 7 black balls
6 3  Total number of balls = 15
 Probability = = (a)
52 26 One ball is drawn at random which is neither
676 Arun Deep's Understanding Math-10
red not black 4 1
 Number of outcomes = 5 (c) (d)
5 3
5 1 Sol. There are 1 to 40 = 40 tickets in a bag
 Probability = = (b)
15 3  No. of tickets which is multiple of 5 = 8
13. A bag contains 4 red balls and 5 green balls. (5, 10, 15, 20, 25, 30, 35, 40)
One ball is drawn at random from the bag. 8 1
The probability of getting either a red ball or Probability = = (a)
40 5
a green ball is
15. If a number is randomly chosen from the
4 5 numbers 1, 2, 3, 4, ..., 25, then the probability
(a) (b)
9 9 of the number to be prime is
(c) 0 (d) 1 7 9
Sol. In a bag, there are (a) (b)
25 25
4 red balls + 5 green balls
11 13
Total = 4 + 5 = 9 (c) (d)
25 25
One ball is drawn at random
Sol. There are 25 number bearing numbers 1, 2,
 Probability of either a red or a green ball
3, ..., 25
9 Prime numbers are 2, 3, 5, 7, 11, 13, 17, 19,
= =1 (d)
9 23 = 9
P.Q. A bag contains 5 red, 4 white and 3 black 9
balls. If a ball is drawn from the bag at  Probability being a prime number = (b)
25
random, then the probability of the ball being
16. A box contains 90 cards numbered 1 to 90.
not black is
If one card is drawn from the box at random,
5 1 then the probability that the number on the
(a) (b) card is a perfect square is
12 3
1 9
3 1 (a) (b)
(c) (d) 10 100
4 4
Sol. In a bag, there are 1 3
(c) (d)
5 red + 4 white + 3 black balls = 12 9 100
One ball is drawn at random Sol. In a box, there are
 Probability of a ball not black 90 cards bearing numbers 1 to 90
 Perfect squares are 1, 4, 9, 16, 25, 36, 49,
54 9 3
= = = (c) 64, 81 = 9
12 12 4
9
14. One ticket is drawn at random from a bag Probability of being a perfect square =
containing tickets numbered 1 to 40. The 90
probability that the selected ticket has a 1
number which is a multiple of 5 is = (a)
10
1 3 17. If a (fair) coin is tossed twice, then the
(a) (b) probability of getting two heads is
5 5
677 Arun Deep's Understanding Math-10
(a) 7 (b) 14
1 1
(a) (b) (c) 21 (d) 28
4 2
Sol. Total number of eggs = 400
3 Probability of getting a bad egg = 0.035
(c) (d) 0
4  Number of bad eggs = 0.035 of 400
Sol. A coin is tossed twice 35
Number of outcomes = 2 × 2 = 4 = 400 × = 14 (b)
1000
 Probability of getting two heads
P.Q. A girl calculates that the probability of her
= (HH = 1) winning the first prize in a lottery is 0.08. If
1 6000 tickets are sold, how many tickets she
= (a) has bought?
4
(a) 40 (b) 240
18. If two coins are tossed simultaneously, then
(c) 480 (d) 750
the probability of getting atleast one head is
Sol. For a girl,
1 1 Winning a first prize = 0.08
(a) (b)
4 2 Number of total tickets = 6000
3  Number of tickets she bought = 0.08 of 6000
(c) (d) 1
4 8
= 6000 × = 480 (c)
Sol. Two coins are tossed 100
 Total outcomes = 2 × 2 = 4
CHAPTER TEST
Probability of getting atleast one head
3 1. In a single throw of a die, find the probability
(HT, TH, H, H) = (c) of getting
4
(i) a number greater than 5
19. Lakshmi tosses two coins simultaneously.
(ii) an odd prime number
The probability that she gets almost one head
(iii) a number which is multiple of 3 or 4.
is
Sol. In a single throw of a die
3 Number of total outcomes = 6
(a) 1 (b)
4 (1, 2, 3, 4, 5, 6)
1 1 (i) Numbers greater than 5 = 6 i.e., one number
(c) (d)
2 7 1
 Probability =
Sol. Two coins are tossed 6
 Total number of outcomes = 2 × 2 = 4 (ii) An odd prime number 2 i.e., one number
Probability of getting atleast one head 1
 Probability =
3 6
= (HT, TH, HH = 3) = (b)
4 (iii) A number which is a multiple of 3 or 4 which
20. The probability of getting a bad egg in a lot are 3, 6, 4 = 3 numbers
of 400 eggs is 0.035. The number of bad 3 1
eggs in the lot is  Probability = =
6 2
678 Arun Deep's Understanding Math-10
P.Q. A lot consists of 144 ball pens of which 20 Number of good mobiles = 42
are defective and the others are good. Number having minor defect = 3
Rohana will buy a pen if it is good, but will Number having major defect = 3
not buy it if it is defective. The shopkeeper (i) Acceptable to Varnika = 42
draws one pen at random and gives it to
42 7
her. What is the probability that :  Probability = =
48 8
(i) She will buy it ?
(ii) Acceptable to trader = 42 + 3 = 45
(ii) She will not buy it ?
Sol. In a lot, there are 144 ball pens in which 45 15
 Probability = =
defective ball pens are = 20 48 16
and good ball pens are = 144 – 20 = 124 P.Q. A game consists of spinning an arrow which
comes to rest at one of the regions 1, 2 or 3
Rohana buys a pen which is good only.
(shown in the given figure). Are the outcomes
(i) Now number of possible outcomes = 144 1, 2 and 3 equally likely to occur? Give
and number of favourable outcomes = 124 reasons.
 Probability of good pen will be,
2
Number of favourable outcome 1
P (E) = Number of possible outcome

3
124 31
= =
144 36
Sol. In a game,
(ii) Probability of not buying a defective pen
No, the outcomes are not equally likely.

will be P E Outcome 3 is more likely to occur than the
outcomes of 1 and 2.

But P (E) + P E = 1 P.Q. A bag contains 6 red, 5 black and 4 white
balls. A ball is drawn from the bag at
31 31 5

36

+ P E =1 P E =1– 
=
36 36
random. Find the probability that the ball
drawn is
(i) white (ii) red (iii) not black
5

Hence P E =
36
Ans. (iv) red or white.
Sol. Total number of balls = 6 + 5 + 4 = 15
2. A lot consists of 48 mobile phones of which Number of red balls = 6
42 are good, 3 have only minor defects and
Number of black balls = 5
3 have major defects. Varnika will buy a
phone if it is good but the trader will only Number of white balls = 4
buy a mobile if it has no major defect. One (i) Probability of a white ball will be
phone is selected at random from the lot.
Number of favourable outcome
What is the probability that it is P (E) =
Number of possible outcome
(i) acceptable to Varnika?
(ii) acceptable to the trader? 4
Sol. Number of total mobiles = 48 =
15
679 Arun Deep's Understanding Math-10
(ii) Probability of red ball wil be
Number of favourable outcome
Number of favourable outcome P (E) =
Number of possible outcome
P (E) =
Number of possible outcome
13
=
6 2 20
= =
15 5 (iii) Number of ball which are neither white nor
(iii) Probability of not black ball will be black = Number of ball which are only red
=5
Number of favourable outcome
P (E) =  Probability of neither white nor black ball
Number of possible outcome will be

15  5 10 2 Number of favourable outcome


= = = P (E) =
15 15 3 Number of possible outcome
(iv) Probability of red or white ball will be
5 1
Number of favourable outcome = = Ans.
20 4
P (E) =
Number of possible outcome 4. A bag contains 5 white balls, 7 red balls, 4
black balls and 2 blue balls. One ball is
64 10 2 drawn at random from the bag. What is
= = = Ans.
15 15 3 the probability that the ball drawn is :
3. A bag contains 5 red, 8 white and 7 black (i) white or blue
balls. A ball is drawn from the bag at (ii) red or balck
random. Find the probability that the drawn
(iii) not white
ball is :
(iv) neither white nor black ?
(i) red or white
Sol. Number of total balls = 5 + 7 + 4 + 2 = 18
(ii) not black
Number of white balls = 5,
(iii) neither white nor black
number of red balls = 7,
Sol. Total number of balls in a bag = 5 + 8 + 7
= 20 number of black balls = 4 and
(i) Number of red or white balls = 5 + 8 = 13 number of blue balls = 2.
Probability of red or white ball will be (i) Number of white and blue balls = 5 + 2 = 7
 Probability of white or blue balls will be
Number of favourable outcome
P (E) = Number of possible outcome Number of favourable outcome
P (E) =
Number of possible outcome
13
=
20 7
= .
(ii) Number of ball which are not black 18
= 20 – 7 = 13 (ii) Number of red and black balls = 7 + 4 = 11
 Probability of not black ball will be  Probability of red or black balls will be
680 Arun Deep's Understanding Math-10

Number of favourable outcome Number of favourable outcome


P (E) = P (E) =
Number of possible outcome Number of possible outcome

11 10 1
= = =
18 20 2
(iii) Number of ball which are not white (ii) Numbers which are divisible by 2 or 3 will
be 2, 3, 4, 6, 8, 9, 10, 12, 14, 15, 16, 18,
= 7 + 4 + 2 = 13
20 = 13 balls
 Probability of not white balls will be
 Probability of ball which is divisible by 2
Number of favourable outcome or 3 will be
P (E) =
Number of possible outcome Number of favourable outcome
P (E) =
Number of possible outcome
13
=
18 13
=
(iv) Number of balls which are neither white 20
nor black (iii) Prime numbers will be 2, 3, 5, 7, 11, 13,
= 18 – (5 + 4) = 18 – 9 = 9 17, 19 = 8
 Probability of ball which is neither white  Probability of prime number will be
nor black will be
Number of favourable outcome
Number of favourable outcome P (E) =
Number of possible outcome
P (E) =
Number of possible outcome
8 2
= =
9 1 20 5
= = Ans.
18 2 (iv) Numbers not divisible by 10 will be
5. A box contains 20 balls bearing numbers 1, 2, 3, 4, 5, 6, 7, 8, 9, 11, 12, 13, 14, 15,
1, 2, 3, 4, ....., 20. A ball is drawn at random 16, 17, 18, 19 = 18
from the box. What is the probability that  Probability of prime number will be
the number on the ball is
Number of favourable outcome
(i) an odd number P (E) =
Number of possible outcome
(ii) divisible by 2 or 3
(iii) prime number 18 9
(iv) not divisible by 10 ? = = Ans.
20 10
Sol. In a box, there are 20 balls containing 1 to 6. Find the probability that a number selected
20 number at random from the numbers 1, 2, 3,
 Number of possible outcomes = 20 ........., 35 is a
(i) Numbers which are odd will be, (i) prime number
1, 3, 5, 7, 9, 11, 13, 15, 17, 19 = 10 balls. (ii) multiple of 7
 Probability of odd ball will be (iii) multiple of 3 or 5.
681 Arun Deep's Understanding Math-10
Sol. Numbers are 1, 2, 3, 4, 5, ........, 30, 31, (i) Numbers which are divisible by 5 will be
32, 33, 34, 35 15, 20, 25, 30, 35, 40, 45, 50, 55, 60 = 10
Total = 35  Probability of number divisible by 5 will be
(i) Prime numbers are 2, 3, 5, 7, 11, 13, 17,
Number of favourable outcome
19, 23, 29, 31 P (E) =
Number of possible outcome
which are 11
 Probability of prime number will be 10 5
= =
Number of favourable outcome 48 24
P (E) = (ii) Numbers which is a perfect square are 16,
Number of possible outcome
25, 36, 49 which are 4 in numbers.
11  Probability of number which is a perfect
=
35 square will be
(ii) Multiple of 7 are 7, 14, 21, 28, 35 which Number of favourable outcome
are 5 P (E) =
Number of possible outcome
 Probability of multiple of 7 will be

Number of favourable outcome


4 1
= = Ans.
P (E) =
Number of possible outcome 48 12
8. The box has cards numbered 14 to 99.
5 1 Cards are mixed thoroughly and a card is
= = drawn at random from the box. Find the
35 7
probability that the card drawn from the
(iii) Multiple of 3 or 5 are 3, 5, 6, 9, 10, 12, 15, box has
18, 20, 21, 24, 25, 27, 30, 33, 35. Which
(i) an odd number
are 16 in numbers
(ii) a perfect square number.
Probability of multiple of 3 or 5 will be
Sol. Cards in a box are from 14 to 99 = 86
Number of favourable outcome  No. of total cards = 86
 P (E) = Number of possible outcome One card is drawn at random
Cards bearing odd numbers are 15, 17, 19,
16
= Ans. 21, ..., 97, 99
35
Which are 43
7. Cards marked with numbers 13, 14, 15,
....., 60 are placed in a box and mixed No. of actual events
(i)  P (E) = No. of total events
thoroughly. One card is drawn at random
from the box. Find the probability that the
number on the card is 43 1
= =
(i) divisible by 5 86 2
(ii) a number which is a perfect square. (ii) Cards bearing number which are a perfect
Sol. Number of cards which are marked with square
numbers 13, 14, 15, 16, 17, ....... to 59, = 16, 25, 36, 49, 64, 81
60 are = 48 Which are 6
682 Arun Deep's Understanding Math-10

6 3 9 x x
 P (E) = = = × =
86 43 8 18 16
9. A bag contains 5 red balls and some blue and number of balls = 18 + 2 = 20
balls. If the probability of drawing a blue x2 x x2 x
ball is four times that of a red ball, find the  =  =
18  2 16 20 16
number of balls in the bags.
20x = 16x + 32  20x – 16x = 32
Sol. Number of red balls = 5
and let number of blue balls = x 32
 4x = 32  x = =8
 Total balls in the bag = 5 + x 4
 x=8
5 11. A card is drawn from a well-shuffled pack
and that of red balls =
5 x of 52 cards. Find the probability that the
According to the condition, card drawn is :
(i) a red face card
x 5 x 20
=4×  = (ii) neither a club nor a spade
5 x 5 x 5 x 5 x
(iii) neither an ace nor a king of red colour
 x  –5
(iv) neither a red card nor a queen
 x = 20
(v) neither a red card nor a black king.
Hence, number of blue balls = 20
Sol. Number of cards in a pack of well-shuffled
and number of balls in the bag cards = 52
= 20 + 5 = 25 Ans.
(i) Number of a red face card = 3 + 3 = 6
10. A bag contains 18 balls out of which x balls
are white.  Probability of red face card will be
(i) If one ball is drawn at random from the bag,
Number of favourable outcome
what is the probability that it is white ball? P (E) =
Number of possible outcome
(ii) If 2 more white balls are put in the bag, the
9
probability of drawing a white ball will be 6 3
8 = =
52 26
times that of probability of white ball coming
in part (i). Find the value of x. (ii) Number of cards which is neither a club
Sol. Total numbers of balls in a bag = 18 nor a spade = 52 – 26 = 26
No. of white balls = x  Probability of card which is neither a club
(i) One ball is drawn a random
nor a spade will be
x
Probability of being a white ball = Number of favourable outcome
18
P (E) = Number of possible outcome
(ii) If 2 more white balls an put, then number of
white balls = x + 2
9 26 1
and probability is times = =
8 52 2
683 Arun Deep's Understanding Math-10
(iii) Number of cards which is neither an ace (ii)a face card
nor a king of red colour (iii)a diamond or a club
= 52 – (4 + 2) = 52 – 6 = 46 (iv) a queen or a spade.
Sol. Total number of cards = 52
 Probability of card which is neither ace nor
Black jacks, black kings and black aces are
a king of red colour will be removed
Now number of cards = 52 – (2 + 2 + 2)
Number of favourable outcome
P (E) = = 52 – 6 = 46
Number of possible outcome
One card is drawn
(i) No. of red cards = 13 + 13 = 26
46 23
= = 26 13
52 26  Probability = =
46 23
(iv) Number of cards which are neither a red (ii) Face cards = 4 queens, 2 red jacks, 2 kings
card nor a queen are = 52 – (26 + 2) = 52 =8
– 28 = 24
8 4
 Probability of card which is neither red nor  Probability = =
46 23
a queen will be
(iii) a diamond on a club
= 13 + 10 = 23
Number of favourable outcome
P (E) = 23 1
Number of possible outcome
 Probability = =
46 2
24 6 (iv) A queen or a spade
= = = 4 + 10 = 14
52 13
14 7
(v) Number of cards which are neither red card  Probability = =
nor a black king = 52 – (26 + 2) 46 23
= 52 – 28 = 24 13. Two different dice are thrown
simultaneously. Find the probability of
 Probability of cards which is neither red getting :
nor a black king will be (i) sum 7 (ii) sum < 3
Number of favourable outcome (iii) sum < 10
P (E) = Sol. (i) Numbers whose sum is 7 will be (1, 6),
Number of possible outcome
(2, 5), (4, 3), (5, 2), (6, 1), (3, 4) = 6

24 6 Number of favourable outcome


= = Ans. P(E) = Number of possible outcome
52 13
12. From pack of 52 playing cards, black jacks, 6 1
black kings and black aces are removed and = =
36 6
then the remaining pack is well-shuffled. A
card is drawn at random from the remaining (ii) Sum < 3
pack. Find the probability of getting Then numbers can be (1, 2), (2, 1), (1, 1)
(i) a red card which are 3 in numbers
684 Arun Deep's Understanding Math-10

 Probability will be 14. Two dice are thrown together. Find the
probability that the product of the numbers
Number of favourable outcome on the top of two dice is
P (E) = (i) 4 (ii) 12
Number of possible outcome
(iii) 7
Sol. Two dice are thrown together
3 1
= =  Total number of events = 6 × 6 = 36
36 12 (i) Product 4 = (1, 4), (2, 2), (4, 1) = 3
(iii) Sum < 10 3 1
 Probability = =
The numbers can be, 36 12
(1, 1), (1, 2), (1, 3), (1, 4), (1, 5), (1, 6), (ii) Product 12 = (2, 6), (3, 4), (4, 3), (6, 2) = 4
(2, 1), (2, 2), (2, 3), (2, 4), (2, 5), (2, 6), 4 1
(3, 1), (3, 2), (3, 3), (3, 4), (3, 5), (3, 6),  Probability = =
36 9
(4, 1), (4, 2), (4, 3), (4, 4), (4, 5), (4, 6),
(iii) Product 7 = 0 (no outcomes)
(5, 1), (5, 2), (5, 3), (5, 4), (5, 5), (6, 1),
(6, 2), (6, 3), (6, 4) = 33 0
 Probability = =0
36
 Probability will be

Number of favourable outcome


P (E) =
Number of possible outcome

33 11
= = Ans.
36 12
9 789384 447243

You might also like